You are on page 1of 832

BIBLIOTECA DEL PROFESSORAT

Matemàtiques
SÈRIE RESOL

BATXILLERAT
SOLUCIONARI
El Solucionari Matemàtiques I, del projecte Saber fer,
per a 1r de Batxillerat, és una obra col·lectiva concebuda,
dissenyada i creada al Departament d’Edicions
Educatives de Grup Promotor / Santillana Educación, S. L.,
dirigit per Teresa Grence Ruiz i Pere Macià Arqué.

En l’elaboració ha participat l’equip següent:


Francisco Calvo Pérez
David Campos Ortega
Miquel Escolano Sánchez
Ana de la Cruz Fayos
Ana María Gaztelu Villoria
Augusto González García
Silvia Martín García
Virgilio Nieto Barrera
Laura Sánchez Barrera

EDICIÓ
Ana de la Cruz Fayos
Núria Grinyó Martorell
Silvia Martín García
Virgilio Nieto Barrera
Laura Sánchez Fernández

EDITOR EXECUTIU
Carlos Pérez Saavedra

DIRECCIÓ DEL PROJECTE


Domingo Sánchez Figueroa
Índex
Unitat 1 . . . . . . . . . . . . . . . . . . . . . . . . . . . . . . . . . . . . . . . . . . . . . . . . . . . . . . . . . . . . . . . . . 5
Unitat 2 . . . . . . . . . . . . . . . . . . . . . . . . . . . . . . . . . . . . . . . . . . . . . . . . . . . . . . . . . . . . . . . . . 57
Unitat 3 . . . . . . . . . . . . . . . . . . . . . . . . . . . . . . . . . . . . . . . . . . . . . . . . . . . . . . . . . . . . . . . . . 109
Unitat 4. . . . . . . . . . . . . . . . . . . . . . . . . . . . . . . . . . . . . . . . . . . . . . . . . . . . . . . . . . . . . . . . . 185
Unitat 5 . . . . . . . . . . . . . . . . . . . . . . . . . . . . . . . . . . . . . . . . . . . . . . . . . . . . . . . . . . . . . . . . . 247
Unitat 6. . . . . . . . . . . . . . . . . . . . . . . . . . . . . . . . . . . . . . . . . . . . . . . . . . . . . . . . . . . . . . . . . 311
Unitat 7 . . . . . . . . . . . . . . . . . . . . . . . . . . . . . . . . . . . . . . . . . . . . . . . . . . . . . . . . . . . . . . . . . 371
Unitat 8. . . . . . . . . . . . . . . . . . . . . . . . . . . . . . . . . . . . . . . . . . . . . . . . . . . . . . . . . . . . . . . . . 441
Unitat 9. . . . . . . . . . . . . . . . . . . . . . . . . . . . . . . . . . . . . . . . . . . . . . . . . . . . . . . . . . . . . . . . . 503
Unitat 10. . . . . . . . . . . . . . . . . . . . . . . . . . . . . . . . . . . . . . . . . . . . . . . . . . . . . . . . . . . . . . . . 563
Unitat 11 . . . . . . . . . . . . . . . . . . . . . . . . . . . . . . . . . . . . . . . . . . . . . . . . . . . . . . . . . . . . . . . . 625
Unitat 12 . . . . . . . . . . . . . . . . . . . . . . . . . . . . . . . . . . . . . . . . . . . . . . . . . . . . . . . . . . . . . . . . 673
Unitat 13 . . . . . . . . . . . . . . . . . . . . . . . . . . . . . . . . . . . . . . . . . . . . . . . . . . . . . . . . . . . . . . . . 743
Unitat 14. . . . . . . . . . . . . . . . . . . . . . . . . . . . . . . . . . . . . . . . . . . . . . . . . . . . . . . . . . . . . . . . 785
 
 
 
 
 
 

1
 

1
 
 
 
 
 

Nombres reals
 

 
Nombres reals      

 
 
 
 
 

ACTIVITATS 
 
1. Calcula el representant canònic d’aquests nombres:
��� �� ���
a) b) c)
�� �� ���
��� � �� � ��� �
a) �� b) � c) �
�� � �� �� ��� �

2. Escriu dos representants dels nombres racionals:


� � �
a) b) c)
�� � ��
Resposta oberta.
� �� ��
a) = �� , , ,��
�� �� ��
� �� ��
b) = �� , , ,��
� � �
� �� ��
c) = �� , , ,��
�� �� ��

3. Determina quants nombres racionals diferents hi ha en aquesta seqüència:


� � �� � ��
� �, �
� � � �� �

Hi ha dos nombres racionals diferents:

� �� � �� �
� �
� �, � - � �
� � � � ��

4. Una fracció que tingui un terme negatiu i una altra que tingui els seus dos termes positius,
poden ser representants del mateix nombre racional?

No poden representar el mateix nombre racional perquè si una fracció té un terme negatiu, el quocient
és negatiu; i si els dos termes són positius, el quocient és positiu.

5. Escriu quatre nombres irracionals i especifica’n la regla de formació.


Resposta oberta.
Després de la coma, escrivim tots els múltiples de 3: 0,3691215...
Després de la coma, escrivim tots els múltiples de 4: 0,481216...
Sumem 1 al nombre racional √2: √2 � �
Sumem 2 al nombre racional √2: √2 � 2

5
 
 
 
 
 
 

Nombres reals
Nombres reals 1
 
 
 
 
 
 

6. Determina si els nombres següents són irracionals.


�� ��
a) 0,51015202530... b) c) � � � d)
�� ��

a) És un nombre irracional perquè té infinites xifres decimals que no es repeteixen de manera


periòdica.

b) És un nombre decimal exacte; per tant, no és un nombre irracional.

c) És un nombre irracional perquè, si a un nombre irracional se li resta un nombre enter, el resultat és


un nombre irracional.

d) No és un nombre irracional perquè és una fracció.

7. Sense fer operacions amb decimals, escriu un nombre irracional inclòs entre -√� i √� .
Resposta oberta. Per exemple: √2 � �

8. Justifica si les afirmacions següents són certes o no:


a. L’arrel d’un nombre irracional és racional.
b. Un nombre irracional al quadrat no és racional.
a) Certa, perquè té infinites xifres decimals no periòdiques.

b) Falsa. Per exemple: ( 2)2  2

9. Indica el conjunt numèric mínim al qual pertanyen aquests nombres:


a) 8,0999... c) √�� e) 2,5
b) 1,22333444... �
d) �� ��� f) -11

10. Representa les arrels següents:


a) √�� b) √��� c) √� d) √��

6
 
 
 
 
 
 

Nombres reals 11
 
 
 
 
 
 

11. Col·loca aquest nombre en la recta real:


� � � √�
��

12. Representa els nombres 1 i 2 en la recta real següent:

13. Aplica la propietat distributiva i opera.

14. Escriu tres nombres situats entre aquests:


��� ��� �
a) ��� b) √����√� � �
��� ��� ��

3011 3012 3013


a) Resposta oberta, per exemple: , i
2000 2000 2000
1 2 3
b) Resposta oberta, per exemple: 5 , 5 i 5
100 100 100

7
 
 
 
 
 
 

Nombres reals
Nombres reals 1
 
 
 
 
 
 

15. Ordena, de més petit a més gran els nombres racionals i irracionals següents:
�� ����
3 �
� ���
���� ��
3 �� � � ��
��� �

16. Per mitjà de la propietat distributiva, calcula sense realitzar els quadrats.
a) ��� b) ����
a) 99�� = 99 · 99 = 99(100 – 1) = 9 900 – 99 = 9 801
b) 999�� = 999 · 999 = 999(1 000 – 1) = 999 000 – 999 = 998 001

17. Representa els següents conjunts numèrics de totes les maneres que coneguis.
a. Nombres més petits que �.
b. Nombres més grans que √� i més petits o iguals que 7.
c. Nombres més petits o iguals que 2 i més grans que -2.
d. Nombres compresos entre els dos primers nombres parells, tots dos inclosos.
e. Nombres compresos entre √� i √�.


e) ( 2, 3)  x : 2  x  3 

18. Escriu, de totes les maneres que coneguis, aquests intervals de la recta real.

a) (��� ��� � � � ��� � ���� c) ��� ��� � � ��� � � ��


b) ������ � � ��� �� � � � �� d) ������ � � ��� |�| � ��

8
 
 
 
 
 
 

Nombres reals 11
 
 
 
 
 
 

19. Representa el conjunt ��: |� � �| � ��, de totes les maneres que coneguis.

20. Amb l’ajut de la calculadora, escriu √� en forma decimal i les aproximacions corresponents
per excés i per defecte als deumil·lèsims i als centmil·lèsims.
√� � �� �������� �
a) Aproximació per excés: 1,7321
Aproximació per defecte: 1,7320

b) Aproximació per excés: 1,73205


Aproximació per defecte: 1,73205

c) Aproximació per excés: 1,732051


Aproximació per defecte: 1,732052

21. Pensa en una situació en la qual dos mesuraments tinguin els mateixos errors absoluts però
diferents errors relatius.
Resposta oberta.
Vreal = 12,5
Valors aproximats: 12 i 13. En tots dos casos, l’error absolut és 0,5, però els errors relatius són
diferents:

22. Indica dos exemples de mesurament i dóna’n les corresponents cotes d’error.
• Velocitat a l’autopista → 115,45 km/h
 1
Cota d'error absolut   0,5
 2100
Aproximació → 115 km/h→ 
 0,5
Cota d'error relatiu  115 0,5  0,00437

• Mitjana d’edat de jubilació → 64,3 anys


 1
Cota d'error absolut   0,5
 2100
Aproximació → 64 anys → 
 0,5
Cota d'error relatiu  64  0,5  0,007874

23. Calcula les cotes d’error absolut i relatiu quan arrodonim el nombre √�:
a) Als centèsims. b) Als mil·lèsims.

9
 
 
 
 
 
 

Nombres reals
Nombres reals 1
 
 
 
 
 
 

24. Una localitat té 310 habitants, xifra arrodonida a les desenes. En pots indicar els errors?
Sabries assenyalar les cotes d’error comès?
No ho podem calcular, ja que per calcular els errors relatius i absoluts s’ha de conèixer el valor real.

25. Calcula una cota d’error absolut quan truquem un nombre als dècims. I si fos als centèsims?
1 1
Ea  0,05 Ea  0,005
2  101 2  10 2

26. Escriu els següents nombres en notació científica:


a) 0,0000085 c) 31 940 000 000
b) 5 000 000 000 000 c) 0,000000000479

27. Opera i expressa el resultat en notació científica.

28. Determina si les igualtats següents són certes. Justifica la resposta.

29. Calcula el valor numèric, si existeix, dels següents radicals:

a) 4
16  2 c) No té cap arrel real.
3
b) 8  2 d) 5
243  3

30. Transforma aquests radicals en potències, i viceversa.

10
 
 
 
 
 
 

Nombres reals 11
 
 
 
 
 
 

31. Indica si els radicals següents són equivalents:


� �
a) √�� i √�� c) √�� i √�
� �
b) √��� i √� d) √��� i √��
a) Són equivalents. c) Són equivalents.
b) No són equivalents. d) No són equivalents.

32. Efectua aquestes operacions:

33. Opera i simplifica.

34. Racionalitza les expressions següents:

11
 
 
 
 
 
 

Nombres reals
Nombres reals 1
 
 
 
 
 
 

35. Racionalitza i opera.

36. Calcula aquests logaritmes mitjançant la definició:

37. Determina aquests logaritmes per mitjà de la definició:

38. Calcula aquests logaritmes i deixa indicat el resultat:

12
 
 
 
 
 
 

Nombres reals 11
 
 
 
 
 
 

39. Si sabem que log 2 = 0,3010; log 3 = 0,4771 i log 7 = 0.8451, determina els logaritmes decimals
dels 10 primers nombres naturals. Amb aquestes dades, sabries calcular log 3,5? I log 1,5?

40. Sense l’ajuda de la calculadora, determina, log2 5 i log5 2. Comprova que el seu producte és 1.
En l’activitat anterior hem vist que log2  0,3010 .
Utilitzem canvis de base:
log10 1
log2 10    3,32
log2 0,3010
log2 10  log2 (25)  log2 2log2 5  log2 5  2,32
log2 2 1
log2 2    0,43
log2 5 log2 5

Com que els nombres són inversos, el seu producte és 1.També es pot comprovar d’aquesta manera:

log5 log2
log2 5log5 2   1
log2 log5

41. Calcula el valor de x en les igualtats següents:



a) ����� ���� � � �� c) ���� √��� � �

b) ���� �� � � d) ���� �� � �.

� �
a) b) 2,0801… c) d) √3
� �

42. Calcula loga b · logb a


����� �����
��� � �� � � ��� � � � � �� ��
����� �����

SABER FER

43. Multiplica i resta.

13
 
 
 
 
 
 

Nombres reals
Nombres reals 1
 
 
 
 
 
 

  27  2 43 4 64    49 69 118 
2, 7 4, 3     7,1 5, 4  7,6     13,1
a) 9 9 9 d) 9 9 9

  1 819 68 1139    628 4171 10451 


20,21  7,5     12,65 6,34  4,213     10,556
b) 90 9 90 e) 99 990 990

   607 47 1124    122 167 109 


6,313 5,22     11,35 1,23 1,012     0,220
c) 99 9 99 f) 99 165 495

44. Multiplica i divideix.

 6 19 38    287 76 21812 
a) 1,22,1     2,53 c) 6,378, 4     53,8567901234
5 9 15 45 9 405
 6 19 54    287 76 287 
b) 1,22,1     0,5684210526315789473 d) 6,378, 4     0,75526315789473684210
5 9 95 45 9 380

45. Fes les operacions següents:

2 1 2 2
 5 4   3   2   25 24   2  4 302 2 4 600 4 15004
a)                  2     
 6 5   2   3   30 30   3  9 49 3 9 2401 9 21609
2 1 2 2
 5 4   3   2   25 24  2 9 1 2 9 1 9 6077
b)    :             2     
 6 5   2   3   30 30  3 4 30 3 4 1350 4 2700
46. Factoritza aquestes expressions:

6 30 54 23 235 233 23  5 32  6  5 9 


a)        1     1  
35 105 245 57 357 572 57  3 7  35  3 7 

9 45 81 32 532 34 32  5 32  9  5 9 
b)    2  5  2 2  2  1 3  2    1  
4 32 100 2 2 2 5 2  2 5  4  8 25 

47. Calcula la unió d’aquests intervals:

a) (4,2](3,0)  (4,0) b) (2, 8][2, 0)

48. Calcula la intersecció dels intervals següents:

14
 
 
 
 
 
 

Nombres reals 11
 
 
 
 
 
 

a) ( 4,  2]  ( 3, 0)  ( 3, 1] b) (2, 8]  [ 2, 0) 

49. Escriu els cinc primers intervals encaixats dels nombres √��, � i ɸ.

a) 22  4,6904157598234295545656301135445...

4  22  5 → (4, 5) → Error < 5 – 4 = 1

4,6  22  4,7 → (4,6; 4,7) → Error < 4,7 – 4,6 = 0,1

4,69  22  4,70 → (4,69; 4,7) → Error < 4,7 – 4,69 = 0,01

4,690  22  4,691 → (4,69; 4,691) → Error < 4,691 – 4,69 = 0,001

4,6904  22  4,6905 → (4,6904; 4,691) → Error < 4,691 – 4,6904 = 0,0001


b)  3,1415926535897932384626433832795...

3    4 → (3, 4) → Error < 4 – 3 = 1


3,1    3,2 → (3,1; 3,2) → Error < 3,2 – 3,1 = 0,1
3,14  π  3,15→ (3,14; 3,15) → Error < 3,15 – 3,14 = 0,01
3,141    3,142 → (3,141; 3,142) → Error < 3,142 – 3,141 = 0,001
3,1415    3,1416 → (3,1415; 3,1416) → Error < 3,1416 – 3,1415 = 0,0001
c)  1,6180339887498948482045868343656...

1    2 → (1, 2) → Error < 2 – 1 = 1


1,6    1,7 → (1,6; 1,7) → Error < 1,7 – 1,6 = 0,1
1,61    1,62 → (1,61; 1,62) → Error < 1,62 – 1,61 = 0,01
1,618    1,619 → (1,618; 1,619) → Error < 1,619 – 1,618 = 0,001
1,6180    1,6181 → (1,618; 1,6181) → Error < 1,6181 – 1,618 = 0,0001

50. Opera en notació científica.

a) 6,4106 5,1104 9,3102  9,24964102 b) 5,1106 5,210 4 5,3102  5,04853106

51. Converteix les expressions següents en un sol radical:

2
1 13 2

 

3 4
a) 5 3   5 c) 5 3  3 52 e) 23  12 23
3
5 2
5

15
 
 
 
 
 
 

Nombres reals
Nombres reals 1
 
 
 
 
 
 
2
2  1 13 3 3
b) 5 3   3 52 d) (5) 3   5 f) 393
3
(5) 2
5

52. Introdueix els factors de les expressions següents dins del signe radical:

a) 3x 2 3 3y  3 27x 6 3y  3 81x 6 y c) 2ab2c 4 4  4 64a 4b8c 4

b) 8b 8a3b  512a 3b3 d) (2a  b) b  (2a  b)2 b  4a2b  b3  4ab2

53. Racionalitza les expressions següents:

3 3  4 53
a) 
3 4 5 5

6 6
3
 22  2 3   3
b)   3
22   
2  3  3 3 22   3 2 
3
2  2 3  3
2  2 3 2  
3 2
2 3

c)

16
 
 
 
 
 
 

Nombres reals 11
 
 
 
 
 
 

ACTIVITATS FINALS

54. Classifica aquestes fraccions en reductibles i irreductibles:

ିହ ଷ
a) → És irreductible, perquè m.c.d.(5,12) = 1. d) → És irreductible, perquè m.c.d.(3,8) = 1.
ଵଶ ଼
9 3 ଵହ
b)  → És una fracció reductible. e) െ →És irreductible, perquè m.c.d.(15,28) = 1
6 2 ଶ଼

15 5 104 52
c)  → És una fracció reductible. f)   → És una fracció reductible.
18 6 206 103

55. Classifica aquestes fraccions en reductibles i irreductibles:

5 1 26 1 12 3 88 1
a)  c)  e)  f) 
200 40 130 5 400 100 176 2

1080 5 702 2 72 8 104 13


b)  d)  f)  g) 
200 2 1 053 3 243 27 216 27

56. Calcula x perquè les fraccions siguin equivalents…

3 6 x 4
a)   3x  30  x  10 c)   6x  12  x  2
5 x 3 6
5 x 4 1
b)   40  2x  x  20 d)   12  x  x  12
2 8 x 3

57. Determina els valors de x perquè siguin irreductibles.

a) m.c.d.(x, 18) = 1, x < 18 → x = {5, 7, 11, 13, 17} b) m.c.d.(12, x) = 1, x < 12 → x = {5, 7, 11}

58. Determina els valors de x perquè siguin irreductibles.


17
 
 
 
 
 
 

Nombres reals
Nombres reals 1
 
 
 
 
 
 

2 2 2 2
1  2   2 1  1  3   1  1 9 1 17
a)              
2  3   3 6  2  2   2  2 4 4 16
2 2 2 2
 4 1   5 1   1   5  109
b)  :            
 3 6   2 6   8   12  576

59. Fes les operacions següents:

2 1 2 2 2
 5 4   2   1   25 24  3 1  1  3 1 3 1 1 5 401
a)                      900   1 350 
 6 5   3   2   30 30  2 4  30  2 4 2 4 4 4
1 1 2 1 1
5 2 7 4  25 4  3 16  29  3 16 10 3 16 70 16 254
b)    :      
   :    : 
 :   

 2 5   3 3
   10 10  7 9 10
  7 9 29 7 9 87 9 261

60. Expressa aquests nombres en forma decimal:

22  43 12  42
a)  1,692307 b)  0,043 c)  0,0109 d)  8,4
13 1 000 1 100 5

61. Indica de quin tipus són aquests nombres decimals:

a) 2,331 Decimal exacte


b) 4,1234… Irracional
c) 6,2727… Decimal periòdic puro
d) 0,03131… Decimal periòdic mixt
e) 4 Decimal exacte
f) –32,207 Decimal exacte

18
 
 
 
 
 
 

Nombres reals 11
 
 
 
 
 
 

62. Determina la fracció generatriu dels nombres decimals següents:

1 40001 1
a) 0,2  d) 8,0002  g) 0,01 
5 5000 100

 32  1412  5897
b) 3,5  e) 42,78  h) 5,902 
9 33 999

237  5209  13
c) 2,37  f) 10,523  i) 0,0157 
100 495 825

63. Efectua per mitjà de les fraccions generatrius.

 4 17 71   571 23 89
a) 1,3 3,4    c) 6,34 2,5   
3 5 15 90 9 10
  923 52 403  108 316 608
b) 10,25  5,7    d) 4,32 7,02   
90 9 90 25 45 225

64. Fes les operacions següents:

� �� ��� ��� �� ���� ����


a) � � c) � � =
� � �� �� � ��� ���
� ��� �� � � ��� ��� ���
b) ∶ � � d) ∶ � =
�� �� ���� ��� � �� ��� ���

65. Mitjançant les fraccions generatrius, comprova se aquestes igualtats són certes o falses:

19
 
 
 
 
 
 

Nombres reals
Nombres reals 1
 
 
 
 
 
 

66. Ordena aquests nombres decimals de més petit a més gran:

67. Escriu un nombre racional i un d’irracional inclosos entre:



a) �� ������ ����� c) 1 i 2 e) ��� �� �
��� � �� ��

b) �� �� �
������� �� �
d) �� ������ �� f) 0,2 i 0,25
Resposta oberta
a) Racional: 3,40022 d) Racional: 5,62
Irracional: 3,4002201001... Irracional: 5,6201001...
b) Racional: 2,523 e) Racional: -2,67
Irracional: 2,52301001... Irracional: -2,6701001...
c) Racional: 1,1 f) Racional: 0,21
Irracional: 1,101001... Irracional: 0,2101001...

68. Sense fer operacions, escriu un nombre irracional comprès entre – √� i √�


√�
Resposta oberta. Per exemple:

69. Demostra que 2·√� és un nombre irracional

La prova més senzilla per a demostrar que és irracional és mitjançant reducció a l’absurd.

Suposem que és un nombre racional, i llavors es pot escriure com 2√5 = , amb a i b nombres primers

entre ells.

Ara s’eleven ambdós costats de la igualtat al quadrat i s’obté.

2
a a2
20     20  2  20b2  a2
b b

D’aquí s’entén que es pugui escriure a2 = (2k)2, amb k un nombre enter divisor de a, així s’obté per tant
5b2 = k2.

Això assegura que 5 és múltiple de k2, i implica que 5 també és múltiple de k, i aquí està l’absurd: se
suposava que b i k no tenien factors comuns i es manté que els dos són múltiples de 5, és a dir, que
tenen 5 com a factor comú, i per tant el seu m.c.d. ha de ser com a mínim 5.

Aquesta és la contradicció que es buscava, per tant 5 és irracional, i per tant 2 5 també ho és.

20
 
 
 
 
 
 

Nombres reals 11
 
 
 
 
 
 

70. Diferencia entre nombres racionals i irracionals.


a) √� b) √� c) √�� d) √�� e) √�� f) √��
Són tots nombres irracionals excepte √16 � ��, el qual és un nombre enter i, per tant, racional.

71. Indica els nombres que són irracionals.

Irracionals → a), c), d), i f)

Racionals → b) 2 9  23  6 i e) 1 16  1 4  3

72. Quins nombres estan representats en les construccions següents?

a) 20 b) 2 5

73. Quins nombres estan representen sobre aquesta recta numèrica els punts A, B, C i D, en què n
és un segment qualsevol?

74. Representa els nombres següents en la recta real:

a) 10  32 12 →

0  1 2 3

21
 
 
 
 
 
 

Nombres reals
Nombres reals 1
 
 
 
 
 
 

b)  10   22 12 12 →

–2  –1 0
                   

c) 1 2  1 12 12 →

0 1
     

    d) 3 1  22 12 1 →

–1 0 1
     

e) 2  3  12 12  22 12 →

1 1
2
0  1 2 3
                     

f) 2  3  12 12  22 12 →

1 1
2
0 1 2 3
   

75. Representa aquests nombres en la recta real:

a) 5  22 12 →  

0  1 2

                   

b) 13  32 22 →

22
 
 
 
 
 
 

Nombres reals 11
 
 
 
 
 
 

0  1 2 3

                   

c) 18  32 32 →

0  2 3 4

   

76. Ordena i representa els nombres següents en la recta real:



�� �, � b) √� c)

a)  2,3  2  0,3     
b)  5 = 2 +1  2 2

9 1 9
c)  = 2 +   4
4 4 5 2,3
0  1 2 3 4 

 
 
77. Opera i classifica el tipus de nombre real.

�� �
a) És un nombre racional: �2, 7� � � � �
� �

b) És un nombre irracional: �4, 9� � ��� � √5



�,� �� � �
c) És un nombre racional: � � � � � � �
� �� � �

78. Expressa mitjançant intervals de totes les maneres possibles.

a) x  (2,3] y 2  x  3 c) x  (, 0] y x  0

b) x  1, 4y 1  x  4 d) x  [8,) y x  8

79. Descriu i representa els intervals següents:


23
 
 
 
 
 
 

Nombres reals
Nombres reals 1
 
 
 
 
 
 

a) (0, 10) d) ��� �� g) ���� ��


b) (3, 7] e) ��� ��� h) ����� ���
c) (��� ��� f) ���� ��� i) ���� √��

a) x : 0  x  10
b) x : 3  x  7
c) x : x  2
d ) x : 2  x  5
e) x : 5  x  10
f ) x : 4  x
g) x : x  6
h) x : 100  x
i) x : 7  x  2

                        –7  

80. Escriu l’interval que correspon a aquestes desigualtats:

a) ( 1 , 3 ) b) ( 6 , 7 ] c) [ 5 , 9 ) d) [ 10 , 12 ]

81. Escriu l’interval corresponent.

a) (��� ��� c) ���� ��� e) ���� ���


b) (��� �� d) ��� ��� h) ���� ���

24
 
 
 
 
 
 

Nombres reals 11
 
 
 
 
 
 

82. Calcula les unions d’intervals següents:

a) (3,16)(2, 5)  (2,16)

b) [2, 2)[11, 0]  [11, 2)

 5 7   15 9   15 7 
c)  ,   ,    , 
 4 3  2 5  2 3

d) [ 7, 5][ 5, 7]  [ 7, 7]

83. Determina les interseccions d’aquests intervals:

a) (1,10)(3, 8)  (1, 8)

 4   5   4 
b)  , 5  , 0   , 0
 7   8   7 

   
5 7   15 9   5 , 9 
c)   ,   ,  
 2 3  4 5  2 5
   

d) [ 7, 5][ 5, 7] [ 5, 5]

84. Donats els intervals següents, calcula:

a) AB  [4, 2)

b) AC  [4, 4)

c) BC  2, 2
d) ABC  (2, 1]

85. Donats aquests intervals, determina:

25
 
 
 
 
 
 

Nombres reals
Nombres reals 1
 
 
 
 
 
 

a) AB  (,5)

b) AC  (, 3]

c) BC  [0,3]

d) ABC  [0,1]

86. Expressa els intervals següents com una intersecció de dues semirectes:

 13   
a)  1,  
 2
1,  , 132
b) [5, 5 3] (, 5 3][5, )

c) x : 6  x  40   (, 40](6,)

 51   51   51 
d)  x :   x  3   , 3   , , 3
 4   4   4 

   
2  2
e) 3,
 2 
 3,   ,
 2 

   
f) 
30
 2

, 90   
  2
 
30
,   , 90


 
   7   7 

7
g)  x :   x   3    ,  3    , ,  3
2   2   2 
 
h) x :  3
    
5  x  3 5   3 5 , 3 5  , 3 5   3 5 ,
87. Escriu en forma d’interval i expressa-ho després com a intersecció de dues semirectes.

26
 
 
 
 
 
 

Nombres reals 11
 
 
 
 
 
 

a) La temperatura prevista per a demà variarà dels -1 ºC de mínima als 13 ºC de màxima.


b) Aquest jugador de futbol té menys de 27 anys.
c) L’aigua es manté en estat líquid entre 0 i 100 ºC.
d) Es pot votar des dels 18 anys.
e) El meu pressupost màxim per a comprar un cotxe és de 11 000 €.


a) [1,13]  ,13[1, ) d) [18, ) → Ja escrit en forma de semirecta.

b) [0, 27)  [0, )(, 27) e) (0,11000] (0, )(,11000]

c) (0,100)  (0, )(,100)

88. Opera i arrodoneix el resultat als dècims.


a) 43,295 + 4,57 - 7,367 c) 3,56 · (7,4009 – 3,48)
b) 5,32 + 4,05 · 7,361 d) 7,37 – 5,3519 : 2,1
a) 43,295 4,577,367  40,498  40,5
b) 5,32 4,057,361  35,31205  35,3

 
c) 3,56 7,4009 3,48  13,958404 14,0


d) 7,37 5,3519 : 2,1  4,8214761904  4,8
89. Al llarg de la història s’han utilitzat diferents aproximacions del nombre ࣊ (de valor
3,14159265…).
 A la Bíblia, el valor de ࣊ és 3.
૛૞૟
 A l’antic Egipte s’estimava el valor en , fracció que resulta de suposar que l’àrea d’un
ૡ૚

cercle coincideix amb la d’un quadrat que tingui de costat de la mida del seu

diàmetre.

 A Mesopotàmia, el valor de ࣊ era 3 · ൌ ૜ǡ ૚૛૞.

૜૞૞
 A l’antiga Xina, .
૚૚૜
 I, finalment, en els càlculs pràctics s’utilitza 3.14.
Calcula l’error absolut i relatiu de cada aproximació prenent com a valor exacte de ࣊ =
3,14159265.
▪ Bíblia → Error absolut = 0,14159265
Error relatiu = 0,0450703
▪ A l’antic Egipte → Error absolut = 0,01890
Error relatiu = 0,006016
▪ Mesopotàmia → Error absolut = 0,01659265
Error relatiu = 0,0052816
▪ A l’antiga Xina → Error absolut = 2,70 · 10–7
Error relatiu = 8,60 · 10–8
▪ En càlculs pràctics → Error absolut = 0,00159265
Error relatiu =
0,00050696

27
 
 
 
 
 
 

Nombres reals
Nombres reals 1
 
 
 
 
 
 

90. Determina l’aproximació per arrodoniment fins als deumil·lèsims per a aquests casos:
a) √� � � √� c) √� � � √�
� �
b) � � √� d) � � √�
� ��

a) 3,1463 b) 3,5029 c) 0,5040 d) 3,0951

91. Calcula l’error absolut i l’error relatiu quan trunquem 5,73691 al centèsim.
El nombre 5,73691, trucat al centèsim, és 5,73, així els seus errors absolut i relatiu seran:

Ea
Ea  5,736915,73  0,00691 Er   0,00120
5,73691

92. Determina l’error absolut i relatiu quan arrodonim els nombres següents:

a) al deumil·lèsim,
��
b) 4,3964 al centèsim
��
c) al dècim,

3
a) al deumil·lèsim → 0,2727
11

Ea
Ea  0,272727 0,2727  0,000027 Er   0,000099
0,272727
b) 4,3964 al centèsim → 4,4

0,0035
Ea  4,3965 4,4  0,0035 Er   0,000796
4,3965
29
c) al dècim → 7,3
4

Ea
Ea  7,3 7,25  0,05 Er   0,0068
7,25

93. Aproxima el nombre � perquè l’error sigui més petit que un centèsim.

Perquè l’error absolut comès sigui més petit que al centèsim cal calcular el quocient amb dues xifres
decimals. L’aproximació demanada és 0,14.

94. Aproxima el nombre 12,3456 de manera que l’error absolut sigui més petit que 0,001.

Perquè l'error absolut sigui menor que al mil·lèsim, s'escriu el nombre amb tres xifres decimals. Per
tant, l'aproximació demanada és 12,345.

28
 
 
 
 
 
 

Nombres reals 11
 
 
 
 
 
 

95. Determina per a quin nombre 5.432,723 serà una aproximació als mil·lèsims per defecte. És
l’única resposta? Quantes respostes hi ha?

Resposta oberta

Una aproximació als mil·lèsims és 5 432,7231. La resposta no és única, ja que hi ha infinits nombres.

96. Calcula una aproximació als següents nombres.


a)  amb una cota d’error inferior a un mil·lèsim.
b) √� amb una cota d’error inferior a mig centèsim.
���√�
c)  =   amb una cota d’error més petita que 0,0001.

��
d) amb una cota d’error inferior a 0,00001.

1
a)  aproximació
  3,141  cota d'error absolut   0,0005  0,001
2103
1
b) 2 aproximació
 1,4142  cota d'error absolut   0,00005  0,0005
210 4

1 5 aproximació 1
c)      0,2236  cota d'error absolut   0,00005  0,0001
2 2∙10 4
22 aproximació 1
d)   3,14285  cota d'error absolut   0,000005  0,00001
7 2105

97. Indica quins d’aquests nombres estan escrits en notació científica:

a) 4,678 d) 9,34 · ���

b) 0,45 · ��� e) 4,62 · ����

c) 3,001 · ���� f) 34,709 · ���

Amb notació científica tenim: 3,001 · 1017 y 4,62 · 10–6.

98. Escriu en notació científica i indica’n la mantissa i l’ordre de magnitud:

a) 15 000 000 000 e) 4 598 000 000

b) 0,00000051 f) 0,0967254

c) 31 940 000 g) 329 000 000

d) 0,0000000009 h) 111 000

a) 5 000 000 000 = 5 · 109 Mantissa: 5 Ordre de magnitud: 9

b) 0,00000051 = 5,1 · 10-7 Mantissa: 5,1 Ordre de magnitud: –7

c) 31 940 000 = 3,194 · 107 Mantissa: 3,194 Ordre de magnitud: 7


29
 
 
 
 
 
 

Nombres reals
Nombres reals 1
 
 
 
 
 
 

d) 0,0000000009 = 9 · 10-10 Mantissa: 9 Ordre de magnitud: –10

e) 4 598 000 000 = 4,598 · 109 Mantissa: 4,598 Ordre de magnitud: 9

f) 0,0967254 = 9,67254 · 10–2 Mantissa: 9,67254 Ordre de magnitud: –2

g) 329 000 000 = 3,29 · 108 Mantissa: 3,29 Ordre de magnitud: 8

h) 111 000 = 1,11 · 105 Mantissa: 1,11 Ordre de magnitud: 5

99. Efectua aquestes operacions amb nombres en notació científica:


a) 1,32 · 104 + 2,57 · 104
b) 8,75 · 102 + 9,46 · 103
c) 3,62 · 104 + 5,85 · 10-3
d) 2,3 · 102 + 3,5 · 10-1 + 4,75 · 10-2
e) 3,46 · 10-2 + 5,9 · 104 + 3,83 · 102
a) 1,32 · 104 + 2,57 · 104 = 3,89 · 104
b) 8,75 · 102 + 9,46 · 103 = 1,0335 · 104
c) 3,62 · 104 + 5,85 · 10-3 = 3,620000585 · 104
d) 2,3 · 102 + 3,5 · 10-1 + 4,75 · 10-2 = 2,30975 · 102
e) 3,46 · 10-2 + 5,9 · 104 + 3,83 · 102 = 5,93830346 · 104

100. Calcula el resultat d’aquestes operacions:


a) 9,5 · 104 – 3,72 · 104
b) 8,6 · 103 – 5,45 · 102
c) 7,9 · 10-4 – 1,3 · 10-6
d) 4,6 · 106 + 5,3 · 104 + 3,9 · 102
e) 5 · 102 - 3 · 10-1 + 7 · 10-2
a) 9,5 · 104 – 3,72 · 104 = 5,78 · 104
b) 8,6 · 103 – 5,45 · 102 = 8,055 · 103
c) 7,9 · 10-4 – 1,3 · 10-6 = 7,887 · 10-4
d) 4,6 · 106 + 5,3 · 104 + 3,9 · 102 = 4,652610 · 106
e) 5 · 102 - 3 · 10-1 + 7 · 10-2 = 4,997 · 102

101. Efectua les operacions següents:

30
 
 
 
 
 
 

Nombres reals 11
 
 
 
 
 
 

102. Simplifica el resultat d’aquestes operacions:

103. Donats els nombres següents escrits en notació científica, calcula:


A = 2,7 · 108 B = 5,4 · 109 C = 7,1 · 1012
a) A · B : C c) A + B · C
b) B – A + C d) (B + C) : A
a) A · B : C= 2,7 · 10 · 5,4 · 109 : (7,1 · 1012)= 2,0535211 · 105
8

b) B – A + C = 7,10513 · 1012
c) A + B · C = 2,7 · 108 + 5,4 · 109 · 7,1 · 1012 = 2,7·108 + 3,834 · 1022 =3,834000000000027 · 1022
d) (B + C) : A = (5,4 · 109 + 7,1 · 1012) : (2,7 · 108) = 2,63 · 104

104. Donats els nombres següents en notació científica, calcula:


A = 3,2 · 106 B = 8,2 · 109 C = 5,1 · 10-6
c) A · B · C c) A + B · C
d) (A : C) · B d) A · C2
a) A · B · C = 3,2 · 106· 8,2 · 1011· 5,1 · 10–6 = 1,33824 · 1013

b) (A : C) · B = [3,2 · 106 : (5,1 · 10–6)] · 8,2 · 1011 = 5,1405 · 1023

c) A + B · C =3,2 · 106 + 8,2 · 1011 · 5,1 · 10–6 = 7,382 · 106

d) A · C2 = 3,2 · 106 · (5,1 · 10–6)2= 3,2 · 106 · 26,01 · 10–12 = 8,3232· 10–5

105. Calcula el valor numèric dels radicals que tens a continuació:

106. Escriu dos radicals equivalents a cadascun dels següents:


3 6 8
a) 25 c) 53 e) 26

31
 
 
 
 
 
 

Nombres reals
Nombres reals 1
 
 
 
 
 
 

12 20
b) 74 d) 23 f) 315
Resposta oberta. Per exemple:
a) 3
25  6 210  9 215 c) 6
53  5  4 52 e) 8
26  4 23  12 29

b) 12
74  6 72  120 740 d) 23  4 26  6 29 f) 20
315  4 33  8 36

107. Simplifica els radicals que tens a continuació:

3
i) 3
343
 73 7

108. Escriu en cada cas si el desenvolupament de la igualtat és cert o fals. Si és fals, corregeix-lo.

a) Fals 8  4 26  2 2 c) Fals 5
2510  54  58

b) Fals 3 34  6 94  3 3 3 d) Cert

109. Escriu aquestes potències d’exponent fraccionari com un radical:

3 4 3 4 1 1

2 2 2 3     79  2 3 16
2 3 5 30 79
a) 1
2 2  2
30
c) 55 5  53  515  15 516
2 5  

32
 
 
 
 
 
 

Nombres reals 11
 
 
 
 
 
 
1

 1  2
2  7 5 7
 1 1 3

47   7
    1
b) 3   32 : 3 3   3 36  36 347
4 d) 4
7 5 
5
  7 5 77

110. Escriu aquestes potències d’exponent fraccionari com un radical:

1
1 81 
i) 4  a 8
a a

111. Expressa mitjançant un sol radical.

1
 1 5 1 1 2 1
4 2 26
a) 3 5   35 2   35 510  310 510  32 5
5 10
f) 4 4  24
  3
3
6
3 3

33
 
 
 
 
 
 

Nombres reals
Nombres reals 1
 
 
 
 
 
 
1
 1 2 1 1

2  22   6 2 12 12  1 3
1 1 1
b)   1   2   2  2 g) 3 4
2  2 4   212  12 2
3
2  3    
2 

2 1 1 1 1
c) 32 h)  1
 1

4
3
2 5  1 2  5 4 5
5 2 
 
 
1
 1 2
 1  2  1
5 4 3
d) 3   32    3 8  8 3 i) 256  120 256  15 2
  
 
1
1
 2  1 2 1
1 1 1
e)  1   2 2   2 4 
  4
2  2   2
2
112. Extreu tots els factors possibles dels radicals següents:

a) a3b4 d) a3b4  a2 b2
3
b) a2 b 5 c 3 e) a3 b3  c 3
3
c) a3b2 c7 f) a4 c2  a4 b2

a) a3b 4  ab2 a d) a3b4  a2b2  ab ab2 1

b) a2b5c 3  ab2c bc e) 3
a 3 b 3  c 3  3 a 3b 3  c 3

c) 3
a3b2c 7  ac 2 3 b2c f) a 4 c 2  a 4 b2  a 2 c 2  b2

113. Extreu els factors que puguis d’aquests radicals:

 
3
a) 125  5 5 d) 250  5 3 2 g) 4
224  2 4 14
3
b) 80  4 5 e) 1080  6 3 5 h) 5
486  3 5 2
3
c) 189  3 3 7 f) 4
720  2 4 45 i) 3528  42 2

34
 
 
 
 
 
 

Nombres reals 11
 
 
 
 
 
 

114. Aquesta expressió amb radicals és un nombre enter. Calcula aquest nombre.
� �
�√�� � � √�� � � √� 
1 2 3

 
2 10  4 25  6 8  2 25 2 5 4 2 6  225  20    

115. Extreu factors dels radicals següents:

a) 32x 3 y 2 d) 4
256x 3 y15

b) 3
55 x6 e) 4
x 12 y 9 z19

a)  32x 3 y 2  4xy 2x           d)  4 256x 3 y 15  4y 3 4 x 3 y 3  

b)  3
55 x 6  5x 2 3 52            e)  4 x 12 y 9 z19  x 3 y 2 z 4 4 yz 3  

c)  3
125x 7 y 2  5x 2 3 xy 2         f) 5
729x 4 y 22 z15  3y 4 z 3 5 3x 4 y 2  

116. L’expressió 3  2 2  3  2 2 és un nombre enter. Determina’l.

Si fem el quadrat de l’expressió s’obté:


2
 
 3 2 2  32 2   32 2  32 2 2∙ 32 2  32 2  6 21  4
 

 
2  
 32 2  32 2   4  32 2  32 2  2  
 
 
117. Simplifica les expressions següents:

35
 
 
 
 
 
 

Nombres reals
Nombres reals 1
 
 
 
 
 
 

 
118. Simplifica les expressions següents:

a) 3 d) 3
b) 3 e) 3
c) 4 f) 6

119. Efectua les sumes i restes de radicals següents:


a) √��  ‐ √�  + √�� 
� �
b) � √�� + � √���
c) √�  + �√��√��  ‐ √�  + √�� 
d)   √�� ‐ 2√�� ‐ √���  + �√� 

a) 32  8  98  9 2

b) 5 3 81  4 3 108  15 3 3 12 3 4

2
c) 6  7 24  54  18  13 6  3 2
3

d) 75 2 12  363  4 3  6 3

120. Introdueix els factors dins del radical.


� �
a) � √� d) √�

� ��
b� � √�� e� � √�

� �
c� � √�� f� � √�
� � �
a) � √� = √�� � � = √��
� � �
b) � √�� = √�� � �� = √�� ���
� � � �
c) � √�� = � √�� � �� = √�� ��� f) � √�
� �� �� ��
d)

√� = � ��
= �
��

�� � ��� � � � �
e) √� =� =� = �
� �� �� �
� � �
f) � √� = √��� � = √��

36
 
 
 
 
 
 

Nombres reals 11
 
 
 
 
 
 

121. Introdueix els factors en el radical.



� � �� � � √�
a) �� b) � c) �
� � � � �


� � � �� � � �� � √� � �
a) � � = √�� b) � =� c) � =�
� � � ��� � � �����

122. Introdueix els factors dins del radical si és possible.


���� �
a) a ·� c) -2a�� √�� e) 5 + √�
��

��� � �� � � ��
·�

b) d) · � f) -�� √�
� �� � �

���� �� ������ ��� ��


a) a ·� = � =�
�� �� �

��� � � �� � � � �� �� �� � � � � �� �� �� � �� � � � �
b) ·� = � = � = �
� �� � � �� �� �� � ��

�� � �
c) -2a� √�� = √��� �� � � �� = √��� �� � �

� �� �� �� �
d) ·� =� = �
� � �� � � ��

e) No és possible introduir factors, ja que 5 no és factor.


� � �
f) -�� √� = √��� � = √���

123. Efectua les operacions següents i simplifica:

  
a)  5 2  3 2 2  16 13 2             d)   2 2 3 3 2   3 2 26 3  2 6  

b)  12 5 3 2   3 2 6 5  2 10       e)   2  5 4 2  3  231 2   

c)   3  55 2 3   3115 3           


f)  2 7 5 7  3 5   14 15 5 5 7 6 35  

124. Expressa amb un sol radical el resultat de les operacions següents:

a)  4
53  6 53  53  12 59  12 56  12 518  52 4 53   

b)   3

72 8  4 85 : 783  12 78 819 : 12 76 818  12 72 8
 
37
 
 
 
 
 
 

Nombres reals
Nombres reals 1
 
 
 
 
 
 

c)  3 45  4 2 42  8 45 53  8 34  4 4 54  8 22  4 4  8 45 53  8 22 34  413 57  8 228 34 57  23 8 24 34 57
 
d)  3
235 :  3
 6 2 2 2 6
23  23  2 3 5 : 2 3  2 3 5   5 5 6 3 3 2

 
125. Expressa amb un sol radical el resultat de les operacions següents:
� � � � �
a) √�� � √�� � √�� c) √��� � � : √��� �
� � �
b) √��� � � √��� � d) �√�� � �� √�

126. Calcula el resultat d’aquests productes:

d)

127. Efectua les operacions que tens a continuació i simplifica:

38
 
 
 
 
 
 

Nombres reals 11
 
 
 
 
 
 
1 1 13 13
4
23 24  3 2 2 4 24 2 3 212 212 1 12
2
a)  5
 1 5
 4  48   3  
2
  2 12
2 35 2
2  2 2 2
22 2 2 2 2
2 12

3
d)  6  20  47 4 16  6  3 20  7  6  3  9  3  

 
128. Efectua les operacions amb radicals que tens a continuació:

2 2 2
      2

 a a   16a  9a   25a   5  144


a)      a    
 9 16     
144   144   12  25a
  
4 4 4
 
a 2a  9a 
2
 9a  2  10  100
b)          
 2 5   10   10   9a  81a
2
   
 
2
 
a a a
c)    2a   2a  2a 
 2  2 2
   
2
 
2a  2a 2a 32a
d)   6a   6a   4a  10a    
 3  3 3 3

   

39
 
 
 
 
 
 

Nombres reals
Nombres reals 1
 
 
 
 
 
 

129. Simplifica els radicals següents:



a) √�� � � � �� b) � � ��� � ��

a 2
2
a) a2  4  4a   a 2

2
 
1 2 2
b) 2a2 2a    2a    2a
2  
 2  2

130. Racionalitza les expressions següents i simplifica’n el resultat:


� �� √�
a) c) � e) �
�√� √� √�
√� √� �√�
b) � d) � f) �
√�� � ��� √�
4
3 3 3 33
a)  d) 
4 3 4 3 4 33 9

2 2  3 232 6
23 22 34 6 25 34 2
b)    e) 1
3
12 3 2
2 3  23 3 2 23 6 4
4

12 3 3
c)  43 3 f)  33 3
3 6
9 3

131. Racionalitza aquestes expressions i simplifica’n el resultat:


��√� �√���
a) � c) �
√� ���

�√��� �√���
b) � d� �
√� ����

a)
3 2

3

2 3 2  3 2 3 6
25
c)
3 5 2 3 5  4 5 2 4 5 3 4 53 2 4 5
 
3
4 2 2 4 3
5 5 5

b)
7 7 7
3
7
  7 1  3
72 d)
3 5 1
5
53

310 57  5 52
5

132. Elimina les arrels del denominador.

40
 
 
 
 
 
 

Nombres reals 11
 
 
 
 
 
 

 
 
133. Racionalitza les expressions següents:

a)
2
3 2
 2  3 2  e)
2 3
 3 1

  3 1 3 
1 7 5 2 3 2
b)  f)  2 6
5 7 2 3 2 2

3 3 8 2 4 3 2 6
c)  3 2 g) 
2 2 2 6 2 3 3

2 3 6 2 2 3 5 30 3 15
d)  h) 
3 3 6 7 2 5  3 17

134. Elimina arrels del denominador de les expressions que tens a continuació:

41
 
 
 
 
 
 

Nombres reals
Nombres reals 1
 
 
 
 
 
 

6
3
5 52 2 6 55
a) 
12 5 19

b)
5
2

5
2

5
2  2 3 3  10
27  310 22 35
2 3  2 5 3 2 3 3 227 25

c)
3 3 5
2 3 5

3
43  6
200  3 6 3125 
d)

8 5 18   2 2 1
2 8 2

e)
12

2
 5 2 
27  5 2  3

f)
34 2
2 8 5

3 4
13 
3 8  4 50 
135. Racionalitza les expressions següents i simplifica’n el resultat:

42
 
 
 
 
 
 

Nombres reals 11
 
 
 
 
 
 

136. Racionalitza aquestes expressions:

a)
3 5
3 6

5 5
3 7

1
33 5  
3 6 
5
4
5  
3 7 

12 3 12 6 19 15  4 30 15 35



12

b)
12 6
2 3 3 2
 12  2 3 
137. Racionalitza les expressions següents:

 
 
138. Efectua aquestes operacions:

43
 
 
 
 
 
 

Nombres reals
Nombres reals 1
 
 
 
 
 
 

 
 
139. Efectua aquestes operacions:

2 1 30  9 5 1 1 1 5  3 52  6 55
a)   c)   
3 2 5 5 55 5 3
5 6
5 5

b)
2

1
3
1 3 2 3 7 37
13 3 10
   d)
5
5  10

2 5
4

1
2

15 2 43

46 23

140. Efectua l’expressió següent:

128 2 8  3 2 8 2 4 2 3 2 15
4 2  4 2  4 2  
2 32 8 2 8
141. Calcula, a partir de la definició els logaritmes següents:

 
 
142. Utilitzant la definició, calcula els logaritmes següents:

5 2
a) log 9 243  c) log32 4 
2 5
3 9
b) log25 125  d) log 4 512 
2 2

143. Determina quines de les igualtats següents són certes i corregeix les que no ho siguin:
44
 
 
 
 
 
 

Nombres reals 11
 
 
 
 
 
 

   
a) Falsa: log a  b  loga logb  log ab  loga logb

b) Falsa: log0 1 log1  0

 
c) Certa: log a : b  loga logb

d) Falsa: log  a   logbloga  log  a   bloga


b b

144. Calcula el resultat d’aquestes expressions aplicant les propietats dels logaritmes:

 
 
145. Si sabem que log 7 = 0.8451 calcula aplicant les propietats dels logaritmes:
log 28 + log 15 – log 6
 2815 
log28 log15log6  log    log70  log7 log10  log71  1,8451
 6 

146. Calcula els logaritmes següents utilitzant la calculadora:

 
 
147. Si log e = 0.4343, quant val ln 10 ? i ln 0,1?

45
 
 
 
 
 
 

Nombres reals
Nombres reals 1
 
 
 
 
 
 
‫܏ܗܔ‬૚૙ ૚
‫ܖܔ‬૚૙ = = = 2,3025
‫܍܏ܗܔ‬ ૙ǡ૝૜૝૜
‫܏ܗܔ‬૙ǡ૚ ି૚
‫ܖܔ‬૙ǡ ૚= = = -2,3025
‫܍܏ܗܔ‬ ૙ǡ૝૜૝૜

148. Si sabem que ln 4 = 0,6021, calcula els logaritmes següents:


a) log 2 c) log 0,2 

b) log d) log 4 000 

log 4
a)  log2   0,30105  
2
1
b)  log  log4  0,6021  
4
log4
c)  log0,2  log10  0,301051  0,69895  
2
d)  log4000  log4 log1000  0,3021 3  3,6021  

 
149. Si sabem que ln a = 0,6 i que ln b = 2,2 calcula els logaritmes següents:

 
 
 

1
a) ln a  ln a  0,3
2
1
b) ln 3 b  ln b  0,7333
3

ab 1
c) ln 4 
 lna lnb 2lne  0,2
e2 4

a5 5 1
d) ln   ln a  ln b  2,23333
3
b 2 3

150. Calcula el valor de x.

 
 

a) ࢒࢕ࢍ࢞ ૜ ൌ  െ૚ ՜  ࢞ି૚ ൌ ૜ ՜ ࢞ ൌ 

46
 
 
 
 
 
 

Nombres reals 11
 
 
 
 
 
 

b) ���� � � ��� � � �� � �� � � � � √�

� �
c) ���� � � � ��� � � ��� � �� � �� � ����
� �


d) ���� � � ��� � � �� � �� � � � √�


e) ���� � � � ��� � �� � ���� � � � �
���


f) ���� � � � ��� � �� � ���� � � � �
���

g) ���� �� � �� � ��� � � � ��

h) ����� � � �� � � � � ��� � � � �������.

151. Indica si les afirmacions següents són certes o falses. Justifica la resposta.
a) Tots els nombres decimals es poden escriure en forma de fracció.
b) Tots els nombres reals són racionals.
c) Qualsevol nombre irracional és real.
d) Hi ha nombres enters que són irracionals.
e) Hi ha nombres reals que són racionals.
f) Tot nombre decimal és racional.
g) Cada nombre irracional té infinites xifres decimals.
h) Tots els nombres racionals tenen infinites xifres decimals que es repeteixen.
i) Tots els nombres racionals es poden escriure mitjançant fraccions.
a) Falsa, els nombres irracionals tenen infinites xifres decimals no periòdiques i no es poden escriure
com a fracció.

b) Falsa, perquè hi ha nombres reals que són irracionals.

c) Certa, els nombres racionals i els irracionals formen el conjunt dels nombres reals.

d) Falsa, perquè si són enters no poden tenir infinites xifres decimals no periòdiques.

e) Certa, tots els nombres que es poden expressar com a fracció, són nombres reals, que, a més a
més, són racionals.

f) Falsa, perquè els nombres decimals amb infinites xifres decimals no periòdiques són irracionals.

g) Certa, tenen infinites xifres decimals no periòdiques.

h) Falsa, els decimals exactes també són racionals.

i) Certa, per definició.

152. Per què l’arrel quadrada de qualsevol nombre acabat en 2 és un nombre irracional? Hi ha algun
altre conjunt de nombres amb aquesta característica?

47
 
 
 
 
 
 

Nombres reals
Nombres reals 1
 
 
 
 
 
 

Perquè no hi ha cap nombre que, en multiplicar-se per ell mateix, doni un nombre acabat en 2.

Totes les famílies de nombres acabats en 3, 7 i 8 tenen aquesta característica.  

153. Escriu en notació científica les quantitats següents:

a) L’any llum: 9 460 000 000 km

b) Velocitat de la llum: 300 000 km/s

c) Diàmetre del Sol: 1 400 000 km

d) Càrrega elèctrica de l’electró: 0,0000000000000000001602 C

e) Massa del protó: 0,0000000000000000000001673 kg

f) Distància de Mercuri al Sol: 58 000 000 km

g) Massa de l’electró: 0,00000000000000000000000000009109 kg

h) Distància entre la Terra i la Lluna: 384 000 000 m

a) Any-llum 9 460 000 000 km = 9,46 · 109 km

b) Velocitat de la llum → 300 000 km/s = 3 · 105 km/s

c) Diàmetre del Sol → 1 400 000 km = 1,4 · 106 km

d) Carga elèctrica del electró → 0,0000000000000000001602 C = 1,602 · 10-19 C

e) Massa del protó → 0,0000000000000000000001673 kg = 1,673 · 10–22 kg

f) Distància de Mercuri al Sol → 58 000 000 km = 5,8 · 107 km

g) Massa de l’electró → 0,00000000000000000000000000009109 kg = 9,109 · 10–29 kg

h) Distància entre la Terra i la Lluna → 384 000 000 m = 3,84 · 108 m

154. Els formats de paper estàndard es basen en una norma internacional. Aquests formats de
paper tenen unes mides que, quan es talla per la meitat un dels rectangles estàndard,
s’obtenen dos rectangles semblants al primer. Així, si es divideix per la meitat un full DIN A4
en surten dos rectangles iguals (de la mida DIN A5) semblants al primer.

 
a) Quina relació hi ha entre els costats dels rectangles?

48
 
 
 
 
 
 

Nombres reals 11
 
 
 
 
 
 

b) Se sap que un rectangle de format DIN A0 té una àrea d’1 m2. Determina quines són les
dimensions del full de format DIN A4.

���
a) L’àrea de l’A4 és a · b, el doble que la de l’A5, que és .

� �
b) � � � �� → ��� � � ��

155. La distància entre la Terra i Júpiter és de 6,32 · 106 km. Una nau que fes el viatge entre els dos
planetes en un any, a quina velocitat hauria d’anar?

����� ���������
Un any en hores és 24 · 365 → v = = = 7,2 · ��� km/h
����� ���������

156. Des de l’antiguitat apareix sovint el nombre d’or,  , , en proporcions de la natura però també
en obres d’art o bé en construccions com, exemple, el Partenó grec.

���√�
=

Comprova la propietat següent de l’invers del nombre d’or:



=  –1


 1 1

2

22 5

5 1
   1 5  1 5 4 2
1   
  1      
   2 
 1 5 5 1
  1  1 
 2 2
I es comprova que ambdós costats de la igualtat mesuren el mateix.


157. Quin tipus de decimal obtenim de la fracció , on a és un nombre enter?
�� ����

Com que el sistema de numeració és decimal, en dividir un nombre enter entre un nombre que sigui
potència de 2 o de 5, o de tots dos, s’obté un decimal exacte. Si el numerador és múltiple del
denominador, s’obté un nombre enter.

49
 
 
 
 
 
 

Nombres reals
Nombres reals 1
 
 
 
 
 
 

158. Existeix algun cas en el qual l’aproximació per excés i per defecte coincideixin?

Si considerem l’arrodoniment, determina si aquesta aproximació pot coincidir amb


l’aproximació per excés o per defecte?

No poden coincidir, ja que per aproximar per defecte s’eliminen les xifres a partir de l’ordre considerat,
i per aproximar per excés s’eliminen les xifres a partir de l’ordre considerat, mentre s’augmenta una
unitat l’última xifra que es manté.

L’aproximació per arrodoniment coincideix amb l’aproximació per defecte si la xifra anterior a l’ordre
considerat és més petit que cinc, i coincideix amb l’aproximació per l’excés a la resta de casos.

159. Comprova cadascuna de les igualtats que tens a continuació:


� � ���
a) √� · √� = √��
� � ���
b) √� · √� = √��
� � �
c) √� � � = √� + √�

d) a √�� = ���� � ���

e) √� � √� � √� � √� � �√� � �
f) �√� � � = √�� � ��

g) √�� �� = �√�
h) √��� �� = � � �
1 1 m n
n
a) Fals: a  m b  n∙m ab atès que n
a  m b  a n b m  a nm b mn  nm am bn

b) Fals: n
a  m b  nm ab  n a  m b  nm am bn
1 1 1
c) Fals: n
 
a  b  n a  n b ja que n a  b  a  b n  a n  b n  n a  n b
m n m

 a ∙ b  ja que a n bm a  b n a n  b n n a n bm
m
d) Fals: a n b m  n

e) Cert: a  a  a  b  a ab

f) Fals: a b  c  ab  ac  a b  c  a2b  a2c


4
g) Fals: a 8b2  a b  4 a 8 b2  a 2 b

h) Fals: a2  b2  a  b  a2  b2 2ab  a  b

160. Escriu el nombre ���� en notació científica.


a) Sabent que log 2 = 0,3010 i que √�� = 3,1622.
b) Ho podries fer amb una calculadora científica?
c) Expressa ���� en notació científica tenint en compte el primer apartat.

50
 
 
 
 
 
 

Nombres reals 11
 
 
 
 
 
 

a) Anomenem x al nombre: ૛૞૙૙ = x


Hem de trobar y on ૚૙࢟ = x.
࢒࢕ࢍ࢞
૛૞૙૙ = x 500 = log2 x =
࢒࢕ࢍ૛

D’altra banda com,࢒࢕ࢍ࢞ ൌ ࢟ǡ


y = 500 · log 2 = 150,5
૚૙૚૞૙ǡ૞ = ૚૙૙ǡ૞ ൉ ૚૙૚૞૙ = 3,1622 · ૚૙૚૞૙
b) No es pot trobar amb calculadora, ja que és un nombre massa gran.
c) Anomenem x al nombre ૞૞૙૙ ൌ ࢞
Hem de trobar y perquè ૚૙࢟ = x:
࢒࢕ࢍ࢞
૞૞૙૙ = x 500 = log5 x =
࢒࢕ࢍ૞
D’altra banda, com que log x = y:
y = 500 · log 5 = 349,5
૚૙૜૝ૢǡ૞ = ૚૙૙ǡ૞ · ૚૙૜૝ૢ = 3,1622 · ૚૙૜૝ૢ
   

51
 
 
 
 
 
 

Nombres reals
Nombres reals 1
 
 
 
 
 
 

PER APROFUNDIR 
161. Escull la resposta adequada.

     
2 2 2
log x y log y x  7  log x y log y x  49  log x y  log y x 2log x ylog y x  49 
log y log x
 log y   log x  2  49  log y   log x  2  49  log y   log x 
2 2 2 2 2 2

x y x y x y
 47
□ log x log y

□ Elevant ambdós termes al quadrat s’obté:


2
 
 96 2  9 6 2   9 6 2  9 6 2 6  24  
 

Així, quan calculem la seva arrel quadrada per obtenir el valor inicial, resulta 24  2 6 .

□ Perquè tingui un nombre imparell de divisors ha de ser un nombre l’arrel quadrada del qual sigui
exacta; per tant, només resten com a possibles els nombres de tres xifres menors de 322.

Els únics dos nombres que compleixen aquesta propietat són 182 = 324 i 242 = 576.

1 1
□ 
1

1
 log3 10  2  log 3 10  3 log3 10  2,3    
log 1   log 1  
3
2 
3
5 

x xy
□ y
x

x  y 1
 
 y x  y 1  x  y  yx  y 2  y  x  y  0  y 2  xy  x  0
x
xy

3 9 12
Quan x = –3, y  .
2
ࢇ ࢇା࢈ ࢇି࢈
162. Si una fracció ࢈ és irreductible, digues si les fraccions i també són irreductibles?
ࢇ൉࢈ ࢇ൉࢈
Com que els divisors de a + b són els divisors comuns de a i b:
ࢇା࢈
(a + b) i a · b no tenen divisors comuns, i la fracció és irreductible.
ࢇ൉࢈
Com que els divisors de a – b són els divisors comuns de a i b:
ࢇି࢈
(a – b) i a · b no tenen divisors comuns, i la fracció és irreductible.
ࢇ൉࢈

52
 
 
 
 
 
 

Nombres reals 11
 
 
 
 
 
 

163. Explica com es racionalitzen les fraccions del tipus:



�� ��
√� � √�
Multipliquem el denominador pel conjugat:

Per tant, multiplicant pel conjugat n vegades:

 
 
164. Dues peces mòbils d’una màquina es desplacen a la mateixa velocitat.

La primera peça descriu una circumferència de 5 cm de radi i la segona es desplaça d’un extrem
a l’altre del diàmetre d’aquesta circumferència.

Si les dues peces surten d’un mateix punt, coincidiran en algun moment?

CLAU: Considera el següent esquema, a més a més, recorda que  és un nombre irracional:

Suposem que ambdues peces parteixen d’A. Anomenem v a la


velocitat que porten els dos mòbils. La distància recorreguda pel
mòbil que es desplaça per la circumferència en els punts A i B és
5π(k − 1), essent k un nombre natural. La distància recorreguda
pel mòbil que es desplaça pel diàmetre en els punts A i B és 10(k
− 1), essent k un nombre natural. Les distàncies recorregudes pel
mòbil que es desplaça per la circumferència són nombres
irracionals, mentre que les distàncies recorregudes pel mòbil que es desplaça pel diàmetre són nombres
naturals.

Per tant, mai coincidiran tots dos mòbils.

165. Demostra la igualtat següent:


���
∑��
��� ���� ��1

��� � ��� � ���


∑��
��� ���� �� ∑��
��� ��� �� ∑��
��� =
� � � � �

� �
= ∑��
��������� � �� � ��� �� = (log 100 – log 1) =1
� �

53
 
 
 
 
 
 

Nombres reals
Nombres reals 1
 
 
 
 
 
 
� � �
166. Demostra que � � , expressat en forma decimal, és un nombre mixt per a qualsevol
� ��� ���
valor de n.
Perquè una fracció irreductible generi un nombre decimal periòdic mixt ha de tenir al denominador algun
factor primer 2 o 5 i algun que no sigui 2 ni 5.

Trobem la suma:

� � � ��� � �� � �
� � �
� ��� ��� ��� � ���� � ��

Veiem que és irreductible, ja que el numerador no s’anul·la per cap valor natural de n i, a més a més,
el denominador és el producte de tres nombres consecutius i té com a divisors, com a mínim, el 2 i el
3.

167. Apilen en capes un munt de taronges, de manera que en el forat de 4 taronges d’una capa en
col·loquen una altra de la capa superior.

La primera capa, comptant per sota, té m files i n columnes, i l’última capa té només una fila; si
m és el nombre de diagonals d’un decàgon i n és el nombre més petit que dividit entre 4 dóna
residu 3, entre 5 dóna residu 4 i entre 6 dóna residu 5, quantes taronges hi ha?

La quantitat de files, m, és igual al nombre de diagonals del decàgon:

��������
m= � ��

La quantitat de columnes, n, és un nombre tal que n + 1 és el més petit múltiple comú de 4, 5 i 6, que
és 60, per això n = 59.

La quantitat de taronges que hi ha a cada capa és:

1a capa: 59 · 35 = 2.065

2a capa: (59 – 1) · (35 – 1) = 2.065 – 94 + 1

3a capa: (59 – 2) · (35 – 2) = 2.065 – 2 · 94 + 22

...

34a capa: (59 – 33) · (35 – 33) = 2.065 – 33 · 94 + 332

35a capa: (59 – 34) · (35 – 34) = 2.065 – 34 · 94 + 342

Fem la suma:

2.065 · 35 – (1 + 2 + 3 + ... + 34) · 94 + (1 + 22 + 33 + ... + 342) =

���� ��������������
= 72.275 - · 94 + = 30.030 taronges
� �

MATEMÀTIQUES A LA TEVA VIDA 
54
  2.065 · 35 – (1 + 2 + 3 + ... + 34) · 94 + (1 + 22 + 33 + ... + 342) =
 
 
 
 
  = 72.275 -
����
· 94 +
��������������
= 30.030 taronges 1
� �
Nombres reals 1
 
   
 

MATEMÀTIQUES A LA TEVA VIDA 
 
 
 

 
1. Contesta.
a) Per què es fan marques a la carretera quan un cotxe frena bruscament?
35 
  Quin és el valor de la gravetat g?
b)

a) Perquè dos o quatre rodes es bloquegen i es produeix una transferència del pes del cotxe sobre
elles.

b) g = 9,8 m/s2

2. Consulta què és el coeficient de fregament d’una superfície.

El coeficient de fregament és una constant adimensional que expressa l’oposició al lliscament que
ofereixen les superfícies de dos cossos en contacte. És una característica de cada parell de materials
en contacte. A més, depèn de factors com la temperatura, la velocitat relativa entre les superfícies, etc.

3. Quines magnituds representen les variables µ, g i x en l’expressió de la velocitat inicial respecte


de la distància de frenada? 

µ representa el coeficient de fregament. És una magnitud sense unitats.

g representa l’acceleració de la gravetat. És una constant fixa, que val 9,8 m / s2 .

x representa la longitud de les marques de frenada expressades en metres.

4. És correcta aquesta igualtat? 

v = ��μ�� = √� ·√μ ·��� ·√� ·

Si, la igualtat és correcta, ja que s’ha aplicat una propietat elemental de les arrels.

5. Quin és l’índex de l’expressió radical? 

L’índex de l’expressió radical és 2 (arrel quadrada)

6. Calcula la velocitat d’un automòbil si sabem que ha frenat i ha deixat una marca de frenada de
30 m en una carretera d’asfalt?  

Prenem µ = 0,75 per ser d’asfalt la carretera i llavors:

v = �� � �� �� � �� � � �� = √��� = 21 m / s = 75,6 km / h

55
5. Quin és l’índex de l’expressió radical? 

Nombres reals
L’índex de l’expressió radical és 2 (arrel quadrada)

6. Calcula la velocitat d’un automòbil si sabem que ha frenat i ha deixat una marca de frenada de
30 m en una carretera d’asfalt?  
 
 

Prenem µ = 0,75 per ser d’asfalt la carretera i llavors:


 
 
 
 

v = �� reals
Nombres � �� �� � �� � � �� = √��� = 21 m / s = 75,6 km / h 1
 
 
 
 
 
 

7. Esbrina quines són les campanyes dels responsables de trànsit de Catalunya per evitar
accidents. 
36  Algunes campanyes per evitar accidents són:
 
▪ Campanya de l'ús del cinturó de seguretat i sistemes de retenció infantil adequats a l'alçada i pes dels
nens.

▪ Campanya contra les drogues i l'alcoholisme

▪ Campanya contra les distraccions al volant, com l'ús del telèfon mòbil.

▪ Campanya de control de motocicletes i ciclomotors.

56
 
 
 
 
 
 

2
Sucessions numèriques.
 
 
 
 

Successions numèriques. 
Progressions  

 
Progressions 
 
 
 
 
 

ACTIVITATS 
1. Calcula els cinc primers termes de les successions següents.
a) �� � ����� � �
���
b) �� �
��� ��

a) �� � ����� � � � ��
�� � ����� � � � �
�� � ����� � � � ��
�� � ����� � � � �
�� � ����� � � � ��
��� �� ��
b) �� � � �
���� �� � �
2�3 �1
�2 � �

2 � 2 � 2 10
3�3 0
�3 � �
� �0
2 � 3 � 2 20
��� 1
�4 � �
���� �� 34

5�3 2 1
�5 � �
� �
2 � 5 � 2 52 26

2. Calcula els termes 20è i 40è de les dues successions anteriors.
a) ��� � ������ � �� � ��
�40 � ��1��� � 40 � 40
���� ��
b) ��� � �
����� �� ���
���� ��
�40 � �
����� �� ����


3. Escriu una successió la llei de recurrència de la qual sigui: a1 = 5, an = na-1 + 5
Resposta oberta. Per exemple:
5, 10, 15, 20, 25, 30, ... 
 
4. Escriu una successió la llei de formació de la qual sigui:
�� � �
�� �  
�� � �

Resposta oberta. Per exemple:

57
 
 
 

Successions numèriques. 2
Sucessions numèriques. Progressions
Progressions
 
 
 
 
 
 
�� �� � � � � �� � � � �
, , 0, , , , , ...  ‐2, , 0, , , , , ...
� � �� �� �� �� � �� �� �� ��


5. Troba el terme general de les successions següents:
a) {‐1, +1, ‐1, +1, ‐1,...} 
b) {1, 8, 27, 64,...} 
c) {8, 27, 64, 125,...} 
a) �� � ���� � �������
b) Si ens fixem, observarem que els nombres d’aquesta successió es correspon amb la potència 3.
Per tant, el terme general serà:
�� � ��  
c) Si ens fixem, observarem que els nombres d’aquesta successió mantenen la relació de l’exercici
anterior, però amb un decalatge respecte n, per tant:
  an = (n+1)3 
 
6. Donada la progressió amb a1 = 3 i d = 5, calcula el terme 25è.
Recordem les expressions que caracteritzen les progressions aritmètiques
a1  an = an‐1 + d    an = a1 + (n ‐1)d 
Així,
a25 = 3 + (25 ‐1)5 = 123 
 
7. Calcula el terme 1 000è d’una progressió aritmètica el primer terme de la qual és 20 i la diferència
és 4.

a1000 = 20 + (1000 – 1) ∙ 4 = 4016 
 
8. Si el quart terme d’una progressió aritmètica és 7 i un terme és 25, calcula quin lloc ocupa
aquest terme.

Resposta oberta.
Té múltiples solucions com es pot observar a la següent taula:
  n  4  5  6  7  8  9  10  11  12  13    n 
P1  an 7  9  11  13  15  17  19  21  23  25    13 
P2  an 7  10  13  16  19  22  25          10 
P3  an 7  13  19  25                7 
P4  an 7  16  25                  6 
 
Això representa una part de les que donen resposta a aquests condicionants.    

58

40 
 
 
 
 
 
 

Equacions i inequacions 23
 
 
 
 
 
 

9. Calcula la suma dels 25 primers termes de la progressió aritmètica an = 2n – 5.


�� ���
Recordem que �� � ��

a1 = 2 · 1 – 5 = – 3
a25 = 2 · 25 – 5 = 45
�� � ��
��� � � �� � ���

 
10. Troba la suma de tots els nombres parells més petits que 151.
an = 2n < 151 
n = 75 
� � ���
��� � � �� � ���� 

 
11. La suma de set nombres parells consecutius és 140. Esbrina aquest nombres.

n = 7; 
d = 2;  
an = a1 + (n ‐1)d→ a7 = a1 + (7 ‐1)d= a1 + 6∙2 
� � � �� �� � �� � ��
�� ��→ � � � ��� → 
� �
→ ��� � �� � � � ��� → ��� � �� � �� → �� � ��
Així la progressió és: 14, 16, 18, 20, 22, 24, 26.

12. Troba el terme general i la suma dels n primers termes de la progressió aritmètica {0, 5, 10, 15,
20,…}.

d = 5; 
�� � �; 
an = a1 + (n – 1) ∙ d→ an = (n – 1) ∙ 5 = 5n – 5  
�� � �� �� � � �
�� � � � → �� � � � � ��� � �� 
� � �
 
13. El primer terme d’una progressió geomètrica és 0,3 i el tercer terme és 0,012. Calcula’n la raó i
escriu-ne els cinc primers termes.

�� � �� �
�� � �� ���

�� �� ��� � �
�� � �� � ���� → �� � �� � �� → � � � �� �� �
�� �� � �� �

59
 
 
 

Successions numèriques. 2
Sucessions numèriques. Progressions
Progressions
 
 
 
 
 
 


�� � �� � �
��
� �
�� � �� � � �  
� ��
� � �
�� � � �  
�� � ���
� � �
�� � �� � � � � �  
��� � ����
� � �
�� � �� � � � � �  
���� � ����
 
� ��
14. D’una progressió geomètrica sabem que �� � i �� � . Calcula’n el terme general.
� ���


�� � ;

��
�� � ;
���
��
�� �
� � ��� �
�� � �� � � ���
� �� � �� � � � � � � � � ���

� � � �
�� ���� �


�� � ��
�� � �� � � � �� � � � � � � 
� ��

� ���
�� � �� � ���� � �� � � � � �  

15. En una progressió geomètrica sabem que r = 2 i a7 = 96. Calcula’n el terme general.
Apliquem a la fórmula general el cas particular que coneixem per aïllar �� .
�� �� �� �
�� � �� � �� � �� � � � �  
� � �� �� �
I ara sí que podem escriure l’expressió general:
� ���
�� � �� � ���� � �� � �� � � � ����  


16. Troba el terme 15è d’una progressió geomètrica si saps que a2 = 5 i r = .

Apliquem a la fórmula general el cas particular que coneixem per aïllar �� .
�� � ��
�� � �� � � � �� � � � � � ��� �
� �

I ara sí que podem escriure l’expressió general:
��
�� � ���
�� � �� � ���� � ��� � �� � � � �� ����� � ����  
� � ���

60

42 
 
 
 
 
 
 

Equacions i inequacions 23
 
 
 
 
 
 

17. Troba el producte dels sis termes d’una progressió geomètrica se saps que �� � � i r = 0,1.
Sabem que �� � ���� � �
���
� �
�� � �� � �� � � � � � � �� �����
��
Aplicarem la fórmula coneguda del producte dels primers termes d’una progressió geomètrica i
obtindrem:

� � � �� �
�� � ���� � �� �� → �� � ���� � �� �� � ��� � � � � � � � �� � � � � � ��
�� �� � ��
 
18. Troba el producte dels cinc primers termes d’una progressió geomètrica si saps que �� � � i
�� � � .

Repetim els processos aïllant la variable que ens interessa i utilitzant les dades conegudes.

� �� � � �
�� � �� � �� → � � � � � � √�
�� �



�� � �� � � � � √� � ��

Repetirem el procés final de l’exercici anterior i obtindrem:

� �� � �� ��
�� � ���� � �� �� → �� � ���� � �� �� → �� � ��� � �� �� � ��� � � � � � � � � � ����� �� 

 
�����
19. El producte dels cinc primers termes d’una progressió geomètrica és i el primer terme
�������
és a1 = 2. Calcular r i el terme següent.

�����
�� � ���� � �� �� → � ��� � �� ��  
�������


����� � � ����� � � �����
→ � �� � �� � → � �� � �� � → �� � � � � �� ���� 
������� ������� �
������� � ��

� �� � �� ����
�� � �� � �� → � � � � � � �� � 
�� �

�� � �� � � � �� � 
�� � �� � �� � � � �� �� � �� ����� 
 
20. Calcula la suma dels 10 primers termes d’una progressió geomètrica el primer terme de la qual
és a1 = 2 i la raó és r =√�.

61
 
 

Successions numèriques. 2
Progressions
Sucessions numèriques. Progressions
 
 
 
 
 
 
��
�� � � √� � �
�� � � � � → ��� � � � � ���, ������ 
��� √� � �
 
21. Calcula el valor de n de manera que es verifiqui que 4 + 42 + .... + 4n = 5.460

��
�� � ��

�� � � �� � � �
�� � � � � → �� � � � � ���� → �� � � � ���� � → 
��� ��� �

→ �� � � � ���� � � ���� 


→ � � ���� 
→ � � ���� ����� � � 
 
22. La raó d’una progressió geomètrica és r = √� i el tercer terme és √���. Calcula la suma dels
sis primers termes de la progressió.

�� √���
�� � �� � �� → �� � �
�� �

�� � � √��� √� � �
�� � � � � → �� � � � ���, ���� 
��� � √� � �
 
23. En una progressió geomètrica, la suma dels tres primers termes és 42 i el terme central és 12.
Calcula’n el primer terme i la raó.

�� � �� � �

�� ��
�� � � � � � → � � � �
� �

�� � �� � � � �� � �� ��
� �� � �� � � � �� → �� � � � � � �� � �� → � → � � � � �� � ��→
� � � �� ��
��
→ �� � �����
��
2
���  
0,5
Com ja hem, vist es tracta d’una mateixa solució ja que són els nombres inversos 2 i ½.
�� 12
�� � � � � 
� 2
�� 12
�� � � � � 2� 

62

44 
 
 
 

Equacions i inequacions 3
 
 
 
 
2
 
 

 
 
24. Calcula la suma dels infinits termes de la progressió geomètrica el terme general de la qual és

�� � .
��
� �
�� � � � � �
� �

�� �


��
�� � � ��
��� ���

25. Calcula el valor de ����√� …

√�, ��√�, ����√�, ������√�, … 

�� � √� 

�� � ��√� 

�� � ����√� 

�� ��
Per tal que una progressió sigui geomètrica s’ha de complir que � � �  
�� ��

�� ��√� ����√� ��
� � �  
�� √� ��√� ��
Això implica que no es tracta d’una progressió geomètrica.
No obstant, per límits, podem trobar que:
 
� .
26. Calcula la fracció generatriu del decimal periòdic ��, ���
��� ���
� �
�, ��� � ��
���� �������
���
������� �
�� ��� �
����
����

���
���� ���
�� � �
�� ���
����
��� ����� � ��� ����� ����
� � �� � �, ���
��, ��� � � �� � � � �
��� ��� ��� ���

63

45 
Successions numèriques. 2
Progressions
 
 
 
 
Sucessions numèriques. Progressions
 
 

 
27. Esbrina quina és la progressió geomètrica la suma dels infinits termes de la qual és 5 i el
primer terme és a1 = 2.
� � � �
�� ����� ����� �
��� � � �
� ���
�� � � � � �

 
� �
28. Interpola quatre mitjans diferencials entre els valors i � .
� �

� �
��� ��� ��
�� � �  
��� � ���
�� � �; �� � � � �; �� � � � ��; �� � � � ��; �� � � � ��; 
� � �� ��
�� � � ; �� � �
� �� ; 
� � ��� ���
� �� �� � �� � � ��
�� � � � � � �� ; �� � � � � � �� ; �� � � � � �
� ��� ��� � ��� ��� � ���
��
�  
���
 
29. Interpola sis mitjans proporcionals entre aquests dos valors √� i 81.

��� � � 81 �
�� � � � � �27√3 � √3 
� √3
�� � √3 � � � √3 � √3 � 3; 
�� � 3 � √3 � ��1��;     
�� � ��1�� � √3 � �;   
�� � � � √3 � 1���8�; 
�� � 1���8� � √3 � 27;   

�� � 27 � √3 � ���7�� 
 
 
30. Per una quantitat determinada de diners, invertida en un dipòsit financer a un interès del 3,5 %
anual durant 3 anys, hem rebut 735 € d’interessos. Quina era la quantitat inicial?

Utilitzem la fórmula de l’interès simple i aïllem la variable que ens interessa, aquesta ocasió és el
capital inicial:

�� � � � � ��� � � ��� � ���


�� � �� � � � � ��� � 
��� ��� �� � � �
 

46 
64
 
 
 
 
 
 
 

Equacions i inequacions 23
 
 
 
 
 
 

31. Quin interès ofereix un compte bancari amb el qual, si inverteixes 5.000 € durant dos anys,
obtens uns beneficis de 400 €?

Utilitzem la fórmula de l’interès simple i aïllem la variable que ens interessa:

����� �������
�� �� � ����
�� �� ������

 
32. Si el lloguer d’un pis és de 850 € al mes i puja un 5 % anual, què pagarem d’aquí a 10 anys?
Apliquem la fórmula de l’interès compost:

� � � ��
�� � �� �� � � � ��� � �� � � � ������ ��������
��� ���

 
33. Una empresa rep un crèdit al 8 % anual amb la condició de retornar en un sol pagament la
quantitat prestada més els interessos. Quant temps trigarà a duplicar-se el deute?

Com que tenim un únic pagament final apliquem la fórmula de l’interès simple i veurem quan els
interessos més el capital són dues vegades el capital inicial.

C� � � � � 100 � � 100 � �� 100


�� �� � � � � � 1��������
100 �� � � �� � � 8
 
 
34. Sabent que el primer terme d’una progressió aritmètica és 5 i que el cinquè és 13, calcula’n el
terme 36è.

Apliquem la fórmula de les progressions aritmètiques i substituïm els valors coneguts per trobar la
relació entre els termes:

�� � � � �� � �� � � � �� � � � �
��� � � � ��� � �� � � � ��

35. En una progressió aritmètica de vuit termes el primer i l’últim sumen 21, i el tercer terme és 6.
Escriu la progressió.

Apliquem la fórmula coneguda tantes vegades com podem fins a trobar un sistema d’equacions que
disposi de tantes equacions com de variables, i així descobrirem el que s’ha demanat.

Primer exposem el conegut:

�� � �� � �1

�� � �
Després posarem totes les variables en funció de dues:
�� � �� � �8 � 1�� � �1 � �� � �� � ��
�� � � � �� � �� � 1��
I ara reescrivim el sistema d’equacions amb les dues úniques variables que defineixen la progressió:

65
 
 
 

Successions numèriques. 2
Sucessions numèriques. Progressions
Progressions
 
 
 
 
 
 
21 � 2��
21 � 2�� � 7� ��
� →� 7
� � �� � 2� 21 � 2�� 42 4 7 42
� � �� � 2 � � � � �� � � �� → �� � �� � � � �
7 7 7 3 7
21
→�� �3
7
Finalment, escrivim la progressió. Si es vol es pot reduir fins una expressió de forma habitual:

�� � �� � 1�3 � 3� � 3
 
 
36. Tinc 200 € i cada mes em donen 1,5 € més. Calcula quants diners tindré al cap d’un any i mig.

I = 1,5 · 18 = 27

Cf = 200 + 27 = 227
 
37. Digues per a quin valor de n es compleix que �� � �� � � � �� � �����
Generem una successió:
�� ;��� ;��� ; …
Com es pot comprovar la raó de la mateixa és:
r=3
Aprofitem que coneixem el valor de la suma dels n valors finits primers i apliquem la corresponent
fórmula:
�� � � �� � �
�� � � � � → ���� � � � � �� � � ��� � ��
��� ���
Aïllem i trobem n
�� � ����
n = log3 2187 = 7
 
38. El producte de 5 termes d’una progressió geomètrica és 282 475 249 i l’últim terme és 7
vegades més gran que el tercer. Calcula la progressió.

Plantegem les dades de l’enunciat:

�� � ���� � �� �� � ����������� 

�� � � � ��    
Aprofitem les propietats de les progressions geomètriques:     
� � � �� � �
�� � � � �  → �� � �� � �� → � � �� → � � √� 
� �

Tornem a aplicar la definició de progressió geomètrica i aïllem la incògnita necessària de l’expressió 
del producte: 
�� � �� � �� � �� � ��  

66

48 
 
 
 
 
 
 

Equacions i inequacions 23
 
 
 
 
 
 

� � �����������
����������� � ���� � �� � �� �� � ��� � �� � → � � �� � � 
������
���
�� � � � �√��  
 
39. Determina la fracció generatriu de cada decimal periòdic següent com la suma d’una
progressió geomètrica: a) 2,4545454545...

� � � � �� ��
Tenim el nombre �� �� �

� = 0,45 + 0,0045 + 0,000045+...


El segon terme és la progressió geomètrica il·limitada: 0,��

�� �
Primer terme és �� �� � i la raó és
��� ���

�� ��
��� ��� ��
�� � � � �� �
�� ��
��� ���
�� ���
� ���
�� �� �
�� ��
 
40. Un banc té dos tipus de dipòsit:
A: tipus d’interès del 4,75 % anual durant 5 anys.
B: durada de 5 anys, però el tipus d’interès, 6 % anual, només es cobra durant els 3 primers
anys. Els dos darrers anys no es cobren interessos i regalen un televisor valorat en 580 €.

Si la inversió mínima és de 5.000 €, quin dipòsit és més avantatjós si s’hi inverteix aquesta
quantitat?
����� ������������
A) � � � � ���������
��� ���

����� ���������
B) � � � � ��� � �������
��� ���
 
 
41. Faig un ingrés al banc de 20 000 € i es comprometen a pagar-me el 3 % anual abonant els
interessos semestralment. Quants diners tindré d’aquí a cinc anys?


 �� � ������ � �� � ���� � ������ � ������� � ������ � ���������� � ����������� 
�����

 
42. Calcula el temps necessari perquè un capital de 50.000 € al 3 % anual es converteixi en 60 000
€.
NOTA: Tema d’interès compost. Considerar que els interessos es reinverteixen anualment.
Ci = 50 000; Cf = 60 000; r = 3 %
� ������
������ � ������ � �� � �� → � �� ��� � �� �
��� ������
�� ���
T · ln 1,03 = ln 1,2 → � � � 6,16809691 anys
�� ����
67
Successions numèriques. 2
Progressions
 
 
 
 
Sucessions numèriques. Progressions
 
 

 
43. Dipositem 5 000 € en un banc al 4 % d’interès compost anual. Un banc té dos tipus de dipòsit:
a) Cada any. b) Cada semestre. c) Cada trimestre.
Quin capital rebrem després de 3 anys en els supòsits anteriors.
� �
a) �� � � ��� � �� � � � � ��� � ��� ���� � � ��� � �� ������ � � ���� ��
���

� ���
b) �� � � ��� � �� � � � � ��� � ��� ���� � � ��� � �� ��������� � � ���� ����
�����

� ���
c) �� � � ��� � �� � � � � ��� � ��� ����� � � ��� � �� �������� � � ���� ����
�����

50 
68
 
 
 
 
 
 
 

Equacions i inequacions 23
 
 
 
 
 
 

ACTIVITATS

44. Continua els termes d’aquestes successions:


a) 5, 7, 9, 11, 13, …
b) 20, 30, 50, 80, 130, …
c) 1, 4, 16, 64, …
d) 1, 8, 27, 64, …
Quin criteri de formació segueix cadascuna d’elles?
a) �� � �� �� � ���� � �
b) �� � ������ � ��� �� � ���� � ����
c) �� � �� �� � ���� � �
d) �� � ��

45. Donada la successió:


� � �� ��
, , , ,…
� � �� ��
a) Quin n'és el 6è terme?
b) I el criteri de formació?
�� ��� ��
a) �� � � �� �
�� �� �� ��

b) Numerador: 1, 8, 27, 64,... → �� � � ��


Denominador: 4, 7, 12, 19,... → �� � � �� � �

46. El terme general de la successió 1, 4, 9, 16, 25, … és an = n2.Troba el terme general de les
successions:

a) 0, 3, 8, 15, 24, …
b) 3, 6, 11, 18, 27, …
c) 4, 9, 16, 25, …
d) 16, 25, 36, 49, …
a) �� � �� � �
b) �� � �� � �
c) �� � �� � ���
d) �� � �� � ���

47. La successió 3, 5, 7, 9, 11, … té per terme general an = 2n + 1. Determina el terme general de les
successions:

a) –1, 1, 3, 5, 7, …
b) 6, 9, 12, 15, …
c) –2, –5, –8 , –11, …

69
 
 
 

Successions numèriques. 2
Sucessions numèriques. Progressions
Progressions
 
 
 
 
 
 

d) 5, 10, 15, 20, 25, …


a) �� � �� � �
b) �� � ��� � ��
c) �� � � � ��
d) �� � ��

48. Troba els cinc primers termes de la successió el terme general de la qual és:

a) an = 5n-1
b) an = 5n-1
c) an = (-5) · 2n
d) an = n2 + 5n – 2
���
e) �� �
��

a) 1, 5, 25, 125, 625.


b) 4, 9, 14, 19, 24.
c) -10, -20, -40, -80, -160.
d) 4, 12, 22, 34, 48.
� � � �
e) 4, , , , .
� � �� ��

49. Troba la regla de formació d’aquestes successions recurrents:

a) 1, 3, 4, 7, 11, 18, 29, …


b) 1, -2, 3, -4, 5, -6, …
c) 1, 2, 3, 6, 18, 108, …
d) -5, 1, 6, 5, -1, -6, …
a) �� � �; �� � �; ��� � � � � �� � ���� � ����
b) ��� � � � � �� � ��������� � �
c) �� � �; �� � �; �� � �; ��� � � � � �� � ���� � ����
d) �� � ��; �� � �; ��� � � � � �� � ����� � ���� �

50. Troba la diferència i el terme general de les progressions aritmètiques següents:

a) 10, 5, 0, −5, …
b) √�, 2√�, 3√�, 4√�,…
c) 7, 11, 15, 19, …
d) 16, 2, −12, …
a) � � ��; �� � �� � �� � ������
b) � � √�; �� � � � √�;
c) �� � � � �� � ���; � � � � �� � �� � �

70

52 
 
 
 
 
 
 

Equacions i inequacions 23
 
 
 
 
 
 

d) �� � �� � �� � �������� � � ��� � �� � �� � ���

51. Troba el terme general de les progressions aritmètiques següents:

a) 1,33, 1,31, 1,29, 1,27, …


b) 5, 2, -1, -4, -7, …
� �
c) , 1, , 2, …
� �
� � � �
d) , , , , …
� � � �
� � �
a) �� � �� �� � �� � �� �� � � � � � � �� � �� �� � �
�� �� ��

b) �� � � � �� � ������� � � �� � �� � � � ��
� � � �
c) �� � � �� � �� � � � � �� � �
� � � �
� � � � �
d) �� � � �� � �� � � � � �� � � �
� � � � �

� �
52. En una progressió aritmètica, �� � i �� �
� �

Troba a1, d i determina el terme general.


Sabent la posició i valor de dos termes consecutius d’una progressió aritmètica podem fer el següent
per obtenir d:
� � �
� � �� � � � � � �
� � �
Per obtenir el primer terme de la progressió fem:
� � �
�� � �� � �� � ��� �
� � �
Així tenim i podem agrupar al terme general:
� � � �
�� � � �� � �� � �� � � �
� � � �

53. En una progressió aritmètica, a8 = 10 i a15 = 34.


Calcula’n la diferència i el terme general.
Repetim el procés considerant que es tracta de termes no contigus amb la fórmula general pel cas
particular:
���� � �� � ��
�� �
� �
A partir d’aquest moment repetim procediment de l’exercici anterior.
��
�� � �� � �� � �� � � � � ���

�� �� ���
�� � ��� � �� � �� � �� � ��
� � �
Agrupem termes dins el terme general.

71
 
 
 

Successions numèriques. 2
Sucessions numèriques. Progressions
Progressions
 
 
 
 
 
 

54. En una progressió aritmètica, a5 = 10 i d = 5.


Esbrina el lloc que ocupa el terme que val 180.
Amb les dades procedim a trobar el primer terme de la progressió.
�� � �� � �� � �� � � � � � ���
Determinem el terme general i el presentem de forma normal.
�� � ��� � �� � ��� → �� � �� � ��
Apliquem el cas demanat a l’enunciat i aïllem la variable sol·licitada:
��� � ��� � �� � �����→ ��� � �� � �� → � � ��

55. Troba el terme general d’una progressió aritmètica en què a3 = 7 i a2 + a9 = 29.


Plantegem les relacions entre els termes dels quals tenim informació:
�� � �� � ��
Aïllem les variables desitjades:
�� � �� � �� � ��
� →� � � �� � ��� � �� � � �� → �� � �
�� � �� � � � � � � � � �� → �
→�������
Així trobem el primer terme de la progressió:
�� � �� � � � � � � � �
I com sempre escrivim el terme general i el presentem en forma normal:
�� � � � �� � ���� → �� � �� � �

56. Sigui �� � �� � � el terme general d’una progressió aritmètica. Calcula a20 i la suma dels vint
primers termes.

Apliquem pel cas demanat l’expressió general:

�� � �� � � → � ��� � � � �� � � � ��
Utilitzant la fórmula de la suma dels primers termes:
� � ��
��� � � �� � ���

57. En una progressió aritmètica, a8 = 100 i la diferència és 7. Troba’n el primer terme i la suma
dels vuit primers termes.

Seguint procediment similar a l’exercici anterior:

�� � �� � �� � ��� � � � � � ��
�� � ���
�� � � � � ���

72

54 
 
 
 
 
 
 

Equacions i inequacions 23
 
 
 
 
 
 

58. Calcula la suma dels 10 primers termes d’una progressió aritmètica en què el tercer terme és
24 i el desè és 66.

Amb les dades del problema trobem la diferència:

૟૟ െ ૛૝
ࢇ૜ ൌ ࢇ૚૙ െ ૠࢊ ՜ ࢊ ൌ ൌ ૟

Amb la d trobem el primer terme:

ࢇ૚ ൌ ࢇ૜ െ ૛ࢊ ൌ ૛૝ െ ૛ ൉ ૟ ൌ ૚૛
I ara ja podem calcular la suma dels primers 10 termes:
૚૛ ൅ ૟૟
ࡿ૚૙ ൌ ൉ ૚૙ ൌ ૜ૢ૙

59. Troba la suma dels 100 primers nombres parells.


Sabem que el primer nombre parell és ࢇ૚ ൌ ૛, i sabem que el nombre parell 100è és ࢇ૚૙૙ ൌ ૛૙૙, és a
dir, que n = 100. Així, apliquem la coneguda fórmula i tenim:

૛ ൅ ૛૙૙
ࡿ૚૙૙ ൌ ൉ ૚૙૙ ൌ ૚૙૚૙૙

60. Calcula la suma dels múltiples de 7 més grans que 200 i més petits que 400.
Tenim que la suma de la progressió aritmètica dels primer n termes és:
ࢇ૚ ൅ ࢇ‫ܖ‬
ࡿ࢔ ൌ ൉ ‫ܖ‬Ǣ

Per tant, podem fer que entre dos termes la suma dels valors sigui ࡿ‫ ܂‬ൌ ࡿழ۰ି ࡿழ‫ۯ‬
Al nostre cas haurem de trobar els valors múltiples de 7 que són més petits que 400 i 200.
Es pot mirar la successió i es troba que:
ࢇ૚ ൌ ૠ
ࢇ૛ૡ ൌ ૚ૢ૟
ࢇ૞ૠ ൌ ૜ૢૢ
Així substituïm i trobem el valor demanat:
ૠ ൅ ૜ૢૢ ૠ ൅ ૚ૢ૟
ࡿ‫ ܂‬ൌ ࡿழ૝૙૙ି ࡿழ૛૙૙ ൌ ൉ ૞ૠ െ ൉ ૛ૡ ൌ ૡૠ૛ૢ
૛ ૛

61. Troba la suma dels 15 primers termes d’una progressió aritmètica en què a1 = 7 i a4 = 40.
Com sempre aplicant les fórmules generals sobre les dades aportades a l’enunciat del problema
tenim que:

૝૙ െ ૠ
ࢇ૝ ൌ ࢇ૚ ൅ ૜ࢊ ՜ ࢊ ൌ ൌ ૚૚

Trobem el valor del terme 15 i apliquem la fórmula de la suma dels primers termes:
ࢇ૚૞ ൌ ࢇ૚ ൅ ૚૝ࢊ ൌ ૠ ൅ ૚૝ ൉ ૚૚ ൌ ૚૟૚
ૠ ൅ ૚૟૚
ࡿ૚૙૙ ൌ ൉ ૚૞ ൌ ૚૛૟૙

73
Successions numèriques. 2
Progressions
 
 
  Sucessions numèriques. Progressions
 
 
 


62. Quants nombres imparells consecutius a partir d’1 sumen 2 916?
Apliquem la teoria de progressions aritmètiques al conjunt dels nombre imparells i tenim que:
�� � �
���
Amb el terme general següent:
�� � �� � �� � ��� → �� � � � �� � ��� � �� � �
Sabem que la suma és 2916, i per tant, apliquem la fórmula corresponent i aïllem la variable
desconeguda:
�� � �� � � �� � �
�� � � � → ���� � � � → ���� � ��
� �
� � √2916 � ��

63. La suma dels cinc primers termes d’una progressió aritmètica és 2,5. La suma dels vuit
primers termes és 5,2. Escriu la progressió.

Primer de tot, procedim a escriure com un sistema d’equacions emprant les fórmules generals i de la
suma dels primers termes aplicats a les dades de l’enunciat:

�� � ��
��� � � � � �� � �� � �� � �� � �
� �
� �� � �� �� � � � �� � ��� �
�� � � � � �� � → � �
� → �� � �� �
� � � � � � � ��
� �� � �� � �� � � �
� �� � �� � ��
� �� � �� � ��
Hem aplicat el mètode de substitució per resoldre el sistema, aïllant les variables que ens interessen i
així, disposem de les dades requerides.
�� � �� �
�� � �
�� � � �� �
�� � �� �

Com fem sempre escrivim l’expressió general i la transformem a la forma normal:
�� � �� � � �� � �� � �� � � �� � � � � �� �

En fraccions: �� � � �� � ��
��

64. Interpola:
a) Set mitjans diferencials entre 4 i 44.

b) Cinc mitjans diferencials entre i 1.

����
a) � � ��
���
�� � � � � � �� �� � ��� �� � ��� �� � ��� �� � ��� �� � ��� �� � ��

56 
74
 
 
 
 
 
 
 

Equacions i inequacions 23
 
 
 
 
 
 

�� �
b) � � �

��� ��
� � � � � �� ��
�� � � � ���� � ����� � ������ � � ����� �
� �� �� �� �� �� ��

� � �
65. Donada la successió 5, 5 √�, 5 √�� , 5 √�� ,… , quin lloc hi ocupa el nombre 625?
Primer trobem la raó dividint un terme pel seu predecessor:

� � √�
Tenim la raó i el primer terme, ara ja podem plantejar el cas particular amb la posició com a única
incògnita, la qual aïllarem i trobarem:
� ��� ��� � � � � �
�� � �� � ���� → ��� � � � � √�� → √��� � √� → ���� � �� → � →
��� �
→ �� � � � � → � � ��

66. Quants termes de la progressió 4, 8, 12, … hem de sumar perquè la suma sigui 840?
Trobem el terme general:
�� � � � �
Apliquem les fórmules conegudes aplicades al cas indicat a l’enunciat i aïllem la variable demanada:
� � �� ����� ��� � ��
�� � ��� � ��� ��� � � � ���� � �� � ��� → �� � � � ��� � �
� � �
�� � √� � ���� �� � �� ��
�� � ��
� � ���
Escollim com resposta correcte n = 20, ja que la iteració negativa no té cap sentit.

67. Troba tres nombres naturals en progressió aritmètica si la seva suma és 24 i el seu producte
és 440.
Plantegem el problema definint amb les fórmules conegudes:
La suma dels tres termes és 24:
��
� �� � �� � � → �� � �� � � � �� � �� � �� → � � � � ��
�� � �� � ��
El producte dels tres termes és 440 i amb l’equació del sistema anterior tenim:
� � � � ��
� → �� � ��� � � � �� � � ��� � ��� � �� �� � ��� →
�� � ��� � �� � ��� � ��� � ���
→ �� � � � ��� � �� � ��� � � ��� → ��� � ��� � �� � � ���
→ ����� � ��� � � ��� → ��� � � ����� � ��� � � → �� � � ���� � �� � �
Tenim una equació quadràtica que resolem i ens dona:

�� � ������� � � � � � �� ��
�� � ��
��� �
��
� � � � �� � �

�� � ��
� �� � �� � � � �� � � � �
�� � �� � �� � �� � � � �
75
 
 
 

Successions numèriques. 2
Sucessions numèriques. Progressions
Progressions
 
 
 
 
 
 
ࢇ૚ ൌ ૞
൝ ࢇ૛ ൌ ࢇ૚ ൅ ࢊ ൌ ૞ ൅ ૜ ൌ ૡ
ࢇ૜ ൌ ࢇ૚ ൅ ૛ࢊ ൌ ૞ ൅ ૟ ൌ ૚૚

Es pot comprovar que les dues solucions que ens dona l’equació de grau dos són en realitat la
mateixa solució.

68. Volem construir un dipòsit d’aigua amb forma prismàtica de 80 m3 de volum i de manera que
les seves dimensions estiguin en progressió aritmètica. Calcula les dimensions que ha de tenir
el dipòsit.

Utilitzem l’expressió coneguda pel volum d’un prisma com a producte de les seves arestes:
ࢂ ൌ ࢇ૚ ൉ ࢇ૛ ൉ ࢇ૜ ൌ ૡ૙
Atès que ens diuen que aquestes formen una progressió aritmètica aplicarem les següents
expressions:
ࢇ૛ ൌ ࢇ૚ ൅ ࢊ
ࢇ૜ ൌ ࢇ૚ ൅ ૛ࢊ
ࢇ૚ ൉ ሺࢇ૚ ൅ ࢊሻ ൉ ሺࢇ૚ ൅ ૛ࢊሻ ൌ ૡ૙ ՜ ࢇ૚ ൉ ሺࢇ૚ ૛ ൅ ࢊࢇ૚ ൅ ૛ࢊࢇ૚ ൅ ૛ࢊ૛ ሻ ൌ ૡ૙
՜ ࢇ૚ ૜ ൅ ૜ࢊࢇ૚ ૛ ൅ ૛ࢊ૛ ࢇ૚ െ ૡ૙ ൌ ૙
Tenim una expressió polinòmica de grau 3 que podem resoldre emprant el mètode de Ruffini:

1 3d 2d2 -80
2 2 6d+4 4d2+12d+8
1 3d+2 2d2+6d+4 4d2+12d-72

Per trobar les solucions possibles s’ha de complir que 4d2 + 12 d - 72 = 0, i com que es tracta d’una
equació quadràtica la resolem directament amb la seva fórmula:

െ૝ േ ඥሺ૝ሻ૛ െ ૝ ൉ ૝ ൉ ሺെૠ૛ሻ ૜
ࢊൌ ൌቄ
૛൉૟ െ૟

De l’expressió trobada anteriorment, substituïm i tenim que:

ࢇ૚ ૜ ൅ ૜ࢊࢇ૚ ૛ ൅ ૛ࢊ૛ ࢇ૚ െ ૡ૙ ൌ ૙ ՜ ࢇ૚ ૜ ൅ ૢࢇ૚ ૛ ൅ ૚ૡࢇ૚ െ ૡ૙ ൌ ૙

1 9 18 -80
2 2 22 80
1 11 40 0

Tenim que ࢇ૚ ൌ ૛ m

Descartem ara la solució de d = - 6 per manca de sentit i emprant d = 3 es troba:


ࢇ૛ ൌ ࢇ૚ ൅ ࢊ ൌ ૛ ൅ ૜ ൌ ૞࢓
ࢇ૜ ൌ ࢇ૚ ൅ ૛ࢊ ൌ ૛ ൅ ૛ ൉ ૜ ൌ ૡ࢓

76

58 
 
 
 
 
 
 

Equacions i inequacions 23
 
 
 
 
 
 

Ara podreu comprovar que és compleixen totes les condicions de l’enunciat.

69. El cometa Encke va ser descobert l’any 1821 i té un període de 3,3 anys. Quantes vegades s’ha
vist fins a l’any 2000?

d = 3,3
an = 2000
a1 = 1821
�� � �� � � ���� � ���� � �� �
�� � �� � �� � �� � �� � � � � �
� ��� ��
� �� �
S’ha vist 55 vegades

70. El cometa Halley visita la Terra cada 76 anys. El 1986 el vam veure per quarta vegada des que
l’astrònom Edmund Halley el va descobrir. Quin any va descobrir-lo?

d = 76
a4 = 1986
a1 = ?
�� � �� � �� � �� � �� � �� � �� � �� � �� � � � ���� � �� � �� � �� � ����

71. Un director d’un centre comprova que en col·locar els 435 alumnes en files de manera que a la
primera fila n’hi hagi un, a la segona, dos, a la tercera, tres, i així successivament, hi caben
tots. Quantes files ha fet?

Definim la progressió aritmètica:

�� � �� � �� � �� � � � � � �� � �� � �
Aplicant la fórmula de la suma dels n primers termes:
�� � �� ���
�� � � � � ��� � � � � �� � � � ��� � �
� �
Trobem una equació quadràtica que resoldrem amb la seva fórmula directa:
�� � ��
�� � √�� � ��� �� � ��� � � � � � ������ �� � √���� �� � �� � ��
�� � � � �� �
�� ��� � � �� � ��
� ���

Solament és acceptable un resultat positiu, atès que no és possible tenir alumnes en negatiu, així
tenim, 29 files

72. En Miquel té 381 monedes i vol construir torres de monedes de manera que la primera torre en
tingui 3 i cada una de les següents en tingui el doble que l’anterior. Determina el nombre de
torres que pot construir.

Tenim la progressió geomètrica: 3, 6, 12, 24,...

Amb raó: r=2

77
 
 

Successions numèriques. 2
Progressions
Sucessions numèriques. Progressions
 
 
 
 
 
 

I expressió o terme general: �� � �� � � ����� � � � ������


Aplicarem la fórmula de la suma dels primers termes d’una progressió geomètrica:
�� � � �� � �
�� � � � � →�� � ��� → ��� � �� � � → ��� ��� � � � ��� � → � � �
��� �
Aïllada la variable cercada trobem el resultat.

73. La Carme ha comprat a un amic seu un cotxe de segona mà per 8 500 €, i decideix pagar-lo en
10 vegades de manera que cada vegada pagarà 100 € més que l’anterior. Quant pagarà en el
primer termini? I en l’últim?

Plantegem les dades de l’enunciat en forma d’equacions:


�� � �� � �� � �� � ��� � �� � ���
I amb la suma total a pagar sabem que:
�� � �� �� � �� � ��� � ��� � ���
�� � � � → � ��� � �� � → �� � � ���
� � �
Tornem a aplicar la primera equació generada i trobem:
��� � � ���

74. Un club d’esports es va fundar amb 33 socis i ara, que compleix el 20è aniversari, té 253 socis.
Si cada any ha tingut la mateixa quantitat d’altes i mai no s’ha donat ningú de baixa, quants
socis tenia quan va celebrar el 10è aniversari? Quantes altes ha tingut cada any?

Tenim que en 20 anys s’han incorporat al club 253 - 33 socis, per tant, l’increment anual que ha tingut
en aquest període és:

��� � �� ���
�� � � ��
�� ��
Així el 10è aniversari, tindran:
�������� � ��� socis

75. Troba els costats d’un triangle rectangle si saps que estan en progressió aritmètica de
diferència 6.

�� � �� � �
�� � �� � ��
Apliquem Pitàgores:
��� � ���� � �� � � ��� � ��� →

→ �� � � ���� � ��� � �� � � �� � � ���� � �� → �� � � ���� � ��� � � →

78

60 
 
 
 
 
 
 

Equacions i inequacions 23
 
 
 
 
 
 

A l’equació quadràtica trobada, aplicarem la fórmula de solució directa i trobem


���� � ������ � � � � � ������
�� � �
���
� � ��
�� �
�� � ��

Descartem la solució negativa i tenim que:


�� � ���
Apliquem les equacions inicials i tenim que:
�� � �� � � � ���
�� � �� � �� � ��

76. La hipotenusa d’un triangle rectangle mesura 20 cm. Calcula’n el perímetre si saps que les
mides dels tres costats estan en progressió aritmètica.

�� � �� � � � �� � �
�� � �� � �� � �� � ��
Apliquem Pitàgores:
��� � ��� � ��� � ��� � ���� � ��� � ��� � ��� � �� � ��� � ��� ���� � ��� � ���� � ��� � �
� �� � ��� � �� � �
A l’equació quadràtica trobada, aplicarem la fórmula de solució directa i trobem


������ � ������ � � � � � ����
�� �
���
� � ��
��
���
Descartem la solució d = 20 atès que en un triangle no pot haver-hi dos costats amb la mateixa
longitud.
Apliquem les equacions inicials i tenim que:
�� � �� � � � ��
�� � �� � � � ��
El perímetre serà = 20 + 16 + 12 = 48 cm

77. L’Enric decideix estalviar i posa a la guardiola 1 € la primera setmana, 1 € i 20 cèntims la


segona, 1 € i 40 cèntims la tercera, i així successivament. Quant haurà estalviat al cap d’un
any?

Sabem que un any té 52 setmanes, i que d = 0,2 €,


Tenim �� � �� � �� � �� � � � � � ��� � �� � �� � � ��� � �
Per tant,
�� � �� � � ��� �
�� � � � � �� � � ���� � �
� �

79
Successions numèriques. 2
Progressions
 
 
  Sucessions numèriques. Progressions
 
 
 

78. En una granja hi ha 300 pollastres i cada dia en neixen 12. Quants n’hi ha al cap de 10 dies si
no se n’ha mort cap?

Descrivim matemàticament l’enunciat:


�� � ���� � � �� � � � ��
Apliquem la teoria coneguda de les progressions aritmètiques i cerquem el desè element:
�� � �� � �� � �� � � → ��� � ��� � ��� � �� � �� � ���

79. En una progressió geomètrica a1 = 0,2 i a2 = 0,3. Troba’n el terme general i a25.
Simplement apliquem les definicions de progressió geomètrica i raó a les dades de
l’enunciat:
�� � �� � � ��� →
�� � �
�� �
�� � �
Un cop tenim la raó escrivim el terme general i cerquem el terme 25è.

� ��� � ��
�� � �� � � � � → ��� � �� � � � � � ����� ��
� �

80. En una progressió geomètrica a1 = 6 i a3 = 18. Troba’n a8 i el terme general.


Procedim amb el mateix procediment de l’exercici anterior:

�� ��
�� � �� � �� → � � � � � � √�
�� �
��� �
�� � � � √� → �� � � � �√�� � ���√� � ���� ��

81. El tercer terme d’una progressió geomètrica és 6, i el cinquè és 54. Determina’n el terme
general.

��
�� � �� � �� → � � � ��

�� � �
�� � � �
�� � �
� ���
�� � �� � � ��� → �� � ��

82. Dos termes consecutius d’una progressió geomètrica són 6 i 8. Esbrina el lloc que ocupen si
��
�� � .
��

62 
80
 
 
 
 
 
 
 

Equacions i inequacions 23
 
 
 
 
 
 
� �
�� �
� �

�� � ��� �� � ��� ��� � ���


�� � �� � →�� �� � → �� � →
�� � �� � �� �

��� �����
� � �� � ������� � → � �
→ ��� �� ��� ����

������

La resposta és quart i cinquè respectivament.

83. Un dels termes d’una progressió geomètrica és 885 735. Si el primer terme és 5 i la raó és 3, de
quin terme es tracta?

�� � �� � ���� → ������ � � � ���� → ������ � ���� → ��� ������ � �� � �� � ����


��� ������
→�� � � � ��
����


84. El terme sisè d’una progressió geomètrica és 12, i el quart és .

a) Troba’n el terme general.
b) Troba el producte dels deu primers termes de la progressió.
a) Comencem trobant la raó amb els dos termes coneguts:

�� ��
�� � �� � �� → � � � � � √�� � �√�
�� � �

Per trobar el primer terme tenim alguns camins que podem escollir:
���
�� � �� � � ��� → �� � �� � �√��� →
� � � � �
� �� � �√��� → �� � � �
� � ��√� ���√�
� � �
�� � �� � �√��� → �� � �� � � � �� ���������
����√� ���√�

� ���
�� � � ��√��
���√�

b) Per cercar el producte dels primers termes d’una progressió geomètrica necessitem
addicionalment el darrer terme, així:
� �
��� � � ��√�� � ����
���√�

I apliquem la corresponent fórmula:


�� � ���� � �� �� → ��� � ���� ��������� � ������� � ��������� ��

85. El terme vuitè d’una progressió geomètrica és 1 458 i la raó és 3.


a) Troba’n el terme general.
b) Troba el producte dels vuit primers termes de la progressió.

81
 
 
 

Successions numèriques. 2
Sucessions numèriques. Progressions
Progressions
 
 
 
 
 
 

Amb el mateix mètode duem a tere la recerca del primer terme, i amb ell i les dades de l’enunciat
podrem determinar tot el que s’ha demanat:
���� �
���� � �� � �� � �� � �
���� �

a) �� � � ����

� �
b) �� � ���� � �� �� � �� � �� � �4��� � ����6�� � �0��

86. El terme cinquè d’una progressió geomètrica és 160 i el segon és 20.


a) Troba’n el setè terme.
b) Calcula el producte dels set primers termes de la progressió.
Seguim les fórmules conegudes:

� �� �
�� � �� � �� � � � � � √� � �
��

�� ��
�� � �� � � � �� � � � ��
� �
a) �� � �0 � ���� � �� � �0 � �� � 640 

b) �� � ���� � �� �� � �� � ���0 � �0 � �� �� � ��6 400�� � ��0���� � �0�

87. El primer terme d’una progressió geomètrica és 7 i la raó és 3. Calcula la suma dels vuit
primers termes de la progressió.

Simple aplicació de les fórmules conegudes de progressió geomètrica:

�� � �� � ���� � �� � � � ����
�� � � � �� � �� ���
�� � � �� � �
�� � � � � � �� � � � � �� ���
��� ���

88. La suma dels set primers termes d’una progressió geomètrica és 7 651 i la raó és 3. Troba el
primer terme de la progressió.

Definim les dades indicades:

�� � � ���

���

���

Substituïm i aïllem de la fórmula la variable demanada:

82

64 
 
 
 
 
 
 

Equacions i inequacions 23
 
 
 
 
 
 

�� � � �� � � � ���
�� � � � � → � ��� � �� � → �� � � �
��� ��� � ���


89. El segon terme d’una progressió geomètrica és 2 i el quart és . Calcula la suma dels sis

primers termes de la progressió.

Cerquem la raó i el primer terme:




�� � �� � � → � � � � �

� �
�� �
�� � � � � � → �� � � � ��

Procedim amb la fórmula de la suma dels primers termes


� � � ��
�� � � � � �� �� � ��
�� � �� � → �� � � � �� � � � �� � � � � ��
� � �
��� �� � �
� � �


��
90. Troba la suma dels infinits termes de la successió 16, 12, 9, ,…

Primer de tot hem de trobar el tipus de successió que tenim, i es pot comprovar que es tracta d’una
progressió geomètrica; per tant podrem aplicar les fórmules conegudes de les mateixes:
�� � �
�� � � � � � → � � � �
�� �� �
Hem vist que la raó es manté igual entre tots els termes de l’enunciat. I atès que el valor absolut de la
raó és menor que 1, es pot aplicar la fórmula següent:
�� �� ��
�� � � � � ��
��� ���
� �

�� �
91. La suma dels infinits termes d’una progressió geomètrica és i la raó és . Troba els quatre
� �
primers termes de la successió.
Veient la fórmula de la suma de infinits termes i aïllant, trobarem el primer terme de la progressió:
�� �� �� ��� ��
�� → � � → � → ���� � �� → �� � �
��� �� � � �

83
 
 
 

Successions numèriques. 2
Sucessions numèriques. Progressions
Progressions
 
 
 
 
 
 

Amb aquest i la raó trobarem els successius termes:


� �
�� � �� � � � � � �
� �
� � �
�� � �� � � � � �
� � ��
� � �
�� � �� � � � � �
�� � ���

92. Determina la fracció generatriu de cada decimal periòdic següent com la suma d’una
progressió geomètrica.

a) 0,711711711…
b) 1,324324324324…
c) 0,644444…
a) Considerem el nombre com una progressió de termes amb raó menor que 1 i utilitzant la fórmula
de la suma de termes infinits podrem trobar el quocient que resulta.

Aquest procés es repetirà per cada exercici, amb la consideració d’addicionar cada part
independent al final com un tot per cada nombre (part entera més part decimal definida més part
decimal periòdica):

��� ��� ���


� �
�� ��� � � ��
���� ������� ����������

��
����
���
�� �
����
��� ���
��� ��
� �
�� ��� ����
� � ����
��� � �
������ ��� ���
����
��� ��� ���
� ���
b) �� ��� � � � �
���� ������� ����������

��
����
���
�� �
����
��� ���
��� ��
� �
�� ��� ����
� � ����
��� � �
������ ��� ��
����
�� ��
� ���
�� ��� �
�� ��
� � �
� � �� � �
c) �� �� � � ��
��� ���� �����

��
��

�� �
���

84

66 
 
 
 
 
 
 

Equacions i inequacions 23
 
 
 
 
 
 
� �
�� �
��
�� �� ���
� � ���
� � �
���� ��� ��
��
� � ��� ��
��
�� �� � � �
�� �� ��� ��

93. Interpola:
a) Quatre mitjans geomètrics entre 2 i 64.
b) Cinc mitjans geomètrics entre 161 i 4 347.
Apliquem les fórmules conegudes, trobem la raó i continuem definint els mitjans entremitjos
demanats:
��� � � ��
a) � � � �� ��
� �

�� � � � � � � � � � �;
�� � � � �� � � � �� � �;
�� � � � �� � � � �� � ��;
�� � � � �� � � � �� � ��
��� � � ���� �
b) � � � �� � √�� � √�
� ���

�� � � � � � ��� � √� � ���√�;


�� � � � �� � ��� � √� � ���;


�� � � � �� � ��� � √� � ���√�;


�� � � � �� � ��� � √� � ����;


�� � � � �� � ��� � √� � ����√�

� com una suma d’infinites fraccions. Suma totes aquestes fraccions.


94. Expressa el nombre �� ���
Com s’interpreta aquesta suma?

Emprem el mateix procediment que a l’exercici 92.

� �� ��
� ���
�� ��� � � ��
�� ���� ������

��
���
�� ��
�� ��
��
�� ��� ����
� ����
� �
� �� ��� ���
��
��� ���

� �� ����� ����
� ���
�� ��� � � �
�� ��� ���� ���

85
 
 
 

Successions numèriques. 2
Sucessions numèriques. Progressions
Progressions
 
 
 
 
 
 

95. Calcula el valor de l’expressió:

� � �
� � ��
� � ��
� �
�� � ��
� ��
� � �
Numerador: � � ��
� � ��
� �� �
�� � � � � �
� �� �
� �
�� �
�� � � � � �

��� ��� � �
� �
� �
Denominador: � � � ��
� ��
�� �
�� � �� � � �
�� �
�� � � �
�� � � � � � �
��� �� �
� �
Així, ara podem dir que:
� � � �
� � �� �� �
� � �� �
� � � � � �
�� � � � �� �
� �� �

96. El producte de 5 termes d’una progressió geomètrica és 282 475 249 i l’últim terme és 7
vegades més gran que el tercer. Calcula la progressió.

Amb les dades aportades trobem la raó:

�� � �� � ���� � �� � �� � �� � � � �� � � � √�
�� � �� � �� � �� � ��
Amb la fórmula del producte dels primers termes cerquem el primer terme:

�� � ���� � �� �� � �� � ���� � �� �� � ���� � �� � ���� � ��� ��� ��� �


��
� �� �� � ��� ��� ��� � ���� ��� ����� � �� � √��� ��� ��� � �
Ara ja podem escriure el terme general de la progressió.
���
�� � �� � ���� � �� � � � √�

97. El temps necessari perquè una determinada substància radioactiva es desintegri i redueixi a la
meitat la quantitat inicial és de 2,5 minuts.

Si tenim 1 600 g d’aquesta substància, quanta en quedarà després de 25 minuts?

86

68 
 
 
 
 
 
 

Equacions i inequacions 23
 
 
 
 
 
 

El factor conegut d’increment/reducció és la raó:



�� ;

El temps de 2,5 min es considera com una iteració, i simplifiquem la dificultat:
� ���
�� � �� � � ��� → �� � ���� � � �

��
�� � ��
�, �
� �� ����
→ ��� � ���� � � � � � �, ���� �
� ����

98. Els tres termes de lloc imparell d’una progressió geomètrica de sis termes sumen 105 i els tres
termes que ocupen els llocs parells sumen 230. Calcula’n el primer terme i la raó.

Primer escrivim les relacions indicades a l’enunciat:

�� � �� � �� � ��� �� ��
� → �� � ��� � �� � (1)
�� � �� � �� � ��� ���

�� � �� � ��
� → �� � �� � �� � �� � �� � ��� � �� � �� � �� � �� � (2)
�� � �� � ��

�� � �� � ��
��� � �� � �� → �� � � � �� � �� � �� � �� � ��� � �� � �� � �� � �� � (3)
�� � �� � ��

Mirem d’agrupar les equacions de forma que se simplifiquin amb un polinomi de grau el més petit
possible i trobem:

��� �� � �� � �� � �� � ��� ��
→ � � →
��� �� � �� � �� � �� � ��� ��
→ �� � �� � �� � �� � � �� � �� � �� � �� � →
���� � ���� � ���� � ���� � ��� � �� � � → � � �, �����������
Si resolem aquest polinomi pels mètodes gràfics o iteratius, veurem que l’única solució que compleix
és la indicada. A continuació, emprem les mateixes relacions conegudes:

��� � �� � �� � �� � �� � → �� � �, ������
�� � 3,643205 � 2,19047619111 � 7,980354647 
�� � �, ��������� � �, ����������� � ��, �������� 
�� � 17,48077685 � 2,19047619111 � 38,29122549 
�� � 38,29122549 � 2,19047619111 � 83,87601777 
�� � 83,87601777 � 2,19047619111 � 183,7284199 

87
 
 
 

Successions numèriques. 2
Sucessions numèriques. Progressions
Progressions
 
 
 
 
 
 

99. Una llagosta és a la vora d’una bassa de forma circular de 7 metres de diàmetre. Fa un primer
salt de 3 metres i, en cada un dels següents, salta la meitat que en l’anterior. Arribarà a l’altre
extrem de la bassa?

� � � �
�, , , , ,…
� � � ��


��


���� � � �, � �

��
� � �� � � ��, ����

�� �
�� � � � � � � No arriba a l’altre extrem atès que 6 < D.
��� ��

100. Des d’una altura de 10 metres tirem una pilota que, després de tocar a terra, rebota de manera
que, en cada rebot, arriba a les tres quartes parts de l’altura anterior. Suposant que rebota
indefinidament, calcula la distancia total recorreguda per la pilota.


��

�� � ��

�� ��
�� � � � �� �
��� ���

101. Hi ha uns bacteris que es reprodueixen de manera que cada 40 minuts es duplica la població.
Si inicialment tenim 1 000 bacteris, quants n’hi haurà al cap de 24 hores?

���

�� � ����

�� � ��
�� � ��
��

�� � �� � � ��� � ��� � �� � � �� � ���� � ��� � �,����� � ����  

102. Volem plantar 1 125 arbres de manera que a la primera fila n’hi hagi 5, a la setena, 65 i que la
quantitat d’arbres de més que es planta d’una fila a la següent sigui sempre igual. Esbrina
quantes files podem plantar.

�� � ��

�� � �

88

70 
 
 
 
 
 
 

Equacions i inequacions 23
 
 
 
 
 
 

��
�� � �� � �� � �� � � � �� � �� � � � � ��

�� � �� � � �� � � � � �� � ���� ���
�� � � � � ���� � � � � ���� � � � � ���� � ��
� � � �
� ���� � ��� � � � √��� � ��

103. Un corredor de maratons ha de donar 500 voltes a la pista d’atletisme. Per entrenar, decideix
fer 10 voltes el primer dia i cada dia dels següents fer 4 voltes més. Determina quants dies
trigarà a fer les 500 voltes.

� � �

�� � ��

�� � �� � �� � �� � �

�� � �� �� � �� �� � �� � �� � ��� �� � ��
�� � � � � ��� � � � � ��� � � � � ��� � ��
� � � �
� ��� � �� � ��� � � � � � ��� ������

Trigarà 14 dies. Es pot comprovar a continuació:


�� � �� � �� � �
��� � � �� � ���

104. Troba la profunditat d’un pou si per excavar el primer metre s’han pagat 20 €, per a cada un
dels restants 5 € més que per l’anterior i el cost total ha estat de 1 350 €.

� � �
�� � ��
�� � �� � �� � �� � �
�� � �� �� � �� �� � �� � �� � ���
�� � � � � ���� � � � � ���� � � � � ����
� � �
�� � �� � ��
� � � � �� � � � ���� � � � �� � �� � ��� � � � � � ��
� � �

�� � ����� � � � ��� � � � � ��
�� ��
��� � � ���

Descartem el resultat negatiu per irreal i considerem que la fondària del pou és de 20 m.

105. La Teresa ha comprat un cavall i vol posar-li ferradures. Ha de posar 28 claus: el primer li costa
1 cèntim d’euro i cada un dels següents 1 cèntim més que l’anterior. Quant ha de pagar en total
per ferrar el cavall?

89
 
 

Successions numèriques. 2
Progressions
Sucessions numèriques. Progressions
 
 
 
 
 
 

� � �� ��
�� � �� ��
� � ��
�� � �� � �� � �� � � � ��� � �� �� � ��� � �� � �� �� � �� ��
�� ��� ���������������������
�� � � � � ��� � � �� � �� �� €
� �

106. Quant pagaria la Teresa si el preu del primer clau fos el mateix que en l’activitat anterior però
cada un dels següents valgués el doble que l’anterior?

�� � �� ��
� � ��
� � �
�� � � ��� � �
�� � � � � � ��� � �� �� � � �������� �� €
��� ���

107. En Felip puja dalt d’una muntanya i observa que la pressió atmosfèrica disminueix un 2 % cada
100 m d’ascensió. Quin percentatge de la pressió atmosfèrica a nivell del mar hi haurà a 2,3 km
d’alçada?

�� � � atm
��
�� � �� ��
���
����
�� � ��
���
�� � �� � ���� � ��� � �� � ��� � � � �� ���� � �� ���� ���

108. Les preguntes d’un examen estan ordenades per ordre de dificultat. La primera val 2 punts i
cada una de les restants val tres punts més que l’anterior. Si el total de les preguntes val 40
punts, quantes preguntes té l’examen?

Escrivim les dades i plantegem l’equació de la suma dels n primers termes d’una progressió
aritmètica:

90

72 
 
 
 
 
 
 

Equacions i inequacions 23
 
 
 
 
 
 

�� � � punts
� � �
�� ��� �����������
�� � � � → �� � � � → �� � ��� � �� � � → ��� � � � �� � �
� �

Resolem directament l’equació quadràtica trobada:

�� � ����� � � � � � ����� ���


�� ��
��� � � ��� �

Descartem el resultat negatiu per irreal i es determina que el total de preguntes és 5.

109. Una corda de 55 metres s’ha dividit en trossos de manera que les seves longituds estan en
progressió aritmètica. Si saps que la diferència entre el tros més llarg i el més curt és de 12
metres, quants se n’han fet?

�� � �� � �� → �� � �� � �� (0)
�� � �� � �� � � � �� � �� → ��� � �� � �� (1)
�� ��� �� ��� ���
�� � � � → �� � � � → ��� � ��� � � ��� → �� � ���� � �� (2)
� �

Arribats a aquest punt, tenim dues equacions i tres incògnites; per tant, hem de començar a acotar el
problema:

Conjunt de solucions pertanyent als nombres enters

De (2) es dedueix que només la solució n = 5 és correcta. n = 11 també compliria, però es correspon
a una d negativa i, per tant, no lògica i la descartem.

� � � → �� � �� ���

�� � �� � � ��
→ ���� � � → �� � �� �� � ��� �� � ��� �� � ��� → �� � ��� � � � ��
� �
Conjunt de solucions pertanyent als nombres reals

Per a aquest conjunt tenim que hi ha múltiples solucions, p. e.:

A cada cas emprarem la mateixa metodologia. Primer apliquem la fórmula de la suma reduint i aïllant
al temps el primer terme (2), i amb l’equació (0) cerquem el darrer terme �� , trobem d (1) i a partir
d’aquí la resta de termes.

També inclourem una petita comprovació de sumatori dels termes.

�� � �� �� ��
� � � → �� � � → �� � → � � ��
� � �
�� �� � � � ��
� � � → ��� �� � � ��� � → ��� �� � ��� � � → ��� � � � → �� � ��
� � �
�� ��

� �
� � � � ��

91
 
 
 

Successions numèriques. 2
Sucessions numèriques. Progressions
Progressions
 
 
 
 
 
 
��
�� � � ��� �

�� �� � � � ��
� � � → ��� �� � � �� �� → ��� �� � ��� �� � → ��� � � � → �� � ��
� � �
�� ��

� �
� � � � ��

�� �� �� ��
�� � � �� � � �� � � �� �
� � � �
� � � �� � � ��
� � � → ��� �� � � → ��� �� � �� → ��� � � � → �� � ��� � � � ��
� �
�� � �� �� � �� �� � ��� �� � ��� �� � ��
�� �� � �� �� �� � � � ��
� � � → ��� �� � → ��� �� � � → ��� � � → �� � ��
� � � � �
�� ��
� ���
� � �
��� � � � ��
� �
�� ��� ��� ��� ��� ���
�� � � �� � � �� � � �� � � �� � � �� �
� �� �� �� �� ��
I així podríem continuar.

110. Tenim un dipòsit amb 1 litre d’una substància A i canvien 1 dL d’aquesta substància per 1 dL
d’aigua. Quina quantitat exacta de la substància A hi quedarà després de 5 canvis? I després
de 15 canvis? I després de n canvis?

�� � � l
��
�� � �� ��
���
a) ���
b) ����
a) �� � �� � ���� → �� � �� � �� � � � �� ��� � �� ���� �
b) �� � �� � ���� → ��� � �� � ��� � � � �� ���� � �� ��������� �

92

74 
 
 
 

Successions numèriques. 2
Progressions 2
 
 
 
 
 
 
��
�� � � ��� �

�� �� � � � ��
� � � → ��� �� � � �� �� → ��� �� � ��� �� � → ��� � � � → �� � ��
� � �
�� ��

� �
� � � � ��

�� �� �� ��
�� � � �� � � �� � � �� �
� � � �
� � � �� � � ��
� � � → ��� �� � � → ��� �� � �� → ��� � � � → �� � ��� � � � ��
� �
�� � �� �� � �� �� � ��� �� � ��� �� � ��
�� �� � �� �� �� � � � ��
� � � → ��� �� � → ��� �� � � → ��� � � → �� � ��
� � � � �
�� ��
� ���
� �
� ��� � � � ��
� �
�� ��� ��� ��� ��� ���
�� � � �� � � �� � � �� � � �� � � �� �
� �� �� �� �� ��
I així podríem continuar.

110. Tenim un dipòsit amb 1 litre d’una substància A i canvien 1 dL d’aquesta substància per 1 dL
d’aigua. Quina quantitat exacta de la substància A hi quedarà després de 5 canvis? I després
de 15 canvis? I després de n canvis?

�� � � l
��
�� � �� ��
���
a) ���
b) ����
a) �� � �� � ���� → �� � �� � �� � � � �� ��� � �� ���� �
b) �� � �� � ���� → ��� � �� � ��� � � � �� ���� � �� ��������� �

93

74 
 
 
 
 
 
 

Sucessions
Equacionsnumèriques. Progressions
i inequacions 3
 
 
 
 
 
 

111. Una persona envia una carta a dos amics, i els demana que, al seu torn, cada un d’ells n’enviï
una còpia a uns altres dos amics, i així successivament. Després de 10 enviaments, quantes
còpies de les cartes s’han fet?

�� � � cartes
� � �
�� � �� � ���� � ��� � �� � �� � � � �� � ����
�� � � ��� � �
�� � � � � � ��� � � � � � � ���� � ���� ������ ���������
��� ���
Com que considerem les dues primeres com a originals, s’hauran fet un total de 2.044 còpies.

112. Una bóta conté 4 096 litres de vi. Si un dia se’n treu la meitat del contingut, l’endemà se’n torna
a treure la meitat del que quedava, i així successivament, quina quantitat de vi es treu l’onzè
dia?

�� � � ��� ������ �� ��


��

����
� ��
�� � �� � ���� � ��� � �� � ��� � � ��� � � � � � ������

113. Les dimensions d’un ortoedre estan en progressió geomètrica. Calcula-les si saps que sumen
28 m i que el volum és de 512 m3.

Plantegem les equacions amb les relacions indicades a l’enunciat i les agruparem/reduirem fins que
tinguem el mínim i mateix nombre de variables que d’equacions:

�� � �� � �� � ���
�� � �� � �� � �� �� � �� � � � �� � �� � ���
� � �
�� � �� � �� �� � �� � � � �� � �� � ��
�� � �� � �


��
��� � �� � ��� � � �
�� �� � � � � � ��→� �� � � � � �
�� �� � � � �� � � �� �� � �� � � � � � � ��
�� ��

��
�� � � � � �� � ��� � ��� � �� � ���� � ��� � ���� � �� � � �
��

Atès que tenim una equació quadràtica, la podem resoldre directament:

������ � ������� � � � �� � � ��
�� � ��
��� �
� �
�� � ��
��

94
 
 
 

Successions numèriques. 2
Progressions 2
 
 
 
 
 
 

I ara trobarem la resta de termes:


�� � ��
� �
�� � �� � �� � �� � � � � �


�� � �� � � � �� � � �

�� � �
�� � �� � �� � � � �� � ��
�� � � � � � � � � � � �
Com es pot observar les dues respostes són la mateixa i, per tant, totes dues compleixen:
�� � ����� � �� �� � ��

114. Quants diners produeixen 15 000 € al 6 % d’interès en un any? I si hem de retirar els diners tres
mesos abans del termini però ens abonen la part proporcional?

����� ����������
a) � � � ����9�����
��� ���

b) I9mesos���9����� ���9������������������
��

115. A quin rèdit anual es van invertir 1 250 € si al cap de l’any han produït 30 € d’interès?
De la mateixa fórmula d’interès simple aïllem r:
��� � � ��� � ��
�� � � �� ���
��� ����� � �

116. La Montserrat inverteix en lletres del Tresor una quantitat de 35 600 €. Les lletres produeixen
cada any un 3,2 % l’interès que li ingressen en un compte bancari. Quant li hauran ingressat al
cap de 8 anys?

Com que no es reinverteix, utilitzem la fórmula d’interès simple.

����� ������ � �� � � �
�� � � ������ ���
��� ���

117. Fa uns quants anys, l’Arnau va demanar un préstec de 15 000 € al seu cosí Marc, i es va
comprometre a tornar-lo en cinc anys i pagar-li al final de cada any un 2,8 % d’interessos de
tots els diners que li va deixar.

Completa la taula on el Marc ha anotat els pagaments que li ha fet l’Arnau.

Any 2010 2011 2012 2013 2014


Quantitat 420 420 420 420 15 420

95

76 
 
 
 
 
 
 

Sucessions
Equacionsnumèriques. Progressions
i inequacions 3
 
 
 
 
 
 

����� ����� � �� � � �
�� � � ���
��� ���

118. En Joan va deixar diners a la seva germana Bet. Amb les dades de la taula, esbrina la quantitat
prestada i el tipus d’interès que es va comprometre a pagar la Bet.

Any 2012 2013 2014 2015


Quantitat 135 135 135 4 635

Suposem que el quart any retorna els interessos anuals compromesos més el capital inicial. Per tant,
podem dir que el capital inicial fou: �� � 4 635 -135 = 4 500 €
���
L’interès anual serà � � � ���
4500

119. L’Esther va aconseguir que un banc li deixés 25 000 € amb la condició que retornaria en un sol
termini tots els diners més el 5 % per cada any que trigués a tornar-los. Avui, després d’uns
quants anys, ha pagat 35 000 € i ha cancel·lat el deute amb el banc. Quants anys ha trigat a
pagar?

Els interessos totals rebuts són 35 000 – 25 000 = 10 000 €. Com que paga en un sol termini
apliquem la fórmula de l’interès simple:
�����
��
���
����� ����������
�� � = 8 anys
��� ��������

120. Calcula en quant es convertiran 1 200 € si els ingressem:


a) Durant 8 anys a un interès compost del 4 %.
b) Durant 6 anys a un interès compost del 5,4 %.
c) Durant 4 anys a un interès compost del 8 %.
� � � �
a) �� � �� � �� � � � ����� � �� � � � ������ ����
��� ���

� ��� �
b) �� � �� � �� � �� � ����� � �� � � � ������ ����
��� ���

� �
c) �� � �� � �� � �� � ����� � �� � �� � ������ ����
��� ���

121. Avui he posat al dia el compte corrent i tenia un saldo de 31 633 €. Quants diners vaig
ingressar-hi fa sis anys si es van comprometre a pagar-me un 4 % anual acumulant els
interessos al capital al final de cada any?

Aïllem C0 de la fórmula de l’interès compost:

96
 
 
 

Successions numèriques. 2
Progressions 2
 
 
 
 
 
 

� �
�� � �� � �� � �
���
� ��
→ �� � �� � �� � �
���

I substituïm els valors:


� ��
�� � �� ��� � �� � � � �� ��� �
���

122. A quin rèdit anual estava sotmesa una operació bancària per la qual 120 € es van convertir al
cap de cinc anys en 146 €?

a) Si tenia reinversió anual (habitual):



Aïllem de la fórmula de l’interès compost: �� � �� � �� � �� →
���
� ���
Anomenem x a �� � � , així tenim que: �� � �
� �� ���
��� ���

� = 1,040002353
x = ��� ���

x=�� ⟹ r = 4,0002353 %
���

b) Si no tenia reinversió anual:


��� � � ��� � � ��� � ��
�� � → � � � � �� � %
��� �� � � ��� � �

123. En Ferran va a una caixa d’estalvis amb el propòsit d’obrir un compte amb 1 400 € i deixar-los-
hi durant quatre anys. Li ofereixen tres alternatives:

a) Un rèdit del 3,49 % anual amb pagament trimestral d’interessos.


b) Un rèdit del 3,5 % anual amb pagament d’interessos cada semestre.
c) Un rèdit del 3,51 % anual amb capitalitzacions anuals. Quina opció li interessa més?
Apliquem la fórmula de l’interès compost
� �
�� � �� � �� � �
���
���� ���
a) �� � � ��� � �� � � � � ��� � �� ��������� � � ���� �� �
�����

��� ���
b) �� � � ��� � �� � � � � ��� � �� ��������� � � ���� �� �
�����

���� �
c) �� � � ��� � �� � � � � ��� � �� ��������� � � ���� �� �
���
L’opció més interessant és la a) ja que rep més interessos.

97

78 
 
 
 
 
 
 
Sucessions numèriques. Progressions
Equacions i inequacions 3
 
 
 
 
 
 

124. Fa cinc anys, en Xavier va obrir tres comptes amb 2 000 € en cada un. Les condicions eren les
següents:

a) Rèdit anual: a %. Pagament d’interessos trimestral.


b) Rèdit anual: b %. Pagament d’interessos semestral.
c) Rèdit anual: c %. Pagament d’interessos trimestral.
Avui, als comptes, hi té aquestes quantitats: 2 322,37 €, 2 378,89 € i 2 433,31 €.
Quin valor tenen a, b i c

� � � ��� � ������ �� �
a) �� � �� � �� � � → � ���� �� � � ��� � �� � � → � � � �� � �→
��� ����� � ��� ���

→ �� ��������� � �� � � → � � �� ��������� �
���

� � � ��� � ������ �� �
b) �� � �� � �� � � → � ���� �� � � ��� � �� � � → � � � �� � �→
��� ����� � ��� ���

→ �� ��������� � �� � � → � � �� �������� �
���

� � � ��� � ������ �� �
c) �� � �� � �� � � → � ���� �� � � ��� � �� � � → � � � �� � �→
��� ����� � ��� ���

→ �� ��������� � �� � � → � � �� �������� �
���

125. La Marta va demanar un préstec de 20 000 € i l’ha estat pagant al 4 % d’interès durant 6 anys.
El mateix dia que va rebre els diners, els va invertir a un 3 % d’interès compost. Si sumes totes
les quantitats que va haver de pagar i totes les que va rebre, hi va sortir guanyant o perdent?

Quants diners?

Els diners pagats anualment són:


����� �� �������
�� � � ��� € / any, així, els sis anys acumulen pagament de 800 · 6 = 4 800 €
��� ���
abonats d’interessos més el capital inicial a retornar paga un total de 24 800 €.

Els diners rebuts al final dels 6 anys amb reinversió són:

� �
�� � �� � �� � � = 20 000 · (1 + 0.03)6 = 23 881,05 €
���
Així, ha pagat 918,95 € més que els que ha rebut.

126. L’Elena i en Dídac van rebre, fa quatre anys, una sorprenent herència des de l’Argentina de
180.000 € cadascun. En Dídac va invertir a la borsa i ha aconseguit una revaloració mitjana
anual del 5 %. L’Elena va comprar lletres del Tresor, que donaven un 5% anual i els interessos
els hi ingressaven anualment en un compte que li donava un rèdit de l’1 % anual. Avui dia, qui
té més diners?

En Dídac a la borsa té reinversió de la mateixa de manera que aconsegueix el mateix interès.

98
 
 
 

Successions numèriques. 2
Progressions 2
 
 
 
 
 
 

‫ܜ ܚ‬ ૞ ૝
۱܎ ൌ ۱૙ ൉ ቀ૚ ൅ ቁ ൌ ૚ૡ૙૙૙૙ ൉  ൬૚ ൅ ൰ ൌ ૛૚ૡૠૢ૚̀
૚૙૙ ૚૙૙
L’Elena ho fa amb reinversió al 1 %
۱൉‫ܚ‬൉‫ܜ‬ ૚ૡ૙૙૙૙൉૞
۷ൌ ൌ ൌ ૢ૙૙૙ €/any
૚૙૙ ૚૙૙
Cal afegir el que es guanya al compte fix al 1%. L’any 2 tindrà 9 000 € que aportaran 90 € addicionals.
L’any 3 tindrà 18 090 € que rendiran 180,9 €, i el quart any tindrà 27 270,9 € que aportaran 272,71 €
addicionals. La suma total dels interessos de les lletres i del compte fix és de 216 543,61 €.

Per veure l’evolució més fàcilment pots observar la següent taula:

Interès  Interessos  Int. lletres  Interès  Capital 


Anys Capital lletres Lletres acumulats fixe Interessos total
€ % € € % € €
1 180.000 5 9.000 1 0 189.000
2 180.000 5 9.000 9.000 1 90 198.090
3 180.000 5 9.000 18.090 1 181 207.271
4 180.000 5 9.000 27.271 1 273 216.544
127. Un banc ofereix un dipòsit AP12, que remunera la inversió al 12 % el primer mes i la resta al 3,7
%. Quants beneficis s’obtindrien al cap d’un any amb una inversió de 100 €?

Aplicarem la fórmula de l’interès com sempre i dividirem per 12, al tenir en consideració que només és
un mes el 12 %.

۱૙ ൉ ‫ ܚ‬൉ ‫ܜ‬ ૚૙૙ ൉ ૚૛ ൉ ૚


۷૚ ൌ ൌ ൌ ૚̀
૚૙૙ ൉ ૚૛ ૚૙૙ ൉ ૚૛
Ara calculem els interessos dels 11 mesos següents i tindrem:
۱૙ ൉ ‫ ܚ‬൉ ‫ ܜ‬൉ ૚૚ ૚૙૙ ൉ ૜ǡ ૠ ൉ ૚ ൉ ૚૚
۷૛ െ ۷૚૙ ൌ ൌ ൌ ૜ǡ ૜ૢ̀
૚૙૙ ൉ ૚૛ ૚૙૙ ൉ ૚૛
Ara sumem els interessos dels dos períodes:
Itotal = 4,39 €

128. El sou íntegre d’un treballador en una ciutat catalana està reflectit a la taula següent:

99

80 
 
 
 
 
 
 
Sucessions numèriques. Progressions 3
Equacions i inequacions
 
 
 
 
 
 

a) Quin percentatge d’augment ha sofert el sou en 5 anys?

b) Quina ha estat la puja acumulada de l’IPC?

c) En aquests cinc anys, ha guanyat poder adquisitiu o n'ha perdut?

�������������
a) El sou ha augmentat en un � �, ����.
������

b) El IPC ha sofert un increment acumulat del 10 % entre 2010 i 2014.


c) Ha perdut un 10 – 7,18 = 2,82 % de poder adquisitiu en aquest període.

129. La taula següent mostra el cost d’un producte que l’any 1996 valia 1 pesseta i el d’un altre que
l’any 2006 valia 1 euro.

Pots completar la taula?

Any  Pessetes  Euros 


1996  1 0,763532678
1998  1,037322 0,792029244
2000  1,07704832 0,822361588
2002  1,15926373 0,88513574
2004  1,223850946 0,93445019
2006  1,30970164 1
  Producte 1  Producte 2 

En groc posarem els valors calculats de la següent manera:


Període 1996 – 2002:
Utilitzem les dades del producte 1 per calcular la inflació (percentatge d’increment de valor en anys
consecutius) amb els valors de les caselles grises, així, mirem quan ha crescut (%) el valor del
producte en pessetes i considerant que tots dos productes han sofert la mateixa inflació l’apliquem al
producte 2.

1,15926373
P. Ex.: Inflació 2000-2002 = � 1,076334003 � ���������1
1,07704832
�,��������
Producte 2 → L’any 2000 hauria tingut un valor en euros de � 0,822361588
�,076334003

Període 2002-2006:
Es procedeix de manera inversa amb el mateix criteri: considerar que la inflació del producte 2 és
igual a la del producte 1 en el mateix període.

100
 
 
 

Successions numèriques. 2
Progressions 2
 
 
 
 
 
 

AMPLIA
130. Quin és el terme central d’una progressió aritmètica de 2n + 1 termes? Quina relació hi ha
entre aquest terme i els extrems?

Per una banda tenim que:

ࢇ࢔ ൌ ࢇ૚ ൅ ሺ࢔ െ ૚ሻ ൉ ࢊ

ࢇ૛࢔ ൌ ࢇ૚ ൅ ሺ૛࢔ െ ૚ሻ ൉ ࢊ

ࢇ૛࢔ା૚ ൌ ࢇ૚ ൅ ሺ૛࢔ሻ ൉ ࢊ ൌ ࢇ૛࢔ ൌ ࢇ૚ ൅ ૛࢔ࢊ

Per altra banda tenim que el terme mig de qualsevol població és:

ࢇ૚ ൅ ࢇ૛ ൅ ࢇ૜ ൅ ‫ ڮ‬൅ ࢇ ࢔
ഥൌ

I també sabem que la suma de tots els termes d’una progressió aritmètica (nostre cas n = 2n + 1) és:
ࢇ૚ ାࢇ૛࢔శ૚
ࡿ૛࢔ା૚ ൌ ൉ ሺ૛࢔ ൅ ૚ሻ

Així podem dir:


ࢇ૚ ାࢇ૛࢔శ૚
ࡿ૛࢔ା૚ ൉ ሺ૛࢔ ൅ ૚ሻ ࢇ૚ ൅ ࢇ૛࢔ା૚ ࢇ૚ ൅ ࢇ૚ ൅ ૛࢔ࢊ

ഥ૛࢔ା૚
ࢇ ൌ ൌ ൌ ൌ ൌ ࢇ૚ ൅ ‫ ܌ܖ‬ൌ ࢇ࢔ା૚
૛࢔ ൅ ૚ ૛࢔ ൅ ૚ ૛ ૛

Així es determina que el terme central és n+1.

Ara comprovem el resultat:

ࢇ૛࢔ା૚ െ ࢇ࢔ା૚ ൌ ࢇ૚ ൅ ૛࢔ࢊ െ ࢇ૚ െ ࢔ࢊ ൌ ࢔ࢊ

ࢇ࢔ା૚ െ ࢇ૚ ൌ ࢇ૚ ൅ ࢔ࢊ െ ࢇ૚ ൌ ࢔ࢊ

Hi ha la mateixa distància (diferència) entre el terme central i els extrems màxim i mínim i, per tant, es
tracta del terme mig.

131. Demostra que si c és la mitjana aritmètica entre a i b, llavors a, c i b estan en progressió


aritmètica.

Definim c:

ࢇ൅࢈
ࢉൌ

Definim d com la distància entre c i a, i definim d’ com la distància entre b i c. Ara haurem de
demostrar que d = d’:

d = c – a; d’ = b – c

d = d’ ?

101

82 
 
 
 
 
 
 
Sucessions numèriques. Progressions 3
Equacions i inequacions
 
 
 
 
 
 

��� � �
�� � � �� �
� � �
��� � �
�� � � � �� �
� � �
Es pot comprovar que d = d’ i, per tant, es tracta d’una progressió aritmètica.

132. De la mateixa manera, si c és la mitjana geomètrica entre a i b, llavors a, c i b estan en


progressió geomètrica.

Definim c:

� � √� � �

Definim r com la raó entre c i a, i definim r’ com la raó entre b i c. Ara haurem de demostrar que r = r’:

� √� � �
�� � ;
� �

� � � √� � � � � √� � � √� � �
�� � � � � � � ;
� √� � � √� � � √� � � ��� �

Es pot veure que les dues expressions són la mateixa i, per tant, es verifica que es compleix la
condició de progressió geomètrica als tres punts.

133. Troba els angles d’un pentàgon sabent que l’angle més petit és recte i que formen una
progressió aritmètica.

�� � ��� 

�� � �� � �� 

� � � �� �� � �� � ��
�� � � � � ��� � � � � ��� � �� � �� � �� � �� � �� 
� �
��
�� � � 

�� � ��� 

102
 
 
 

Successions numèriques. 2
Progressions 2
 
 
 
 
 
 

ࢇ૛ ൌ ૢ૙ι ൅ ૢι ൌ ૢૢ

ࢇ૜ ൌ ૢૢι ൅ ૢι ൌ ૚૙ૡ

ࢇ૝ ൌ ૚૙ૡι ൅ ૢι ൌ ૚૚ૠ

ࢇ૞ ൌ ૚૚ૠι ൅ ૢι ൌ ૚૛૟ 

134. Divideix el polinomi ࢞ૠ െ ૛ૠ entre࢞ െ ૛ mitjançant la regla de Ruffini i comprova que els
termes del polinomi quocient formen una progressió geomètrica.

Aplicant Ruffini:

1 0 0 0 0 0 0 -128
2 2 4 8 16 32 64 128
1 2 4 8 16 32 64 0
 

Per tant:

࢞ૠ െ ૛ૠ
ൌ ࢞૟ ൅ ૛࢞૞ ൅ ૝࢞૝ ൅ ૡ࢞૜ ൅ ૚૟࢞૛ ൅ ૜૛‫ ܠ‬൅ ૟૝
࢞െ૛
Els termes del polinomi quocient són:

1, 2, 4, 8, 16, 32, 64,...

Amb raó r = 2; es comprova atès que es manté r per a tots i cadascun dels termes de la progressió
amb ࢇ૚ ൌ ૚Ǥ

ࢇ૛ ࢇ૜ ࢇ૝ ࢇ૞ ࢇ૟
࢘ൌ ൌ ൌ ൌ ൌ ൌ૛
ࢇ૚ ࢇ૛ ࢇ૜ ࢇ૝ ࢇ૞

135. Fes el mateix amb els polinomis xn - 2n i x – a. Quin és el primer terme de la progressió? I la
raó?

Seguirem els mateixos passos de l’exercici anterior i també les mateixes deduccions:

1 0 0 0 ... െʹ௡
a a ܽଶ ܽଷ --- ܽ௡
1 a ܽଶ ܽଷ --- ܽ െʹ௡

Els termes del polinomi quocient són:

૚ǡ ࢇǡ ࢇ૛ ǡ ࢇ૜ ǡ ǥ ǡ ࢇ࢔

103

84 
 
 
 
 
 
 
Sucessions numèriques. Progressions 3
Equacions i inequacions
 
 
 
 
 
 

Amb raó r =ࢇ; ss comprova atès que es manté r per a tots i cadascun dels termes de la progressió
amb ࢇ૚ ൌ ૚Ǥ

ࢇ૛ ࢇ૜ ࢇ૝ ࢇ࢔
࢘ൌ ൌ ൌ ൌ‫ڮ‬ൌ ൌࢇ
ࢇ૚ ࢇ૛ ࢇ૜ ࢇ࢔ି૚

136. Quina d’aquestes opcions produeix més interessos?

a) Un dipòsit financer en el qual ingressem un capital Ci a un rèdit r durant un temps 2t a


interès compost.

b) Un dipòsit financer en el qual ingressem un capital Ci a un rèdit r durant un temps t a


interès compost.

c) Un dipòsit financer en el qual ingressem un capital Ci a un rèdit 2r durant un temps t a


interès compost.

‫ܚ‬ ‫ܜ‬
Per l’interès compost tenim ۱܎ ൌ ۱ܑ ൉ ቀ૚ ൅ ቁ , per tant, pels casos sol·licitats hem de comprovar que
૚૙૙

donat un Ci com afecten les condicions en el rendiment. Així, només caldrà comprovar per a cada
supòsit el terme exponencial, com es veu a continuació:

‫ܚ‬ ૛‫ܜ‬
a) ቀ૚ ൅ ቁ
૚૙૙

‫ܚ‬ ‫ܜ‬
b) ቀ૚ ൅ ቁ
૚૙૙

૛‫ܚ‬ ‫ܜ‬
c) ቀ૚ ൅ ቁ
૚૙૙

Entre b) i c) es veu fàcilment que sempre que r > 0 c) serà més gran que b).

Entre c) i a) hem de comparar les expressions:

‫ܚ‬ ૛‫ܜ‬ ‫ܚ‬ ૛ ૛࢘ ࢘૛


a) ቀ૚ ൅ ቁ ՜ ቀ૚ ൅ ቁ ՜૚൅ ൅
૚૙૙ ૚૙૙ ૚૙૙ ૚૙૙૙૙

૛൉‫ܜ ܚ‬ ૛൉‫ܚ‬ ૛࢘
c) ቀ૚ ൅ ቁ ՜ ቀ૚ ൅ ቁ՜૚൅
૚૙૙ ૚૙૙ ૚૙૙

૛࢘ ࢘૛ ૛࢘
૚൅ ൅ ൒ ૚൅
૚૙૙ ૚૙૙૙૙ ૚૙૙

Com es pot observar un altre cop sempre que r és diferent de 0 a) serà més gran que c).

137. Compara el capital final que obtindrem per un capital inicial C a un interès r anual si els
interessos es paguen:

a) Al final de l’any.

b) Al final de cada quadrimestre.


104
 
 
 

Successions numèriques. 2
Progressions 2
 
 
 
 
 
 

c) Al final de cada trimestre.

d) Al final de cada mes.

� �
Utilitzem el mateix mètode comparatiu: Tenim l’expressió �� � �� � �� � � que compararem per
���

cada cas aplicant els seus condicionants.

� �
a) �� � ����

� ���
b) �� � ������

� ���
c) �� � ������

� ����
d) �� � �������

Es comprova que per qualsevol r i qualsevol t, sempre es compleix que d > c > b > a.

I, per tant, podem dir que, donats un mateix interès i un mateix capital inicial, la freqüència de
pagaments és directament proporcional al rendiment d’un capital.

138. Per a quin valor de x estan en progressió geomètrica els valors x – 1, x + 3 i x + 15? Per a
aquest valor, troba la suma il·limitada següent:

� � �
� � ��
� � � � � � � � ��

�� � � � �
� �� � � � �
�� � � � ��

�� �� � � � � � ��
�� � → � → �� � ��� � �� � ���� � ��� →
�� �� � � � ���

→ �� � �� � � � �� � ��� � �� → �� � �� →

��
→�� ��

� � �
, , , ….
� � ��

�� � � �
�� � � � �
�� � �

�� � � �
�� � �
� �

��� �� �
� �

105

86 
 
 
 
 
 
 
Sucessions numèriques. Progressions
Equacions i inequacions 3
 
 
 
 
 
 

139. En un quadrat de costat 1, unim els punts mitjans de cada costat i obtenim un altre quadrat, i
així successivament.

a) Calcula el perímetre i les àrees dels diferents quadrats i demostra que formen dues
progressions geomètriques.

b) Calcula la suma de tots els perímetres i de totes les àrees.

� � � �
Longitud del costat segueix la progressió: �, , , , ,…
√� � �√� �

� � � � � √�
Perímetre de cada quadrat segueix la progressió: �, , , , ,… . �� �
√� � �√� � √� �

� � � � �
Àrea de cada quadrat segueix la progressió: �, , , , ,… . ��
� � � �� �

Suma de perímetres:

�� � � � � ��√� ����√�� ����√�


�� � � � � ��√�
� � � � � � � � �√� � ��� ��������
��� �� ��√� ��√� ��√� � �
√� �

�� �
Suma d’àrees: �� � � � ��
���

106
 
 
 

Successions numèriques. 2
Progressions 2
 
 
 
 
 
 

MATEMÀTIQUES A LA TEVA VIDA


1. Quines dues eines s’han de fer servir per avaluar un crèdit?

Les dues eines més importants per avaluar aquestes dades són la fórmula que dóna les anualitats
que s’han d’anar amortitzant i les taules d’amortització.

2. Quins dos tipus d’interessos hi ha en el mercat dels préstecs?

Hi ha préstecs d’interès fix i d’interès variable en el temps perioditzats en funció d’uns índex de
referència.

3. Quina és la fórmula que s’esmenta en el text?

La fórmula de l’interès compost.

4. En aquesta fórmula de la qual es parla en el text, què representa la lletra i?

5. En la fórmula de les quotes d’amortització hi ha un denominador. En quins casos no té sentit


l’expressió que ens dóna el capital final?

6. Un banc ofereix crèdits al 8 % per tornar en 4 anys. L’oferta d’una altra entitat és un interès al 4
% amb un temps d’amortització de 8 anys. Si necessites 33 000 €, quin banc has d’escollir per
demanar-hi els diners?

Suposem que l’interès esmentat és fix, anual i compensat al llarg de tot el període compromès. Això,
ens portarà a la següent taula:

Any  A  B 
1  2.640  1.320 
2  2.640  1.320 
3  2.640  1.320 
4  2.640  1.320 
5    1.320 
6    1.320 
7    1.320 
8    1.320 
  10.560  10.560 

107

88 
 
 
 
 
 
 

Sucessions
Equacionsnumèriques. Progressions
i inequacions 3
 
 
 
 
 
 

Com es pot comprovar no hi ha diferència econòmica, ja que els interessos a pagar sumen la mateixa
xifra. No obstant, el període de retorn de la quantia total és menor en el primer banc. Així la opció que
es permet gaudir més dels beneficis del préstec és la B

Suposem que el tipus de interès es fix però que el capital s’amortitza proporcionalment cada any que
passa. Veiem a les taules següents els resultats d’aquesta hipòtesis.

Banc A    Banc B 
Any  Capital  Interès  Restant    Capital  Interès  Restant 
1  33.000,00  2.640,00  30.360,00    33.000,00  1.320,00  31.680,00 
2  30.360,00  2.428,80  27.931,20    31.680,00  1.267,20  30.412,80 
3  27.931,20  2.234,50  25.696,70    30.412,80  1.216,51  29.196,29 
4  25.696,70  2.055,74  23.640,97    29.196,29  1.167,85  28.028,44 
5             28.028,44  1.121,14  26.907,30 
6             26.907,30  1.076,29  25.831,01 
7             25.831,01  1.033,24  24.797,77 
8             24.797,77  991,91  23.805,86 
  9.359,03    9.194,14   

Si observem els resultats obtinguts, la opció més favorable torna a ser el banc B, per quantitat total
retornada, per cost (interessos) i també perquè els pagaments intermitjos seran més petits.

108
 
 
 
 
 
 

3
 
 
 
 
 
 

Equacions i inequacions
 

 
Equacions i inequacions   
 
 
 
 
 

ACTIVITATS 
 
1. Escriu un polinomi de grau 3 i amb terme independent -1. Determina’n els termes i el valor
numèric per a x = 2 i x = -2.
Resposta oberta. Per exemple:
P(2)  223 321  21
P(x)  2x 3 3x 1  
P(2)  2(2) 3(2)1  23
3

2. Efectua l’operació de polinomis següent:


(-2x3 + x2 + x – 1) + (3x + 1)(x+3)
(‐2x3 + x2 + x – 1) + (3x + 1)(x+3) = ‐2x4 + 5x3 + 2x2 + 7x + 5 
 
3. Donats els polinomis següents, calcula:
P(x) = x3 – 3x2 + 1 Q(x) = 3x2 – 2x – 2
a) P(1) + P(-1) d) P(-2) · Q(-2)
b) P(0) – 2Q(0) e) P(-1) – 3Q(-1)
c) P(3) + Q(2) f) Q(-4) + Q(1)
a) P(1)P(1)  13  4 d) P(2)Q(2)  (19)14  266
b) P(0) – 2Q(0)  1 4  5 e) P(1) – 3Q(1)  3 9  12

c) P(3)Q(2) 16  7 f) Q(4) 4Q(1)  54  4  50

4. Efectua les divisions de polinomis següents:

a) (10 x 4 – 3 x 2  1) : ( x 2 – 1) b) (6 x 3  5x ) : (2 x 2 )

a) (10x 4 – 3x 2 1): (x 2 – 1)  10x 2  7 i el residu és 8.

b) (6x 3  5x): (2x 2 )  3xy i el residu és 5x.


5. Divideix aquests polinomis per mitjà de la regla de Ruffini:

a) (x 3  3) : (x  1) b) (4 x 5 – 12 x 3 – 20 x  2) : ( x  2)

a) (x 3  3): (x 1)  x 2  x 1 i el residu és 2.


1 0 0 3
-1 –1 1 –1
1 –1 1 2
b) (4x 5 – 12x 3 – 20x  2): (x  2)  4x 4  8x 3  4x 2  8x  4 i el residu és 10.
4 0 –12 0 –20 2
–2 –8 16 –8 16 8
4 –8 4 –8 –4 10

109
 
 
 
 
 
 

Equacions i inequacions
Equacions i inequacions 3
 
 
 
 
 
 

6. Comprova si els nombres següents són arrels del polinomi


P(x) = x4 + 3x3 – 2x2 + 6x – 8.
a) x = 1 b) x = 2 b) x = -1 b) x = -4
a) P(1) = 1 + 3 · 1 – 2 · 1 + 6 · 1 – 8 = 0
4 3 2

Per tant, x = 1 és una arrel del polinomi.


b) P(2) = 24 + 3 · 23 – 2 · 22 + 6 · 2 – 8 = 36
c) P(-1) = (-1)4 + 3 · (-1)3 – 2 · (-1)2 + 6 · (-1) – 8 = -18
d) P(-4) = (-4)4 + 3 · (-4)3 – 2 · (-4)2 + 6 · (-4) – 8 = 0
Per tant, x = -4 és una arrel del polinomi.

7. Calcula les arrels enteres dels polinomis que tens a continuació.

a) x 5  4 x 4  x 3 – 6 x 2 b) x 3 – 5 x 2  29 x  105
 x  0
5 4 3 2 2 3 2
a) x  4x  x – 6x  x (x  4x  x – 6)  
 x 3  4x 2  x – 6  0
1 4 1 –6
1 1 5 6
1 5 6 0
–2 –2 –6
1 3 0
–3 –3
1 0
Les arrels enteres són {–3, –2, 0, 1}.
b) S’aplica Ruffini directament:
1 –5 –29 105
7 7 14 –105
1 2 –15 0
–5 –5 15
1 –3 0
3 3
1 0
Les arrels enteres són {–5, 3, 7}.

8. Factoritza els polinomis següents: 

b) x 5  4 x 4  5x 3 – 4 x2  4 x  
4 2
a) 2 x – 2 x
4 2 2
a) 2x – 2x  2x (x 1)(x 1)
5 4 3 2 2 2
b) x  4x  5x – 4x  4x  x(x 2) (x 1)

9. Determina les arrels enteres d’aquests polinomis: 

a) 2 x 4 – 13x 3  27 x 2 – 18 x b) 3x 3  7x 2 – 7x  3  

4 3 2 3 2
 x  0
a) 2x – 13x 27x – 18x  x(2x – 13x 27x – 18)  
2x 3 – 13x 2 27x – 18  0

110
 
 
 
 
 
 

Equacions i inequacions 33
 
 
 
 
 
 

2 –13 27 –18
2 4 –18 18
2 –9 9 0
3 6 –9
2 –3 0
3
L’última arrel no és entera: 2x  3  0  x 
2
Per tant, les arrels enteres són {0, 2, 3}.
b) S’aplica Ruffini directament:
3 –7 –7 3
–1 –3 10 –3
3 –10 3 0
3 9 –3
3 –1 0
1
L’última arrel no és entera: 3x 1  0  x 
3
Per tant, les arrels enteres són {–1, 3}.

10. Simplifica les fraccions algèbriques següents: 


�� �� ��� ������
a) b)
�� ����� �����
2 2
x x x(x 1) x
a)   b) 2x 2x 12  2(x 2)(x  3)  x 2
x 2  2x 1 (x 1)(x 1) x 1 4x 12 4(x  3) 2

11. Redueix a comú denominador aquestes fraccions: 


� �� ���
a) b) c)
�� ����� �� �� �� �����

m.c.m. (x 2  2x  3, x 2 1, x 2  2x 1)  (x 1)2 (x  3)(x 1)

5 5(x 1)2

x  2x  3 (x 1)2 (x  3)(x 1)
2

3x 3x (x 1)(x  3)

x 1 (x 1)2 (x  3)(x 1)
2

x 1 (x 1)2 (x  3)

x  2x 1 (x 1)2 (x  3)(x 1)
2

12. Opera i simplifica. 


� � ���
� �
��� ��� � �����
3 1 2 x x 2  4x 18
  
32  6x x 6x 12 6x(x  2)

13. Efectua les operacions següents: 


� �� �� ��� �� �����
a) �     b)  :  
�� ����� �� �� ����� ���
111
 
 
 
 
 
 

Equacions i inequacions
Equacions i inequacions 3
 
 
 
 
 
 

5 x 2 1 5(x 1)(x 1) 5x  5


a) ∙  
x  2x  3 3x
2
(x  3)(x 1)3x 3x 2  9x

b) x 1 x 2  3x  2 (x 1)(x 1) 1
:  
2
x  2x 1 x 1 (x 1) (x 1)(x  2) (x 1)(x  2)
2

14. Classifica i resol aquestes equacions de segon grau: 

a) �� � ��� � �� � � e) 3�� � ��� � �

b) ��� � ��� � �� � � f) 4�� � �� � �

c) ��� � �� � � � � g) -8�� � �� � �

d) ��� � ��� � � � � h) -5�� � ��� � �

a) Equació completa:
������ � ������� � � � � � ��
�� � ��� � �� � � � � �
���
���� � ��
��� � � �
� �� � �
b) Equació completa:
��� � √��� � � � � � ��
�� � ��� � �� � � � � �
���

������ � ��
��� � � � �

�� � ��
c) Equació completa:
�� � ��� � � � � � ����
�� � � �� � � � � � � �
���
���√��� � � � �� ��
��� � �
� � � � ��� ��
d) Equació completa:
������ � ������� � � � � � �
�� � � ��� � � � � � � �
���
�� �
��� �
� �

e) Equació incompleta:
�� � � ��� � � � ���� � �� � � � � � � � �� � �
f) Equació incompleta:
�� � � �� � � � � � √� � � � � �� �� � �

g) Equació incompleta:
��� � � �� � � � � � √�
h) Equació incompleta:
��� � � ��� � � � ����� � �� � � � � � � � �� � �
112
 
 
 
 
 
 

Equacions i inequacions 33
 
 
 
 
 
 

15. Resol les equacions següents: 


a) x2 + 2x = 15
b) 2x2 = 7x + 2
c) 3x2 – 3 = 20 – 2(x – 5)
d) 8x2 + (2 – x)(5x + 1) = 15x + (3 + x)(x – 1) – 3
a) x 2 2x  15  x 2 2x 15  0

2 22  41(15) 2 8  x1  5


x x 
21 2  x2  3

b) 2x 2  7x 2  2x 2  7x 2  0

 x1  7 65  0,27
2
7 (7)  42(2) 7 65  4
x x 
22 4  7 65
 x2   3,77
4
2
  2
c) 3x – 3  20 – 2 x – 5  3x 2x 33  0

 11
2 22  4 3(33) 220  x1  
x x  3
23 6 x  3
 2

d) 8x 2 (2  x)(5x 1)  15x (3 x)(x 1) – 3  2x 2  8x  8  0

8 (8)2  428 8 0
x x  x 2
22 4

16. Determina el nombre de solucions que tenen aquestes equacions sense resoldre-les: 

 
Calculem el discriminant:
a) Δ = b2 -4ac = 52 - 4 · (-2) · (-8) = -39 < 0. No té solució real.
b) Δ = b2 -4ac = 302 - 4 · 9 · 25 = 0. Té una solució
c) Δ = b2 -4ac = 92 - 4 · (-5) · (-6) = -39 < 0 No té solució real.
d) Δ = b2 -4ac = (-1)2 - 4 · 2 · (-3) = 25 > 0 Té dues solucions.
e) Δ = b2 -4ac = 92 - 4 · (-1) · (-2) = 73 > 0 Té dues solucions.
f) Δ = b2 -4ac = 0,52 - 4 · 0,34 · (-1) = 1,61 > 0 Té dues solucions.
17. Resol les equacions biquadrades que tens a continuació:

a) x 4  5x 2 – 36 
0

b) x 4 – x 2  2 16 x 2 – 14

c) 11( x 4  1) – 7 
25 x 2 (1 – x 2 )

113
 
 
 
 
 
 

Equacions i inequacions
Equacions i inequacions 3
 
 
 
 
 
 

a) x 4 5x 2 – 36  0  z 2 5z – 36  0

513  z1  9  z1  9  x  9  No té solució real


2
5 52  41(36)
z z  
21 2  z2  4  z2  4  x1  2 x2  2

b) x 4 – x 2 2  16x 2 – 14  x 4 – 17x 2 16  0  z 2 – 17z 16  0

17  (17)2  4116 1715  z1  1  z  1  x  1 x  1


z z   1 1 2

21 2  z2  16  z2  16  x 3  4 x 4  4

c) 11(x 4 1) – 7  25x 2 (1 – x 2 )  36x 4 – 25x 2  4  0  36z 2 – 25z  4  0

 1  1 1 1
2  z1   z1   x1   x2 
25 (25)  4 4 36 25 7 4 4 2 2
z z  
236 72 z  4 4
z   x   2 2
x4 
 2 9  2 9 3
3 3
 
18. Resol aquestes equacions amb fraccions algèbriques:
x 1 1 2x 4  4 x2  3
a) 1    b)   2 2
x 1 x x4 x

 
a) x 1 1  1  x(x 1)  x(x 1)  x 1  x  1
x 1 x x(x 1) x(x 1) x(x 1)

2x 4  4 x 2  3 2x 4  4 x 2 (x 2  3) 2x 4
b) 4
 2 2  4
 4
 4  x 4  3x 2  4  0
x x x x x
x 4  3x 2  4  0  z2  3z  4  0

3 (3)2  4 1(4) 3 5  z1  1
z z 
21 2  z2  4

z1  1 x 2  1 No té solució real

z2  4  x1  2 x2  2

19. Resol les equacions amb radicals següents:

a) x  41 2 x  1  

b) 1  2 x 
1 3 4x  3  4  

a) x 41  2 x 1  x 2 82x 1681  4(x 1) x 2 86x 1677  0



86  (86)2  411677 86  688  x1  432 43
x x 
21 2  x2  432 43

Comprovem i obtenim que x1  432 43 és l’única solució vàlida.

114
 
 
 
 
 
 

Equacions i inequacions 33
 
 
 
 
 
 

b) 12 x 1  3 4x 3  4  4(x 1)  36x 30 4x 3 2


 900(4x 3)  1024x 2 384x  36  64x 2  249x 171  0
 57
249 (249)2  464171 249135  x1 
x x  64
264 128 x  3
 2
Comprovem i obtenim que x2 = 3 és l’única solució vàlida.
20. Resol les equacions següents, que es troben en forma factoritzada:

a) x1  5 x2 1 x3  3

b) x1  4 x2  0 x3  0 x 4  4 x5  9
3 1
c) x1  2 x2   x3 
2 3
1
d) x1  5 x2  5 x3  5 x 4  0 x5 
3

e) x1  3 x2  2 x3 1 x4  3

21. Factoritza aquestes equacions i soluciona-les:

a) (x - 1)(x - 2)(x - 3)(x + 4) = 0


x1 = 1 x2 = 2 x3 = 3 x4 = -4
b) x(x - 3)2(x2 + 1) = 0
x1 = 0 x2 = 3
c) (x + 4)2(x2 + 1) = 0
x1 = -4
22. Escriu una equació que tingui com a solucions 2, 3 i 7. Quin és el grau mínim que pot tenir?
Resposta oberta, per exemple:
(x 2)(x 3)(x 7)  0  El mínim grau que pot tenir és 3.

23. Resol aquestes equacions logarítmiques: 


a) log (x – 2) + log (2x – 1) = log (3x – 4)  

115
 
 
 
 
 
 

Equacions i inequacions
Equacions i inequacions 3
 
 
 
 
 
 

b) log (x + 3) = log (7x – 27) – log (x – 4)  


 x  1
a) log(x  2)log(2x 1)  log(3x  4)  (x 2)(2x 1)  3x  4  2x 2  8x 6  0   1
 x2  3
 x  5
b) log(x  3)  log(7x 27)log(x  4)  (x  3)(x  4)  7x 27  x 2 8x 15  0   1
 x2  3

24. Soluciona les equacions següents: 


a) log3 (2x – 1) = 1 c) logx 2 + logx 5 = 1
b) log2 (3x – 2) = 2 d) logx 12 + logx 18 = 3

a) log3 (2x 1)13  2x 1 x  2

b) log2 (3x 2)  2 4  3x 2 x  2

c) log x 2log x 5 1 log x 10 1 x 10

d) log x 12log x 18  3log x 216  3 x 3  216  x  6


 
25. Resol les equacions següents: 

 
 
a) 53x1  625  3x 1  4  x  1
3x1
1 3x 1
b) 2 4
   5  x  7
32 4

2
 x  6
c) 3x 7x12  729  x 2  7x 12  6  x 2  7x  6  0   1
 x2  1
x1
d) 5 x2  25  x 1  2  x  5
x 2
x  0
 1
e) 4 x 3 7 x 2 12x3 3 2 2
 64  x  7x 12x  3  3  x(x  7x 12)  0   x2  3

 x3  4

116
 
 
 
 
 
 

Equacions i inequacions 33
 
 
 
 
 
 

17
f) 4 x6  32  2(x  6)  5  x 
2
x2
1 x 2
g) 3 3
   1  x  1
3 3
3 23
h) 6
2x 2  3 23 17
5
 6 10  2x 2    20x 2  6  23  20x 2  17  x  
5 10 20
x3
x 3 1
i) 17 3x2  289   2  5x  1  x 
3x  2 5

4 2 3 x   2
j) 3x 13x 4  36 x 12x  x 4 6x 3 13x 2 12x  4  0   1
 x2  2

26. Calcula el valor de x en aquestes equacions: 

 
a) 4  2 x x6
 2x  x  6  x  6
7 x3
7x  3
b) 3 4
 9 x2   2(x  2)  x  12
4

2  x  4
c) 49 x  73x  2x  3 x 2  x 2 2x  3  0   1
 x2  1
5 x2  x  2 i x  2
x 4 4
d) 5  25 2
 x 4  4  5x 2   1 2

 x 3  1 i x 4  1

27. Resol les equacions següents: 

log3
a) 32x1  2 x  (2x 1)log3  xlog2  x (2log3log2)  log3  x   0,7304
2log3log2
3x
1 1 
b)    32x  0  (3 x)log    (2 x)log(3)
2 2
No existeixen logaritmes de nombres més grans o iguals que 0. L’equació no té solució real.

c) 53x2  35x2 (3x 2)log5 (5x 2)log3 x(3log55log3) 2log32log5


2 log3  2 log5
x  1,5369
3 log5  5 log3

x2 
3 x2 x1  x  3 3
d) 3 9  3  32(x1)  x 2 6x 6  0   1
3

 x2  3 3

117
 
 
 
 
 
 

Equacions i inequacions
Equacions i inequacions 3
 
 
 
 
 
 

28. Resol les inequacions següents:


a) 3x –5 4x –7 c) 5 – ( 2 – 3 x )  2(3 x – 5)  x

b) 2x –30  5x  3 d) 2(2  x )  x  5

a) 3x – 5  4x – 7  x  2  2  x   (2,)
b) 2x – 30  5x  3  x  11  11 x    [11,)
 
c) 5 – (2 – 3x)  2(3x – 5)  x  x  13     x  13   , 13
4 4  4

d) 2(2  x)  x  5  x  3     x  3  (,3)

29. Resol les inequacions que tens a continuació:


3  2x x 2 x  5 1  3x x
a) 1   b)   1
3 4 3 6 4
3 2x x
a) 1   x  0     x  0  (,0]
3 4
2x  5 1 3x x
b)   1  x  2  2  x    (2,)
3 6 4

30. Resol les inequacions de segon grau amb una incògnita següents:

�� � 1
a) Resolem l’equació: x2 – 3x + 2 = 0 → �
�� � 2
Prenem un punt de cada interval on la recta queda dividida.
x=0 x = 1,5 x=3
Si x = 0 → 0 – 3 · 0 + 2 > 0 → (-∞, 1) no és solució de la inequació.
2

Si x = 1,5 → 1,52 – 3 · 1,5 + 2 < 0 → (1, 2) és solució de la inequació.


Si x = 3 → 32 – 3 · 3 + 2 > 0 → (2,+∞) no és solució de la inequació.
Les solucions de l’equació ho són també de la inequació.
Per tant, la solució és �1,2�.

b) Es dedueix de l’apartat anterior que les solucions de la inequació són:


(-∞,1] ∪ [2,��∞)
�� � �
c) Resolem l’equació: x2 – 9x = 0→ �
�� � �
Prenem un punt de cada interval on la recta queda dividida:

118
 
 
 
 
 
 

Equacions i inequacions 33
 
 
 
 
 
 

x = -1 x=1 x = 10
Si x = -1 → (-1)2 – 9 · (-1) > 0 → (-∞, 0) és solució de la inequació.
Si x = 1 → 12 – 9 · 1 < 0 → (0,92) no és solució de la inequació.
Si x = 10 → 102 – 9 · 10 > 0 → (9,+∞) és solució de la inequació.
Les solucions de l’equació no ho són de la inequació.
Per tant, la solució és (-∞, 0) ∪ (9,+∞)
�� � ��
d) Resolem l’equació: x2 – 9 = 0 → �
�� � �
Prenem un punt de cada interval on la recta queda dividida.
x = -10 x=0 x = 10
Si x = -10 → (-10)2 – 9 > 0 → (-∞, -3) no és solució de la inequació.
Si x = 0 → 02 – 9 < 0 → (-3, 3) és solució de la inequació.
Si x = 10 → 102 – 9 > 0 → (3,+∞) no és solució de la inequació.
Les solucions de l’equació no ho són de la inequació.
Per tant, la solució és (-3, 3).
e) El primer membre de la inequació sempre serà positiu.
Per tant, la inequació no té solució.
x� � ��
f) Resolem l’equació: (x – 3)(x + 4) = 0 → �
x� � �
Prenem un punt de cada interval on la recta queda dividida.
x = -10 x=0 x = 10
Si x = -10 → (-10 – 3)(-10 + 4) > 0 → (-∞, -4) és solució de la inequació.
Si x = 0 → (0 – 3)(0 + 4) < 0 → (-4, 3) no és solució de la inequació.
Si x = 10 → (10 – 3)(10 + 4) > 0 → (3,+∞) és solució de la inequació.
Les solucions de l’equació ho són també de la inequació.
Per tant, la solució és (-∞, -4] ∪ [3,+∞)
x� � ��
g) Resolem l’equació: (x + 3)x = 4 → �
x� � �
Prenem un punt de cada interval on la recta queda dividida.
x = -10 x=0 x = 10
Si x = -10 → (-10 + 3) · (-10) – 4 > 0 → (-∞, -4) no és solució de la inequació.
Si x = 0 → (0 + 3) · 0 - 4 < 0 → (-4, 1) és solució de la inequació.
Si x = 10 → (10 + 3) · 10 + 4) > 0 → (1,+∞) no és solució de la inequació.
Les solucions de l’equació ho són de la inequació.
Per tant, la solució és (-4,1).
x � ��
h) Resolem l’equació: x2 – x – 30 = 0 → � �
x� � �
Prenem un punt de cada interval on la recta queda dividida.
x = -10 x=0 x = 10
Si x = -10 → (-10) – 10 – 30 > 0 → (-∞, -5) no és solució de la inequació.
2

Si x = 0 → 02 - 0 – 30 < 0 → (-5, 6) és solució de la inequació.


119
 
 
 
 
 
 

Equacions i inequacions
Equacions i inequacions 3
 
 
 
 
 
 

Si x = 10 → 102 - 10 – 30 > 0 → (6,+∞) no és solució de la inequació.


Les solucions de l’equació no ho són de la inequació.
Per tant, la solució és (-5,6).
i) El primer membre de la inequació és sempre més gran o igual que zero.
Per tant, la inequació no té solució.
j) El primer membre de la inequació és sempre més gran o igual que zero.
Per tant, la inequació no té solució.

31. Resol aquestes inequacions de grau superior seguint el mètode utilitzat per a les inequacions
de segon grau:
a) (x – 2)(x – 3)(x2 – 2) � 0
b) x(x – 4)(x + 1)(x3 – 1) � 0
c) x3 + 2x2 + 3x - 6 < 0
d) x4 – 5x3 + 4x2 + 9x - 9 > 0

��� � �√�

a) Resolem l’equació: (x – 2)(x – 3)(x2 – 2) = 0 → �� � √�
� �� � �
� � � �
� �
Prenem un punt de cada interval on queda dividida la recta:
x = -10 x=0 x = 1,5 x = 2,5 x = 10
Si x = -10 → (-10 – 2)(-10 – 3)((-10)2 – 2) > 0 → (-∞, √2) és solució.
Si x = 0 → (0 – 2)(0 – 3)(02 – 2) < 0 → (√2, -√2) no és solució.
Si x = 1,5 → (1,5 – 2)(1,5 – 3)(1,52 – 2) > 0 → (√2,2) és solució.
Si x = 2,5 → (2,5 – 2)(2,5 – 3)(2,52 – 2) < 0 → (2,3) no és solució.
Si x = 10 → (10 – 2)(10 – 3)(102 – 2) > 0 → (3, +∞) és solució.
Les solucions de l’equació ho són també de la inequació.
Per tant, la solució és (-∞, -√2,] ∪ [√2, 2� ∪ [3,+∞)
�� � ��
� � � �
b) Resolem l’equació: x(x – 4)(x + 1)(x3 – 1) = 0 → �
� � � �
� � � �
Prenem un punt de cada interval on queda dividida la recta:
x = -10 x = - 0,5 x = 0,5 x=2 x = 10
Si x = -10 → �10 · (-10 – 4)(-10 + 1)((-10) – 1) > 0 → (-∞, -1) no és solució.
3

Si x = -0,5 → -0,5 · (-0,5 – 4)(-0,5 + 1)((-0,5)3 – 1) < 0 → (-1, 0) és solució.


Si x = 0,5 → 0,5 · (0,5 – 4)(0,5 + 1)(0,53 – 1) < 0 → (0, 1) no és solució.
Si x = 2 → 2 · (2 – 4)(2 + 1)(23 – 1) < 0 → (1, 4) és solució.
Si x = 10 → 10 · (10 – 4)(10 + 1)(103 – 1) > 0 → (4,+∞) no és solució.
Les solucions de l’equació ho són també de la inequació.

120
 
 
 
 
 
 

Equacions i inequacions 33
 
 
 
 
 
 

Per tant, la solució és [-1,0� ∪ [1,4].

c) Resolem l’equació: x3 + 2x2 + 3x - 6 = 0 → x = 1


Prenem un punt de cada interval on queda dividida la recta:
x=0 x = 10
Si x = 0 → 03 + 2 · 02 + 3 · 0 – 6 < 0 → (-∞, 1) és solució.
Si x = 10 → 103 + 2 · 102 + 3 · 10 – 6 > 0 → (1,+∞) no és solució.
Les solucions de l’equació no ho són de la inequació.
Per tant, la solució és (-∞, 1) .
�� √��
��� � �
� � � �

d) Resolem l’equació: x4 – 5x3 + 4x2 + 9x - 9 = 0 →
� �� � ��√��
� �
� �� � �
Prenem un punt de cada interval on queda dividida la recta:
x = -10 x=0 x=2 x = 2,5 x = 10
�� √��
Si x = -10 → (-10)4 – 5 · (-10)3 + 4 · (-10)2 + 9 · (-10) – 9 > 0 → (-∞, ) és solució.

�� √��
Si x = 0 → 04 – 5 · 03 + 4 · 02 + 9 · 0 – 9 < 0 → ( ,1) no és solució.

�� √��
Si x = 2 → 24 – 5 · 23 + 4 · 22 + 9 · 2 – 9 > 0 → (1, ) és solució.

�� √��
Si x = 2,5 → 2,54 – 5 · 2,53 + 4 · 2,52 + 9 · 2,5 – 9 < 0 → ( , 3) no és solució.

Si x = 10 → 104 – 5 · 103 + 4 · 102 + 9 · 10 – 9 > 0 → (3,+ ∞) és solució.

Les solucions de l’equació no ho són de la inequació.


�� √�� �� √��
Per tant, la solució és (-∞, ) ∪ (1, ) ∪ (3, +∞).
� �

121
 
 
 
 
 
 

Equacions i inequacions
Equacions i inequacions 3
 
 
 
 
 
 

SABER FER 
 
32. Determina el valor de k per a cadascun dels casos següents:
a) 3x2 – 6x + k = 0 tingui 2 solucions. d) x2 + kx + 25 = 0 tingui 2 solucions.
b) 3x2 – 6x + k = 0 tingui 1 solució. e) x2 + kx + 25 = 0 tingui 1 solució.
c) 3x2 – 6x + k = 0 no tingui solució. f) x2 + kx + 25 = 0 no tingui solució.
a) (-6)2 – 4 · 3 · k > 0 → 36 – 12k > 0 → k < 3
b) (-6)2 – 4 · 3 · k > 0 → 36 – 12k > 0 → k < 3
c) (-6)2 – 4 · 3 · k > 0 → 36 – 12k > 0 → k < 3
d) k2 – 4 · 25 · 1 > 0 → k2 – 100 > 0
e) k ha de pertànyer a l’interval (-∞, -10) ∪ (10, �∞)
f) k2 – 4 · 25 · 1 = 0 → k2 – 100 = 0 → k1 = -10 k2 = 10
g) k2 – 4 · 25 · 1 < 0 → k2 – 100 < 0
h) k ha de pertànyer a l’interval (-10, 10)

33. Resol les equacions següents:


a) x6 – 9x3 + 8 = 0 c) x6 – 7x3 - 8 = 0 e) x8 – 15x4 – 16 = 0
b) x6 + 7x3 - 8 = 0 d) x6 + 9x3 + 8 = 0 f) x8 – 17x4 + 16 = 0
a) 
x 6  9 x3  8 0  z 2  9 z  8 
0

9  (9)2  418 97  z  1


z  z 1
21 2  z2  8

z1  1  x3  1  x1  1 z2  8  x3  8  x2  2

b) x6  7x3  8  0  z2  7z 8  0

7  72  41 (8) 7  9  z  8
z  z 1
21 2  z2 1

z1  8  x3  8  x1  2 z2 1  x3 1  x2 1
c) x6  7x3  8  0  z2  7z 8  0

7  (7) 2  4 1 (8) 79  z  1


z  z  1
21 2  z2  8

z1  1  x3  1  x1  1 z2  8  x3  8  x2  2
d) x6  9x3  8  0  z2  9z 8  0

9  92  4 18 9  7  z  8
z  z  1
21 2  z2  1

z1  8  x3  8  x1  2 z2  1  x3  1  x2  1

122
 
 
 
 
 
 

Equacions i inequacions 33
 
 
 
 
 
 

e) x8 15x4 16  0  z2 15z16  0

15 (15) 2  4 1 (16) 1517  z  1


z  z  1
2 1 2  z2  16

z1  1  x4  1  No té solució real z2 16  x4 16  x1  2 x2  2


f) x8 17x4 16  0  z2 17z16  0

17  (17)2  4 116 17 15  z  1


z  z  1
2 1 2  z2  16

z1 1  x4 1  x1  1 x2 1 z2 16  x4 16  x3  2 x4  2

34. Resol les equacions amb les fraccions algèbriques següents:


૛‫ ܠ‬૜ ି‫ ܠ‬૛ ି૛‫ܠ‬ା૛૞ ࢞૜ ା࢞૛ ାૠ࢞ା૛ ࢞૜ ା૞࢞૛ ି૞࢞ି૛૚
a) ൌ ૛‫ܠ‬ b) ൌ࢞െ૛ c) ൌ࢞൅૜
‫ ܠ‬૛ ି૚ ࢞૛ ା૛࢞ା૝ ࢞૛ ା࢞ି૟

2x3  x2  2x  25
a)  2x  x2  25  0  x1  5 x2  5
x2 1
x3  x2  7x  2
b) 2
 x  2  x2  7x 10  0  x1  2 x2  5
x  2x  4
x3  5x2  5x  21
c)  x  3  x2  2x  3  0  x1  1 x2  3
x2  x  6

35. Resol aquestes equacions amb fraccions algèbriques:


x2  x x x 1 x  2
a)    c)  
3x  1 2 x  1 x3 x4
x x5 x3 x4
b)    d)  
x6 x3 x 1 x  2

x2  x x
a)   2x3  2x  0  x  0
3x 1 2x 1
x x5 15
b)   4x  30  0  x 
x6 x3 2
x 1 x  2
c)   2  0  No té solució
x3 x 4
x3 x4
d)   2  0  No té solució
x 1 x  2

36. Resol aquestes equacions:


x x
a) x2  3 
1 c)  6
2 2

b) 
x x6 d) 3x  19 x  3

123
 
 
 
 
 
 

Equacions i inequacions
Equacions i inequacions 3
 
 
 
 
 
 

a) x2  3 1  x2  4  0  x1  2 x2  2

b) x  x 6  x2  x 6  0  x1  2 x2  3

c) x  x  6  x2  2x  24  0  x1  4 x2  6
2 2

d) 3x19  x 3  x2  3x 10  0  x1  2 x2  5
Si substituïm els parells de solucions obtingudes en l’equació corresponent, podem afirmar que
totes són solucions vàlides.

37. Resol les equacions següents:

a) 2x8  x  2  x2 8x16  0  x  4

b) x2 5  x 2  3  x4  2x3  23x2 12x 216  0  x  3

c) 2x  4x 7  x1  x4 8x3 8x 64  0  x  2 x  8

d) 3x1  x2  3x 9  2x1  9x4  24x3  90x2 84x 45  0  x  5

38. Resol aquestes equacions:

a) x3 – 3x2  2x  0  x(x1)(x 2)  0  x1  0 x2 1 x3  2
5 4 3 2 2 2 1
b) 3x – 13x 16x – 4x  0  x (3x 1)(x  2)  0  x1  0 x2  x3  2
3

1
c) 4x5 – 12x4  9x3 – 2x2  0  x2 (2x 1) 2 (x  2)  0  x1  0 x2  x3  2
2

d) x4 –1 0  (x1)(x1)(x2 1)  0  x1  1 x2 1

39. Calcula el valor de x en els logaritmes que tens a continuació:

a) log5 x  4  x  54  625
b) log (x 1)  2  x 1 100  x  101

124
 
 
 
 
 
 

Equacions i inequacions 33
 
 
 
 
 
 

c) log 3 (7x 1)  3  7x 1  27  x  4

 x  8
2 2 2
d) log (x  36)  2  x  36  100  x  64   1

 x2  8

40. Calcula el valor de x en les expressions següents:

a) log x 32  5  x5  25  x  2

b) log x 0,1 1 x1 101  x 10

c) log x2 64  3 x6  26  x  2

d) log ( x2) 27  3 (x 2)3  33  x 2  3  x  5

41. Resol la inequació


࢞૛ ି૛࢞ା૝
൒࢞
࢞ି૝
x2  2x 4 2(x 2) 2(x  2)  0  x  2
x  0  
x4 x 4  x4  0  x  4
Hi ha tres intervals diferents. Per saber quin és interval solució, s’agafa un punt de cada un i es
substitueix en la inequació:
2  (10  2) 16 8
x  10  (,2)     0  És interval solució
10  4 14 7
2 (0  2) 4
x  0  (2,4)    1 0  No és interval solució
04 4
2  (10  2) 24
x  10  (4,)    4  0  És interval solució
10  4 6
A continuació, es comprova si els extrems dels intervals són solucions:
2  (2  2) 0
x  2    0  0  És solució
2  4 6
2  (4  2) 12
x4    No és solució
44 0
Per tant, la solució és (,2](4,).

125
 
 
 
 
 
 

Equacions i inequacions
Equacions i inequacions 3
 
 
 
 
 
 

ACTIVITATS FINALS 
 
42. Escriu en cada cas un polinomi que et demanem i calcula’n el valor per a x = 3 i x = -1.
a) De grau 4 i sense terme independent.
b) De grau 3 i sense termes de grau 2 ni 1.
c) De grau 2 i que la suma dels coeficients sigui 10.
d) Que sigui un binomi de grau 3 amb terme independent.
Resposta oberta, per exemple:

4
 P(3)  34  3  84
a) P(x)  x  x  
 P(1)  (1)4 1 0

3
 P(3)  4·33  2  110
b) P(x)  4x  2  
 P(1)  4·( 1)  2  2
3

2  P(3)  2  32  5 3 3  36
c) P(x)  2x  5x  3  
 P(1)  2  ( 1)  5 ( 1)  3  0
2

3
 P(3)  33 1 26
d) P(x)  x 1 
 P(1)  (1)3 1 2

43. Efectua les operacions de polinomis següents:

a) (3x2  2x  5)  (x3  5x2  2x 1)  (x4 1)  x4  2x2  3


   
b)  2x3  5 x 1   x  2   2x4  4 x3  5 x2  2 x  2
 2   3 3 2 3 3

 x3   4 1 2  28 5
2
c) (5 3x )  9     x  x  5  3x6  x5  x4  x3 11x2  20
 3  5  5 3

 x2 x3 x4  x2  x3 x4 x4 x5 x5 x6
(3x  2)  (2x  3)      x    6x2  9x  4x  6       
 2 3 4  2  2 4 3 6 4 8
d)
6
x 5 1 1
   x5  x4  x3  6x2 13x  6
8 12 12 2

126
 
 
 
 
 
 

Equacions i inequacions 33
 
 
 
 
 
 

44. Calcula el valor numèric d’aquest polinomi per als valors de x següents:

a) P(5)  6  (  5) 4  61 (  5)3 185 (  5) 2 158  (  5)  40  16830

b) P(5)  6  54  61 53 185 52 158  5 40  0

c) P(4)  6  44  61 43 185 42 158  4  40  0

d) P(4)  6  (  4)4  61 (  4)3 185 (  4)2 158  (  4)  40  1264


4 3 2
 1  1  1  1  1 693
e) P     6·    61    185    158     40 
 2  2  2  2  2 4
4 3 2
1 1 1 1 1
f) P    6     61   185   158    40  0
2 2 2 2 2
4 3 2
 2  2  2  2  2 6664
g) P     6      61    185    158     40 
 3  3  3  3  3 27
4 3 2
2 2 2 2 2
h) P    6     61   185   158    40  0
 3  3  3  3  3

45. Fes les divisions de polinomis següents i digues quin el polinomi quocient i el residu en cada
cas:

a) (x5  x4  x3  x2 1) : (x2  3)  x3  x2  4x  4 amb residu 12x11


5 9 35
b) (5x2  3x  2) : (2x  3)  x amb residu
2 4 4
c) (3x6  5x3  x  3) : (x3  2x 1)  3x3  6x  2 amb residu 12x2  3x 1

d) (x8  x6  x4  x2 1) : (x3  x 1)  x5  x2  x 1 amb residu x2  2x  2

46. Divideix els polinomis següents per mitjà de la regla de Ruffini:

127
 
 
 
 
 
 

Equacions i inequacions
Equacions i inequacions 3
 
 
 
 
 
 

a) 2 3 7 –11 0 –1
1 2 5 12 1 1
2 5 12 1 1 0

Quocient: 2x4  5x3 12x2  x 1 Residu: 0


b) 4 –1 1
–1 –4 5
4 –5 6

Quocient: 4x5 Residu: 6


c) 3 0 0 5 0 –1 3
–3 –9 27 –81 228 –684 2 055
3 –9 27 –76 228 –685 2 058

Quocient: 3x5  9x4  27x3  76x2  228x  685 Residu: 2058


d) 10 –1 0 1 0 –1 0 1
2 2 4 6 12 26 52 102 204
1 2 3 6 13 26 51 102 205
Quocient: x 2x  3x 6x 13x  26x 51x 102
7 6 5 4 3 2
Residu: 205

47. Comprova si els valors x = -1, x = 0 i x = 1 són arrels d’aquests polinomis:

 
  P(x)  P(–1) P(0) P(1) Arrels 
a)  5 4
x  x 3x  x  2x   3 2
0  0  0  x1 = –1   x2 = 1    x3 = 0 
b)  5 4 3
x  4x  6x  6x  7x 10   2
0  –10  –24  x = –1 
c)  x5 1  –2  –1  0  x = 1 
13 3 13 2 21
d)  x5  x4  x  x  3x  3     3  0  x = 1 
4 4 2
 
48. Indica quins dels polinomis següents tenen entre les seves arrels els valors x = -2 i x = 1.

 
  P(x)  P(–2) P(1) Arrels
 
a)  3
4x  3x  9x 2   2
0  0  x = –2    x = 1
b)  x5  3x4  7x3  27x2 18x   120  0  x = 1
c)  5 4 3
x  2x  22x  8x 117x  90   2
468  –43  Cap no és arrel. 
d)  x5  6x4  6x3  64x2  27x  90   140  –64  Cap no és arrel. 
 

128
 
 
 
 
 
 

Equacions i inequacions 33
 
 
 
 
 
 

49. Comprova si M(x) = 2 x3 - 5x2 + 4x - 4 és divisible entre x - 2 i, en cas afirmatiu, troba un


polinomi N(x) que permeti escriure M(x) de la forma M(x) = (x - 2) · N(x).

Dividim el polinomi entre x – 2:


2 -5 4 -4
2 4 -2 4
2 -1 2 0

El polinomi N(x) és quocient:


N(x) = 2x2 – x + 2

50. Determina les arrels dels polinomis següents: 

129
 
 
 
 
 
 

Equacions i inequacions
Equacions i inequacions 3
 
 
 
 
 
 

 
 
51. Calcula les arrels d’aquests polinomis:

a) x1  2 x2 1 x3  5
b) x1  2 x2  0 x3 
1
3

c) x1  3 x2  2 x3  2 x4  3
d) x1  3 2 1 x4 1
x2   x3 
3 2

52. Escriu un polinomi de segon grau, Q(x), que tingui les arrels 1 i 3, i tal que Q(0) = 6.

Q(x)  c(x1)(x 3)  cx2  4cx 3c


Q(0)  3c  6  c  2  Q(x)  2x2 8x 6

53. Determina el valor de m perquè el polinomi P(x) = mx3 - 6x2 - 4x + 8 tingui arrel.
P(2)  8m 24  8  8  0  8m 24  0  m 3
 
54. Calcula perquè el polinomi x3 - 2x2 + qx + 5 sigui divisible entre el polinomi x + 1.
1 –2 q 5
–1 –1 3 –3 – q
1 –3 3+q 2–q  2q 0  q 2
 
55. Quin valor ha de prendre a perquè el residu de dividir x3 + ax2 - 3x - a entre x - 4 sigui 67?
Dividim el polinomi entre x – 4:
1 a -3 -a
4 4 16+4a 52+16a
1 4+ a 13+4a 52+15a
Igualem el residu a 67:
130
 
 
 
 
 
 

Equacions i inequacions 33
 
 
 
 
 
 

52 + 15a = 67  a = 1

56. Calcula a i b perquè el polinomi x3 + ax2 + bx – 6 sigui divisible entre x - 2 i entre x + 3.


Dividim el polinomi entre x – 2:
1 a b -6
2 2 4+2a 8+4a+2b
1 2+a 4+2a+b 2+4a+2b

Dividim el polinomi entre x + 3:


1 a b -6
-3 -3 9-3a -27+9a-3b
1 -3+a 9-3a+b -33+9a-3b

Resolem el sistema:
૛ ൅ ૝ࢇ ൅ ૛࢈ ൌ ૙
ቅ՜ࢇ=2 ࢈ = -5
െ૜૜ ൅ ૢࢇ െ ૜࢈ ൌ ૙

57. Escriu dos polinomis de segon grau que tinguin arrels 2 i -3.
Resposta oberta, per exemple:

P(x)  x2  x6                 Q(x)  2x2  2x12  


 
58. Escriu un polinomi de tercer grau que tingui com a única arrel -1
Resposta oberta, per exemple:

P(x)  x3  x2  x1  
 
59. Determina un polinomi P(x) de segon grau que tingui arrels 1 i -2, i tal que P(3) = 30.

P(x)  c(x1)(x 2)  cx2  cx 2c 


P(3)  9c 3c 2c  30  c  3  P(x)  3x2  3x 6  
 
60. Escriu un polinomi Q(x) de tercer grau que tingui arrels 1, -1 i -2, i tal que Q(0) = -6.

Q(x)  c(x1)(x1)(x 2)  cx3  2cx2  cx 2c  


Q(0)  2c  6  c  3  Q(x)  3x3  6x2  3x 6  
   

131
 
 
 
 
 
 

Equacions i inequacions
Equacions i inequacions 3
 
 
 
 
 
 

61. Efectua les sumes i les restes següents de fraccions algèbriques:

y2 x3 3 xy4 x5 3y2 x5  xy4  3y2


a)  2 2 2 2 2 2 2 2
x y x x y x y x y x2 y2
5 3x  2 5x  5 3x2  2x 3x2  3x  5
b)    
x x 1 x(x 1) x(x 1) x2  x
3x2 2 5x 3x2 2x  4 5x2 10x 8x2 8x 4
c)      2 
x 2  4 x  2 x  2 x2  4 x 2  4 x 4 x2  4
1 x x 1 2 x2  2x 1 x2  2x 1 2 4x  2
d)   2    2  2
x1 x 1 x 1 2
x 1 2
x 1 x 1 x 1

62. Fes aquestes operacions i simplifica el resultat:

 
63. Comprova si el nombre que hi ha indicat en cada apartat és solució de l’equació.

a) No, les solucions són x1 = 2 i x2 = 3.


132
 
 
 
 
 
 

Equacions i inequacions 33
 
 
 
 
 
 

b) Si, les solucions són x1 = �√3 i x2 = √3.



c) Si, la solució és x = �

d) No aquesta equació no té solució real.

64. Resol aquestes equacions de segon grau:

a) 3x2 – 48  0  x2 16  x1  4 x2  4

b) 3x2 – 48x  0  x(3x 48)  0  x1  0 x2 16

c) 3x2  48  0  x2  16  No té solució real


2 3 32  41 9 3 27
d) x  3x 9  0  x   x  No té solució real
2 1 2
3 (3) 2  4 1 9 3 27
e) x2  3x 9  0  x   x  No té solució real
2 1 2

2 18  182  4  (3)  (3) 18  288


–3x 18x – 3  0  x   x 
f) 2  (3) 6
 x1  3 2 2 x2  3 2 2

2 18  (18)2  4 (3)  3 18  360


–3x 18x  3  0  x   x 
g) 2 (3) 6
 x1  3 10 x2  3 10
 x1  3 10 x2  3 10

1 12  4 1 (18) 1 73


x2  x – 18  0  x   x 
h)
2 1 2
1 73 1 73
 x1  x2 
2 2

133
 
 
 
 
 
 

Equacions i inequacions
Equacions i inequacions 3
 
 
 
 
 
 

65. Resol aquestes equacions de segon grau amb denominadors:

3x2 1 x2  x 2 3
a)   x  0  5x2  2x  3  0  x1   x 1
2 3 5 2
x2 3x2  2x 3 19x
b) 1   x2  2x1 0  x  1
2 3 6
x(x1) 10 x2  2x 8
c)   2  3x2 8x  0  x1  0 x2  
5 2 3
 x  3
11x  5 2x2 1 
d) x  2
 2
 x  2x  5x  3  0   1
6 3 x 
 2
66. La suma de les solucions d’una equació de segon grau és 4 i el producte és – 21.
a) Escriu l’equació corresponent.
b) Determina aquestes solucions.
a) x(4 – x) = – 21
������ ���������� � � ��
b) – x2 + 4x + 21 = 0 → x = →� �
������ �� � �
Les solucions són – 3 i 7.

67. Indica el nombre de solucions de cadascuna de les equacions següents:

a)  b2  4ac  (4)2  435  44  0  No té solucions reals


b)  b2  4ac  32  4 (2)12 105  0  Té dues solucions
c)  b2  4ac  (1)2  41(3) 13  0  Té dues solucions

d)  b2  4ac  62  4(1)9  72  0  Té dues solucions

68. Resol l’equació general de segon grau ax2 + bx + c = 0 utilitzant les igualtats notables.
Relaciona el resultat que obtinguis amb el nombre de solucions que té l’equació de segon
grau.

�� �
� x2 + �x + c = 0 → x2 + � ��
� �

134
 
 
 
 
 
 

Equacions i inequacions 33
 
 
 
 
 
 

�� � �� �� �� �
x2 + � � → x2 + � � �
� � � ��� ��� �

�� �� � � �� �
�� � �� � �
- → x+ = �� �
� �� � �� ��� �

�� �� ���� ���√�� ����


x= � � → x=
�� ��� ��

Si √b � � ��� < 0 → No té solució


Si √b � � ��� = 0 → Té una solució
Si √b � � ��� > 0 → Té dues solucions
Determina el valor de k perquè aquesta equació tingui solució x = 7.
x2 – 13x + k = 0
Per a aquest valor de k, quina és l’altra solució?

72 137  k  0  k  42  

13 (13)2  4 1 42 131


x2 13x  42  0  x    x1  6 x2  7
2 1 2  

 
69. Quins són els valors que han de prendre a i b perquè l’equació ax2 + bx – 30 = 0 tingui les
solucions, x1 = 5 i x2 = – 3?
Substituïm les dues solucions dins l’equació i formem un sistema on les incògnites son a i b:
25a + 5b – 30 = 0
��� � �� � �� � � � � � → ��� � ��� � ��
� → � � → ���� � �� � ��� → � � �
�� � �� � �� � � �� � � → ��� � ��� � ���

→ 9· 2 – 3b -30 = 0 → b=-4

70. Sense resoldre-les, digues quina és la suma i quin és el producte de les arrels de les
equacions següents, i després calcula-les per comprovar-ho:

Partim d’una equació de segon grau de solucions a i b: (x – a)(x – b) = 0


Després, multipliquem: x2 – ax – bx + ab = 0 → x2 – (a + b)x + ab = 0
Per tant, el producte de les arrels és el terme independent i la suma de les arrels és l’oposat al
coeficient del terme de primer grau.
a) El producte de les arrels és -14 i la suma és -5.
Les arrels són x1 = -7 i x2 = 2.
b) El producte de les arrels és 0 i la suma és -1.
Les arrels són x1 = -1 i x2 = 0.

135
 
 
 
 
 
 

Equacions i inequacions
Equacions i inequacions 3
 
 
 
 
 
 
� ��
c) El producte de les arrels és � i la suma és � .
� �
� �
Les arrels són x1 = � i x2 = .
� �
��
d) El producte de les arrels és � i la suma és �1

� �
Les arrels són x1 = � i x2 = .
� �

e) El producte de les arrels és i la suma és 1


L’arrel és x = .

1 3
 
f) El producte de les arrels és 10 i la suma és 10 .

1 x 1
x1   2
Les arrels són 2 i 5.

71. Escriu equacions de segon grau que tinguin les solucions següents:

a) (x 2)(x5)  x2  3x10 c) x(x 2)  x2  2x


 1  2 
b) (x 4)(x 4)  x2 16 d) 9  x    x    9x2  9x  2
 3  3

72. Resol les equacions biquadrades que tens a continuació:


a) 25x4 – 101x2 + 4 = 0 b) x4 – 25x2 + 144 = 0 c) 3x4 – 30x2 + 27 = 0

a) 25x4 –101x2  4  0  25z2 –101z 4  0


 1
101 (101) 2  4 25 4 101 99  z1 
z  z  25
2  25 50 z  4
 2

z1 
1
 x1  
1
x2 
1 z2  4  x3  2 x4  2
25 5 5

b) x4 – 25x2 144  0  z2 – 25z144  0


25 (25) 2  4 1144 25 7  z  9
z  z  1
2 1 2  z2  16

z1  9  x1  3 x2  3 z2 16  x3  4 x4  4

136
 
 
 
 
 
 

Equacions i inequacions 33
 
 
 
 
 
 

c) 3x4 – 30x2  27  0  3z2 – 30z 27  0


30  (30) 2  4  3 27 30  24  z  1
z  z  1
23 6  z2  9

z1 1  x1  1 x2 1 z2  9  x3  3 x4  3

73. Determina les solucions de les equacions següents:

x3  x 1
a)   0  4x(x3  x)  (x2 1)  0  4x4  5x2 1 És una equació biquadrada:
x 1 4x
2

 1
z  1 x 
 1  x  1 1  3 2
2 2
z  x  4z  5z1   1 z1  1  1 z2   
 z2   x2  1 4  1
 4  x4   2

b) 9(1  x 2 )(1  x 2 )  80 x 2  0   9 x 4  80 x 2  9  0  És una equació biquadrada:

 1
z  9 x 
 1  x  3 1  3 3
2 2
z  x  9z  80z 9   1 z1  9   1 z2    
 z2    x2  3 9  1
 9  x4   3

18 18
c) 1   0  x4 18x2  81  0  És una equació biquadrada:
x 2 x4

 x  3
z  x2  z2 18z 81 z  9   1
 x2  3

74. Resol les equacions biquadrades que hi ha a continuació:

a) x8  3x4 – 4  0  z2  3z– 4  0
3 32  4 1 (4) 3 5  z  4
z  z  1
2 1 2  z2  1

z1  4  x4  4  No té solució real z2  1  x4 1  x1  1 x2 1

137
 
 
 
 
 
 

Equacions i inequacions
Equacions i inequacions 3
 
 
 
 
 
 

b) x6 – 19x3 – 216  0  z2 19z– 216  0


 19  35
2
19  19  4 1 (216) 19  1225  z1  2  8

z  z 
2 1 2  z  19  35  27

 2 2

z1  8  x3  8  x  2 z2  27  x3  27  x  3

c) x12  7x6 12  0  z2  7z12  0


7  72  4112 7 1  z  4
z  z  1
2 1 2  z2  3

z1  4  x6  4  No té solució real
z2  3  x6  3  No té solució real

d) 36x10  x5 – 6  0  36z2  z– 6  0
 1 865
 z1   0,42
1 12  4  36  (6) 1 865  72
z  z 
2  36 72  1 865
 z2   0,39
72

1 865 1 865 1 865


z1   x5   x1  5  0,84
72 72 72

1 865 1 865 1 865


z2   x5   x2  5  0,83
72 72 72
75. Busca les solucions de les equacions amb fraccions algèbriques següents i comprova’n, com
a mínim, una de les solucions:

138
 
 
 
 
 
 

Equacions i inequacions 33
 
 
 
 
 
 

 
 
76. Resol les equacions amb fraccions algèbriques que hi ha a continuació:

x2 1 7x2  7  6x 6x4  6 6x4  6x2 7x3  6x2  7x


a) x    
x 6x2  6 6x(x2 1) 6x(x2 1) 6x(x2 1)

1 22 1 22
 7x3 12x2  7x  6  0  x1  2 x2  x3 
7 7
x 4 x 3
b)  2  (x 4)(x2  x  6)  (4x 7)(x  3) 
4x 7 x  x  6

139
 
 
 
 
 
 

Equacions i inequacions
Equacions i inequacions 3
 
 
 
 
 
 

 x  3
 x3  x2  5x  3  0   1
 x2  1
L’única solució vàlida és x  1 , ja que x  3 es descarta perquè fa 0 el segon denominador.
x1 7 x 2x2  3x1 7 2x2  x
c)  2     
2x 1 4x 1 2x 1 4x2 1 4x2 1 4x2 1
3
 4x  6  0  x 
2

4 x 4 x2 1 x2  x
d) 1     3 x  0  x  3
x2 1 x 1 x2 1 x2 1 x2 1
1 1 1 x 3 x 3 1
e)   2  2  2  2  7  0  No té solució real.
x 3 x 3 x  9 x  9 x  9 x  9

77. Determina les solucions d'aquestes equacions amb fraccions algèbriques:

���� � ����
a) � � → � � � ��� � � �� � � → �� � � � � → � � � ��� � �� � �
�� � ���

�������������� ������� ���� �� � �


z = � →�
��� � �� � �
�� � � → � � � �� �� � �
�� � � → � � � �� �� � �
� � ��
b) �� � �� � � �� � → �� � � ��� � � � � � → �� � � � � → �� � � ��� � � � �

�������������� ������ �����


�� � �
z = � →� �
��� �� �� �

� �
�� � � → � � � � �� �
� �
� � �
�� � → � � � � �� �
� � �
�� ��
c) �� � � � � → �� � � �� � � � � � → �� � � � � → �� � � �� � � � �
� �

������ ��������� ����


�� � �
z = � →� �
��� � �� � �

�� � � → � � � �� �� � �

�� � � → No té solució real.

140
 
 
 
 
 
 

Equacions i inequacions 33
 
 
 
 
 
 


� � � �� � �� � �� � � � � � �� � � � � ��� � �� � � � �
� � � �
d)
���
������������ ������ ��√����
z = �
��� ��
No té solució real.

78. Resol les equacions amb fraccions algèbriques següents:

4 1 1
a)   8x  4  x  3  7x  1 x  
x  3 2x 1 7
 9  73
 x1 
x  2 3x x 2 3x  2
b)  0   x2  9x  2  0  
2  x 2x 1 2  x 1 2x  9  73
 x2  2

3 2 3 2
c)  0   9x 15  2x  6  x  3
x  3 3x  5 x  3 3x  5
x x3 3
d)   x2  2x  x2  4x  3  2x  4x  3  x 
x 1 x  2 2

79. Troba les solucions de les equacions amb fraccions algèbriques següents:

x 1 2 1 2(x 1)(x 1) 4x2  x(x 1)


    
a) x x 1 2x 2x2 (x 1) 2x2 (x 1)
 2x2  2  4x2  x2 1 x2  1 No existeix solució real.
x  2
12  1
b) x  3x  4   0  x  3x  4x 12  0   x2  2
2 3 2

x 
 x3  3
 x  2  10
c) x  x  6  3  0  x2  x  6  3x  0  x2  4x  6  0   1
2

x 
 x2  2  10

2x x  x  0
d)   0  2x(x 1)  (3x  4)x  x2  2x  0  x(x  2)  0   1
3x  4 x 1  x2  2

141
 
 
 
 
 
 

Equacions i inequacions
Equacions i inequacions 3
 
 
 
 
 
 

80. Calcula el valor de x en les equacions següents:

5 x 3
a) 1   0  1 x2  5(1 x)  x(1 x)  0  6  4x  x 
1 x 1 x 2

10x 1 4x2  3x  4
  3  (10x 1)(x 1)  (4x2  3x  4)  6(x 1) 2 
b) 2(x 1) 2(x 1) 2

1
10x2  x 10x 1 4x2  3x  4  6x2 12x  6  4x 1 0  x  
4
2x 1 x 2x  3
   1 (2x 1)(x 1)x  x2 (x  4)  (2x  3)(x  4)(x 1)  x(x  4)(x 1) 
x  4 x 1 x
c)  x  2
2  1
 5x  4x 12  0   6
 x2 
 5

d) 1 x2 1 x2
1  2  1   (x  4)  (x2  3x  4)  x2  4x  0  x  0  
x 1 x  3x  4 x 1 (x  4)(x 1)

x2  5x  2 x  2 2x  5 3x
   1 
e) 2x  5 x 1 x 1 4

 4( x 2  5 x  2)( x 2  1)  4( x  2)(2 x  5)( x  1)  4(2 x  5) 2 ( x  1)  4(2 x  5)( x 2  1)  3 x (2 x  5)( x 2  1) 

 x1  3
 x  3,8186
 2
 10 x 4  3x3  130 x 2  123x  72 0  
 x3  0, 41092
 x4  1,5295

L’única solució que es pot obtenir amb els mètodes que hem vist aquest curs és x = 3.
x2  4 x 1 1
  2x   0 
f) x  x 1 x  2
2
7

 7(x2  4)(x  2)  7(x 1)(x2  x 1) 14x(x2  x 1)(x  2)  (x2  x 1)(x  2)  0 

14x4  43x3 13x2 15x  47  0  No té solució real.

142
 
 
 
 
 
 

Equacions i inequacions 33
 
 
 
 
 
 

x x 2x x x 2x
 3  2 0   2 0
2
g) x 1 x 1 x  x 1 (x 1)(x 1) (x 1)(x  x 1) x  x 1
2

x  0
2 3 2 2  1
x(x  x 1)  x(x 1)  2x(x 1)(x 1)  0  3x  2x  0  x (3x  2)  0   2
 x2  
 3
81. Completa les equacions següents escrivint un nombre en el segon membre, de manera que
tinguin la solució indicada:

a) √� � � � �√�� � � � ��
� � �� �
b) � � �
√� √��� �� √��

82. Determina la solució d’aquestes equacions amb radicals:

a) 6x 2  4  6x 2 16  6x18  0  x  3


b) 6x  8  x  6x  8  x2  x2  6x  8  0  x1  2 x2  4

c) x 2  9  1  x  x 2  9  x2  2 x  1  2 x  8  0  x  4

5 33 5 33
d) 2x2  7x1  x1 2x2 7x1 x2  2x1 x2 5x 2  0  x1  x2 
2 2

83. Resol les equacions amb radicals següents:

a)
3
x 9  4  x9  64  x55  0  x  55
3
b) x2  7x  2  x2  7x  8  x2  7x  8  0  x1  1 x2  8
 
84. Resol aquestes equacions amb radicals:

a) 240
2x 10  5 x 10  2x 10  25(x 10)  23x  240  0  x 
23

143
 
 
 
 
 
 

Equacions i inequacions
Equacions i inequacions 3
 
 
 
 
 
 

b) 94 94
x  2  x  3  5  x  2  x 10 x  3  28  x  0 x
25 25
En comprovar el resultat s’observa que no és solució.

c) 4x 11  7 2x  29  4x 11 49(2x  29)  94x 1410  0  x  15


d) 13 13
x  3  x 1  3  x  3  x  6 x 1 10  x  0 x
36 36
En comprovar el resultat s’observa que no és solució.

85. Resol les equacions amb fraccions algèbriques següents:

2
 x2 
x2
 
2
a) 3x2  4x  5  3x  5   3x2  4x  5    3x  5 
5 5 
 x  3,3205
 1
x 4
6x 3
x 6x 4
 x  24,456
3
 3x2  4x  5    7x2  30x  25     4x2  34x  20  0   2
25 5 25 5  x3  0,64731

 x4  9,5122
Després de la comprovació, les úniques solucions vàlides són x1 i x2.
Cap d’aquestes dues solucions es pot obtenir amb els mètodes que hem vist en aquest curs.

   5 
2 2
b) x2  4x  4  x  3  5  x2  4x  4 x3 

  
2


2
 x2  4x  4  x  28 10 x  3  x2  3x  24  10 x  3 

 x  1
 x4  6x3  39x2  244x 276  0   1
 x2  6
Després de la comprovació, s’observa que l’única solució vàlida és x1 = 1.
2
 5
5
 
2
c) x  x   4  3x  8   x2  x    4  3x  8
2

9  9

2
 211 
5
 
2
 x  x   16  3x  8  8 3x  8   x2  2x 
2
  8 3x  8 
9  9 
 1
386 2 884 3049 x 
 x4  4x3  x  x  0 1 3
9 9 81  x  9,535
 2
1
Després de la comprovació, s’observa que l’única solució vàlida és x1  .
3

144
 
 
 
 
 
 

Equacions i inequacions 33
 
 
 
 
 
 
2
 3
3
 
2
d) 5  8x  x2  6x   10x  5  8x   10x  x2  6x   
4  4

2
3  17 
5 8x  101x  6x  10x 4x2  24x  3  101x2  2x    100x2 (4x2  24x  3) 
2

4  4

 1
4 2309 x 2
3 289  x1  
 9801x  2804 x   17 x  0 2
2 16  x2  0,1221

1
L’única solució que es pot obtenir amb els mètodes vistos en el curs és x   .
2

86. Resol les equacions amb radicals següents:

a) x7  x1 2 x 2  0  2x 2 x7 x16  4x8 


 x 7 x1  x1 x2  6x 7  x2  2x1 4x8  0  x  2
b) x 1 x  x 1 x  1 x  x  2 x 1

 2 x 1 1 x  4x  x 2 1 x  2  5x 2  2 1 x 


x  0
2  1
2
 25x  20x  4  4  4x  25x 16x  0   16
 x2 
 25
16
En comprovar el resultat veiem que l’única solució vàlida és x2  .
25
87. Calcula la solució de les equacions següents:

a) (x2 – 4)(x2 – 3x 2)  0  (x1)(x 2)(x 2)2  0 


 x1  2 x2 1 x3  2
b) (x2 – x)(x2 16)  0  x(x1)(x2 16)  0 
 x1  0 x2 1

c) (x1)(x2  4)(x2 – 9)  0  (x1)(x 3)(x 3)(x2  4)  0 


 x1  3 x2 1 x3  3

145
 
 
 
 
 
 

Equacions i inequacions
Equacions i inequacions 3
 
 
 
 
 
 

d) (x2 – 4x – 5)(x2 – 2x – 8)  0  (x1)(x 2)(x5)(x 4)  0 


 x1  2 x2  1 x3  4 x4  5

88. Determina la solució de les equacions que tens a continuació:

146
 
 
 
 
 
 

Equacions i inequacions 33
 
 
 
 
 
 

 
89. Troba la solució d’aquestes equacions:

Resolem l’equació 6x2 – x – 2 = 0

Resolem l’equació 4x2 – 8x – 5 = 0

147
 
 
 
 
 
 

Equacions i inequacions
Equacions i inequacions 3
 
 
 
 
 
 

Resolem l’equació x2 + 5x – 1 = 0:

d) x2 (x2  x 6)  3(x2  3x)  x2 (x 3)(x 2)  3x(x 3)  0  x(x 3) x(x 2)  3 

 x(x 3)(x2  2x 3)  0  x(x 3)(x 3)(x1)  0  x1  0 x2  3 x3  3 x4 1

90. Calcula la solució de les equacions que tens a continuació:

a) x3 – 6x2 11x – 6  0
1 –6 11 –6
1 1 –5 6
1 –5 6 0
2 2 –6
1 –3 0
3 3
1 0

Les arrels enteres són {1, 2, 3}.


3 2
b) x – 3x – 13x 15  0
1 –3 –13 15
1 1 –2 –15
1 –2 –15 0
5 5 15
1 3 0
–3 –3
1 0

Les arrels enteres són {3, 1, 5}.


5 4 3 2
c) x  x – 5x – 5x  4x  4  0
1 1 –5 –5 4 4
1 1 2 –3 –8 –4
1 2 –3 –8 –4 0
–1 –1 –1 4 4
1 1 –4 –4 0
2 2 6 4
1 3 2 0
148
 
 
 
 
 
 

Equacions i inequacions 33
 
 
 
 
 
 

–2 –2 –2
1 1 0
–1 –1
1 0

Les arrels enteres són {–2, –1, 1, 2}.


3 2
d) x  7x  4x  28  0
1 –7 4 –28
7 7 0 28
1 0 4 0

x2 + 4 = 0 no té solucions reals, per tant, l’única arrel entera és x = 7.

91. Resol les equacions següents:

149
 
 
 
 
 
 

Equacions i inequacions
Equacions i inequacions 3
 
 
 
 
 
 

 
Resolem l’equació x + 3x + 4 = 0:
2

���√�� ������ ���√��


x = � → No té solució real.
��� �

Resolem l’equació -5x2 - 2x - 1 = 0:

������������ ������������ ��√���


x = � → No té solució real.
������ ���
x=2
�� � ���� �
e) � � → �� � � �� � � �� � � � �
� � ��

Resolem l’equació 2x2 - x + 1 = 0:

������������ ������ ��√��


x = � → No té solució real.
��� �
x=3

150
 
 
 
 
 
 

Equacions i inequacions 33
 
 
 
 
 
 

92. Escriu alguna equació que tingui les característiques següents:

a) Els valors x = 1, x = 2 i x = 3 són solucions de l’equació.



b) Una equació de grau 3 amb una única solució real, que és x = .

c) Una equació de grau 3 amb dues solucions reals que siguin l’una oposada de l’altra.
� �
d) Una equació de grau 2 amb coeficients enters i amb solucions x = � i x = .
� �

√�
e) Una equació de grau 3 amb coeficients enters i els valors x = i x = -3 son dues de les

seves solucions.

Resposta oberta, per exemple:


a) 7(x 1)(x  2)(x  3)  0  7x3  42x2  77x  42  0
3
 1 3 3 2 3 1 3 2
b)  x    0  x  x  x   0  8x 12x  6x 1 0
 2 2 4 8
c) Si dues de les solucions són reals, aleshores la tercera solució també serà real.
(x 1)(x 1)x  0  x3  x  0

  
d)  x  1  x  2   0  x2  x  2  0  15x2  x  2  0
 3  5 15 15

e) Agafant com a tercera solució x3   2 :


2
  
 x  2   x  2  (x  3)  0  2x3  6x2  x  3  0
 2   2 

93. Determina el valor de x en les expressions següents:

 3
1  x1 
1  3
a) log x 3  1 x  3  x 
1
c) log x 3  2  x  3  x   
2 2
3 3  3
 x 
 2 3

L’única solució vàlida és x1  3.


3
x  5
b) log x 5  2  x2  5   1 d) log x 2  5  x5  2  x  5 2

 x2   5

L’única solució vàlida és x1  5 .

151
 
 
 
 
 
 

Equacions i inequacions
Equacions i inequacions 3
 
 
 
 
 
 

94. Determina el valor de x en les expressions següents:

a) log x 8  4  x4  8  x  4 8
1 1
b) log x  4  x4   x  4 4  2
4 4

c) log x 3  5  x5  3  x  5 3
 3
x 
4 4 2 9  1 2
d) log x  2  x   x   
2

9 9 4  3
 x2   2

3
L’única solució vàlida és x1  .
2

e) log x 343  3 x3  343 x  7

f) log x 2  4  x4  2  x  4 2
 
g) log x   125   3  x3   125  x   2
 8  8 5

h) log x 49  6  x6  49  x  6 49  3 7

95. Determina el valor de x en les expressions que tens a continuació:

a) log3 9 x  2  32  9 x  x 1

b) log 2 x  3  xlog 2  3  x  3  4,983


2 2 2log 2

1
c) ln 3  1 xln 3  1 x  
x
 0,91
ln 3

d) log 2 4 x4  2  22  4 x4  41  4 x4  1 x 4  x  5


3
e) log 3 9 x3  3  33  9 x3  33  32 x6  3  2x  6  x  
2
152
 
 
 
 
 
 

Equacions i inequacions 33
 
 
 
 
 
 
x
f) log 2 2  3  x log 2  3  x  3  9,966
2 2 2 log 2

3
g) ln 3  3  (x  6)ln 3  3  x   6  3,269
x6

ln 3

3x4 14
h) log 3 27  2  (3x  4)log 3 27  2  3·(3x  4)  2  x  
9

96. Resol les equacions logarítmiques següents:

x x 1 1
a) log x  1 log5  log  1   x 
5 5 10 2
x x
b) log 5 x  2  log 5 7  log 5  2   25  x  175
7 7
x x
c) log 2 x  3 log 2 9  log 2  3   8  x  72
9 9

97. Resol les equacions logarítmiques que hi ha a continuació:

1
a) log 4 x  2  log 4  log 4 2x  2  2x  16  x  8
2
3x 1 3x 1 1 1
b) log (3x 1)  2  log50  log  2    x
50 50 100 2
3x 1 1 16(3x 1) 2
c) log 2  2  log 2  log 2  2  12x  4  4  x 
4 16 4 3

98. Calcula el valor de x en les equacions següents:

 x  6
1
  2
a) log x 9  log x 16  2  log x 9 16  2  x  36  
2
1

 x2  6

153
 
 
 
 
 
 

Equacions i inequacions
Equacions i inequacions 3
 
 
 
 
 
 

L’única solució vàlida és x1  6 .

 x  0
2 2
b) log x1 (6x 1)  2  (x1)  6x1 x  4x  0   1

 x2  4
L’única solució vàlida és x2  4 .

2 2 2  x1  1
c) log 1 (x  3x  3)  0 1 x  3x  3  x  3x  2  0  
2  x2  2

d) log 2 x3  log 2 x2  4  log 2 x  4  x 16

2 2 2  x1  1
e) log 2 (x  4x 1)  2  4  x  4x 1 x  4x  5  0  
 x2  5

99. Resol aquestes equacions:

a) log(x 3)  log(x1) 1 log(x 5)  log (x 3)(x1)(x5) 1


 x  0,47419
3 2
 1
 (x  3)(x 1)(x  5)  10  x  7x  7x  5  0   x2  1,8873

 x3  5,5869

L’única solució vàlida és x3  5,5869

 x  2,7614·106
b) log 3 (x2  x 1)  27  x2  x 1 327   1
 x2  2,7614·106

         
c) log 5  x  1   log 5  x  3   1 log 5  x  1    x  1   x  3  x  1   5 
 2  4  3  2  4  3

19 2 19 39
x3  x  x   0  x  1,1906
12 24 8
         
d) log  x  2   log  x  3   log  2x 1   1   x  2  x  3  2x 1   10 
 2   2   3   2  2  3 

1 11 17 21
 x3  x2  x   0  x  5,8775
6 12 12 2

154
 
 
 
 
 
 

Equacions i inequacions 33
 
 
 
 
 
 

100. Determina el valor de x en les equacions següents:

1
a) log 3 x  5  log 3 2x  3  1 log 3 (x  5)(2x  3)  1
2
x  6
 1
(x  5)(2x  3)  9  2x2 13x  6  0   1
 x2 
 2
L’única solució vàlida és x1 = 6.
6
2 x
2 1 2  2
b) log 2 x  log 2 3
x   2  x  2  x   2 3   16
3 6 3
3 3
x  

101. Resol les equacions logarítmiques que tens a continuació:

 3 13
 x1   3,303
 2
a) log 5 (x 1)  log 5 (x 1)  log5 3x  x 1  3x  x  3x 1  0  
2 2

 3 13
 x   0,303
 2 2

L’única solució vàlida és x1  3 13  3,303


2
( x  3)( x  2)
b) log 2 ( x  3)  log 2 (2 x  21) 1  log 2 ( x  2)  2 
2 x  21
 x  3
x2  9x  36  0   1
 x2  12
L’única solució vàlida és x2 = 12.

102. Calcula el valor de x en les equacions exponencials següents:

1
a) 4   4 x  43  x  3
x
e) 162 x4  1 24(2 x4)  20  8x  16  x  2
64
b) 2 x5  32  x  5  5  x  10 f) 2 x1  8  x 1  3  x  2

c) 36x  27 x2  6  x  3x  6  x  3 g) 2 x1  16  x 1  4  x  3


11
d) 32  2  5x 10  1 x 
x2
h) 2 x1  128  x 1  7  x  6
5

155
 
 
 
 
 
 

Equacions i inequacions
Equacions i inequacions 3
 
 
 
 
 
 

103. Resol les equacions exponencials que tens a continuació:

11
a) 642 x5  16 x2  3(2x  5)  2(x  2)  6x 15  2x  4  x 
4
b) 125x3  25x3  No existeix solució real.

c) 5x3  1 x  3  0  x  3
1
d) 2   x 1  3  x  4
x1

8
1
e) 2 
x1
 x 1  4  x  5
16
1
f) 2 
x1
 x 1  7  x  8
128

104. Resol les equacions exponencials següents:

a) 3 27 x2  9 x  33x61  32 x  3x  5  2x  x  5

b) 5x4  125x4  x  4  3x 12  x  8


1 28
c)  34x  34( x6)  34x  4x  24  4  x  x 
816x
5
d) 322 x3  2 x3  10x 15  x  3  x  2

e) 125x2  52 x  3x  6  2x  x  6

f) 256 x  4·42 x3  44  42 x2  6  2x  x  3

105. Opera amb aquestes potències i calcula el valor de x en les equacions exponencials següents:
3 2
x3 
x2 4 x 3
  2 x  x 5 x
a) 3 10 5 10
=1 b) =1
8
x2 4 x 3
a) 3
x3    1 3
10 5 10
 1 10x3  x2  8x  3  0  x1  1 x2  x3 
2 5

 x  1
3 2
2 x x 5x x3 x2 5x0
b)  1 23  2  x3  x2  5x  3  0   1
8  x2  3

156
 
 
 
 
 
 

Equacions i inequacions 33
 
 
 
 
 
 

106. Resol les equacions exponencials que tens a continuació:


5 x 3  34 x 2  48 x 1 5  x3
x3  (2 x 2 16 x )
3 5 3 7 3 1 2
a)   =1 b)      c)    713 x 13 x
1
2 7 3 7
34x2 48x 5x3 34 x2 48x
x3   0
 3 5 5  3 5  3 6
a)   1       5x3  34x2  48x  0  x1  0 x2   x 8
2 2 2 5 3
1
x3  (2 x2 16 x)
   
b)  3    7  1 2 16 8
3
 x3  (2x2 16x)  x3  x2  x  0  x1  0 x2  x3  2
7  3 3 3 3 3

5x3
1 13x2 13x x3 5
13x2 13x
1
c)    7  1 7 7 2
 70  2x3 13x2 13x 10  0  x1  5 x2  x3  2
7 2

107. Calcula el valor de la incògnita x si suposem que la resta de lletres que apareixen són
constants.

3
a) a2x1  a2  2x 1 2  x 
2
b) mx3  (m2 ) 2 x  x  3  4x  x  1

7
c) (3a)2 x5  9a2  (3a) 2 x5  (3a)2  2x  5  2  x 
2
5x 2 5x 2 2
d) ( p 3)  p  6 p 9  ( p 3)  ( p 3)  5x  2  x 
5
e) (a2  2ab b2 ) 2  (a  b)2 x  (a  b) 4  (a  b) 2 x  4  2x  x  2

x  3  0  x  3
1

f) (9  2x  x2 ) x3  1  
 x2  4
2
9  2x  x  1 
  x3  2

 x 1 0  x1  1
g) (x 1)  1 
x1

 x 1 1 x2  0

157
 
 
 
 
 
 

Equacions i inequacions
Equacions i inequacions 3
 
 
 
 
 
 

108. Resol aquestes equacions exponencials per mitjà d’un canvi de variable:
a) 92 x  3  9 x  2 0
2x x
b)  2  2
 2  
35
 3  3

c) 74 x  3  73 x  5  72 x  13  7 x  6 0
4x x
d)  5  5
 2  1 0
6  6

t  1 9 x  1 x  0
2x x
2 1
a) 9  3 9  2  0   t  3t  2  0  
9 t
x
log 2
t2  2  9  2  x   0,3155
x

 log 9
  
x
t  5   2   5  Impossible
2
2x
2
x  2 x 1  3
  t 
b)    2    35  t  2t  35  0  
 3 2
x
 
 3  3 t  7   2   7  x  log 7  4,8
2  3 2
 log
3
La solució obtinguda es descarta, ja que no és vàlida en substituir-la a l’equació. Per tant, aquesta
equació no té solució.
c) 74 x  3 73x  5 72 x 13 7 x  6  0 7
x
t
  t 4  3t 3  5t 2 13t  6  0 
t  2  7 x  2  Impossible
1
 log3
t2  3  7  3  x   0,5646
x

 log7

t3  1 2  7  1 2  Impossible
x

t  1 2  7 x  1 2  Impossible
4
4x 5 x  x
5 5   t
d)    2   1  0 
6
 t 4  2t 1  0 
6 6
  
x
t  1  5   1 x  0
1 6

  
x

t2  0,54369   5   0,54369  x  log 0,54369  3,3423


 6 5
 log  
 6

109. Resol les equacions exponencials següents mitjançant un canvi de variable:

158
 
 
 
 
 
 

Equacions i inequacions 33
 
 
 
 
 
 

3x 2 x1 x 5 11
a) 3  5 3 11 3 1  0 3
x1 t
  t 3  t 2  t 1  0 
3 3

t1  1 3x  1 x1  0

 t2  3  3x  3  Impossible

t  1  3x  1  x  1
3 3 3 2

3x
1 x 4 9 2
b) 4    9  2  2  4  (2 )  9  (2 )  2  (2 )  0 2  3  2  0
2 x  x1 x 3 x 2 x 1 t

2 t t t
t  4  2 x  4  x  2
2 1 1
2t  9t  4  0  2   x
1 1
t2   2 x   x2  1
 2 2
3x1 2x x1  3 x
 3  3  3 3
  t
3
c)   1        t 3  t 2  t 1 0 
 2

2 2 2 2 2

  
x
t  1  3   1 x  0
1 2
1

 x
  3
 t2  1    1 Impossible
 2
 x
t  2   3   2  x  1
 3 3 2 3 2

110. Indica si x = 2 es troba entre les solucions de les inequacions següents:


a) 3(x – 2) > 1 – 3(2x – 3) b) 5(4 – 3x) + 3 ൑ 1 + 3x c) 2(x – 3) – 4(3 – 2x) > 0
a) 3(x  2)  1 3(2x  3)  3 (2  2)  1 3 (2 2  3)  0  2

x = 2 és solució de la inequació.
b) 5(4  3x)  3  1 3x  5(4  3 2)  3  1 3 2  7  7

x = 2 és solució de la inequació.
c) 2(x  3)  4(3 2x)  0  2(2  3)  4(3 2 2)  0  2  0

x = 2 és solució de la inequació.

111. Determina la solució de les inequacions següents:

a) 2x – 30  5x  3  –33  3x  x  11, x  [11,)

b) 2x – 6  5x18  –24  3x  x  8, x  (8,)

c) 11– 3x  23  –12  3x  x  4, x  [4,)

d) 3x 2  x10  2x  8  x  4, x  [4,)

159
 
 
 
 
 
 

Equacions i inequacions
Equacions i inequacions 3
 
 
 
 
 
 

3  3
e) 6  5x  2x 3  3  7x  x  , x   , 
7  7

1 1 
f) 14x  5  x  5  15x  x  , x   , 
3 3 

112. Resol les inequacions que hi ha a continuació:

a) 3(x  5)  4(x  2)  5  7x  28  x  4, x  (,4]


 
b) x – 2(x  2) – 3(2  4x)  9  11x  19  x  19 , x   , 19 
11  11 

c) 4(10 – 2x) – 3(2x1)  –3(x1) – (2 – 3x)  42 14x  x  3, x  ,3 

113. Troba la solució de les inequacions següents:


� ��
a) ��� � �
� �
������ �
b) � � � � �
� �
��� ��� ���
c) � �
�� �� ��

x 3x 24  24 
a)  1   3  4x 12  9x  36   24  5x  x   , x    ,  
3 4 5  5 
3  x  1 x 57  57 
b)  x   8  9 x  9  6 x  2 x  48  13x  57  x  , x   , 
2 3 13  13 
x 1 x  2 x  2
c)    12 x  12  3 x  6  4 x  8  5 x  10  x  2, x    , 2 
10 40 30

114. Quina és la solució d’aquestes inequacions? 

 
�� � ��
a) Resolem l’equació: �� � � � � � � → �
�� � �
Prenem un punt de cada interval on queda dividida la recta:
x = -10 x=0 x = 10
Si x = -10 → ������ + 10 – 6 > 0 → (��� ��� no és solució de la inequació.
Si x = 0 → �� + 0 – 6 < 0 → (��� �� és solució de la inequació.

160
 
 
 
 
 
 

Equacions i inequacions 33
 
 
 
 
 
 

Si x = 10 → ��� - 10 – 6 > 0 → (3,+�� no és solució de la inequació.


Les solucions de l’equació no ho són de la inequació.
Per tant, la solució és (-2,3)
� � ��
b) Resolem l’equació: ��� � �� � � � � → � �
�� � �
Prenem un punt de cada interval on queda dividida la recta:
x = -10 x=0 x = 10
Si x = -10 → ������ - 2 ·(-10) + 8 < 0 → (��� ��� és solució de la inequació.
Si x = 0 → �� + 0 + 8 > 0 → (��� �� no és solució de la inequació.
Si x = 10 → ���� - 2·10 + 8 < 0 → (2,+�� és solució de la inequació.
Les solucions de l’equació no ho són de la inequació.
Per tant, la solució és (��� ��� � (�� ���.
c) Resolem l’equació: ��� � �� � � � � → No té solució real.
El primer membre de l’equació sempre pren valors positius.
No té solució.
d) Resolem l’equació: �� � � �� � � � � → No té solució real.
El primer membre de l’equació sempre pren valors negatius.
És una identitat.
�� � ��
e) Resolem l’equació: �� � � �� � � � � → � �
�� �

Prenem un punt de cada interval on queda dividida la recta:
x = -10 x=0 x = 10
Si x = -10 → � � ����� + 5 ·(-10) - 3 > 0 → (��� ��� és solució de la inequació.


Si x = 0 → � � �� + 5 · 0 - 3 < 0 → (��� � no és solució de la inequació.


Si x = 10 → � � ��� + 5 ·10 - 3 > 0 → ( ,+�� és solució de la inequació.

Les solucions de l’equació no ho són de la inequació.

Per tant, la solució és (��� ��� � ( � ���.


�� � �
� �
f) Resolem l’equació: �� � ��� � �� � � → � �
�� � �

Prenem un punt de cada interval on queda dividida la recta:


x = -10 x = -1 x=0

Si x = -10 → � � ������ + 31 ·(-10) + 18 > 0 → (��� � � no és solució de la inequació.

� �
Si x = -1 → � � ����� + 31 � ���� + 18 < 0 → (� � � � és solució de la inequació.
� �

Si x = 10 → � � �� + 31 ·0 + 18 > 0 → (� ,+�� no és solució de la inequació.

Les solucions de l’equació ho són de la inequació.

161
 
 
 
 
 
 

Equacions i inequacions
Equacions i inequacions 3
 
 
 
 
 
 
ૢ ૛
Per tant, la solució és (െ ǡ െ ሻǤ
૛ ૜

115. Determina la solució de les inequacions que tens a continuació:


a) x4 – 5x2 + 4 > 0 b) x4 + 8x2 – 20 ൑ 0

a) x 4 5x 2  4  0  z2 5z  4  0  z1 1 z2  4

z1 1 x1  1 x2 1

z2  4  x3  2 x4  2
Ens queden els intervals (,2),(2,1),(1,1),(1,2) i (2,). Vegem quins compleixen la inequació
agafant un punt de cada un i comprovant si la satisfan.

10   10   5  10   4 


4 2
x 9504  0  Compleix la inequació.

1, 5   1, 5   5  1, 5   4 


4 2
x 2,1875  0  No compleix la inequació.

x  0  0  50  4  4  0 
4 2
Compleix la inequació.

1, 5  1,5   5 1, 5   4 2,1875  0  No compleix la inequació.


4 2
x

x  10  10   5 10   4  9504  0  Compleix la inequació.


4 2

Veiem també que succeeix als extrems dels intervals.

x 2   2   5  2   4 0  No compleix la inequació.


4 2

x 1   1  5  1  4 0  No compleix la inequació.


4 2

x  1  1  5 1  4  0  No compleix la inequació.


4 2

x  2   2   5  2   4  0  No compleix la inequació.
4 2

Per tant, la solució és:

 , 2  1,1  2,  .


b) x4  8x2  20  0  z2  8z  20  0  z1 
10 2
z2 

10  No té solucions reals


z1 
2
z2   2
x1  x2 2

Ens resten els intervals  ,  2 ,  2, 2   i 


2,  , veiem quins acompleixen la
inequació prenent un punt de cadascun i comprovant si la satisfan.

x 10   10   8  10   20 


4 2
10780  0  No compleix la inequació.

0   0   8  0   20 
4 2
x 20  0  Compleix la inequació.

162
 
 
 
 
 
 

Equacions i inequacions 33
 
 
 
 
 
 

x  10  10   8 10   20  10780  0  No compleix la inequació.


4 2

Comprovem també si els extrems dels intervals compleixen.

   
4 2
 2   2  8  2  20 
x 0  0  Compleix la inequació.

 2   8 2 
4 2
x 2   20 0  0  Compleix la inequació.

Per tant, la solució és   2 , 2  .


 

116. Resol les inequacions que hi ha a continuació:

5
a) x 2  6 x – 1  3 x 2  3 x – 6  2 x 2  3 x – 5  0  x1 1 x2 
2
5 5
Tenim els intervals  , 1 ,  1, 

i  ,   , comprovem quins compleixen la inequació
 2 2 
prenent un punt de cadascun i veient si la satisfan.

10  2  10   3  10  – 5 


2
x 225  0  Compleix la inequació.

x 0  2  0   3  0  – 5 5  0  No compleix la inequació.


2

x 10  2 10   3 10  – 5  165  0  Compleix la inequació.


2

Comprovem també si els extrems dels intervals compleixen.

x 1  2  1  3  1 – 5 0 


2
No compleix la inequació.

2
5 5 5
x
  2    3   – 5 0  No compleix la inequació.
2 2 2
5
Per tant, la solució és  , 1   
,   .
2 
b) 2x2  25x  x  x –10  x2  35x  0  x1 
35 0
x2 

Tenim els intervals  , 35 ,  35,0 i  0,  , comprovem quins compleixen la inequació
prenent un punt de cadascun i veient si la satisfan.

x 100   100   35  100  6500  0  Compleix la inequació.


2

x 10   10   35  10  250  0  No compleix la inequació.


2

x 100  100   35 100  13500  0  Compleix la inequació.


2
 
Comprovem també si els extrems dels intervals compleixen.

163
 
 
 
 
 
 

Equacions i inequacions
Equacions i inequacions 3
 
 
 
 
 
 

x 35   35   35  35  0  No compleix la inequació.


2

0   0   35  0  
2
x 0  No compleix la inequació.

Per tant, la solució és:  , 35  0,  .


c) x2 –  2x 1· x –1  7  x2  x  6  0  No té solucions reals.
Així, o tots els punts compleixen la inequació o cap d’ells ho compleix.
Per exemple, prenem x = 0 → Compleix la inequació.

Per tant la solució és tot el conjunt dels nombres reals, 


3  39 3  39
 x –1   2x 1
2 2
d) –10  3x2  6x –10
 0  x1  x2
3 3
 3 39   3 39 3 39   3 39 
Tenim els intervals  , , ,  i  ,  , comprovem quins compleixen
 3  3 3   3
 
la inequació prenent un punt de cadascun i veient si la satisfan.

10  3  10   6  10  – 10 


2
x 230  0  Compleix la inequació.

x 0  3  0   6  0  – 10 
2
10  0  No compleix la inequació.

x 10  3 10   6 10  –
2
 10 350  0  Compleix la inequació.
Comprovem també si els extrems dels intervals compleixen.
2
3  39  3  39   3  39 
x  3   6   –10 0  No compleix la inequació.
3  3   3 
2
3  39  3  39   3  39 
x  3   6   –10 0  No compleix la inequació.
3  3   3 
 3  39   3  39 
Per tant, la solució és:  ,    ,   .
 3   3 

117. Determina les solucions d’aquestes inequacions:


��� ������
a) � ��
� �
���� ��� �
b) � ����
� �
���� �� � ��
c) �� � ��
� �
���� ����� �
d) 3 ‐ � � �
� �

164
 
 
 
 
 
 

Equacions i inequacions 33
 
 
 
 
 
 

��� ����� � �� � ��
e) � � ��
� � �
��� ������
a) � � � → ��5� � 1� � �� � � �� � �� → �� � � 2� � 1� � �
� �
El primer membre de la inequació és sempre positiu, així, sempre es compleix. Es certa per a tots
els nombres reals.
���� ��� �
b) � � 1 � � → ���� � � � 2� � 2� � � � � �� → 2� � � �� � � � �
� �
Resolem l’equació:
�� � ��
� 1
2� � �� � � � �� → �
�� � �
2
Prenem un punt de cada interval que es divideix la recta:
x = -10 x = -1 x=0
Si x = -10 → � � ������ + 7 ·(-10) + 3 > 0 → (��, ��� no és solució de la inequació.

Si x = -1 → � � ����� + 7 ·(-1) + 3 > 0 → (��, � � és solució de la inequació.


Si x = 0 → � � �� + 7 ·0 + 3 > 0 → (� ,+�� no és solució de la inequació.

Les solucions de l’equació no ho són de la inequació.

Per tant, la solució és (��, � ��

���� �� � ��
c) � � � � 5 → ��12� � � � �� � �� � � � � ��� → �� � � 2�� � �� � �
� �
Resolem l’equació �� � � 2�� � �� � �
No té solució real. Com el primer nombre de l’equació sempre pren valors negatius, la inequació
no té solució.
���� ������
d) 3 - � �� → 1� � �x � � � �2x � 2x � � �
� �
→ 2x � � 2�x � 2� � �
Resolem l’equació

� �1� � √115
�x� � 2
2x � � 2�x � 2� � � →
� �1� � √115
�x� �
� 2
Prenem un punt de cada interval que es divideix la recta:
x = -10 x = -5 x=0
����√���
Si x = -10 → � � ������ + 26 ·(-10) + 27 > 0 → (��, � és solució de la inequació.

����√��� ����√���
Si x = -5 → � � ����� + 26 ·(-5) + 27 < 0 → ( , � no és solució de la inequació.
� �
����√���
Si x = 0 → � � �� + 26 ·0 + 27 > 0 → � ,+�� és solució de la inequació.

Les solucions de l’equació ho són de la inequació.

165
 
 
 
 
 
 

Equacions i inequacions
Equacions i inequacions 3
 
 
 
 
 
 

 13  115   13  115 


Per tant, la solució és:  ,    ,   .
 2 2
   
��� ������ ��� ��
e) � � � x� → 4x � � ��x � 7 � �
� � �
Resolem l’equació

� ��� � √977
�x� � 8
4x � � ��x � 7 � � →
� ��� � √977
�x� �
� 8
Prenem un punt de cada interval que es divideix la recta:
x = -10 x = -5 x=0
����√���
Si x = -10 → 4� ������ + 33 ·(-10) + 7 > 0 → (��, � és solució de la inequació.

����√��� ����√���
Si x = -5 → � � ����� + 33 ·(-5) + 7 < 0 → ( , � no és solució de la inequació.
� �

����√���
Si x = 0 → � � �� + 26 ·0 - 27 > 0 → � ,+�� és solució de la inequació.

Les solucions de l’equació ho són de la inequació.

 33  977   33  977 


Per tant, la solució és:  ,    ,  .
 8   8 

118. Resol les inequacions amb fraccions algèbriques següents:

166
 
 
 
 
 
 

Equacions i inequacions 33
 
 
 
 
 
 

119. Troba les solucions de les inequacions següents:

x2 1 x2  3x
a) 0 c) 0
x 1 x2  4
 x2  3 x  3
b) 0 d) 0
2x  3 2 x 2  18
 

x2 1 x2 1 0 x1  1 x2 
1
a) 0    
x 1 x 1 0 x  1 

Tenim els intervals  , 1 ,  1,1 i 1,  , comprovem quins compleixen la inequació prenent
un punt de cadascun i veient si la satisfan.
������ �� ��
x=-10 → � � ��� � � → Compleix la inequació.
������� ��
���� �� ��
x=0 → � � �� � � → Compleix la inequació.
����� �
����� �� ��
x=10 → � � � � � → No compleix la inequació.
������ ��

Comprovem també si els extrems dels intervals compleixen.


����� �� �
x = -1 → � → No existeix solució.
������ �
���� �� �
x=1→ � � � → Compleix la inequació.
����� �

Per tant, la solució és:  , 1  1,1.  


3
 3 x2 
x1 
 x2  3  x2  3 0 
b) 0    3 
2x  3 2x  3 0 x 
2 

167
 
 
 
 
 
 

Equacions i inequacions
Equacions i inequacions 3
 
 
 
 
 
 

3  3
Tenim els intervals  ,  3  ,   
3,  ,  , 3  i
2  2 
 3,   , comprovem quins compleixen
la inequació prenent un punt de cadascun i veient si la satisfan.
  10  3 97 97
2

10 
x   0  No compleix la inequació.
2  10  3 23 23

  0  3 3
2

x 0   1  0  Compleix la inequació.


2  0  3 3

 1,6  3 0,44 11
2

x  1,6    0  No compleix la inequació.


2 1,6  3 0,2 5

 10  3 97
2
97
10 
x    0  Compleix la inequació.
2 10  3 17 17
Comprovem també si els extrems dels intervals compleixen.

   3
2
  3 0
 3 
x  0  No compleix la inequació.
2 3  3 2 3 3
2
3
   3 3
3 2
x    4 
 0  No existeix solució.
2 3 0
2   3
2

 
2
 3 3 0
3 
x  0  No compleix la inequació.
 
2 3 3 2 3 3

3
Per tant, la solució és:

  3,  
 2
 
3,  .

x2  3x x2  3x 
0 x1 0 x2 3 
c) 0  2  
2
x 4 x 4  0  2 x4 
x3  2

Tenim els intervals  , 2 ,  2,0 ,  0,2 ,  2,3 i  3,  , comprovem quins compleixen la
inequació prenent un punt de cadascun i veient si la satisfan.
 10  3 10 
2
130 65
10 
x  0  Compleix la inequació.
 10  4
2
96 48

 1  3 1 4  4  0
2

x 1   No compleix la inequació.


 1  4 3 3
2

168
 
 
 
 
 
 

Equacions i inequacions 33
 
 
 
 
 
 

1  31  3  3  0
2

x 1   Compleix la inequació.
1  4 4 4
2

 2,5  3 2,5 1, 25  5  0


2

x2,5   No compleix la inequació.


 2,5  4
2
2, 25 9

10  3 10 
2
70 35
x  10   0  Compleix la inequació.
10  4
2
96 48
Comprovem també si els extrems dels intervals compleixen.

 2  3  
2
2
10
x2   No existeix solució.
 2  4
2
0

 0  3 0 
2
0
0 
x 0  No compleix la inequació.

 0  4 4
2

 2  3  2 
2
2
2 
x  No existeix solució.
 2  4
2
0

 3  3 3 
2
0
3
x 0  No compleix la inequació.

 3  4 5
2

Per tant, la solució és:  , 2  0,2  3,  .


x  3 x  3  0  x3 
d) 0    
2
2 x  18 2
2 x  18 0  3 x2 
x1  3

Tenim els intervals  , 3 ,  3,3 i  3,  , comprovem quins compleixen la inequació
prenent un punt de cadascun i veient si la satisfan.
   10   3 13 1
 10 
x    0  Compleix la inequació.
2   10   18
2
182 14

 0   3 3 1
x0     0  No compleix la inequació.
2 0 
2
 18  18 6

 10   3 7 1
10 
x     0  No compleix la inequació.
2 10 
2
 18 182 26

Comprovem també si els extrems dels intervals compleixen.


  3  3 6
x  3    No existeix solució.
2   3   18 0
2

169
 
 
 
 
 
 

Equacions i inequacions
Equacions i inequacions 3
 
 
 
 
 
 

 3  3 0
3 
x   No existeix solució.
2  3   18 0
2

Per tant, la solució és:  , 3 .

120. Determina per a quins valors de x és possible efectuar les operacions següents:

5  5
a) 5  3 x  0  5  3 x  x   Es pot realitzar l’operació per a valors d’ x   ,  .
3  3

b) x  3  0  x  3  Es pot realitzar l’operació per a valors d’ x  3,   .

c) 4  3 x  x 2  0  ( x  4)( x  1)  0  La recta real es divideix en tres intervals, que són

  ,  4  , ( 4,1) y (1,  ) . Prenent un punt de cadascun d’ells, s’obtenen els intervals que satisfan
la inequació:

x 5  ( 5  4)( 5  1) 4  0  L’interval   ,  4  si satisfà la inequació.

x 0  (0  4)(0  1) 4  0  L’interval (  4,1) no satisfà la inequació.

x  2  (2  4)(2  1)  6  0  L’interval (1,  ) si satisfà la inequació.

Els punts x  4 i x1són inclosos, ja que el valor dins la inequació és 0.

Per tant, els valors de x per als que es pot realitzar l’operació són x     ,  4   1,   .

2 2
d) 2  5 x  0  2  5 x  x   Es pot realitzar l’operació per a valors d’ x   ,  .
5  5

e) 6  x  x 2  0  ( x  3)( x  2)  0  La recta real es divideix en tres intervals, que són

  ,  3  , (3, 2) y (2,  ) . Prenent un punt de cadascun d’ells, s’obtenen els intervals que satisfan
la inequació:

x  4  (  4  3)(  4  2) 6  0  L’interval   ,  3  si satisfà la inequació.

x 0  (0  3)(0  2) 6  0  L’interval ( 3, 2) no satisfà la inequació.

x  3  (3  3)(3  2)  6  0  L’interval (2,  ) si satisfà la inequació.

Els punts x  3 y x  2 no hi figuren, ja que són els que fan 0 la inequació, i per definició, no
existeix log 0.

Per tant, els valors de x per als quals es pot fer l’operació són x  (  , 3)  (2,  ) .

170
 
 
 
 
 
 

Equacions i inequacions 33
 
 
 
 
 
 

f) x 2  2 x  1  0  Per resoldre aquesta inequació no s’ha estudiat al curs un mètode particular. Per
això, les arrels de l’equació x2  2x  1 0 s’obtindran representant gràficament f ( x
) x2  1 y

g ( x )  2 x . Els seus punts de tall seran les arrels de l’equació:

Observant la gràfica, es veu que els punts de tall són x  0 , x 1i aproximadament x  4,255 , per tant,
la recta real queda dividida en quatre intervals, que són    , 0  , (0,1) , (1; 4, 255) i (4, 255;  ) . Prenent
un punt de cadascun d’ells, s’obtenen els intervals que satisfan la inequació:
1
x 1  1   1  0  L’interval    , 0  satisfà la inequació.
2
1 1
x   2  1  0  L’interval (0,1) no satisfà la inequació.
2 4
x  3 9 8 1  0  L’interval (1; 4, 255) satisfà la inequació.

x  5  25  32 1  0  L’interval (4, 255;  ) no satisfà la inequació. 

Els punts x  0 , x 1 i x  4,255 no són inclosos, ja que són els que fan 0 la inequació, i per definició,
no existeix log 0.
Per tant, els valors de x per als que es pot realitzar l’operació

121. El director d’un supermercat s’ha fixat que el nombre de clients que un dependent atén cada
hora està relacionat amb la seva experiència. Ha estimat que aquest nombre es pot calcular de
manera aproximada amb la funció:

૝૙ࢊ
࡯ሺࢊሻ ൌ 
ࢊା૜

on d és el nombre de dies que el dependent fa que hi treballa i C és el nombre de clients que


atén en una hora.

171
 
 
 
 
 
 

Equacions i inequacions
Equacions i inequacions 3
 
 
 
 
 
 

a) Quants clients per hora atendria un depenent que porta treballant dos dies?

b) El director sap que un depenent comença a ser rendible a l’empresa quan atén a 32 clients
per hora. Quan succeeix això?

c) Investiga el que succeeix amb el nombre de clients atesos per dependents que tenen molta
experiència. Pots constatar alguna característica especial?

40·2
a) C (2)
  16  Un depenent que porti treballant 2 dies atendrà 16 clients per hora.
23
40d
b) 32   32d  96  40d  8d  96  d  12  Un dependent comença a ser rendible a partir de
d 3
12 dies treballats.
c) Calculant el nombre de clients atesos per dependents amb molta experiència, s’observa que com
a màxim, cadascun podrà atendre a 40 clients per hora:

40·100 40·500 40·10000


C
 (100)  38,835 , C
(500)  39, 761 , C
 (10000)  39,988
100  3 500  3 10000  3

122. Determina la suma i el producte de les solucions d’aquesta equació:


x2 - 9x + 14 = 0
Troba les solucions de l’equació. Pots explicar què passa?

El producte d’arrels és 14 i la suma és 9.

Les arrels són x1 = 2 i x2 = 7.

Si el coeficient del terme de segon grau és 1, el producte de les arrels és el terme independent i la
suma de les arrels és l’oposat al coeficient del terme de primer grau.

123. Estudia el valor dels coeficients de l’equació biquadrada ax4 + bx2 + c = 0 perquè tingui quatre,
tres, dues, una o cap solució.
Analitzem el nombre d’arrels de l’equació biquadrada ax4 + bx2 + c = 0 a partir de les arrels
obtingudes a l’equació de segon grau associada.

az2 + bz + c = 0.

172
 
 
 
 
 
 

Equacions i inequacions 33
 
 
 
 
 
 

Si ��� � √� � � ��� � �� → No té solució.


�� ��
Si ��� � √� � � ��� � �� → � � � � → ��� � � → No té solució.
�� ��
��
→ ��� � ���� � �� � � �� → Té una solució x = 0.
��
��
→ ��� � � → Té dues solucions oposades.
��

Si ��� � √� � � ��� � �� → L’equació de segon grau té dues solucions.


Si les dos solucions són negatives, l’equació biquadrada no té solució.
������ ���� ������ ����
� ���� � � →� No té solució.
�� ��
Si una solució és negativa i l’altra és cero:
��
c=0i � �� →Té una solució x = 0.

Si una solució és positiva i l’altra és cero:

�� ��
c=0i � �� →Té tres solucions x = 0, x= �� .
� �

Si les dues solucions són positives, l’equació biquadrada té quatre solucions.


������ ���� ������ ����
� ���� � � →� Té quatre solucions.
�� ��

��������
� ����

��
x=
� ������ ����

�� ��
124. Fa quatre anys una persona tenia la meitat més la tercera part de l’edat que té ara. Quina edat
té?

Anomenem x a l’edat actual de l’individu i plantegem l’equació:


� �
� � � � � � � → �� � �� � �� � ��� → � � ���
� �
Actualment té 24 anys.
125. Una llanxa recorre 50 m per minut quan baixa per un riu i 20 m per minut quan hi puja. Si uns
amics volen fer una excursió d’anada i tornada pel riu en 3 hores, fins a quina distància poden
baixar?

173
 
 
 
 
 
 

Equacions i inequacions
Equacions i inequacions 3
 
 
 
 
 
 

Anomenem x a la distància que podem recórrer en 3 hores, és a dir, en 180 minuts. Per recórrer 50
metres pujant o baixant es necessiten 3 minuts i mig (un per baixar i dos i mig per pujar), per això:

 3,5min 
50 m  7 18000
  9000 x  x  2571,43 m
x m 180min  2 7
Es poden baixar fins a 2.571,43 metres del riu.

126. Descompon el nombre 60 en dues parts de manera que, dividint entre l’altra part, el quocient
doni 3 i el residu sigui 8.

Si x és un dels sumands, l’altre serà 60-x. Llavors, utilitzant l’algoritme de la prova de la divisió, es té
la següent equació:

x = 3(60 – x) + 8 x = 180 – 3x + 8 4x = 188 x = 47, i per tant el segon sumand serà 13.

127. Troba dos nombres consecutius si sabem que la suma de la quarta part i la cinquena part del
més petit i la suma de la tercera part i la setena part del més gran també són nombres
consecutius.

Anomenant x al nombre més petit, x + 1 és el nombre més gran consecutiu. Llavors:

105x + 84x + 420 = 140x + 140 + 60x + 60 220 = 11x  x = 20


� � ��� ���
� � � � �
� � � �

Les solucions són x = 20, x + 1 = 21.

128. Un botiguer compra melons a 40 cènt./kg i els ven a 60 cènt./kg. Calcula quants quilos de meló
havia comprat si se li han fet malbé 10 kg i ha guanyat 42 €.

Anomenem x al nombre de quilograms de melons que va comprar:

0,20(x – 10) = 42

x = 220

El botiguer va comprar 220 kg de melons.

129. Entre dos gots A i B de la mateixa capacitat distribueixen en parts desiguals 10 litres d’aigua.
El got A s’ompliria si hi aboquessin els dos terços de l’aigua que conté B. El got B s’ompliria si
hi afegissin la meitat de l’aigua del got A. Quanta aigua hi ha a cada got i quina capacitat tenen
plens?

Si al got A hi ha x litres d’aigua, llavors, el got B contindrà 10 – x litres.

� �
La capacitat total del got A ve donada per � � ��� � �� i la capacitat del got B, per � ��� � ��.
� �

Com que el volum dels dos gots és igual:

174
 
 
 
 
 
 

Equacions i inequacions 33
 
 
 
 
 
 
� �
� � ��� � �� � ��� � �� � � → �� � �� � ��� � �� � �� � ��� → �� � ��� →
� �

��������������→ ����� � �� → El got A conté 4 litres d’aigua i el got B, 6 litres.



La capacitat total d’ambdós gots és de 10 – 4 + = 8 litres.

130. Una mare, per estimular el seu fill, li dóna 1 € per cada exercici que faci bé. Si li surt malament,
el fill ha de donar 50 cèntims a la mare. Després de 20 exercicis, el fill ha guanyat 15,50 €.
Quants exercicis ha fet bé?

Sigui x el nombre d’exercicis que ha fet malament. Llavors, 20 – x serà el nombre d’exercicis ben
solucionats.

Com que duu guanyats 15,50 €, es planteja i es resol la següent equació:

( 20 – x) – 0,5x = 15,50 → 20 – 1,5x = 15,5 → x = 1,5x = 4,5 → x = 3

És a dir, ha fet 3 exercicis malament i 17 bé.

131. Si augmentéssim de 4 cm l’aresta d’un cub, el volum es multiplicaria per 8. Calcula la mida de
l’aresta.

Sigui x la longitud en cm de l’aresta del cub petit. Llavors, l’aresta del cub gran mesurarà x + 4 cm.
Com que el volum del cub gran és 8 vegades el del cub petit, es té la següent equació:

8 x 3  ( x  4) 3  8 x 3  x 3  12 x 2  48  64  7 x 3  12 x 2  48  64  0  x  4

Així, les arestes mesuren 4 i 8 cm respectivament.

132. Si r i s són les solucions de ax2 + bx + c = 0, quines són les solucions de cx2 - bx + a = 0?

Per ser r i s les arrels de ax 2  bx  c 0 , es té que

ax2 bx c k  x r x  s kx2  kr ks x  krs , on s’obtenen els valors d’ a, b i c en funció de k,
r i s:

a=k b = -k(r + s) c = krs

Ara se substitueixen els valors a la nova equació i es resol:

cx2 bx  a  0 krsx2  k  r  s x  k  0

k  r  s    k  r  s    4  krs  k
2
k  r  s   k 2 s 2  2k 2 rs  k 2 r 2  4rsk 2
x   x 
2  krs 2krs

175
 
 
 
 
 
 

Equacions i inequacions
Equacions i inequacions 3
 
 
 
 
 
 

k  r  s   k 2  r  s 
2
k  r  s   k 2 r 2  2rsk 2  k 2 s2
 x  x 
2krs 2krs
  r  s  r  s 1
k  r  s   k  r  s   r  s  r  s  x1   
 2rs s
 x  x  
2krs 2rs x    r  s    1
 r  s
 
 2 2rs r
S’ha de veure el cas que r i s són cero. Per això, s’estudien diversos casos i s’aïlla de l’equació
s x  k k  rsx2   r  s
krsx2  k  r   x 1 0.
Se suposa que k ��0, ja que així s’evita arribar a 0 = 0.
1 1
Cas 1: r � 0 i s � 0 → Les arrels són x1   i x2  
r s

Cas 2: r  0 y s  0  sx  1 0  x  1
s

1
Cas 3: r  0 y s 0  rx  1 0  x 
r
Cas 4: r  0 y s 0  k 0  1 0  No hi ha solució.

133. En una empresa que es dedica a fabricar recipients de vidre han calculat que per elaborar un
tipus de got de vidre hi ha unes despeses fixes de 3.000 € i una despesa en matèria primera
d’1,50 € per got. Quants gots podran fabricar en aquesta empresa amb una despesa màxima de
7.000 €?

Sigui el nombre màxim de gots que es podrà fabricar. Llavors, com que la despesa màxima permesa
és de 7.000 €:

���� � �� �� � ���� → �� �� � ���� → � � �� ���� ���

Per tant, com a màxim es podran fabricar 2.666 gots.

134. Volen formar un rectangle doblegant 8 m de filferro. Entre quins valors estarà l’àrea d’aquest
rectangle?

Com que el filferro té una longitud de 8 m, el semiperímetre del rectangle serà de 4m.

Si x és la base del rectangle, llavors 4 – x serà l’alçada, amb x ∈ (0 , 4).

L’àrea del rectangle ve determinada per l’expressió A = x(4-x) = 4x - x2 , que és una paràbola on la
imatge per x ∈ (0 , 4) és l’interval (0 , 4].

És a dir, l’àrea mínima és pràcticament nul·la, i l’àrea màxima val 4 m2.

176
 
 
 
 
 
 

Equacions i inequacions 33
 
 
 
 
 
 
૚ ૛
135. Una dona de 24 anys acaba de tenir un fill. L’edat del fill, quan estarà entre i de la de la
૞ ૞
mare?
  Actualitat Dintre de x anys
Mare  24  24 + x 
Fill  0  x 
2 
x   24  x  
1 2 5  5 x  48  2 x  x  16 
 24  x   x   24  x     
5 5 1  24  x  5 x  x6 
 24  x   x 
5 
૚ ૛
Quan el fill tingui entre 6 i 16 anys la seva edat serà compresa entre el i de la de la seva mare.
૞ ૞

136. El triple d’un nombre menys la seva meitat sempre és més gran que 3. Quins nombres
compleixen aquesta propietat?

x 6
3x   3  6x  x  6  x   
2 5
଺ 6 
Els nombre que compleixen aquesta propietat són els nombres més grans que , és a dir x   ,   .
ହ 5 

137. D’un nombre sabem que si al seu quadrat hi restem la seva meitat, obtenim un nombre més
petit que 1. Quin nombre pot ser?

 1  17
  x1  1, 281
x  4
x2   1  2x2  x  2  0  
2  x 1  17  0, 781
  2 4
La recta real queda dividida en tres trams. Prenent un valor de cada un d’ells, comprovem si es satisfà
la inequació:

 1 17  x1 (1) 1 3
 ,  (1)2   1    1  No se satisfà la inequació en aquest interval.
 4  2 2 2

 1 17 1 17  x0 2 0
 ,  0   0  1  Es satisfà la inequació en aquest interval.
 4 4  2

 1 17  x2 2 2
 ,   2   4 1  3  1  No se satisfà la inequació en aquest interval.

 4  2

1  17 1  17 2 x
En x1  i x2  , es compleix que x   1 , i, per tant, no poden formar part del conjunt
4 4 2
de nombres cercats.

 1 17 1 17 
Així, els nombres que satisfan la propietat donada són x   , .
 4 4 

177
 
 
 
 
 
 

Equacions i inequacions
Equacions i inequacions 3
 
 
 
 
 
 

138. Una companyia elèctrica ofereix tres tarifes que tenen una part fixa i una part proporcional al
consum.
· Tarifa A: 6,70 € quantitat fixa més 0,18 € per quilowatt hora de consum.
· Tarifa B: 9,60 € quantitat fixa més 0,13 € per quilowatt hora de consum.
· Tarifa C: 14 € quantitat fixa més 0,09 € per quilowatt hora de consum.
a) A partir de quina quantitat de consum la tarifa B és millor que la A?
b) A partir de quina quantitat de consum la tarifa C és millor que la A?
c) A partir de quina quantitat de consum la tarifa C és la millor de totes?
Que una tarifa sigui millor que una altra vol dir que un client gasti menys amb la primera tarifa que
amb la segona.

Anomenant x als kwh consumits, es té:


a) 6,70  0,18x  9,60  0,13x  5x  290  x  58
Quan es consumeixin més de 58 kwh la tarifa B serà més rendible que la tarifa A.
730
b) 6, 70  0,18 x  14  0, 09 x  9 x  730  x   x  81,11
9
Quan es consumeixin més de 81,11 kwh la tarifa C serà més rendible que la tarifa A.
6, 70  0,18 x  14  0, 09 x  9 x  730  x  81,11
c)     x  110
9, 60  0,13 x  14  0, 09 x  4 x  440  x  110 

Quan es consumeixin més de 110 kwh la tarifa C serà la més rendible de totes.
 
139. Volen construir una piscina rectangular en un jardí i, abans que res, dibuixen un esquema amb
les dimensions del jardí i de la piscina. Quines són les dimensions de la piscina si la diferència
d’àrees entre el jardí i la piscina és de 135 m2?

Anomenant x al costat més petit de la piscina, es té:


2 x 4  3x 2 
135  2 x 4  3x 2  135 
0 2 z 2  3z  135 0 

 15
 3  4  2   135 
2
3 3  33  z1  
z z  2  
22 4  z2  9

15 15  x  3
  x2 
z1    No té solució real.        z2  9   1
2 2  x2  3  

Es descarta x1  3 com a solució, ja que la longitud ha de ser positiva.


Per tant, la piscina mesura 9 metres de llarg i 3 metres d’ample. 

178
 
 
 
 
 
 

Equacions i inequacions 33
 
 
 
 
 
 

AMPLIA 
 
140. Escull la resposta adequada

□ Sigui x el nombre d’ovelles i y el nombre de gallines. Com cada ovella té 4 potes i cada gallina 2, hi
ha en total x + y animals i 4x + 2y potes..

4x  2 y x l2
l  4 x  2 y lx  ly   4  l  x  l  2  y  
x y y 4l

 2x  3x
logb 2 logb 3
□ b 1 , x 0 , 0

Es prenen logaritmes en la igualtat  2 x    3x 


log b 2 log b 3
, s’apliquen propietats dels logaritmes, es
simplifica i s’aïlla:
log b 2  log b 2 x log b 3  log b 3 x

log b 2   log b 2  log b x   log b 3   log b 3  log b x 

 logb 2  logb 2 logb x  logb 3  logb 3 logb x


2 2

 logb 2   logb 3
2 2
 logb 3 logb x  logb 2 logb x

 logb 2  logb 3   logb 3 logb 2  logb x


2 2

 lo g b 2  lo g b 3    lo g b 2  lo g b 3  
  1  log b 2  log b 3   lo g b x

  1    log b 2  log b 3  log b x

1 1
 log b 2  log
b3 logb x  logb log b x 
x
6 6

 1
 x 1 3, x   , x  1
2

 1  1
 x  2 x
 1 3, x    1 3, x   , x  1
 x
□ x  2 x  1 3   2   2


 1 3x 1 1
x  2 x
 1 3, x    1 3, x   , x  
 2  2 3
 1 1
3x  1 3, x   , x  
 2 3

179
 
 
 
 
 
 

Equacions i inequacions
Equacions i inequacions 3
 
 
 
 
 
 

 1
 x 2, x ,
2
x  1 
 x 2

 x  4, 1
x , x  1  No té solució.
 2

x 2 , 1
x , x
1
 No té solució.
 3 2 3
 4 1 1 4
x   , x , x  x
 3 2 3 3
Per tant, hi ha dues solucions reals.

 x  5x  24 0, x  0
2

d) 5 x 8  x 2  16  5 x  8  x 2  16   2
 x  5x  24 0, x  0

 5   4 1   24 
2
5 5  121  x  3
x 2  5 x  24 
 0  x  x   1
2 1 2  x2  8

2
5  52  4 1   24  5  121  x1  8
x  5 x  24  0  x    x  
2 1 2  x2  3

 x  5 x  24
2
 0, x  0  
x 8
 2  El producte de les solucions és 8  8 64 .
 x  5 x  24 
0, x  0  x 
8

e) Anomenant x, y, z a les edats dels seus fills:

t  x yz  t
  t  n  2  t  3n  t  n  2t  6n  5n  t   5
t  n 2  x  n  y  n  z  n  n
 
141. Calcula la relació entre els coeficients de l’equació següent i la suma, el producte i la suma
dels dobles productes de les seves tres arrels:

ax3  bx2  cx  d 
0
Siguin x 1 , x 2 i x 3 les arrels de l’equació donada. Llavors:

( x  x1 )( x  x 2 )( x  x 3 ) ax 3  bx 2  cx  d 

 x 3  ( x1  x 2  x3 ) x 2  ( x1 x 2  x1 x3  x 2 x3 ) x  x1 x 2 x3  ax 3  bx 2  cx  d

Comparant els termes s’obtenen les relacions demanades:


x1  x2  x3 b x1 x2  x1 x3  x2 x3 c x1 x2 x3 d

 
142. Estudia les solucions de l’equació següent segons els valors de m.
x2 – 2x + log m = 0
Per la definició de logaritme: m > 0; οൌ  „ଶ െ Ͷƒ… ൌ ሺെʹሻଶ െ Ͷ ൉ ͳ ൉ Ž‘‰
Perquè l’equació no tingui solució: Ͷ െ Ͷ݈‫ ݉݃݋‬൏ Ͳ ՜ ሺͳͲǡ ൅λሻ
Perquè l’equació tingui solució: Ͷ െ Ͷ݈‫ ݉݃݋‬ൌ Ͳ ՜ ݉ ൌ ͳͲ
Perquè l’equació tingui dues solucions: Ͷ െ Ͷ݈‫ ݉݃݋‬൐ Ͳ ՜ ሺെλǡ ͳͲሻ

180
 
 
 
 
 
 

Equacions i inequacions 33
 
 
 
 
 
 

143. Si les solucions de l’equació


ax2 + bx + c = 0
son x1 i x2 escriu equacions de segon grau que tinguin aquestes solucions:
a) Els quadrats de x1 i x2.
b) Els inversos de x1 i x2.
c) Els oposats de x1 i x2.
a) �� � � �� ��� � � �� � � � → � � � �� �� � � �� �� � � �� � � ��
� � � � � �
b) �� � � �� � � � � → �� � � � �� � � ��
�� �� �� �� �� ��

c) �� � � � ��� � � � � � � → � � � �� � � � � � � � � � � � � �

144. En Joan i en Lluís pugen a una escala mecànica En Joan puja tres vegades més ràpid que el
seu amic, i tots dos ho fan esglaó a esglaó.

Quan acaben de pujar, en Joan ha comptat 75 esglaons, i en Lluís, 50 esglaons. Amb aquestes
dades, calcula els esglaons “visibles” de l’escala.

Mentre en Joan puja un esglaó, l’escala mecànica ha pujat x esglaons, i el nombre d’esglaons visibles
és 75 + 75x.

En Lluís puja 50 esglaons. Com que ho fa tres vegades més lent que en Joan, mentre en Lluís puja
un esglaó, l’escala puja 3x. El nombre d’esglaons visibles és 50 + 150x.


Per tant, resulta que: 75 + 75x = 50 + 150x → x =

El nombre d’esglaons “visibles” és 100.

181
 
 
 
 
 
 

Equacions i inequacions
Equacions i inequacions 3
 
 
 
 
 
 

145. Calcula les solucions reals de l’equació


� �
√���� � � � √� � ��

19  x 
3
3 1729  x  3 x  19  1729  x  3
 1729  x  57 3 x 2  x  1083 3 x  6859  3
x 2  19 3 x  90  0
 
Fem   z  3 x  i resolem l’equació de segon grau resultant: 

 19 
2
19   4 1  90  z1 9  x 9  x1 729
3
2 19  1
z  19 z  90  0  z
 z   
2 1 2  z 2  10  x  10  x2  1000
3

146. Descompon el polinomi


P(x) = �� � ���� � ��
en producte de dos factors, si sabem que s’anul·la per a dos valors, ��� ���� , inversos entre ells.

Siguin r i les dues arrels inverses del polinomi P(x).

El polinomi que té les arrels és:


1 1 � ��
�� � �� �� � � � � � � � � 1 � � � � �� � 1
� �
Així, resulta que P(x) = P�x� � � � � � ���� � �� � �� � � �� � 1��� � � �� � � �� � ��.

147. Prova que les sumes de les primeres, les segones i les terceres potències de les arrels del
polinomi

���� � � �� � ��� � �� � �
valen el mateix.
Sigui ax3  bx2  cx  d l’equació general de tercer grau. Llavors, en aquest cas es té que
a 1b 2c 3
d 4.

D’altra banda, siguin x 1 , x 2 y x 3 les arrels de l’equació donada. Fent ús de les fórmules obtingudes a

l’exercici 142:

x1  x2  x3  b 2 x1 x2  x1 x3  x2 x3 c 3 x1 x2 x3  d 4

▪ Suma de les primeres potències de les arrels:

x1  x2  x3 2

▪ Suma de les segones potències de les arrels:


x1  x 2  x3  2  ( x1  x 2  x3 ) 2 (  2) 2  x12  x 2 2  x3 2  2( x1 x 2  x1 x3  x 2 x3 ) 
4

x1 2  x 2 2  x 3 2  2·3 
4  x1 2  x 2 2  x 3 2 
2

▪ Suma de las terceres potències de les arrels:


x1  x 2 2  x3 2  2  ( x1 2  x 2 2  x3 2 )( x1  x 2  x3 )  2·(  2) 
2

 x13  x 2 3  x3 3  x1 2 ( x 2  x3 )  x 2 2 ( x1  x3 )  x3 2 ( x1  x 2 ) 4 

3
x13  x23  x33  x1(x1x2  x1x3)  x2 (x1x2  x2 x3 )  x3 (x1x3  x2 x3) 4 x
1x2 x1x3 x2 x3 

182
 
 
 
 
 
 

Equacions i inequacions 33
 
 
 
 
 
 

 x13  x 2 3  x3 3  x1 (3  x 2 x3 )  x 2 (3  x1 x3 )  x3 (3  x1 x 2 ) 4 

 x13  x 2 3  x3 3  3 x1  4  3 x 2  4  3 x3  4  4 

2
x13  x23  x33 3(x1  x2  x3 ) 12 4 x
1 x2 x3 
 2 
 x13  x 2 3  x3 3  3(  2)  12 
4  x13  x 2 3  x3 3 
4  12  6  x13  x 2 3  x3 3 
2

148. Determina de manera justificada tots els parells de nombres enters (x, y) que verifiquen
l’equació x2 – y4 = 2009.

x 2  y
4
) 2 009 .
2 009  ( x  y 2 )( x  y 2

Sigui a x  y 2 i b x  y 2 . Llavors, ab  2 009 .

 ab
a  b  2 x  x  2
Per una banda, es busca una expressió per a x i per a y2 en funció d’ i y b → 
b  a 2 y 2  y 2 b  a
 2
També, es descomposa 2.009 en factors primers i s’estudien tots els productes possibles, considerant
que x i y són nombres sencers i, per tant, els dos factors polinòmics, 
a (x  y2 ) i
b ( x  y 2 ) també ho

són:

2009
 7·7·41
 49·41
 7·287
 1·2009

Ara s’estudien els casos possibles, tenint en consideració que a  b i que perquè y sigui sencer és
ba
necessari que sigui un quadrat perfecte:
2
 1  2009
  x  1500
 a  1 x
a b
 , y 2 ba
2
▪ 2009
 1·2009   
2 2
 
 b 2009  y2
2009  1
 1004  No és quadrat perfecte.
 2

 7  287
  x  147
a  7  x
a b
 , y 2 ba
 2
▪ 2009
 7·287   
2 2

 b 287  287 7
y2  140  No és quadrat perfecte.
 2

 41  49
b   x  45
a  41

a b
x  , y2
a
2
▪ 2009
 49·41   
2 2

 b 49  y 2  49  41 4 y 2
 2
El tercer cas és l’únic que compleix totes les característiques, i com que hi ha simetria parell, els únics
parells sencers que resolen l’equació donada són:  45,2 ,  45, 2 ,  45,2 i  45, 2 .

183
 
 
 
 
 
 

Equacions i inequacions
Equacions i inequacions 3
 
 
 
 
 
 

MATEMÀTIQUES A LA TEVA VIDA 
 
1. Quins avantatges té l’ús del telèfon mòbil davant del telèfon fix?
El principal avantatge del telèfon mòbil és la seva portabilitat, que permet:
▪ La comunicació (escrita i parlada) entre persones que estiguin en qualsevol lloc del mon.
▪ La connexió a internet instantània, quan sigui requerida per l’usuari.

2. Explica què representen les variables M i N en la inequació del text.


N són les trucades que pot fer.
M són els minuts que pot parlar en cada trucada a partir del primer.

3. Planteja una inequació similar a la de l’exemple per al pla C i un consum màxim de 42 €.

0,0968 N  M  0,1815N  42

4. La Paula truca a la seva família una única vegada al dia. Quants minuts pot parlar per terme
mitjà cada dia perquè el seu consum no superi els 40 € si té la tarifa B? I si té la tarifa E?

A més a més, disposa de 60 minuts gratuïts, que suposen realment 305,96 minuts al mes per
parlar. Això equival a 10,20 minuts cada dia.

▪ Inequació per a la tarifa E:

29,500,2124 N  M 0,1815N  40
Fa 30 trucades al mes, una per dia. Així, N30 i ja es pot aïllar M:

29,50 0,2124 30 M 0,1815 30  40 M 1,317


Els minuts no poden ser negatius, així que no pot pagar menys de 40 euros si fa 30 trucades. Per
tant, sols podria parlar els 300 minuts gratis, que són equivalents a 10 minuts al dia.

 
5. Detalla i compara tres ofertes de diferents companyies de telefonia mòbil que estiguin en vigor
en aquest moment.

Resposta oberta, per exemple:


Pla  Quota en €  Mínim en €  Cèntims/minut Establiment de trucada en cèntims
1  4  0  2,7 15 
2  14,9  0  8,5 0 
3  9  0  3 des del minut 6 15 
 
 

184
 
 
 
 
 
 

4
 

4
 
 
 
 
 

Sistemes d’equacions
 

 
Sistemes d’equacions   

 
 
 
 
 

ACTIVITATS 
1. Determina les incògnites, els coeficients, el terme independent i una solució d’aquestes
equacions lineals:

a) c)

b) d)

  Equació reordenada  Incògnites  Coeficients Terme independent Solució qualsevol 


a)  3x 2y  3   x, y  3, –2  –3  (–1, 0) 
b)  2x 5y  z  8   x, y, z  2, 5, 1  8  (0, 0, 8) 
c)  4x 2y  z  8   x, y, z  –4, 2, –1  –8  (1, 1, 6) 
d)  4x  y  4z 2t  7   x, y, z, t  4, –1, –4, 2  7  (0, 1, 0, 4) 
 
2. Indica quins d’aquests parells de valors són solució del sistema

?
a) x = 1, y = 0 b) x = -1, y = 1 c) x = 5, y = 4
L’únic parell que satisfà les dues equacions que formen el sistema és x = 5, y = 4.
3. Classifica aquests sistemes d’equacions i troba’n la solució pel mètode més adequat:
8 x  2y 4  4 x  6y 
2
a)  c) 
12 x  3y 6  3y  2 x 
1

p  2q 
1  2 x  y 4 
b)  d) 
3p  q 
11  x  2y 7

8x 2y  4  :  2
 4x  y  2
a)  : (3)   Sistema compatible indeterminat.
12x 3y  6   4x  y  2

p 2q  1  p 2q  1  reducció 23


b)    7p  23  p  

3p  q  11  
∙ 2 
 6p 2q  22 7

23 8
 3  q  11  q    Sistema compatible determinat.
7 7

4x  6y  2 : 
2
 2x  3y  1  Sistema compatible indeterminat.
c)  
3y 2x  1  2x  3y  1

2x  y  4 2x  4  y  substitució
d)     x 2(2x  4)  7 
x 2y  7 x 2y  7

1 10
 3x  1  x    y   Sistema compatible determinat.
3 3

185
 
 
 
 
 
 

Sistemes d’equacions
Sistemes d’equacions 4
 
 
 
 
 
 

4. Determina de quin tipus són aquests sistemes d’equacions i representa’n gràficament la


solució:

3 x  2y  5  x  y  3 
a)  c) 
6 x  4 y 0 2 x  y  1 

4 x  6y 2  10 x  y 3 
b)  d) 
6 x  9y 3  4 x  2y 6 

a) 3x 2y  5  3x 2y  5  c) x  y  3 
   x  2 y3x
 reducció  y  5
6x  4y  0 :   3x 2y  0
(2)
  2x  y  1

Sistema incompatible. No hi ha solució: Sistema compatible determinat. Solució única:



1  X  1 

1  X 

   

b)  :   d) 10x  y  3  10x  y  3


4x 6y  2

2
 2x  3y  1  
6x 9y  3 :   2x  3y  1
(3)
 4x  2y  6 :   2x  y  3 
(2)

12x  0  x  0 y310
reducció  x
 y  3
Sistema compatible indeterminat.
Infinites solucions: Sistema compatible determinat. Solució única:
Y  Y 


1  X 
1  X 

 
5. Resol mitjançant els mètodes de substitució i igualació.
3x  5y  2  2 x  7y 4 
a)  b) 
2 x  3y 5 6 x  2y 3

a) ▪ Substitució:

25y 
3x 5y  2  x   25y 
 3   2  3y  5  4 10y  9y  15  y  19  x  31
2x  3y  5   3 
2x  3y  5

▪ Igualació:

186
 
 
 
 
 
 

Sistemes d’equacions 44
 
 
 
 
 
 

25y 
 x  
3x 5y  2 
 3   25y  3y 5  4 10y  9y 15  y  19  x  31
2x  3y  5 3y  5  3 2
x
2 

b) ▪ Substitució:

4  7y 
2x  7y  4  x   4  7y  15 29
 2   6   2y  3  1221y  2y  3  y    x  
6x 2y  3  2  19 38
6x 2y  3

▪ Igualació:

4  7y 
x 
2x  7y  4  2   4  7y  2y  3  12 21y  2y  3  y   15  x   29

6x 2y  3 2y  3  2 6 19 38
x
6 

6. Determina la solució d’aquest sistema d’equacions:

3(2 x  y  1)  6(4 x  y ) 
15
  
 x  y  3( x  2 y  6) 4 
Resolem el sistema per substitució:

3(2x  y 1)6(4x  y)  15 2x  y  2


 
x  y  3(x 2y 6)  4 2x 5y  14

Resolem per reducció:

2x  y  2 

2x 5y  14
 4y  12  y  3

1
Substituïm en una de les equacions: 2x 3  2  x 
2

7. Resol els sistemes següents pel mètode de reducció:


3 x  2y 
4 x  3y  1 
a)  b) 
2 x  3y 
2 4 x  5y  2 

2
3x 2y  4 ∙  4 2
(‐2)
 6x  4y  8 2 5  16
a)    5y  2  y   x 
2x 3y  2  ∙ 3
 6x  9y  6 
 5 3 15

x  3y  1  ∙ 
(4)
 4x 12y  4 17y  6  y   6  x  13   6   11
b)  
4x  5y  2 4x 5y  2  17  17  17
8. Resol gràficament aquests sistemes:

187
 
 
 
 
 
 

Sistemes d’equacions
Sistemes d’equacions 4
 
 
 
 
 
 

a) Y  b) Y 


1  1 

X  X 

9. Determina la solució d’aquests sistemes:


xz 1  xyz  0 
 
a) 2 x  y  2z 4 b) x  y  z 2 
x  3y  z  3  x  y  z 4 

x  z 1 
 2(1 z) y 2z  4 y  4z  2 2 2 2 5
a) 2x  y 2z  4 x1z
     y     4z  2  z   x1z
 x 

1 z  3y  z  3  3y  2 
 3 3 3 3
x  3y  z  3 

5 2 2
La solució és la terna  , , 
 3 3 3

xyz 0 
 z  y  y  z  2  z  y  1
b) x  y  z  2  xzy
     y  2 z1y
  z  3 xzy
 x 1
 
z  y  y  z  4 2y  4  
x  y  z  4

La solució és la terna (1,2,3) .

10. Resol els sistemes següents:


x  2y  5z  4  x  y  1
 
a) 2 x  3y  z 1 b) x  z  1 
3x  y  2z 1 y  z  1 

x 2y 5z  4 
 2(4 5z 2y) 3y  z  1 y  9z  9  y9z9
a) 2x  3y  z  1 x45z2y
     
3(4 5z 2y) y 2z  1 7y 13z  13
3x  y 2z  1

 7(9z  9)13z  13  50z  50  z  1 y9z9


  y  0 x45z2y
  x  1
La solució és la terna (1,0,1) .

x  y  1
 1 y  z  1 yz 1 1 1
b) x  z  1  x1y
    2z  1  z   y   x 
y  z  1 
 2 2 2
y  z  1 

1 1 1
La solució és la terna  , ,  .
2 2 2

188
 
 
 
 
 
 

Sistemes d’equacions 44
 
 
 
 
 
 

11. Resol aquests sistemes:


xy  3  x  y  2z  0 
 
a) 3x  2y 19  b) 2 x  5y  6 z 0
16 
2 x  3y  3x  4 y  z  0 

xy 3  x  y  3
 E 3E E  x  y  3
a) 3x 2y  19 2 1 2
 5y  10    y 2 x 5
2x  3y  16 E
2E1 E3
5y  10  5y  10 
3
 
La solució és la parella (5, 2).

x  y 2z  0  x  y 2z  0 x  y 2z  0


 E 2E E  
b) 2x 5y  6z  0 
2 1 2
 3y 10z  0  3y 10z  0   z  0  y  0  x  0
3x  4y  z  0  E3 3E1 E3
 
 E3 E2 3E2 5z  0
 y 5z  0   


La solució és la terna (0, 0, 0).

12. Resol els sistemes següents:


xyz 2  x ‐ y ‐ 2z  8 
 
a) 3x  y  1  b) 2 x  y ‐ 3z  11
3
5 x  7y  3z  x  2y  3z  5 

xyz 2  xyz 2  xyz2 


 E 3E E   y
53z
1 x2yz 1
a) 3x  y  1  
2 1 2
 4y  3z  5 4y  3z  5  z  1  4  y   x 
2 2
5x  7y  3z  3 E
3
5E1 E3  E3 2E3 E2 19z  19 
 2y  8z  7    
1 1 
La solució és la terna  , ,1 .
2 2 

x  y 2z  8  x  y  2z  8  x  y  2z  8 
 E 2E E  
b) 2x  y  3z  11 
2 1 2
 3y  z  5 3y  z  5 
x 2y  3z  5  E
3
E1 E3  E3 E2 E3
 3y  5z  3   4z  2 

5z
y 11 x8y2z 43
 z  2  
3 y    x 
6 6
 43 11 1 
La solució és la terna  , ,  .
6 6 2

13. Expressa aquests sistemes de manera matricial i soluciona’ls:

a) b)
     
 3 4 5 2  E E  1 1 2 6  E 3E E  1 1 2 6 
a)  1 1 2 6   
1 2
 3 4 5 2  
2 1 2
  0 7 1 20 
    E3 2E1 E3   E3 3E2 7E3
 2 1 1 8   2 1 1 8     0 3 5 4   

189
 
 
 
 
 
 

Sistemes d’equacions
Sistemes d’equacions 4
 
 
 
 
 
 

  x  y 2z  6 y3,


 z1
 x  6  3 2  x  5
 1 1 2 6  
  0 7 1 20   7y  z  20    7y  21  y  3
z1

  32z  32 
 0 0 32 32     z 1

La solució és la terna (5,3,1) .

     
 1 3 2 1  E 2E E  1 3 2 1   1 3 2 1 
b)  2 1 1 3  
2 1 2
  0 7 5 1   0 7 5 1 
  E3 3E1 E3   E3 E3 /2   E3 4E2 7E3
 3 1 0 7     0 8 6 4     0 4 3 2   
  x  3y 2z  1 y7,
 z10
 x 2120  1  x  0
 1 3 2 1   z10
  0 7 5 1   7y  5z  1     7y 50  1  y  7
  z  10 
 0 0 1 10   z  10
 

La solució és la terna (0,7,10) .

14. Expressa els sistemes següents de manera matricial i soluciona’ls:

a) b)

   
 1 1 0 3  E E E  1 1 0 3 
a)  1 0 1 2  
2 1 2
 0 1 1 1 
   
 0 1 1 1   0 1 1 1 
Com que una fila es repeteix, el sistema és compatible indeterminat. Té infinites solucions, que es
donen en funció d’un paràmetre:
 1 1 0 3  x  y  3  z
, y1
 x (1 )  3  x  2 
   z

 0 1 1 1  y  z  1  y  1 
Les solucions vénen determinades per la terna (2 ,1 ,) .
     
 3 1 1 1  E E  1 3 1 2  E 3E E  1 3 1 2 
b)  1 3 1 2   
1 2
 3 1 1 1  
2 1 2
  0 10 4 5 
    E3 4E1 E3   3 E2 E3
 4 2 0 1   4 2 0 1     0 10 4 7  E 

 
 1 3 1 2 
  0 10 4 5   Sistema incompatible. No té solució.
 
 0 0 0 2 

15. Determina el nombre de solucions d’aquests sistemes:

a) b)

190
 
 
 
 
 
 

Sistemes d’equacions 44
 
 
 
 
 
 

     
 1 4 1 2  E 3E E  1 4 1 2   1 4 1 2 
a)  3 1 1 1  
2 1 2
  0 13 4 5   0 13 4 5 
  E3 2E1 E3   E3 9E2 13E3  
 2 1 1 0     0 9 3 4    0 0 3 7 
Sistema compatible determinat. Té una única solució.
     
 2 3 5 1  E E E  2 3 5 1  E E 2E  2 3 5 1 
b)  2 1 1 3   
2 1 2
 0 4 6 2  
3 2 3
 0 4 6 2 
     
 0 2 3 1   0 2 3 1   0 0 0 0 
Sistema compatible indeterminat. Té infinites solucions.

16. Discuteix aquests sistemes i troba’n la solució:

a) b)

     
 0 1 3 3  E E  2 5 5 4   2 5 5 4 
a)  0 3 2 1  
1 3
 0 3 2 1   0 3 2 1 
    E3 E2 3E3  
 2 5 5 4   0 1 3 3      0 0 7 8 
Sistema compatible determinat. Té una única solució:
3 8
y , z
7 x 
3
  2x 5y 5z  4  7

 2 5 5 4   z 8 3 14
 0 3 2 1   3y 2z  1  7 y  
  7z  8   8 7
 0 0 7 8   z 
7
 3 3 8
La solució és la terna  , ,  .
 14 7 7 

     
 1 3 4 1  E 2E E  1 3 4 1   1 3 4 1 
b)  2 2 1 2  
2 1 2
  0 4 9 0   0 4 9 0 
  E3 E1 E3   E3 E2 E3  
 1 1 3 2     0 4 7 3     0 0 2 3 
Sistema compatible determinat. Té una única solució:
27 3
y , z
2 x  
25
  x  3y  4z  1   8

 1 3 4 1   z 3 27 8
 0 4 9 0   4y  9z  0  2 y  
  2z  3  3 8
 0 0 2 3   z 
2
 25 27 3 
La solució és la terna   , ,  .
 8 8 2

17. Resol aquests sistemes d’equacions:


x 1 
 y  3 xy  2 x 
9 
a) 2  b) 2 
x 2y  y 

2
2 x y 2  7 

191
 
 
 
 
 
 

Sistemes d’equacions
Sistemes d’equacions 4
 
 
 
 
 
 

x 1 
 y  3 x  2y  7 
a) 2   2 2   2(2y  7)2  y 2  7  8y 2  98 56y  y 2  7 
2x  y  7
2x 2  y 2  7 

 y  5 2y7
 x1  3
 7y 2 56y 105  0  y 2  8y 15  0   1
 y2  3  x2  1
2y7

Les dues solucions són les parelles (3,5) i (1,3) .

xy 2x  9
 xy 2x  9
b) x  y    (2y 2  y)y  2(2y 2  y)  9  2y 3 5y 2  2y  9  0
 2y  x  2y 2  y 
y 
Obtenim les solucions aplicant Ruffini:
2 –5 2 9
–1 –2 7 –9
2 –7 9 0  y  1 és l’única solució real, i per tant, x  21  1 .
La solució del sistema és la parella (1,1) .

18. Resol els sistemes d’equacions següents:


6 x  6y 
5 
 x  y  4  x  y  2
a) 5 b) 
x2  y2   y  x 2  2(x  y)
2

36 

6x 6y  5  56y  2
x   56y 
a) 2 2 5   6   36    36y 2  5 
x y   2 2   6 
36  36x  36y  5

1 x56y 1
 2536y 2 60y  36y 2  5  60y  20  y   6 x 
3 2
1 1
L’única solució és la parella  ,  .
2 3
  
x  y  4  x  y 2  x  y  4  x  y 2  x  y  4  x  y 2
b)    
y 2  x 2  2(x  y)  Factorizant
  (y  x)(y  x)  2(x  y) y  x  2 


x  y  4  x  y 2
   x 2 x  4  x  2 x 2  2x 6  4  2x 6  16  2x  10  x  5  y  7
y  2 x 

En simplificar la segona equació del sistema, es perd la solució trivial nul·la, que també és vàlida.
Per tant, les dues solucions són les parelles (0, 0) i (5, 7).

19. La diagonal d’un rectangle fa 17 cm, i el perímetre, 46 cm. Planteja un sistema d’equacions i
calcula la longitud dels costats.

192
 
 
 
 
 
 

Sistemes d’equacions 44
 
 
 
 
 
 

Per definició de perímetre i aplicant el teorema de Pitàgores, arribem al sistema de dues equacions i
dues incògnites següent:

2x 2y  46  x  y  23 Substitució y  23 x 


 2 2    2 2 
x  y  17  x  y  289
2 2 2
x  y  289

 x  8  y  15
 x 2 (23 x)2  289  2x 2  46x 240  0   1 1

 x2  15  y2  8
Perquè la solució s’ajusti a la il·lustració, x i y han de mesurar 8 cm i 15 cm, respectivament.

20. La suma d’un nombre més cinc vegades l’invers d’un altre nombre és 2. D’altra banda, el
segon nombre més el quàdruple del primer és 9. Determina quins són aquests nombres.

Sigui x el primer nombre i y el segon. Aleshores, el sistema d’equacions és el següent:

5  x  1  y  5
x   2 Substitució xy  5  2y  2  1 1
y      x(9  4x) 5  2(9 4x)  4x 17x 13  0   13
 y  9 4x   x2   y2  4
4x  y  9  4
 13 
Hi ha dues solucions possibles, que estan determinades per les parelles (1, 5) i  ,4  .
4 

193
 
 
 
 
 
 

Sistemes d’equacions
Sistemes d’equacions 4
 
 
 
 
 
 

SABER FER

21. Resol aquests sistemes d’equacions:


2 x 5
 x  3y    y 
a) 5 b) 3 2
5x  15y  2  2 x  6y 15

2 5
 5x 15y  2 Reducció
x  3y   ∙ 
a) 5   0  0  Sistema compatible indeterminat.
 5x 15y  2
5x 15y  2 

Es busquen les solucions en funció d’un paràmetre:

215 
5x 15 2  x 
5x 15y  2    5 
y    
y  
 215 
Les infinites solucions estan determinades per les parelles  ,  .
 5 

x 5
 y   ∙   2x 6y  15 Reducció
(6)
b) 3 2   0  0  Sistema compatible indeterminat.
2x 6y  15
2x 6y  15

Es busquen les solucions en funció d’un paràmetre:

15 6 
2x 6 15 x 
2x 6y  15   2 
y    
y  
 15 6 
Les infinites solucions estan determinades per les parelles  ,  .
 2 
22. Resol els sistemes en funció del paràmetre a.
2x  y 7  3x  5y 20 
a)  b) 
ax  2y 
12  7 x  ay 
39 

2x  y  7  ∙ 
(2)
 4x 2y  14 Reducció 2 2
a)   (a  4)x  2  x  x
ax 2y  12 ax 2y  12 a4 4a

▪ Si a  4  Sistema incompatible. No té solució.


▪ Si a  4  Sistema compatible determinat. Té una única solució.

3x 5y  20 ∙ (7)




 21x 35y  140 Reducció 23 23
b)   (3a  35)y  23  y  x
7x  ay  39 ∙ 
3
 21x  3ay  117 3a 35 353a

35
▪ Si a   Sistema incompatible. No té solució.
3
35
▪ Si a   Sistema compatible determinat. Té una única solució.
3

194
 
 
 
 
 
 

Sistemes d’equacions 44
 
 
 
 
 
 

23. La diferència de les dues xifres d’un nombre és 2 i la diferència entre aquest nombre i el que
obtenim intercanviant-ne les xifres és 18. Quin nombre és?

Sigui x la xifra de les desenes, i y la xifra de les unitats. Aleshores, es donen dos casos:

▪ yx:

y  x  2 y  x  2 y  x  2


  
10x  y (10y  x)  18 10x  y 10y  x  18 9x  9y  18

y  2 x  Substitució
    9x  9(2 x)  18  9x 18  9x  18  Impossible. No té solució.
9x  9y  18

▪ xy:

x  y  2 x  y  2 x  y  2


    Sistema compatible indeterminat.
10x  y (10y  x)  18 10x  y 10y  x  18 x  y  2

Es busquen les solucions en funció d’un paràmetre:


 x  2 
x  y  2 y
 
 y  
Les infinites solucions estan determinades per les parelles (2 ,) . Per exemple, per a   3 , el
nombre 53 satisfà les condicions donades.

24. Determina el nombre de solucions dels sistemes.

3x  6y  9 3x  6y  9 
a)  b) 
 x  2y 3  x  2y 3

3x 6y  9 3x 6y  9


a)    0  18  Sistema incompatible. No té solució.
x 2y  3 ∙ 
3
 3x 6y  9

3x  6y  9  3x 6y  9
b)    0  0  Sistema compatible indeterminat. Infinites solucions.
 x  2y  3 ∙ 
3
 3x 6y  9
25. Resol aquests sistemes:
x  3y  1 xyz  0
 
a) 2 x  y  z  4 b) 3x  3y  3z 0
3x  4 y  z 5 x  y  z 0 

x  3y  1 x  3y  1
 E 2E E 
a) 2x  y  z  4 2 1 2
 5y  z  2 
3x  4y  z  5 E
3
3E1 E3 
 5y  z  2 

195
 
 
 
 
 
 

Sistemes d’equacions
Sistemes d’equacions 4
 
 
 
 
 
 

Com que es repeteix una equació, el sistema és compatible indeterminat:


z , y
2  2   11 3
x  3y  1 
   5  x  1 3
  x 
  5  5
5y  z  2  z 2 
 y 
5
 11 3 2  
Les infinites solucions estan determinades per les ternes  , ,  .
 5 5 

x  y  z  0 x  y  z  0
 E 3E E 
b) 3x  3y  3z  0 
2 1 2
 0  0
x  y  z  0 EE E3
3 1
 0  0

Sistema compatible indeterminat. Com que hi ha una equació i tres incògnites, les solucions s’han
de donar en funció de dos paràmetres:

x  y  z  0 y,
 z
 x   

Les infinites solucions estan determinades per les ternes   ,, .  

26. Resol aquest sistema:

     
 1 1 1 0  E   1 1 1 0   1 1 1 0 
2 2 E1 E2
 2 1 2 1     0 1 4 1   0 1 4 1 
  E3 
 E1 E3   E3 E2 E3  
 1 0 a 2     0 1 1 a 2     0 0 3 a 1 

▪ 3a  0  a  3  Sistema incompatible. No té solució.


▪ 3a  0  a  3  Sistema compatible determinat:
a1 1
y , z a 1 1 a 2
  x  y  z  0  a3 3a x   x
 1 1 1 0   z 1 4 a 1 a  3 3 a a3
4 1   y  4z  1 
3a y  1 y
 0 1
  (3 a)z  1  1 3 a a3
 0 0 3 a 1   z
3 a
 a 2 a 1 1 
Per a cada valor de a  3 la terna  , ,  és la solució del sistema.
 a  3 a  3 3 a 

27. La Maria, la Marisa i la Manuela volen aplegar 260 € per comprar un regal. Si la Maria posa el
doble que la Marisa, i la Manuela posa dues terceres parts del que posa la Maria, quant ha
posat cadascuna?

Diners en € que aporta
Maria  y  2x  
Marisa x

196
 
 
 
 
 
 

Sistemes d’equacions 44
 
 
 
 
 
 

2
Manuela  z y 
3
TOTAL 260
El sistema és el següent:

x  y  z  260 x  y  z  260  
   1 1 1 260  E 2E E
y  2x   2x  y  0   2 1 0 0   
2 1 2

2   
2y  3z  0  0 2 3 0  
z y
3 
     x  60
 1 1 1 260   1 1 1 260  
  0 3 2 520   0 3 2 520    y  120
  E3 2E2 3E3    z  80
 0 2 3 0     0 0 13 1040  
Així, la Maria, la Marisa i la Manuela aporten per al regal 120 €, 60 € i 80 € respectivament.

28. Resol els sistemes d’equacions següents:

3x 2  y  y 1 3
x 3x  1
5y  2
a)  b) 
5 x  7y  2 x  y  1

 

2  3x 2  y 2  3y 1  0
3x 2  y  y  1 3x 2  y  (y 1)2  
a)   2 7y  
5x  7y  2 x
5x  7y  2  5 

2
 7y 2 
 3   y 2  3y 1  0 122y 2  75y  47  0 
 5 
y  1  x  1
 1 1
 47 939
 y2    0,385  x2    0,939
 122 1000
Les solucions són les parelles (1, 1) i (0,939,  0,385) .


 
3
  (3x 1)3  27x 3  27x 2  7x 5y 1  0
3
5y 2x  3x 1 3
5y 2x
b)   
x  y  1  y  x 1
x  y  1

 x  1,1129  y  2,1129
 27x 3 27x 2  7x  5(x 1)1  0  27x 3 27x 2 2x 6  0   1 1

 x2  2,1129  y2  3,1129

Les solucions són les parelles (1,1129, 2,1129) i (2,1129, 3,1129) .

29. Resol aquests sistemes d’equacions:


1 1  3x  y  2
  1 
a) x y  b) 2 1 
  3
5
x  2y  x 2 y 

197
 
 
 
 
 
 

Sistemes d’equacions
Sistemes d’equacions 4
 
 
 
 
 
 

1 1 
  1 y  x  xy  y  x  xy 
a) x y      y 52y  (52y)y  2y 2 6y  5  0
 x 2y  5 x  52y 
x 2y  5
No té solució real.

3x  y  2
 y  3x 2 y  3x 2
b) 2 1   
  3 2y (x 2)  3y(x 2) 3xy  4y  x 2  0
x 2 y 
x  1  y  1
 1 1
 3x(3x  2) 4(3x 2) x 2  0   10 4
 x 2   y2 
 9 3
 10 4 
Les solucions són les parelles (1, 1) i  ,  .
 9 3

198
 
 
 
 
 
 

Sistemes d’equacions 44
 
 
 
 
 
 

ACTIVITATS FINALS
30. Completa els espais perquè els parells de valors següents siguin solució de l’equació
–x + 5y = 4.

a) 15y  4  y 1  =1 c) 4 5y  4  y  0  =0
b) x 15  4  x  11  =11 d) x 10  4  x  14  = –14

31. Completa els espais perquè les ternes de valors següents siguin solució de l’equació
2x – 3y + z = 8.

a) 23y  9  8  y  1  =1 d) 0 3y 2  8  y  2  = –2

b) 2x 31  8  x  2  =2 e) 215 z  8  z  5  = –5

c) 26 z  8  z  4  =4 f) 2x 6 10  8  x  4  = –4
 
32. Considera l’equació 2x + y = 5.
a) Escriu-ne totes les seves solucions.
b) Raona si (5, -15) n’és una de les solucions.
c) Completa els següents parells de valors perquè en siguin solució: (3, ) i ( , 3).
 5 
2x   5  x 
a) 2x  y  5 y
 
  2
y    
y  
 5  
Les infinites solucions les determina la parella  ,  .
 2 

5  
5 515
b) 2   10  5  No és solució.
 2
  15

5  53
c) x  3   3    1  =1 y  3   3  x   x 1 = 1
2 2

33. Identifica els sistemes per als quals el parell de valors x = 5, y = 2 és solució.
3x  4 y 
7 2  ( x  y ) x  1
 
a) x  1  c) 2x  y 
y 0  y
2  4 

y 
x  6 5y  3x 
19 
2  
b)  d) yx 
xy x  1
 3 
3 5 
199
 
 
 
 
 
 

Sistemes d’equacions
Sistemes d’equacions 4
 
 
 
 
 
 

15  8  7
7  7 
a) 4    No és solució d’aquest sistema.
 2  0 4  0
2 

2 
5  6 
b) 2   6  6  Sí que és solució d’aquest sistema.
3 5  1  1 

3 5 

2(5 2)  6


6  6
c) 10 2     Sí que és solució d’aquest sistema.
 2 2  2 
4 

10 15  19
5  19
d) 25     No és solució d’aquest sistema.
 1 1  1 
3 

34. Completa aquests sistemes perquè x = 1, y = 4 en siguin solució:

 
Per comoditat, denotem el de la primera equació amb a, i el de la segona equació amb b.
Substituint:

a 12  10 a  2 a 12  16  a  4


a)  b) 
5 4b  11 b  4 228  b b  26

 
35. Calcula el valor de a i b perquè el sistema següent tingui com a solucions x = 1 i y = -2.
3 x  2(5 x  y )  3 a
                      
4( x  2y )  3 x  2 b

3x 2(5x  y) 3  a  312(512) 3  a  a  14


   
4(x  2y) 3x 2  b 4(122) 312  b b  21

36. Indica els sistemes per als quals el parell de valors x = 3, y = -2 és solució.
x 1  y  3x 
y 0
 x  2
a) 2  c) 7 
xy 5 2 x  y  8 

8x  4 
5 x  2y 10   y  2
 5 
b) y  d) 
x  4  5
2  4 x  y  7
2 

31 
2  0 4  0
a) 2    No és solució d’aquest sistema.
 5  5
32  5

200
 
 
 
 
 
 

Sistemes d’equacions 44
 
 
 
 
 
 

15  4  10
11 10
b) 2    No és solució d’aquest sistema.
3  4  4  4
2 

2 9 
3 2 2  2 
c) 7    No és solució d’aquest sistema.
4  8
6  2  8

24  4 
2  2 2  2
d) 5    Sí que és solució d’aquest sistema.
 7  7
125  7

37. Agrupa de dues en dues aquestes equacions lineals:


4x – y – 1 = 0 y + 1 = -2x
2y = 8x + 10 y + 2x = 3
6x – 9 + 3y = 0 y – 4x = 5
Perquè formin:
a) Dos sistemes compatibles determinats.
b) Dos sistemes compatibles indeterminats.
c) Dos sistemes incompatibles.

4x  y 1  0 2y  8x 10
a)  
y 2x  3 y 2x  3

6x  9 3y  0 2y  8x 10
b)  
y 2x  3 y  4x  5

4x  y 1  0 y  1  2x 
c)  
y  4x  5 y  2x  3

38. Afegeix a l’equació 3x – 5y = 3 una altra equació, de manera que en resulti un sistema:

a) Determinat. b) Indeterminat. c) Incompatible.

Resposta oberta, per exemple:

3x  5y  3 3x 5y  3 3x  5y  3


a)  b)  c) 
2x  y  1 10y  6x 6 12x  20y 

39. Considera les equacions següents:


3x – y = 1 ax + by = c
Calcula els valors de a, b i c perquè les dues equacions
a) ax + by = c passa per (2, –3) i el sistema té com a solució (–2, –7).
b) ax + by = c passa per (4, 3) i el sistema té com a solució un punt d’ordenada 5.
c) ax + by = c passa per (–2, 0) i el sistema és incompatible.

201
 
 
 
 
 
 

Sistemes d’equacions
Sistemes d’equacions 4
 
 
 
 
 
 

ax  by  c (2,3)
 2x  3y  c 

a) (2,7) 11  
   
2a  7b  c  
Es forma un sistema amb variables a, b i c, que és compatible indeterminat, perquè hi ha dues
equacions i tres incògnites. Les possibles solucions es donen en funció d’un paràmetre:

a   
2a  3b  c  b , a
c3 
  10b  2c  c  5b  2 b  

2a  7b  c 
c  5 

a  1
 3x  y  1
Per exemple, si  1  b  1   
x  y  5
c  5 

x  by  c (4,3)
 4a  3b  c 

b) 3x  y  1  y5 3x 5  1  x  2 
   
ax  by  c  ax 5b  c  2a 5b  c 

Es forma un sistema amb variables a, b i c, que és compatible indeterminat, perquè hi ha dues


equacions i tres incògnites. Les possibles solucions es donen en funció d’un paràmetre:

a  
4a  3b  c  b , a
c5 
  c  7b   b   
2
2a 5b  c 
c  7 

a  1
 3x  y  1
Per exemple, si  1  b  1  
 x  y  7
c  7

3x  y  1
c) ax  by  c (2,0)
  2a  c  
ax  by  c 

Perquè sigui un sistema incompatible, la parella (a, b) ha de ser proporcional a la parella (3, 1) i, a
més, c 1 .

1
Aleshores, per exemple, si c = 2, tenim que 2a  2  a  1  b  , i el sistema queda així:
3

3x  y  1
1 
x  y  2
3 

202
 
 
 
 
 
 

Sistemes d’equacions 44
 
 
 
 
 
 

40. Quins dels sistemes següents tenen la mateixa solució? Resol els sistemes gràficament.
xy 
x  3y 2 y   2  0
a)  c) 4 
xy  2x 
x  5y 

x y 
  3 
3y  x  16  
b)  d) 5 2 
2 x  4  y  1 x
3
 4y 
2 
a) Y  c) Y 



(0, 0)  X  (10, 2)

1 X 

   

La solució és la parella (0, 0) La solució és la parella (10,2)


b) Y  d)

3  (10, 2) 1 


1  
 
 
 1 27  La solució és la parella (10,2)
 , 
La solució és la parella 5 5 

Per tant, tenen la mateixa solució els sistemes c) i d).


41. Determina el nombre de solucions d’aquests sistemes:
x  2y 2 2x  y  8
x  x  2y 
 
a) y  7  c)  1
4  5 
xy  3  x  1  y 

2y 
x 2  x  4y  15
3 
 x  3y 
b) 2 x  7y  4 d)  1
3
2y  3 x 6  
 5y  3x  0 

203
 
 
 
 
 
 

Sistemes d’equacions
Sistemes d’equacions 4
 
 
 
 
 
 

x  2y  2 x 2y  2      


   1 2 2  Gauss  1 2 2   1 2 2 
x  ∙4
a)  y  7    x  4y  28   1 4 28    0 1 5    0 1 5 
4        
x  y  3  1 1 3   0 1 5   0 0 0 
xy 3  
Sistema de dues equacions i dues incògnites. Té una única solució.

2y 
x  2 · 3
3x  2y  6    
3     3 2 6  Gauss  1 2 2 
b) 2x  7y  4 2x  7y  4   2 7 4     0 25 0 
 
2y  3x  6  3x 2y  6  3 2 6



 0 0 0 



Sistema de dues equacions i dues incògnites. Existeix una única solució.

2x  y  8 2x  y  8    
   2 1 8  Gauss  2 1 8 
x 2y  ∙5
c)  1    x 2y  5   1 2 5    0 1 6  
5      

 x  y  1  1 1 1   0 0 0 
x 1  y 

Sistema de dues equacions i dues incògnites. Existeix una única solució.

x  4y  15 x  4y  15      
   1 4 15  Gauss  2 1 8   2 1 8 
x  3y  ∙3
d)  1   x 3y  3   1 3 3    0 1 12    0 1 12   0  39
3        
 3x 5y  0  3 5 0 
  0 7 45   0 0 39 
5y  3x  0

Sistema incompatible. És impossible que es compleixi la tercera equació. No té solució.

42. Resol els sistemes següents per substitució:


2x  y 
1
 6 x  2y  6 
a) x y  c) 
  2 3( x  1)  y  2 8
3 5 
3 x  2y 
0
 2( x  y )  4 y 4 
b) x y  1 1  d) 
  x  3(y  2)  8 
4 2 2 

2x  y  1
x 1 2x 27 43
a) x y   y12
x
   2  5x  3 6x  30  x  y12x
 y  
  2 3 5 11 11
3 5 

3x  2y  0
3x 2y  0 x22(y1)
b) x y 1 1       3(2 2(y 1)) 2y  0 
   x  2(y 1)  2
4 2 2
126y 2y  0 12  4y  y  3 x22(y1)
  x  2

6x 2y  6 6x 2y  6 6x 2y  6 y3x3


c)      6x 2(3x 3)  6  6  6
3(x 1)(y 2)  8 3x 3 y 2  8 3x  y  3

 x  
Sistema compatible indeterminat. Té infinites solucions, que depenen d’un paràmetre: 
 y  3 3

204
 
 
 
 
 
 

Sistemes d’equacions 44
 
 
 
 
 
 

2(x  y) 4y  4 2x 2y  4  2x 2y  4 x23y


d)      2(23y)2y  4 
x 3(y  2)  8  x  3y 6  8 x  3y  2 

 4 6y 2y  4  8  4y  y  2 x23y


  x  4

43. Resol aquests sistemes per igualació:


x  y  1
yx26  
a)  c) x  y 3 
2( x  y )  1  y   
4 8 

2x y 3 
27   
4x  y   3 2 16 
b) 2 d) 
x
13
x  3y   3
y 

yx2 6  y  4 x 
a)    4  x  2x  1  x  3  y  7
2( x  y )  1  y  y 2 x  1

27  27 2y 
4x  y  8x 2y  27 x  27 2y
2   8   13 3y 
 x  3y  13  8
b) x  3y  13  x  13 3y 

7 5
272y  104 24y  22y  77  y   x 
2 2

x  y  1 y  1  x 
y  1 x  2x  3
c) x  y 3    2x  3  1 x  
  2x  2y  3 y   2
4 8 2 

1 5
22x  2x 3  1  4x  x    y 
4 4
2x y 3 
   9 24y 
3 2 16   32x  24y  9  x  9 24y
 3y 
d) 32  
x  x  3y 
  32
 3 x  3y 
y 

924y  96y  72y  9  y  8  x  24

44. Resol els sistemes següents per reducció:


x 7 
y 2x  2 x  y  7
3  
a)  c) x  y 5 
2x  1  
4y  5
 2 4
5 

4x  2  y  5  x  y  2
 
b) y 2  d) 10 
3x   2 3x   y
5  3 

205
 
 
 
 
 
 

Sistemes d’equacions
Sistemes d’equacions 4
 
 
 
 
 
 

x 7 
 y  2x 
3 5x  3y  7 5x  3y  7 x
73y
a)   E2 5E2   53y  53  y  1  5 x  2

2x 1 2x  20y  24  2  5x 50y  60


4y   5  
5 

4x 2  y  5 
4x  y  3
b) y 2     11x  11  x  1 y4
 x3
y  7
3x   2 15x  y  8
5 

2x  y  7
2x  y  7 2 x 7y 19
c) x  y 5     3y  2  y    2 x 

  2x 2y  5 3 6


2 4

x  y  2
d)
x  y  2 E
1
3∙E1
 3x  3y  6 12x  4  x   1 y2x
 y 
7
10   
3x   y  9x  3y  10 9x  3y  10 3 3
3 

45. Resol els sistemes d’equacions següents:


y 3 
x  1 y  5 3  5(x  3) 
16 
   4 
a) 2 4 2 c) 
3(2  x) y 1 
2(x  1)  3(1  y) 3 x  1  
5 3 15 

4 x 3y  2 5  1  2 x  3y 
  y  1
b) 3 6 6  d)
6 
 
xy 2(x  2y  3)
 x  xy  6 
4  5 

x 1 y  5 3 
a)
  2x  y  9 E
1
3∙E1
 6x 3y  27 11x  33  x  3 y2x9
  y  3
2 4 2  
5x  3y  6 5x  3y  6
2(x 1) 3(1 y)  3x 1

4x 3y 2 5 
   
6   8x  3y  7 8x  3y  7
b) 3 6   7x  7  x  1 y5x
 y  5
xy  5x  y  0 E
2
3E2
 15x  3y  0 

 x
4 

y 3 
5(x 3)  16
c) 4
20x  y  127 E
1
5E1
 100x 5y  635 109x  654  x  6 y12720
 x
y  7
 
3(2 x) y 1  9x 5y  19 9x 5y  19
 
5 3 15 

12x  3y 
y  1
6 2x  3y  7  2x 3y  7
d)     23x  115  x  5 y367x
  y  1
2(x 2y  3)  
7x  y  36 
E2
3E2
 21x 3y  108
xy  6
5 

46. Resol els sistemes d’equacions següents:

2(x  4)3(32y)1  12


a) 
5(x  y) 4(x 1) 2y 10  0

206
 
 
 
 
 
 

Sistemes d’equacions 44
 
 
 
 
 
 

2x  3 y 1 
  3 
b) 3 5 
x 5 2y 1 
 0
2 3 

x 3 y 5 
 0

9 8 
c) 
2x  y 1 x  2 y  3
 0

2 5 
(2 x  3 y  2)  6( x  y  1) 15
d) 
4( x  3)  12( x  y  3) 32 

2x  y  3 x  7 
x  2y  3   0
2 6 
e) 
x  6y x  y y
   0
2 3 6 

2(x  4) 3(32y)1  12 2x 6y  12


a)    x  3y 6
5(x  y) 4(x 1) 2y 10  0 x  3y  6  0 

Sistema compatible indeterminat.

2x  3 y 1 
  3 
3 5 10x 3y  27 (3)
 30x  9y  81 

b)  
x 5 2y 1   30x  40y  130 
3x  4y  13 10
 0
2 3 

 y  1  3x  4(1) 13  x  3  x  3 y  1

x  3 y 5 
  0
9 8 8x  9y  69
c)    Sistema incompatible
2x  y 1 x 2y  3  8x  9y  11
 0
2 5 

(2x  3y 2)6(x  y 1)  15 8x  3y  11 8x  3y  11


d)   (1) 
4(x  3)12(x  y 3)  32 8x 12y  8  8x 12y  8

1 3 3 1
 y   8x  4  8  x   x  y
3 2 2 3

2x  y  3 x  7 
x 2y  3   0
2 6 x 15y  2 
e)    Sistema incompatible. No té solució.
x 6y x  y y  x 15y  0
  0
2 3 6 

47. Resol els sistemes d’equacions que tens a continuació:


3( x  1)  5(2y  3)  2 x  8 
a) 
4( x  y)  3x  2  1  (1  y )

207
 
 
 
 
 
 

Sistemes d’equacions
Sistemes d’equacions 4
 
 
 
 
 
 

x 3 y 4 
 0

3 4 
b) 
x 4 y 2
 3

2 5 

x 3 y 4 
  3
c) 6 2 
2(x  3)  3(y  1)  2 

3  2x 1 1  2x 
 
4 4 6 
d) 
x  3 3  3y 
1 
2 8 

6( x  2 y  3)  5( 2 x  3 y  1)  3  6

e) 3y 
16 
x  1 
 

3(x 1)5(2y  3)  2x  8 x 10y  4 x 10y  4 1


a)     x  2 y 
4(x  y) 3x 2  (1 y) x 5y  3 x  5y  3 5

x 3 y 4 
  0
4x  3y  0  20x 15y  0
b) 3 4    x 6 y 8
x  4 y 2  5x 2y  46 20x  8y  184
 3
2 5 

x  3 y  4 
 3 x  3y  9 2x 6y  18  25
c) 6 2     x  16 y  
2x  3y  7 2x  3y  7 3
2(x  3)  3(y 1)2

3 2x 1 12x 
   x  2
4 4 6 x  2 
d)   4x 1   x  2 y  3
x  3 3 3y  4x  3y  1 y  
1  3 
2 8 

6(x 2y  3)5(2x  3y 1) 3  6


e) 3y  16x  3y  16
16  
x 1 
Com es pot comprovar, es tracta de dues expressions d’una mateixa equació, per tant, no té una
única solució com a sistema.

208
 
 
 
 
 
 

Sistemes d’equacions 44
 
 
 
 
 
 

48. Quantes solucions tenen aquests sistemes d’equacions si a = -1? I si a = -2?


ax  y 3 2 x  3ay 8
a)  b) 
2 x  2y 
7  x  3y 4 

x  y  3
x  y  3
a) Si a  1   7   Sistema incompatible. No té solució.
2x 2y  7 x  y  
2 

2x  y  3
Si a  2    Sistema compatible determinat. Una única solució.
2x 2y  7 

2x  3y  8 
b) Si a  1    Sistema compatible determinades. Una única solució.
x  3y  4

2x 6y  8  x  3y  4
Si a  2     Sistema compatible indeterminat. Infinites solucions.
x 3y  4 x 3y  4

49. Calcula el valor de k perquè el sistema següent sigui compatible determinat:


3x  2y  1

                      x  k  y 3
4 x  y  2


3x  2y  1 3x  2y  1 (3k  2)y  8 
   y 8  8
x  k  y  3  3x  3ky  9  5 5  3k  2    2  k  2 
   y     3k 2
4x  y  2 3
3x  y   
3 4 2 y  2 
4 2 
 
50. Si és possible, troba un valor de a perquè aquest sistema d’equacions:

8 x  12y  20 

4 x  9y a
a) Sigui incompatible.
b) Sigui compatible indeterminat.
c) Sigui compatible determinat.

8 12
  El sistema és sempre compatible determinat.
4 9

51. Si és possible, troba valors de a perquè aquest sistema:


ax  2y 
2

                y 
2 x   a
2 
a) Sigui incompatible.
b) Sigui compatible indeterminat.
c) Sigui compatible determinat.

209
 
 
 
 
 
 

Sistemes d’equacions
Sistemes d’equacions 4
 
 
 
 
 
 

ax 2y  2
ax 2y  2 a 2
y       a  8
2x   a 8x 2y  4a 
 8 2
2 
El sistema és incompatible si a = –8. En la resta de casos, el sistema és compatible determinat.

52. Classifica els sistemes següents segons el valor del paràmetre a.


3x  ay  5 ax  y 0 
a)  c) 
3 x  y a  x  3ay 0

 x  2y a 3x  y a  1
b)  d) 
ax  y 3 2ax  2y  4 

   5 
a)  3 a 5  Gauss
  3 a
 3 1 a   0 a 1 5 a 

▪ a 1  0  a 1  Sistema incompatible, ja que 0  6 .

▪ a 1  0  a  1  Sistema compatible determinat.


No es pot donar el cas compatible indeterminat perquè a – 1 i 5 + a no s’anul·len alhora.
   2 a 
b)  1 2 a  Gauss
  1  
 a 1 3   0 12a a 2
3 

1 11
▪ 1 2a  0  a    Sistema incompatible, ja que 0   .
2 4
1
▪ 12a  0  a    Sistema compatible determinat.
2
No es pot donar el cas compatible indeterminat perquè 12a i a2  3 no s’anul·len alhora.
   
c)  a 1 0  Gauss
  a 1 0 
 1 3a 0   0 1 3a
2
0 

Com que 1  3a2 no s’anul·la mai, el sistema és sempre compatible determinat.


   
d)  3 1 a 1  Gauss
  3 1 a 1
2

 2a 2 4   0 3 a a  a 6 

▪ 3a  0  a  3  Sistema incompatible, ja que 0  12 .

▪ 3a  0  a  3  Sistema compatible determinat.

No es pot donar el cas compatible indeterminat perquè 3 – a i a2  a 6 no s’anul·len alhora; de

fet, a2  a 6 no s’anul·la mai en els nombres reals.

53. Considera el parell de valors (3 , ), en què pot ser qualsevol nombre.

a) Escriu una equació que tingui com a solucions aquest parell de valors.

210
 
 
 
 
 
 

Sistemes d’equacions 44
 
 
 
 
 
 

b) Escriu un sistema de dues equacions lineals que tingui com a solucions aquest parell de
valors.

Resposta oberta, per exemple:

x  3y 
a) x 3y  0 b) 
6y  2x  0

54. Donat el parell de valors (1 - ,2 ), en què és qualsevol nombre real:

a) Escriu una equació lineal que tingui com a solucions aquest parell de valors.

b) Escriu un sistema d’equacions lineals que tingui com a solucions aquest parell de valors.

Resposta oberta, per exemple:

2x  y  2
a) 2(1 x)  y b) 
10x 10  5y 

55. Troba tres sistemes d’equacions compatibles indeterminats que tinguin com a solució x = -2,

y= , en què pot ser qualsevol nombre.

Resposta oberta, per exemple:

6x 6(y  2)  0
x  y  2  20x 10  10(1 y)
  x 1 y 1 
3x 6  3y  5(x  y)2  8  
2 2 

56. Si les solucions de l’equació ax + by = c són de la forma ( , 2 ) i les de l'equació dx + ey = f

són de la forma ( , - 3· + 10), troba la solució del sistema format per les dues equacions.

S’obté una equació equivalent de cada una de les equacions donades a partir de la forma que té la
solució:

ax  by  c  x , y2 y  2x 
 x,    2x  10  3x  x  2  y  4
dx  ey  f  y103
 y  10  3x 

57. L'equació ax + by = c té com a solució els parells ( +1, - 3) i l’equació dx + ey = f té com a

solució els parells ( - 2, 2 + 1).

Calcula la solució del sistema d’equacions següent:

ax  by 
c

dx  ey 
f
 

211
 
 
 
 
 
 

Sistemes d’equacions
Sistemes d’equacions 4
 
 
 
 
 
 

S’obté una equació equivalent de cada una de les equacions donades a partir de la forma que té la
solució:
ax  by 
c  
x 1,  y 3
x  y 4 
 x2,  y1   y x  4  2x  (x  4) 5  x 9
dx  ey 
f   2x  y 5
 y 13

58. Resol els sistemes següents amb tres equacions i dues incògnites, i representa les solucions:

a) c)

b) d)
     
 2 1 1   2 1 1   2 1 1 
a)  1 1 4    0 3 9    0 1 3   y  3 2
xy1
x 1
     
 4 1 1   0 1 3   0 0 0 
     
 1 1 7   1 1 7   1 1 7 
b)  2 1 2  0 1 4  0 1 4   Sistema incompatible. No té solució.
     
 3 2 0   0 5 21   0 0 1 
     
 2 1 0   2 1 0   2 1 0 
  y  2   x  1
2xy0
c)  3 2 1    0 1 2    0 1 2
     
 1 3 7   0 7 14   0 0 0 
   
 4 2 0   4 2 0 
d)  6 3 2  0 0 8   Sistema incompatible. No té solució.
   
 3 2 2   0 2 8 

59. Resol aquests sistemes de tres equacions lineals amb dues incògnites:
x  y  1 5  x 
 
2(x  4  1)  y  1 2 3 

 2(x  y)  3 
a) 4 x  y 2 x  3y  2 b) x  y  2  
3
3 x  2y  
2x  3y  1   x  y  5  7  2(x  y)
5  


2(x  4 1)  y 1 2x  y  11      
   2 1 11   2 1 11   2 1 11 
a) 4x  y  2x  3y 2  x 2y  1  
1 2 1    0 5 13    0 5 13 
 
3x 2y  7x 13y  5  7 13 5   0 19 67
   
2x  3y 1    0 0 582 
5 
Sistema incompatible. No té solució.

212
 
 
 
 
 
 

Sistemes d’equacions 44
 
 
 
 
 
 

x  y 1 5 x 
 
2 3  x  3y  13    
2(x  y)3    1 3 13   1 3 13 
b) x  y 2  x  y  3   1 1
3
 3  0 1 4 
  
x  3y  2  1 3   
x  y 5  72(x  y) 2   0 0 11 


   
 2 1 11   2 1 11 
  0 5 13    0 5 13   Sistema incompatible. No té solució.
   
 0 19 67   0 0 582 

60. Afegeix al sistema següent una equació de la forma ax + by = c (diferent de les anteriors)
perquè el sistema de tres equacions i dues incògnites que en resulta continuï sent compatible:

2 x  3y 0

11 
3x  y  
6
Perquè el sistema continuï sent compatible, la nova equació ha de ser una equació equivalent a
algunes de les donades. Per exemple, 6x 24y  11 .

61. Escriu en cada cas un sistema de tres equacions que tingui la solució indicada:
a) x = 4, y = –2, z = 0 c) x = 2, y = 7, z = –1

b) x = –3, y = 5, z = 1 d) x = 0, y = 1, z =
Resposta oberta.
a) x + 2y + 3z b) 2x + y – 2z = –3 c) 5x – y + z = 2 d) 3x + 2y – 2z = 1

62. Calcula el valor de a i b a l’equació ax – 4y + bz = -4 si saps que (-1, 3, 2) i (3, 5, 2) en són dues
solucions. Després escriu les altres dues solucions.

ax  4y  bz  4 (1,
 3, 2)
 a 12 2b  4 3a 6b  24
   8b  40  b  5 a2b8
 a  2
ax  4y  bz  4 
(3, 5, 2)
3a 20 2b  4 
 3a 2b  16 

Per tant, l’equació resultant és 2x  4y 5z  4 .

Dues solucions són, per exemple, (0, 1, 0) i (2, 7, 4).

63. Troba el valor de a i b a l’equació 3x+ay+bz = 6 si saps que (1, 5, 1) i (-1, 7, -1) en són dues
solucions. Després escriu tres solucions que compleixin:

a) Solució en la qual x = 2.

b) Solució en la qual y = -3.

213
 
 
 
 
 
 

Sistemes d’equacions
Sistemes d’equacions 4
 
 
 
 
 
 

c) Solució en la qual z = 5.

3x  ay  bz  6 (1,
5, 1)
 5a  b  3
 12a  12  a  1 b7a9
  b  2
3x  ay  bz  6 (1,
 7, ‐1)
 7a  b  9

Per tant, l’equació resultant és 3x  y 2z  6 .

 y  2z  0  Una solució és (2, 2, 1).


a) 3x  y 2z  6 x2

 3x  2z  9  Una solució és (3, –3, 0).


b) 3x  y 2z  6 y3

 3x  y  16  Una solució és (7, –5, 5).


c) 3x  y 2z  6 z5

64. Resol els sistemes següents:


xyz 1 x  2y  5z 4
 
a) x  2y  z  11 c) x  3y  4 z 9
2 x  y  2z 4  x  y  4z 11

xyz 2 x  2y  2 z 7
 
b) x  2y 5 d) 2 x  3y  2z  2
2 x  y  2z 2  x  2y  3z 2

      x 5 
 1 1 1 1   1 1 1 1   1 1 1 1  
a)  1 2 1 11    0 3 2 10    0 3 2 10   y  2
      
 2 1 2 4   0 1 0 2   0 0 1 2  z 2 

      x 1 
 1 1 1 2   1 1 1 2   1 1 1 2  
b)  1 2 0 5    0 1 1 3    0 1 1 3   y  2 
      
 2 1 2 2   0 3 4 2   0 0 7 7  z  1

      x 1 
 1 2 5 4   1 2 5 4   1 2 5 4  
c)  1 3 4 9    0 5 9 5    0 5 9 5   y  10
      z 5 
 1 1 4 11   0 3 9 15   0 0 1 5  

67 
 x
      31 
 1 2 2 7   1 2 5 4   1 2 5 4  46 
d)  2 3 2 2    0 7 6 16    0 7 6 16   y   31 
      
 1 2 3 2   0 4 1 5   0 0 31 29  29 
z 
31 

65. Resol aquests sistemes per mitjà del mètode de Gauss:


x  y  z  5  x  3y  z 15
 
2 x  y  4 z 5 3 x  y  3z 9
x  y  5z 5 x  4 y  2z  4 
a) d)

214
 
 
 
 
 
 

Sistemes d’equacions 44
 
 
 
 
 
 

x  2y  z 1 3 x  2y  2 z 
1
 
x  y  2z 2 2x  y  z 0
x  2y  z 3 3y  2z  1
b) e)
2x  y  z 24  3 x  2y  4 z 5
 
7 x  10y  2z 6 5 x  2y  2 z  1
2 x  6y  4 z 10   x  6y  10 z 11
c) f)
      x  0
 1 1 1 5   1 1 1 5   1 1 1 5  
a)  2 1 4 5    0 1 6 5    0 1 6 5   y  5
      
 1 1 5 5   0 1 2 5   0 0 8 0  z  0

      x  2
 1 2 1 1   1 2 1 1   1 2 1 1  
b)  1 1 2 2    0 1 3 3    0 1 3 3   y  0
      
 1 2 1 3   0 2 1 1   0 0 5 5  z  1 

      x  10 
 2 1 1 24   2 1 1 24   2 1 1 24  
c)  7 10 2 6    0 27 11 156    0 27 11 156   y  7
      
 2 6 4 10   0 7 5 34   0 0 58 174  z  3 

      x4 
 1 3 1 15   1 3 1 15   1 3 1 15  
d)  3 1 3 9    0 4 3 18    0 4 3 18   y  3
      z  2 
 1 4 2 4   0 7 1 19   0 0 25 50  

      x  1
 3 2 2 1   3 2 2 1   3 2 2 1  
e)  2 1 1 0    0 1 1 2    0 1 1 2   y  5
      
 0 3 2 1   0 3 2 1   0 0 1 7  z  7 

     
 3 2 4 5   3 2 4 5   3 2 4 5 
f)  5 2 2 1    0 8 13 14    0 8 13 14 
     
 1 6 10 11   0 8 13 14   0 0 0 0 
Sistema compatible indeterminat. Les infinites solucions estan determinades per aquesta terna, en
 2  13 14 
funció del paràmetre  :  , , 
 4 8 

66. Resol els sistemes següents per mitjà del mètode de Gauss utilitzant la forma matricial:


xyz  3 x  y  5z  2
 
2 x  y  5z 0 x  y  z 4 
x 13  y  2z  3
a)  3y  2z  d)
2 2

xyz  1 x  2z  11
 
2 x  y  z 4  2y  z  5
x  2y  2 z 7 x  4 y  1
b) e)

215
 
 
 
 
 
 

Sistemes d’equacions
Sistemes d’equacions 4
 
 
 
 
 
 

x  y  z  5  x  y  z 7
 
2 x  y  4 z 5 3x  y  z  15
x  y  5z 5 4 x  2y 22
c) f)
 
    
 1 1 1 3   1 1 1 3   1 1 1 3 
a)  2 1 5 0  2 1 5 0  0 3 3 6 
 1 13   1 6 4 13   0 0 0 0 
 3 2     
 2 2 
Sistema compatible indeterminat. Les infinites solucions estan determinades per aquesta terna, en
funció del paràmetre  :

12,2 ,
 1 1 1 1   1 1 1 1   1 1 1 1 
  
b)  2 1 1 4    0 1 1 2    0 1 1 2 
 1 2 2 7   0 3 3 6   0 0 0 0 
     

Sistema compatible indeterminat. Les infinites solucions estan determinades per aquesta terna, en
funció del paràmetre  :

3, 2 , 
 1 1 1 5   1 1 1 5   1 1 1 5 
     
c)  2 1 4 5    0 1 2 5  0 1 2 5 
 1 1 5 5   0 2 4 10   0 0 0 0 
     

Sistema compatible indeterminat. Les infinites solucions estan determinades per aquesta terna, en
funció del paràmetre  :

3, 5 2, 
 1 1 5 2   1 1 5 2   1 1 5 2 

d)  1 1 1 4    0 2 4 6    0 2 4 6 
 0 1 2 3   0 1 2 3   0 0 0 0 
     
Sistema compatible indeterminat. Les infinites solucions estan determinades per aquesta terna, en
funció del paràmetre  :
3 1, 32, 
 1 0 2 11   1 0 2 11   1 0 2 11 
     
e)  0 2 1 5    0 2 1 5    0 2 1 5 
 1 4 0 1   0 4 2 10   0 0 0 0 
     

Sistema compatible indeterminat. Les infinites solucions estan determinades per aquesta terna, en
funció del paràmetre  :
 5 
 2 11, , 
 2 

216
 
 
 
 
 
 

Sistemes d’equacions 44
 
 
 
 
 
 

 1 1 1 7   1 1 1 7   1 1 1 7 
     
f)  3 1 1 15    0 2 4 6    0 2 4 6 
 4 2 0 22   0 2 4 6   0 0 0 0 
     

Sistema compatible indeterminat. Les infinites solucions estan determinades per aquesta terna, en
funció del paràmetre  :

   4, 2  3,  
67. Comprova si aquests sistemes d’equacions tenen solució o no:
xyz  3 x  2y  z 5
 
2x  4z 2 2 x  y  2z 7
2 x  y  3z 1 xyz  0 
a) d)
2x  y  z 0 x  5y  5z  12
 
x  3y  2z 1 2 x  3y  3z  1
x  2y  z 2 3x  y  z 0 
b) e)
 x  y  2z 3 x  5y  3z  10 
 
2 x  3y  z 2 2 x  2y  z 4 
2y  10 z 8  3x  y  z 5
c) f)
 1 1 1 3   1 1 1 3   1 1 1 3 
     
a)  2 0 4 2    0 1 1 2    0 1 1 2 
 2 1 3 1   0 1 1 5   0 0 0 3 
     

Sistema incompatible. No té solució.


 2 1 1 0   2 1 1 0   2 1 1 0 
     
b)  1 3 2 1    0 5 3 2    0 5 3 2 
 1 2 1 2   0 5 3 4   0 0 0 6 
     
Sistema incompatible. No té solució.
 1 1 2 3   1 1 2 3   1 1 2 3 
     
c)  2 3 1 2    0 1 5 4    0 1 5 4 
 0 2 10 8   0 1 5 4   0 0 0 0 
     

Sistema compatible indeterminat.


 1 2 1 5   1 2 1 5   1 2 1 5 
     
d)  2 1 2 7    0 3 0 3    0 3 0 3 
 1 1 1 0   0 3 0 5   0 0 0 2 
     
Sistema incompatible. No té solució.
 1 5 5 12   1 5 5 12   1 5 5 12 
  
e)  2 3 3 1    0 7 7 23    0 7 7 23 
 3 1 1 0   0 14 14 36   0 0 0 10 
     

Sistema incompatible. No té solució.

217
 
 
 
 
 
 

Sistemes d’equacions
Sistemes d’equacions 4
 
 
 
 
 
 

 
 1 5 3 10 
 1 5 3 10   1 5  
   3 10   
f)  2 2 1 4    0 8 5 24    0 8 5 24 
 3 1 1 5   0 16 10 25   
     23 
 0 0 0  
 2 
 
Sistema incompatible. No té solució.

68. Expressa de manera matricial aquests sistemes i aplica el mètode de Gauss per resoldre’ls:
x  y  z 6 x  y  z 4 
 
y  4z  7 x  10  z 
x  4  y  z  2
a) c)
5z  y  2 x  3  y  2(z  x  2)
 
7x  z  2  5y  2(z  y)  3(1  x)
x  3y  2z  2(1  x) z  5x  3y  2
b) d)

x  y  z  6  1 1 1 6   1 1 1 6   1 1 1 6   x  4
       
a) y  4z  7   0 1 4 7    0 1 4 7    0 1 4 7    y  5
x  4  1 0 0 4   0 1 1 2   0 0 3 9   z  3

5z  y  2x  3 2x  y  5z  3
 
b) 7x  z  2 5y   7x  5y  z  2 
x  3y 2z  2(1 x) 3x  3y  2z  2

 121
x 
 2 1 5 3   2 1 5 3   2 1  217
     5 3  
 69
  7 5 1 2    0 17 37 17    0 17 37 17    y 
 3 3 2 2   0 3 19 5   0 0 434 136   217
       68
z  
 217

x  y  z  4 x  y  z  4
 
c) x  10  z   x  z  10 
y  z 2 y  z  2

 1 1 1 4   1 1 1 4   1 1 1 4   x  2
      
  1 0 1 10    0 1 2 6    0 1 2 6    y  10
 0 1 1 2   0 1 1 2   0 0 1 8   z  8
      

y  2(z  x 2) 2x  y 2z  2


 
d) 2(z  y)  3(1 x)  3x  2y 2z  3 
z  5x  3y 2 5x  3y  z  2

218
 
 
 
 
 
 

Sistemes d’equacions 44
 
 
 
 
 
 

 19
x 
      9
 2 1 2 4   2 1 2 4   2 1 2 4  
 32
 3 2 2 3    0 7 10 6    0 7 10 6    y 
 5 3 1 2   0 11 8 24   0 0 54 102   9
       17
z 
 9

69. Aplica el mètode de Gauss per determinar el nombre de solucions dels sistemes següents:
2x  4y  z 0 6 x  4 y  12z 6
 
x  y  2z 0 7 x  6y  18 z 7
3x  2y  z 0  6 x  2y  3 z 3
a) c)
2 x  3y  z 0 3 x  7y  4 z 4
 
x  9 y  3z 0 10 x  71y  9 z 9
5x  3y  z  0  6 x  3y  2z  10 
b) d)
 2 4 1 0   2 4 1 0   2 4 1 0  x  0
      
a)  1 1 2 0    0 6 3 0    0 2 1 0    y  0
 3 2 1 0   0 16 5 0   0 0 13 0   z  0
      

 2 3 1 0   2 3 1 0   2 3 1 0 
     
b)  1 9 3 0    0 3 1 0    0 3 1 0 
 5 3 1 0   0 3 1 0   0 0 0 0 
     
Sistema compatible indeterminat. Les infinites solucions estan determinades per aquesta terna, en
funció del paràmetre  :
  
 0, ,  
 3 
 6 4 12 6   6 4 12 6 
   
c)  7 6 18 7    0 1 24 0 
 3 2 6 3   0 0 0 0 
   
Sistema compatible indeterminat. Les infinites solucions estan determinades per aquesta terna, en
funció del paràmetre  :

118, 24, 
 4164
x 
 3 7 4 4   3 7 4 4   3 7 4 4   2167
       804
d)  10 71 9 9    0 143 67 67    0 143 67 67    y 
 6 3 2 10   0 11 10 2   0 0 0 0   2167
       41
z 
 197

219
 
 
 
 
 
 

Sistemes d’equacions
Sistemes d’equacions 4
 
 
 
 
 
 

70. Opera i resol pel mètode de Gauss els sistemes d’equacions següents:
x  3y 2  y  x  y  2(2  z)
 
y  2z  1  2 x  y z 2 x  3
3(x  y)  4 x  3(1  y) x  2y  z  1  2 x 
a) b)

x  3y  2  y  x  4y  2  1 4 0 2   1 4 0 2 
     
a) y  2z  12x   2x  y 2z  1   2 1 2 1    2 1 2 1  
3(x  y) 4x  3(1 y) 7x  3  7 0 0 3   7 0 0 3 
 3
x 
 1 4 0 2   1 4 0 2   7
     11
  0 9 2 3    0 9 2 3    y 
 0 28 0 11   0 0 56 15   28
     15
z 
 56

x  y  2(2  z) x  y 2z  4  1 1 2 4   1 1 2 4   1 1 2 4 


       
b) y  z  2x  3  2x  y  z  3   2 1 1 3    0 3 3 11    0 3 3 11 
x 2y  z  1 2x  x 2y  z  1  1 2 1 1   0 3 3 5   0 0 0 6 

Sistema incompatible. No té solució.

71. Opera i resol pel mètode de Gauss aquests sistemes d’equacions utilitzant la forma matricial:
x 1 y 3 
  12  z 
2 4

y z 6 
a) 2 x    6 
5 2 
xy 
 z  4
2 


3(x  y)  4 z 1

b) 2 x  (y  z)  5
x y 5 z 4
   
2 3 6 5 

x 1 y  3 
  12 z 
2 4  2x  y  4z  43
y z 6  
a) 2x    6  20x 2y 5z  90 
5 2 
 x  y 2z  8
xy
 z  4
2 
 2 1 4 43   2 1
   4 43   2 1
  4 43   x  3
  20 2 5 90    0 12 35 340    0 12 35 340    y  5
 1 1 2 8   0 1 8 59   0 0 131 1048   z  8
      

220
 
 
 
 
 
 

Sistemes d’equacions 44
 
 
 
 
 
 


3(x  y) 4z  1 3x  3y  4z  1
 
b) 2x (y  z)  5  2x  y  z  5 
x y 5 z  4  15x 10y 6z  1
  
2 3 6 5 
 3 3 4 1   3 3 4
   1   3 3 4 1   x  1
  2 1 1 5    0 9 5 13    0 9 5 13    y  2
 15 10 6 1   0 5 14 4   0 0 101 101   z  1
      

72. Els valors x = 3 ,y= ,z= són solució d’un sistema de tres equacions amb tres
incògnites. Escriu cinc solucions diferents per al sistema.

Resposta oberta, segons els valors que es triïn per  . Per exemple:
Si   2  (6, 1, 2)

 1 
Si   1   3,  , 1
 2 
Si  0  (0, 0, 0)

 1 
Si  1   3, ,1
 2 
Si  2  (6,1, 2)

73. Els valors (2 , - 3, ) són solució d’un sistema de tres equacions amb tres incògnites.
Completa els espais a la llibreta perquè les ternes de valors següents siguin solució del mateix
sistema:

a) x  4  2 4    2  (4, 1, 2)

b) z  3   3  (6, 0, 3)

c) y  5   3  5    8  (16, 5, 8)

d) z  2    2  (4, 5, 2)

1  5 1
e) x  1  2 1     1,  , 
2  2 2

f) y  1   3  1    4  (8,1, 4)
74. Determina quins dels valors següents són solució d’aquest sistema:
2x  y  z 8

xz 3
xy  5

221
 
 
 
 
 
 

Sistemes d’equacions
Sistemes d’equacions 4
 
 
 
 
 
 

a) d) (

b) ( e) (

c) ( f) (

2 (  2)( 6)  8 8  8


 
a)  (  6)  3  6  3  No és solució del sistema.
 ( 2)  5 2  5

2(  3)( 2)  8 8  8


 
b) (  3)  3  3  3   Sí que és solució del sistema.
(  3)( 2)  5 5  5 

2(  3)  (  3)  8 9  8


 
c) (  3)(  3)  3  6  3  No és solució del sistema.
(  3)  5 3  5 

2( 5)  ( 2)  8 8  8


 
d) ( 5)(  2)  3  3  3   Sí que és solució del sistema.
( 5)  5 5  5 

2 ( 5)(  3)  8 8  8
 
e)  (  3)  3  3  3   Sí que és solució del sistema.
 ( 5)  5 5  5 

2 (  3)( 5)  8 8  8


 
f)  ( 5)  3  5  3   No és solució del sistema.
 (  3)  5 3  5 

75. Si sabem que els valors (3 - 1, 4 +3, ) són solució d’un sistema de tres equacions amb
tres incògnites, determina quines solucions són de la forma:

a  3 1 
a)   3 1  4 3    4  (13, 13,  4)
a  4  3

a  4 3
b)   4  3      1  (4, 1, 1)
a  

76. Si la terna ( , + 1, - 3) és solució d’un sistema de tres equacions amb tres incògnites,
determina quines de les següents també ho són i justifica per què:

a)
b)
c)
d)
e)
222
 
 
 
 
 
 

Sistemes d’equacions 44
 
 
 
 
 
 

Un sistema de tres equacions i tres incògnites del qual (,  1,  3) és solució, és, per exemple:
2x  y  z  2

x  y  1
x  z  3

Substituint cada terna en el sistema, es veu si n’és solució o no:

2(2)(2 2)(2  6)  2 42 


 
a) 2 (2 2)  1  2  1  No
2 (2  6)  3 6  3 

2(2)(2 1)(2  3)  2 22 


 
b) 2 (2 1)  1  1  1  Sí
2 (2  3)  3 3  3 

2( 1)( 2)( 2)  2 22 


 
c)  1 ( 2)  1  1  1  Sí
 1( 2)  3 3  3 

2(  3)( 2)( 6)  2 22 


 
d)   3(  2)  1  1  1  Sí
  3( 6)  3 3  3 

2( 1)  (  4)  2 22 


 
e)  1    1  1  1  Sí
 1 (  4)  3 3  3 

77. Completa a la llibreta les ternes de valors perquè siguin solució del sistema següent:

Primer es resol el sistema en funció d’un paràmetre  , i després es calculen les solucions
demanades:

x  y  3z  2   6  4 5 
y   ...  Les infinites solucions estan determinades per la terna  ,  ,  .
2x   z  4  3 3 
2 
 6  4 5 
a)      ,  ,  
 3 3 
b)  3  (6, 5, 3)

223
 
 
 
 
 
 

Sistemes d’equacions
Sistemes d’equacions 4
 
 
 
 
 
 

  3  
 6  4     
5   5  3   10  4 3 
c)       , ,      , ,   
3 3 5 5 5 
  
 
 

5 9  2 9
d)   3        , 3,  
3 5  5 5

6  4 3 6  5  3 6  3 6   10 5 3 6 
e)       ,   ,    , , 
3 4  3  4  4   4 4 

6  4 3  5 3
f)  3     3,  , 
3 4  4 4

78. Resol el sistema


x  y  z

x  3y  2 z 
13

a) Per a x = 1.
b) Per a x = -4.
c) Per qualsevol valor de x.

1  y  z  y  z  1 z1y  y  2
a) x  1      3y 2(1 y)  12    (1,2, 3)
13y 2z  13 3y 2z  12  z  3

4  y  z  y  z  4 z4y  y  5
b) x  4      3y 2(4  y)  17    (4, 5,1)
4  3y 2z  13 3y 2z  17  z  1

c) Hi ha infinites solucions, dependents d’un paràmetre  :


 13
 y  z  yz  y   
2 13 132 
x      3(  z)2z  13     ,  , 
  3y 2z  13  z  13 2  2 2 
 2

79. Considera
x  2  z  y
.
3x  y  1  2z 

a) Resol el sistema per a y = 3.


b) Resol el sistema per a qualsevol valor de y.
c) Calcula y per a x = 7.
d) Calcula z per a x = -2.
 4
x 2  z  3 
 

x  z  1 Reducció  x    4 1
a) y  3     5z  1   5    , 3, 
3x  3  12z  3x 2z  2 z  1  5 5
 5

224
 
 
 
 
 
 

Sistemes d’equacions 44
 
 
 
 
 
 

b) Òbviament, en aquest cas hi ha infinites solucions, determinades per un paràmetre  :


 2 5
z
x 2  z   
 x  z  2  Reducció 

  
5   5  3 , , 2  5 
y    
3x   1 2z  3x 2z  1    x  5  3  5 5 
 5
c) A partir de l’apartat anterior:

5 3
x 7  7   y  10
5
d) També a partir de l’apartat b):

53 25 5
x  2   2   5  z  1
5 5

80. Resol aquests sistemes


xy  1  2 x  y  3z  4 
2x  y  z 0  x  y  z  4 
a)  c) 
xz 1  xyz 2
yz 1  x  2z  0 

2 x  2 y  3z 10  2 x  3y  5z  0
 x  2z 5 x  5y  3x  7
b)  d) 
4 x  3y  z 5 6 x  7y  10 z  4
3x  y  4 z 4  x  2y  2 z 7

x  y  1 x  y 1  x  y  1
  
2x  y  z  0 E
2
2E1 E2
 3y  z  2 3y  z  2
a)   
x  z  1 E
3
E1 E3
 y  z  0  E
3
3E3 E2
 4z  2
y  z  1 y z 1  E4 3E4 E2
   2z  1

Les equacions tercera i quarta són proporcionals. Se n’elimina una i es continua resolent el
sistema:

x  y  1
 1 3y122 1 1
3y  z  2  z    y  xy1
 x 
2 2 2
2z  1

2x 2y  3z  10 2x 2y  3z  10 2x 2y  3z  10


  
x 2z  5 E E 2E2
2 1
 2y  7z  0 2y  7z  0
b)   
4x  3y  z  5 E
3
2E1 E3
 y  5z  15 E
3
2E3 E2
 3z  30
3x  y  4z  4    4y 17z  38 E 3z  38
E4 3E1 2E4
4
E4 2E2
 

Sistema incompatible

2x  y  3z  4 2x  y  3z  4
 E 2E E 
x  y  z  4 
2 2 1
 y  z  4
c)  
x  y  z  2 E
3
2E3 E1
 3y  z  8
x  2z  0
 E
4
2E4 E1
 y  z  4 

225
 
 
 
 
 
 

Sistemes d’equacions
Sistemes d’equacions 4
 
 
 
 
 
 

Les equacions segona i quarta són proporcionals. Se n’elimina una i es continua resolent el
sistema:

2x  y  3z  4 2x  y  3z  4
 
y  z  4 y  z  4  z  1 yz4
  y  3 2xy3z4
 x  2

3y  z  8 
E3 E3 3E1
4z  4

2x  3y  5z  0 2x  3y 5z  0 2x  3y  5z  0 2x 3y 5z  0


  E 2E E  
x  5y  3x  7 4x  5y  7  
2 2 1
 7y 10z  14 7y 10z  14
d)    
6x  7y 10z  4 6x  7y 10z  4 E  2y 25z  4  5z  18
3E1 E3 E3 2E2 E3
3

x  2y  2z  7
 x  2y 2z  7
 E
4
2E4 E1
 7y  z  14 E
4
7E2 E4
 23z  21

El sistema és incompatible, perquè de les equacions tercera i quarta s’obtenen valors diferents de
z. Per tant, no té solució.

81. Resol els sistemes d’equacions no lineals següents:

25
x2  y2  x2  y2 10 
a)  d) 
xy  7   x  y 2 

xy  3 x 2  3y 2 6 
b)  e) 
xy  2  x  y  2 

2x  y 0 xy  2 
c)  f) 
xy  3y 20  x  y 2  3
2

x 2  y 2  25 x7y  y  3  x  4
a)    (7 y)2  y 2  25  y 2  7y 12  0   1 1

x  y  7  y2  4  x2  3

 y1  3 17  x1  3 17
x  y  3 xy3  2 2
b)   2
 (y  3)y  2  y  3y  2  0  
xy  2  3 17 3 17
 y2   x2 
2 2

2x  y  0 y2 x  x  5  y  10
c)  2x 2 6x  20  x 2  3x 10  0   1 1

xy  3y  20  x2  2  y2  4

x 2  y 2  10 yx2 2  x  3  y1  1
d)    x (x 2)2  10  x 2 2x  3  0   1
x  y  2   x2  1  y2  3

x 2 3y 2  6 xy2


e)    (y 2)2  3y 2  6  y 2 2y 1  0  y  1  x  3
x  y  2
2
xy  2 y 2x  2 4
f)    x 2
     3  x 2  2  3  x 4  3x 2  4  0  És una equació biquadrada.
x 2  y 2  3   x x
  x  2  y1  1
4 2 2
2
t1  4   1
x  3x  4  0  t  3t  4  0  
x t
 x2  2  y2  1

t2  1  Impossible

226
 
 
 
 
 
 

Sistemes d’equacions 44
 
 
 
 
 
 

82. Determina les solucions dels sistemes no lineals següents:


x  3y  5 0
 (x  2y)2  4 
a) x d) 
xy  10  2 x  y 16 
2

2x  y 4  (y  x)2  xy 31
b) 2 2  e) 
x y  13 x  (y  1)2 
xy 

x2 
x 1   2
x 11
 9y 
c) y  f) 
x
xy 4  x   3 
y 

x  3y 5  0 y  3  x  4
3y 5 2  1 1
a) x3y5
x    (3y  5)y 10   6y 13y 15  0   5 15
xy 10   2  y2    x 2  
2   6 2
 x  3  y  2
2x  y  4 y42x 2 2 2  1 1
b)    x (4 2x)  13  5x 16x  3  0   1 18
x 2  y 2  13  x 2   y2 
 5 5

x 1 
x  11 x4y (4  y)1  y  5  x  9
c) y   4  y   11  y 2 6y 5  0   1 1

 y  y2  1  x2  5
x  y  4
 x  10  y  4
(x  2y)2  4 y2x16  1
 
2 1
2
d)   x  2(2x 16)  4  3x  64x  340  0   34 20
2x  y  16  x 2   y2 
 3 3

(y  x)2  xy  31 x 2  y 2 3xy  31 x 1y


12yy 2
e)    
x (y 1)2  xy  x(1 y) y 2 2y  1
 y  3  x  2
 1 1

 y2  2  x2  3
2
 
 1 2y  y 2    x  3 129
2 
2 1 2y  y 2
    y  3y    31  y  y 6  0    3 2
 1 y   1 y   y3  1  
  3 129
  x 4 
2

x2 
 2
9y  x 2  18y  y18
x2  x2   x2 
f)     x    x  3  
x
x   3
xy  x  3y   18   18 
y 
 x  0  y  0 No és vàlida.
 1 1

3 2 2  1
 x  3x 18x  0  x(x  3x 18)  0   x2  3  y2 
 2
 x3  6  y3  2

227
 
 
 
 
 
 

Sistemes d’equacions
Sistemes d’equacions 4
 
 
 
 
 
 

83. Resol aquests sistemes:


3xy  2 x 100  x 3 
  y  1
a) x  d) 7 
 2
y 1  x  2  y 

3 4 
  1 y 2 x  15
b) x y  e) 
1
5x  6y  y  x 1 1

(x  3)(y  1) 
50 
 x 2  xy  y 1
c) x 2  f) 
 2 y 1  x 
y 4 

3xy 2x  100  y  4  x  10


 x2(y1) 2  1 1
a) x   32(y 1)y 22(y 1)  100  3y  y 52  0   13 20
 2  y2    x 2  
y 1   3 3

3 4 
  1 4x 3y  xy  y 15x 6  4x  3
 15x   1 5x 
b) x y       x  
 5x  6y  1  6   6 
5x 6y  1
 x  1  y  1
2  1 1
 5x  8x  3  0   3 2
 x 2   y2  
 5 3

(x 3)(y 1)  50


 (x  3)(y 1)  50 x2y10
c) x 2     (2y 10 3)(y 1)  50 
 2 x 2  2y  8
y4 
y  9  x  8
2 2  1 1
 2y  2y 10y 10  3y  350  0  2y 11y 63  0   7
 y2    x2  17
 2

x 3 
 x  11  y  3
 y 1 x  3
 
2

 
2
d) 7   x  2 1  x 10  7 x 2  x 2 29x 198   1 1

 7  x
 2  18  y 4
x 2  y  2


 y   152 x    y  225 4x 6 x  225 4x  6
2 2

y 2 x  15 Igualació
e)  x  2 x 
 y  x 1  1 
2
y  x 1  1 2 y  2  x 

 
2
 (5x  223)2  6 x  25x2  2194x 49729  0  No té solució

 y  0  x no existeix
x 2  xy  y  1  1
   
2 1

f)  y 1  y y 1  y  1  y 2 y 1  0    x  2
 y2  5  
2
y 1  x 
  x3  2  no vàlida

228
 
 
 
 
 
 

Sistemes d’equacions 44
 
 
 
 
 
 

84. Determina la solució d’aquests sistemes:

16 
2 x  2y  log2 x 1  log y 

a) 3y  c) 
x
9   x  1 y  1
3

xy  2  1
 25x  5y 
b) 125 x
y
d) 5
 25 
625  x  11  y 

2x 2y  16
2 xy  24  x  y  4 x  y  4 Reducció
a) 3 y        3x  3  x  1  y  3
9 x   32x  3y1  2x  y 1 2x  y  1
3

xy  2  x  2  y  1
 xy  2  xy  2 y 2x 4 2  1 1
b) 125 x         3x  4   3x  4x  4  0   2
 25y  53x4  52y  3x  4  2y  x  x2    y2  3
625   3

log2x  1log y  log2x  log10y  x  5y 


 
2

 
2
c)    5y 1  y 1 
x 1  y 1 x 1  y 1 x 1  y 1
 y  1  x  5
 5y 1  y 2 2y 1  y 2 3y 2  0   1 1

 y2  2  x2  10

1
25 x 5y   52x 5y  51  2x  y  1
d) 5     2x  x 11  1 
 x 11  y  x 11  y 
x 11  y 
  7
  y1  
5
x    2
 1
4 7
 y   No vàlida
 4x 2  3x 10  0    2 2

  y3  3
 x2  2  
  y 4  3  No vàlida

85. Per produir llet semidesnatada que conservi el 40 % del greix mesclen dos tipus de llet: una
amb el 20 % de greix i una altra amb el 70 % de greix. Quants litres de cada mena de llet
necessiten per produir 200 ℓ de llet amb el 40 % de greix?

229
 
 
 
 
 
 

Sistemes d’equacions
Sistemes d’equacions 4
 
 
 
 
 
 

Siguin x i y el nombre de litres que es necessiten de la llet amb un 20% de greix i amb un 70%,
respectivament. Aleshores, es planteja el sistema següent:

x  y  200
x  y  200 2x 2y  400  x  120
20 70 40     5y  400  
x y 200 2x 7y  800 2x  7y  800  y  80
100 100 100 
Per tant, es necessiten 120 litres de llet amb un 20% de greix i 80 litres de llet amb un 70% de greix.

86. En una bodega venen dos tipus de vi: de criança i reserva. Determina quin preu tenen si saps
que en Joan ha pagat 69 € per 3 ampolles de reserva i 12 ampolles de criança, i la Betlem n’ha
comprat 6 ampolles de criança i 8 ampolles de reserva i ha pagat 80 €.

 
Siguin x i y els preus en € de les ampolles de criança i de reserva, respectivament. La despesa que
han fet en Joan i la Betlem queda reflectit en el sistema d’equacions següent:

12x 3y  69 4x  y  23 y234 x  x  4


   3x  4(23 4x)  40  
6x  8y  80 3x  4y  40  y  7

És a dir, una ampolla de criança costa 4 €; i una de reserva, 7€.

87. Un ciclista i un cotxe surten l’un en direcció cap a l’altre des de dues ciutats separades per 180
km. Si saps que el ciclista va quatre vegades més a poc a poc que el cotxe i que triguen 1 h 48
min a trobar-se, quina és la velocitat de cada un?

Plantegem un sistema d’equacions, tenint en compte que e  v t .

Anomenem x la distància recorreguda pel ciclista, i y la velocitat que porta:

1 h 48 min  1,8 h
x  1,8y   y  20
 180 1,8y  7,2y  
180  x  7,2y   x  1,820  36

La velocitat del ciclista és de 20 km/h, i la del cotxe és de 80 km/h.

230
 
 
 
 
 
 

Sistemes d’equacions 44
 
 
 
 
 
 

88. Calcula un nombre si saps que la suma de les seves xifres es 14, i que si invertim l’ordre en
que estan col·locades el nombre disminueix de 18 unitats.

Anomenem x la xifra de les desenes, i y la de les unitats:

x  y  14 y  14  x   x  8
  126  9x  9x 18  0  18x  144  
10y  x 18  10x  y  9y  9x 18  0  y  14  8  6

El nombre és el 86.

89. Un camió surt d’una ciutat a 80 km/h i dues hores després surt un cotxe en la mateixa direcció
a 100 km/h. Quant trigarà a trobar-lo i quanta distància ha recorregut el cotxe fins aquell
moment?

Plantegem un sistema d’equacions, tenint en compte que e = v · t.

Anomenem x la distància recorreguda pel camió, i y el temps que triga a trobar-lo:

x  80y   y  8
  80y 160  100y  
x 160  100y   x  808  640

Trigarà 8 hores a trobar-lo i haurà recorregut 800 km.

90. Els costats d’un rectangle es diferencien de 2 m. Si augmentéssim 2 m cada costat, l’àrea
s’incrementaria de 40 m2. Calcula les dimensions del polígon.

Anomenem x el costat més petit del polígon i y la seva àrea:

x(x 2)  y   x  8
  x 2 6x  8  x 2 2x  40  4x  32  
(x 2)(x  4)  y  40  y  8(8 2)  80

Els costats del polígon original fan 8 m i 10 m, respectivament.

91. En Jaume vol tancar un terreny de forma rectangular. Després de posar filferro a dos costats
consecutius, s’adona que ha utilitzat 170 m de filferro. Si sap que la diagonal del rectangle fa
130 m, quines són les dimensions i l’àrea del terreny?

Anomenem x i y les dimensions del terreny:

x  y  170  2 (170) (170)2  4 16000 170  70  y1  120


  y 170y 6000  0  y   
x 2  y 2  1302  21 2  y2  50
Si y1  120  x1  170 120  50
Si y2  50  x2  170  50  120

Les dimensions del terreny són 120 m i 50 m, respectivament.

L’àrea del terreny és de 6 000 m2.

231
 
 
 
 
 
 

Sistemes d’equacions
Sistemes d’equacions 4
 
 
 
 
 
 

92. Determina les dimensions que te un plec rectangular de paper si sabem que si deixem els
marges laterals d’1 cm i els verticals de 2,5 cm, l’àrea és 360 cm2, i que si els marges laterals
són de 2 cm i els verticals d’1,25 cm, l’àrea és la mateixa.

Anomenem x i y les dimensions del plec:

350 2y 
 x 
(x  2)(y 5)  360  y 5  (15) (15)2  4 2(875) 15 85
   2y 2 15y  875  0  y   
(x  4)(y  2,5)  360 350  4y  22 4
x
y 2,5 
 35
 350  2
35 2
 y1    x1 
 2 35
 5  14
 2
 350 225
 y2  25  x2   20
255

Les dimensions del plec són 20 cm i 25 cm, respectivament.

93. Les habitacions de l’Aurora i la Marga tenen forma quadrada. La suma de les superfícies és
29,89 m2 i la diferència d’aquestes superfícies és 5,39 m2. Calcula’n les dimensions.

Siguin x i y les longituds dels costats de les habitacions respectives. Suposem que x  y , aleshores:

  y  3,5
x 2  y 2  29,89 Reducció  y 2 29,89x 2
2
x  4,2     1
    2x  35,28   1
 y2  3,5  No vàlida
x 2  y 2  5,39 
 x2  4,2 No vàlida

Les solucions negatives no tenen sentit. Per tant, les úniques mides vàlides de les habitacions són
4,2 m i 3,5 m respectivament.

94. Considera un quadrat i la seva diagonal. Formem un rectangle de base igual a la diagonal del

quadrat i de perímetre 4+6 cm. Determina les dimensions de les dues figures si sabem que

la suma de les àrees és 9+6 cm2.

Sigui x la longitud del costat del quadrat en cm.

Primer, pel teorema de Pitàgores, se calcula la longitud de la diagonal del quadrat, en funció de x:

d 2  x 2  x 2  d  x 2 cm

Sigui y la longitud de l’altura del rectangle. Les equacions que formaran el sistema són:

2
▪ Perímetre = 4 6 2  2x 2 2y ▪ Suma d’àrees = 96 2  x  xy 2

232
 
 
 
 
 
 

Sistemes d’equacions 44
 
 
 
 
 
 


 y  2 3 2  x 2 
4  6 2  2x 2 2y  9 6 2  x 2
 9 6 2  x
Igualació
2     2 3 2  x 2  
9 6 2  x 2  xy 2  y  x 2
x 2 
 x  3  y  2
 x 2  x(6 2 2) 9 6 2  0   1 1

 x2  32 2  y2  2 No vàlida

Per tant, el costat del quadrat fa 3 cm; l’altura del rectangle, 2 cm; i la base del rectangle, 3 2 cm.

95. Calcula les dimensions d’un rectangle de perímetre 40 cm del qual coneixem que la suma de

les seves diagonals és 8 cm.

Sigui x la longitud de la base del rectangle, i y l’altura. Primer, es calcula la longitud de la diagonal
amb el teorema de Pitàgores. Després, tenint en compte que les diagonals d’un rectangle són iguals i
que el semiperímetre fa 20 cm, es planteja i resol el sistema d’equacions següent:


2 x 2  y 2  8 13  y20x
   
2 2
   2 x 2 (20  x)2  8 13 
x  y  20
 x  8  y1  12
 4(x 2  400  x 2  40x)  832  x 2 20x  96  0   1
 x2  12  y2  8

Per tant, el rectangle té unes dimensions de 8 x 12 cm.

96. L’altura d’un rectangle és tres cinquenes parts de la base. La tercera part de la diagonal del

rectangle fa cm. Calcula les dimensions del rectangle.

Sigui x la longitud de la base del rectangle, i y l’altura. Amb el teorema de Pitàgores s’obté la diagonal,
i amb ella es planteja i resol el sistema d’equacions següent:

2
3 
3  x2  x  
2
y  x 2
 
 
5  5
   3  2
  34   x 2   x    3 34 
2
x y 2
 3  5  
 34   
3 
3 
2
 x  15  y  9
 x   x   306  34x 2  7650  x 2  225   1
2 1

5   x2  15  No vàlida

Per tant, el rectangle té unes dimensions de 15 x 9 cm.

233
 
 
 
 
 
 

Sistemes d’equacions
Sistemes d’equacions 4
 
 
 
 
 
 

97. La xifra de les unitats d’un nombre de dues xifres és 3 unitats més gran que el quadrat de la
xifra de les desenes. El quadrat del consecutiu del nombre original supera de 55 unitats el
quadrat de l’original. Determina el nombre del qual partim.

Siguin x i y les xifres de les unitats i les desenes respectivament. Per trobar el nombre de partida s’ha
de resoldre el sistema següent:

x  3 y 2 
  (3 y 2 10y 1)2  55(3 y 2 10y)2 
(x 10y 1)  55(x 10y) 
2 2

 y  12  No vàlida
108y 2  80y 16  55106y 2 60y  9  y 2 10y 24  0   1
 y2  2  x2  7

El nombre de partida és 27.

98. El lloguer d’una tenda de campanya és de 80 € al dia. L’Agnès fa els preparatius d’una excursió
amb els seus amics i fa aquesta reflexió: ≪Si fóssim tres amics més, hauríem de pagar 6 €
menys cada un.≫ Quants amics van d’excursió?

Nre. d’amics Preu en € per persona TOTAL en €

x  y 80

x + 3  y–6 80

 
S’ha de resoldre el sistema següent:

xy  80  y 80  80   x  5  y  16
 x (x 3) 6   80  x 2  3x  40  0   1 1

(x  3)(y 6)  80 x   x2  8 No vàlid

Per tant, van d’excursió 5 amics, i cada un paga 16 €.

99. La diferència de dos nombres és 5. La diferencia dels quadrats dels seus consecutius és 95.
Troba els dos nombres.

234
 
 
 
 
 
 

Sistemes d’equacions 44
 
 
 
 
 
 

Siguin x i y els nombres que busquem. Suposem x  y . S’ha de resoldre el sistema següent:

xy 5  xy5
   (y 51)2 (y 1)2  85 10y  60  y  6  x  11
2 2
(x 1) (y 1)  95

Els nombres són 6 i 11.

100. La suma de les arrels quadrades de dos nombres és 6. El quocient dels dos nombres és igual
al més petit dels dos nombres. Troba aquests nombres.

Siguin x i y els nombres que busquem. Suposem x  y . S’ha de resoldre el sistema següent:

x  y  6  y  4  x  16
 xy2 2 2
x   y  y  6  y 13y 36  0  
1 1

 y  y2  9  x2  81
y 

Es comproven les solucions i es descarta (81, 9). Per tant, els nombres que es busquen són 16 i 4.

101. Busca dos nombres naturals que verifiquin que el seu producte menys 12 unitats coincideix
amb el quàdruple del més petit, si sabem a més que la diferència del triple del més petit i el
més gran és 1 unitat.

Siguin x i y els nombres que busquem. Suposem x  y . S’ha de resoldre el sistema següent:

y  3  x  8
xy 12  4y  x3y1 2  1 1
  (3y 1)y 12  4y  3y 5y 12  0   4
3y  x  1  y2   No vàlida
 3

Per tant, els nombres són 8 i 3.

102. Dues vaques i tres vedells valen el mateix que setze ovelles. Una vaca i quatre ovelles valen
igual que tres vedells. Tres vedells i vuit ovelles costen el mateix que quatre vaques.
Determina el preu de cada animal.

Siguin x, y, z els preus respectius, en €, d’una vaca, un vedell i una ovella.

235
 
 
 
 
 
 

Sistemes d’equacions
Sistemes d’equacions 4
 
 
 
 
 
 

2x  3y  16z   2 3 16 0   2 3 16 0 


    
x  4z  3y    1 3 4 0  E2 2E2 E1  0 9 36 0 
  
3y  8z  4x   4 3 8 0  E 2E E
 
 0 9 24 0 
3 1 3
   E
3
E2 E3

 2 3 16 0 
 
 0 9 36 0   x  y  z  0
 0 0 12 0 
 

S’obté la solució trivial nul·la. Òbviament, no té sentit per a aquest problema.

103. Troba dos nombres naturals si saps que la cinquena part de la seva diferència és 2 i que la
13
suma dels seus inversos és .
72

.Siguin x i y els nombres que busquem. Suposem x  y . S’ha de resoldre el sistema següent:

xy 
 2  x  y 10 
5   1 1 13
  1 1 13  xy10
     72(2y 10)  13y(y 10) 
1 1 13     y 10 y 72
  x y 72 
x y 72 
 y  8  x  18
2  1 1
13y 14y  720  0   90
 y2   No vàlida
 13

Els nombres són 18 i 8.

104. Troba dos nombres naturals que verifiquin que l’invers del primer més el triple del segon és

i la diferència entre el doble del primer i l’invers del segon és .


Siguin x i y els nombres que busquem. Suposem x  y . S’ha de resoldre el sistema següent:

1 31 
 3y  
5   x(3115y)  5 x  20 
5
x 3115y
y 19    
1 19 y(4x 19)  2  3115y 
2x   
y 2 
y  2  x  5
2  1 1
 285y  539y 62  0   31
 y2    No vàlida
 285
Els nombres són 5 i 2.

105. Calcula tres nombres si saps que sumen 6, la suma del doble del més gran i el triple de la
diferència dels altres dos és -4, i la diferència del triple del més gran i el doble de la suma dels
altres dos és 8.

Siguin x i y els nombres que busquem. Suposem x  y,z . S’ha de resoldre el sistema següent:

236
 
 
 
 
 
 

Sistemes d’equacions 44
 
 
 
 
 
 

x  y  z  6  1 1 1 6   1 1 1 6   1 1 1 6   x  4
   
2x  3(y  z)  4   2 3 3 4    0 1 5 16    0 1 5 16    y  1
3x 2(y  z)  8  3 2 2 8   0 1 1 2   0 0 6 18   z  3

Els nombres són 4, –1 i 3.

106. La suma de les tres xifres d’un nombre és 16. Calcula el nombre si saps que la diferència de
les unitats i les desenes és el doble de les centenes, i la suma de les unitats i les centenes
supera d’1 unitat el doble de les desenes.

Siguin x, y, z les xifres de les centenes, desenes i unitats respectivament.

x  y  z  16  1 1 1 16   1 1 1 16   1 1 1 16   x  2
       
z  y  2x    2 1 1 0    0 1 3 32    0 1 3 32    y  5
z  x  1 2y   1 2 1 1   0 1 0 5   0 0 3 27   z  9

El nombre que es busca és 259.

107. Al portamonedes d’en Martí hi ha 11 monedes d’1 €, 0,5 € i 0,2 €, amb un valor total de 4,9 €.
Calcula quantes monedes hi ha de cada tipus si saps que la suma del doble de monedes d’1 €
més les monedes de 0,5 € coincideix amb el nombre de monedes de 0,2 €.

1 € 0,5 € 0,2 € TOTAL

Nombre de monedes x y z 11

€ que aporten x 0,5y 0,2z 4,9

x  y  z  11  1 1

1 11 

 1 1

1 11   x  3
 
2x  y  z    2 1 1 0  ...   0 1 3 22    y  1
    0 0 1,4 9,8  
x  0,5y  0,2z  4,9  1 0,5 0,2 4,9    z  7
   

Per tant, en Martí té tres monedes d’1 €, una moneda de 0,5 € i set monedes de 0,2 €.

108. Una llibreta gran, una de mitjana i una de petita costen juntes 3,9 €. Tres de grans, quatre de
mitjanes i una de petita costen 11,1 €, i sis de petites i tres de mitjanes costen el mateix que
cinc de grans. Calcula el preu de cada tipus de llibreta.

Siguin x, y, z els preus d’una llibreta gran, una de mitjana i una de petita respectivament.

x  y  z  3,9  1 1 1 3,9   1 1 1 3,9   1 1 1 3,9   x  1,8


       
3x  4y  z  11,1   3 4 1 11,1    0 1 2 0,6    0 1 2 0,6    y  1,2
     
6z  3y  5x   5 3 6 0   0 8 11 19,5   0 0 27 24,3   z  0,9

Per tant, la llibreta gran val 1,8 €; la mitjana, 1,2 €; i la petita, 0,9 €.

237
 
 
 
 
 
 

Sistemes d’equacions
Sistemes d’equacions 4
 
 
 
 
 
 

109. La Laura té bitllets de 5 €, 10 € i 20 €. En total són 16. Els triple dels bitllets de més valor és
igual al total dels altres, i la meitat dels bitllets de més valor és igual a la diferència dels de
menys valor i els de valor intermedi. Calcula quants bitllets de cada tipus té la Laura.

Siguin x, y, z el nombre de bitllets de 5 €, 10 € i 20 € respectivament.

3z  x  y   
 1 1 3 0   1 1 3 0   x  7
z  
 x  y    2 2 1 0    0 4 5 0    y  5
2   1 1 1 16   0 0 4 16   z  4
x  y  z  16     

La Laura té 7 bitllets de 5 €, 5 de10 € i 4 de 20 €.

110. Sobre un camió carreguen tres bidons. El doble del pes del primer menys el triple del pes del
segon és 4 kg. El quíntuple del pes del segon menys un terç del pes del tercer és 50 kg.
Calcula el pes de cada bidó si entre tots tres pesen 275 kg.

Siguin x, y, z els pesos del primer, segon i tercer bidó respectivament.

 
x  y  z  275 x  275 y  z
  5y 2z  546 5y  2z  546
2x  3y  4  2(275 y  z) 3y  4   2 
1  1  15y  z  150   30y 2z  300
5y  z  50 5y  z  50
3  3 
 1339
x   38,26
 35
 846
 35y  846   y   24,17
 35
 1488
z   212,57
 7

El primer bidó pesa 38,26 kg; el segon, 24,17 kg, i el tercer 212,57 kg.

111. Un nombre que té tres xifres el representem xyz. Determina’l si saps que si l’escrivim xyz, el
nombre disminueix de 459 unitats; si l’escrivim yxz, disminueix de 360 unitats, i que yzx és 45
unitats més petit que yxz.

Anomenem a, b i c les xifres de les centenes, desenes i unitats, respectivament.

100a 10b  c  100c 10a  b  459 90a  9b  99c  459


 
100a 10b  c  100b 10a  c  360  90a  90b  360 
100b 10c  a  100b 10a  c  45  9a  9c  45
10a  b 11c  51
 b  c  1 a  c 5
10a 10b  40 ac5
 
10b 10c  10 b  c 1
a  c  5

238
 
 
 
 
 
 

Sistemes d’equacions 44
 
 
 
 
 
 

Per determinar la solució, sabem que els tres nombres són enters i, per tant, c és un nombre de 0 a 9.
Com que a = c + 5, c només pot ser 0, 1, 2, 3 i 4. Per a cada un d’aquests valors de c:

Si c  0  a  5 i b 1 . El nombre és 510.

Si c  1  a  6 i b  2 . El nombre és 621.

Si c  2  a  7 i b  3 . El nombre és 732.

Si c  3  a  8 i b  4 . El nombre és 843.

Si c  4  a  9 i b  5. El nombre és 954.

112. Al bloc de pisos on visc han fet obres. L’administrador de la comunitat intenta esbrinar quant
cobren per hora un electricista, un lampista i un paleta. Sabem que:

 Al 4t A, l’electricista es va estar 1 hora i el paleta, 2 hores, i han hagut de pagar 78 € de


mà d’obra.

 Al 3r D han pagat 85 € per les 2 hores que s’hi ha estat el lampista i l’hora del paleta.

 A casa meva s’hi han estat 1 hora el lampista, 1 hora l’electricista i 3 hores el paleta, i
ens han cobrat 133 €.

Determina les tarifes horàries de cada professional.

  €/h hores 4t A hores 3r D hores a casa 

Electricista  x 1 0 1

Lampista  y 0 2 1

Paleta  z 2 1 3

TOTAL en €  87 85 133

x 2z  87  1 0 2 87   1 0 2 87   1 0 2 87   x  73
       
2y  z  85   0 2 1 85    0 2 1 85    0 2 1 85    y  39
x  y  3z  133  1 1 3 133   0 1 1 46   0 0 1 7   z  7

L’electricista cobra 73 € per hora; el lampista, 39 €; i el paleta, 7 €.

239
 
 
 
 
 
 

Sistemes d’equacions
Sistemes d’equacions 4
 
 
 
 
 
 

113. Quan l’any 1800 Beethoven escriu la seva primera simfonia, és deu vegades més gran d’edat
que el jove Franz Schubert. Passa el temps i és Schubert qui compon la cèlebre Simfonia
inacabada. Aleshores la suma de les edats dels dos músics és igual a 77 anys. Cinc anys
després mor Beethoven i en aquell moment Schubert té els mateixos anys que tenia
Beethoven quan va compondre la seva primera simfonia.

Determina l’any de naixement de cadascun d’aquests dos compositors.

  Beethoven Schubert EQUACIÓ 

Edat el 1800  10x x

Edat el 1800 + y anys  10x  y   xy 10x  y  x  y  77  

Edat el 1800 + y + 5 anys  10x  y 5   x  y 5   x  y 5  10x  

10x  y  x  y  77 11x 2y  77 11x 2y  77 Reducció  x  3


    29x  87  
x  y 5  10x  9x  y  5 18x 2y  10  y  22

Beethoven va morir l’any 1800 + 22 + 5 = 1827 a l’edat de 30 + 3 + 22 + 5 = 57 anys. Per tant, va


néixer el 1770.

Schubert tenia 3 anys el 1800, per tant va néixer el 1797.

240
 
 
 
 
 
 

Sistemes d’equacions 44
 
 
 
 
 
 

Amplia 

 
114. Escull la resposta adequada.

□ De la successió d’igualtats obtenim el sistema següent:

a 1  b 2  a  b  3  1 1 0 0 3   1 1 0 0 3 
     
a 1  c  3   a  c  2   1 0 1 0 2   ...   0 1 1 0 5 
a 1  d  4 a  d  5  1 0 0 1 5   0 0 1 1 7 
 

És un sistema compatible indeterminat. Les infinites solucions vénen donades per un paràmetre  :
a   5 b   2 c   7 d
A la vista dels resultats, està clar que la variable c sempre és la més gran de les quatre.
m2n  (*) ha de ser un múltiple de 3.
□ Es planteja el sistema següent, on (*) representa la dada desconeguda:

2m n  3  Reducció 4m2n  6


    3m  6 (*)  3(2 m)  (*) 
m2n  (*) m2n  (*)

  Edat del gos en mesos Edat Isabel en mesos 
Actualment  y x 
D’aquí 5 anys = 60 mesos  y 60   x 60  
 

x 
y   x  360
212  y
x
x x 60
24 60  5  x 1440  120  4x 240  
x  60  24 6  y  15
y  60  52
12 
Per tant, el gos té 15 mesos d’edat.

241
Sistemes d’equacions
Sistemes d’equacions 4

Llançaments de 3 punts (z) Llançaments de 2 punts (y) Llançaments d’1 punt (x)


Punts  3z  2y x 

Es planteja i resol el sistema següent:

3z  2y  x  61  1 2 3 61   1 2 3 61   1 2 3 61   x  13
       
x  1 y    1 1 0 1    0 1 1 20    0 1 1 20    y  12
3z  2y   0 2 3 0   0 2 3 0   0 0 1 8   z  8

Per tant, han encistellat 13 llançaments lliures.

115. Escriu un sistema lineal de dues equacions amb dues incògnites de manera que compleixi la
condició indicada en els casos següents:

a) Que sigui compatible determinat amb solució x = -1, y = -2.

b) Que sigui compatible indeterminat i x = -1, y = -2, sigui una solució del sistema.

c) Que sigui compatible indeterminat i totes les solucions de la forma x = -1, y = .

Resposta oberta, per exemple:

4 x  y 2  7x  5y  3  2 x  1  4 y 3·(1  y )  y 
a)  b)  c) 
5 x  y 3  5(2 y  x)  
3 3(3x  1)  3 y  2( y  1)  y  2 

116. Escriu un sistema d’equacions lineals de tres equacions amb tres incògnites que compleixi les
condicions següents:

a) Que sigui compatible determinat amb solució x = 3, y = 2, z = 2.

b) Que sigui compatible indeterminat x = 3, y = 2 i z = 2, sigui una solució del sistema.


c) Que sigui compatible indeterminat i x = 3, y = 2 i z = 2 i x = 1, y = 1 i z = 1, dues solucions.
Resposta oberta, per exemple:

3x 2y  3z  7 x  y 2z  1 x 2y  1


  
a) x  y 2z  1 b) 7x  y  8z  3 c) x 2z  1
7x  y  8z  3 6x 6z  2(y 1) x  y  z  1

117. Resol aquest sistema d’equacions:


1 2 3 
  1
x y z 
2 4 5 17 
  
x y z 3
3 6 2 2
  
x y z 3

Seguint el suggeriment, es fa el canvi de variable següent:


242
 
 
 
 
 
 

Sistemes d’equacions 44
 
 
 
 
 
 

1 2 3   
  1   11 6
x y z X 2Y  3Z  1  X  x 
  6  11
1  1
2 4 5 17  Xx 17   11 xX  12
    2X  4Y  5Z   ...  Y   y  
x y z 3  Y 1 3  12 y
1
 11
3 6 2 2   1 
y Y
2    z  3
    1 3X 6Y 2Z    Z   1 
x y z 3  Z z
 3   3 z Z 

118. Tres nombres, a, b i c, diferents de zero, estan en progressió aritmètica. Si augmentem a d’1
unitat o c de 2 unitats, resulten progressions geomètriques. Determina aquests nombres.

Sigui d la diferència de la progressió aritmètica.

Siguin r1 i r2 les raons de les dues progressions geomètriques que es poden obtenir. Les dades de
l’enunciat s’expressen de la manera següent:

▪ Progressió aritmètica: ▪ Progressió geomètrica 1 ▪ Progressió geomètrica 2

a a 1 a
b  ad b  (a 1)r1 b  ar2

c  b  d  a 2d c  br1  ar12 c 2  br2  ar22

Eliminant d de les dades de la progressió aritmètica: a  2b  c

Eliminant r1 de les dades de la progressió geomètrica 1: b  c(a 1)

Eliminant r2 de les dades de la progressió geomètrica 2: b  a(c 2)


Resolent el sistema format per les tres equacions anteriors es determinen a, b i c:

a  2b  c 
 
Substituint E2 en E1 a  2 c(a 1)  c  a  2 c(a 1)  c  Substitució
b  c(a 1)   
Igualant E2 i E3
     
 c(a 1)  a(c 2)  c  2a
b  a(c 2) 
a  8

 a  2 2a(a 1) 2a  b  12
c  16

119. En una cafeteria, un got de llimonada, tres entrepans i set pastissets han costat 1 xíling i 2
penics. Si tenim en compte que 1 xíling val 12 penics, calcula:

a) El preu d’un got de llimonada, un entrepà i un pastisset.

b) El preu de dos gots de llimonada, tres entrepans i cinc pastissets.

Siguin x, y, z els preus respectius d’un got de llimonada, un entrepà i un pastisset, i a, b són els preus
que es busquen.

Es verifica que:

243
 
 
 
 
 
 

Sistemes d’equacions
Sistemes d’equacions 4
 
 
 
 
 
 

x  3y  7z  14

x  4y 10z  17

x  y  z  a
2x  3y  5z  b

Considerant les dues primeres equacions del sistema, i agafant z com a paràmetre, s’obté:

x 3y  14  7z  x  5 2z 
 
x  4y  1710z  y  3 3z 

Substituint aquests valors en les equacions tercera i quarta, es verifica que:

52z 33z  z  a


  a  8 i b  19
10  4z  9  9z 5z  b

244
 
 
 
 
 
 

Sistemes d’equacions 44
 
 
 
 
 
 

MATEMÀTIQUES A LA TEVA VIDA 
 
1. Explica per què la demanda i l’oferta no pugen o baixen al mateix temps.

Si la demanda d’un determinat producte puja, se’n faran més compres, hi haurà menys quantitat
d’exemplars per vendre i, per tant, l’oferta disminueix.

Al contrari, si l’oferta augmenta, hi ha més exemplars per vendre, i per tant, la demanda disminueix.

2. El dia de l’espectador, els encarregats del cinema decideixen posar les entrades a 4 €. Si les
equacions d’oferta i de demanda es mantenen, et sembla que hi haurà un excés de demanda o
un excés d’oferta?

Com que el preu baixa, la demanda augmenta, i, en conseqüència, l’oferta disminueix. Això és
reflecteix en les equacions de l’enunciat:

D(Px )  1500 100Px P


x
4
 D(Px )  1100
700(Px 1) P 4
O(Px )  
x
 O(Px )  700
3

3. Quan el preu d’un producte, per exemple el pa, està intervingut, es pot aplicar la llei de l’oferta i
la demanda?

No, ja que en aquest cas, el preu del producte no depèn directament ni de l’oferta ni de la demanda,
sinó d’altres factors.

4. Resol de manera gràfica el sistema que varia l’equació d’oferta perquè el preu de mercat:

a) Pugi a 6 €.

b) Baixi a 5 €.

a) b)

 
   

245
 
 
 
 
 
 

5
 
 
 
 
 
 

Trigonometria
 

 
Trigonometria   
 
 
 
 
 

ACTIVITATS 
 
1. Determina l’equivalència en radians d’aquests angles:
a) 10º b) 135º c) -60º

2 x 20 
a)  x  rad
360 10 360 18
2 x 270 3
b)  x  rad
360 135 360 4
2 x 120 
c)  x  rad
360 60 360 3

2. Calcula la mida en graus dels angles següents:


૛࣊ ସ஠
a) rad b) 3 rad c) - rad 
૜ ଷ
360 x 3602
a)  x  120o
2 2 6
3
360 x 3603
b)  x  171,88o
2 3 2
360 x 360 4
c)  x  240o
2 4 6

3

3. Calcula les raons trigonomètriques dels triangles aguts següents:

 
60 60 109
a) sin  tg  sec 
109 91 91
91 109 91
cos  cosec  cotg 
109 60 60
91 91 109
sin= tg  sec 
109 60 60
60 109 60
cos  cosec  cotg 
109 91 91

247
 
 
 
 
 
 

Trigonometria
Trigonometria 4
 
 
 
 
 
 

8 8 17
b) sin sin  tg tg   sec sec 
17 15 15
15 17 15
  cos cos           cosec  cosec         cotg cotg    
17 8 8
15 15 17
  sin=sin            tg tg            sec sec   
17 8 8
8 17 8
  cos cos           cosec cosec        cotg cotg   
17 15 15
4. Demostra que es compleixen les igualtats següents:
1
a) sec  
cos 
1
b) cosec  
sin 
1 cos 
c) cotg
  
tg  sin 
a 1 1
a) sec   
c c cos
a
a 1 1
b) cosec   
b b sin
a
c 1 1 1 cos
c) cotg     
b b tg  sin sin
c cos

5. Calcula les raons trigonomètriques de l’angle a si:

7 2
a) cos   c) sin  
25 2
b) tg  1,67 d) tg   0,3

2
 7  24
a) cos2   sin2  1  sin  1   
 25  25

24 25 25 7
tg  cosec  sec  cotg 
7 24 7 24

2 1 1
b) 1 tg   2
 cos   0,51
cos  11,672
sin tgcos 1,670,51  0,85
cotg 0,60 cosec 1,18 sec 1,96

248
 
 
 
 
 
 

Trigonometría 54
 
 
 
 
 
 

2
 
2 2
c) cos  1    tg 1 cosec  2 sec  2 cotg 1
 2  2
 

1
cos   0,96 sin tgcos 0,30,96  0,29
d) 1 0,32
cosec 3,45 sec 1,04 cotg 3,33

6. Justifica si hi ha cap angle per al qual es verifiqui que:


a) sin  = 0,3 i cos  = 0,8
b) sin  = 0,72 i tg  = 1,04
c) cos  = 0,1 i sin  = 0,99
a) No existeix, ja que no verifica les relacions trigonomètriques.

sin2   cos2  1  0,32  0,82  0,73  1


b) Sí que existeix, ja que verifica les relacions trigonomètriques.

0,72
Calculem el cosinus: 1,04   cos 0,69  0,722  0,692  1
cos
c) Sí que existeix, ja que verifica les relacions trigonomètriques.

0,12  0,992  1

7. Calcula el valor de les expressions següents:


a) cos 30º - sin 60º + tg 45º
b) cos2 60º - sin2 45º
c) tg 60º + sin 45º - cos2 30º
d) tg 30º + tg 60º - sin 30º cos 30º

3 3
a) cos30  sin60 tg 45   1  1
2 2
2
1   2 
2

b) cos 60 sin 45    


2 2   1

 2   2  4

2
 
2 2  3  3 4 3 2 2
c) tg60 sin45  cos 30  3   
2  2  4

3 1 3 13 3
d) tg30 tg60 sin30cos30   3  
3 2 2 12

8. Determina aquests valors amb la calculadora:

a) cos 79º b) sin 43,5º c) tg 10º 28’



a) cos79o  0,19 b) sin43,5o  0,69 c) 10o28'  10,46  tg10o28'  0,18

249
 
 
 
 
 
 

Trigonometria
Trigonometria 4
 
 
 
 
 
 

9. Troba aquestes raons trigonomètriques amb la calculadora:

a) sin (0,35 rad) b) cos (1 rad) c) tg (1,27 rad)

a) sin 0,35rad  0,34 b) cos1 rad  0,54 c) tg 1,27 rad  3,22

10. Indica el signe de les raons trigonomètriques d’aquests angles i identifica el quadrant on són:

a) 66º c) 175º e) 342º

b) 18º d) 135º f) 120º

a) És al 1r quadrant; totes les raons trigonomètriques són positives.

b) És al 1r quadrant; totes les raons trigonomètriques són positives.

c) És al 2n quadrant; el sinus i la cosecant són positius, i la resta de les raons trigonomètriques són
negatives.

d) És al 2n quadrant; el sinus i la cosecant són positius, i la resta de les raons trigonomètriques són
negatives.

e) És al 4t quadrant; el cosinus i la secant són positius, i la resta de les raons trigonomètriques són
negatives.

f) És al 2n quadrant; el sinus i la cosecant són positius, i la resta de les raons trigonomètriques són
negatives.

11. Ordena de més petit a més gran els cosinus dels angles següents sense calcular-los:

34º 72º 98º 160º 251º 345º

S’ordenen tenint en compte que el cosinus és positiu en angles del primer i del quart quadrants:

cos160  cos251  cos98  cos72  cos34  cos345

12. Justifica la resposta.

a) Per què no existeix tg 90º?

b) Passa el mateix amb tots els angles que són múltiples de 90º?

No existeix, perquè cos 90º = 0

Si multipliquem 90º per un nombre parell, la tg és cero, perquè el sinus és 0 i el cosinus és 1.

Si multipliquem 90º per un nombre imparell, la tangent no és definida, perquè el cosinus és 0.

250
 
 
 
 
 
 

Trigonometría 54
 
 
 
 
 
 

13. Ordena de més petita a més gran les tangents dels angles següents sense calcular-les:

65º 110º 170º 210º 315º

Les tangents del primer i del tercer quadrants són positives, les del segon i del quart són negatives.

tg110o  tg315o  tg170o  tg210o  tg65o

14. Si saps que cos 50° = 0,6428 troba les raons trigonomètriques d'aquests angles.

a) 130º b) 230º c) -50º d) 310º

2 2
Calculem el sinus de 500: sin 50 0,6428  1  sin50 0,766

cos50 cos130  0,6428 sin50  sin130  0,766 tg130  1,1918


a)

sec130  1,5557 cosec130 1,3054 cotg130  0,8391

cos50 cos230  0,6428  sin50  sin230  0,766 tg230  1,1918


b)

sec230  1,5557 cosec230  1,3054 cotg230  0,8391

cos50 cos(50)  0,6428  sin50  sin(50)  0,766 tg(50)  1,1918


c)

sec(50)  1,5557 cosec(50)  1,3054 cotg(50)  0,8391

d) cos50 cos310 0,6428  sin50  sin310  0,766 tg310  1,1918

sec310 1,5557 cosec310  1,3054 cotg310  0,8391

15. Si saps que sin 25° = 0,4226 troba les raons trigonomètriques dels angles següents.

a) 745º b) 565º c) 1055º d) 1235º

a) 745o  2360o 25o  sin745o  0,4226

cos745o  0,9063  tg745o  0,4663

b) 565o  360o 180o 25o  sin565o  0,4226

cos565o  0,9063  tg565o  0,4663

c) 1055o  3360o 25o  sin1055o  0,4226

cos1055o  0,9063  tg1055o  0,4663

d) 1235o  3360o 18025o  sin1235o  0,4226

cos1235o  0,9063  tg1235o  0,4663

251
 
 
 
 
 
 

Trigonometria
Trigonometria 4
 
 
 
 
 
 

16. Si saps que ‫ ࢻ ܖܑܛ‬ൌ , calcula:

a) sin ( 90º–  ) b) sin (180º–  ) c) sin (–  )

1
a)  sin cos(90 )
5
Substituïm en l’expressió per calcular sin(180 ) :
1 2 6
cos2 (90 ) sin2 (90  )  1  sin2 (90  )  1  
25 5
1
b) sin(180 )  sin 
5
1
c) sin()  sin  
5

17. Si sin 18° = 0,309 i cos 18° = 0,951; calcula:

a) sin 72º b) cos 162º c) tg(-72º)

a) sin72 sin(9018)  cos18 0,951

b) cos162 cos(18018)  cos18 0,951

1 cos18 0,951
c) tg(72)   tg72   tg(9018)      3,0777
tg18 sin18 0,309

18. Donats els angles següents, contesta les preguntes que tens a continuació:

25º 65º 115º 155º -25 -65º

a) Quins angles tenen el mateix sinus? I el mateix cosinus?

b) Quins angles tenen igual la tangent?

o o
a) sin25  sin155 sin65o  sin115o cos65o  cos(65o ) cos25o  cos(25o )

o o
b) tg115  tg(65) tg155  tg(25)

19. A partir de les raons de 45º i 60º, calcula les raons trigonomètriques de 105º i 15º.

3 2 1 2
sin(60o  45o )  sin60o cos45o  cos60o sin45o      0,9659
2 2 2 2

1 2 3 2
cos(60o  45o )  cos60o cos45o  sin60o sin45o      0,2588
2 2 2 2

tg60o  tg45o 3 1
tg (60o  45o )    3,7321
1 tg60o tg45o 1 3

252
 
 
 
 
 
 

Trigonometría 54
 
 
 
 
 
 

3 2 1 2
 
sin 60o  45o  sin60o cos45o  cos60o sin45o  
2 2 2 2
 0,2588

1 2 3 2
cos(60o  45o )  cos60o cos45o  sin60o sin45o    0,9659
2 2 2 2

tg60o  tg 45o 3 1
tg (60o  45o )  o o
  0,2679
1 tg60 tg45 1 3

20. Calcula les raons trigonomètriques de 76º i 19º si saps que cos 38° = 0,788 i sin 38° = 0,6157.

cos76o  cos(238o )  cos2 38o  sin2 38o  0,2419


sin76o  sin(238o )  2sin38o cos38o  0,9703
0,6157 2tg38o
tg38o   0,7813  tg76o  tg(238o )   4,011
0,788 1  tg2 38o

38o 1 cos38o
cos19o  cos   0,9455
2 2
38o 1 cos38o
sin19o  sin   0,3256
2 2
38o 1  cos38o
tg19o  tg   0,3443
2 1 cos38o

21. Resol les equacions trigonomètriques següents en l’interval [0º, 360º]:


a) 5 sin x = 2 c) 5 tg x = 12
b) 7 cos x = –1 d) 2 tg x = 2

2  x  2334'41,44"
a) 5sin x  2  sin x    1
5  x2  15625'18,56"

1  x  9812'47,56"
b) 7cos x  1  cos x     1
7  x2  26147'12,44"

12  x1  6722'48,49"
c) 5tg x  12  tg x  
5  x2  24722'48,49"

 x  45
d) 2tg x  2  tg x  1   1
 x2  225
22. Resol aquestes equacions trigonomètriques i simplifica’n el resultat:

a) cos 2x = 1 c) sin 2x – cos x = 0

b) cos 2x + sin x = 1 d) 2 tg 4x = 1

a) cos2x 1  x  k amb k  

b) cos2x  sinx  1  cos2 x  sin2 x  sinx  1  2sin2 x  sinx  0

253
 
 
 
 
 
 

Trigonometria
Trigonometria 4
 
 
 
 
 
 

 5
x1  k amb k   x2  2k amb k   x3  2k amb k  
6 6
c) sin2x  cos x  0  2sinx cos x cos x  cos x (2sinx 1)  0

(2k 1)  5
x1  amb k   x2  2k amb k   x3  2k amb k  
2 6 6
1 26,56o 180o k
d) 2tg4x  1  tg4x   4x  26,56o 180o k  x  amb k  
2 4

23. D’un triangle sabem que els catets fan 7 m i 24 m. Calcula’n la hipotenusa i l’amplitud dels
angles.

Calcula la hipotenusa utilizant el Teorema de Pitàgores:

h 242  7 2 25 m

7 24
 arcsin
 16, 26o  arcsin
 73, 74o
25 25

24. D’un triangle rectangle,  , sabem que = 62° i que la hipotenusa a fa 1 m. Troba'n els
elements.

Apliquem la relació d’angles complementaris per calcular el tercer angle:

�� � ��� � ��� � ���

Utilitzem una de les seves raons trigonomètriques, trobem l’altre dels seus costats:


��� � � � � �� � � � ��� ��� � ��������

Amb el Teorema de Pitàgores determinem el tercer costat:

� � �1� � ������� � ��������

25. Calcula b i c en els triangles següents:

254
 
 
 
 
 
 

Trigonometría 54
 
 
 
 
 
 

 
Apliquem el teorema del sinus com a l’apartat anterior:
b = 25,85 cm c = 19,85 cm

26. Justifica si poden ser certes les igualtats següents:


� �
a) � � �
� �
����� �����

b) � � � � �
�����

c) � � � � �

a) És possible si sin 
A  1 , és a dir, si 
A = 90  .
c  c 
b) No és possible perquè 
a A 90o i b 
 B 90 o , però no existeix cap triangle amb
sinC sinC
dos angles rectes.
  
c) És possible si A = B = C = 60  .

27. Calcula la longitud del costat desconegut.

a
a)  122  122  2 12 12 cos 38
o
7,81 cm

a
b)  102  162  2 10 16cos 48
o
11,91 cm

28. Determina si les mides següents corresponen a les longituds dels costats d’un triangle i indica
si és acutangle, rectangle o obtusangle:

a) 12, 11 i 9 cm c) 26, 24 i 10 cm

b) 23, 14 i 8 cm d) 40, 30 i 20 m

a) �� � �� � � � � ��� � ��� � →���� � ��� � �� � � � �� � � ��� �

→ ��� � � �� ����� → �� � � ������ ��� ��� →El triangle és acutangle

b) Les mides no formen un triangle, perquè la suma dels costats menors és mes petita que el costat
més gran.

c) �� � �� � � � � ��� � ��� � →���� � ��� � ��� � � � �� � �� ��� �

255
 
 
 
 
 
 

Trigonometria
Trigonometria 4
 
 
 
 
 
 

→ ��� � � � → � � ��� →El triangle és rectangle.

d) �� � �� � � � � ��� � ��� � → ��� � ��� � ��� � � � �� � �� ��� �

→ ��� � � �� �� → � � ����������� →El triangle és obtusangle.

29. Resol el triangle següent:

132 102  42  2 10  4 cos 


A 
A 131, 49o
B
102 132  4 2  2 13  4 cos B  35,18o

180o  131, 49o  35,18o 


13,33o

30. Resol el triangle següent:

a
 152  92  2  9 15cos 71
o
14, 77 cm

Aplica el teorema del sinus:

14,77 9  35,18o
 B
o
sin71 sinB

  180o  71o  35,18o  73,82o


B

31. Resol un triangle sabent que dos dels costats fan 14 cm i 18 cm, i que l’angle oposat a un dels
costats és de 70º. Dibuixa el triangle.
Apliquem el teorema del sinus per calcular l’angle oposat al costat conegut:
� � � �� ��
� � → � → � � ������ ��� ���
� ��� �
��� � � ��� �
� ��� �
� ��� ���
La suma dels angles d’un triangle és 180º, així trobem el tercer angle:
� � � � � � ���� → � � ������ ��� ��� � ��� � ���� → � � ����� ��� ���

256
 
 
 
 
 
 

Trigonometría 54
 
 
 
 
 
 

Utilitzem el teorema del sinus per trobar el tercer costat:


� � � � ��
� � → � → � � ��� �� ��
� ��� �
��� � � ��� �
� ��� ���� ��� ��� ��� ���

32. Resolem el triangle a = 4 m, c = 6 m i = 25°, i obtenim com a solucions dos triangles


obtusangles. Comprova que això és possible i dibuixa’n les solucions.

Utilitzem el teorema del sinus per calcular l’angle oposat al costat conegut:

� � � � � � � ������ ��� ���


� � → � →�
� ��� �
��� � � ��� �
� ��� ��� ��� � � � �����������

Si els angles d’un triangle sumen 180 º:

� � � � � � ���� → ��� � � � ������ ��� ��� � ���� → � � ������� ����

1a solució: � � � � � � ���� → ��� � � � ������ ��� ��� � ���� → � � ������� ����

� � � � � � ���� → ��� � � � ����������� � ���� → � � ������ ����


2a solució:�

Utilitzem el teorema del sinus per calcular l’angle oposat al costat conegut:

1a solució:
� � � � �
� � → � → � � �� �� ��
� ��� �
��� � � ��� �
� ��� ������� ���� ��� ���

2a solució:
� � � � �
� � → � → � � �� �� ��
� � �
��� � ��� � ��� � ��� ����� � ���� ��� ���

               
 

   

257
 
 
 
 
 
 

Trigonometria
Trigonometria 4
 
 
 
 
 
 

SABER FER 
33. Calcula el sinus, el cosinus i la tangent de l’angle ∝.

a) cos = amb 270º < < 360º b) tg = -2 amb 90º < < 180º
2
1 0,9682
a) sin  1  cos 2   1   0,9682 tg   3,8728
4 0, 25

1
b) cos  
 0, 4472
 sin 1  cos 2  
0,8944
2
1  tg 

34. Calcula els cosinus següents mitjançant les raons trigonomètriques de 50º:

a) cos 40º c) cos 310º e) cos 5º g) cos 25º

b) cos 130º d) cos 80º f) cos 100º

cos50o  0,6428 sin50o  0,7660 tg 50o  1,1918

o
 o o

a) cos 40 cos 90  50  cos 90 cos 50  sen90 sin50 0, 7660
o o o o

b) cos130 
o

cos 180  50  o o
cos180 cos 50  sin180 sin50  0, 6428 o o o o

c)  cos  360  50


cos 310 o
 cos 360 cos 50  sin360 sin50 0, 6428
o o o o o o

d) cos 80 cos  30  50 


o
 cos 30 cos 50  sin30 sin50 0,1736
o o o o o o

e) cos 5 cos  50  45 
o
 cos 50 cos 45  sin50 sin45 0, 9962
o o o o o o

f) cos100  cos  2  50  
o o
cos 50  sin 50  0,1736 2 o 2 o

50o 1  cos 50o


g) cos
 25o cos
  0,9063
2 2

35. Resol les equacions següents:


a) sin (2x + 5º) = 1 b) cos (2x + 5º) = 1

85o
a) sin  90  k  360   1  2 x  5  90  k  360  x 
o o o o o
 k 180o amb k 
2
5o
b) cos  k  360   1  2 x  5  k  360  x  k 180 
o o o o
amb k  
2

36. Resol les equacions següents:

a) sin2x  2sinx 
0 b) sin2 x  sinx 
0

258
 
 
 
 
 
 

Trigonometría 54
 
 
 
 
 
 

sin 2 x  2 sinx  0  2 sinx  cos x  1 


0  2 sinx cos x  2 sinx  0
a)

x1 k amb k  x2  2k amb k 

sin 2 x  sinx  0  2 sinx cos x  sinx  0  sinx  2 cos x  1  0


b)
2
x1 k amb k  x2   2k amb k  
3
4
x3   2k amb k  
3

37. Resol aquesta equació:

cos 2 x  3cos x  2 0
cos 2 x  sin 2 x  3 cos x  2  0  2 cos 2 x  3 cos x  1 0
1
3 9 8   
cos x   1
 x
1  2k x2   2 k x3  2k
4  2 3 3

38. Resol l’equació següent:


cos 3x + cos x = 0

0   cos 2 x  sin 2 x  cos x  2 sin x cos x sin x  cos x 


cos 2 x cos x  sin 2 x sin x  cos x  0

2 cos 3 x  2 cos x  2 cos3 x  0  2 cos x  2 cos 2 x  1  0

x1
 2k 1  amb k 
 
x2  k amb k  
2 4 2

39. En Joan vol saber l’amplada d’un riu sense haver-se de desplaçar a l’altra riba. Pren mides
amb els seus passos i arriba a la situació següent:

 
3 a
tg    a  15 m
5 25

259
 
 
 
 
 
 

Trigonometria
Trigonometria 4
 
 
 
 
 
 

40. Calcula l’àrea d’aquest triangle:

h 3 bh 3 3
sin 60o   h 3 Àrea
  =2,60cm2
2 2 2 2
41. Calcula l’altura d’un triangle de base 100 cm i amb els angles adjacents = 45° i = 60°.

h 
tg 45o
  1
100  x  100
  100  x 3 x  x  36, 6 cm
h  3 1
tg 60o  3
x 
h  63, 4 cm

42. Determina l’àrea d’un pentàgon regular de 15 cm de radi.

o l/2
sin 36 l 17, 64 cm

15
152   l / 2   ap 2  ap  12,13 cm
2

p  ap 88,167 12,13
Àrea
   534,86 cm 2
2 2

43. Dos vaixells surten simultàniament d’un port amb rumbs que formen un angle de 82º. El primer
navega a una velocitat de 18 milles per hora, i el segon a 25 milles per hora. Si mantenen els
rumbs inalterats i l’abast dels seus equips de ràdio és de 180 milles, es podran mantenir en
contacte al cap de 3 hores?

El primer recorre 54 milles i el segon 75.

a 542  752  2  54  75  cos82o 86,1 milles

Com 86,1<180 si podran mantenir el contacte.

44. Calcula la longitud dels costats que falten en el trapezi rectangle següent:

260
 
 
 
 
 
 

Trigonometría 54
 
 
 
 
 
 

Afegint un triangle amb els angles de 40º, 70º i 70º.


3 y x
  
 y 3 cm
, x 2, 05 cm
sin 70 o
sin 70 o
sin 40o
 
ACTIVITATS FINALS 
 
45. Determina l’equivalència en radians d’aquests angles donats en graus sexagesimals:
a) 35º c) 84º 12’
b) 185º d) 62º 25’
7 o 42,1
a) rad c) 84, 2  rad=1,47 rad
36 90
37 o 31, 21
b) rad d) 62, 42  rad=1,09 rad
36 90

46. Calcula l’equivalència en graus d’aquests angles donats en radians:

a) 1,5 rad c) 1,5 π rad

��
b) 3,5 rad d) rad

a) 85,94o c) 270o

b) 200,54o d) 216o

47. En els triangles rectangles ABC següents, A és el vèrtex de l’angle recte. Tenim les mides de la
hipotenusa, a, i dels catets, b i c. Contesta les qüestions que hi ha plantejades.

a) Troba sin i tg si b = 12 cm, c = 16 cm.

b) Troba cos i cotg si a = 30 cm, c = 22 cm.

c) Troba cos i cosec si a = 3 cm, b = 2 cm.

d) Troba cos i tg si b = 20 cm, c = 20 cm.

b b
a) a  12 2  16 2  20 cm sin B  0, 6 tgB  0, 75
a c
c 11 c 11
b) b  30 2  22 2  416 cm cos B  cotg B 
a 15 b 104

261
 
 
 
 
 
 

Trigonometria
Trigonometria 4
 
 
 
 
 
 

b 2 a 3
c) c  32  2 2  5 cm cos C   cos ecC 
a 3 c 5
c 20 2 b
d) a  202  202  800 cm cos B   tgB  1
a 800 2 c

෡ i࡮
48. Dibuixa dos angles ࡭ ෡ que verifiquin que:

2
sin Aˆ  tgBˆ  5
2
Resposta oberta. Per exemple:

 
       
49. Sense fer servir la calculadora, determina el valor més simplificat possible de les expressions
següents:

a) sin 30º + sin 60º

஠ ஠
b) tg + sec
ଷ ଺

c) cotg 90º – cotg 30º

d) cosec 60º – cos 60º

1 3 1 3 1
a)   c) 0   3
2 2 2 3
3
1 2 3 2 5 1 1 4 3
b) 3  3   d).  
3 3 3 3 3 2 2 3
2 2

50. Sense fer servir la calculadora, indica si les afirmacions següents són certes o falses:

a) sin 30º + sin 60º = sin 90º c) sin 90º = 2 sin 45º

b) cos 30º + cos 60º = cos 90º d) cos 90º = 2 cos 45º

262
 
 
 
 
 
 

Trigonometría 54
 
 
 
 
 
 

1 3 2
a)   1  Falsa c) 1  2  Falsa
2 2 2

3 1 2
b)   0  Falsa d) 0  2  Falsa
2 2 2
51. Calcula la cosecant, la secant i la cotangent:

a) De l’angle de 30°. c) De l’angle de 60°.

b) De l’angle de 45°. d) De l’angle de 90°.

2
a) cos ec 30 o  2 s ec30o  cot g 30o  3
3

b) cos ec45o  2 sec45o  2 cot g 45o  1

2 3
c) cos ec60o  s ec 60 o  2 cot g 60o 
3 3
d) cos ec 90 o  1 La secant no existeix cotg90o   0

52. Els angles , i són aguts. Completa la taula següent a la llibreta sense arribar a
determinar els angles:

53. Sinus Cosinus Tangent


sin = 0,5602
cos = 0,1849
tg = 2,7804
 
 
 
Sinus Cosinus Tangent
sin = 0,5602 cos = 0,8284 tg = 0,6763
sin = 0,9828 cos = 0,1849 tg = 5,3151
sin = 0,6616 cos = 0,3384 tg = 2,7804

෡ ൌ ૚ ՜ ‫࡭ ܛܗ܋‬
૙ǡ ૞૟૙૛૛ ൅ ‫ܛܗ܋‬૛ ࡭ ෡ ൌ ඥ૚ െ ૙ǡ ૞૟૙૛૛ ൌ ૙ǡ ૡ૛ૡ૝

૙ǡ ૞૟૙૛
෡ൌ
‫࡭ ܏ܜ‬ ൌ ૙ǡ ૟ૠ૟૜
૙ǡ ૡ૛ૡ૝

෡ ൅ ૙ǡ ૚ૡ૝ૢ૛ ൌ ૚ ՜ ‫࡮ ܖܑܛ‬
‫ܖܑܛ‬૛ ࡮ ෡ ൌ ඥ૚ െ ૙ǡ ૚ૡ૝ૢ૛ ൌ ૙ǡ ૢૡ૛ૡ

૙ǡ ૢૡ૛ૡ
෡ൌ
‫࡮ ܏ܜ‬ ൌ ૞ǡ ૜૚૞૚
૙ǡ ૚ૡ૝ૢ

263
 
 
 
 
 
 

Trigonometria
Trigonometria 4
 
 
 
 
 
 


෡ൌඨ
‫࡯ ܛܗ܋‬ ൌ ૙ǡ ૜૜ૡ૝
૚ ൅ ૛ǡ ૠૡ૙૝૛

෡ ൅ ૙ǡ ૜૜ૡ૝૛ ൌ ૚ ՜ ‫࡯ ܖܑܛ‬
‫ܖܑܛ‬૛ ࡯ ෡ ൌ ඥ૚ െ ૙ǡ ૜૜ૡ૝૛ ൌ ૙ǡ ૟૟૚૟

53. Utilitza la calculadora per determinar els angles aguts que compleixen que:

a) cos = 0,3453 e) tg = 0,3554

b) tg = 2,3688 f) sin = 0,0968

c) cosec = 1,9044 g) sin = 0,2494

d) cos = 0,9726 h) cotg = 2,5

a) cos = 0,3453

b) tg = 2,3688

c) cosec = 1,9044

d) cos = 0,9726

e) tg = 0,3554

f) sin = 0,0968

g) sin = 0,2494

h) cotg = 2,5

54. Determina les raons trigonomètriques següents:

a) sin 19° 22' 37" g) tg 83° 41' 57"

b) cos 44° 52' h) sin 37° 25"



c) cos 1,03 i ) tg

૛࣊
d) ‫ܖܑܛ‬ j ) cos 0,845

e) sec 54° 28' k) cotg 35° 40'



f ) cosec ૈ l ) sec

264
 
 
 
 
 
 

Trigonometría 54
 
 
 
 
 
 

a) sin 19º 22’ 37” = 0,3318 g) tg 83º 41’ 57” = 9,0567

b) cos 44º 52’ = 0,7088 h) sin 37º 25’ = 0,6019

c) cos 1,03 = 0,5148 i) tg = 0,4142

��
d) ��� = 0,9511 j) cos 0,845 = 0,6637

e) sec 54º 28’ = 1,7206 k) cotg 35º 40’ = 1,3934

f) No és definida l) sec = 1,1547

55. Resol els triangles rectangles corresponents considerant que és l’angle recte.

a) b = 7 m, = 48º d) a = 6 cm, = 42º 12’

b) c = 12 m, = 28º e) b = 3 m, c = 6 m

c) a = 13 m, c = 5 m f) b = 8 m, a = 10 m

a) Apliquem la relació d’angles complementaris, i calculem el tercer angle:


� = 90º - 48º = 42º
Amb una de les seves raons trigonomètriques trobem l’altre dels seus costats:
b a 7
   a  a  9, 42 m
sin B sin A sin 48º
Amb el Teoremes de Pitàgores obtenim el tercer costat:

c 9, 42 2 
7 2 6,3m
b) Apliquem la relació d’angles complementaris, i calculem el tercer angle:
� = 90º - 28º = 62º
Amb una de les seves raons trigonomètriques trobem l’altre dels seus costats:
b c b 12
   b 6,38 m

sin B sin C sin 28º sin 62º
Amb el Teoremes de Pitàgores obtenim el tercer costat:

a  12 2  6, 382 13, 59 m
c) Apliquem el Teoremes de Pitàgores, i calculem el tercer costat:

b 132  52 12m


12
��� �� � � �� � ��� 22� 48,5′′
13

��� �� � � �� � 22� 3�� 11,5′′
��

d) Apliquem la relació d’angles complementaris, i calculem el tercer angle:

C� = 90º - 42º12’ = 47º 48’

265
 
 
 
 
 
 

Trigonometria
Trigonometria 4
 
 
 
 
 
 

Amb una de les seves raons trigonomètriques trobem l’altre dels seus costats:
b a b
   6  b  4, 44 m
sin B sin A sin 47º 48 '
Amb el Teoremes de Pitàgores obtenim el tercer costat:

c  6 2  4, 44 2 4, 04m
e) Apliquem el Teoremes de Pitàgores, i calculem el tercer costat:

a 32  6 2  6, 71m
3
��� �� � � �� � 26� 33� 26,6′′
6,71

��� �� � � �� � 63� 26� 33,4′′
�,��
f) Apliquem el Teoremes de Pitàgores, i calculem el tercer costat:

c 10 2  82 6m
8
��� �� � � �� � �3� 7� 48,37′′
10

��� �� � � �� � 36� �2� 11,63′′
��

56. Una barca està lligada a la riba d’un canal amb una corda que fa 8 m. En un moment
determinat, aquesta corda forma un angle de 38º amb la vora. A quina distància de la riba es
troba la barca?

Distància = 8 sin 38º = 4,93 m

57. Si ens trobem a 40 m de la xemeneia d’una fàbrica i la veiem sota un angle de 26º, quina altura
té? Considera que els ulls de l’observador estan situats a 175 cm del terra.


�� ��� � � � � ��, �� �
��

19,51 +1,75 = 21,26 m

L’altura de la xemeneia és de 21,26 m.

58. Calcula l’àrea del quadrat interior.


266
 
 
 
 
 
 

Trigonometría 54
 
 
 
 
 
 

 
8
cos 30o   l  9, 24 cm Àrea l 2 85,3 cm2
l

59. Determina la longitud de l’apotema d’un hexàgon regular de 4 cm de costat.


L’hexàgon regular es pot dividir en dotze triangles rectangles.

360o
Calculem l’angle central:  30o
12
ap
cos 30o   ap  3, 46 cm
4

60. Un pentàgon regular està inscrit en una circumferència de 20 cm de radi. Determina la mida del
costat.

El pentàgon regular pot dividir en cinc triangle isòscels.

���
Calculem l’angle central: � ��°

Els restants angles del triangle:

180º = 72º + 2 � � � � � ��°

Apliquem el Teorema del sinus:

b a b 20
   6 
b 23, 51 m
sin B sin A sin 72º sin 54º
La mida del costat és de 23,51 cm.

61. Escriu de més petit a més gran els cosinus dels angles següents sense calcular-los:

55° 110° 165° 220° 275° 330°

Els cosinus són negatius en el segon i tercer quadrants.

cos165o  cos 220 o  cos110 o  cos 275o  cos 55o  cos 330 o

62. Escriu de més petit a més gran els sinus dels angles següents sense calcular-los:

267
 
 
 
 
 
 

Trigonometria
Trigonometria 4
 
 
 
 
 
 

45° 120° 135° 200° 225° 310°

 
Els sinus són negatius en el tercer i quart quadrants.

sin 310 o  sin 225o  sin 200 o  sin 45o  sin135o  sin120 o
63. La taula següent mostra les raons trigonomètriques d’angles de quadrants diferents. Sense
determinar-los, completa-la a la llibreta amb les raons que falten.

Quadrant sin cos Tg


Segon 0,6702
Tercer -0,4539
Quart -0,7459
Tercer -0,7822
Segon -1,9004
Quart 0,6983

Quadrant sin cos Tg


Segon 0,6702 -0,7422 -0,903
Tercer 0,8911 -0,4539 -1,9631
Quart 0,8016 -0,5979 -0,7459
Tercer -0,7822 -0,623 1,2555
Segon 0,8849 -0,4657 -1,9004
Quart -0,7158 0,6983 -1,0251

0,6702� � ��s� �� � 1 � ��s �� � �1 � 0,6702� � �0,7�22


0,6702
�� �� � � �0,903
�0,7�22
sin� �� � ��0,��39�� � sin �� � �1 � 0,��39� � 0,8911
0,8911
�� �� � � �1,9631
�0,��39

1
��s �� � � � 0,8016
1 � ��0,7��9��

sin� �� � 0,8016 � 1 � sin �� � �1 � 0,8016� � �0,�979

� � 1 � ��s �
��0,7822�� � ��s � � � � �1 � 0,7822� � �0,623

�0,7822 1
�� �� � � 1,2��� ��s �� � � � �0,�6�7
�0,623 1 � ��1,900���

���� �� � ��0,�6�7�� � 1 � sin �


� � �1 � 0,6�7� � 0,88�9

���� �� � ��0,6983�� � 1 � sin �


� � �1 � 0,683� � �0,71�8
�0,71�8
�� �� � � �1,02�1
0,6983

64. Sense fer servir la calculadora, calcula el valor del sinus, el cosinus i la tangent dels angles
següents mesurats en radians:
268
 
 
 
 
 
 

Trigonometría 54
 
 
 
 
 
 

a) e) i)

b) f) j)

c) g) k)

d) h) l)

a) sin    
0 cos    
1 tg    
0

   
b) sin    
1 cos    
0 La tangent no existeix.
 2  2

  1   3   3
c) sin    
 cos     tg    

 6 2  6 2  6 3

  2   2  
d) sin    cos    tg    1
4 2 4 2 4

  3   1  
e) sin    cos    tg    3
3 2 3 2 3

   
f) sin    1 cos    0 La tangent no existeix.
2 2

 3   3 
g) sin    1 cos   0 La tangent no existeix.
 2   2 

h) sin     0 cos    1 tg    0

 3   3 
i) sin    1 cos  0 La tangent no existeix.
 2   2 

j) sin  2   0 cos  2   1 tg  2   0

 7   7 
k) sin    1 cos  0 La tangent no existeix.
 2   2 

 9   9 
l) sin   1 cos  0 La tangent no existeix.
 2   2 

269
 
 
 
 
 
 

Trigonometria
Trigonometria 4
 
 
 
 
 
 

65. Sense fer servir la calculadora, determina el valor del sinus, el cosinus i la tangent dels angles
següents mesurats en graus:

a) –60º e) 120º i) 210º

b) –45º f) 135º j) 225º

c) –30º g) 150 k) 240º

d) –30º h) 180º l) 270º

3 1
a) sin  60o  
 cos  60 o   tg  60o    3
2 2
2 2
b) sin  45o  
 cos  45o   tg  45o   1
2 2
1 3 3

c) sin 30
o

 cos  30o   tg  30o  

2 2 3
d) sin 0 0
o
cos  0o   1 tg  0o   0

3 1
e) sin 120o   cos 120o    tg 120o    3
2 2
2 2
f) sin 135o   cos 135o    tg 135o   1
2 2
3 3

g) sin 150
o
  12 cos 150o   
2
tg 150o   
3

h) sin 180
o
0 cos 180 o   1 tg 180o   0

1 3 3
i) sin  210o    cos  210o    tg  210o  
2 2 3
2 2
j) sin  225o    cos  225o    tg  225o   1
2 2
3 1
k) sin  240o    cos  240 o    tg  240 o   3
2 2
l) sin  270 o   1 cos  270 o   0 La tangent no existeix.

270
 
 
 
 
 
 

Trigonometría 54
 
 
 
 
 
 

66. Utilitza la calculadora per trobar aquestes raons:

a) sin 319º 12’ 52” g) tg 183º13’53”

b) cos 434º26’ h) sin 333º55’’

 11 
c) tg 7,03 i) tg  
 8 

 8 
d) sin   j) cos 3,845
 5 

 11 
e) Cosec 200º 16’ k) cotg  
 6 

 5 
f) sec   l) cosec 5,24
 4 

a) sin 319º 12’ 52” = - 0,6532 g) tg 183º13’53” = 0,0565

b) cos 434º26’ = 0,2684 h) sin 333º55’’ = - 0,4538

 11 
c) tg 7,03 = 0,9257 i) tg   = 2,4142
 8 

 8 
d) sin   = -0,9511 j) cos 3,845 = -0,7626
 5 

 11 
e) Cosec 200º 16’ = -2,8869 k) cotg   = -1,7321
 6 

 5 
f) sec   = -1,4142 l) cosec 5,24 = -1,1574
 4 

67. Calcula aquestes raons trigonomètriques relacionant-les amb les d’un angle del primer
quadrant:

a) 301º 21’ 15’’ c) 190º 43’’ o


e) 386 56’

b) 902º 40’ o
d) 295 12’ 45’’
o
f) 612 43’ 2’’

 360º   58º 38’ 45’’


a) 301º 21’ 15’’

sin  301º 21’ 15’’ 


 sin  58º 38’ 45’’ 0,8540

cos  301º 21’ 15’’  cos  58º 38’ 45’’  0, 5203

271
 
 
 
 
 
 

Trigonometria
Trigonometria 4
 
 
 
 
 
 

tg  301º 21’ 15’’  tg  58º 38’ 45’’  1, 6412

b) 902º 40’ 
2  360º 180º 2º 40’

 sin  2o 40’ 
sin  902º 40’  0, 0465

 cos  2o 40’ 
cos  902º 40’  0, 9989

tg  902º
  2o 40’ 0, 0466
40’ tg

c) 190º 43’’
 180º 10º 43’’

sin 190º 43’’   sin 10 o 43’’  0,1739

cos 190º 43’’   cos 10o 43’’  0, 9848

10o 43’’ 0,1765


190º 43’’ tg
tg 

d) 295 12
o o

’ 45’’ 360  64 47’ 15’’ o

sin  295o 12’ 45’’   sin  64o 47’ 15’’  0, 9047

cos
  295o 12’ 45’’ cos
  64o 47’ 15’’ 0, 4260
tg  295o 12’ 45’’  tg  64o 47’ 15’’  2,1239

o
e) 386
 56’ 360 o  26 o 56’

 386o 56’ sin


sin   26o 56’ 0, 4530
cos
  386o 56’ cos
  26o 56’ 0,8915
 386o 56’ tg
tg  26o 56’ 0, 5081
f) 612 o 43’ 2’’  360 o  180 o  72 o 43’ 2’’

sin  612o 43’ 2’’   sin  72o 43’ 2’’  0,9549

cos  612o 43’ 2’’   cos  72o 43’ 2’’  0, 2971

 612o 43’ 2’’ tg


tg  72o 43’ 2’’ 3, 2140
68. Dibuixa tots els angles possibles, més petits de 360º, que verifiquin les condicions següents:

272
 
 
 
 
 
 

Trigonometría 54
 
 
 
 
 
 

a) Que el sinus valgui 0,8. c) Que la tangent valgui 0,5.

b) Que el cosinus valgui -0,4. d) Que el sinus valgui -0,4.

a)   53,13o   126,87o

α 
β

                
b)   113,58o   246, 42o

α 

β
                   
   

c)   26,57o   206,57o

α  β 

                
d)  203,58
   336,42

β 
α 

        

69. Determina aquests angles per mitjà de la calculadora:

a) arc cos 0,4539 d) arc tg 2,1618

b) arc sin 0,9284 e) arc cos (-0,2926)

c) arc tg (-0,5459) f ) arc sin (-0,3308)

a)  arc cos 0,4539 63 20,95''                 d)  arc tg 2,1618  65 10' 32,9''  

273
 
 
 
 
 
 

Trigonometria
Trigonometria 4
 
 
 
 
 
 

b)  arc sin 0,9284 68 11' 12,3''                e)  arc cos (0, 2926)
 107 49, 2''  

c)  arc tg (0,5459) 


331 22' 12''
             f)  arc sin (0,3308)
 340 40' 58''  

70. Calcula l’angle a del 1r quadrant amb les raons trigonomètriques que verifiquen que:

a)
sin  sin 249 31' c)
tg  tg 249 31'

cos  cos 249 31'


b) tg  tg 183 30 '
d) 

Determina’n la resta de les raons trigonomètriques.

a) sin   sin 249 31'  0,9368    69 31'

b) cos   cos 249 31'  0,3499    69 31'

c) tg   tg 249 31'  2,6770    69 31'

sin 3 30 ' 7, 68''  0, 0611


d) tg  tg 183 30 ' 0, 0612   3 30 ' 7, 68 ''  
 cos 3  30 ' 7, 68 '' 0,9981

71. De l’angle d’un triangle sabem que el sinus és 0,7. Podries determinar de quin angle es tracta?

sin   0,7    44, 43o o,   135,57o

Els dos angles poden pertànyer a un triangle, per tant no podem determinar de quin angle es tracta.

72. De l’angle d’un triangle coneixem el cosinus, que val 0,2. Podries determinar quin angle és?

cos   0, 2    78, 46o o,   281,54o  

Únicament pot pertàyer a un triangle l’angle    78, 46o . 

73. D’un angle donat, a, sabem que sin  = 0,3 i que la tangent és negativa. Indica a quin quadrant

pertany aquest angle i calcula el valor de la tangent.

sin   0 i tg   0 per tant, l’angle pertany al segon quadrant.

sin   0,3    162,54o tg162,54o  0,3145

274
 
 
 
 
 
 

Trigonometría 54
 
 
 
 
 
 

74. Si sabem que la tangent d’un angle és dues vegades el seu sinus, que el signe d’aquest sinus
és positiu i que el del cosinus és negatiu, indica a quin quadrant pertany l’angle i calcula’n la
resta de les raons trigonomètriques.

sin   0 
  L’angle és al segon quadrant 
cos   0 

 3
sin  1 sin  
tg  2sin    2sin   cos
     120  
 2  
cos  2 tg    3

75. D’un angle ∝ del 2n quadrant sabem tan sols que el seu sinus és 0,5. Determina la resta de les
raons trigonomètriques d’aquest angle.

3 3
sin   0,5    150o        cos150o           tg150o    
2 3

76. Si sabem que sin ∝= 0,23 i que a és un angle agut, determina les raons trigonomètriques
següents:

a) cos  c) tg (-  ) e) sin (180° + )

b) tg  d) cos (180° - ) f ) sin (720° + )

a) 0,23� � ��� � ∝�� �� � ���� � � �� � 0,23� � 0,��32

�,��
b) �� ∝�� � � 0,23�3
�,����

c) ����∝� � � ���� ∝�� � �0,23�3

d) �� ����0��∝� � ����� � � �0,��32

e) sin���0� � �� � � ����� � � �0,23

f) sin��20� � �� � sin � � 0,23�

77. Si sabem que la tangent d’un angle és tres vegades el seu sinus i que les dues raons són
negatives, calcula’n la resta de les raons trigonomètriques.

Tenim que:

sin   0 

sen   cos   0  L’angle és al quart quadrant.
tg
  0
cos  

D’altra banda:
275
 
 
 
 
 
 

Trigonometria
Trigonometria 4
 
 
 
 
 
 

sin  1
tg   3sin   cos      289, 47 o .
cos  3
Llavors, la resta de raons trigonomètriques són:
sin 289, 47o  0,9428 tg 289, 47 o  2,8284 .

�.
78. En la circumferència següent, calcula la mida del segment AB i de l’arc de circumferència ��

 
Com que l’angle A = 90º, l’angle B = 90º - 30º = 60º.
��
��� 30° � � �� � � ��

La mida del segment AB és 4 cm.

� és de 60º
Com que l’angle de 30º és inscrit, l’angle central de l’arc ��

Calculem la longitud d’un arc de 60º dins d’una circumferència de radi 4 cm:

�� � � � 60° ��
� �
�� � � ���� ��
360° 3

� és 4,19 cm.
La mida de l’arc ��

79. Calcula les raons trigonomètriques d’aquests angles:


��
a) cos � = -0,54 amb � < � <

��
b) sin � = 0 amb < � < ��

a) ���� � � ���� ���� � � � ��� � � � �� � ���� ���� � ��� ����

�������
��� � � �� ����
�����

b) No existeix cap angle amb aquestes condicions, perquè si sin � =0, llavors � � ���.

80. Resol els triangles que tens a continuació:

a) a = 10 cm b = 14 cm c = 8 cm

b) b = 6 cm c = 9 cm � = 39° 12'

c) a = 7 cm � = 38° 49' � = 66° 40'

a) Apliquem el teorema del cosinus:

276
 
 
 
 
 
 

Trigonometría 54
 
 
 
 
 
 

��� � �� � � � ���� � ��� � ��


� → ����
�� � �� � � � � ��� � ���� � � � � �� ����
��� � � �� � �

� � ������ ���

��� � �� � � � ���� � ��� � ��


� → ����
�� � �� � � � � ��� � ���� � � � � ��� �
��� � � �� � �

� � ������� ����

�� � ���� → ������ ��� → ������� ���� � ����� ����

b) Apliquem el teorema del cosinus:

� → �� � �� � �� � � � � � � � ��������� → � � �� �� ��
�� � �� � � � � ��� � ����

��� � �� � � � ��� � ��� � � ��


� → ����
�� � �� � � � � ��� � ���� � � � � �� ����
��� � � �� �� � �

� � ������ ��� ���� → �� � ���� � ������ � ����� � ���� � ������ ��� ����

c) � � ���� � ������ � ������ � ������

Apliquem el teorema del sinus:

� � � �
� → � → � � �� �� ��
��� � ��� � ��� ������ ��� ������

� � � �
� → � → � � �� �� ��
��� �� ��� � ��� ������ ��� ������

81. Resol els triangles següents:

a) a = 9 cm c = 5 cm B = 103° 27'

b) b = 8,3 cm c = 9,1 cm C = 112° 50'

c) c = 6 cm A = 27° 42' B = 98° 20'

a) Apliquem el teorema del cosinus:

� → � � √�� � �� � �� � ��� ������� � ��� �� ��


�� � �� � �� � ��� � ����

Apliquem el teorema del sinus:

� � ��� �� �
� → � → ��� � � �� ����
��� � ��� � ��� ������� ��� �

� � ������ ��� ��� → �� � ���� � ������ � ��� � ������� � ������ � ���

b) Apliquem el teorema del sinus:

277
 
 
 
 
 
 

Trigonometria
Trigonometria 4
 
 
 
 
 
 

��� � �� � �� ���� � ��� � ��


� → ����
�� � �� � �� � ��� � ���� � � � � ��� �
��� � � �� � �

� � ������� ����

� � �� � �� �
� → � → ��� � � �� ����
��� � ��� � ��� ������� ��� �

� � ������ ��� ��� → � � ���� � ������� � ������ ��� ��� � ����� � ���

�� � � ��� ������ ���


�� � �� �� ��
����������

c) � � ���� � ������ � ������ � ������

Apliquem el teorema del sinus:

� � � �
� → � → � � �� �� ��
��� � ��� � ��� ������ ��� ��� ���

� � � �
� → � → � � �� �� ��
��� � ��� � ��� ������ ��� ��� ���

82. Troba les solucions per als triangles següents:

a) a = 12 cm b = 7 cm c = 6 cm

b) a = 8 cm c = 9 cm � = 42° 55'

c) a = 10 cm c = 9 cm � = 72° 55'

a) Apliquem el teorema del cosinus:

��� � �� � �� ���� � �� � ��
��� � � � � ��� ����
��� �����

� � ���� ��� ���

��� � �� � �� ��� � ��� � ��


��� � � � � �� ����
��� � � �� � �

� � ��� ��� ��� ���

� � ���� � ��� ��� ��� ��� � ���� ��� ��� � ��� ��� ��� ���

b) Apliquem el teorema del sinus:

� � � �
� → � → ��� � � �� ����
��� � ��� � ��� � ��� ��� ���

� � ��� ��� ��� ���� � ��� ��� � ��� ��� � ��� �� ����

� � � �
� → � → � � ��� �� ��
��� � ��� � ��� ��� � � ���� ��� ��� �� �

c) Apliquem el teorema del sinus:

278
 
 
 
 
 
 

Trigonometría 54
 
 
 
 
 
 
� � � ��
� → � → ��� � � �� �����
��� � ��� � ��� � ��� �������

� � ���� � ������� ��� ��� � ������� � ������� �� ����


� � ������� ��� ��� ����������

� � � ��
� → � → � � �� ������
��� � ��� � ��� ����� � �� ���� ��� �������

83. Resol els triangles següents:

a) a = 10 cm, � = 30°, � = 70°

b) a = 25 cm, � = 60°, � = 80°

 = 180  - 30 - 70  = 80 
a) C

Apliquem el teorema del sinus:

a b 10sin 70 a c 10sin 80


 b
  18, 79 cm  c
  19, 70 cm
 
sin A sin B sin 30  
sin A sin C sin 30

 = 180 - 60  - 80  = 50 
b) B

Apliquem el teorema del sinus:

a b 25sin 50 a c 25sin 80


 b
  22,11  c
  28, 43
 
sin A sin B sin 60  
sin A sin C sin 60
cm cm

84. El pedestal d’una estàtua fa 8 m i, quan ens separem 15 m de la base, veiem l’estàtua sota un
angle de 28º. Quina altura té l’estàtua?

8 x
 tg 38  8
 x 0,7813·15 
 x 3, 7193 m
15

85. Determina l’altura del far situat en un illot amb les dades següents:

� = 73° � = 61° � ' = 28° b = 50 m

 
A  180o  73o  61o  46o

Apliquem el teorema del sinus:

279
 
 
 
 
 
 

Trigonometria
Trigonometria 4
 
 
 
 
 
 

50 x 50 x
   x 60,79 m

 sin D
sin A  sin 46 sin 61o
o

h  60,79  tg 28o  32,32 m .


86. En una construcció, dues bigues de 10 m estan soldades pels extrems i formen un triangle de
15 m amb una altra biga. Calcula els angles que formen entre elles.

Apliquem el teorema del sinus:


A   10 
B
10

15
 
15 15
 sin B
sin A  sin 180o  2 A

 
sin 2 A 2sin A cos A

 3
cos A  A  B
  41, 41o  C
  97,18o
4
87. Determina la distància que hi ha entre els punts A i B amb les dades del gràfic.

 
  180o  72o  95o  13o
B
Apliquem el teorema del sinus:
50 c
  c 211,39 m

sin13 sin 72o
o

88. Un far A es troba a 12 km a l’oest d’un altre far B. Un iot surt del far A i navega 9 km en línia
recta. En aquell moment, des del far B, el iot observa que forma un angle de 42º amb la
direcció est-oest. Determina la distància del iot al far B.

280
 
 
 
 
 
 

Trigonometría 54
 
 
 
 
 
 

Apliquem el teorema del sinus:


9 12  63,15o
 C


sin 42o sin C

A  180o  42o  63,15o  74,85o

a 9
 
 a 12,98 km
sin 74,85o sin 42o

89. Dos cables de 10 m i 6 m que subjecten una antena vertical situada sobre un pedestal formen
entre ells un angle de 25º. Calcula l’altura de l’antena.

Apliquem el teorema del cosinus i obtenim la distància a la qual esta lligats els cables a terra:

a
 102  62  2 10  6  cos 25
o
5, 22 m

Sigui x la distància que hi ha des de la base de l’antena fins el connector d’un dels cables.. D’aquesta
manera tenim que la distància de la base de l’antena a l’altre connector és de 5,22 – x. Així:

Altura
 102  x 2 

62   5, 22  x   x 8, 74 m
2 2 2
  10  x 
Altura  6   5, 22  x  
2 2

Substituïm x pel seu valor i obtenim l’altura:

Altura  102  8,742 4,86 m

90. En una paret hi ha dues argolles a 8 m l’una de l’altra. Un nen lliga cada argolla a un extrem
d’una corda i s’allunya de la paret fins que la corda queda tesada. En aquell moment, la corda
forma angles de 50º i 37º amb la paret.

a) Quina mida té la corda?

b) A quina distància es troba el nen de la paret?

L’altura del costat conegut divideix el triangle inicial en dos triangles rectangles. Apliquem la definició
de tangent en els angles coneguts i formem un sistema d’equacions.

h
����� � � � �����
� x � � � h � x � ����� ��� � �����
� h � �� � x� � ������ ����� � �����
������ �
��x

h = 3,1 · tg50º = 3,69 m

El nen es troba a una distància de 3,69 m de la paret.

281
 
 
 
 
 
 

Trigonometria
Trigonometria 4
 
 
 
 
 
 

3, 69 3, 69
sin 50º  BA   4,82m
BA sin 50º

3, 69 3, 69
sin 37º  CA   6,13m
CA sin 37º

Es calcula la longitud de la corda

8 + 4,82 + 6,13 = 18,95 m.

La corda té 18,95 m.

91. Per determinar la distància entre dos turons, tenim les dades del dibuix i sabem que la
distància de l’observador al turó A és 1 km. Quina distància hi ha entre els dos turons?

 
1 d
 o

 d 0, 78 km
sin 40 sin 300

92. Simplifica les expressions següents:

a) ����� � ����� � ���� � ����� � �����

b) ����� � ���� � ����� � ����

� ��
c)��� �� � � � ��� �� � �
� �

d)���� � �� � ���� � ��


e)��� � � �� � �������

a) sin   120   sin   sin   120  


o o

 sin  cos120 o  sin120 o cos   sin   sin  cos120 o  sin120


o
cos  0

282
 
 
 
 
 
 

Trigonometría 54
 
 
 
 
 
 

b) sin   30   cos  
o
45  o
sin  cos 30 o  sin 30 o cos   cos  cos 45o  sin  sin 45o

3 2 2 1
 sin   cos 
2 2

æ pö æ 3p ö p p 3p 3p
c) cos çça + ÷÷÷ + cos çça + ÷÷÷ = cos a cos - sin a sin + cos a cos - sin a sin =
çè 2ø ç
è 4ø 2 2 4 4

2 2 2 +1 2
= - sin a - cos a - sin a = - sin a - cos a
2 2 2 2

tga + tg p tga - tg p
d) tg (a + p) - tg (a -p) = - =0
1- tgatg p 1 + tgatg p

æp ö p p
e) cos çç +a÷÷÷ + cos (-a ) = cos cos a - sin sin a + cos a = - sin a + cos a
çè 2 ø 2 2

93. Simplifica aquestes expressions trigonomètriques:

 
a) 2  tg  cos
2 2
b) 2  cos 2  cos 
2

2sin  cos 
c)
1  2sin 2 
d) cos  1  cos    sin 
2


tg
e) 2 2
cos 

sin
  2  cos   
a)  2  tg  cos 2 2sin  
2 2  2 2
cos
2
b)  2  cos 2  cos 2  
2  cos 2   sin 2   cos 2  
2  sin 2   
2sin  cos  sin 2 sin 2 sin 2
c)     tg 2  
2
1  2sin  sin   cos   2sin  cos   sin  cos 2
2 2 2 2 2

d)  cos  1  cos    sin 2  cos   cos 2   sin 2  cos   1  

1  cos 
 1  cos 2  sin 2  2sin 
tg (1  cos  ) 2
(1  cos  ) 2
e)  2  2 2 1  cos
  2  2  1  cos  2tg (1  cos  )  
cos  cos  cos  cos  cos 

283
 
 
 
 
 
 

Trigonometria
Trigonometria 4
 
 
 
 
 
 

94. Si sabem que la tangent de ∝ és 2,5 i que ∝ és un angle del primer quadrant, calcula tg (∝ +
45°). Determina també en quin quadrant es troba l’angle ∝ + 45°.

tga + tg 45o 2,5 + 1


tg (a + 45o ) = o
= = -2,33
1- tgatg 45 1- 2,5

L’angle és al segon quadrant.


95. D’un angle agut sabem que tg ∝ =

Calcula tg 2∝.

5
2⋅
2tg a 4 = - 40
tg (2a ) = =
1 - tg a 1 - 25
2
9
16

96. Troba una fórmula per calcular sin 3x en funció de sin x. Aplica-la per calcular sin 3x si saps

que sin x = .

sin (2 x + x) = sin 2 x cos x + sin x cos 2 x = 2sin x cos2 x + sin x (cos2 x - sin 2 x) =

= 3sin x (1- sin 2 x) - sin 3 x = 3sin x - 4sin 3 x


3
1 1 æ1ö 23
Si sin x = llavors sin (3x) = 3 - 4 çç ÷÷÷ =
3 3 ç
è 3ø 27

97. Sabem que sin 56° = 0,83 i cos 23° = 0,92.

a) Calcula la resta de raons d’aquests angles.

b) Troba les raons trigonomètriques de 79º.

c) Determina les raons de 33º.

d) Podries calcular les raons de 28º?

e) I les raons de 46º?

a) �� ��� � ���� ��� � �� � ��� ��� � � �� � �� ��� � �� ��

�� ��
�� ��� � � � �� ��
�� ��

���� ��� � �� ��� � �� � ��� ��� � � �� � �� ��� � �� ��

�� ��
�� ��� � � � �� ��
�� ��

284
 
 
 
 
 
 

Trigonometría 54
 
 
 
 
 
 

b) ��� ��� � ������� � ���� � ��� ��� � ��� ��� � ��� ��� � ��� ��� � �� �� � �� �� � �� �� � �� �� �
�� ��

��� ��� � ������� � ����


� ��� ��� � ��� ��� � ��� ��� � ��� ��� � �� �� � �� �� � �� �� � �� �� � �� ��

�� ��� � �� ��� �� �� � �� ��
�� ��� � ������ � ���� � � � �� ��
� � �� ��� � �� ��� � � �� �� � �� ��

c) ��� ��� � ������� � ���� � ��� ��� � ��� ��� � ��� ��� � ��� ��� � �� �� � �� �� � �� �� � �� �� �
�� ��

��� ��� � ������� � ����


� ��� ��� � ��� ��� � ��� ��� � ��� ��� � �� �� � �� �� � �� �� � �� �� � �� ��

�� ��� � �� ��� �� �� � �� ��
�� ��� � ������ � ���� � � � �� ��
� � �� ��� � �� ��� � � �� �� � �� ��

��� ����� ��� ������


d) ��� ��� � ��� �� �� � �� ��
� � �

��� � � ��� ��� � � �� ��


��� ��� � ��� �� �� � �� ��
� � �

��� � � ��� ��� � � �� ��


�� ��� � �� �� �� � �� ��
� � � ��� ��� � � �� ��

e) ��� ��� � ����� � ���� � � � ��� ��� ��� ��� � � � �� �� � �� �� � �� ��

��� ��� � ����� � ���� � ���� ��� � ���� ��� � �� ��� � �� ��� � �� ��

� � �� ��� � � �� ��
�� ��� � ���� � ���� � � � �� ��
� � �� ��� � � �� ���

98. Indica una fórmula simplificada de:

a) sin (30° + � ) �)
c) tg (45° – �

b) cos ( � – 60°) � + 30°)


d) cos (�

� √�
� � � ���� �
a) ������� � � � � ��� ��� � ��� � � ��� ��� � ��� � � ��� � � ��� � � � √� ��� � �
� � �
� √� �
� � ���� � ��� �
b) ����� � � ��� ��� � ��� � � ��� ��� � ��� � � ��� � � ���� � � √� ��� � �
� � �

���� �
��
c) tg (45º �� � �
���� �

� � ���� � ��� �
d) ����� � � ��� ��� � ��� � � � � ��� �
� � ��� ��� � √� ��� � � � � �√� ��� �
� � ��� �
� �
� � �

� �
99. Si sabem que sin x = i que � � � �� sense calcular prèviament el valor de x, troba:
� �
285
 
 
 
 
 
 

Trigonometria
Trigonometria 4
 
 
 
 
 
 
� �
��� �� � � �� �� � �
� �

a) Expressa els resultats utilitzant radicals.

b) Explica com determinaries les raons


� �
de ��� � ����
� �

Trobem les raons trigonomètriques de x:

� � √�� �√��
cos x = ��� � � � � � �
tg =
� � ��

� �
a) i b) Les raons trigonomètriques de ���� ��� � ���� ��� són conegudes.
� �

� √� � √� � √� � � �
��� � ��� � ��� � ��� � �� � √�
� � � � � � � � �

� � � � √� √�� √� �√� � √��


����� � � � ��� � � ��� � ��� � � ��� � � � � �
� � � � � � � ��

� ��� � �� �√�� � ��√�

���� � � � � � �
� � � ��� � �� �

�� �
100. Sabem que � � � � � �� � �
� �

a) Troba sin x i cos x. 

� �
b) Utilitzant radicals, troba les raons dels angles    i   
� �

c) Sense determinar l’angle x, calcula: 
� �
����� � � ���� � �
� �

d) Sense determinar l’angle x, decideix raonant en quin quadrant són aquests angles. 
� �
� � � �
� �


a) cos � � � � ���� ���� � � ���� � � � sin � � �� � ���� � ���6
�������

� � � √� � √�
b) sin � cos � �� �
� � � � � �

� √� � √� �
sin � cos � �� � �
� � � � �
� � � √� √�
c) cos�� � � � cos � � cos � ���� � sin � ���� � � ��6 � � ����√2
� � � � �
� � √�
� ��� � �� � �� � 2�√3
� � �
���� � � � � � �
6 � � ��� � �� �� �
� √� 39
� � �

d) Com que el sinus de l’angle x – és positiu, l’angle és al segon quadrant.

286
 
 
 
 
 
 

Trigonometría 54
 
 
 
 
 
 

e) I com la tangent de l’angle x + és positiva, l’angle és al tercer quadrant.

101. L’angle que forma un ventall és de 170º. Si el ventall té tres varetes centrals, calcula les raons
trigonomètriques dels angles que es formen quan el despleguem vareta a vareta si saps que
cos 170° = – 0,98 i que sin 170° = 0,17.

 
Els angles que es formen són de 42,5º, 85º, 127,5º i 170º.
X=170º

æ xö 1- cos x æ xö 1 + cos x
sin (85o ) = sin çç ÷÷÷ = = 0,99 cos (85o ) = cos çç ÷÷÷ = = 0,1
çè 2 ø 2 çè 2 ø 2

æ xö æ xö
1- cos çç ÷÷÷ 1 + cos çç ÷÷÷
æ xö çè 2 ø æ xö çè 2 ø
sin ( 42,5o ) = sin çç ÷÷÷ = = 0, 67 cos ( 42,5o ) = cos çç ÷÷÷ = = 0, 74
çè 4 ø 2 çè 4 ø 2

sin (127,5o ) = sin (42,5o + 85o ) = sin (42,5o ) cos (85o ) + sin (85o ) cos (42,5o ) = 0,79

cos (127,5o ) = cos (42,5o + 85o ) = cos (42,5o ) cos (85o ) - sin ( 42,5o ) sin (85o ) = -0,59

 
102. Calcula cos 285º a partir de les raons dels angles de 330º i de 45º.

3 2 æç 1 ö÷ 2
cos (285o ) = cos (330o - 45o ) = cos (330o ) cos (45o ) + sin (330o ) sin (45o ) = ⋅ + ç- ÷⋅ = 0, 2588  
2 2 çè 2 ÷ø 2
103. Si les raons de 32º són:

sin 32° = 0,53 cos 32° = 0,848

a) Calcula les raons trigonomètriques de 62º.

b) Determina les raons de 31º.

c) Pots calcular les raons trigonomètriques de qualsevol angle que tingui la mida en graus,
però sense minuts ni segons?

√� �
a) ��� ��� � ������� � ���� � ��� ��� � ������ � ������������ � �� �� � � �� ��� � � �� ��
� �

√� �
��� ��� � ������� � ���� � ��� ��� � ������ � ������������ � �� ��� � � �� �� � � �� ��
� �
�� ��
�� ��� � � �� ��
�� ��
��� ����� ��� ������
b) ��� ��� � ���� �� �� � �� ��
� � �

��� � � ��� ��� � � �� ��


��� ��� � ���� �� �� � �� ���
� � �

287
Trigonometria
Trigonometria 4

�� ��
�� ��� � � �� ���
�� ��

c) A partir de les mides de 32º i de 31º, podem trobar les mides de 1º, i a partir d’aquestes podem
calcular les raons de qualsevol angle.

104. Escriu les expressions que tens a continuació en forma de producte:


a) cos 2∝ – cos ∝ d) sin ∝ – cos ∝
b) cos ∝ – cos 4∝ e) cos ∝ + sin 3∝
c) sin 8∝ – sin 2∝ f ) sin ∝ – sin 3∝

a) cos 2a - cos a = cos2 a - sin 2 a - cos a = cos a (cos a -1) -(1- cos 2 a ) =

= cos a (cos a-1) -(1- cos a)(1+ cos a) = (cos a-1)⋅(2cos a +1)

b) cos a - cos 4a = cos a - cos(3a +a ) = cos a + tga cos a sin 3a - cos a cos 3a =

= cos a (1 + tga sin 3a - cos 3a )

c) sin 8  sin 2  sin(2  6 )  sin 2  sin 2 cos 6  sin 6 cos 2  sin 2 

sin 2 cos 6  (sin 2 cos 4  2sin 2 cos 2 cos 2 ) cos 2  sin 2 

2 sin 2 cos 2 cos 2 ) cos 2  sin 2  sin 2 (cos 6  cos 4 cos 2  2 cos 3 2  1)

d) sin   cos
 tg ·cos   cos
 cos  (tg  1)

e) cos   sin 3  cos   sin(  2 )  cos   sin  cos 2  cos  sin 2 

cos   cos  tg cos 2  cos  sin 2 


 cos  (1  tg cos 2  sin 2 )

f) sin   sin 3 sin   sin(2   ) sin   sin 2 cos   sin  cos 2

sin   2 sin  cos 2   sin  cos 2 


 sin  (1  2 cos 2   cos 2 )

105. Simplifica i escriu les expressions següents amb una sola raó trigonomètrica:

����∝������� ∝
a)
���� ∝����� ∝����� ∝

b) ���� � ∝ ����� � ∝

c) ���� ∝ ���� ∝ ��� ∝ ����� ∝� � ��� ∝

d) ���� ∝ ����� ∝ ����� ∝

sin a - sin 3 a sin a (1 - sin 2 a ) sin a (1 - sin 2 a )


a) = = = sen a
cos 4 a + cos 2 a sin 2 a (1 - sin 2 a )2 + (1 - sin 2 a ) sin 2 a 1 - sin 2 a

2 2
b) cos 5a- sin 5a = cos10a

288
Trigonometría 54

1
c) sec a (sin a cos a + cos 2 a ) - sin a = (cos a (sin a + cos a )) - sin a = cos a
cos a

d) sin 4 a - cos4 a + cos2 a = sin 2 a (1- cos2 a ) - cos2 a (1- sin 2 a ) + cos2 a = sin 2 a

106. Demostra que es verifiquen les igualtats que hi ha a continuació:

a) � � ����� ∝� �������∝ ���������∝ �����

b) ������ � ������� � ��������� � ����

c) ��� �� � ������ � ������� �� � �� � ��

∝ ∝��
d) ��� � � � ���� � �
� �

a) � ����� � ���� ����� � ���� �

������ � ������ � ���� � ������������ � ������ � ���� � ������� �

√� � ��� � √� � ��� � √� � ��� � √� � ��� �


� �� � �� � ��
� � � �

������ ∝ ����∝����∝ ������ ∝


=2� � � ��
� � �

=���� ∝ ����� ∝ ����� ∝ ��� ∝� � � ���� ∝

b) � ����� � ���� ����� � ���� �

������ � ������ � ���� � ������������ � ������ � ���� � ������� �

√� � ��� � √� � ��� � √� � ��� � √� � ��� �


� �� � �� � ��
� � � �

������ ∝ ������ ∝
=2� � � � ���� � � ���� � � ��� ��
� �

(2sin  ) 2sin  ( cos2  )


c) 2tg (   ) sin 2 (   )  1  sin 2   1  2sin  cos  2sin 2
 cos  cos 

   sin(   )  sin  cos 2   1


d) sin
     
 2  2 cos      1  cos  4(1  cos  )
  2 2
 2  2

(1  cos  )(1  cos  ) 1  cos   


   cos  
2(1  cos  ) 2 2
107. Demostra que la igualtat següent és certa:
����
��� �� � �
� � ��� �
����� � ���� � ���� ����
��� �� � ����� � ���� � � � �
���� � � ��� �
���� �

289
Trigonometria
Trigonometria 4

108. Demostra les igualtats trigonomètriques següents:

����� ����� � �
a) � � ���� c) � � ����� �
���� ���� ������ ������

b) ��� � ����� � ���� � ������ d) ��� � � ��� � ���� � ����

2sin  cos  2cos2   1 2cos2   2cos2   1 1


a)     sec 
sin  cos  cos  cos 

sin  cos  sin 2   cos2  1 1


b)      sec   cosec 
cos  sin  cos  sin  cos  sin 

1  sin   1  sin  2
c)   2sec2 
1  sin 
2
cos 
2

sin 2  cos2   sin 2  1


d) cos     sec 
cos  cos  cos 

109. Comprova que la igualtat següent és certa substituint a per un angle conegut:

�����
� ���� � ���� � ���
����

Demostra que aquesta propietat es compleix per a qualsevol angle �.

Escollim l’angle de 30º

������� � √� √� √� √�
� � ��� ��� � ������ � ����� � ‐ �
������� √� � � � �

����� ����� ���� � �� � ��� �� ���� ���� � ��� �


� ���� � � � � ���� � ���� � ���
���� ��� ��� ���
����� �

110. Determina la solució de les equacions següents:

� �
a) cos x tg x = e) sin x cotg x = –
� �

b) cos 2 x + sin 2 x = 1 f ) sin 2 x + sin 2 x = 0

c) cos 2 x – sin 2 x = 0 g) tg x + sin x = 0

d) sin 2 x + cos x = 1 h) tg x – sin 2 x = 0

� � �� � ��� � ���� � �
a) ��� � � �� � � � ��� � � � �
� � �� � ���� � ���� � �

b) ��� �� � ��� �� � � � ���� � � ���� � � ����� � ���� � ���� � � ���� �

290
Trigonometría 54

→ ������ � � � ��� � ���� � � → � ��� � �� ��� � � ��� �� � �

�� � �� � ���� � �
��� � � � → �
�� � ���� � ���� � �

�� � ��� � ���� � �
��� � � ��� � → �
�� � ���� � ���� � �

� � ��� �� � ���� � �
c) ��� �� � ��� �� � � → ��� �� � ��� �� → � �
�� � ���� �� � ���� � �

� � ��� � ���� � � �� � ���� � ���� � �


d) ��� �� � ��� � � �� ��� � � ������ � � → � �
�� � ���� � ���� � � �� � ���� � ���� � �

→ ������ � � � ��� � ���� � � → � ��� � �� ��� � � ��� �� � �

�� � ��� � ���� � �
��� � � � → �
�� � ���� � ���� � �

� � ��� � ���� � �
��� � � ��� � → � �
�� � ���� � ���� � �

1 1  x  104, 48  360 k


e) sin x cot gx   cos x   1
4 4  x2  255,52  360 k

�� � �� � ���� � �
f) ��� �� � ��� �� � � → � ��� �� � � → �
�� � ��� � ���� � �


g) ��� � ��� � � � → ��� � � � 1� � �
��� �

�� � �� � ���� � �
��� � � � → �
�� � 1��� � ���� � �

1
� 1 � � → �� � 1��� � ���� � �
��� �
��� �
h) tg � � ��� �� � � → � � ��� � ��� � � � → ��� ��1 � ���� � �� � �
��� �

x� � �� � ���� � �
��� x � � → �
x� � 1��� � ���� � �


→ 1 � ����� x � � → ��� x � � → x� � ��� � ���� � �

291
Trigonometria
Trigonometria 4

111. Resol les equacions trigonomètriques que tens a continuació:

������°���
a) ��
��� �


b) �� � � �� � �� � � � � �

c) sin (x + 30°) + cos (x + 60°) = 1 + cos 2 x

������°��� √� ��� ����� �


a) � � → � � → √� � �� � � �
��� � � ��� �

→ tg x =-0,2679 → x=345° + 360°·k

� ���� �
b) �� � � �� � �� � � � � � → � �� � � � � �
� ���� �

→��� � � �� � � � → �� ���� � � �� � �

�� � �° � ���° � �
�� � � � → �
�� � 1��° � ���° � �

�� � ��° � ���° � �
�� � � 1 � � → �� � � 1 → �
�� � 22�° � ���° � �

c) ����� � ��°� � ��� �� � ��°� � � � ��� ��

√� ��� � ��� � ��� � √� ��� �


→ � � � � ��� � � � ���� � � ���� � � ���� � →
� � � �

→ ��� x � 2 ���� x → ��� x�2 ��� x�1� � �

x � ��° � ���° � �
��� x � � → � �
x� � 2��° � ���° � �

1 x � ��° � ���° � �
2��� x � 1 � � → ��� x � → � �
2 x � � ���° � ���° � �

112. Resol les equacions trigonomètriques següents:


a) tg (x + 45°) + tg (x - 45°) = 2 cotg x c) tg� � �� – 1 = 0

b) 4 sin 2(x + 30°) = 1 d) sin (x + 30°) - cos x = 0

tgx  tg 45o tgx  tg 45o 2


a) tg ( x  45)  tg ( x  45
) 2cotg x    
1  tgxtg 45o 1  tgxtg 45o tgx

 tgx  1 tgx   tgx  1 tgx  1 tgx 2 1  tg 2 x  


2

3
 6tg 2 x 2  tgx  x30o  180o  k , 150o +180o  k
3

292
 
 
 
 
 
 

Trigonometría 54
 
 
 
 
 
 

2 x  60o  14, 48o  x 22,76o  180o  k


 o

b) sin 2 x  60  0, 25   o
165,52o  x 52,76o  180o  k
 2 x  60
 


tg  tgx
  4 1  tgx
c) tg   x   1  0   1  1  tgx  0  x  0  180k
 4   1  tgx
1  tg ·tgx
4
d) sin( x  30)  cos x  0  sin x cos 30 o  sin 30 o cos x  cos x  0 

3 1
 
2
 cos x  
2
 sin x  cos x cos x  3 sin x cos x  3 1  cos 2 x
2 2
 3
3 cosx  30  360k , 330  360k
 2
 3  3cos 2 x cos 2 x  3 4 cos 2 x  cos x  
2 cos x   3  150  360k , 210  360k
 2
De totes les solucions possibles s’observa que únicament són vàlides:
x 30 360k x 210 360k


113. Resol aquests sistemes d’equacions trigonomètriques:

���� � � ���� � � � � � � � ���


a) � � b) � �
���� � � ���� � � ��� � � � ��� � � ���
� ���� �

sin� � � sin� � � 1 sin� � � 1 � sin� � � ��s � �


a) � � � →� �
cos � � ��s � � � cos � � � sin� � �
� �

1
cos 2� � → � � 3�� � 1��� � �
2

1
cos� y � sin� 3�� → cos y � � → y � ��� � 1��� � �
4
� � � � 12�
b) � � → x=120° - y → cos (120°-y) =
��� � � � ��� � � ���
���� �

1
� sin �12�� � y�tg y
2cos y
� cos � y � √3 sin y � cos y � 1 � √3 sin y � cos y � sin� y
→ cos � y � sin� y � 1 → cos 2y � �1 → y � ��� � 1��� � �
� � 12�� � y � 12�� � ��� � 1��� � � � 3�� � 1��� � �
114. Resol aquests sistemes d’equacions trigonomètriques:

� � � � ��� ��� � � ���� � � �


a) � c) �
��� � � ��� � ���� � � � ��� � � �

� � � � ��� ���� � � ���� � � �


b) � d) �
�� � � ���� � ��� ���� � � ��� � � �

293
 
 
 
 
 
 

Trigonometria
Trigonometria 4
 
 
 
 
 
 

3
a) x 60o  y  cos  60o  y  sin y  cos 60o cos y  sin y sin y
2
1 2 3 1
cos y  sin y  tg y  y 75o  360o  k , x 15o  360o  k
2 2 2 3
tg 30o  tgx tg 30o  tg 2 x
b) y 30o  x  tg  30o  x  tg  30o  2 x   
1  tg 30o tgx 1  tg 30o tg 2 x
tg 30o  tg 2 30o tg 2 x  tgx  tg 30o tg 2 xtgx tg 30o  tg 2 30o tgx  tg 2 x  tg 30o tg 2 xtgx
4 4 2tgx
tg 2 x  tgx 0  2
 tgx 0  tgx  3  tg 2 x  0
3 3 1  tg x
x 180o  k , 
 y 180o  k  30o
x 60o  180o  k , y 90o  180o  k
x  120o  180o  k , y  150o  180o  k
c) 2sin x  4 cos y 
4
+ 3sin x  4cos y 1
5sin x  5
1
5sin x 5  sin x 1  cos y   x 90o  360o  k , y 60o  360o  k
2
d) 4sin x  cos 2 x 
2
– 4sin x  2 cos y 2
cos2 y  2cos y  0
 1 o o o
2 cos y  0  sin x   y 180  k , x  30  360
cos y  2 cos y 
0 2
cos y 2  Sense solució


115. Resol aquestes equacions trigonomètriques:

a) ���� � � ��� � � �

���� �
b) � ��� �
���� ����� �


c) � � ��� � � � ��� �
��� ����� �

d) ��������� � �� � � ���� � � �

e) 2 cosx – 1 = sec x

f) 2 cos x + sin x = 1

g) sin x + cos x = 0

294
 
 
 
 
 
 

Trigonometría 54
 
 
 
 
 
 

a) � ��� � � ��� � � � → � ��� � ��� � � � � � → � ��� �� � � → ��� �� �

� � ��� � ���� � �
� �
�� � ��� � ���� � �

��� � �
b) � ��� � → ���� � � � ��� � ��� � � ���� � → ��� �� � ��� ��
� ��� ����� �

�� � ��� �� � ���� � �

�� � ���� �� � ���� � �

� � ��� ����� ��� ���� �


c) � � ��� � � � ��� � → � �
��� ����� � � ��� � ��� ��� ����� �

��� � ���� � � ��� �� � � ��� � ���� � �


→ ��→� �
��� � ���� � � ��� �� �� � ���� � ���� � �

d) ��� ����� � � �� � � ��� � � � � → ���� � � ��� � � ��� � ���� �� � �

����� � � ��� � � � � �

� � � ����������� � ���� � �
→ ��� � � � → � �
� �� � ����������� � ���� � �

� �� � ��� � ���� � �
→ ��� � � → �
� �� � ���� � ���� � �

e) � ��� � � � � ��� � → � ��� � � � ��� � � � � �

�� � �� � ���� � �
→ ��� � � � → �
�� � ���� � ���� � �

� � � ���� � ���� � �
→ ��� � � � → � �
� �� � ���� � ���� � �

f) � ��� � � ��� � � � → √� � ���� � � � � � ��� � → � ��� � � � � ��� � � �

� �� � ������ ��� ��� � ���� � �


→ ����� ��� � � �� � � → ��� � � → �
� �� � ������ ��� � ���� � �

� � ���� � ���� � �
g) → ��� � � ��� � � � → ��� � � � ��� � → � �
�� � ���� � ���� � �


116. Determina quin és l’angle agut tal que el triple de la seva tangent és igual al doble del seu
cosinus.

sin x
3tgx  2 cos x  3  2 cos x  3sin x  2 cos 2 x  3sin x  2 1  sin 2 x 
cos x

 2  No pot ser
3  9  16 
2 sin 2 x  3sin x  2  0  sin x   1
4  2

o
L’angle agut és x  30 .

295
 
 
 
 
 
 

Trigonometria
Trigonometria 4
 
 
 
 
 
 

117. Indica quin és l’angle obtús tal que el seu sinus sumat amb el triple del seu cosinus dóna -1?

sin x  3 1  sin 2 x  1  sin x 


1  3 1  sin 2 x 

1  x 270o

9 1  sen x  1  2sin x  sin x  5sin x  sin x  4  0  sin x   4
2 2 2

 x 126,87o
5


118. Quin és l’angle agut tal que el seu sinus multiplicat pel seu cosinus dóna ?

o o
1 1 2 x 30  x 15
sin x cos x   sin 2 x 
4 2 2x 150o  
x 75o

119. Si sabem que l’àrea d’un triangle rectangle és 28 cm2 i que un dels angles fa 60º:
a) Quina mida té cadascun dels angles?
b) Calcula'n la longitud dels costats i el perímetre.
a) L’angle desconegut és: 90º - 60º = 30º
b) Prenem com base i altura els catets del triángle rectangle:
࢈൉ࢇ ૞૟
૛ૡ ൌ  ՜ ࢈ ൌ
૛ ࢇ
૞૟
ࢇ ૞૟
࢚ࢍ૜૙ι ൌ  ՜ ࢇ ൌ ඨ ൌ ૢǡ ૡ૞ࢉ࢓
ࢇ ࢚ࢍ૜૙ι

b=5,68 cm
Apliquem el teorema de Pitàgores per calcular la hipotenusa:

ࢉ ൌ ඥૢǡ ૡ૞૛ ൅ ૞ǡ ૟ૡ૛ ൌ ૚૚ǡ ૜ૠࢉ࢓


Els costats són de 11,37, 5,68 i 9,85 cm respectivament.
El perímetre és 26,9 cm.

120. Fixa’t en la situació i, mitjançant la trigonometria, calcula l’altura, h, a la qual es troba el punt
B.

 
Si B es troba a la altura h


–‰ ͵ͷι ൌ 
ଶହା௫
ቐ ௛  ՜ ‫ ݔ‬ൌ ͳǡͳͳ݄ ՜ ݄ ൌ ͳ͹ǡͷͳ ൅ Ͳǡ͸͵݄ ՜ ݄ ൌ Ͷ͹ǡ͵ͺ݉
–‰ Ͷʹι ൌ 

296
Trigonometría 54

El punt B es troba a l’altura de 47,38 m.

121. Determina l’altura de la muntanya de la figura a partir de les dades que ens proporcionen.

180º - 60º = 120º 180º - 120º - 30º = 30º

Els angles del primer triangle són 30º, 120º i 30º. Pel teorema del sinus

2500 a
 o

 a 2500 m
sin 30 sin 30o
Es resol el triangle rectangle sabent que un angle és 60º i la seva hipotenusa és 2.500 m.

h
 sin 60o 
 h 2165,06 m
2500

122. Dos amics estan separats per una distància de 40 m i veuen un arbre a la riba del davant del
riu, com indica la figura. Calcula l’amplada del riu.

Anomenem h a l’amplada del riu.


–‰ ͵ͺι ൌ 

ቐ ୦
 ՜ š ൌ ͳǡʹͺŠ ՜ ͵ͺǡ͸͵ െ ͳǡʹͶŠ ՜ Š ൌ ͳ͹ǡʹͷ
–‰ ͶͶι ൌ 
ସ଴ି୶

L’amplada del riu és 17,25 m.

297
Trigonometria
Trigonometria 4

123. Dos focus, situats en el terra i en costats diferents, il·luminen el campanar d’una església. La
suma de les distàncies dels focus fins al peu de la torre és de 100 m. Si els angles que formen
els feixos de llum amb el terra són de 32º i 46º, determina l’altura que té el campanar.

Anomenem y l’altura del campanar



–‰ ͵ʹι ൌ 

൝ ୷  ՜ š ൌ ͳǡ͸Ͷ ՜ ͳͲ͵ǡͷͷ െ ͳǡ͸͸› ൌ › ՜ › ൌ ͵ͺǡͻ͵
–‰ Ͷ͸ι ൌ 
ଵ଴଴ି୶

L’altura del campanar és 38,93 m.

124. Des d’un turó es poden veure, en línia recta cap a l’est, dos poblets que estan separats 800 m
l’un de l’altre. Des del cim s’observen amb angles de 18º i 26º 40’, respectivament.

a) Quina és l’altura del turó? b) A quina distància es troba cada poblet de l’observador?

a) Anomenem y l’altura del turó.

90º - 18º = 72º 90º - 26º 40’ = 63º 20’

୶ା଼଴଴
–‰ ͹ʹι ൌ 

ቐ ୶  ՜ š ൌ ͳǡͻͻ› ՜ ͵ǡͲͺ› ൌ ͳǡͻͻ› ൅ ͺͲͲ ՜ › ൌ ͹͵ͷǡͻͶ
–‰ ͸͵ιʹͲԢ ൌ 

x = 199y = 1504,42 m 800 + x = 2304,42 m

La distància de l’observador a cada poblet és 1.674,78 m i 2.419,18 m, respectivament.

298
Trigonometría 54

125. El pilot d’un helicòpter de reconeixement que vola sobre el mar a una altura de 2 500 m albira
dues embarcacions que es troben en un mateix pla vertical, amb angles de depressió de 62º
24’ i 37º 18’, respectivament. Calcula la distància que separa una embarcació de l’altra.

2500
a  4125, 49 m
sin  37 o18’
Es calculen els angles:

62 o 24’  37 o 18’ = 25o 6’


90 o  62 o 24’ 
27 o 36’
180 o  90 o  27 o 36’ 
62 o 24’
180 o  62 o 24’ 
117 o 36’
180 o  117 o 36’  25o 6’ 
37 o18’
Pel teorema del sinus:
4125, 49 x
   x 1974,52 m
sin117 36’ sin 25o 6’
o

126. Entre les dues plantes d’un edifici han d’instal·lar una escala. La diferència d’altura entre les
plantes és de 3,5 m i disposen de 5 m en horitzontal per posar l’escala.

a) Calcula l’angle d’inclinació de l’escala.

b) Determina la longitud de l’escala.

3,5 3,5
a) tg     35o b) sin 35o  l 6,10 m
5 l
127. Calcula la longitud del paral·lel 38º nord si considerem que el radi de la Terra és de 6 370 m.

90 o  38 o 
52 o

299
Trigonometria
Trigonometria 4

r 6370  sin 52o 5019,63 km


2 r  31539, 26 km

128. L’Eulàlia i la Miranda volen mesurar l’amplada d’un congost. Per fer-ho, es col·loquen en una
de les vores. L’Eulàlia porta una corda de 6 m de llargada i la deixa lliscar des de la vora del
congost. De la seva banda, la Miranda, que té els ulls a 1,8 m del terra, s’ha d’enretirar 4,5 m
per veure la vora més próxima coincidint amb el final de la corda.

a) Quina amplada té el congost? b) Es podria fer sense utilitzar la trigonometria?


L’amplada del congost és x:
1,8
�� � � � � �,4�
4,5
6
�,4 � � � � � � 15���

a) L’amplada del congost és 15 m.
b) Es podria utilitzar la semblança de triangles per resoldre el problema.

129. Des de la finestra d’un edifici, situada a 3 m d’altura, es veu la base d’un altre edifici amb un
angle de 22º per sota de l’horitzontal. La part superior d’aquest segon edifici no es pot veure,
però sí que se’n pot observar el reflex en un estany amb un angle de 56º sota l’horitzontal.
Quina altura té aquest segon edifici? Quina distància hi ha entre totes dues edificacions?

Sigui V el punt que representa la finestra. Llavors:

90 o  56 o 
34 o

3  tg 34o 
MP  2m MV  22  32  3, 6 m

300
 
 
 
 
 
 

Trigonometría 54
 
 
 
 
 
 

Pel teorema del sinus:

3,6 AM
  AM 5, 4 m
sin 22 sin 34o
o

La distància entre els dos edificis és: 2  5, 4  7, 4 m

Com l’angle d’incidència és igual a l’angle de reflexió, s’utilitza la semblança de triangles per trobar
l’altura.
3 2
  AN  8,1 m
AN 5, 4

130. Les piràmides de Gizeh, a Egipte, són d’un gran interés arqueològic, artístic i històric.
Aquestes tres piràmides –Kheops, Kefren i Micerí– les van construir els antics egipcis com a
cambres mortuòries per als faraons que donen nom a les piràmides. La més gran, la de
Kheops, va ser considerada pels antics com una de les set meravelles del món, es tracta d’una
piràmide recta de base quadrada de 233 m de costat.

Calcula’n l’altura si quan ens en separem 80 m veiem el vèrtex amb un angle de 29º 30’ amb
l’horitzontal.

233
 116,5 m
2
h
tg 29o30’
  h  111,17 m
116,5m  80m
131. Una casa de planta rectangular fa 12 m de llarg i 8 m d’ample. La teulada, amb una inclinació
de 18º, és una superfície plana inclinada que té la part més elevada situada sobre un dels
costats grans del rectangle. Calcula l’àrea de la teulada.

Com sabem que la teulada té forma rectangular i que un dels seus costats té 12 m, trobem la longitud
de l’altre costat, x.
8
��� �8� � � � � � � 8������

Calculem l’àrea de la teulada:
A = 12 · 8,41 = 100,92 m2
L’àrea de la teulada és 100,92 m2

301
 
 
 
 
 
 

Trigonometria
Trigonometria 4
 
 
 
 
 
 

132. Determina l’altura a la qual caminen els senderistes que travessen un congost per un pont
penjant com el de la figura.

 
Anomenem y a l’altura del pont penjant.

�� ��� � �

� � ��� � ��� � ������ � ������� � ����� � �� � �� � ������
�� ��� � �
����

L’altura del pont penjant és de 39,8 m.

133. Demostra que la suma de les tangents dels tres angles d’un triangle és igual que el seu
producte.

La suma dels angles d’un triangle és 180º.


a + b + c = 180º
���� � ����
�� � � ������� � �� � ��� � ����� � �� � �
� � ���� � ����
Per tant, la suma de tangents és:
���� � ����
�� � � ���� � ���� � ����� � ���� � �
� � ���� � ����
���� � ����
���� � �� � � ������� � ���� � ����� � � ����� � ����
� � ���� � ����
� ���� � ���� � ���� � ����� � � ����� � ����
� ���� � ���� � ����� � ����� � ���� � ����

134. Les mides dels costats d’un triangle són proporcionals a 5, 6 i 7, i la seva àrea és ��√�.
Determina la mida dels costats i dels angles del triangle.

Com els costats són proporcionals,els triangles són semblants I els seus angles són iguals. Apliquem
el teorema del cosinus:

��� � �� � � � ��� � �� � ��
�� � �� � � � � ��� � ��� � � ��� � � �� � �� �
��� �����

� � ������ ��� ���

��� � �� � � � ��� � �� � ��
�� � �� � � � � ��� � ��� � � ��� � � �� � �� ����
��� �����

� � ����� �� ����

�� � ���� � ������ ��� ��� � ����� �� ���� � ������ �� ����

302
 
 
 
 
 
 

Trigonometría 54
 
 
 
 
 
 

Per trobar la longitud dels costats apliquem la fórmula d’Heró.

Si anomenem p el semiperímetre, llavors:

� � ���� � ���� � ���� � ��

�� � �� � �� ���
� � ��� � � ��� � � ��� � � � � ��
� �

��√� � ������ � ������ � ������ � ��� � �� ��� � �� � �� � �� � ��� � � � �

La mida dels costats és de 10, 12 i 14, respectivament.

135. Expressa les raons trigonomètriques següents mitjançant raons trigonomètriques de grau 1:

a) cos2 x b) sin2 x

1  cos 2 x 1  cos 2 x
a) cos 2 x  b) sin 2 x 
2 2

136. En l’estructura de la figura, els punts A, C i E están alineats i en coneixem les dades següents:

AB = 15 cm AC = 1m CW = 30°

DE = 2m WD = 14° WE = 124°

Calcula el valor de x.

 
o o o o
180  124  14 
42
2 a
Pel teorema del sinus:  o
 a 2, 48 m

sin 42 sin124o
h
sin 42o   h  2, 48  sin 42o  1, 66 m
a
b
cos 42o   b  2, 48  cos 42o  1,84 m
a
c
tg 60o   c  1, 66  tg 60o  2,88 m
h
x  2,88  1,84  4, 72 m

303
 
 
 
 
 
 

Trigonometria
Trigonometria 4
 
 
 
 
 
 

137. Sabem que tg z = 1,5. Amb aquestes dades, pots calcular tg (z+ ) sense determinar l’angle z?

Si apliques la fórmula de l’angle suma et costarà. Fes servir aquesta expressió:


࣊ ࣊ ࣊
ൌ ൅
૛ ૝ ૝

 
 
138. Dos angles inscrits en una circumferència que abasten el mateix arc tenen la mateixa mida.
Utilitza-ho per demostrar:

ࢇ ࢈ ࢉ
ൌ ൌ
෡ ෡ ෡
࢙࢏࢔࡭ ࢙࢏࢔࡮ ࢙࢏࢔࡯

en què d és el diàmetre de la circumferència circumscrita al triangle.

 
El triangle CBA’ és recte per ser un dels seus costats el diàmetre de la circumferència.
Els costats oposats als angles ‫ܤ‬෠݅‫ܦ‬
෡ són iguals.
෡ són iguals per abastar el mateix arc, així, els seus sinus són iguals.
Els angles ‫ܣ‬መ݅‫ܣ‬Ԣ
ࢇ ࢈ ࢇ
ൌ ൌ ࢇ ൌࢊ
෡ ෡
࢙࢏࢔࡭ ࢙࢏࢔࡭Ԣ

ࢇ ࢉ ࢈
ൌ ൌ ൌࢊ
࢙࢏࢔࡭ ෡ ࢙࢏࢔࡮
෡ ࢙࢏࢔࡯ ෡

304
 
 
 
 
 
 

Trigonometría 54
 
 
 
 
 
 

PER APROFUNDIR 
139. Escull la resposta adequada.

      1    
□   sin  cos   cos  cos  cos   sin  cos   cos  cos   
 32 32  16 8 4 2  16 16  8 4

1    1   1  1
  sin  cos   cos   sin  cos  sin  . 
4 8 8 4 8 4 4  16 2 16
□  Com el cosinus i el sinus sols poden valdre com a molt 1: 
2A  B 
  A  30 B  60
90 
A B 
 
BC
sin 30o   BC  2
4  
□  a  b  c   a  b  c   a2  b2  c 2  2ab  3ab  

c 2  a2  b2  2ab cosC  1 o
 cos C   C  60
2 2 2
c  a  b  ab  2
 
2 2 o 2
□  7  8  x  2  8  x cos 60  x  8 x  15  0  x1  5, x 2  3  

32  72  82  2  7  8  cos     21,79o  El tercer angle no seria agut.

52  72  82  2  7  8  cos     38,21o  Tots els angles són aguts.

El tercer costat mesura 5 m.

140. Per a quins valors de k té solució l’equaciósin x cos x = k ?

Determina’n les possibles solucions.

��� � � ����� � �� � ������� � ����� � ��� � ������ � ��

� �
Com sin x < |�| � � � � � �
� �

Les solucions estarán acotades en [0º, 180º] + 180º · k.

305
 
 
 
 
 
 

Trigonometria
Trigonometria 4
 
 
 
 
 
 

141. Demostra que la bisectriu interior de l’angle � en el triangle ABC divideix el costat oposat en
dos segments proporcionals als costats AB i AC.

Anomenem D al punt de tall de la bisectriu amb el costat CB.

Apliquem el teorema del sinus:

��
�� �� �� �� sin

� � → �
sin

� sin �
� sin �
� �� sin �


��
�� �� �� �� sin

� � → �
sin

� sin �� sin �� �� sin ��

Com que els angles són suplementaris, els seus sinus són iguals.
�� ��

�� ��
 

142. D’un port surten dos vaixells amb rumbs diferents. El rumb del primer és N 23º E, a una
velocitat d’11 milles/h. El segon navega en dirección S 67º E a 15 milles/h.

Calcula, aproximadament, el rumb des del segon vaixell cap al primer una hora després.

Resol el problema també en el cas que el segon angle sigui de 77º.

 180o  90o  67
  o
23o

112  152 
18, 6 milles
11
sin    36, 25o
18,6
    23o  36, 25o  59, 25o
El rumb des del segon vaixell cap al primer serà N 59,25o O.

o
En el cas que el segon angle sigui de 77 :

� � ���� � ��� � ��� � ���

d 2 112  15 2  2 11 15  cos 80 o  d  17 milles

sin  sin 80o


Pel teorema del sinus:  
  39,59o
11 17

306
 
 
 
 
 
 

Trigonometría 54
 
 
 
 
 
 

    13o  39,59o  52,59o


El rumb des del segon vaixell cap al primer serà N 52,59o O.

143. Un punt P dista 12 cm del centre d’una circumferència de 6 cm de radi. Determina l’angle que
formen entre elles les dues tangents traçades des d’aquest punt de la circumferència.

 
 sin   6  1    30o
En el triangle CPA
12 2
Com   2    60 , que és l’angle comprès entre les dues tangents.

144. Considera M el punt mitjà del segment d’extrems A i B; estudia el lloc geomètric dels punts P
del pla que verifiquen que PM és mitjana proporcional entre PA i PB.

PM és una mitjana del triangle PAB.

Apliquem el teorema del cosinus en els triangles PAM i PMB:

PA2 = d2 + PM2 - 2d · PM · cosહ

PB2 = d2 + PM2 - 2d · PM · cos (180º - હ) =

= d2 + PM2 - 2d · PM · cosહ

Sumant, s’obtè: PA2 + PB2 = 2d2 + 2PM2

Com PM2 = PA · PB, resulta que:

PA2 + PB2 = 2d2 + 2PA · PB → PA2 + PB2 – 2PA·PB = 2d2 → (PA – PB)2 = 2d2

307
 
 
 
 
 
 

Trigonometria
Trigonometria 4
 
 
 
 
 
 

Per tant, tenim que PA – PB = d √�, és a dir, la diferència de les distàncies de P als punts A i B és
constant, per la qual cosa el lloc demanat és una hipèrbole de focus A i B, així, la distància focal és

2c = 2d i l’eix real és 2a = d√�.

En una hipèrbole es verifica que b2 = c2 – a2:


�√� ��
�� � �� � � � �� �� � ��� � �√�
� �

Com que els dos eixos són iguals, la hipèrbole és equilàtera.


Una altra manera:

Siguin els punts A(-d , 0) i B(d , 0). El punt mig és l’origen de coordenades i P és el punt de
coordenades (x, y).
Es verifica que:
PM2 = PA · PB

�� � �� � ��� � ��� � �� � ��� � ��� � ��


Després d’operar i simplificar, resol l’equació:
��
�� � �� � , d’una hipèrbole equilàtera referida als seus eixos.

��
�� � �� � ���� � �� � �√�


�√�
�� � � �� � � �� � �� � � � � �� � � ���� � ��

Els seus focus són els punts A i B.

145. Calcula els valors dels cosinus dels angles x que verifiquen l’equació següent:

���� � � ����� �� � ������ � �

1  cos 2 x  1  cos 2 x  1
 2   sin 2 x 
0
2  2  2
sin 2x 1  cos 2x  2  2cos 2x
3cos 2x  sin 2x 1 0

308
 
 
 
 
 
 

Trigonometría 54
 
 
 
 
 
 

Sigui t  cos 2x . Així:


3
3t  1  t 2  1 0   3t  1 1  t 2  10t 2  6t 0  t1 0, t2 
2

5
3
t2   no és solució.
5
1  cos 2 x 2
cos x 
  cos x 

2 2
2 2
Les solucions són cos x  , cos x   .
2 2

146. Determina els dos valors enters de x més propers a 2 013º, tant per defecte com per excés, que
compleix aquesta equació trigonomètrica:
� �� � ��
���� � ���� � �√�
2 2
2sin x  21sin x 
2 2
2
Sigui t  2sin x

2
t  2 2  t 2  2 2t  2  0  t  2
t
2 1 2
2sin x  2  sin 2 x   sin x   x 45o , 135o , 225o , 315o
2 2
2 013 
5 360  213 
1800  213

Aproximació per defecte: 1800o  135o 


1935o
Aproximació per excés: 1800o  225o 
2 025o
 

   

309
 
 
 
 
 
 

Trigonometria
Trigonometria 4
 
 
 
 
 
 

MATEMÀTIQUES A LA TEVA VIDA 
 
1. Què és la fibra òptica? Quina condició compleix el feix de llum per restar en el nucli fins al final
de la Fibra òptica?

La fibra òptica és un fil molt fi de material transparent, vidre o materials plàstics, a través dels quals s’envien 
impulsos de llum que representen dades. 

Un feix de llum amb un angle d’incidència és més petit que l’angle de refracció, romandrà en el nucli fins a 
arribar al final de la fibra óptica. 

2. La conclusió del text anterior és que un feix lluminós amb l’angle d’incidència més petit que
l’angle de refracció, restarà en el nucli fins a arribar al final de la fibra òptica. Com es dedueix
aquesta conclusió a partir de la llei de Snell?

n2
1  sin 1 
Com que sin 90o 
n1

Però com que sin 1  1 és necessari que n1  n2 perquè hi hagi refracció i canviï de mitjà.

3. Investiga i enumera alguns avantatges i inconvenients d’utilitzar la fibra òptica en lloc del
cable de coure tradicional.

Resposta oberta. Per exemple:

▪ Avantatges: Té un cost més petit que el coure i permet transmetre una quantitat de dades més
gran per unitat de temps.

▪ Inconvenients: La fibra és més fràgil que el cable de coure, i a més la seva reparació és més difícil
en cas de trencament.

4. Si l’índex de refracció de l’aire és 1,0003 i l’angle límit a l’entrada d’una fibra òptica és 41,8º,
quin serà aleshores l’índex de refracció del material que forma la fibra òptica?

1,0003
n2  1,5  
sin 41,8o

310
Nombres complexos
Nombrescomplexos
6



ACTIVITATS
 (VFULXDTXHVWVQRPEUHVFRPDQRPEUHVFRPSOH[RV

D ξെ૜ F   ξെ૚૟
E  G ±

D  ξെ͵ ൌ ξ͵ ൉ ξെͳ ൌ ξ͵݅  F   L

ర ξଶ ξଶ
E  ξെͳ͸ ൌ ξͶ݅ ൌ ʹξ݅ ൌ ʹ ቀ ൅ ݅ቁ ൌ ξʹ ൅ ξʹ݅ G   L
ଶ ଶ

 &DOFXODDLESHUTXqOHVLJXDOWDWVVHJHQWVVLJXLQFHUWHV
D Ei D±   E D±E ±Di
a) a 21 3 b 0   

b) a 0 4a2b 2 b 1


‫ܡ‬
 'RQDWHOQRPEUHFRPSOH[] ±[ ࢏GHWHUPLQDHOYDORUGH[L\SHUTXqVLJXL

D 8QQRPEUHUHDO
E 8QQRPEUHLPDJLQDULSXU
F 8QQRPEUHFRPSOH[TXHQRVLJXLUHDOQLLPDJLQDULSXU
a)y 0  b)x 0  c)xz0,yz0

 &DOFXODO¶RSRVDWLHOFRQMXJDWGHOVQRPEUHVFRPSOH[RVVHJHQWV

D  ξ૛ െ ૜ܑ F  ±i H  ܑ J  

૛ ૚ ૛
E  െ ࢏ G  െ૜ ൅ ࢏ I  ± K  ±i
૜ ૞ ૞

a)Oposat:   i  Conjugat:  i 

   
b)Oposat:  i Conjugat: i
   

c)Oposat: 32i Conjugat: 32i

 
d)Oposat:  i  Conjugat:   i 
 

311
293

Nombres complexos
0QODTGUEQORNGZQU 5

 
e)Oposat:  i Conjugat:  i
 

f)Oposat: 7 Conjugat: 7

g)Oposat: 0 Conjugat: 0

h)Oposat: 2i Conjugat: i

 5HSUHVHQWDJUjILFDPHQWHOVQRPEUHVFRPSOH[RVVHJHQWV
૚ ૚ ૞
D  ൅ ܑ F  െ ܑ H  i J  
૛ ૛ ૛
૚ ૚
E  െ ൅ ܑ G  െ െ ܑ I  ± K  
૛ ૛

$UDFRQWHVWDRQVHUjVLWXDWXQQRPEUHUHDO",VLHOQRPEUHpVLPDJLQDULSXU"



8QQRPEUHUHDOVHVLWXDUjHQO¶HL[G¶DEVFLVVHV

8QQRPEUHLPDJLQDULSXUVHVLWXDUjHQO¶HL[G¶RUGHQDGHV

 (VFULXHOVQRPEUHVFRPSOH[RVUHSUHVHQWDWVJUjILFDPHQW

‫ݖ‬ଵ ൌ െͶ ൅ ʹ݅  ‫ݖ‬ଷ ൌ ݅  ‫ݖ‬ହ ൌ ͳ െ ͵݅

‫ݖ‬ଶ ൌ ͵ ൅ Ͷ݅  ‫ݖ‬ସ ൌ ͷ  ‫ ଺ݖ‬ൌ ͸ െ Ͷ݅



 5HVROOHVRSHUDFLRQVVHJHQWV
D  ±±i  ±i  F  ±±i ±i 
ି૚ି࢏ ሺି૛ା࢏ሻሺ૚ା૜࢏ሻ
E   G  െ ૛࢏
ି૝ା૞࢏ ି૚ା૛࢏


312
294

0QODTGUEQORNGZQU 65

D  L  L  L


ିଵି௜ ሺିଵି୧ሻାሺିସିହ୧ሻ ିଵାଽ௜
E  ൌ ൌ 
ିସାହ௜ ሺିସାହ୧ሻሺିସିହ୧ሻ ସଵ

F  ሺെͳ െ ‹ሻሺെͶ ൅ ͷ‹ሻ  ൌ Ͷ െ ͷ‹ ൅ Ͷ‹ ൅ ͷ ൌ ͻȂ ‹


ሺିଶା୧ሻሺଵାଷ୧ሻ ሺିହିହ୧ሻሺିଵିଶ୧ሻ
†ሻ െ ʹ‹ ൌ ሺିଵାଶ୧ሻሺିଵିଶ୧ሻ െ ʹ‹ ൌ ͵ െ ‹ െ ʹ‹ ൌ ͵ െ ͵‹
ିଵାଶ୧


 'HWHUPLQDO¶LQYHUVGHOVQRPEUHVFRPSOH[RVTXHKLKDDFRQWLQXDFLy
D  i F  ±±i  H  ±i
E  i G  i  I  i
  i i   i  i 
D   ¸   i   G   ¸  
i i i  i  i  i  
    i   i    i  i
E   ¸    H   ¸  
 i  i   i    i   i  i 
  i  i     i   i
F  ¸    I   ¸  
 i  i   i  i   i 

 'HWHUPLQDO¶H[SUHVVLySRODUGHOVQRPEUHVFRPSOH[RVUHSUHVHQWDWV


­r z    

°
z o ® o z o 

°tg D o D o
¯ 
­°r z  
z o ® o z o 
°̄D  o

­r z     
°
z o ®  o z  o 
° tg D o D   o

¯ 
­°r z  
z o ® o z  o 
°̄D o

­r z   


°
z o ® o z  o 

°tg D o D o
¯ 

313
295

Nombres complexos
0QODTGUEQORNGZQU 5

­r z   
 
  
°
z o ® o z   o 

°tgD o D o
¯ 

 ([SUHVVDHQIRUPDSRODU
૚ ૚
D  i F  െ ൅ ࢏ H  ±i
૛ ૛

E  ±±i G  ±ξ૜࢏ I  


D  i ξͷଶ଺ιଷଷᇲହସǡଶᇲᇲ  G  ξ͵݅ ξ͹ଷଵ଴ιହଷᇲ ଷ଺ǡଶᇲᇲ 

E  i ξͷଶ଴଺ιଷଷᇲ ହସᇲᇲ  H  Ͷ݅ Ͷଶ଻଴ι 


ଵ ଵ ξଶ
F  െ ൅ ݅ ൌ    I   ͳʹ଴ι 
ଶ ଶ ଶ ଵଷହι


 ([SUHVVDHQIRUPDELQzPLDLWULJRQRPqWULFD
ૈ ૜ૈ
D  ૚૚૛૙ι  E  ૜૛૝૙ι  F  ૛  G ૜ 
૜ ૛
ଵ ξଷ
E  ͳଵଶ଴ι ൌ  ቀെ ǡ ቁ ൌ ሺ…‘• ͳʹͲι ൅ ݅ •‹ ͳʹͲιሻ
ଶ ଶ

ଷ ଷξଷ
F  ͵ଶସ଴ι ൌ  ቀെ ǡ െ ቁ ൌ ͵ሺ…‘• ʹͶͲι ൅ ݅ •‹ ʹͶͲιሻ
ଶ ଶ

ξଷ గ గ
G  ʹഏ ൌ  ቀͳǡ ቁ ൌ ʹሺ…‘• ൅ ݅ •‹ ሻ
య ଶ ଷ ଷ
ଷగ ଷగ
H  ͵૜࣊ ൌ  ሺͲǡ െ͵ሻ ൌ ͵ሺ…‘• ൅ ݅ •‹ ሻ
૛ ଶ ଶ


 ([SUHVVDHQIRUPDSRODULWULJRQRPqWULFD
D  ±i E  i F  ±ξ૜i G  ±±i

D    i R  cos R  i VLQ R 

E    i   R   cos R  i VLQ R 

F     i R  cos R  i VLQ R 

G    i  R  cos R  i VLQ R 



 (IHFWXDOHVRSHUDFLRQVVHJHQWVLH[SUHVVDHOUHVXOWDWHQIRUPDSRODU
D  ૝૚૛૙ι ൉ ૜૟૙ι  F  ૚૛૟૙ι ൉ ૟૚૛૙ι  H  ૜૚૙૙ι ǣ ૜૝૙ι 
E  ૛૛૜૙ι ൉ ૜૚૜૙ι  G  ૝૙ι ǣ ૛૚ૡ૙ι  I  ૚૛૟૙ι ൉ ૟૚૛૙ι 

D  R ˜ R  ˜    


R R R 

E   R ˜ R  ˜    R R R  R 

314
296

0QODTGUEQORNGZQU 65

§· §·
F  R  R ¨ ¸ R R ¨ ¸ R
©  ¹   ©  ¹

§·
G  R  R ¨ ¸ R R R 
©  ¹  

§ ·
H  R  R ¨ ¸ R R R 
©  ¹  
I  R ˜ R ˜     R R R 


 'RQDWVDTXHVWVQRPEUHVFRPSOH[RVFDOFXOD

ࢠ૚ ൌ ૚૛૚૙ι  ࢠ૛ ൌ ૜ሾࢉ࢕࢙ሺെ૜૙ιሻ ൅ ࢏࢙࢏࢔ሺെ૜૙ιሻሿ

ࢠ૚ ሺࢠ૚ ሻ૛ ൉ࢠ૛
D   E  
ࢠ૛ ࢠ૛


‫ݖ‬ଵ ൌ ͳଶଵ଴ι  ‫ݖ‬ଶ ൌ ͵ଷଷ଴ι 
ଵమభబι ሺଵమభబι ሻమ ൉ଷయబι ଵరమబι ൉ଷయబι ଷరఱబι
D   E  ൌ ൌ ൌ ͳଵଶ଴ι 
ଷయయబι ଷయయబι ଷయయబι ଷయయబι


 (IHFWXDOHVRSHUDFLRQVVHJHQWV

D  ሺ૜૝૞ι ሻ૛  F  ቀ૛࣊ ቁ  H  ሺ૝૜૜૙ι ሻ૜ 


E  ሺ૜ െ ૜࢏ሻ૞  G  ൫ξ૞ ൅ ξ૞࢏൯  I  ሺെ૜࢏ሻ૞ 



D  R  ˜ R R i 

  
 
E    i

 R   R 
 ˜ R


 ¬
F  žžž Q ­­­   Q  Q   
žŸ  ®­ ¸


 
G   i 
 R
   ¸ R   R    


H  R
  ¸ R   R  i 


I  i
 o
  ¸ R  R  i 



 5HVRO> FRVƒiVLQƒ @āሺ૛૛૚૙ι ሻ૝ 
> cosqi sin q @ā q  qāq q q

 

315
297

Nombres complexos

a a a a

316
0QODTGUEQORNGZQU 65

(OPzGXOGHOHVVROXFLRQVVHUjO¶DUUHOF~ELFDGHPzGXO

([LVWLUDQWDQWVDUJXPHQWVFRPLQGLTXLHOUDGLFDO

ଶ଻଴ιା଴൉ଷ଺଴ι
6LN ՜Ⱦଵ ൌ ൌ ͸͹ι͵ͲԢ

ଶ଻଴ιାଵ൉ଷ଺଴ι
6LN ՜Ⱦଶ ൌ ൌ ͳͷ͹ι͵ͲԢ

ଶ଻଴ιାଶ൉ଷ଺଴ι
6LN ՜Ⱦଷ ൌ ൌ ʹͶ͹ι͵ͲԢ

ଶ଻଴ιାଷ൉ଷ଺଴ι
6LN ՜Ⱦସ ൌ ൌ ͵͵͹ι͵ͲԢ

3HUWDQWOHVDUUHOVVyQͳ଺଻ιଷ଴ᇲ ǡ ͳଵହ଻ιଷ଴ᇲ ǡ ͳଶସ଻ιଷ଴ᇲ ‹ͳଷଷ଻ιଷ଴ᇲ 

૜ య
G  ξെ૚ ൅ ࢏ ൌ  ටඥʹଵଷହι 


(OPzGXOGHOHVVROXFLRQVVHUjO¶DUUHOF~ELFDGHPzGXO ξʹ

([LVWLUDQWDQWVDUJXPHQWVFRPLQGLTXLHOUDGLFDO

ଵଷହιା଴൉ଷ଺଴ι
6LN ՜Ⱦଵ ൌ ൌ Ͷͷι

ଵଷହιାଵ൉ଷ଺଴ι
6LN ՜Ⱦଶ ൌ ൌ ͳ͸ͷι

ଵଷହιାଶ൉ଷ଺଴ι
6LN ՜Ⱦଷ ൌ ൌ ʹͺͷι

ల ల ల
3HUWDQWOHVDUUHOVVyQඥʹସହι ǡ ඥʹଵ଺ହι ǡ ‹ ඥʹଶ଼ହι 

 5HVROOHVHTXDFLRQVVHJHQWV
D  ࢠ૜ െ ૚ ൌ ૙ F  ࢠ૝ ൅ ૚૟ ൌ ૙
E  ࢠ૞ ൅ ૜૛ ൌ ૙ G  ࢠ૝ െ ૡ૚ ൌ ૙
a)z31 0 l z   R 

Elmòduldelessolucionsserà1.

Existirantantsargumentscomindiquielradical.

Sik 0 l  C  R 

R ¸ R
Sik 1 l C   R 


R  ¸ 
Sik 2 l C   R 


Pertant,elsvalorsdezsón10q,1120q,1240q.

b)z532 0 l z      R 

317
299

Nombres complexos
0QODTGUEQORNGZQU 5

Elmòduldelessolucionsserà2.

Existirantantsargumentscomindiquielradical.

R
Sik 0 l C   R 


R ¸ R
Sik 1 l C   R 


R  ¸ R
Sik 2 l C   R 


R  ¸ R
Sik 3 l C   R 


R  ¸ R
Sik 4 l C   R 


Pertant,elsvalorsdezsón236q,2108q,2180q,2252q,2324q.

c)z416 0 l z      R 

Elmòduldelessolucionsserà2.

Existirantantsargumentscomindiquielradical.

R
Sik 0 l C   R 


R R
Sik 1 l C   R 


R R ¸ 
Sik 2 l C   R 


R R ¸ 
Sik 3 l C   R 


Pertant,elsvalorsdezsón245q,2135q,2225q,2315q.

d)z481 0 l z      R 

Elmòduldelessolucionsserà3.

Existirantantsargumentscomindiquielradical.

Sik 0 l C  R 

318
300

0QODTGUEQORNGZQU 65

R
Sik 1 l C   R 


R ¸ 
Sik 2 l C   R 


R ¸ 
Sik 3 l C   R 


Pertant,elsvalorsdezsón30q,390q,3180q,3270q.
૚ା࢏
 &DOFXODLUHSUHVHQWDOHVDUUHOVF~ELTXHVG¶DTXHVWQRPEUH 
ି૚ି࢏
ͳ൅݅ ሺͳ ൅ ݅ሻሺെͳ ൅ ݅ሻ െͳ െ ͳ
ൌ ൌ ൌ െͳ ൌ ͳଵ଼଴ι 
െͳ െ ݅ ሺെͳ െ ݅ሻሺെͳ ൅ ݅ሻ ͳ൅ͳ

0zGXOξͳ ൌ ͳ

$UJXPHQWV

ଵ଼଴ιା଴൉ଷ଺଴ι
6LN ՜Ⱦଵ ൌ ൌ ͸Ͳι

ଵ଼଴ιାଵ൉ଷ଺଴ι
6LN ՜Ⱦଶ ൌ ൌ ͳͺͲι

ଵ଼଴ιାଶ൉ଷ଺଴ι
6LN ՜Ⱦଷ ൌ ൌ ͵ͲͲι  

 8QTXDGUDWDPEFHQWUHHQO¶RULJHQGHFRRUGHQDGHVWpXQGHOVYqUWH[VHQHOSXQW$  


'HWHUPLQD¶QHOVDOWUHVYqUWH[V
&DOFXOHPOHVDUUHOVTXDUWHVGHi

0zGXOξ͵ଶ ൅ ʹଶ ൌ  ξͳ͵$UJXPHQWVWJ‫ן ݃ݐ ן‬ൌ  ՜‫ן‬ൌ ͵͵ιͶͳᇱ ʹͶǡʹԢԢ

6XPHPžDO¶DUJXPHQWGHFDGDYqUWH[SHUREWHQLUHOVHJHQW

3HUWDQWOHVDUUHOVVyQξͳ͵ଷଷιସଵᇲ ଶସǡଶᇲᇲ ǡ ξͳ͵ଵଶଷιସଵᇲ ଶସǡଶᇲᇲ ǡ ξͳ͵ଶଵଷιସଵᇲ ଶସǡଶᇲᇲ ‹ξͳ͵ଷ଴ଷιସଵᇲ ଶସǡଶᇲᇲ 

319
301

Nombres complexos
0QODTGUEQORNGZQU 5

SABERFER

 5HVRODTXHVWHVHTXDFLRQVLH[SUHVVD¶QOHVVROXFLRQVDPEQRPEUHVFRPSOH[RV
D  x   x    
E  x   x    
F  x   x    
G  x    x    

 o   
D  x   x    l x    o i 


 o   
E  x   x    l x    o i 


 o  


F  x   x    l x    o i 


 o  
G  x   x    l x   oi 


 &DOFXODOHVRSHUDFLRQVVHJHQWV
i  i  i  i 
D   i 
 E  i 
 
 i  i   i   i 
i  i  i  i  i
  i
 i  i 

 i
D   
i   i   
 i  i  i  i  i  

i  i  i  i  i 
i  i
 i

E  i 
  i    i 
 i   i   i
 i 


 &DOFXODHOVLQYHUVRVG¶DTXHVWVQRPEUHVFRPSOH[RV

D  ] ±i E  ] i F ] i

  i  i  i
D  ¸   
  i  i  
  i  i
E  ¸  
 i  i 
  i  i
F  ¸  
 i  i 

 &DOFXODHOVLQYHUVRVG¶DTXHVWVQRPEUHVFRPSOH[RV
૜ି‫ܢ‬൉ܑା૛ܑ ૞ା૛‫ܢ‬൉ܑି૝ܑ
D  ൌ ‫ ܢ‬൅ ࢏ E  ൌ ‫ ܢ‬െ ૛࢏
૛ ૜

D    z ¸ i i   z i l    z zi l z  
 i

320
302

0QODTGUEQORNGZQU 65

   i   i
 z ¸  
 i i 
 i
E    zi  i  z  i l  i  z   i
l z  
  i
 i  i  i
 z ¸  
  i  i 

 &DOFXODHOVFRQMXJDWVGHOVQRPEUHVVHJHQWVHVFULWVHQIRUPDSRODU
D  ૛૜૜ι  E  ૜૛૛ι  F  ૚૚૙૞ι  G  ૛૛૛૛ι 

D (OFRQMXJDWGHqpVq q

E (OFRQMXJDWGHqpVq q

F (OFRQMXJDWGHqpVq q

G (OFRQMXJDWGHqpVq q

 &DOFXODHOVRSRVDWVGHOVQRPEUHVVHJHQWVHVFULWVHQIRUPDSRODU
D  ૛૜૜ι  E  ૜૛૛ι  F  ૚૚૙૞ι  G  ૛૛૛૛ι 
D /¶RSRVDWGHqpVqq q

E /¶RSRVDWGHqpVqq q

F /¶RSRVDWGHqpVqq q

G /¶RSRVDWGHqpVqq q

 &DOFXODHOVLQYHUVRVGHOVQRPEUHVFRPSOH[RVHQIRUPDSRODUVHJHQWV
D  ૛૜૜ι  E  ૜૛૛ι  F  ૚૚૙૞ι  G  ૛૛૛૛ι 

§· §·
D /¶LQYHUVGHqpV ¨ ¸ ¨ ¸ R 
©  ¹ R ©  ¹
§· §·
E /¶LQYHUVGHqpV ¨ ¸ ¨ ¸ R 
©  ¹ R ©  ¹
F /¶LQYHUVGHqpVq q

§· §·
G /¶LQYHUVGHqpV ¨ ¸ ¨ ¸ R 
©  ¹ R ©  ¹

 5HVROOHVRSHUDFLRQVVHJHQWV
D  ૛૜૙ι ൅ ૜૚૜૞ι െ ૜૛ૠ૙ι   E  ૚૝૞ι ൅ ૚૚૜૞ι ൅ ૚૛૛૞ι ൅ ૚૜૚૞ι 

321
303

Nombres complexos
0QODTGUEQORNGZQU 5

D  ((2cos30qi2sin30q)(3cos135qi3sin135q))(3cos270qi3sin270q) 

§    ·   §  ·
 ¨¨   i  i ¸¸  i   i ¨¨   ¸¸ 
©   ¹  ©  ¹
E (cos45qisin45q)(cos135qisin135q)(cos225qisin225q)(cos315qisin315q) 

       
i  i  i  i 
         

 )HVO¶RSHUDFLyVHJHQW


   ¬­­ 
ž
žžž   i­­    i
Ÿ ®

     

 ¬
žž  i­­
žž  ­
Ÿ  ­®


  ¬
žž  i­­    i 

žž 
Ÿ  ­®
­



 R


R R  R
R
     R 
 ¬

  ¬ ¬­

  ¬­   ­¬
žž   i­­ žžž ­­ ­ ž
žžŸ  ­­®R
ž
žžŸ  ­­®R
­ ž ­
žž 
Ÿ  ®­ ŸžŸ  ­®R ­®
žž


 'HWHUPLQDOHVFRRUGHQDGHVGHOVYqUWH[VGHOWULDQJOH$%&VLVDSVTXHVyQHOVDIL[RVGHOHV
DUUHOVF~ELTXHVGH±


   R 

Elmòduldelessolucionsseràl’arrelcúbicade27queés3.

Existirantantsargumentscomindiquielradical.

R
Sik 0 l  C   R 


o
R  ¸
Sik 1 l C   R 


R o ¸ 
Sik 2 l C   R 


Pertant,elsvalorsdelesarrelssón360q,3180q,3300q.

Lescoordenadesdelsvèrtexssón:

    ¬­
A(3cos60q,3sin60q)  žžž  ­­ 
žŸ   ­®

322
304

0QODTGUEQORNGZQU 65

B(3cos180q,3sin180q) (3,0)

    ­¬
C(3cos300q,3sin300q)  žžž   ­­ 
žŸ   ­®

 &DOFXODWRWHVOHVVROXFLRQVGHOHVHTXDFLRQVVHJHQWV
D  ࢠ૝ െ ૚૟ ൌ ૙ E  ࢠ૜ ൅ ૡ ൌ ૙ F  ࢠ૝ െ ૢ ൌ ૙ G  ࢠ૜ ൅ ૢ ൌ ૙
D  z   R l z qz qz qz q

E  z   R l z qz qz q

F  z   R l z   R  z   R  z  R  z   R 

G  z   R l z    R  z    R  z    R 

 

323
305

Nombres complexos
0QODTGUEQORNGZQU 5

ACTIVITATSFINALS
 ([SUHVVDDTXHVWVQRPEUHVFRPSOH[RVHQIRUPDELQzPLD
D  ξെ૚૟ ൅ ૜ E  െ૛ െ ξെ૝ F  ξെૡ ൅ ξ૛
a) ξെͳ͸ ൅ ͵ ൌ ͵ ൅ Ͷ݅

b) െʹ െ ξെͶ ൌ  െʹ െ ʹ݅

c) ξെͺ ൅ ξʹ ൌ ξʹ ൅  ξͺi

 ([SUHVVDDTXHVWVQRPEUHVFRPSOH[RVHQIRUPDELQzPLD




   i
z   i   z   i   z    z  i 
 
  
z   i   z    i   z   i 
  

 5HSUHVHQWDDTXHVWVQRPEUHVHQHOSODFRPSOH[
૜ ૞
D  i G  i J  െ  ࢏
૛ ૜

E  ξ૛ െ ૜࢏ H   K  ξ૜ െ ૛࢏


F  െ૜ െ ࢏ I  i L  ξ૜࢏

    

 5HVROOHVHTXDFLRQVVHJHQWVLH[SUHVVD¶QOHVVROXFLRQVPLWMDQoDQWQRPEUHVFRPSOH[RV
D  ࢞૛ ൅ ૠ ൌ െ૝૛
E  െ࢞૛ െ ૟૝ ൌ ૙

324
306

0QODTGUEQORNGZQU 65

F  ૚ െ ሺെ࢞૛ ሻ ൌ െ૚૛૙
G  െ૜ ൅ ࢞૛ ൌ ૛࢞૛ ൅ ૚
H  ሺ࢞ െ ૚૙ሻ૛ ൌ െ૛૙࢞
D x2 49ox  o   oi 

E x2 64ox  o   oi 

F  x2 121ox  o   oi 

G x2 4ox  o   oi 

H x2 100ox  o   oi 

 (VFULXLGLEXL[DHOFRQMXJDWLO¶RSRVDWGHOVQRPEUHVFRPSOH[RVVHJHQWV
D  ±i G  ±
E  ±i H  ±±i
F  ±i I  i
a) Conjugat: 32i Oposat: 32i (vermell)

b) Conjugat:3i Oposat: 3i (verd)

c) Conjugat:13i Oposat: 13i (blau)

d) Conjugat:4 Oposat: 4 (rosa)

e) Conjugat:22i Oposat: 22i (groc)

f) Conjugat: 4i Oposat: 4i (morat)


 (VFULXHOFRQMXJDWLO¶RSRVDWGHOVQRPEUHVFRPSOH[RVVHJHQWV

 ±i ±i  ±i

7HQLQWHQFRPSWHDTXHVWVH[HPSOHVGHGXHL[
D &RPpVODUHSUHVHQWDFLyGHOFRQMXJDWG¶XQQRPEUHFRPSOH["
E &RPpVODUHSUHVHQWDFLyGHO¶RSRVDWG¶XQQRPEUHFRPSOH["

Conjugat:23i Oposat:23i

Conjugat:32i Oposat:32i

Conjugat:2i Oposat:2i

a) Éssimètricarespectedel’eixX.

b) Éssimètricarespectedel’origendecoordenades.

325
307

Nombres complexos
0QODTGUEQORNGZQU 5

 &DOFXODHOYDORUGHNSHUTXqHOQRPEUHN N± iYHULILTXLOHVFRQGLFLRQVVHJHQWV


D 4XHVLJXLXQQRPEUHLPDJLQDULSXU
E 4XHVLJXLXQQRPEUHUHDO
a)k 0   b)k3 0 l k 3


 &DOFXODLUHSUHVHQWDHQHOSODFRPSOH[HOVQRPEUHV
 iLLLLL«
,QYHVWLJDWDPEpTXqSDVVDDPEHOVQRPEUHV
 iii iii«
݅ ସ௡ିଷ ൌ ݅ ݅ ିସ௡ାଷ ൌ െ݅

݅ ସ௡ିଶ ൌ െͳ ݅ ିସ௡ାଶ ൌ െͳ

݅ ସ௡ିଵ ൌ െ݅ ݅ ିସ௡ାଵ ൌ ݅

݅ ସ௡ ൌ ͳ  ݅ ିସ௡ ൌ ͳ

      

 &DOFXODHOUHVXOWDWG¶DTXHVWHVRSHUDFLRQV
D ሺ૝ െ ࢏ሻ ൅ ሺെ૛ ൅ ૜࢏ሻ F ૞ െ ሺ૛ െ ࢏ሻ
૚ ૚ ૚ ૚ ૛ ૚ ૛
E  ቀ െ ܑቁ െ ቀെ ൅ ܑቁ G  ቀ െ ܑቁ ൅ ቀ െ ܑቁ
૜ ૛ ૜ ૝ ૞ ૜ ૞

   
D 22i E   i F  3i G   i 
   

 (IHFWXDOHVRSHUDFLRQVVHJHQWV
D ሺ૜ െ ૞࢏ሻ ൅ ሺ૛ െ ૠ࢏ሻ ൅ ሺെ૝ ൅ ૡ࢏ሻ
E  ሺെ૚ ൅ ૛࢏ሻ െ ሺ૜ ൅ ૟࢏ሻ െ ሺെ૝ െ ࢏ሻ
F െሺ૚ െ ૛࢏ሻ െ ሺെૠ࢏ሻ െ ሺെ૝ െ ૜࢏ሻ
G ૛ሺ૚ െ ૝࢏ሻ െ ૛ሺ૚ ൅ ૝࢏ሻ െ ૜ሺ૝ െ ૝࢏ሻ
H ૛൫ξ૜ ൅ ࢏൯ െ ૜൫૛ξ૜ ൅ ૝࢏൯
I ൫ξ૛ െ ૜࢏൯ ൅ ૛൫૛ െ ξ૜࢏൯
D  ሺ͵ െ ͷ݅ሻ ൅ ሺʹ െ ͹݅ሻ ൅ ሺെͶ ൅ ͺ݅ሻ ൌ ͳ െ Ͷ݅
E  ሺെͳ ൅ ʹ݅ሻ െ ሺ͵ ൅ ͸݅ሻ െ ሺെͶ െ ݅ሻ ൌ ሺെͳ ൅ ʹ݅ሻ ൅ ሺെ͵ െ ͸݅ሻ െ ሺെͶ െ ݅ሻ ൌ െ͵݅

326
308

0QODTGUEQORNGZQU 65

F  െሺͳ െ ʹ݅ሻ െ ሺെ͹݅ሻ െ ሺെͶ െ ͵݅ሻ ൌ െሺͳ ൅ ʹ݅ሻ ൅ ሺ͹݅ሻ ൅ ሺͶ ൅ ͵݅ሻ ൌ ͵ ൅ ͳʹ݅
G  ʹሺͳ െ Ͷ݅ሻ െ ʹሺͳ ൅ Ͷ݅ሻ െ ͵ሺͶ െ Ͷ݅ሻ ൌ െͳʹ െ Ͷ݅
H  ʹ൫ξ͵ ൅ ݅൯ െ ͵൫ʹξ͵ ൅ Ͷ݅൯ ൌ ʹ൫ξ͵ ൅ ʹ݅൯ ൅ ൫െ͸ξ͵ െ ͳʹ݅൯ ൌ െͶξ͵ െ ͳͲ݅
I  ൫ξʹ െ ͵݅൯ ൅ ʹ൫ʹ െ ξ͵݅൯ ൌ ൫ξʹ െ ͵݅൯ ൅ ൫Ͷ െ ʹξ͵݅൯ ൌ ൫ξʹ െ Ͷ൯ െ ൫͵ ൅ ʹξ͵൯݅

 &DOFXODHOVSURGXFWHVLOHVSRWqQFLHVVHJHQWV
D ሺ૚ െ ૜࢏ሻሺ૛ െ ૟࢏ሻ G ሺ૞ െ ૝࢏ሻሺ૞ ൅ ૝࢏ሻ
E ሺെ૜ െ ૝࢏ሻሺૠ െ ࢏ሻ H ൫െ૜ െ ૛ξ૛࢏൯൫െ૜ ൅ ૛ξ૛࢏൯

F ሺെ૛ ൅ ૞࢏ሻ૛   I ൫ξ૛ െ ࢏൯ 
D  ሺͳ െ ͵݅ሻሺʹ െ ͸݅ሻ ൌ ʹ െ ͸݅ െ ͸݅ െ ͳͺ ൌ െͳ͸ െ ͳʹ݅
E  ሺെ͵ െ Ͷ݅ሻሺ͹ െ ݅ሻ ൌ െʹͳ ൅ ͵݅ െ ʹͺ݅ െ Ͷ ൌ െʹͷ െ ʹͷ݅
F  ሺെʹ ൅ ͷ݅ሻଶ ൌ Ͷ െ ʹͷ െ ʹͲ݅ ൌ െʹͳ െ ʹͲ݅
G  ሺͷ െ Ͷ݅ሻሺͷ ൅ Ͷ݅ሻ ൌ ʹͷ ൌ Ͷͳ
H  ൫െ͵ െ ʹξʹ݅൯൫െ͵ ൅ ʹξʹ݅൯ ൌ ͻ ൅ ͺ ൌ ͳ͹

I  ൫ξʹ െ ݅൯ ൌ ξʹଷ െ ͸݅ െ ͵ξʹ ൅ ݅ ൌ െξʹ െ ͷ݅

 (IHFWXDDTXHVWHVGLYLVLRQV
ି૚ା૞࢏ ૛૙ା૝૙࢏ ି૚ା૞࢏
D   E   F  
૜ି૛࢏ ૡା૟࢏ ૛ି࢏

ି૚ା૞࢏ ሺି૚ା૞࢏ሻሺ૜ା૛࢏ሻ ି૜ି૛࢏ା૚૞࢏ି૚૙ ି૚૜ା૚૜࢏


D  ൌ ൌ ൌ ൌ െ૚ ൅ ࢏
૜ି૛࢏ ሺ૜ି૛࢏ሻሺ૜ା૛࢏ሻ ૢା૝ ૚૜

૛૙ା૝૙࢏ ሺ૛૙ା૝૙࢏ሻሺૡି૟࢏ሻ ૚૟૙ି૚૛૙࢏ା૜૛૙࢏ା૛૝૙


E  ൌ ൌ ൌ ૝ ൅ ૛࢏
ૡା૟࢏ ሺૡା૟࢏ሻሺૡି૟࢏ሻ ૟૝ା૜૟

ି૚ା૞࢏ ሺି૚ା૞࢏ሻሺ૛ା࢏ሻ ି૛ି࢏ା૚૙࢏ି૞ ିૠାૢ࢏


F  ൌ ൌ ൌ 
૛ି࢏ ሺ૛ି࢏ሻሺ૛ା࢏ሻ ૝ା૚ ૞

 )HVOHVGLYLVLRQVGHQRPEUHVFRPSOH[RVVHJHQWV
D  ሺ૜ െ ࢏ሻǣ ሺ૚ െ ࢏ሻ F  ሺ૞ ൅ ૛࢏ሻǣ ሺ૛࢏ሻ
૞ ξ૛
E   G  
૛ା૝࢏ ૚ାξ૛࢏

  i  i  i    i i i   i  


D  ¸    i   F  ˜ 
 i  i  i i  

   i   i   i    i   i
E  ˜   G  ˜ 
  i   i     i   i 

 'HWHUPLQDHOUHVXOWDWGHOHVRSHUDFLRQVVHJHQWVHQIRUPDELQzPLD
૜૙ሺ૚ି࢏ሻ
D  ൅ ሺ૛ െ ૜࢏ሻ࢏
ି૝ି૛࢏

327
309

Nombres complexos
0QODTGUEQORNGZQU 5

ሺ૛ା૜࢏ሻ૜
E  ૛࢏ െ 
ି૜ା࢏
૝ሺ૚૙ି࢏ሻାૡ
F  െ ሺ૜ െ ࢏ሻሺ૛ ൅ ૟࢏ሻ
૛ି૟࢏
૚૙ି૚૙࢏ି૞ሺ૚ା࢏ሻ
G  ሺെ૛ െ ૞࢏ሻ െ 
ሺૡା૛࢏ሻିሺ૞ା૜࢏ሻ

ሺ૚ା૜࢏ሻ૛ ିሺ૛࢏ሻ૛
H  
ି૜ା૝࢏



 (IHFWXDO¶RSHUDFLyVHJHQW

   i   i    i
   
i i i    i  

   i   i    i   i   i   i
      
i i i    i   i i i  i 


  i i  
˜ 
  i i   i 
i  i  

 &DOFXODLVLPSOLILFDOHVH[SUHVVLRQVTXHKLKDDFRQWLQXDFLy
D  ࢏૝ૢ ൉ ࢏ૡૠ  F  ሺെ૜࢏ሻ૜ ൅ ሺ૛࢏ሻ૟ ǣ ሺ૚૛࢏૚ૡ ሻ
E  ࢏૜૝ ൉ ࢏૚૙૜ ൅ ࢏ૠૡ ൉ ࢏૚૚૟  G  ࢏૚ૢ ൉ ሺ૛࢏૜૜ െ ૜࢏૛ૡ ሻ

a) i49ͼi87 i136 1 c) (3i)3(2i)6:(12i18) 27i64:(12) 27i 


b) i34ͼi103i78ͼi116 i137i194 i1 d) i19ͼ(2i33–3i28) iͼ(2i3) 23i


 5HSUHVHQWDL0XOWLSOLFDKRSHUiLUHSUHVHQWDHOUHVXOWDW0XOWLSOLFDKRGXHVYHJDGHVSHU iL
H[SOLFDTXqREWHQV
i)i i

i)i2 i

328
310

0QODTGUEQORNGZQU 65

4XDQPXOWLSOLTXHPSHUiHOSXQWHVGHVSODoDžPLWMDQoDQWXQJLUGHFHQWUHO¶RULJHQHQHOVHQWLW
FRQWUDULDOHVDJXOOHVGHOUHOORWJH

4XDQPXOWLSOLTXHPSHUiREWHQLPHOSXQWVLPqWULFUHVSHFWHGHO¶RULJHQ HOVHXRSRVDW 



 &RPSURYDVLHOVYDORUVGH]TXHHVGRQHQVyQVROXFLRQVGHOHVHTXDFLRQVFRUUHVSRQHQWV

(12i)22(12i)5 0

14i424i5 0

6ípVVROXFLy

(1i)2(3i)(1i)(44i) 0

12i132i144i 0

6ípVVROXFLy


§   ·
¨¨   i ¸¸   
©   ¹

§   ·§   ·
¨¨   i ¸¨
¸¨   i ¸¸      
©   ¹©   ¹
1RpVVROXFLy

 &DOFXODHOYDORUGHNSHUTXqOHVH[SUHVVLRQVVHJHQWVVLJXLQQRPEUHVLPDJLQDULVSXUV
૜ା࢑࢏ ૚ି࢑࢏
D   E  Niā 
࢑ା૛࢏ ૚ି࢏


329
311

Nombres complexos
0QODTGUEQORNGZQU 5

 ki k  i k i  k

a) ¸  lk 
k i k  i k 

 ki  i ki k
 i
k k i k k

  

b) ki ¸ ¸   l k  
 i  i  
Noconsideremlasoluciók 0perquèelnombreseriazero.

 7UREDSLTSHUTXqHVFRPSOHL[LODLJXDOWDWVHJHQW
 ሺ࢖ ൅ ૜࢏ሻሺ૝ ൅ ࢗ࢏ሻ ൌ ૚૞ ൅ ૢ࢏
ሺ‫ ݌‬൅ ͵݅ሻሺͶ ൅ ‫݅ݍ‬ሻ ൌ ͳͷ ൅ ͻ݅ ՜ ሺͶ‫ ݌‬െ ͵‫ݍ‬ሻ ൅ ሺͳʹ ൅ ‫ݍ݌‬ሻ݅ ൌ ͳͷ ൅ ͻ݅

‫݌‬ଵ ൌ ͵Ǣ ‫ݍ‬ଵ ൌ െͳ
Ͷ‫ ݌‬െ ͵‫ ݍ‬ൌ ͳͷ
൝ ՜൝ ͵ 
ͳʹ ൅ ‫ ݍ݌‬ൌ ͻ ‫݌‬ଶ ൌ Ǣ ‫ݍ‬ଶ ൌ െͶ
Ͷ

 'HPRVWUDTXHHOQRPEUHFRPSOH[] iYHULILFDODLJXDOWDW


ࢠ૛
ൌ ࢠ െ ૞

‫ݖ‬ଶ ሺͳ െ ͵݅ሻଶ െͺ െ ͸݅
ൌ ൌ ൌ െͶ െ ͵݅ ൌ ͳ െ ͵݅ െ ͷ ൌ ‫ ݖ‬െ ͷ
ʹ ʹ ʹ

 /DVXPDGHGRVQRPEUHVFRPSOH[RVpVi LODSDUWUHDOGHOVHJRQpV&DOFXODHOVGRV
QRPEUHVVLVDSVTXHHOTXRFLHQWGHOSULPHUHQWUHHOVHJRQpVXQQRPEUHLPDJLQDULSXU
ሺƒ ൅ ܾ݅ሻ ൅ ሺʹ ൅ ݀݅ሻ ൌ ͵ ൅ ʹ݅ ՜ ƒ ൌ ͳǡ ܾ ൅ ݀ ൌ ʹ

ͳ ൅ ܾ݅ ʹ െ ݀݅ ʹ ൅ ܾ݀ ൅ ሺʹܾ െ ݀ሻ݅
൉ ൌ ՜ ʹ ൅ ܾ݀ ൌ Ͳ
ʹ ൅ ݀݅ ʹ െ ݀݅ ʹ ൅ ݀ଶ

ܾ൅݀ ൌʹ
ቄ ՜ ݀ ൌ ͳ ൅ ξ͵ǡ ܾ ൌ ͳ െ ξ͵‫ܾ݋‬±݀ ൌ ͳ െ ξ͵ǡ ܾ ൌ ͳ ൅ ξ͵
ʹ ൅ ܾ݀ ൌ Ͳ

Elsnombressón:

z     i \z     i


z    
i \z     
i

 'HWHUPLQDGRVQRPEUHVFRPSOH[RVVLVDEHPTXHODVHYDVXPDpVLTXHHOVHXTXRFLHQWpVi

abi  cdi  oac bd 

a bi   c  di c  di c  c   d   di
 ¸   i l c  c   d   di  i c  d 

c di c di c  di c d 

330
312

0QODTGUEQORNGZQU 65

c  c   d   ²¦¦    
 ¦
» l c    d    a    b  

d  c d ¦¼    

   
(OVQRPEUHVVyQ z  i \z   i 
   

 5HSUHVHQWDJUjILFDPHQWDTXHVWVQRPEUHVFRPSOH[RVLH[SUHVVD¶OVHQIRUPDELQzPLD
D  ૚૟૙ι  F  ૛૚૜૞ι  H  ξ૞૟૙ι 
E  ૞ૢ૙ι  F  ૜૚૛૙ι  H  ξ૜૝૞ι 

 
D cos60qisin60q  i 
 
E 5cos90qisin90q 5i

F  2cos135qi2sin135q    i  

  
G 3cos120qi3sin120q   i 
 

 
H   cos R i  VLQ R  i 
  
 
I   cos R i  VLQ R  i 
 

 ([SUHVVDDTXHVWVQRPEUHVFRPSOH[RVHQIRUPDSRODU


z1 34i 553,13q  z2 2 20q   z3 3i 3270q

z   i   R   z   i   R 


 (VFULXHOVQRPEUHVVHJHQWVHQIRUPDSRODULUHSUHVHQWD¶OVJUjILFDPHQW
D ±i G ±i
E ξ૜i H ±

F ±ξ૛±ξ૛i I  ࢏

a) ͵ െ Ͷ݅ ൌ ͷଷ଴଺ιହଶᇲ ଵଵǡ଺ଷᇲᇲ 

b) ξ͵ ൅ ݅ ൌ ʹଷ଴ι 
c) െξʹ െ ξʹ݅ ൌ ʹଶଶହι 

331
313

Nombres complexos
0QODTGUEQORNGZQU 5

d) െ͵݅ ൌ ͵ଶ଻଴ι 


e) െ͵ ൌ ͵ଵ଼଴ι 
ଵ ଵ
f) ݅ൌ 
ଶ ଶవబι


 (VFULXHQIRUPDELQzPLDHOVQRPEUHVFRPSOH[RVVHJHQWV
D  ૝૟૙ι  F  ૜࣊  H  ૜૚૞૙ι  J  ξ૛ૠ࣊ 
૛ ૝

E  ૛૛૚૞ι  G  ૛࣊  I  ૚૜࣊  K  ξ૜૜૙૙ι 


ଷξଷ ଷ
D  ʹ ൅ ʹξ͵݅ F  ͵݅ H  െ ൅ ݅ J  ͳ െ ݅
ଶ ଶ

ξଷ ଷ
E  െͳǡ͸Ͷ െ ͳǡͳͷ݅ G   I  െ݅ K  െ ݅
ଶ ଶ


 'RQDWVDTXHVWVQRPEUHVFRPSOH[RV
ࢠ૚ ൌ ૞૛૝૙ι ࢠ૚ ൌ ૜૚૜૞ι ࢠ૚ ൌ ξ૜࣊ 

HVFULXQHHOFRQMXJDWLO¶RSRVDWGHFDGDVFXQHQIRUPDSRODULELQzPLD

7HQLPHQFRQVLGHUDFLyTXHHOFRQMXJDWpVHOSXQWVLPqWULFUHVSHFWHO¶HL[G¶DEVFLVVHVLO¶RSRVDWpVHO
VLPqWULFUHVSHFWHGHO¶RULJHQ


Nombre Conjugat Oposat
Polar Binòmia Polar Binòmia Polar Binòmia
૞૛૝૙ι  ͷ ͷξ͵ ͷଵଶ଴ι ͷ ͷξ͵ ͷ଺଴ι ͷ ͷξ͵
ቆെ ǡ െ ቇ ቆെ ǡ ቇ ቆ ǡ ቇ
ʹ ʹ ʹ ʹ ʹ ʹ
૜૚૜૞ι  ͵ξʹ ͵ξʹ ͵ଶଶହι ͵ξʹ ͵ξʹ ͵ଷଵହι ͵ξʹ ͵ξʹ
ቆെ ǡ ቇ ቆെ ǡെ ቇ ቆ ǡെ ቇ
ʹ ʹ ʹ ʹ ʹ ʹ

ξ૜࣊  ͵ ξ͵ ξ͵ଵଵగ  ͵ ξ͵ ξ͵଻గ  ͵ ξ͵


૟ ቆ ǡ ቇ ଺ ቆ ǡെ ቇ ଺ ቆെ ǡ െ ቇ
ʹ ʹ ʹ ʹ ʹ ʹ

 (VFULXHQIRUPDSRODUHOVQRPEUHVFRPSOH[RVTXHKLKDDFRQWLQXDFLy
࣊ ࣊ ࣊ ࣊
D ξ૜ሺ‫ ܛܗ܋‬൅ ࢏ ‫ ܖܑܛ‬ሻ E  ૜ሺ‫ ܛܗ܋‬൅ ࢏ ‫ ܖܑܛ‬ሻ
૟ ૟ ૛ ૛

D   Q     E   Q 



332
314

0QODTGUEQORNGZQU 65

 'RQDWHOQRPEUHHVFULWHQIRUPDSRODU‫ܚ‬હ GLJXHVFRPVHULHQO¶RSRVDWLHOFRQMXJDW


FRUUHVSRQHQWV

2SRVDWrD rqD  &RQMXJDWrD



 (IHFWXDOHVRSHUDFLRQVVHJHQWVDPEQRPEUHVFRPSOH[RV
 S

D  ૝૚૛૙ι ൉ ૜૟૙ι  G  ž  J  
 
ž S



 § ·
E  S  H  ¨¨  S ¸¸  K  q 
S ©  ¹


ž
F   S ˜  ž  I  qāq L  

ž

 S
  
a)4120qͼ360q 12180q  d) ž ž    g) 
ž S S




S § ·
b) S    e) ¨¨ S ¸¸  S   h)(2120q)5 32600q=32240q
S  ©  ¹ 


ž
c)  S ˜ ž ž ˜ ž ž  f)2260qͼ5130q=10390q=1030q i) ž 

ž


 ([SUHVVDHQIRUPDSRODUO¶LQYHUVG¶DTXHVWVQRPEUHV
D  ૛૚૞૙ι  F  ૝࣊  H  ࢋ࣊ 
૜ ૟


E  ࢋ࣊  G  ቀ૝ቁS I  ξૠ૜૞ι 

3HUFDOFXODUO¶LQYHUVG¶XQQRPEUHHQIRUPDSRODUFDOFXOHPO¶LQYHUVGHOPzGXOLO¶RSRVDWGHO¶DUJXPHQW
¬ ¬ ¬
a) žž ­­­    c) žž ­­­    e) žž ­­­ 
žŸ  ® R žŸ  ®Q žŸ e ®Q

 

¬  ¬
b) žž ­­­    d)4S    f) žžž ­­­ 
žŸ  ®Q žŸ  ®­R



 &DOFXODOHVSRWqQFLHVGHQRPEUHVFRPSOH[RVVHJHQWVHQIRUPDSRODU
૛ ૟
D  ሺ૛૚૙૞ι ሻ૝  E  ቀ૝࣊ ቁ  F  ሺ૜ି૛૞ι ሻ૟  G  ൬ξ૞૜࣊ ൰ 
૜ ૝

a) 1660q b) 162S/3 c) 729210q d) 1259S/2



 

333
315

Nombres complexos
0QODTGUEQORNGZQU 5

 (IHFWXDOHVRSHUDFLRQVFRPELQDGHVGHQRPEUHVFRPSOH[RVVHJHQWV
૟૟૙ι ൉૜૝૙ι
D  ሺ૝૛૙ι ൉ ૚૞૙ι ሻ ൉ ૜૜૞ι  F  
ૢ૞૟ι

E  ሺૢ૜ૢι ǣ ૜૛૞ι ሻ ൉ ૞૚૙૙ι  G  ሺ૚૚૙૞ι ሻૡ ൉ ሺ૚૟૞ι ሻ૞ 


R
D 470qͼ335q 12105q F   R 
R

E 314qͼ5100q 15114q G 1120qͼ1325q 185q



 )HVOHVRSHUDFLRQVFRPELQDGHVDPEQRPEUHVFRPSOH[RVVHJHQWV

ሺ૚ାඥ૜࢏ሻ ି૝૜࣊
D  ሺ૝ െ ૛࢏ሻ െ ൫૜ െ ૛ξ૛࢏൯ ൉ ૛૜૙ι  E  ૛

ି૜ା૜࢏

  i       i   
 
D  i  i   i   i   



            i   i    i     i     i  



2
 i R  i    i  i   i      i   i     
E  ˜ i 
  i   i   i   i   

 )HVOHVRSHUDFLRQVVHJHQWVH[SUHVVDQWSULPHUHOVQRPEUHVHQIRUPDSRODU

D  ሺ૚ െ ࢏ሻ૝  F  ൫െ૚ ൅ ξ૜࢏൯ 
૟ ૠ
E  ൫െξ૛ ൅ ξ૛࢏൯  G  ൫ξ૛ ൅ ࢏൯ 
ସ ସ
D  ሺͳ െ ݅ሻସ ൌ ൫ξʹଷଵହι ൯ ൌ Ͷଵ଼଴ι  F  ൫െͳ ൅ ξ͵݅൯ ൌ ሺʹଵଶ଴ι ሻସ ൌ ͳ͸ଵଶ଴ι 
଺ ଻
E  ൫െξʹ ൅ ξʹ݅൯ ൌ ሺʹଵଷହι ሻ଺ ൌ ͸Ͷଽ଴ι  G  ൫ξʹ ൅ ݅൯ ൌ ξ͵ଷଶସιସସᇲ଼Ǥଶᇲᇲ 

 5HSUHVHQWDDTXHVWVQRPEUHVLHOVUHVXOWDWVGHOHVRSHUDFLRQVHQHOSODFRPSOH[([SOLFDTXq
SDVVDHQFDGDFDV
૟૚૞૙ι ૝
D  ૛૚૞૙ι ൉ ૜૚૛૙ι  E   F  ቀ૛࣊ ቁ 
૛૟૙ι ૜

D  (OPzGXOGHOUHVXOWDWpVHOSURGXFWHGHOVPzGXOVLO¶DUJXPHQWpVODVXPDGHOVDUJXPHQWVGHOV
QRPEUHVGRQDWV

334
316

0QODTGUEQORNGZQU 65

E  (OPzGXOGHOUHVXOWDWpVHOTXRFLHQWGHOVPzGXOVLO¶DUJXPHQWpVODUHVWDGHOVDUJXPHQWVGHOV
QRPEUHVGRQDWV

 
F  (OPzGXOGHOUHVXOWDWpVODTXDUWDSRWqQFLDGHOPzGXO,O¶DUJXPHQWpVHOTXjGUXSOHGHO¶DUJXPHQW
GHOQRPEUHGRQDW


 (IHFWXDOHVSRWqQFLHVVHJHQWVPLWMDQoDQWODIyUPXODGH'H0RLYUH
D  (3(cos25qisin25q))4
E (2(cos40qisin40q))9
F (5(cos115qisin115q))7

§ S S·
G  ¨ cos  i sen ¸ 
©  ¹

§ § S S · ·
H  ¨  ¨ cos  i sen ¸ 
© ©   ¸¹ ¹


D (3(cos25qisin25q))4 81(cos100qisin100q)
E (2(cos40qisin40q))9 512(cos0qisin0q) 512
F  (5(cos115qisin115q))7 78125(cos85qisin85q)

§ S S·
G  ¨ cos  i sen ¸ cos S  i sen S  
©  ¹

§ § S S ··
H  ¨  ¨ cos  i sen ¸¸  cos   i sen   
© ©   ¹¹

 

335
317

Nombres complexos
0QODTGUEQORNGZQU 5

 'LEXL[DHOVQRPEUHV૛૜૙ι L૟૚૞૙ι 'HWHUPLQDSHUTXLQQRPEUHFRPSOH[KHPGHPXOWLSOLFDUHO


SULPHUSHUREWHQLUHOVHJRQ

     
6¶KDGHPXOWLSOLFDUSHU͵ଵଶ଴ι 

 'LEXL[DHOVQRPEUHV૚૛૜૙૙ι L૝૚૛૙ι ,QGLFDSHUTXLQQRPEUHFRPSOH[KHPGHGLYLGLUHOSULPHUSHU
REWHQLUHOVHJRQ

     
6¶KDGHGLYLGLUSHU͵ଵ଼଴ι 

 &DOFXODOHVVROXFLRQVGHOHVDUUHOVVHJHQWV

D  ૜ඥ૟૝૚૛૙ι   F  ට૜૛૞࣊

 H  ඥૢ૛૛૙ι 

E  ૞ඥ૚૚૞૙ι   G  ඥ૟૝૚ૡ૙ι 

I  ඥ૛ૠ૞ι 

D   ž  

(OPzGXOGHOHVVROXFLRQVVHUjO¶DUUHOF~ELFDGHOPzGXO

([LVWLUDQWDQWVDUJXPHQWVFRPLQGLTXLHOUDGLFDO

ž   ˜ ž
E ž
6LN o  
ž   ˜ ž
E ž
6LN o  
ž   ˜ ž
E ž
6LN o  
3HUWDQWOHVDUUHOVVyQqqq

 

336
318

0QODTGUEQORNGZQU 65

E (OPzGXOGHOHVVROXFLRQVVHUjO¶DUUHOFLQTXHQDGH

([LVWLUDQWDQWVQRPEUHVFRPLQGLTXLHOUDGLFDO

R   ˜ R
E R
6LN o  

R  ˜ R
E R
6LN o  

R   ˜ R
E R
6LN o  

R   ˜ R
E R
6LN o  

R   ˜ R
E R
6LN o  
3HUWDQWOHVDUUHOVVyQqqqqq

F (OPzGXOGHOHVVROXFLRQVVHUjO¶DUUHOFLQTXHQDGH

([LVWLUDQWDQWVDUJXPHQWVFRPLQGLTXLHOUDGLFDO

R   ˜ R
E R
6LN o  

R   ˜ R
E  R
6LN o  

R   ˜ R
E R
6LN o  

R   ˜ R
E R
6LN o  

R   ˜ R
E R
6LN o  
3HUWDQWOHVDUUHOVVyQqqqqq

G (OPzGXOGHOHVVROXFLRQVVHUjO¶DUUHOVLVHQDGH

([LVWLUDQWDQWVQRPEUHVFRPLQGLTXLHOUDGLFDO

R   ˜ R
E R
6LN o  

R   ˜ R
E R
6LN o  

337
319

Nombres complexos
0QODTGUEQORNGZQU 5

R   ˜ R
E R
6LN o  

R   ˜ R
E R
6LN o  

R   ˜ R
E R
6LN o  

R   ˜ R
E R
6LN o  
3HUWDQWOHVDUUHOVVyQqqqqqq

H (OPzGXOGHOHVVROXFLRQVVHUjO¶DUUHOF~ELFDGH  

([LVWLUDQWDQWVDUJXPHQWVFRPLQGLTXLHOUDGLFDO

R   ˜ R
E R
6LN o  

R  ˜ R
E R
6LN o  

R   ˜ R
E R
6LN o  

R   ˜ R
E R
6LN o  

3HUWDQWOHVDUUHOVVyQ
 ž  ž   ž   ž 

I  (OPzGXOGHOHVVROXFLRQVVHUjO¶DUUHOVLVHQDGH

([LVWLUDQWDQWVQRPEUHVFRPLQGLTXLHOUDGLFDO

R   ˜ R
E R
6LN o  

R  ˜ R
E R
6LN o  

R   ˜ R
E R
6LN o  

R   ˜ R
E R
6LN o  

338
320

0QODTGUEQORNGZQU 65

R   ˜ R
E R
6LN o  

R   ˜ R
E R
6LN o  

3HUWDQWOHVDUUHOVVyQ                  


 R
 R
 R
 R
 R
 R

 &DOFXODOHVDUUHOVVHJHQWVGHQRPEUHVFRPSOH[RV
૝ ૜
D  ξ૚ F  ξ૚ H  ξ࢏
૜ ૝
E  ξ૚ G  ξ࢏ I  ξ࢏

D  ξͳ ൌ  ͳబιశೖ൉యలబι 

6L݇ ൌ Ͳ ՜ ‫ݔ‬ଵ ൌ ͳଵ଼଴ι ൌ െͳ  6L݇ ൌ ͳ ՜ ‫ݔ‬ଶ ൌ ͳ଴ι ൌ ͳ


E  ξͳ ൌ  ͳబιశౡ൉యలబι 

6L݇ ൌ Ͳ ՜ ‫ݔ‬ଵ ൌ ͳଵଶ଴ι  6L݇ ൌ ʹ ՜ ‫ݔ‬ଷ ൌ ͳ଴ι ൌ ͳ

6L݇ ൌ ͳ ՜ ‫ݔ‬ଶ ൌ ͳଶସ଴ι 


F  ξͳ ൌ  ͳబιశೖ൉͵͸Ͳι 

6L݇ ൌ Ͳ ՜ ‫ݔ‬ଵ ൌ ͳଽ଴ι ൌ ݅  6L݇ ൌ ʹ ՜ ‫ݔ‬ଷ ൌ ͳଶ଻଴ι ൌ െ݅

6L݇ ൌ ͳ ՜ ‫ݔ‬ଶ ൌ ͳଵ଼଴ι ൌ െͳ 6L݇ ൌ ͵ ՜ ‫ݔ‬ସ ൌ ͳ଴ι ൌ ͳ

G  ξ݅ ൌ  ͳవబιశೖ൉యలబι 

6L݇ ൌ Ͳ ՜ ‫ݔ‬ଵ ൌ ͳସହι

6L݇ ൌ ͳ ՜ ‫ݔ‬ଶ ൌ ͳଶଶହι  



H  ξ݅ ൌ  ͳవబιశೖ൉యలబι 

6L݇ ൌ Ͳ ՜ ‫ݔ‬ଵ ൌ ͳଷ଴ι 6L݇ ൌ ʹ ՜ ‫ݔ‬ଷ ൌ ͳଶ଻଴ι ൌ െ݅

6L݇ ൌ ͳ ՜ ‫ݔ‬ଶ ൌ ͳଵହ଴ι  


I  ξ݅ ൌ  ͳవబιశೖ൉యలబι 

6L݇ ൌ Ͳ ՜ ‫ݔ‬ଵ ൌ ͳଶଶǡହι   6L݇ ൌ ʹ ՜ ‫ݔ‬ଷ ൌ ͳଶ଴ଶǡହι

6L݇ ൌ ͳ ՜ ‫ݔ‬ଶ ൌ ͳଵଵଶǡହι  6L݇ ൌ ͵ ՜ ‫ݔ‬ସ ൌ ͳଶଽଶǡହι 

 &DOFXODDTXHVWHVDUUHOVLUHSUHVHQWDOHV
૟ ૜ ૞
D  ξെ૚૟ F  ξ૚૟࢏ H  ඥ૚ െ ξ૜࢏

339
321

Nombres complexos
0QODTGUEQORNGZQU 5

૜ ૜ ξ૛ ξ૛ ૝
E  ξെ࢏ G  ට ൅ ࢏ I  ξ૟૛૞
૛ ૛

a)    ž 

(OPzGXOGHOHVVROXFLRQVVHUjO¶DUUHOVLVHQDGH

 

R   ˜ R
6Lk 0o E R 


R   ˜ R
6Lk 1o E  R 


R   ˜ R
6Lk 2o E R 


R   ˜ R
6Lk 3o E R 


R   ˜ R
6Lk 4o E R 


R   ˜ R
6Lk 5o E R 


3HUWDQWOHVDUUHOVVyQ   ž    ž 


i  ž    ž    ž   i   ž  

E   i  R 

(OPzGXOGHOHVVROXFLRQVVHUjO¶DUUHOF~ELFDGH

([LVWLUDQWDQWVDUJXPHQWVFRPLQGLTXLHOUDGLFDO.

R   ˜ R
6Lk 0o E R 


R   ˜ R
6Lk 1o E R 
 

R   ˜ R
6Lk 2o E R 


3HUWDQWOHVDUUHOVVyQ190q,1210q,1330q

F   i  ž 

340
322

0QODTGUEQORNGZQU 65

(OPzGXOGHOHVVROXFLRQVVHUjO¶DUUHOF~ELFDGH:   .


([LVWLUDQWDQWVDUJXPHQWVFRPLQGLTXLHOUDGLFDO

R   ˜ R
6Lk 0o E R 


R  ˜ R
6Lk 1o E R 


R   ˜ R
6Lk 2o E R 


3HUWDQWOHVDUUHOVVyQ    ž    ž     ž   i .

 
G    i  R 
 
(OPzGXOGHOHVVROXFLRQVVHUjO¶DUUHOF~ELFDGH
([LVWLUDQWDQWVDUJXPHQWVFRPLQGLTXLHOUDGLFDO

R   ˜ R
6Lk 0o E R 

R   ˜ R
6Lk 1o E  R 

R   ˜ R
6Lk 2o E R 
 
3HUWDQWOHVDUUHOVVyQ115q,1135q,1255q


H    i  R 
(OPzGXOGHOHVVROXFLRQVVHUjO¶DUUHOFLQTXHQDGH
([LVWLUDQWDQWVDUJXPHQWVFRPLQGLTXLHOUDGLFDO

R   ˜ R
6Lk 0o E R 

R  ˜ R
6Lk 1o E R 

R   ˜ R
6Lk 2o E R 

R   ˜ R
6Lk 3o E R  


341
323

Nombres complexos
0QODTGUEQORNGZQU 5

R   ˜ R
6Lk 4o E R 

3HUWDQWOHVDUUHOVVyQ   R     R     R     R     R .

I     R 
(OPzGXOGHOHVVROXFLRQVVHUjO¶DUUHOF~ELFDGH
([LVWLUDQWDQWVDUJXPHQWVFRPLQGLTXLHOUDGLFDO

R   ˜ R
6Lk 0o E R 

R  ˜ R
6Lk 1o E R 

R   ˜ R
6Lk 2o E R 

R   ˜ R
6Lk 3o E R 
 
3HUWDQWOHVDUUHOVVyQ50q,590q,5180q,5270q.

 8QDGHOHVDUUHOVF~ELTXHVG¶XQQRPEUHFRPSOH[pVL&DOFXOD¶QOHVDOWUHVGXHV


/HVDOWUHVGXHVDUUHOVWLQGUDQHOPDWHL[PzGXO

z   i  q 

z  q ˜R  q 

z  q ˜R  q 



 (OVYqUWH[VGHOSROtJRQVHJHQWVyQOHVDUUHOVTXDUWHVG¶XQQRPEUHFRPSOH[

342
324

0QODTGUEQORNGZQU 65

'HWHUPLQDHOQRPEUHLOHVVHYHVDUUHOV
/HVDUUHOVVyQ
z1 44i   ž  z2 44i   ž  z3 44i   ž  z4 44i   ž 

(OQRPEUHpV:z 1024180q 1024.



 (OQRPEUHFRPSOH[૛૜૙ι pVXQGHOVYqUWH[VG¶XQSHQWjJRQUHJXODU&DOFXOD¶QHOVDOWUHVTXDWUH
YqUWH[VLHOQRPEUHFRPSOH[TXHWpDTXHVWVQRPEUHVFRPDDUUHOVFLQTXHQHV

/HVDUUHOVVyQ:z1 230q,z2 2102q,z3 2174q,z4 2246q,z5 2318q.


(OQRPEUHpV:z 32150q.

‫ܖ‬
 7UREDQL]GHPDQHUDTXHGXHVGHOHVVROXFLRQVGH ξ‫ܢ‬QVLJXLQ૟૜૙ι L૟૚૛૙ι +LKDXQD~QLFD
VROXFLy"4XLQpVHOQRPEUHQPpVSHWLWTXHSRWVWUREDU"

6LJXLz rD
/¶DUUHOHQqVLPDGHrKDGHVHU
/¶DUJXPHQWKDGHVHUP~OWLSOHGHLGH
/DVROXFLyQRpV~QLFD
(OQRPEUHPpVSHWLWTXHFRPSOHL[OHVFRQGLFLRQVpVn 
z q
8QDDOWUDVROXFLypVn 
z q

 &DOFXODWRWHVOHVVROXFLRQVFRPSOH[RVGHOHVHTXDFLRQVVHJHQWV
D [   F  [ 
E [  G  [ 

D  x 
  R 
(OPzGXOGHOHVVROXFLRQVVHUjO¶DUUHOFLQTXHQDGH

([LVWLUDQWDQWVDUJXPHQWVFRPLQGLTXLHOUDGLFDO

R   ˜ R
6Lk 0o E R 

R  ˜ R
6Lk 1o ȕ  R 

R   ˜ R
6Lk 2o ȕ  R 

R   ˜ R
6Lk 3o ȕ  R 


343
325

Nombres complexos
0QODTGUEQORNGZQU 5

R   ˜ R
6Lk 4o ȕ  R 

3HUWDQWOHVDUUHOVVyQ136q,1108q,1180q,1252q,1324q.

E  x 
  R 
(OPzGXOGHOHVVROXFLRQVVHUjO¶DUUHOF~ELFDGH

([LVWLUDQWDQWVDUJXPHQWVFRPLQGLTXLHOUDGLFDO

R   ˜ R
6Lk 0o ȕ R 

R   ˜ R
6Lk 1o ȕ  R 

R   ˜ R
6Lk 2o ȕ R 

R   ˜ R
6Lk 3o ȕ  R 

3HUWDQWOHVDUUHOVVyQ50q,590q,5180q,5270q.

F  x 
  R 
(OPzGXOGHOHVVROXFLRQVVHUjO¶DUUHOFLQTXHQDGH

([LVWLUDQWDQWVDUJXPHQWVFRPLQGLTXLHOUDGLFDO

R   ˜ R
6Lk 0o ȕ R 

R   ˜ R
6Lk 1o ȕ  R 

R   ˜ R
6Lk 2o ȕ R 

R   ˜ R
6Lk 3o ȕ  R 

R   ˜ R
6Lk 4o ȕ  R 

3HUWDQWOHVDUUHOVVyQ10q,172q,1144q,1216q,1288q.

G  x 
  R 
(OPzGXOGHOHVVROXFLRQVVHUjO¶DUUHOF~ELFDGH

([LVWLUDQWDQWVDUJXPHQWVFRPLQGLTXLHOUDGLFDO

344
326

0QODTGUEQORNGZQU 65

R   ˜ R
6Lk 0o ȕ R 

R  ˜ R
6Lk 1o ȕ  R 

R   ˜ R
6Lk 2o ȕ R 

R   ˜ R
6Lk 3o ȕ  R 

3HUWDQWOHVDUUHOVVyQ245q,2135q,2225q,2315q.

 5HVROOHVHTXDFLRQVVHJHQWV
D [±[  F [±[± 
E [±[  G [[ 

 r     r 
D  x  r i 
 
E  x x   
o x 
 R 
(OPzGXOGHOHVVROXFLRQVVHUjO¶DUUHOF~ELFDGH
([LVWLUDQWDQWVDUJXPHQWVFRPLQGLTXLHOUDGLFDO

R   ˜ R
6Lk 0o ȕ R 

R  ˜ R
6Lk 1o ȕ  R 

R   ˜ R
6Lk 2o ȕ  R 

3HUWDQWOHVDUUHOVVyQ0,2,2120q,2140q.
F  t x2

 r   
t r  


x   

x    

x   i    
x    i 
  

G  x(x327) 0
345
327

Nombres complexos
0QODTGUEQORNGZQU 5

x 
  R 
(OPzGXOGHOHVVROXFLRQVVHUjO¶DUUHOF~ELFDGH
([LVWLUDQWDQWVDUJXPHQWVFRPLQGLTXLHOUDGLFDO

R   ˜ R
6Lk 0o ȕ R 

R  ˜ R
6Lk 1o ȕ  R 

R   ˜ R
6Lk 2o ȕ R 

3HUWDQWOHVDUUHOVVyQ0,360q,3180q,3300q.

 )HVDTXHVWHVRSHUDFLRQVDPEQRPEUHVFRPSOH[RV

   i 
D    E   F  
  i
   i

i  i

 
D    i   R 
i
(OPzGXOGHOHVVROXFLRQVVHUjO¶DUUHOF~ELFDGH
([LVWLUDQWDQWVDUJXPHQWVFRPLQGLTXLHOUDGLFDO

R   ˜ R
6Lk 0o ȕ R 

R   ˜ R
6Lk 1o ȕ  R 

R   ˜ R
6Lk 2o ȕ R 

R   ˜ R
6Lk 3o ȕ  R 

3HUWDQWOHVDUUHOVVyQ267,5q,2157,5q,2247,5q,2337,5q.

  i   i   i
E  ˜ i R 
 i  i  i
(OPzGXOGHOHVVROXFLRQVVHUjO¶DUUHOTXDGUDGDGH
([LVWLUDQWDQWVDUJXPHQWVFRPLQGLTXLHOUDGLFDO

R   ˜ R
6Lk 0o ȕ R 


346
328

0QODTGUEQORNGZQU 65

R   ˜ R
6Lk 1o ȕ  R 

3HUWDQWOHVDUUHOVVyQ145q,1225q.

F     i
    i
  i   R 

(OPzGXOGHOHVVROXFLRQVVHUjO¶DUUHOF~ELFDGH
([LVWLUDQWDQWVDUJXPHQWVFRPLQGLTXLHOUDGLFDO

R   ˜ R
6Lk 0o ȕ R 

R  ˜ R
6Lk 1o ȕ  R 

R   ˜ R
6Lk 2o ȕ R 

3HUWDQWOHVDUUHOVVyQ190q,1210q,1330q.

 &DOFXODLH[SUHVVDHOUHVXOWDWHQIRUPDELQzPLD


ž ¸ 

       
  i


ž ¸ 
ž ¸R
    R 
  i R

(OPzGXOGHOHVVROXFLRQVVHUjO¶DUUHOF~ELFDGH
([LVWLUDQWDQWVDUJXPHQWVFRPLQGLTXLHOUDGLFDO

R   ˜ R
6Lk 0o ȕ R 

R  ˜ R
6Lk 1o ȕ  R 

R   ˜ R
6Lk 2o ȕ R 

R   ˜ R
6Lk 3o ȕ  R 

3HUWDQWOHVDUUHOVVyQ   R     R     R     R .

 

347
329

Nombres complexos
0QODTGUEQORNGZQU 5

 (VFULXXQDHTXDFLyGHVHJRQJUDXDPEFRHILFLHQWVUHDOVTXHWLQJXLGHVROXFLRQVDTXHVWV
QRPEUHVFRPSOH[RV
D ±i E ±i
D &RPTXH±ipVVROXFLyOODYRUViWDPEpKRpV
 (x(1i))(x(1i)) x22x2
E &RPTXH±LpVVROXFLyOODYRUVLWDPEpKRpV
 (x(32i))(x(32i)) x26x13

 &DOFXODHOYDORUGHPSHUTXqDTXHVWSROLQRPL
      [±[±P
WLQJXLO¶DUUHO±i7UREDO¶DUUHOTXHIDOWD

(3i)26(3i)m 0o86i186i mom 10


/¶DUUHOTXHIDOWDpV3i.

 &DOFXODHOYDORUGHOVQRPEUHVFRPSOH[RVSLTSHUTXqHOSROLQRPLVHJHQW
      [S[T[±
WLQJXLOHVDUUHOViLi7UREDO¶DUUHOTXHIDOWD

(xi)(x2i) x23xi2
(x23xi2)(xa) x3(a3i)x2(23ia)x2aoa 1
p 13i
q 23i
/¶DUUHOTXHIDOWDpVx a 1.

 5HVROOHVHTXDFLRQVVHJHQWV
D [i[   F i[[±L 
E [±[i   G i[ 

i r    i r i      
D  x o x i x i
   
 r     r i 
E  x o x   i  x  i  
 
 r     r 
F  x o x i x i 
i i
 
G  x i o x  R 
i
(OPzGXOGHOHVVROXFLRQVVHUjO¶DUUHOF~ELFDGH
([LVWLUDQWDQWVDUJXPHQWVFRPLQGLTXLHOUDGLFDO

348
330

0QODTGUEQORNGZQU 65

R   ˜ R
6Lk 0o ȕ R 

R  ˜ R
6Lk 1o ȕ  R 

R   ˜ R
6Lk 2o ȕ R 

3HUWDQWOHVDUUHOVVyQ R  R  R .


 &DOFXODHOYDORUGHESHUTXqO¶RSHUDFLyVHJHQWVLJXLFHUWD

     ૝ૠ૛ι ൉ ሺ૚ ൅ ࢈࢏ሻ ൌ ૡ૚૜૛ι 

R
  bi R o tg R  b
R

 4XLQQRPEUHFRPSOH[KHPGHVXPDUD±LSHUTXqUHVXOWL૞૛ૠ૙ι ",SHUTXqGRQL૟૞ૈ "


ሺെ͵ ൅ ʹ݅ሻ ൅ ሺܽ ൅ ܾ݅ሻ ൌ െͷ݅ ՜ ܽ ൌ ͵ǡ ܾ ൌ െ͹


ሺെ͵ ൅ ʹ݅ሻ ൅ ሺܽ ൅ ܾ݅ሻ ൌ ͵ െ ͵ξ͵݅ ՜ ܽ ൌ ͸ǡ ܾ ൌ െʹ െ ͵ξ͵

 &RPSURYDTXHOHVDUUHOVVHJHQWVVXPHQ
D /HVDUUHOVTXDUWHVGH±i
E /HVDUUHOVVLVHQHVGH

D /HVVROXFLRQVGH  R VyQ   R     R     R     R .



 cos R  i sin R    cos R  i sin R  

   cos R  i sin R    cos  R  i sin R 

§        ·

 ¨¨ i  i  i  i ¸ 
©         ¸¹

E /HVVROXFLRQVGH   VyQ20q,260q,2120q,2180q,2240q,2300q.
2(cos0qisin0q)2(cos60qisin60q)2(cos120qisin120q)
2(cos180qisin180q)2(cos240qisin240q)2(cos300qisin300q) 

§        ·
 ¨¨   i  i    i  i ¸ 
©         ¸¹
 

349
331

Nombres complexos
0QODTGUEQORNGZQU 5

૛ାܑ
 'HWHUPLQDHOYDORUTXHKDGHWHQLUNSHUTXq XQQRPEUHUHDO'HTXLQQRPEUHUHDOHVWUDFWD"
‫ܑିܓ‬

i k i k      k i
˜ ok  
k i k i k  
  
(OQRPEUHpV  .
 
‫ିܓ‬ξ૛ܑ
 (VEULQDHOYDORUTXHKDGHWHQLUNSHUTXq VLJXLXQQRPEUHLPDJLQDULSXU4XLQpVDTXHVW
ξ૛ିܑ

QRPEUHLPDJLQDULSXU"

k  i   i k    k   i
˜ ok  
 i  i  
i
(OQRPEUHpV i .


૟ି૛ܑ
 &DOFXODHOYDORUGHDSHUTXqHOQRPEUH VLJXL
૚ା‫ܑ܉‬
D 8QQRPEUHLPDJLQDULSXU
E 8QQRPEUHUHDO
F (OQRPEUHFRPSOH[±i
G 8QQRPEUHFRPSOH[DPEPzGXO
ૠૈ
H 8QQRPEUHFRPSOH[DPEDUJXPHQW 

  i   ai   a  a   i
 ˜ 
  ai   ai   a
D 62a 0oa 3
E 62a 0oa 3

  i   i   i   i
F  ˜  i o a 
  i   i   i 

G   o    a  o a r  

 a
 arctg §¨   ·¸
© ¹
H  rS 

 a arctg a 

§ · S § · 
arctg ¨  ¸  arctg a o arctg a arctg ¨  ¸  R o a 
© ¹  © ¹ 
 

350
332

0QODTGUEQORNGZQU 65

 &DOFXODHOYDORUGHDSHUTXqHOQRPEUHFRPSOH[VHJHQWYHULILTXLTXHHOVHXTXDGUDWVLJXLLJXDO
DOVHXFRQMXJDW

ξ૜
ࢇ൅ ࢏


ξ૜ ૜
&DOFXOHPHOTXDGUDWቀࢇ ൅ ࢏ቁ ൌ ࢇ૛ െ ൅ ξ૜ࢇ࢏
૛ ૝

ξ૜
(OFRQMXJDWܽ െ ࢏

͵
‫ܽۓ‬ଶ ൅ ൌܽ ͳ
Ͷ
 ՜ ܽ ൌ  െ 
‫۔‬ ξ͵ ʹ
‫ە‬ξ͵ ൌ െ ʹ

‫ܑܠ‬ ૛‫ܠ‬
 5HVRODTXHVWDHTXDFLy െ ൌ ૚
૚ା૜ܑ ૝ିܑ

xi(4i)2x(13i) (13i)(4i)ox(4i126i) 711io x   i   i   i 


˜
  i   i 

 7UREDHOQRPEUHFRPSOH[TXHYHULILFDTXHHOVHXFXEpVXQQRPEUHUHDOLTXHODSDUWUHDOGHO
PDWHL[QRPEUHpVXQDXQLWDWVXSHULRUDODSDUWLPDJLQjULD

a b1
(b1bi)3 2b36b2i2b3i3b3bi1
2b36b23b 0ob(2b26b3) 0

§       · §       ·
(OVQRPEUHVVyQ    ¨
¨   ¸  ¨
 ¸¹ ¨© 

 ¸¹
¸ 
©

 'HWHUPLQDHOVQRPEUHVFRPSOH[RVHOFXEGHOVTXDOVpVLJXDODOVHXFRQMXJDW

(rD)3 rDor 1

3D DoD 0q

3D 360qDoD 90q

3D 720qDoD 180q

3D 1080qDoD 270q

(OVQRPEUHVVyQ1,i,1,i.

 

351
333

Nombres complexos
0QODTGUEQORNGZQU 5

૚૛ା‫ܑ܋‬
 &DOFXODFVDEHQWTXHODUHSUHVHQWDFLyJUjILFDGH pVVREUHODELVHFWULXGHOSULPHU
ି૞ା૛ܑ
TXDGUDQW
3HUTXqHVWLJXLVREUHODELVHFWULXGHOSULPHUTXDGUDQWODSDUWLPDJLQjULDKDGHVHULJXDODODSDUWUHDO

ͳʹ ൅ ܿ݅ ሺͳʹ ൅ ܿ݅ሻሺെͷ െ ʹ݅ሻ െ͸Ͳ ൅ ʹܿ ൅ ሺെʹͶ െ ͷܿሻ݅


ൌ ൌ 
െͷ ൅ ʹ݅ ሺെͷ ൅ ʹ݅ሻሺെͷ െ ʹ݅ሻ ʹͻ
͵͸
െ͸Ͳ ൅ ʹܿ ൌ െʹͶ െ ͷܿ ՜ ܿ ൌ 
͹

 7UREDGRVQRPEUHVFRPSOH[RVFRQMXJDWVWDOTXHODVHYDGLIHUqQFLDVLJXLLHOVHXTXRFLHQW
VLJXLODXQLWDWLPDJLQjULD

(abi)(abi) 2bi 6iob 3

a  bi a  bi a   b   abi a     ai
˜ i
a  bi a  bi a   b   a
a
a    \ o a 

(OVQRPEUHVVyQ33ii33i.

/¶HTXDFLy]D]E]±L WpG¶DUUHOVL&DOFXODHOYDORUGHDELODUHVWDG¶DUUHOV

  a  b  i  ½
¾oa   i b   i 
  a  b  i  ¿

z     i z     i z  i z   z   z  a z   az    z    za   z  a o a i 
/¶DOWUDDUUHOpVz i.

7UREDGRVQRPEUHVFRPSOH[RV]LZWDOTXHODVHYDVXPDVLJXLLLTXHLVLJXLXQDDUUHO
TXDGUDGDGHOVHXTXRFLHQW

z abi  w cdi

(abi)(cdi) ioa c,b 1d

c    d i 
i o c    d i  c  di o c  d  
c  di 
 
z i  w  i
 
 

352
334

0QODTGUEQORNGZQU 65


'HWHUPLQDXQQRPEUHFRPSOH[TXHVLKLVXPHP GRQLFRPDUHVXOWDWXQQRPEUHFRPSOH[GH

PzGXOFLDUJXPHQWƒ

 
a  bi  a   bi 
 

§ · 
c ¨a  ¸ b 
© ¹

b § ·
tg R  ob ¨a  ¸ 
 © ¹
a

 
§ · § · § ·
c ¨ a  ¸  ¨ a  ¸ ¨ a  ¸ 
© ¹ © ¹ © ¹
c  c
a  b  
 
c  c
(OQRPEUHFRPSOH[pV   i .
 

૚ାܑ
%XVFDXQQRPEUHFRPSOH[TXHVLKLVXPHP GRQLXQQRPEUHFRPSOH[GHPzGXOξ૛L
૛ି૛ܑ
DUJXPHQWƒ

 i  i i
˜ 
  i   i 
 § ·
a  bi  i a  ¨b  ¸i 
 © ¹
 
§ · § ·
 a  ¨ b  ¸ o a  ¨ b  ¸ 
© ¹ © ¹

b
R  oa 
tg   b 
a 

a  b 


(OQRPEUHFRPSOH[pV   i .

 

353
335

Nombres complexos
0QODTGUEQORNGZQU 5

&DOFXODO¶jUHDG¶XQTXDGULOjWHUTXHWpGHYqUWH[VOHVVROXFLRQVGHO¶HTXDFLy[ 

x 
  R 

(OPzGXOGHOHVVROXFLRQVVHUjO¶DUUHOF~ELFDGH  
([LVWLUDQWDQWVDUJXPHQWVFRPLQGLTXLHOUDGLFDO

R   ˜ R
6Lk 0o ȕ R 

R  ˜ R
6Lk 1o ȕ  R 

R   ˜ R
6Lk 2o ȕ R 

R   ˜ R
6Lk 3o ȕ  R 

3HUWDQWOHVDUUHOVVyQ1i,1i,1i,1i.
eVXQTXDGUDWGHFRVWDW2:A l2 22 4.

8QSHQWjJRQUHJXODUDPEFHQWUHHQO¶RULJHQGHFRRUGHQDGHVWpXQGHOVYqUWH[VHQ  
7UREDHOVDOWUHVYqUWH[VXVDQWQRPEUHVFRPSOH[RV
‫ݖ‬ଶ ൌ െ͵ െ ʹ݅
)HPSRWqQFLDFLQTXHQD‫ ݖ‬ൌ ξͳ͵ହ ଶଵଷιସଵᇲଶସǡଶᇲᇲ 

&DOFXOHPODUHVWDGHOHVDUUHOVξͳ͵మభయιరభᇲ మరǡమᇲᇲ 

6LN ՜  ‫ݔ‬ଵ ൌ ξͳ͵ସଶιସସᇲ ଵ଺ǡ଼ହᇲᇲ    6LN ՜  ‫ݔ‬ଷ ൌ ξͳ͵ଵ଼଴ιସସᇲଵ଺ǡ଼ହᇲᇲ  

6LN ՜  ‫ݔ‬ଶ ൌ ξͳ͵ଵଵସιସସᇲଵ଺ǡ଼ହᇲᇲ   6LN ՜  ‫ݔ‬ସ ൌ ξͳ͵ଶହ଼ιସସᇲ ଵ଺ǡ଼ହᇲᇲ  

'HWHUPLQDTXLQQRPEUHFRPSOH[IRUPDXQWULDQJOHHTXLOjWHUDPEHOVHXFRQMXJDWLDPE"

6LJXL/ODORQJLWXGGHOFRVWDWGHOVWULDQJOHHTXLOjWHUXQGHOVYqUWH[VpVHOFRPSOH[DEiLO¶DOWUHYqUWH[
pVHOVHXFRQMXJDWD±Ei.

௅ ௅൉ξଷ
„ ൌ  ൉ •‹ ͵Ͳι ൌ  ܽ ൌ െͷ ൅ ‫ ܮ‬൉ …‘• ͵Ͳι ൌ െͷ ൅ 
ଶ ଶ

7RWVHOVWULDQJOHVWHQHQFRPDYqUWH[VLWXDWDO¶HVTXHUUD

6LHOYqUWH[HVWLJXpVVLWXDWDODGUHWDGHOWULDQJOHOHVFRRUGHQDGHVGHOVDOWUHVYqUWH[VVHUDQ

௅ ௅൉ξଷ
„ ൌ  ൉ •‹ ͵Ͳι ൌ  ܽ ൌ െͷ െ ‫ ܮ‬൉ …‘• ͵Ͳι ൌ െͷ െ 
ଶ ଶ

 

354
336

0QODTGUEQORNGZQU 65

&DOFXODO¶jUHDGHO¶KH[jJRQUHJXODUTXHGHWHUPLQHQHOVDIL[RVGHOHVDUUHOVVLVHQHVGH


  R 
(OPzGXOGHOHVVROXFLRQVVHUjO¶DUUHOVLVHQDGH
([LVWLUDQWDQWVDUJXPHQWVFRPLQGLTXLHOUDGLFDO

R   ˜ R
6Lk 0o ȕ R 

R  ˜ R
6Lk 1o ȕ  R 

R   ˜ R
6Lk 2o ȕ  R 

R   ˜ R
6Lk 3o ȕ  R 
 
R R
   ˜ 
6Lk 4o ȕ  R 

R   ˜ R
6Lk 5o ȕ  R 

3HUWDQWOHVDUUHOVVyQ230q,290q,2150q,2210q,2270q,2330q.

p ˜ ap  ˜ 
/¶jUHDGHO¶KH[jJRQpV A   .
 

/HVTXDWUHDUUHOVF~ELTXHVGH±GHVFULXHQXQTXDGUDWFDOFXOD¶QO¶jUHD$PpVOHVDUUHOV
F~ELTXHVGHVFULXHQXQWULDQJOHHTXLOjWHUGHWHUPLQD¶QO¶jUHD
/HVDUUHOVTXDUWHVGHVyQ


z1 845q       z2 8135q      
z3 8225q       
z4 8315q       

  
 
&DOFXOHPHOFRVWDW        .

3HUWDQWO¶jUHDpVGH

/HVDUUHOVF~ELTXHVGHVyQ


z1 1660q      z2 16180q (16,0)
z3 16300q      

6HIRUPDXQWULDQJOHDPEEDVHGHLODVHYDDOWXUDpVGH

3HUWDQWODVHYDjUHDpV 3 

355
337

Nombres complexos
0QODTGUEQORNGZQU 5

'RVYqUWH[VFRQVHFXWLXVG¶XQTXDGUDWVyQHOVDIL[RVGHOVQRPEUHViLi'HWHUPLQDHOV
DOWUHVYqUWH[VVLVDSVTXHHQWpXQDOTXDUWTXDGUDQW
(OYHFWRUGHOFRVWDWpV(65i)(3i) 34iiHOSHUSHQGLFXODUpV43i.

(QHOTXDUWTXDGUDQWHVWDUjHOYqUWH[(3i)(43i) 72i.

(OYqUWH[TXHIDOWDpV(65i)(43i) 102i.

(OQRPEUHFRPSOH[ipVXQDGHOHVDUUHOVF~ELTXHVGH]7UREDOHVDOWUHVGXHVDUUHOV
‫ݖ‬ଵ ൌ ͵ ൅ ͷ݅ ൌ ξ͵ͷହଽιଶᇲଵ଴ǡସ଼ᇲᇲ 

/HVDUUHOVWLQGUDQHOPDWHL[PzGXO

&DOFXOHPOHVDOWUHVDUUHOVξ͵ͷହଽιଶᇲଵ଴ǡସ଼ᇲᇲାೖ൉యలబι 

6Lk=0՜  ‫ݔ‬ଵ ൌ ξ͵ͷହଽιଶᇲ ଵ଴ǡସ଼ᇲᇲ   6Lk=2՜  ‫ݔ‬ଷ ൌ ξ͵ͷଶଽଽιଶᇲ ଵ଴ǡସ଼ᇲᇲ 

6Lk=1՜  ‫ݔ‬ଶ ൌ ξ͵ͷଵ଻ιଶᇲଵ଴ǡସ଼ᇲᇲ 



(VFULXXQDHTXDFLyGHVHJRQJUDXTXHWLQJXLFRPDVROXFLRQViL±i)HVHOPDWHL[DPE
±iL±i
ሺ‫ ݔ‬െ ͵ ൅ ݅ሻሺ‫ ݔ‬െ ͵ െ ݅ሻ ൌ Ͳ ՜ ‫ ݔ‬ଶ െ ݅‫ ݔ‬െ ͵‫ ݔ‬൅ ͻ ൅ ͵݅ ൅ ݅‫ ݔ‬െ ͵݅ ൅ ͳ ൌ Ͳ
    ՜ ‫ ݔ‬ଶ െ ͸‫ ݔ‬൅ ͳͲ ൌ Ͳ
ሺ‫ ݔ‬൅ ʹ ൅ ͷ݅ሻሺ‫ ݔ‬൅ ʹ െ ͷ݅ሻ ൌ Ͳ ՜ ‫ ݔ‬ଶ ൅ ʹ‫ ݔ‬െ ͷ݅‫ ݔ‬൅ ʹ‫ ݔ‬൅ Ͷ െ ͳͲ݅ ൅ ͷ݅‫ ݔ‬൅ ͳͲ݅ ൅ ʹͷ ൌ Ͳ
    ՜ ‫ ݔ‬ଶ ൅ Ͷ‫ ݔ‬൅ ʹͻ ൌ Ͳ

'HPRVWUDTXHVLXQDHTXDFLyGHVHJRQJUDXDPEQRPEUHVUHDOVFRPDFRHILFLHQWVWpGXHV
DUUHOVFRPSOH[HVDTXHVWHVDUUHOVKDQGHVHUQRPEUHVFRQMXJDWV

7HQLPO¶HTXDFLyܽ‫ ݔ‬ଶ ൅ ܾ‫ ݔ‬൅ ܿ ൌ Ͳ

5HVROHPO¶HTXDFLy

ି௕ାቀඥି௕మ ାସ൉௔൉௖ቁ௜
ି௕േඥ௕మ ିସ൉௔൉௖
‫ݔ‬ଵ ൌ
ଶ൉௔
‫ݔ‬ൌ ՜  ൞ 
ଶ൉௔ ି௕ିቀඥି௕మ ାସ൉௔൉௖ቁ௜
‫ݔ‬ଶ ൌ
ଶ൉௔

/HVVHYHVVROXFLRQVVyQGRVQRPEUHVFRPSOH[RVFRQMXJDWV



&DOFXODHOSURGXFWHGHOHVGXHVDUUHOVGHξ૚HOSURGXFWHGHOHVWUHVDUUHOVGH ξ૚$UDWUREDXQD
IyUPXODSHUDOSURGXFWHGHOHVQDUUHOVQqVLPHVGHODXQLWDW

/HVDUUHOVTXDGUDGHVGHVyQ10q,1180qiHOVHXSURGXFWHpV1180q 1.

356
338

0QODTGUEQORNGZQU 65

/HVDUUHOVF~ELTXHVGHVyQ10q,1120q,1240qiHOVHXSURGXFWHpV1360q 1.

(OPzGXOGHOSURGXFWHGHOHVnDUUHOVn-èVLPHVVHUj

/¶DUJXPHQWGHOSURGXFWHGHOHVnDUUHOVQèVLPHVVHUj

R ˜  R ˜ R ˜ n   R R ˜ n ˜ n  


    ˜       n   R n   
n n n n n

(OSURGXFWHGHOHVnDUUHOVnqVLPHVVHUj(1)n1.

'HWHUPLQDODUHODFLyTXHKLKDHQWUHHOPzGXOGHODVXPDGHGRVFRPSOH[RVLODVXPDGHOVVHXV
PzGXOV

(OPzGXOGHODVXPDGHGRVQRPEUHVFRPSOH[RVpVVHPSUHPpVSHWLWTXHODVXPDGHOVPzGXOVGHOV
QRPEUHV

eVFHUWTXHVHPSUHTXHPXOWLSOLTXHVXQQRPEUHUHDOSHUXQQRPEUHFRPSOH[]HOUHVXOWDWWpHO
PDWHL[DUJXPHQWTXH]"
6LQRpVFHUWHQXQFLDXQDSURSLHWDWFRUUHFWD

1RpVFHUWMDTXHͳଵ଼଴ι ൉ ͳଽ଴ι ൌ ͳଶ଻଴ι 

6ROVpVFHUWVLHOQRPEUHUHDOpVSRVLWLX

6LPXOWLSOLTXHPXQQRPEUHUHDOSRVLWLXSHUXQQRPEUHFRPSOH[]HOUHVXOWDWWpHOPDWHL[DUJXPHQWTXH
]


,QGLFDVLpVFHUWTXHO¶LQYHUVGHOSURGXFWHGHGRVQRPEUHVFRPSOH[RVpVHOSURGXFWHGHOVVHXV
LQYHUVRV
eVFHUWMDTXHGRQDWV‫ݖ‬ଵ L‫ݖ‬ଶ GRVQRPEUHVFRPSOH[RVTXDOVHYROpVFRPSOHL[TXH

  
zāz  ā zāz  
zāz z z

'HPRVWUDTXHSHUDTXDOVHYROSDUHOOGHQRPEUHVFRPSOH[RVHVFRPSOHL[TXH

തതതതതതതതതത
ࢠ ૚ ൅ ࢠ૛ ൌ തതത തതത
ࢠ૚ ൅ ࢠ ૛ തതതതതതതത
ࢠ૚ ൉ ࢠ૛ ൌ തതത
ࢠ૚ ൉ തതത
ࢠ૛
8WLOLW]DDTXHVWHVSURSLHWDWVSHUGHPRVWUDUTXHVL] DELpVXQDVROXFLyGHO¶HTXDFLyGHJUDX
QVHJHQW
ࢇ࢔ ൉ ࢞࢔ ൅ ࢇ࢔ି૚ ൉ ࢞࢔ି૚ ൅ ‫ ڮ‬൅ ࢇ૚ ൉ ࢞૚ ൅ ࢇ૙ ൌ ૙
DPEࢇ࢏ ‫ א‬Թ5DOHVKRUHVHOYDORU]WDPEppVVROXFLyGHODPDWHL[DHTXDFLy

357
339

Nombres complexos
0QODTGUEQORNGZQU 5

z  z a  bi  c  di a  c  b  d i a  c  b  d i a  bi  c  di z  z  

z ˜ z a  bi c  di ac  bd  bc  ad i ac  bd  bc  ad i a  bi c  di z ˜ z


6Lz abipVXQDVROXFLyGHODHTXDFLyGHJUDXn,OODYRUV z WDPEppVVROXFLyGHODPDWHL[DHTXDFLy

an a  bi    a a  bi  a  o an a  bi    a a  bi  a
n n
 

an a  bi    a a  bi  a an a  bi    a a  bi  a
n n


n
an a  bi    a a  bi  a an a  bi    a a  bi  a
n


)HVVHUYLUHOVQRPEUHVFRPSOH[RVSHUUHVROGUHOHVHTXDFLRQVVHJHQWV
D  ࢠ૛ െ ૛ࢠ െ ૛ ൅ ૝࢏ ൌ ૙
E  ࢠ૝ ൅ ሺ૝ െ ૛࢏ሻࢠ૛ െ ૛࢏ ൌ ૙
F  ࢠ૝ ൅ ૚૙ࢠ૛ ൅ ૚૟ૢ ൌ ૙

 r      i
D  z  r   i  r  R  r   i 


z1 1i,z2 3i

  i r   i
E  t   i r   i   i r  R   i r   i 


t1 2i,t2 4

z  R   i  z  R R   i  z  R i  z  R  R i 


   

 r   


F  t  r i 


t1 Ͳ512i,t2 Ͳ512i

z  R   i  z    i 

 z     i  z     i 

5HVROOHVHTXDFLRQVDPEQRPEUHVFRPSOH[RVVHJHQWV

D  ൅ ሺ૛ െ ࢏ሻ૟࢏ ൌ െ૜ ൅ ૛࢏
૞ି࢏

358
340

0QODTGUEQORNGZQU 65

ି૝૚ା૜ૠ࢏
E  ࢠሺെ૛ ൅ ૟࢏ሻ ൅ ൅ ૚૙ െ ૡ࢏ ൌ ࢠሺ૚ ൅ ૠ࢏ሻ
૝ି૜࢏
௭ ௭
D  ൅ ሺʹ െ ݅ሻ͸݅ ൌ െ͵ ൅ ʹ݅ ՜ ൅ ͸ ൅ ͳʹ݅ ൌ െ͵ ൅ ʹ݅
ହି௜ ହି௜
‫ݖ‬
՜ ൌ െͻ െ ͳͲ݅ ՜ ‫ ݖ‬ൌ ሺͷ െ ݅ሻሺെͻ െ ͳͲ݅ሻ ՜ ‫ ݖ‬ൌ െͷͷ െ Ͷͳ݅
ͷെ݅
ିସଵାଷ଻୧
„ሻ ሺെʹ ൅ ͸‹ሻ ൅ ൅ ͳͲ െ ͺ‹ ൌ œሺͳ ൅ ͹‹ሻ ՜ œሺെʹ ൅ ͸‹ሻ െ ͳͳ ൅ ‹ ൅ ͳͲ െ ͺ‹ ൌ œሺͳ ൅ ͹‹ሻ ՜
ସିଷ୧

   ՜ ‫ݖ‬ሺെʹ ൅ ͸݅ሻ െ ͳ െ ͹݅ ൌ ‫ݖ‬ሺͳ ൅ ͹݅ሻ

   ՜ ‫ݖ‬ሺെʹ ൅ ͸݅ሻ െ ‫ݖ‬ሺͳ ൅ ͹݅ሻ ൌ ͳ ൅ ͹݅ ՜ ‫ݖ‬ሺെʹ ൅ ͸݅ െ ͳ െ ͹݅ሻ ൌ ͳ ൅ ͹݅

ଵା଻௜
   ՜‫ݖ‬ൌ ՜ ‫ ݖ‬ൌ െͳ െ ʹ݅
ିଷି௜

5HVROOHVHTXDFLRQVVHJHQWV
D  ࢞૛ െ ૡ࢏࢞ ൅ ૝࢏ െ ૚ૢ ൌ ૙
࢞ െ ࢏࢟ ൌ ૙
E ൜ 
࢟ െ ࢏࢞ ൌ ૝ െ ૟࢏
଼௜േඥି଺ସିସ൉ଵ൉ሺସ௜ିଵଽሻ ଼௜േξଵ଴ିଵ଺௜
D  ‫ ݔ‬ଶ െ ͺ݅‫ ݔ‬൅ Ͷ݅ െ ͳͻ ൌ Ͳ ՜ ‫ ݔ‬ൌ ൌ 
ଶ൉ଵ ଶ
‫ ݔ‬െ ݅‫ ݕ‬ൌ Ͳ
E  ൜ ՜ ‫ ݔ‬ൌ ݅‫ ݕ‬՜ ‫ ݕ‬െ ݅ሺ݅‫ݕ‬ሻ ൌ Ͷ െ ͸݅ ՜ ‫ ݕ‬൅ ‫ ݕ‬ൌ Ͷ െ ͸݅ ՜ ‫ ݕ‬ൌ ʹ െ ͵݅
‫ ݕ‬െ ݅‫ ݔ‬ൌ Ͷ െ ͸݅
 ‫ ݔ‬ൌ ݅ሺʹ െ ͵݅ሻ ൌ ͵ ൅ ʹ݅

5HSUHVHQWDHOQRPEUHFRPSOH[૚ ൅ ૛ξ૜࢏LIHVHQDTXHVWSXQWXQJLUGHžFHQWUDWHQO¶RULJHQ
(VFULXOHVH[SUHVVLRQVELQzPLDLSRODUGHOQRPEUHFRPSOH[TXHHQUHVXOWD

      
œ ൌ ͳ ൅ ʹξ͵‹ ൌ ξͳ͵଻ଷιହଷᇲ ହଶǡଷଽᇲᇲ 
)HPXQJLUGHž
ξͳ͵ଵଷଷιହଷᇲହଶǡଷଽᇱᇱ ൌ െʹǡͷ ൅ ʹǡ͸݅

/DVXPDGHGRVQRPEUHVFRPSOH[RVFRQMXJDWVpVLODVXPDGHOVVHXVPzGXOVpV4XLQV
QRPEUHVVyQ"

6LJXL] DEi.

ܽ ൅ ܾ݅ ൅ ܽ െ ܾ݅ ൌ ͳ͸ ܽൌͺ
ቊ ଶ ՜൜ ଶ 
ඥܽ ൅ ܾ ൅ ඥܽ ൅ ሺെܾሻ ൌ ʹͲ
ଶ ଶ ଶ ඥܽ ൅ ܾ ଶ ൌ ͳͲ

     ՜͸Ͷ ൅ ܾ ଶ ൌ ͳͲͲ ՜ ܾ ൌ േ͸

359
341

Nombres complexos
0QODTGUEQORNGZQU 5

(OVQRPEUHVVyQiL±i.

૚ ξ૜
&RQVLGHUD‫ ܝ‬ൌ െ ൅ ܑ&RPSURYDTXHVL] iDOHVKRUHV]Xā]LXā]VyQOHVWUHV
૛ ૛
DUUHOVF~ELTXHVG¶XQQRPEUHFRPSOH['HPRVWUDTXHDL[zSDVVDSHUDTXDOVHYROQRPEUH]
4XqWpGHSDUWLFXODUHOQRPEUHX"
‫ ݖ‬ൌ െʹ ൅ ͷ݅ ൌ ξʹͻଵଵଵιସ଼ᇲ ହǡ଴଻ᇲᇲ 

ͳ ξ͵
‫ ݑ‬൉ ‫ ݖ‬ൌ ቆെ ൅ ݅ቇ ሺെʹ ൅ ͷ݅ሻ ൌ ͳଵଶ଴ι ൉ ξʹͻଵଵଵιସ଼ᇲ ହǡ଴଻ᇲᇲ ൌ ξʹͻଶଷଵιସ଼ᇲ ହǡ଴଻ᇲᇲ 
ʹ ʹ

ͳ ξ͵
‫ݑ‬ଶ ൉ ‫ ݖ‬ൌ ቆെ ൅ ݅ቇ ሺെʹ ൅ ͷ݅ሻ ൌ ͳଵଶ଴ι ൉ ξʹͻଵଵଵιସ଼ᇲହǡ଴଻ᇲᇲ ൌ ξʹͻଷହଵιସ଼ᇲ ହǡ଴଻ᇲᇲ 
ʹ ʹ
6yQOHVDUUHOVF~ELTXHVGHʹͻଷଷହιଶସᇲ ଵହǡଶᇲᇲ 
$L[zVXFFHHL[SHUDTXDOVHYROQRPEUHFRPSOH[MDTXHOHVDUUHOVF~ELTXHVG¶XQQRPEUHFRPSOH[WHQHQ
HOPDWHL[PzGXOLHOVHXDUJXPHQWHVGLIHUHQFLDHQž

4XDQPXOWLSOLTXHPTXDOVHYROQRPEUHSHUXHOVHXPzGXOQRYDULDLHOVHXDUJXPHQWDXJPHQWDž

6DEHPTXHO
DUJXPHQWGHOQRPEUHFRPSOH[]pVƒLTXHHOPzGXOGH]pV&DOFXOD]L]
VLHOVHXSURGXFWHpVi

‫ݖ‬ଵ ൌ ‫ݎ‬ଵହ଴ι 

‫ݖ‬ଶ ൌ ʹఈ 

െͺ݅ ൌ ͺଶ଻଴ι 

‫ݎ‬ଵହ଴ι ൉ ʹఈ ൌ ͺଶ଻଴ι 

‫ݎ‬൉ʹൌͺ ‫ݎ‬ൌͶ ‫ ݖ‬ൌ Ͷଵହ଴ι


൜ ՜൜ ՜൜ ଵ 
ͳͷͲι ൅ ߙ ൌ ʹ͹Ͳι ߙ ൌ ͳʹͲι ‫ݖ‬ଶ ൌ ʹଵଶ଴ι

8QGHOVYqUWH[VG¶XQTXDGUDWDPEFHQWUHHQO¶RULJHQWpFRRUGHQDGHV  )HVVHUYLUHOV


QRPEUHVFRPSOH[RVSHUGHWHUPLQDUQHHOVDOWUHVYqUWH[VLO¶jUHD



0zGXO    


$UJXPHQW tg D o D  o 

(OVYqUWH[VVyQ  o  cos  o  i sen  o   i o  

 o o      o o     o o   



/¶jUHDGHOTXDGUDWpV    

360
342

0QODTGUEQORNGZQU 65


&DOFXODODVXPDGHOVSULPHUVWHUPHVG¶XQDSURJUHVVLyDULWPqWLFDGHGLIHUqQFLDiLTXHWp
GHSULPHUWHUPH±±i

݀ ൌ ͳ ൅ ʹ݅
ܽଵ ൌ െ͸ െ ʹ݅
ܽ௡ ൌ ܽଵ ൅ ሺ݊ െ ͳሻ݀
ܽଵ଴ ൌ െ͸ െ ʹ݅ ൅ ሺͳͲ െ ͳሻሺͳ ൅ ʹ݅ሻ ൌ ͵ ൅ ͳ͸݅
ሺܽଵ ൅ ܽ௡ ሻ݊
ܵ௡ ൌ 
ʹ
െ͸ െ ʹ݅ ൅ ͵ ൅ ͳ͸݅ሻͳͲ
ܵଵ଴ ൌ ൌ െͳͷ ൅ ͹Ͳ݅
ʹ
6LPSOLILFDO¶H[SUHVVLyVHJHQWSHUDFDGDQ‫א‬Գ
LLL«LQ

2EVHUYHPTXHI +ii 

6LJXLmԳ

6Ln mOODYRUVO¶H[SUHVVLyYDO

6Ln mOODYRUVO¶H[SUHVVLyYDOi

6Ln mOODYRUVO¶H[SUHVVLyYDOi

6Ln mOODYRUVO¶H[SUHVVLyYDO

6LHOQRPEUHFRPSOH[DEiWpPzGXOPLDUJXPHQWહFRPH[SUHVVDULHVHQIRUPDELQzPLDXQ
૜ૈ
QRPEUHFRPSOH[DPEPzGXOPLDUJXPHQWૈહ",VLHOPzGXOpVPLO¶DUJXPHQWpVહ ൅ 

6LJXLZ FGiHOQRPEUHFRPSOH[TXHWpSHUPzGXOPLDUJXPHQWɎ±Ƚ

ܿ ൌ ͸ඥܽଶ ൅ ܾ ଶ …‘•ሺʹߨ െ ߙሻ ൌ ͸ඥܽଶ ൅ ܾ ଶ …‘• ߙ

݀ ൌ ͸ඥܽଶ ൅ ܾ ଶ •‹ሺʹߨ െ ߙሻ ൌ െ͸ඥܽଶ ൅ ܾ ଶ •‹ ߙ


ଷ஠
6LJXLY HIi HOQRPEUHFRPSOH[TXHWpSHUPzGXOPLDUJXPHQWȽ ൅ 

͵Ɏ
݁ ൌ ͵ඥܽଶ ൅ ܾ ଶ …‘• ൬ߙ ൅ ൰ ൌ ͵ඥܽଶ ൅ ܾ ଶ •‹ ߙ
ʹ
͵Ɏ
݂ ൌ ͵ඥܽଶ ൅ ܾ ଶ •‹ ൬ߙ ൅ ൰ ൌ െ͵ඥܽଶ ൅ ܾ ଶ …‘• ߙ
ʹ
 

361
343

Nombres complexos
0QODTGUEQORNGZQU 5

&RQVLGHUD‫ ܢ‬ൌ ‫ܚ‬હ XQQRPEUHFRPSOH[HQIRUPDSRODUL]HOVHXFRQMXJDW&DOFXODHOYDORUGHO


TXRFLHQW

‫ ࢻ ܛܗ܋‬൉ ࢉ࢕࢙ࢻ ൉ ǥ ൉ ࢉ࢕࢙࢔ࢻ



ሺࢠ ൅ ࢠതሻ ൉ ሾࢠ૛ ൅ ሺࢠതሻ૛ ሿ ൉ ǥ ൉ ሾࢠ࢔ ൅ ሺࢠതሻ࢔ ሿ

‫ ݖ‬ൌ ‫ݎ‬ఈ ൌ ‫ݎ‬ሺ…‘• ߙ ൅ ݅ •‹ ߙሻ


‫ݖ‬ҧ ൌ ‫ݎ‬ଷ଺଴ιିఈ ൌ ‫ݎ‬ሺ…‘• ߙ െ ݅ •‹ ߙሻ
…‘• ߙ ൉ …‘• ʹߙ ൉ ǥ ൉ …‘• ߙ ݊
ൌ
ሺ‫ ݖ‬൅ ‫ݖ‬ҧሻ ൉ ሾ‫ ݖ‬ଶ ൅ ሺ‫ݖ‬ҧሻଶ ሿ ൉ ǥ ൉ ሾ‫ ݖ‬௡ ൅ ሺ‫ݖ‬ҧሻ௡ ሿ
…‘• ߙ ൉ …‘• ʹߙ ൉ ǥ ൉ …‘• ߙ ݊
ൌ
ሺ‫ ݖ‬൅ ‫ݖ‬ҧሻ ൉ ሾ‫ ݖ‬ଶ ൅ ሺ‫ݖ‬ҧሻଶ ሿ ൉ ǥ ൉ ሾ‫ ݖ‬௡ ൅ ሺ‫ݖ‬ҧሻ௡ ሿ
…‘• ߙ ൉ …‘• ʹߙ ൉ ǥ ൉ …‘• ߙ ݊
ൌ ൌ
ሾ‫ݎ‬ሺ…‘• ߙ ൅ ݅ •‹ ߙሻ ൅ ‫ݎ‬ሺ…‘• ߙ െ ݅ •‹ ߙሻሿ ൉ ǥ ൉ ሾ‫ ݎ‬௡ ሺ…‘• ߙ ൅ ݅ •‹ ߙሻ ൅ ‫ ݎ‬௡ ሺ…‘• ߙ െ ݅ •‹ ߙሻሿ
…‘• ߙ ൉ …‘• ʹߙ ൉ ǥ ൉ …‘• ߙ ݊ ͳ
ൌ ൌ ௡ା௡మ

ʹ‫ ߙ •‘… ݎ‬൉ ʹ‫ ݎ‬ଶ …‘• ʹߙ ൉ ʹ‫ ݎ‬௡ …‘• ݊ߙ
ʹ௡ ൉ ‫ݎ‬ ଶ


'RQDGDO¶HTXDFLy] DEi ]FGL DPEDEFLGQRPEUHVUHDOVWUREDODUHODFLyHQWUH
DTXHVWVQRPEUHVSHUTXqOHVVHYHVDUUHOVWLQJXLQHOPDWHL[DUJXPHQW

‫ ݖ‬ଶ ൅ ሺܽ ൅ ܾ݅ሻ‫ ݖ‬൅ ܿ ൅ ݀݅ ൌ Ͳ

െܽ െ ܾ݅ േ ඥሺܽ ൅ ܾ݅ሻଶ െ Ͷሺܿ ൅ ݀݅ሻ െܽ െ ܾ݅ േ ξܽଶ െ ܾ ଶ ൅ ʹܾܽ݅ െ Ͷܿ െ Ͷ݀݅


‫ݖ‬ൌ ൌ 
ʹ ʹ

3HUTXqWLQJXLQHOPDWHL[DUJXPHQWHOTXRFLHQWHQWUHODSDUWLPDJLQjULDLODSDUWHQWHUDKDGHVHUHO
PDWHL[
ଶ ଶ
ቄܽ െ ܾ ൅ ʹܾܽ݅ െ Ͷܿ െ Ͷ݀݅ ൌ Ͳ
ʹܾܽ െ Ͷ݀ ൌ Ͳ

&RPTXHWHQLPVRODPHQWGXHVHTXDFLRQVLTXDWUHLQFzJQLWHVKHPGHGHL[DUGXHVGHOHVLQFzJQLWHVHQ
IXQFLyGHOHVDOWUHV

ቀඥଶ௖ మ ାଶௗమ ାξଶ௖ቁටඥ௖ మ ାௗ మ ି௖


ܽଵ ൌ   ܾଵ ൌ ඥʹξܿ ଶ ൅ ݀ ଶ െ ʹܿ

ቀඥଶ௖ మ ାଶௗమ ାξଶ௖ቁටඥ௖ మ ାௗ మ ି௖


ܽଶ ൌ െ   ܾଶ ൌ െඥʹξܿ ଶ ൅ ݀ ଶ െ ʹܿ

ටඥ௖ మ ାௗ మ ା௖ቀඥଶ௖ మ ାଶௗ మ ିξଶ௖ቁ௜


ܽଷ ൌ   ܾଷ ൌ െඥʹξܿ ଶ ൅ ݀ ଶ ൅ ʹܿ݅

ටିඥ௖ మ ାௗమ ା௖ቀඥଶ௖ మ ାଶௗ మ ିξଶ௖ቁ௜


ܽସ ൌ   ܾସ ൌ ඥʹξܿ ଶ ൅ ݀ ଶ ൅ ʹܿ݅




AMPLIA
362
344

0QODTGUEQORNGZQU 65

(VFXOOODUHVSRVWDDGHTXDGD

D  3HUTXqHQXQDWHUQDGRVHOHPHQWVVLJXLQGLIHUHQWVKDGHVXFFHLUTXHGRVG¶HOOVVLJXLQLJXDOVLTXH
HOWHUFHUVLJXLGLIHUHQW(QDTXHVWFDVSRWVXFFHLUTXH݅ ௫ ൌ ‫ݖ‬ǡ ݅ ௫ ൌ ሺͳ ൅ ݅ሻ௬ ‫݋‬ሺͳ ൅ ݅ሻ௬ ൌ ‫ݖ‬Ǥ

6LKRSDVVHPDSRODUVWHQLP{190x,  y ā y ,z0}

Siix ziizzLPSOLFDTXH190x z0i  y ā y z190x

190x z0їz 1ixpVP~OWLSOHGH PRG 

 y ā y z10їyz0

+LKDFDVRVSHUODx(0,4,8,12,16),19FDVRVSHUODy(1,2,3,4,…18i19)XQFDVSHUODz  IDXQ
WRWDOGH5ͼ19ͼ1 95FDVRV.

Siix (1i)yeixzzLPSOLFDTXH190x   y ā y iz0z190x.

190x   y ā y їy 0ixpVP~OWLSOHGH4(0mod4).

z0z190xїzz1oxQRpVP~OWLSOHGH PRG DL[zQRpVSRVVLEOHSHUTXqKDGHVHUP~OWLSOHGH


 

+LKDFDVRVSHUODx(0,4,8,12,16),XQFDVSHUODy(0),19FDVRVSHUODz(1,2,3,4,…18i19),IDXQ
WRWDOGH5ͼ19ͼ1 FDVRV.

6L(1i)y zeixzzLPSOLFDTXH  y ā y x z0iz0z190x.

 y ā y x z0oz 2y/2eipVP~OWLSOHGH PRG  HOYDORUGHiQRpVYjOLGSHUTXqz 28!19)

z0z190xoxQRpVP~OWLSOHGH PRG ozz1

363
345

Nombres complexos

364
0QODTGUEQORNGZQU 65

 ൌ ܽ݀ ଶ െ ܽ݁ ଶ െ ʹܽ݀݁݅ ൅ ܾ݀ െ ܾ݁݅ ൅ ܿ ൌ
 ൌ ܽ݀ ଶ െ ܽ݁ ଶ ൅ ܾ݀ ൅ ܿ െ ሺʹܽ݀݁݅ ൅ ܾ݁݅ሻ ൌ Ͳ

&DOFXODOHVFLQFVROXFLRQVFRPSOH[HVGHO¶HTXDFLyVHJHQW

[[[[[ 

(OSULPHUWHUPHpVODVXPDGHOVWHUPHVG¶XQDSURJUHVVLyJHRPqWULFD
ሺ௫ ల ିଵሻ
ܽଵ ൌ ͳǡ ‫ ݎ‬ൌ ‫ݔ‬ ܵହ ൌ 
௫ିଵ

‫ݔ‬ଵ ൌ ͳ
‫ ݔۓ‬ൌ ͳ
ଶ ଺଴ι
ۖ
ሺ௫ ల ିଵሻ ఱ ‫ݔ‬ଷ ൌ ͳଵଶ଴ι
3HUWDQWUHVXOWD: ଺
ൌ Ͳ ՜  ‫ ݔ‬െ ͳ ൌ Ͳ ՜ ‫ ݔ‬ൌ ξͳ  ՜ 
௫ିଵ
‫ݔ۔‬ସ ൌ ͳଵ଼଴ι
ۖ‫ݔ‬ହ ൌ ͳଶସ଴ι
‫ ଺ݔە‬ൌ ͳଷ଴଴ι
/HVVROXFLRQVVyQ‫ݔ‬ଶ ǡ ‫ݔ‬ଷ ǡ ‫ݔ‬ସ ǡ ‫ݔ‬ହ ǡ ‹‫ ଺ݔ‬ǡ

7UREDO¶H[SUHVVLyGH]VLVDEHPTXHHOVDIL[RVGHOVQRPEUHVFRPSOH[RV]L]HVWDQDOLQHDWV



/HVFRRUGHQDGHVGHOVQRPEUHVFRPSOH[RVVyQ

$   % DE  & ܽଶ െ ܾ ଶ ǡ ʹܾܽሻ

&DOFXOHPHOVYHFWRUV

ሬሬሬሬሬԦ ൌ ሺܽ െ ͳǡ ܾሻ
‫ܤܣ‬ ሬሬሬሬሬԦ ൌ ሺܽଶ െ ܾ ଶ െ ͳǡʹܾܽሻ
‫ܥܣ‬

(OVSXQWVHVWUREHQDOLQHDWVVLHOVYHFWRUVVyQSURSRUFLRQDOV

ሬሬሬሬሬԦ ൌ ‫ܥܣݐ‬
‫ܤܣ‬ ሬሬሬሬሬԦ  ՜ ሺܽ െ ͳǡ ܾሻ ൌ ‫ݐ‬ሺܽଶ െ ܾ ଶ െ ͳǡʹܾܽሻ

ଶ ଶ ͳ ͳ
൜ܽ െ ͳ ൌ ‫ݐ‬ሺܽ െ ܾ െ ͳሻ  ՜ ‫ ݐ‬ൌ  ՜ ܽ െ ͳ ൌ ൬ ൰ ሺܽଶ െ ܾ ଶ െ ͳሻ
ܾ ൌ ‫ ݐ‬൉ ʹܾܽ ʹܽ ʹܽ

 ՜ ܽଶ െ ʹܽ ൅ ͳ ൌ „ ՜ „ ൌ ƒ െ ͳ

3HUWDQWUHVXOWDTXH‫ ݖ‬ൌ ܽ ൅ ሺܽ െ ͳሻ݅

 

365
347

Nombres complexos
0QODTGUEQORNGZQU 5

5HSUHVHQWDHOQRPEUHL3DVVD¶ODIRUPDSRODUFDOFXOD¶QOHVSULPHUHVSRWqQFLHVL
UHSUHVHQWDOHVHQHOSODFRPSOH[2EVHUYDTXHHOVDIL[RVG¶DTXHVWVQRPEUHVFRPSOH[RV
GHVFULXHQXQDFRUEDHVSLUDO

     
 
&DOFXODHQHOFDPSFRPSOH[OHVDUUHOVGHOSROLQRPLD[E[FVLVDEHPTXHVyQLJXDOVTXH
OHVGHOVSROLQRPLVF[D[ELE[F[D
6¶DSOLFDODSURSLHWDWGHODVXPDGHOHVDUUHOVG¶XQSROLQRPLGHVHJRQJUDXLVXSRVHPTXHOHVDUUHOV
FRPXQHVVyQߙLߚ
ܾ ܽ ܿ
ߙ ൅ ߚ ൌ െ ൌ െ ൌ െ ՜ ܽ ൌ ܾ ൌ ܿ
ܽ ܾ ܾ
ଵ ξଷ ଵ ξଷ
(OSROLQRPLpVD[D[DLOHVVHYHVDUUHOVെ ൅ ݅Lെ െ ݅
ଶ ଶ ଶ ଶ

&RQVLGHUDHOVFRQMXQWVGHQRPEUHVFRPSOH[RV

$ ^]DUJ>] L @  

% ^]ȁ‫ ܢ‬െ ሺ૛ ൅ ܑሻȁ`


'HWHUPLQDODSURMHFFLyRUWRJRQDOGHOFRQMXQWLQWHUVHFFLyGH$L%VREUHO¶HL[;
(OVFRQMXQWVALBVyQ

­ S½
A ®z x  yi  DUJ> z    i @ ¾ ^ x y  x  y   `
¯ ¿ 
B ^z x  yi  z    i   ` ^ x y  x     y    `
6LJXLAHOFRQMXQWIRUPDWSHOVSXQWVGHODUHFWDxy1 0,iBHOGHOVSXQWVLQWHULRUVGHOD
FLUFXPIHUqQFLDG¶HTXDFLy(x2)2(y1)2 4DPEFHQWUHC(2,1)LUDGLr 2.

366
348

0QODTGUEQORNGZQU 65

x  y   ½
/DLQWHUVHFFLyG¶DPEGyVFRQMXQWVpVODVROXFLyGHOVLVWHPD   ¾ ,TXHpVHOVHJPHQW
x    y     ¿
REHUWD(0,1)iE(2,3).
/DVHYDSURMHFFLyVREUHO¶HL[XpVHOVHJPHQWG¶H[WUHPVO(0,0)iE’(2,0),VHQVHLQFORXUHOyE’.

&RQVLGHUDODVXFFHVVLyGHQRPEUHVFRPSOH[RV^DQ`Q•
ܑ ܑ
‫ ܖ܉‬ൌ ሺ૚ ൅ ܑሻ ൉ ൬૚ ൅ ൰ ൉ ǥ ൉ ൬૚ ൅ ൰
ξ૛ ξ‫ܖ‬
'HWHUPLQDVLH[LVWHL[XQQRPEUHQDWXUDOPWDOTXH

෍ȁࢇ࢔ െ ࢇ࢔ା૚ ȁ ൌ ૚ૢૢ૙


࢔ୀ૚

(OVPzGXOVGHOHVGLIHUqQFLHVGHOVWHUPHVGHODVXFFHVVLyYDOHQ

§ i · § i · § i · § i ·§ i ·
an  an    i ā¨  ¸ āā¨  ¸    i ā¨  ¸ āā¨  ¸ā¨  ¸
© ¹ © n¹ © ¹ © n¹ © n  ¹

ª § i · § i ·º ª § i ·º
«   i ā¨  ¸ āā¨  ¸ » ā «  ¨  ¸»
¬ © ¹ © n ¹¼ ¬ © n   ¹¼

&RPTXHHOPzGXOGHOSURGXFWHpVHOSURGXFWHGHOVPzGXOVUHVXOWD

 n  
ā āā ā 
 n n 

m

¦a n 
n  an         m o m  

367
349

Nombres complexos
0QODTGUEQORNGZQU 5

MATEMÀTIQUESALATEVAVIDA
 4XqpVXQFLUFXLWHOqFWULFLSHUTXLQVHOHPHQWVHVWjIRUPDW"

8QFLUFXLWHOqFWULFpVXQFDPtWDQFDWSHOTXDOFLUFXODHOFRUUHQW(VWjIRUPDWSHUUHVLVWqQFLHV
LQGXFWjQFLHVFRQGHQVDGRUVIRQWVRGLVSRVLWLXVHOHFWUzQLFVVHPLFRQGXFWRUV

 4XqVyQODLPSHGjQFLDLO¶DGPLWjQFLDHQXQFLUFXLWHOqFWULF"

/DLPSHGjQFLDLO¶DGPLWjQFLDVyQSDUjPHWUHVSHUDQDOLW]DUXQFLUFXLWHOqFWULF/DLPSHGjQFLDpVO¶RSRVLFLy
G¶XQFRQGXFWRUDOIOX[G¶XQFRUUHQWDOWHUQ

/¶DGPLWjQFLDpVODIDFLOLWDWTXHHOFLUFXLWRIHUHL[DOSDVGHOFRUUHQW

 4XLQDUHODFLyQXPqULFDKLKDHQWUHODLPSHGjQFLDLO¶DGPLWjQFLD"


/¶DGPLWjQFLDY,pVODLQYHUVDGHODLPSHGjQFLDZ,pVDGLU Y 
Z


 'HWHUPLQDOHVDSOLFDFLRQVGHODLPSHGjQFLDLO¶DGPLWjQFLD3HUH[HPSOHTXqpVODLPSHGjQFLD
GHOFRVKXPj"4XLQDLPSRUWjQFLDHWVHPEODTXHWp"

/DLPSHGjQFLDLO¶DGPLWjQFLDV¶XWLOLW]HQHQODUHVROXFLyGHFLUFXLWVHOqFWULFV

/DLPSHGjQFLDpVODJHQHUDOLW]DFLyGHODUHVLVWqQFLDTXHRSRVDHOFRVKXPjDO¶HOHFWURFXFLyVHJRQV
OHVFRQGLFLRQVGHODSHOO

 6LXQFLUFXLWHOqFWULFWpXQDLPSHGjQFLDGH= jGHWHUPLQD¶QODPDJQLWXGDSDUWLUGHOPzGXO


GHOQRPEUHFRPSOH[LO¶DOWUDEDQGDFDOFXOD¶QO¶DGPLWjQFLDHQVLHPHQVFRPXQQRPEUH
FRPSOH[

0DJQLWXG    : 

   j   j
$GPLWjQFLD ˜ S
  j   j 

 6LHQXQFLUFXLWHOqFWULFO¶DGPLWjQFLDpV6FDOFXOD¶QODLPSHGjQFLD

 
Z : 
Y  

368
350

0QODTGUEQORNGZQU 65

 0XOWLSOLFDHOQXPHUDGRULHOGHQRPLQDGRUHQO¶H[SUHVVLyG¶DGPLWjQFLD < SHOFRQMXJDW5±;j


SHUREWHQLUODSDUWUHDOGHO¶DGPLWjQFLDFRQHJXGDFRPDFRQGXFWjQFLDLODSDUWLPDJLQjULD
DQRPHQDGDVXVFHSWjQFLD

 R  Xj R  Xj
Y ˜ 
R  Xj R  Xj R  X 

R
&RQGXFWjQFLD 
R  X


X
6XVFHSWjQFLD 
R  X


 6LXQFLUFXLWHOqFWULFWpXQDLPSHGjQFLDGH= jGHWHUPLQD¶QODFRQGXFWjQFLDLOD


VXVFHSWjQFLD

 j   j
˜ 
j j 

 
&RQGXFWjQFLD    6XVFHSWjQFLD 
 
 

369
351

 
 
 
 
 
 

7
 
 
 
 
 
 

Geometria analítica
 

 
Geometria analítica   
 
 
 
 
 

ACTIVITATS 
 
1. Copia aquests vectors i calcula gràficament
� �� i �
�� � � �� � �
��

 
2. Efectua aquestes sumes:
a) ��
�� � �
��� � �
���� b) �
�� � ��
�� � �
�����

 
3. ��� ��� � �
Copia els vectors � �� i efectua gràficament les operacions següents:
a) ��
�� � ��

b) �
��� � ���� � ��
c) �� � ��� � ����

371
 
 
 
 
 
 

Geometria analítica
Geometria analítica 7
 
 
 
 
 
 

4. �� com a combinació lineal dels vectors ��� � �


Escriu el vector � �� .
Expressa ��� en funció de �
�� i �
��, i també � �� i ���.
�� en funció de �

 
 
5. Comprova que els vectors �
�� i �
�� formen una base i expressa el vector �
���� en funció d’aquests
vectors.

 
Els vectors �
�� i �� tenen direcció diferent, per la qual cosa, formen una base.

 
 
6. Donada la base B = ��
��� �
���:
a) Calcula � �� � �
�� � � �� � �
�� i � �� � �
��.
b) Comprova que � �� formen una base.
�� i �
c) Expressa �
�� i �
�� com a combinació lineal de � ��.
�� i �

 
a)

 
b) Els vectors �� i ��� tenen direcció diferent, així doncs, formen una base.
��� ��
� ��� ��

c) �
�� � � ; �� � �
� � � �
 
 

372
 
 
 
 
 
 

Geometría analítica 77
 
 
 
 
 
 

 
7. ������� i ��
Dibuixa els punts A(3, 1) i B(–1, –1), i calcula les coordenades dels vectors �� �������, i els seus
mòduls.

AB   1  3, 1  1   4, 2 
 

 4   2
2 2
AB   20
 

BA   3   1 ,1   1    4, 2   

BA  42  2 2  20
 
 
8. Determina x perquè aquests parells de vectors siguin paral·lels:
a) (3, 2) i (9, x) b) (–1, 4) i (x, –2)

9 x 29 1 4 2   1 1
a)  x 6 b)   x 
3 2 3 x 2 4 2

9. Donats els punts A(3, –1), B(–1, 2), C (0, 2) i D (–1, –2), troba els vectors següents:
a) �� �������
������� � �� ������ � ��
b� ��� ������� ������� � ���
c� ��� �������

a) ������ � ��
�� ������ � ���,3� � ��1,�� � ��5, �1�

b) ������ � ��
2�� ������� � 2��3, �3� � �0, ��� � ��6,6� � �0, ��� � ��6,10�

c) ������ � 2��
��� ������ � ��1,0� � 2���, �1� � ��1,0� � ���, �2� � ���, �2�

10. Donats �
�� = (2, –1) i �
�� = (0, 3), efectua les operacions de vectors següents:
a) �
�� � ��
�� b) ��
�� � �
�� c) ��
�� � ��
��
a) �u� � 3�
�� � �2, �1� � �0, ��� � �2,��

b) 5u �� � �10, �5� � �0,3� � �10, �2�


�� � �

c) �u �� � ��2,1� � �0,6� � ��2,��


�� � 2�

11. Donats �
�� = (0, 2), �
�� = (1, -1) i �
���� = (0, -1), calcula.
a) �� � �
� �� c) � ��
���� � � e) �� � ��
� �� � ��
�����
b) �� � ��
� �� � �
����� d) �
�� � �
���� f ) ���
�� � ��
��
a) �
�� � �� � �0,2� � �1, �1� � �2
b) �
�� � ��� � �
���� � �0,2� � ��1, �1� � �0, �1�� � �0,2� � �1, �2� � ��
c) �
��� � �� � �0, �1� � �1, �1� � 1
d) � ��� � �0,2� � �0, �1� � �2
�� � �
e) �
�� � ��� � 2�
���� � �0,2� � ��1, �1� � �0, �2�� � �0,2� � �1,1� � 2

373
 
 
 
 
 
 

Geometria analítica
Geometria analítica 7
 
 
 
 
 
 

f) �2�
�� � 3�� � �0, �4� � �3, �3� � 12
12. Indica quins dels vectors següents són perpendiculars entre ells i quins no ho són:

�� = (�1, 3)
� �� = (12, 4)
� ���� = � , ���


 
u  v 112  3  4 0
  1 10
uw  1  3   1   0
3 3
  1
v  w  12   4   1  0
3
   
Són perpendiculars uiv,v i w.

13. Calcula l’angle dels vectors següents:


a) a = (2, -1) i b = (3, 2) c) a = (–3, –1) i b = (2, 3)
b) a = (5, 2) i b = (–1, 1) d) a = (–1, 5) i b = (4, –2)

2  3  2   1 4
a) cos
      60,3o
22  12 32  22 65
1 5  2 1 3
b) cos
      113, 2o
52  22 12  12 58
3  2  3   1 9
c) cos      217,9o
 3   1 22  32
2 2
130

1 4  5   2  14
d) cos      232,1o
 1  52 42   2 
2 2
520

14. Troba tres vectors perpendiculars i tres més de paral·lels als vectors següents:
a) �� = (1, 1)
� b) ��� = (3, 2) c) �
�� = (0, 1)
Resposta oberta.
a) �� = (1, 1)
Vectors paral·lels: �2,2�, �3,3� � �4,4�
Vectors perpendiculars: ��1,1�, ��2,2� � ��3,3�
�� = (3, 2)
b) b
Vectors paral·lels: �6,4�, �9,6� � �12,��
Vectors perpendiculars: �2, �3�, �4, �6� � �6, �9�
c) �� = (0, 1)
Vectors paral·lels: �0,2�, �0,3� � �0,4�
Vectors perpendiculars: �1,0�, �2,0� � �3,0�

374
 
 
 
 
 
 

Geometría analítica 77
 
 
 
 
 
 

15. Donats els punts A (–1, 3), B (5, 1) i C (0, 3), calcula la distància del punt C al punt mitjà de A i
B.

 1  5 3  1 
Punt mitjà de A i B: M ,    2, 2 
 2 2 

d  C, M   CM   2  0   2  3 
2 2
5
         
 
16. Determina els simètrics dels punts A(2, –5) i B(–1, 3) respecte del punt C(2, –1).

 2  x 5  y 
 2, 1 
 ,   x 2, y 3
 2 2   

El simètric respecte de A és  2,3 .


 1  x 3  y 
 2, 1 
 , x5, y 5
 2 2   

El simètric respecte de B és  5, 5 .

17. Escriu l’equació vectorial i les paramètriques de la recta que passa pels punts A(7, 3) i B(2, 2).
������ � ���, ���
��
Equació vectorial:
������ � ��
�� ������ � ���
������ → ��, �� � �7,3� � ����, ���
Equacions paramètriques:
� � 7 � ��

� �3��

18. Determina les equacions paramètriques de la recta que té de vector director �


�� = (-1, 0) i que
passa pel punt A(3, 2).
� �3��
�  
���
 
19. Calcula l’equació vectorial i les paramètriques de les rectes bisectrius dels quadrants.
Bisectriu del primer i tercer quadrants.

Equació vectorial:

(x, y) = t(1, 1)

Equacions paramètriques:

���
�� � �

Bisectriu del segon i quart quadrants.

375
 
 
 
 
 
 

Geometria analítica
Geometria analítica 7
 
 
 
 
 
 

Equació vectorial:

(x, y) = t(1, -1)

Equacions paramètriques:

���
�� � ��

20. Troba l’equació paramètrica de la recta que passa pel punt mitjà de A (3, 1) i B (5, –3), i pel punt
C (0, 3).

 3  5 1 3  
M ,   4, 1 x t 4 
 2 2    
  y  3  t  4 
CM   4, 4  
 

21. Troba l’equació contínua de la recta que passa per A(2, –1) i té la direcció del vector ��� � �
���� ���.
Esbrina si el punt P(3, 1) és a la recta.
Calculem l’equació contínua de la recta:
� � 2 � � ����

� ��
Comprovem si el punt P compleix les equacions de la recta:
� � 2 � � ����
� � �2
2 ��
Així, el punt P no pertany a la recta.
 
22. Determina l’equació general de la recta que passa per A (0, –2) i B (4, –1).
 x  0 y   2 x
AB   4,1      y  2  0  x  4 y  8  0  
4 1 4
23. Donades les següents equacions paramètriques d’una recta, determina:
� � � � ��

� � �� � �
a) L’equació contínua de la recta.
b) L’equació general de la recta.
x 1
x  1  2t  t   x 1
a)  2   y3

y 3  t  2
t y  3

x 1
 y  3  2 y  6  x  1  x  2 y  5  0
b) 2
   

376
 
 
 
 
 
 

Geometría analítica 77
 
 
 
 
 
 

24. Troba l’equació punt-pendent i l’explícita de la recta que passa per A(2, –3) i té la direcció del
vector �
�� = (–2, 1).
d2 1
m  
d1 2
1
Equació punt-pendent: y  3   x  2
2
1 1
y  x  n  3   2  n  n 2
2 2
1
Equació explícita: y   x2
2
 
25. Calcula la recta que passa pel punt A(2, 7) i forma un angle de 60º amb l’eix d’abscisses.
Explica com ho fas.
Calculem el pendent: tg 60º = m → m = √3

Esbrina l’equació punt-pendent: ��� ��� � � �3�� � 2�


 
26. Estudia la posició relativa de les rectes r i s.
� �����
�� � � � � ��� �
� � � ��

Les equacions generals de r i s són:


r : x  3 y  15 
0 s : 3x  y  6 0
� ��
Donat � , les rectes són secants.
� �

27. Estudia la posició relativa de dues rectes que tenen vectors directors no proporcionals. Quina
condició cal que verifiquin perquè les rectes siguin perpendiculars?
Si els vectors directors no són proporcionals, les rectes són secants.

Sigui �� � ��� � �� � el vector director de la recta r, i sigui �� � ��� � �� � el vector director de la recta s.

Les rectes són perpendiculars si el producte escalar dels vectors és zero.

�� � �� � �� � �� � �� � � �� � �� � ��� � �� � � �� � ���� � �� �

28. Troba la distància entre el punt P(2, -1) i la recta r, que té d’equació:
��� ���
�� �
� �
Expressem la recta en forma general
��� 2��
�� � � 2� � 3� � � � �
3 2

377
 
 
 
 
 
 

Geometria analítica
Geometria analítica 7
 
 
 
 
 
 

���� � ��� � �� |2 � 2 � 3 � ��1� � 8| 7 7√13


d�P, r� � � � �� �
√�� � �� √2� � 3� √13 13
 
29. Calcula la distància que separa la recta següent de l’orígen de coordenades:
�����
�� �
� � � � ��
Expressem la recta general: 
����� ��2
� � � � �� � 3 � � �2� � � � 8 � � 
� � � � �� 2
���� � ��� � �� |�2 � � � 1 � � � 8| 8 8√5
d�P, r� � � � �� �
√�� � �� �2� � ��1�� √5 5

 
30. Calcula la distància entre aquestes dues rectes paral·leles:
� ���
�� �� � �� � � � � ��� � �
� �

Prenem el punt P  0,1 que pertany a la recta r.

L’equació general de la recta s és 5 x  2 y  6 0.

5  0  2 1  6 4 4 29
d  P
,s   
52  2 2 29 29

31. Calcula l’angle que formen aquestes dues rectes quan es tallen:
���
�� � � � � � ��� � �� � �
��

1  1  1  2 1 10
cos       71,57 o
 1  12 12  22
2
10 10
 
 
   

378
 
 
 
 
 
 

Geometría analítica 77
 
 
 
 
 
 

SABER FER 
 
32. Indica si els vectors ࢛ ሬԦ formen una base i determina les coordenades del vector ࢝
ሬԦܑ࢜ ሬሬሬԦ respecte
de la base en cada cas:
a) ࢛
ሬԦ ൌ ሺെ૚ǡ ૙ሻǡ ࢜
ሬԦ ൌ ሺ૛ǡ െ૜ሻ࢏࢝
ሬሬሬԦ ൌ ሺ૜ǡ െ૜ሻ b) ሬԦ ൌ ሺ૜ǡ െ૜ሻǡ ࢜
࢛ ሬԦ ൌ ሺ૚ǡ െ૝ሻܑ࢝
ሬሬሬԦ ൌ ሺ૞ǡ െ૛ሻ
1 0
a) Formen una base, ja que  .
2 3
 3, 3  a  1,0  b  2, 3
3 a  2b   
  b 1, a 1  w u  v
3 3b 
3 3
b) Formen una base, ja que  .
1 4
 5, 2 a  3, 3  b 1, 4  
5 3a  b
    
  b 1, a 2  w 2u  v  
2 3a  4b
 
ሬሬሬሬሬሬԦ i CD són equivalents, amb A (2, 1) i B (0, -2), calcula l’extrem desconegut
ሬሬሬሬሬሬԦ࢏࡯ࡰ
33. Si els vectors ࡭࡮
en aquests casos:
a) C(–5, 7) c) C(0, –2) e) C(7, –5) g) C(0, 0)
b) D(–5, 7) d) D(0, –2) f ) D(7, –5) h) D(0, 0)
 

AB  2, 3 
 

a)  CD  a  5, b  7   2, 3  D  7, 4   

b)  CD   5  a,7  b    2, 3  C  3,10   

c)  CD  a, b  2  2, 3  D  2, 5  

d)  CD   a, 2  b    2, 3  C  2,1  

e)  CD  a  7, b  5  2, 3  D  5, 8  

f)  CD   7  a, 5  b    2, 3  C  9, 2  

g)  CD  a, b   2, 3  D  2, 3  

h)  CD  a, b   2, 3  C  2,3  
34. Determina les coordenades de dos vectors si saps que la suma i la diferència són:

379
 
 
 
 
 
 

Geometria analítica
Geometria analítica 7
 
 
 
 
 
 

�� � ��� �� �
�� � �
� �� � ���� ��
�� � �
 
u   a, b  v   c, d 
         
   
u  v   a  c, b  d    2,3 u  v  a  c, b  d   1, 4
            
ac  2  1 3 3
bd  7 1
a
  ,
c   b  , d 
a  c 1 2 2            bd 4 2 2 
 
35. Calcula els vectors següents:
a) Perpendicular a �
�� = (–2, 1) i de mòdul 2.
b) Perpendicular a �
�� = (3, 1) i de mòdul 1.
c) Perpendicular a �
�� = (–3, 4) i de mòdul 5.
d) Perpendicular a �
�� = (1, 1) i de mòdul 1.
 
a) Un vector perpendicular a u = (–2, 1) és v  1, 2  .

Per obtenir el vector de mòdul 2, multipliquem per 2 i dividim entre el mòdul del vector v:
 2 1, 2   2 4 
w   , 
12  22  5 5 
 
b) Un vector perpendicular a u = (3, 1) és v  1,3 .

Per obtenir el vector de mòdul 1, dividim entre el mòdul del vector v :

w
 1,3   1
,
3 
 
 1  32
2
 10 10 
 
c) Un vector perpendicular a u = (–3, 4) és v   4,3 .

Per obtenir el vector de mòdul 5, multipliquem per 5 i dividim entre el mòdul del vector v :
 5  4,3
w   4,3
42  32
 
d) Un vector perpendicular a u = (1, 1) és v  1,1 .

Per obtenir el vector de mòdul 1, dividim entre el mòdul del vector v :

w
 1,1   1 1 
,
 
 1  12
2
 2 2

   

380
 
 
 
 
 
 

Geometría analítica 77
 
 
 
 
 
 

36. Dibuixa el quadrilàter ABCD en uns eixos de coordenades i calcula’n el perímetre; has de tenir
present que les coordenades dels vèrtexs són A (–4, –3), B (2, –3), C (2, 1) i D (–2, 2).
 
AB
 
6, 0  6 CD   4,1 17
        
 
BC
 
0, 4  4 DA   2, 5   29
        

El perímetre del quadrilàter és P10  17  29 .


 
37. Divideix el segment AB en tres parts iguals.
a) A(–2, 3) i B(0, –1) b) A(1,1) i B(3, 6)

a) 
AB  2, 4  
1  1  4 5
P1 A  AB  2,3   2, 4    , 
  3 3  3 3 
2  2  2 1
P2 A  AB  2,3   2, 4    , 
  3 3  3 3 

AB   2,5
b)   

1  1 5 8
P1 
A  AB  1,1   2,5 
 , 
  3 3  3 3 
2  2  7 13 
P2 
A AB  1,1   2,5 
 , 
  3 3 3 3  
 
38. Comprova si els punts A, B i C estan alineats.
a) A(–2, 0), B(1, –1) i C(–5, –1) b) A(0, 0), B(3, –1) i C(2, –2)
 
a) AB  3, 1 AC  3, 1
3 1
No es troben alineats ja que  .
3 1
 
b) AB  3, 1 AC  2, 2 
3 1
No es troben alineats ja que  .
2 2
 
39. Calcula l’equació de la recta que passa per O (0, 0):
a) I passa també pel punt A(–5, 2).
b) I té pendent m = –2.

a) El vector director és d  ( 5,2) i passa pel punt O.

381
 
 
 
 
 
 

Geometria analítica
Geometria analítica 7
 
 
 
 
 
 

x  5t 
Equacions paramètriques → 
y  2t 

b) Equació punt-pendent → y  2x


40. Determina l’equació de la recta paral·lela a ࢘ ‫ ׷‬૜࢞ െ ࢟ െ ૜ ൌ ૙ que passa pel punt P (0, -4).

El vector director és d  1,3 .

xt 
Equacions paramètriques → 
y 4  3t 
 
࢞ି૚ ࢟ି૚
41. Determina la recta s paral·lela a r : ൌ que és a 2 unitats de distància.
૜ ૝

L’equació de la recta s és 4 x  3 y  C 
0.

Un punt de la recta r és P 1,1 .

4 1  3  1  C 1 C
d  P, s    2  C1 9, C2 11
2
4 3 2 5
 
42. Determina l’equació de la recta perpendicular a ࢘ǣ ૜࢞ െ ࢟ െ ૜ ൌ ૙ que passa pel punt P (0, –4).

El vector perpendicular al vector director de r és  3, 1 .


x y4
Equació en forma contínua → 
3 1

43. Calcula l’equació de la recta que passa per P (0, 1) i forma un angle de 45º amb ࢘ ‫ ࢞ ׷‬൅ ࢟ െ
૚ ൌ ૙.
Equació punt-pendent → y  1 mx
Equació general → mx  y  1 0

o 2 m  2  1 1
cos 45
 
2 m2  1  5
5 1
 2m  1  m2  1  3m2  8m  3 
2
0  m1 
3, m2 

2 3
Hi ha dues rectes:
1
s1 : y 1 3x s2 : y  1  x
3
   

382
 
 
 
 
 
 

Geometría analítica 77
 
 
 
 
 
 

ACTIVITATS FINALS 
 
44. Fixa’t en la figura i efectua les operacions següents:
     
a) AB + BC d) DA + DC g) OA - CB
     
b) DB + CD e) DB - CB h) OD - BC
     
c) OA + BC f) DC - AC i) OA + AB

 
        
a) AB + BC = AC d) DA + DC = DB g) OA - CB = OD
        
b) DB + CD = DA e) DB - CB = DC h) OD - BC = OA
        
c) OA + BC = OD f) DC - AC = DA i) OA + AB = OB

45. Fes les operacions següents:

383
 
 
 
 
 
 

Geometria analítica
Geometria analítica 7
 
 
 
 
 
 

 
 
 
 

46. Donats els vectors següents, calcula gràficament ��


�� � ����� � ��
����.

   
           
 
�� i ��� de la figura formen una base.
47. Comprova que els vectors �

                   
Dibuixa en aquesta base els vectors amb aquestes coordenades:
a) (2, 1) b) (3, –1) c) (–2, 3)
Com que els vectors tenen diferent direcció, formen una base.

 
 
�� = (–1, –3) i �
48. Comprova si els vectors � �� = (4, 2) formen una base i, si és així, calcula les
coordenades dels vectors següents respecte de la base:

a) �
�� = (2, 1) b) ��� = (3, –1) c) �� = (–2, 3)

384
 
 
 
 
 
 

Geometría analítica 77
 
 
 
 
 
 

-1 -3
Formen una base ja que ¹ .
4 2
a) (2,1) = a (-1, -3) + b (4, 2)
2 = -a + 4b ü
ï
ï 1
ý  a = 0, b =
1 = -3a + 2bï
ï
þ 2

b) (3, -1) = a (-1, - 3) + b (4, 2)


3 = -a + 4b ü
ï
ï
ý  a = 1, b = 1
-1 = -3a + 2bï
ï
þ
c) (-2,3) = a (-1, -3) + b (4, 2)
-2 = -a + 4büï
ï 8 9
ý a =- , b =-
3 = -3a + 2b ï
ï
þ 5 10

49. Escriu els vectors ���, �������, b i c com a combinació lineal de �


�� i ���.

 
Escriu les coordenades dels vectors respecte de la base {� ��, ��� }.
    
u = (1, -1) v = (1, 2) a = (0, -5) b = (1, -4) c = (2,1)

0 = a +b ü
ï
ï 5 5
(0, -5) = a (1, -1) + b (1, 2)  ý a = , b =-
-5 = -a + 2bï
ï
þ 3 3

1= a +b ü
ï
(1, -4) = a (1, 2) + b (1, -1)  ýï  a = 2, b = -1
-4 = -a + 2bïï
þ
2 = a +b ü ï
ï
(2,1) = a (1, 2) + b (1, -1)  ý  a = 1, b = 1
1 = -a + 2bï
ï
þ
 

50. Expressa el vector � � = (–1, 3).


�� (7, 6) com a combinació lineal dels vectors ��� = (6, –3) i �
7 = 6a - b üï
ï 27 19
(7, 6) = a (6, -3) + b (-1,3)  ý a = , b =  
6 = -3a + 3bï
ï
þ 15 5

 
� , ��, ��� � �
51. Calcula el mòdul dels vectors ���, � � i ��
� � ��, donats ��� = (–3, 4), �
� = (–5, –12) i �� = (3, –1).

|���| � ���3�� � �� � �       �� � ��8, �8� 


��� � b
��� � ������ � ��12�� � 13  
�b ��� � ���8�� � ��8�� � √128 � 8√2 
���� � b

|��| � �3� � ��1�� � √10        �� � � � ��10, �2�� � ��3,1� � ��13, �23� 


2b

                    �� � �� � ���13�� � ��23�� � √698 


�2b
 

385
 
 
 
 
 
 

Geometria analítica
Geometria analítica 7
 
 
 
 
 
 

�� i ������
52. Calcula les coordenades del punt B perquè els vectors � �� siguin equivalents si sabem que
�� = (2, –3) i A(–1, 2).


AB = ( x +1, y - 2) = (2, -3)  B = (1, -1)
 
 
53. El triangle equilàter següent està inscrit en una circumferència de 5 cm de radi.

 
Calcula els productes escalars següents:
   
a) OA ⋅ OB b) OA ⋅ OC
   
a) OA ⋅ OB = OA ⋅ OB ⋅ cos a = 25 ⋅ cos (-120o ) = -12,5
   
b) OA ⋅ OC = OA ⋅ OC ⋅ cos a = 25 ⋅ cos120o = -12,5

 
54. Considera que A, B, C i D són els vèrtexs d’un quadrat d’1 cm de costat. Calcula els productes
escalars següents:
a) �������
�� � �� ������� c) �������
�� � �� �������

b� ������
�� � ��������� d� ������
�� � ���������
Prenem el punt A com a orígen, i tenim les següents coordenades: A(0, 0), B(1, 0), C(1, 1) i D(0, 1)

������ � �1,0�, ��
a) �� ������ � �0,1� 

  �1,0� � �0,1� � 0 

������ � �1,1�, ��
b) �� ������� � �1, �1� 

�1,1� � �1, �1� � 1 � 1 � 0 

������ � �0,1�, ������


c) �� �� � �0, �1� 

  �0,1� � �0, �1� � �1 

������ � �1,1�, ��
d) �� ������ � �0, �1� 

  �1,1� � �0, �1� � �1 

 
55. Si �
�� = (3, 1) i ��� = (2, -1), calcula.
�� � ���
c� � c� ���
�� � ����� � ���
�� � ����
d� � �� � ��
d� � �� � ����

386
 
 
 
 
 
 

Geometría analítica 77
 
 
 
 
 
 

a) u �� � �3, 1� � �2, �1� � 6 � 1 � 5 


�� � �
b) u �� � �3,1� � �4, �2� � 12 � 2 � 1�
�� � 2�
c) �2u
�� � 3� �� � ��6,2� � �6, �3� � �2, �1� � �2, �1� � �12, �1� � �2, �1� � 24 � 1 � 25
��� � �
d) �u� � �u ��� � �3,1� � ��3,1� � ��2,1�� � �3,1� � �1,2� � 3 � 2 � 5
�� � �
 
� = (–3, 1) i �� = (4, 3), calcula.
56. Donats els vectors ��� = (2, –1), �
� � � ��
a) (���� � �� b ) ��� � �� � �� � ��

a)   2, 1  2  3,1    4,3 


8, 3   4,3 
23

b)  2, 1 3,1   3,1 4,3 7   9 2


 
�� � ��� � �, si �
57. Calcula el valor de t perquè � �� = (–1, 2) i ��� = (3, t ). Troba el mòdul dels dos vectors.
�u� � �
�� � � 
��1, 2� � �3, t� � � � �3 � 2t � � � t � 5 
|u
��| � ���1�� � 2� � √5 

��| � �3� � 5� � √34 


|�
 
58. Determina un vector perpendicular a � �� = (–3, 2) amb mòdul 2.
 
Un vector perpendicular a u és v   2,3 . Perquè tingui mòdul 2, multipliquem per 2 i dividim entre el

mòdul de v :
 2  2,3  4 6 
w   , 
13  13 13 

59. Donat el vector �


�� = (6, 2), troba un vector �
�� amb mòdul √�� tal que �
�� � �
�� � ��.
�� � �6, 2� � ��, �� � 6� � 2� � 14 � � � � � 3�
��� � �

4
� ��
��| � ��� � �� � 3��� � √89 � 1��� � 42� � 4� � � � � �
|� 5
�� � 5
� ��
Si � � � � � �
� �

Si � � 5 � � � �8

 
60. Calcula el valor de k perquè els vectors siguin perpendiculars.
� �
a) �
�� = (2, k), �
�� = (1, –6) c) �
�� � � , � ,��
�� = (k, –1)
� �

b) �
�� = (� , k), �
�� = (5, –1) d) �
�� = (2, –3),��
�� = (1, k)

387
 
 
 
 
 
 

Geometria analítica
Geometria analítica 7
 
 
 
 
 
 

1
a)  2, k   1, 6   2  6k  0  k 
3
 2  10 10
b)   , k    5, 1   k 0  k 
 3  3 3
1 2 1 2 6
c)  ,    k , 1  k   0  k 
3 5 3 5 5
2
d)  2, 3  1, k   2  3k  0  k 
3
 
�� = (7, –2) i �
61. Calcula m perquè � ���� = (m, 6):
a) Siguin perpendiculars.
b) Siguin paral·lels.
c) Tinguin el mateix mòdul.
��
a) �7, 2� � ��, 6� � 7� � 12 � 0 � � � �

� ��
b) � � � � �21
� �

c) |��| � �7� � ��2�� � √53


���| � �� � 6� � � � √17
|�

62. Donats � �� = (16, 12), calcula el valor de m perquè els vectors �
�� = (6, –2) i � �� + ��� i �
�� = m� �� = m�
�� –
�� siguin perpendiculars. Hi ha una solució única?

�� � ��6, �2� � �16, 12� � �16 � 6�, 12 � 2�� 

�� � ��6, �2� � �16, 12� � ��16 � 6�, �12 � 2�� 
Perquè els vectors siguin perpendiculars, el seu producte escalar ha de ser zero.
�16 � 6�, 12 � 2�� � ��16 � 6�, �12 � 2�� � 36�� � 256 � ��� � 1�� � 0 
� � � �√10 
La solució no és única.

63. Calcula l’angle que formen els parells de vectors següents:


�� = (0, –2) i ��� = (–4, –3)
a) � �� = (–4, –3) i ��� = (1, 1)
c) �

�� = ( , 5) i �
b) � �� = (3, –1) d) � �� = (1, √�)
�� = (�, �√�) i �


0, 2   4, 3 3
a) cos 
    306,87o
25 5
1 
 ,5    3, 1 12
3 
cos  
b)      255,38o
226 2260
10
9

388
 
 
 
 
 
 

Geometría analítica 77
 
 
 
 
 
 

c) cos 
 4, 3  1,1 7
  188,13o
5 2 5 2

d) cos 


1,  3   1, 3 

1
   120 o
22 2
 

�� = (1, k) i ��� = (2, –3) formin un angle de 120º.


64. Troba el valor de k perquè els vectors �

1 2  3k 24  13 3
cos120o   k 
2 1  k 2 13 23  
 
�� = (3, –4) i tal que |�
65. Calcula un vector a que formi un angle de 30º amb � ���.
��| = √� � ��
��
a�� � � √3 3a � 4�
��� 30� � → � → 15 � 5√3 � 6a � 8�
|a��| � �� ��� 2 5√3 � 5

�a� � � � � 5√3 → a� � �� � �5
El sistema es resol:

15 � 5√3 � 8� 15 � 5√3 � 8�
�6� � �8� ��15 � 5√3 → � � →� � � � � � �5
� � � � �5 6 6

50� � � 40√3�√3 � 1�� � �5√3 � 1200 � 0

�√��√���� �������√�
→��� ��
� ��

 
66. Determina si el triangle de vèrtexs A(12, 10), B(20, 16) i C(8, 32) és rectangle.
Calculem els vectors formats pels vèrtexs del triangle:
�� ������ � ��12,16� � ��
������ � �8, 6�, �� ������ � ��4,22�
Calculem els mòduls dels vectors:
������ � √64 � 36 � 10 �
��
������ � √144 � 256 � √400 � 20 �
��
������ � √16 � 484 � √500 �
��
Si el triangle és rectangle, ha de verificar el teorema de Pitàgores.
� � �
������ � � ���
��� ������� � ���
�������

10� � 20� � 500


Així es comprova que el triangle és rectangle.
   

389
 
 
 
 
 
 

Geometria analítica
Geometria analítica 7
 
 
 
 
 
 

67. Tres dels vèrtexs d’un paral·lelogram són A(2,1), B(6, –1) i C(7, 1). Quines són les possibles
coordenades de l’altre vèrtex?
 
▪  AB  DC   4, 2    7  a,1  b   D  3,3  
 
▪  AC  DB   5, 0    6  a, 1  b   D 1, 1  
 
▪  AC  BD   5, 0    a  6, b  1  D 11, 1  
 

�� = (1, 5) i ��� = (–4, –3), calcula.


68. Donats els vectors �
�� · ��� i ��� · �
a) � ��
b) |� ���
��| i ��

�� i ���.
c) L’angle que formen els vectors �
d) El valor de k perquè el vector (3, k) sigui perpendicular al vector �
��.
��.
e) El valor de k perquè el vector (k, –1) sigui paral·lel al vector �
f ) Un vector perpendicular al vector ���.
   
a) a  b b  a 1,5   4, 3 19
 
b) a  26 b 5

19

c) cos  138,18o

5 26

3
d)  3, k   1,5 
3 5k 0  k 
5
k 1 4
e)   k 
4 3 3
 
f) Un vector perpendicular a b és, per exemple, c   3, 4  .

 
69. Quines condicions han de complir els vectors �
�� i �
�� perquè (�
�� + ���) · (�
�� - �
��) = 0?
Amb aquest resultat, demostra que si un paral·lelogram té les diagonals perpendiculars,
només pot ser un quadrat o un rombe.
�� � ��� ��
� �� � ��� ��
�� � �� � � �� � �� � �� � � �� � ���� → �� � � � � � � � � � � � → �� � � � � � � � � �
→ |�
��| � |��|
Si �
������� són els costats d’un paral·lelogram, � �� � �� són les seves diagonals.
�� � ������
Per tant, si les diagonals són perpendiculars, els mòduls mesuren el mateix, així doncs únicament pot
ser un quadrat o un rombe.
   

390
 
 
 
 
 
 

Geometría analítica 77
 
 
 
 
 
 

70. Calcula el perímetre d’un triangle que té els vèrtexs situats en els punts A(1, 2), B(3, 2) i C(–1,
3).
Calculem els vectors �� ������ � ��4,1������
������ � �2,0�, �� ������ � �2, �1��
Calculem el mòdul dels vectors:
������� � �2� � 0� � 2
���
������ � � ���4�� � 1� � √17
���
������� � �2� � ��1�� � √5
���
El perímetre és:
2 � √17 � √5 � �,3���

71. Demostra que el quadrilàter format pels punts A(2, –2), B(5, 3), C(0, 6) i D(–3, 1) és un quadrat.
   
Els costats AB i DC , BC i AD són paral·lels:
   
AB
  3,5
  DC BC   5,3 
AD
   
A més a més, AB i BC , AB i AD són perpendiculars:
 
AB  BC
  3,5   5,3
  0
Com que tots els costats són iguals:
   
AB
 DC  BC AD  34

Llavors és un quadrat.

72. Demostra que el triangle de vèrtexs A(3, 1), B(9, –1) i C(5, –5) és isòsceles. També és equilàter?
Quins són els costats iguals? Calcula’n l’àrea.
Determinem els vectors formats pels vèrtexs del triangle:

�� ������ � ��4, �4������


������ � ��, �2�,����� ������ � �2, ���

Calculem els mòduls dels vectors:

������� � √3� � 4 � √40��


���

������ � � √1� � 1� � √40��


���

������ � � √4 � 3� � √40��
���

Com que el triangle té dos costats iguals, AB I AC, és un triangle isòsceles.

Per determinar l’àrea calculem l’altura, h, sobre el costat BC aplicant el teorema de Pitàgores:


√��
h = ��√40�� � � � � √40 � � � √32��

��� √���√��
A= � � 1����
� �

73. Determina el punt mitjà dels segments que tenen els extrems següents:

391
 
 
 
 
 
 

Geometria analítica
Geometria analítica 7
 
 
 
 
 
 

a) A(3, 5) i B(9, 11) c) A(4, 5) i B(7, 1)


b) A(–3, 1) i B(7, –4) d) A(–6, –1) i B(–9, –3)
M és el punt mitjà de A i B.
��� ����
a) � � � , � � ��,��
� �
���� ��� �
b) � � � , � � �2, � �
� � �
��� ��� ��
c) � � � , � � � , 3�
� � �
���� ���� ��
d) � � � , � � �� , �2�
� � �

 
74. Si el punt mitjà del segment AB és M(3, 5), donat A(9, 7), calcula el punt B. Després, calcula A
amb M(–1, 5) i B(4, –9).
Sigui B(x,y).
���
��� ��� 3�
�3,�� � � , ��� 2 � �� � �3
2 2 ��� ��3
��
2
Sigui A(x,y).
���
� � � �� � � �� �
���,�� � � , ��� 2 � �� � ��
2 2 �� � � � � ��
��
2
75. Dos vèrtexs consecutius d’un quadrat són A(–2, 3) i B(1, –2). Si les dues diagonals es tallen en
el punt O(2, 2), calcula els dos vèrtexs que falten.
 
AO  4, 1        
BO  1, 4 
 

C A  2 AO  2,3   8, 2   6,1
 

D  B  2 BO  1, 2    2,8    3, 6 
 
76. Tres vèrtexs consecutius d’un hexàgon regular tenen de coordenades (0, 0), (2, 0) i (3,√�).
Determina els altres tres vèrtexs.
Sabem que A(��0,0�, ��2,0� � ��3, √3��

Calculem el vèrtex D, amb una translació amb orígen en C i vector guia (-1, √�):

� � �3, √3� � ���, √3� � �2,2√3�

Determinem el vèrtex E, amb una translació amb orígen en D i vector guia (-2,0):

� � �2,2√3� � ��2,0� � �0,2√3�

Determinem el vèrtex F, amb una translació amb orígen en E i vector guia (-1,- √3):

� � �0,2√3� � ���, �√3� � ���, √3�

392
 
 
 
 
 
 

Geometría analítica 77
 
 
 
 
 
 

77. Calcula les coordenades dels punts que divideixen el segment d’extrems A(5, –1) i B(17, 8) en
tres parts iguals.
Calculem el vector �� ������ � �12,9�   
������:���������������  
� �
El primer punt estarà situat a   de distància d’un dels extrems del segment, i el segon, a   de distància. 
� �
1 1
������ � �5, �1� � �12,9� � �9,2� 
�� � � � ��
3 3
2 2
������ � �5, �1� � �12,9� � �13,5� 
�� � � � ��
3 3
 

78. Determina el valor de k perquè els punts A(2, –3), B(9, k) i C(6, –1) estiguin alineats.
 
AB  7, k  3           AC   4, 2   

7 k 3 1
 k
  
4 2 2
 
79. Calcula la longitud de la mitjana amb l’orígen a A en el triangle de vèrtexs A(–1, 4), B(6, 5) i
C(10, –3). En el cas d’aquest triangle, la mitjana coincideix amb l’altura? Justifica la resposta.

 
Calculem el punt mitjà, M, del segment CB:

� � 10 5 � 3
��� , � � ��,1�
2 2

�������:
Per determinar la longitud de la mitjana, determinem el mòdul del vector ��

������� � �9, �3�


�� �������� � √�1 � 9 � √90
���

Perquè la mitjana, AM, coincideixi amb l’altura sobre el costat CB, els costats AC i AB han de ser
iguals.

������ � �11, �7�


�� ������ � �7,1�
��

������ � � √121 � �9 � √170


��� ������� � √�9 � 1 � √50
���

Així doncs, la mitjana no coincideix amb l’altura.

   

393
 
 
 
 
 
 

Geometria analítica
Geometria analítica 7
 
 
 
 
 
 

80. Dos vèrtexs d’un triangle equilàter són A(3, 1) i B(5, –2). Calcula quines poden ser les
coordenades del vèrtex que falta.

AB  2, 3 
 
La recta que passa pel tercer vèrtex, C(cx, cy), i pel punt mitjà de A i B i té com a vector director
   1
d  (3, 2) . El punt mitjà del vector AB és M  4,   . Així, aquesta recta té com a equació:
 2
1
y
x4 2  4 x – 6 y – 19 
 0
3 2
 
A més a més, el mòdul de AC és el mateix que el mòdul de AB .

  c y  1  cx 2  6 x  c y 2  2c y  3  0
2
 cx  3
2
13 

Resolem el sistema d’equacions:



19  6c y  cx 4  3 3 , c y  1  3 
cx   2 2 
4   Serien les possibles coordenades del punt C.
0  cx 4  3 3 1
, c y   3 
2 2
c x  6c x  c y  2 c y  3 
2 2 
81. Donats els punts A(3, 0) i B(–3, 0), determina el punt C sobre l’eix de coordenades de manera
que el triangle que descriguin sigui equilàter. Hi ha una solució única? Calcula l’àrea dels
triangles que en resultin.
Els vectors formats pels vèrtexs han de tenir la mateixa longitud.

Si C(0,c):

��� ������� � √� � � �
������� � √� � � � ��� ������� � �
���

� � �� � � � → √27

Els punts demanats són: �� ���3√3������ ��� �3√3��

Calculem l’àrea dels triangles:

� � � � � 3√3
�� � � �√3���
2 2

82. Dos vèrtexs consecutius d’un quadrat són A(2, –4) i B(8, –2). Calcula els altres vèrtexs sabent
que un dels que falten és sobre l’eix d’ordenades.
 
AB   6, 2 AB  40

El vèrtex D és sobre l’eix Y: D  0, y 


 
AD   2    y  4   40  y 2  8 y  20  0  y  2
2 2
AD   2, y  4

C D  AB  0, 2    6, 2  6, 4

394
 
 
 
 
 
 

Geometría analítica 77
 
 
 
 
 
 

83. Escriu les equacions vectorial, paramètrica, contínua, explícita i general de la recta que passa
pel punt P(0, –3) ), i té de vector director ࢜
ሬԦ = (3, –1).
Equació vectorial →  x, y    0, 3  t  3, 1
x  3t 
Equació paramètrica → 
y 3  t 

x y 3
Equació contínua → 
3 1
1
 x 3
Equació explícita → y 
3
Equació general → x  3 y  9 0
 
84. Escriu les equacions vectorial, paramètrica, contínua explícita i general de la recta que passa
pels punts P(–2, 3) i Q(5, 1).

El vector director és v   7, 2 . Llavors:

Equació vectorial →  x, y  
 2,3   t  7, 2 
x 2  7t 
Equació paramètrica → 
y 3  2t 

x  2 y 3
Equació contínua → 
7 2
2 17
Equació explícita → y 
 x
7 7
Equació general → 2x  7 y 17 
0
 
85. Determina l’equació contínua i la general de les rectes que contenen els costats del triangle de
vèrtexs A(1, 4), B(–6, –5) i C(3, –1).
▪ Costat 1:

El vector director és AB  7, 9  . Llavors:

x 1 y  4
Equació contínua → 
7 9
Equació general → 9 x  7 y  19 
0
▪ Costat 2:

El vector director és BC   9, 4  . Llavors:

x6 y5
Equació contínua → 
9 4
Equació general → 4 x  9 y  21 
0

395
 
 
 
 
 
 

Geometria analítica
Geometria analítica 7
 
 
 
 
 
 

▪ Costat 3:

El vector director és CA   2,5
x  3 y 1
Equació contínua → 
2 5
Equació general → 5 x  2 y  13 
0
 
86. Escriu les equacions d’aquestes rectes en les formes següents:
a) �� � � ���� � �� en forma paramètrica
b) ����� � ��� � ��� � �� en forma contínua
� ���
c) � en forma explícita
�� �
� � ��
d) � en forma general
� � �� � �

xt 
a) Un punt de la recta és (0, 4) i el vector director és (1, –3) → 
y 4  3t 
y7
b) Un punt de la recta és (0, –7) i el vector director és (1, 2) → x 
2
5 5
c) El pendent de la recta és  i l’ordenada en l’orígen és 3 → y   x  3
2 2
d) Un punt de la recta és (0, –2) i el vector normal és (1, 3) → x  3 y  6 0
 
87. Comprova si els punts A(–3, 2) i B(5, –1) són a les rectes següents:
� � � � ��
a) � c) ��� � ���� � ���� � ��
� � � � ��
���� ��� ���
b) � � d) �
� � �

�3 � 3 � �� ��3
a) �   → �� � �� 
� � 3 � �� �

El punt (‐3,2) no pertany a la recta. 
� � 3 � �� � � ��
�   → �  
�� � 3 � �� � � ��
El punt (5,‐1) pertany a la recta. 
�����
b) � �  

El punt (‐3,2) no pertany a la recta. 
�� � 3
�� �  
3
El punt (5,‐1) no pertany a la recta. 
c) ��� � � � �� � � 
El punt (‐3,2)  pertany a la recta. 
�� � � � �� � � 

396
 
 
 
 
 
 

Geometría analítica 77
 
 
 
 
 
 

El punt (5,‐1) no pertany a la recta. 
���� ���
d) �  
� �
El punt (‐3,2)  no pertany a la recta. 
� � � �� � �

3 2
El punt (5,‐1) pertany a la recta. 
 
88. Calcula el valor de k perquè la recta �� � �� � � � � passi pel punt P(–1, 4).

3   1  4  7  k 0  k 25
 
 
89. Calcula el valor de k perquè la recta �� � � � � � � passi per l’orígen de coordenades.
40  0  k  0  k  0  
 
90. Escriu dos punts de les rectes següents:
a) � � �� � � c) �� � �� � � � �
� � �� � � ��� ���
b) � d) �
� � � � �� � ��

Resposta oberta. Per exemple:

 1  1 
a)  0, 1 i 1, 2  c)  0,  i   ,0
 3  4 
b)  2, 4  i  3,1 d)  2, 1 i  1, 0 
 
91. Escriu l’equació punt-pendent de la recta que compleix les condicions següents:
a) Passa pels punts A(2, –3) i B(0, –2).
b) Passa pel punt A(0, 3) i té per direcció la del vector �
�� = (3, –1).

A(2, 3)  1  1


a)  2,1  y  3   x  2
  m   AB 
B(0, 2)  2 2
1
m   1
b) 3   y  3  x
3
A(0,3) 

 
92. Calcula el valor de k perquè la recta �� � �� � � � � tingui pendent 2.
8
El vector director és (k, 8) →. m  2k 4
k
 
93. Determina l’equació d’una recta que passa pels punts (–1, –10) i (2, c), sabent que té pendent 7.
Expressa la recta en forma contínua i general.

397
 
 
 
 
 
 

Geometria analítica
Geometria analítica 7
 
 
 
 
 
 

Determinem el vector director: (3, c + 10)


����
Com que sabem que el pendent és: � � � � 21 � � � 1� � � � 11

�� � �3� 21�

Expressem la recta en forma contínua:
��1 � � 1�
���
3 21
I l’expressem en forma general:
21x + 21 = 3y + 30 → 21x – 3y – 9 = 0
 
94. Determina l’equació en forma contínua, paramètrica, general i explícita de la recta que passa
pel punt A(2, –4) i és perpendicular a la direcció del vector ��� = (4, –1).
El vector director de la recta és el (1, 4).
y4
Equació contínua → x2
4
x 2  t 
Equació paramètrica → 
y 4  4t 
Equació general → 4 x  y  12 0
y 4 x  12
Equació explícita → 

 
95. Expressa en forma vectorial, paramètrica i contínua l’equació de la recta que:
��� ���
a) Passa pel punt (1, 3) i és paral·lela a la recta �
� ��
b) És paral·lela a la recta �� � �� � ��� � ���i passa pel punt (–2, 5).
a) Vector director de la recta: (2, -4)

Equació vectorial → (x, y) = (1, 3) + t(2, -4)

� � 1 � 2�
Equacions paramètriques → �
� � 3 � ��

��� ���
Equació contínua → �
� ��

b) Vector director de la recta: (2, 5)

Equació vectorial → (x, y) = (-2, 5) + t(2, 5)

� � �2 � 2�
Equacions paramètriques → �
� � � � ��

��� ���
Equació contínua → �
� �

96. Expressa en forma explícita la recta que:


� � � � ��
a) Passa pel punt (0, -1) i és paral·lela a la recta �
� � ��

398
 
 
 
 
 
 

Geometría analítica 77
 
 
 
 
 
 
࢞ି૜ ࢟
b) És paral·lela a la recta ൌ y passa pel punt (5, –2).
૝ ି૚
a) Vector director de la recta: (3, 0)

Pendent: m = 0
-1 = 0x + n → n = -1
Equació explícita → y = -1
b) Vector director de la recta: (4, -1)

Pendent: m = െ

ହ ଷ
-2 = െ + n → n = െ
ସ ସ
ଵ ଷ
Equació explícita → y = െ ‫ ݔ‬െ
ସ ସ
 
97. Escriu l’equació general de la recta que:
࢞ ൌ െ૚ െ ૜࢚
a) Passa pel punt (10, –2) i és paral·lela a la recta൜
࢟ ൌ ૛ െ ૛࢚
ૡ࢞ି૜
b) És paral·lela a la recta ࢟ ൌ i passa pel punt (4, 0).

௫ିଵ଴ ௬ାଶ
a) Expressem la recta en forma contínua: ൌ
ିଷ ିଶ
I l’expressem en forma general: -2x + 20 = -3y - 6 → -2x + 3y + 26 = 0
b) Vector director de la recta: (1, 4)
௫ିସ ௬
Expressem la recta en forma contínua: ൌ
ଵ ସ

I l’expressem en forma general: 4x – 16 = y → 4x – y – 16 = 0

98. Escriu en forma vectorial i paramètrica la recta que:


࢞ା૛ ࢟ା૚
a) Passa pel punt (0, -3) i és perpendicular a la recta ൌ
ି૜ ૝
b) Passa pel punt (-5, 0) i és perpendicular a la recta െ૜࢞ െ ૛࢟ ൅ ૠ ൌ ૙.
a) Un vector perpendicular a (-3, 4) és (4, 3).
Equació vectorial → (x, y) = (0, -3) + t(4, 3)
‫ ݔ‬ൌ Ͷ‫ݐ‬
Equació paramètriques → ൜
‫ ݕ‬ൌ െ͵ ൅ ͵‫ݐ‬
b) Calculem el vector director de la recta.
ሬሬሬሬሬԦ ൌ ሺെʹǡ ͵ሻ
A(1, 2), B(-1, 5) → ‫ܤܣ‬
Un vector perpendicular a (-2, 3) és (-3, -2)
Equació vectorial → (x, y) = (-5, 0) + t(-3, -2)
‫ ݔ‬ൌ െͷ െ ͵‫ݐ‬
Equació paramètriques → ൜
‫ ݕ‬ൌ െʹ‫ݐ‬
99. Determina l’equació contínua de la recta que compleix les condicions següents:
ି࢞ା૟
a) Passa pel punt (7, -1) i és perpendicular a la recta ࢟ ൌ .

b) Passa pel punt (-4, 4) i és perpendicular a la recta െ૛࢞ ൅ ࢟ ൅ ૠ ൌ ૙.

399
 
 
 
 
 
 

Geometria analítica
Geometria analítica 7
 
 
 
 
 
 

a) Calculem el vector director de la recta:


������ � �3� �1�
��
��� ���

� �

Un vector perpendicular a (3,-1) és (1,3).

Expressem la recta en forma contínua:

��� ��1

1 3

b) Calculem el vector director de la recta:

������ � �1� ��
A(0, -7), B (1, -5) → ��
Un vector perpendicular a (1, 2 ) és (-2, 1).
Expressem la recta en forma contínua:
��� ���

�� 1

��� ���
100. Determina el punt de la recta � que dista 2 unitats del punt P(-2, 2).
� ��
Expressem la recta en forma paramètrica:

� � 1 � ��

� � �1 � �

Els punts de la recta són de la forma:

�� �1 � ��� �1 � ��

Calculem els vectors que van de la recta al punt P:

��������
� � � � ��3 � ��� 3 � ��

Veiem quins d’aquests vectors tenen mòdul 2.

� � ���3 � ���� � �3 � ��� � � � � � 1�� � �� � � � � �� � � �

���√��
�� �
� �
� �� � 1�� � 1� � � � �
���√��
�� �

Per tant, els punts són:

�����√�� ��√�� �����√�� ��√��


�� �1 � � �1 � � �� �1 � � �1 � �
� � � �

101. Estudia la posició relativa que tenen aquestes rectes:


� � � � �� � � �� � ��
a) r :� s :�
��� � � � � ��
� � � � �� � � � � ��
b) r :� s :�
� � �� � �� � � ��
400
 
 
 
 
 
 

Geometría analítica 77
 
 
 
 
 
 
 
a) u r   3,1 i u
s  6, 2  .
Són vectors paral·lels i, a més a més, les rectes coincideixen en un punt, per exemple  2, 0  , llavors
són rectes coincidents.
 
b) u r   6, 2  i us   3, 1 .
Com que els vectors són proporcionals, les rectes són paral·leles o coincidents.

Determinem si el punt �� �1� �3�, de la recta r, pertany a la recta s.

1
1 � 2 � 3� ���
� �� 3
�3 � ��
��3

Les rectes són paral·leles.

102. Determina quina posició relativa tenen aquestes rectes:


��� ��� � ���
a) �� � �� �
� �� � ��
��� ��� ��� ���
b) �� � �� �
� �� � ��
 
a) ur  3, 2  us  2, 4  .
3 2
  Són rectes secants.
2 4
 
b) ur  2, 3 us  2, 3 .
Com que els vectors són iguals, les rectes són paral·leles o coincidents.

Determinem si el punt �� ��1� 2�, de la recta r, pertany a la recta s.

�1 � 1 �2 � 3

2 �3

Les rectes són paral·leles.

 
103. Investiga la posició relativa que tenen els parells de rectes següents:
a) r : 2 x – 6y + 4 = 0 s: –x + 3y – 2 = 0
b) r : 6x – 4y + 11 = 0 s: –9x + 6y – 1 = 0
c) r : 4x – y + 1 = 0 s: 2 x – 3y + 13 = 0
2 6 4
a)     Les rectes són coincidents. 
1 3 2
9 6 1
b)      Les rectes són paral∙leles. 
6 4 11

401
 
 
 
 
 
 

Geometria analítica
Geometria analítica 7
 
 
 
 
 
 

4 1
c)     Les rectes són secants. 
2 3
 
104. Assenyala la posició relativa d’aquestes rectes:
���� �����
a) �� � � �� � �
� ��
����� ���
b) �� � � �� � �
� ��
� �
a) El pendent de la recta r és: �� � �
� �

El pendent de la recta s és: �� �

Com que els pendents són iguals , les rectes són paral·leles o coincidents.
Determinem si el punt �� ��� ���, de la recta s, pertany a la recta r.
���
�� � → Les rectes són paral·leles.

3
b) El pendent de la recta r és: mr  
2
1
El pendent de la recta s és: ms  
3
Com que els pendents són diferents, són rectes secants.
 
105. En quina posició relativa es troben aquests parells de rectes?
� � �� � 4�
a) �� � �� � � �� � � � �
� � � � ��

b) �� � � � � �� �� � �� � 4 � �

� � 4 � ��
c) �� � � �� � � �� �
� � ���
���� ��� ���
d) �� � � �� �
�� � ��
a) Expressem la recta r en forma general:
��� ���
� → ��� � � � 4� � �� → ��� � 4� � �� � �
4 ��

Les rectes són paral·leles

b) Expressem la recta r en forma general:


r : x –2y  6 0
Les rectes són paral·leles.
c) Expressem les rectes en forma general:
r : 3x – y –1  0 s : 2x – 3y – 8  0
Les rectes són secants.
d) Expressem les rectes en forma general:

402
 
 
 
 
 
 

Geometría analítica 77
 
 
 
 
 
 

r : –2
y 5x  1 s : –5x  15
 2y  14
5x  2y  1 0 5x  2y  1 0
Les rectes són coincidents.

106. Determina la posició relativa de la recta r que passa pels punts (–1, 2) i (3, 4), i la recta s que
queda definida pels punts (–3, 4) i (2, –1).
Describim les rectes en forma general:
r : x  2y  5 0 s : x  y 1 0
Les rectes són secants.
 
107. Quina posició relativa mantenen els parells de rectes següents?
a) r passa per (–3, 4) i per (8, –1). b) r passa per (–1, 4) i per (2, –5).
��� ���
s: x – 2y + 15 = 0 �� �
�� �
� � �� � �� → ���� �� → � � ��� � �
a) �
� � �� � �� → ��� ��� → �� � �� � �
� ��
� � � � � → �� � ��� � �� � � → Les rectes són secants.
�� ��
 
b) ur  3, 9  us   1,3 .

Com que els vectors són proporcionals, les rectes són paral·leles o coincidents.
Determinem si el punt �� ���� ��, de la recta r, pertany a la recta s.
���� ���
� → Les rectes són coincidents.
�� �
 
108. Assenyala en quina posició relativa es troben els parells de rectes següents:
a) r és perpendicular a 3x – 4y + 11 = 0.
� � ���
s:�
� � �� � ��
b) r és paral·lela a y = 2 x - 3.
��� �
�� �
� �
a) Determinem el vector director de la recta:

� � → ��� ��

Aquest vector és perpendicular al vector director de la recta s.


 
ur   4,3 us   6,8  .

Les rectes r i s són perpendiculars.


b) Sigui a   :

r : 2 x  y  a  mr  2 Si a 8  Coincidents
  Són paral·leles o coincidents: 
s : 2 x  y  8  ms  2  Si a  8  Paral∙leles
 

403
 
 
 
 
 
 

Geometria analítica
Geometria analítica 7
 
 
 
 
 
 

109. Determina si les rectes r i s es tallen. En cas afirmatiu, calcula’n el punt de tall.
a) r : 2 x + 3y – 3 = 0 s: 3x - 4y – 13 = 0
b) r : 6x + 9y – 3 = 0 s: 2 x + 3y – 1 = 0
c) r : x – 2y + 3 = 0 s: –3x + 6y – 8 = 0
2 3
a)   Es tallen
3 4
Resolem el sistema d’equacions:
2x  3y  3  0 
 x3, y 1
3x  4 y  13 0

6 9 3
b)    Són rectes coincidents, es tallen en tots els punts.
2 3 1
1 2 3
c)    No es tallen, són rectes paral·leles.
3 6 8

110. Indica, si és possible, els punts de tall dels parells de rectes següents:
࢞ൌ૜െࣅ ࢞ ൌ െ૚ െ ૝ࣆ
a) ࢘ǣ ൜ ࢙ǣ ൜
࢟ ൌ െ૚ ൅ ૛ࣅ ࢟ ൌ ૞ ൅ ૜ࣆ
࢞ ൌ ૠ െ ૛ࣅ ࢞ ൌ ૞ െ ૝ࣆ
b) ࢘ǣ ൜ ࢙ǣ ൜
࢟ ൌ ૜ࣅ ࢟ ൌ ૜ െ ૟ࣆ
࢞ ൌ ૚ െ ૛ࣅ ࢞ ൌ െ૜ ൅ ࣆ
c) ࢘ǣ ൜ ࢙ǣ ൜
࢟ ൌ െ૝ ൅ ૝ࣅ ࢟ ൌ െ૚ ൅ ૛ࣆ
 2
3   1  4   4(  1)    5
a)   1  8(  1)  5  3  
1  2  5  3    12
 5

 3 19 
El punt d’intersecció és per tant,  ,  .
5 5 
7  2 5  4    2  1
b)     1   0
3 3 6    2  1

El punt d’intersecció és per tant,  5,3 .

 11

1  2 3    2   4    4  2  8
c)     4  2(4  2 ) 3  
4  4 1  2  4  2 3   5
 4

 7 3
El punt d’intersecció és per tant,   ,  .
 4 2
 
111. Calcula els punts de tall, si és possible, dels parells de rectes següents:
࢞ ൌ ૛ ൅ ૜࢚
a) ࢘ǣ ૛‫ ܠ‬െ ‫ ܡ‬൅ ૡ ൌ ૙ ࢙ǣ ൜
࢟ൌૠ൅࢚
ି૛࢞ାૠ ࢞ି૛ ࢟ି૚
b) ࢘ǣ ‫ ܡ‬ൌ ࢙ǣ ൌ
૜ ૜ ି૛

404
 
 
 
 
 
 

Geometría analítica 77
 
 
 
 
 
 
��� ���
c) �� � �� �� � � � � � �
� ��
� � �� � �� ��� ���
d) �� � �� �
� � � � �� � ��

���� �����
e) �� � � �� � �
� � � � ��

a) 2(2 + 3t) - (7 + t) + 8 = 0 → 4 + 6t – 7- t + 8 = 0 → t = -1
El punt de tall és P(-1, 6).
3� � ��� � � �� � 3� � �
b) � →�
��� � � � 3� � 3 ��� � 3� � ��
Hi ha infinits punts de tall, i les rectes són coincidents.

��� � � � �� � � ��� � �� � � 3� � � � �
c) � →� →�
3� � � � � � � 3� � � � �� 3� � � � ��
No hi ha punts de tall, i les rectes són paral·leles.
������� ������
d) � → ��� � �� � �� � �� → � � ��
� ��
El punt de tall és P(2, -3).
� ������
e) � 3� � → 3 � �� � �� � �
� �
No té solució, les rectes són paral·leles.

112. Calcula els punts de tall d’aquestes rectes:


��� ���
a) �� � � �� � � � � �� �
� ��
� � � � ��
b) �� � �� � � �� � � � �
�����
�����
c) �� � �� �� � � � � � �
� � � � ��
x  3 y 1
a) s :   x  3  2 y  2  x  2 y  5  0
2 1
Són rectes coincidents, tots els seus punts són de tall.
b)  2  3t   3  5  t   2  0  2  3t  15  3t  2  0  No es tallen.
 8
x
6  7
c) 2  2  t    3  5t   5  0  4  2t  3  5t  5  0  t    
7  51
y
 7
��� ��� �����
113. Calcula el valor que ha de prendre k perquè la recta � sigui paral·lela a � .
� � � � � � ��
Per tal que les rectes siguin paral·leles, els vectors directors han de ser proporcionals.
� 3 3
� →���
�� 5 5

 
���
114. Troba el valor de a perquè la recta ax + 3y – 7 = 0 sigui paral·lela a � � � �.

405
Geometria analítica
Geometria analítica 7

Perquè les rectes siguin paral·leles, els vectors directors han de ser proporcionals.
�3 � 3
� ����
5 1 5

115. Calcula el valor de k perquè les rectes r i s siguin paral·leles.


r : kx + (k + 1)y + 8 = 0 s: 5x + 6y - 12 = 0
k k 1 8
   6k  5k  5  k  5  
5 6 12

116. Determina el valor de k perquè les rectes r i s siguin perpendiculars.


r : k2x + (k + 1)y + 8 = 0 s: 16x – 9y – 5 = 0

 a, b   t  b, a   o també   a, b   t  b, a 
k 2  9t  k 2 k 1 9  657
  t   k
k  1 16t  9 16 32

117. Prova que totes les rectes amb l’equació del tipus y =ax + a passen pel mateix punt. Troba el
� � �� � ��
punt i la recta d’aquest tipus que és paral·lela a �
� � �� � ��
Determinem el punt de tall:
� � �� � �

� � �� � �
Totes les rectes passen per (-1, 0).
Com que la recta és paral·lela a la recta donada, el seu vector director és (-3,2).
2
���
3
� �
La recta és: � � � � �
� �

118. Calcula la distància entre el punt P i la recta r en els casos següents:


a) P(0, 0) r : x – 4y + 1 = 0
b) P(2, 1) r : 2 x + 3y – 1 = 0
c) P(–3, –1) r : 3x – 2y – 3 = 0

1 0  4  0  1 17
a) d  P, r 
 
17 17
2  2  3 1  1 6 13
b) d  P, r 
 
13 13
3   3  2   1  3 10 13
c) d  P, r 
 
13 13

406
 
 
 
 
 
 

Geometría analítica 77
 
 
 
 
 
 

119. Calcula la distància del punt P (4, –2) a les rectes.


��� ���
a) –6x + 8y – 5 = 0 c) �
� �
����� ����
b) �
� � � � ��
d) ��

|��� ���� ��| |�������������| �� �
a) ���� �� � � � � �
√�� ��� ������ ��� �� �

b) Calculem l’equació general de la recta: 
��2
�� � 2 � � 2� � � � � � 0
2
|��� � ��� � �| |2 � � � 1 � ��2� � �|
���� �� � � � 0 � 
√�� � �� √2� � 1�
c) Determinem l’equació general de la recta: 

6x – 6 = 3y – 6 → 6x - 3y = 0 → 2x – y = 0
|��� � ��� � �| |2 � � � 1 � ��2�| 10
���� �� � � � � 2√5 � 
√�� � �� �2� � ��1�� √5
d) Determinem l’equació general de la recta: 

3y = 4x – 5 → -4x + 3y + 5 = 0
|��� � ��� � �| |�� � � � � � ��2� � 5| 17
���� �� � � � � 
√�� � �� ������ � �� 5

120. Determina la distància entre la recta que passa pels punts A(–2, 4) i B(4, –2), i el punt P(–3, 2).
La recta en forma general és: 
r:x y20 

1  3  1 2  2 3 2
d  P, r  
2 2  
   

407
 
 
 
 
 
 

Geometria analítica
Geometria analítica 7
 
 
 
 
 
 

121. Calcula la distància de l’orígen de coordenades a la recta que passa per (-3, 6) i és paral·lela a
��� ���

�� �
��� ���
Determinem la recta demanada: �
�� �
Calculem l’equació general de la recta:
8x + 24 = -6y +36 → 8x + 6y – 12 = 0

|��� ���� ��| |����������| �� �


���� �� � � � � �
√�� ��� √�� ��� �� �
 
122. Assenyala la distància entre les rectes paral·leles següents:
a) r : 4x – 3y + 1 = 0 s: 8x – 6y – 5 = 0
b) r : x + 2y – 5 = 0 s: 4x + 8y + 2 = 0
c) r : 5x – y – 16 = 0 s: 5x – y + 16 = 0
a) Un punt de la recta r és P(2, 3).
8 2  63  5 7
d  P, s 
 
10 10
b) Un punt de la recta r és P(–1, 3).
4   1  8  3  2 11 6
d  P, s  
96 12

c) Un punt de la recta r és P(3, –1).


5  3  1   1  16 16 26
d  P, s    
26 13

 
123. Quina distància hi ha entre les rectes r i s?
� � �� � ��
a) �� � �� � � �� � � � �
�����
��� ��� ����
b) �� � �� � �
� � �
� � � � �� ����
c) �� � �� � �
� � �� � � �

a) ����� ������ � ��2� � 1�. Els vectors són proporcionals. Un punt de r és P (-1,3)
�� � �2� 1�, �
12√5
���� �� �
5
b) ����� ������ � �����. Els vectors no són proporcionals. Per tant, les rectes són secants.
�� � �2� ��, �
c) ����� �� � ��� �1�. Com que els vectors són iguals, les rectes són paral·leles o coincidents.
�� � ��� �1�, �����
����
Prenem un punt de la recta r, A(2,-1), i veiem si pertany a �1 � .

Les rectes són coincidents.
   

408
 
 
 
 
 
 

Geometría analítica 77
 
 
 
 
 
 

124. Calcula el valor de a perquè la distància entre el punt A(2, a) i la recta r : 13x – 12y + 2 = 0 sigui
de 3 unitats.

13  2  12  a  2
d  A, r 
  3  28  12a  3 313
313  
▪  Si  28 12a  0 : 

28  3 313
28  12
 a 3 313 
 a
12  
▪  Si  28 12a  0 : 

3 313  28
28  12
 a 3 313 
 a
12  
 
125. Indica el valor de b perquè la recta i el punt es trobin a 5 unitats de distància.
��� ���
�� � �������������������������������������������������� ��
� �

Expressem la recta en forma general:


3x + 3 = by + 3b → 3x – by + 3 - 3b = 0
|3 � ��4� � � � � � 3 � 3�|
�� → 5�� � � � � |�� � 4�|
�3� � �����
�� � � ��� � �44 � � → � � 4
 
126. Calcula el valor de k perquè la distància entre les rectes r : 3x + 4y + k = 0 i s: 6x – 8y – 10 = 0
sigui 4.
Un punt de la recta s és P(–5,–5).

3   5   4   5   k
d  P, r   4  35  k  20
5
▪ Si 35  k  0 :
35  k  20  k  55
▪ Si 35  k  0 :
35  k  20  k  15
 
��� ���
127. Determina l’equació d’una recta que sigui paral·lela a � i que s’hi trobi a 8 unitats de
�� �
distància.
La recta té aquesta equació general.
4x + 3y + C = 0
|4 � ���� � 3 � 5 � �|
�� → 4� � |�� � �|
√4� � 3�

409
 
 
 
 
 
 

Geometria analítica
Geometria analítica 7
 
 
 
 
 
 

C = 29, C = -51
Les següents rectes compleixen les condicions indicades.
4x + 3y + 29 = 0
4x + 3y – 51 = 0
 
128. Assenyala l’angle que formen les rectes r i s.
� � � � �� � � ��
a) �� � �� �
� � �� � �� � � � � ��
����
b) �� � � �� � � �� � �
��
� ��� ���� �
c) �� � �� �
� � �� �

d) �� ��� � �� � � � � �� ���� � �� � � � �
a) �
������ � �3, 2�, �
������ � �2, 3�.

|�� � �� � �� � �� | |3 � 2 � 2 � 3| 12
��� � � � �
���� � ��� � ���� � ��� √3� � 2� � √2� � 3� 13

� � 22�3�� 11,51��

b) �
������ � �1, 3�, �
������ � ��4, 8�.

|�� � �� � �� � �� | |1 � ��4� � 3 � 8| 20 √2
��� � � � � �
���� � ��� � ���� � ��� √1� � 3� � ���4�� � 8� √800 2

� � 45�
 
c) ur   2, 6  us   1, 4 

2   1  6  4 11
cos
      32 o 28´ 16,3´´
40 17 170

d) �
������ � �4, 20�, �
������ � �3, 15�.

|�� � �� � �� � �� | |4 � 3 � 20 � 15| 312


��� � � � � �1
���� � ��� � ���� � ��� √4� � 20� � √3� � 15� 312

� � 0�

129. Quin angle formen les rectes següents?


��� ���
a) �� � �� � � ��� � �
�� �
�����
b) �� � � �� � � �� �
� � � � ��
���
c) �� �� � � � � �� �� � �� � � � �
��� �
d) �� �� � � � � � � �� �
� ��

410
 
 
 
 
 
 

Geometría analítica 77
 
 
 
 
 

 
 

a) ur   2,3 , us 1, 5


2  15 17 2

cos   22o 22´ 48´´

13 26 26
 
b) ur  1,3 , us   1,3

1  9 4
cos       36o 52´ 11,6´´
10 10 5
 
c) ur  (1,1) , us (2, 2)

1·2  1·(  2)
cos   0    90
1  22 12  (2) 2
2

 
d) ur  (1,3) , u
s (2, 4)

1·2  3·(  4) 10 2
cos       45
1  32 22  (4)2
2
200 2

��� ���
130. Calcula l’angle que formen entre elles les rectes r : y = x + 1, s: x + 4y + 4 = 0 i t: �
� ��

r : x  y 1 0
s : x  4y  4 0
t  3 x  2 y  12  0

cos  r  11  1 4
, s
3
  59o
2 17 34

cos  r  11  1 2
, t
1
  78, 7 o
2 13 26
1 3  4  2 11
cos  st
,   42,3o

17 13 221
 
��� ���
131. Troba l’angle que formen la recta que passa pels punts P(-1, 4) i Q(3, 8) i la recta �
� �
������
�� � �4,4�, �
������ � �2,�� 
|�� � �� � �� � �� | |4 � 2 � 4 � �| 40
��� �� � �  
���� � ��� � ���� � ��� √4� � 4� � √2� � �� √2176
� � �0��7�4�,�2��
 
132. Determina la mida dels angles que formen els parells de rectes r i s següents:
����
a) �� � �

s passa per (-1, 6) i és paral·lela a 4x + 2y + 7 = 0.

411
 
 
 
 
 
 

Geometria analítica
Geometria analítica 7
 
 
 
 
 
 

b) �� � � � �

��� ���
s passa per (3, 8) i és paral·lela a � .
� �
c) �� �� � �� � � � �
�����
s és perpendicular a � i passa per (3, -2).
� � �� � ��
a) �
������ � �2, 4�, �
������ � �2, �4�.
|�� ��� ��� ��� | |����������| �� �
��� � � � � �
���� ���� ����� ���� ��� ������ �√�� ��� �� �

� � �3� 7� 48��
b) �
������ � �1, 2�, �
������ � �2, 4�.
|�� ��� ��� ��� | |�������| ��
��� � � � � �1
√�� ��� �√�� ��� ��
���� ���� ����� ����

� � ��
c) �����
�� � �2, 8�,
El vector �
������ ha de ser perpendicular a (-1, 3); per tant pot ser �
�� � �3, 1�
|�� ��� ��� ��� | |�������| �� �√���
��� � � � � �
√�� ��� �√�� ��� √��� ���
���� ���� ����� ����

� � �7� 31� 43,71��

133. Calcula el valor que ha de tenir b perquè la recta 3x + by + 6 = 0 formi un angle de 60º amb la
���
recta � �
��
.
������ � ��3, 1�, �
� ������ � ��, �3�.
|�� � �� � �� � �� | 1 |�3 � � � 1 � ��3�|
��� � � → �
���� � ��� � ���� � ��� 2 ���3�� � 1� � �� � � ��3��

�18 � 1�√3 �18 � 1�√3


→�� ,� �
13 13
 
134. Assenyala el valor de k perquè les rectes r i s formin un angle de 45º.
�����
�� � �� �� � �� � � � �
� � �� � ��
r : 2x  y  7 0

2 2k  2 1
cos 45o    k1 , k2 3
2 k2  4 5 3

412
 
 
 
 
 
 

Geometría analítica 77
 
 
 
 
 
 

135. Escriu les equacions en la forma general de les rectes que configuren el triangle següent:

 
5
La recta r passa pels punts  3,1 i  3, :
 2

ur   4,1 r : x  4y  7 0

 5  5
La recta s passa pels punts  3,  i  6,   :
 2  4

us  4, 5 s : 5 x  4 y  25 
0

 5
La recta t passa pels punts  6,   i  3,1 :
 4

ut   4,1 t : x  4 y 1 0
 
136. Determina la recta paral·lela a r que passa pel punt simètric de P respecte de la recta r.
a) �� �� � �� � �� � �����������, ���
� � ���
b) �� � ���������������������������������, ��
�����
����
c) �� � � �������������������������������������, ���

a) Calculem la recta s, perpendicular a r, i que passa per P:
��� ���
�� �
�2 3
Determinem el punt de tall de les rectes:
2� � 3� � 1� � � 2� � 3� � �1� � � �2
� �� ��
3� � 12 � �2� � 1� 3� � 2� � �2 ��2
� � � �� � � � � ��
��2, 2� � � , ���
2 2 � � 11
� � �� � 3�
La recta és �
� � 11 � 2�
b) Calculem la recta s, perpendicular a r, i que passa per P:
��� ���
�� �
1 �2
Determinem el punt de tall de les rectes:
�2� � � 2 � � � �
� ���2
1 �2
(-4,0) és el punt mitjà de P i el seu simètric,P’, respecte de r.
Calculem P’(x, y):

413
 
 
 
 
 
 

Geometria analítica
Geometria analítica 7
 
 
 
 
 
 
4�� 4�� � � ��2
��4, 0� � � , ���
2 2 � � �4
� � ��2 � 2�
La recta és: �
� � �4 � �
c) Calculem la recta s, perpendicular a r, i que passa per P:
��� ���
�� �
�3 4
Determinem el punt de tall de les rectes:
4� � 3� � � �3� � 4� � �� � � ��
� �� ��
4� � 20 � �3� � 2� 4� � 3� � �� � � ��
(-1, -1) és el punt mitjà de P i el seu simètric, P’, respecte de r.
Calculem P’ (x,y):
� � � �� � � � � ��
���, ��� � � , ���
2 2 ���
� � �� � 4�
La recta és: �
� ���3

137. Assenyala l’equació de la recta simètrica de r respecte de la recta s.


���
a) �� � � �� �� � 2� � 4 � 0

� �2��
b) �� � �� 2� � � � � � 0
� � 3 � 2�
a) �����
�� � �2, ��, �
������ � �2, ��.

Les rectes són paral·leles o coincidents.

Escollim un punt, P, de r i calculem la distància fins a s:

|��� ���� ��| |��������|


��4,0��������������, �� � � �0
√�� ��� ������ ���

Les rectes són coincidents i, per tant, la recta simètrica és la mateixa.

b) �����
�� � ���, 2�, �
������ � ��, �2�.

Com que els vectors són proporcionals, les rectes són paral·leles o coincidents.

Escollim un punt, P, de r i calculem la distància fins s:

|��� ���� ��| |���������|


��2,3��������������, �� � � �0
√�� ��� √�� ���

Les rectes són coincidents i, per tant, la recta simètrica és la mateixa.

138. Els punts P (3, 3) i Q (6, –1) són simètrics respecte d’una recta. Troba l’equació general
d’aquesta recta.

PQ  3, 4 és el vector normal de la recta.

414
 
 
 
 
 
 

Geometría analítica 77
 
 
 
 
 
 

 3  6 1  3   9 
M ,    ,1
 2 2  2 

19
r : 3x  4 y  0  6 x  8 y  19 0
2
 
139. Comprova si els punts P(–1, 4), Q(3, 1) i R(11, –5) estan alineats. En cas afirmatiu, escriu
l’equació de la recta que els inclou.
Calculem els vectors formats pels punts:
ሬሬሬሬሬԦ
ܲܳ ൌ ሺͶǡ െ͵ሻ
ሬሬሬሬሬԦ ൌ ሺͳʹǡ െͻሻ
ܴܲ
Com que els vectors són proporcionals, els punts són alineats.
Calculem l’equació de la recta que els conté:
‫ ݔ‬ൌ െͳ ൅ Ͷ‫ݐ‬
‫ݎ‬ǣ ൜
‫ ݕ‬ൌ Ͷ െ ͵‫ݐ‬
 
140. Demostra que els punts A(3, –2), B(9, 6) i C(10, 5) són els vèrtexs d’un triangle rectangle.
 
AC   7, 7  BC 1, 1
 
Són ortogonals: AC  BC  7 1  7   1  0

 
141. Calcula els valors següents per a aquest triangle:

 
a) La longitud del segment AC
b) L’equació de la recta que passa per A i per C.
c) L’àrea del triangle ABC.
A(1,3)   
 1
2
a)   AC  C  A  (1, 7)  AC   (7)2  50  5 2
C (0, 4) 

b) La recta passa pel punt A i té vector director AC . Per tant, la seva equació és rAC : 7 x  y 4.

c) Per calcular l’àrea és necessari conèixer la base, b, i l’altura, h, del triangle:

415
 
 
 
 
 
 

Geometria analítica
Geometria analítica 7
 
 
 
 
 
 

rAC 
A
3

rh  B
D

1 X


 

  5 2.
b AC

La recta que defineix h compleix:


  
vh  AC vh  (7, 1) x  2 y 1
rh :   rh :     rh : x  7 y  5
 B( 2,1)  B(2,1) 7 1

El punt d’intersecció de rh amb la recta rAC , permet obtenir la longitud de l’altura, h:

7x  y 4  33 31    133 19  19 2


  D  ,   h BD  ,  
x  7y 5  50 50   50 50  10

19 2
5 2·
b ·h 10 19 2
Per tant, Atriángulo
   u .
2 2 2
 
142. Calcula l’àrea del triangle format quan unim els punts mitjans del triangle que té com a vèrtexs
els punts A (5, 3), B (–3, 4) i C (0, –3).
Primer, es determinen els vèrtexs del triangle del que es
cerca l’àrea:

 53 3 4   7 
▪ 
Punt mitjà de AB : P  ,  1, 
 2 2   2

 5 0 33  5 
▪ de AC : Q 
Punt mitjà ,   ,0
 2 2  2 

 3  0 4  3   3 1 
▪ 
Punt mitjà de BC : R  ,   , 
 2 2   2 2

Prenem com a base el segment R Q , és a dir:

  1
2
65
b
 RQ (4) 2      4, 03
 2 2

   1
  4,  
 RQ
  2
rRQ :  2 x  16 y 
5
Q   5 , 0 
  
2 

416
 
 
 
 
 
 

Geometría analítica 77
 
 
 
 
 
 

La recta que definirà h complirà que:

     1  7
vh  RQ vh   2 , 4  y
    x 1 2  16 x  2 y 9
rh :   7  , és a dir, rh :   
 P  1,  7
 P 1,  1 4
  2   2  2

El punt de intersecció de rh amb rRQ permet obtenir la longitud de l’altura:

2 x  16 y 
5  31 337    99 396 
  E ,   h EP  ,   6,14
16 x  2 y 
9  130 130   130 65 

b ·h 4,03·6,14
Per tant, Atriángulo
   12,3721 u 2 u2.
2 2
 
143. La recta que passa per M (2, 3) i és paral·lela a la recta r : y = 3x + 1 determina un triangle amb
els eixos de coordenades. Calcula’n l’àrea.
Primer calculem l’equació de la recta s:

s  r vs  (1,3)
s:   3x  y 3
 M  s  M (2,3)
A continuació, s’obtenen els punts d’intersecció de la recta s amb els
eixos de coordenades:

x0  y0 
  B(0, 3)   C (1,0)
3x  y 
3 3x  y 
3

El triangle está format pels punts O, B, C. Com que són punts que es troben sobre els eixos de
 
coordenades, els vectors OC i OB són perpendiculars. Llavors:
 
OC  12  02  1 OB 02  (3)
2
3

I finalment s’obté l’àrea:


 
OC · OB 1·3 3
Atriángulo
   u2  
2 2 2
 
144. Els punts A(2, 2) i B(–10, –2) són els vèrtexs que corresponen al costat desigual d’un triangle
࢞ ൌ ૚ െ ૟࢚
isòsceles. L’altre costat es troba sobre la recta ൜ . Determina el triangle i calcula’n
࢟ ൌ ૚ ൅ ૛࢚
l’àrea.
Igualem el mòdul dels vectors que van de la recta fins als punts A i B.

ඥሺͳ െ ͸‫ ݐ‬െ ʹሻଶ ൅ሺͳ ൅ ʹ‫ ݐ‬െ ʹሻଶ =ඥሺͳ െ ͸‫ ݐ‬൅ ͳͲሻଶ ൅ሺͳ ൅ ʹ‫ ݐ‬൅ ʹሻଶ

՜ ‫ ݐ‬ൌ ͳ ՜ ‫ܥ‬ሺെͷǡ ͵ሻ

ሬሬሬሬሬԦ ൌ ሺെͳʹǡ െͶሻǡ ‫ܥܣ‬


Determinem les tongituds dels costats: ‫ܤܣ‬ ሬሬሬሬሬԦ ൌ ሺെ͹ǡ ͳሻ݅‫ܥܤ‬
ሬሬሬሬሬԦ ൌ ሺͷǡͷሻ

417
 
 
 
 
 
 

Geometria analítica
Geometria analítica 7
 
 
 
 
 
 

������� � ���1��� � ����� =√160


���

������ � � ������ � 1� =√50


���

������ � � √5� � 5� =√50


���

Determinem l’altura sobre el costat diferent, utilitzant el teorema de Pitàgores:


√���
H=��√50�� � � � � √10��

��� √����√��
Per tant, l’àrea és: A= � � �0���
� �

145. Calcula l’àrea del quadrilàter que formen els punts (1, 3), (–2, 4), (–1, –3) i (3, –2).
Siguin A(1, 3), B(–2, 4), C(–1, –3), D(3, –2) els punts donats.

La representació gràfica mostra que el quadrilàter no és un paral·lelogram. Així doncs, per calcular
l’àrea l’hem de dividir en dos triangles mitjançant una de les seves diagonals:

La diagonal, rAC , té per equació:


 AC (2, 6)
rAC :  3x
y
 A(1,3)

▪ Triangle ABC:
 
AC·AB (2, 6)·(3,1) 6  6 0  Els costats AC i AB són perpendiculares. Llavors:

AC  4  36  40  2 10  2 10 · 10

 AABC
   10 u2.
AB  9  1  10  2

▪ Triangle ADC:
 
AD·CD  (2, 5)·(4,1)  8  5  0  Els costats AD i CD són perpendiculars. Llavors, hem de calcular
l’altura, h, relativa al costat del triangle que es triï com a base:

Base = AC  2 10

La recta que definirà l’altura, h , compleix que:


  
vh  AC v (6, 2)
rh :  , és a dir, rh :  h  x  3 y 3
 D  3, 2   D  3, 2 

El punt d’intersecció de rh amb rAC permet obtenir la longitud de l’altura:

y  3x   3 9    33 11  11 10
  E   ,    h ED  ,  
x  3y 
3   10 10   10 10  10

418
 
 
 
 
 
 

Geometría analítica 77
 
 
 
 
 
 

11 10
2 10 ·
Per
 tant, AADC  10 11 u2.
2
Així, AABCD  AABC  AADC  10  11  21 u 2 .

 
146. Determina el centre d’un paral·lelogram del qual coneixem tres vèrtexs: (5, –1), (9, 5) i (–1, –5).
Quantes solucions té aquest problema? Per què? Fes un dibuix en què es vegin totes les
solucions.
Assignem nom als punts indicats A(5, -1), B(9, 5) i C(-1,, -5)

������ � �4, 6�
��

Fem una translació amb orígen en C i vector (4, 6) , i obtenim un punt D, que forma un paral·lelogram:
D(3,1)

Calculem el centre del paral·lelogram, E:

��� ���
��, �� � � , � � ��4, ��
2 2

Fem una translació amb orígen en C i vector (-4, -6), i obtenim un punt D’, que forma un
paral·lelogram: D’(-5,-11)

Calculem el centre del paral·lelogram, E’:

� � � � � ��
��, �� � � , � � ��2, ���
2 2

������ � ���, ���


��

Fem una translació amb orígen en A i vector (10, 10), i obtenim un punt D’’=(15, 9)

Calculem el centre del paral·lelogram, E’’:

�� � � � � �
��, �� � � , � � �����, 2�
2 2

Aquest problema té tres solucions.

 
 
147. Calcula l’equació de les rectes paral·leles a la recta r: 10x – 3y + 4 = 0 que és a una distància 2
del punt P(3, 1).

419
 
 
 
 
 
 

Geometria analítica
Geometria analítica 7
 
 
 
 
 
 

Únicament hi ha dues rectes que compleixin les característiques donades. Com que són paral·leles a
r, tenen el mateix vector director, és a dir són:

s :10 x  3 y  c 0 t :10 x  3 y  c ' 


0

Llavors, imposem la condició de la distància:

 27  c
 =2 si c  27
10·3  3·1  c 27  c  109 c 2 109  27
d  2  
9  100 109  27  c =2 si c  27 c '  2 109  27
 109

 
 
 
148. Troba l’equació de la recta que passa pel punt P(–3, 3) i que és a una distància de 2 unitats del
punt Q(0, 5).
Com que la recta passa pel punt P:
P ( 3,3)
ax  by  c  0   3a  3b  c  0

Es crea un triangle rectangle, amb vèrtexs P, Q i R, on R és la projecció del punt Q sobre la recta
cercada:

 
PQ  94  13 QR  2 ( xR , yR  5)  xR 2  ( yR  5)2  4 xR 2  ( yR  5)2

Pel teorema de Pitàgores:


 2 
13  4  PR  3  PR  ( xR  3) 2  ( yR  3) 2  9

Resolem el sistema, s’obtenen les coordenades del punt R:


x  0, yR 3  No vàlida
4  xR 2  ( yR  5) 2   R  24 75 

  24 75  R , 
9   xR 
( xR  3) 2  ( yR  3) 2   , yR   13 13 
 13 13

420
 
 
 
 
 
 

Geometría analítica 77
 
 
 
 
 
 

La recta que cerquem, és troba determinada pels punts P i R:

   15 36 
 PR  R  P   , 
r:  13 13   r : 36 x  15 y 
153
 P ( 3,3)

 
149. Assenyala l’equació de la recta que passa pel punt P(2, –5) i per la intersecció de les rectes
r : 5x – 2y + 1 = 0 i s: 4x – y + 7 = 0.

Primer s’obté el punt Q, intersecció de les rectes r i s:


5 x  2 y  1 0 y 4 x  7  13 31 
   5 x  2(4 x  7)  1  0  Q   ,  
4x  y  7  0  3 3

Per finalitzar es calcula l’equació de la recta que passa pels punts P i Q:

P (2, 5)  31
5 
 3  16  t :16 x  19 y  127  0  
 13 31    m  13 19
Q   ,  
 3 3  2
3
 

150. Calcula el valor de k perquè aquestes tres rectes es tallin en el mateix punt. Determina les
coordenades d’aquest punt.
2 x + 5y – 1 = 0 –x + 2y + k = 0 4x + 7y – 5 = 0
Posem nom a les rectes donades r, s, i t, respectivament:

r : 2 x  5 y  1 0
s :  x  2 y  k 0
t : 4x  7 y  5  0

Calculem el punt d’intersecció, P, de les rectes r i t, i fem que aquest punt pertanyi a la recta, s’obté k:

2 x  5 y
 1 0  4 x  10 y 
2 0 Reducción
    3 y  3 0  P(3, 1)
4 x  7 y
 5 0  4 x  7 y
5 0 

P (3, 1)  s
 x  2 y  k 0    3  2  k 0  k 5

421
 
 
 
 
 
 

Geometria analítica
Geometria analítica 7
 
 
 
 
 
 

151. Considera el triangle de vèrtexs A(1, 1), B(4, 6) i C(7, 2). Les rectes paral·leles per cada vèrtex
al costat oposat formen un triangle A'B'C'

.
Determina les coordenades d’aquests vèrtexs i comprova que els dos triangles són semblants
calculant-ne els angles respectius.
Primer es calculen les equacions de les rectes determinades pels punts A ' , B ' i C ' dos a dos:
 
v AB  v A ' B '  B  A  (3,5) 
  rA ' B ' : 5 x  3 y  29
C (7, 2) 
 
v AC  v A ' C '  C  A  (6,1) 
  rA ' C ' : x  6 y  32
B(4, 6) 
 
vBC  vB ' C '  C  B  (3, 4) 
  rB ' C ' : 4 x  3 y 7
A(1,1) 

A continuació, s’obtenen les coordenades de A ' , B ' i C ' amb les respectives interseccions de les
rectes anteriors:

5x  3 y 
29  x 6 y 32
A ' rA ' B '  rA ' C ' 
    5(6 y  32)  3 y  29  A '(10, 7)
x  6y 32

5x  3 y 
29 Reducció
B ' rA ' B '  rB ' C ' 
    B '(4, 3)
4x  3 y 
7 

x  6y  32  4 x  24 y 
128 Reducció
C '  rA ' C '  rB ' C '      C '(2,5)
4 x
 3 y 7  4 x  3y 7 
Per acabar, es comprova que els dos triángles són semblants:
   
v · vBC v A ' B ' · vB ' C ' -11 =
cos b = AB
  =   = cos b ' =  b = b ' = 67,834  ABC A ' B 'C '
v AB · vBC v A ' B ' · vB ' C ' 9 + 25 · 9 + 16

   


v AC · vBC v A ' C ' · vB ' C ' 14 =B 
cos g =   =   = cos g ' =  g = g ' = 62,592  BCA 'C ' A '
v AC · vBC v A ' C ' · vB ' C ' 36 + 1 · 9 + 16

   


v AB · v AC vA ' B ' · vA 'C ' 23  =C
cos a =   =   = cos a ' =  a = 49,574  CAB ' A' B '
v AB · v AC vA ' B ' · vA ' C ' 9 + 25 · 36 + 1

   

422
 
 
 
 
 
 

Geometría analítica 77
 
 
 
 
 
 

152. En un triangle ABC, un vèrtex és A(2, 5). El punt mitjà del costat BC és (3, 1) i el punt mitjà del
costat AB és (0, 4). Calcula els vèrtexs B i C del triangle.

 
A B (2,5)  (b1  b2 )
M AB
  (0, 4)
  B(2,3)  
2 2
BC (2,3)  (c1  c2 )
M BC   (3,1)  C (8, 1)  
2 2
 
153. Determina les coordenades d’un punt P sabent que pertany a la recta r : x – y + 1 = 0 i que dista
5 unitats de l’orígen de coordenades.
d ( P, O )  5
Sigui el punt P( x, y) . P ha de complir  . Per tant:
P  r

PO  a 2  b 2 5  a 2  b 2 25 a b 1 2 2  P1 (4, 3)
 (b  1)  b  25  
a  b 1 0   P2  (3, 4)

 
154. Calcula les coordenades dels punts de la recta r : 2 x + 3y + 4 = 0 que es troben a una distancia
de 2 unitats de la recta s: 3x + 4y – 6 = 0.
Anomenarem P (a, b) al punt o punts buscats:

▪ Per una banda, P  r  2a  3b  4 0.


3a  4b  6 3a  4b  6
▪ D’altra banda, imposem la condició de la distància: d ( P, s )    2
16  9 5
Resolem el sistema format per ambes equacions, s’obté P:
2a  3b  4 0

3a  4b  6  x64  y 44  P1 (64, 44)
 2
5 
2a  3b  4 0 

3a  4b  6   x 4  y 4  P2 (4, 4)
 2
5 

423
 
 
 
 
 
 

Geometria analítica
Geometria analítica 7
 
 
 
 
 
 

155. Un dels vèrtexs d’un paral·lelogram és l’orígen de coordenades i un altre vèrtex és el punt (3,
5). Troba les coordenades dels altres dos vèrtexs si un és a la recta d’equació x – 2y = 0 i l’altre
es troba a la recta d’equació 2 x + y = 0.

 
Sigui A(3,5) , r : x  2 y 
0 i s : 2x  y 0 el punt i les rectes conegudes.
Les rectes donades són perpendiculars, ja que el seu producte escalar és zero.
(2,1) · (1, 2) 2  2 0

Amb aquesta observació és fàcil calcular les equacions dels costats que manquen:
▪ Recta t:
Aquella paral·lela a s que passa per A:
  At
vs  (1, 2)  vt  (1, 2)  t : 2 x  y  11
▪ Recta z:
Aquella paral·lela a r que passa per A:
 
vr (2,1) 
 vz (2,1)  Az
 z : x  2y 
7
Llavors, els vèrtexs desconeguts del rectangle són els punts d’intersecció següents:
x  2y 
0  22 11  2x  y 0   7 14 
B  r t    B ,  C  sz    C , 
2x  y 
11  5 5 x  2y 7   5 5
156. Determina la projecció ortogonal del punt P (7, 4) sobre la recta x – 2y – 4 = 0.

 
Sigui r : x  2 y  4 0.

Per començar es calcula la recta, s, perpendicular a r que passa per P:


  Ps
vr  (2,1)  vs  (1, 2)    s : 2x  y 
18

Així, la projecció ortogonal del punt P sobre la recta r, és un altre punt, Q, que ve determinat per
l’intersecció entre r i s:

x  2y 4  x 2 y  4
   2(2 y  4)  y  18  y  2  x  8  Q(8, 2)
2x  y 
18

424
 
 
 
 
 
 

Geometría analítica 77
 
 
 
 
 
 

157. Troba un punt a l’eix d’abscisses que estigui a la mateixa distància del punt A(5, 4) que de la
��� ���
recta ��� � .
� �

Sigui P (a, 0) el punt que busquem. Llavors:

 3a  19 
2
3a  19
 
2
d ( P, A
) d ( P, r )  (5  a ) 2  4
2
 (5  a ) 2  4 2  
42  32 25

 x1  2
2 
 16a  364a  664 
0  83
 x2  4

És senzill comprovar que ambdues solucions són vàlides. Per tant, hi ha dos punts que són solució:
 83 
P1 (2,0) P2  , 0 
 4 

158. Un angle recte té el vèrtex en el punt A(3, 4) i l'equació de la seva bisectriu és 2 x – y – 2 = 0.


Troba les equacions dels costats.

 
La bisectriu té per vector director (1, 2).
Escrivim l’equació punt-pendent dels costats:
Y - 4 = m ( x – 3 ) → -mx + y – 4 + 3m = 0
Apliquem la fórmula de l’angle entre dues rectes:
|�� � �� � �� � �� | √2 |1 � 1 � 2 � �|
��� ��� � → �
���� � ��� � ���� � ��� 2 √1 � 2� � √1� � ��


→ � � ��� � �


Les equacions són: �� � � � 1� � � � �������

 
159. De totes les rectes que passen pel punt A(2, 3), calcula la recta que determina segments iguals
quan talla els eixos cartesians.

425
 
 
 
 
 
 

Geometria analítica
Geometria analítica 7
 
 
 
 
 
 

Les rectes que passen per A són de la forma


� � 3� � ���� � 2�
Aquestes rectes tallen als eixos en els punts:
�3 � 2�
��� 3 � 2�� � � ��

La distància a l’orígen (0,0) ha de ser sigual:
2
��3 � 2��2 � ���3�2�� → Hi ha tres solucions.

�� � ��� → � � �� � �
�� � �� → � � � � �
3 3
�� � � → � � �
2 2
 
 
160. Indica les coordenades del circumcentre del triangle de vèrtexs A(2, 3), B(4, –1) i C(5, 2), sabent
que el circumcentre equidista dels vèrtexs.

 
Per trobar les coordenades del circumcentre només cal calcular les equacions de dues de les
mediatrius del triangle i obtenir la seva intersecció:

▪ Mediatriu 1:
 
AC  (3, 1)  vm1  (1, 3) 

AC  7 5    m1 : 3x  y 
8
M1  M 1  , 
2  2 2 

▪ Mediatriu 2:
 
AB  (2, 4)  vm2  (4, 2) 

A B   m2 : x  2 y 
1
M 2  M 2 (3,1) 
2 

Així, el circumcentre és el punt CC


 m1  m2 :

3x  y 
8 x 2 y 1
  CC (3,1)
x  2y 
1

426
 
 
 
 
 
 

Geometría analítica 77
 
 
 
 
 
 

Obtenim la tercera mediatriu, i veiem si passa pel circumcentre, es comprova si l´hem calculat bé:

 3  3 6  És correcte.
CC (3,1)
m3 : x  3 y 6 

 
161. Determina l’equació de la recta que talla els eixos de coordenades en els punts (3, 0) i (0, 5).
࢞ ࢟
Confirma que aquesta equació es pot escriure en la forma ൅ ൌ ૚.
૜ ૞
Comprova que si una recta talla els eixos en els punts (a, 0) i (0, b), la seva equació es pot
࢞ ࢟
escriure en la forma ൅ ൌ ૚.
ࢇ ࢈
Aquesta manera d’escriure una recta s’anomena forma canònica o segmentària.
Siguin A(3, 0) i B (0,5) :

La recta r, que passa per (3,0) i (0,5) és:



vr  (3,0)  (0,5) (3, 5) 
  r : 5x  3 y 
15
A(3, 0) 

En general, si una recta talla als eixos en els punts P(a,0) i Q(0, b) , la seva equació és:

( a, b) 
PQ Q  P  xa y x y x y
     1  s:   1
P(a, 0)   a b a b a b

 
162. Donat el segment AB, en què A(–2, 1) i B(2, 3), construeix els possibles triangles equilàters en
els quals AB és un dels costats.

AB  (4, 2)
  rAB : x  2 y 
4
A(2,1) 

L’equació genèrica de les rectes cercades que passen per A és:


y  2 m( x  1)  mx  y  (2  m) 0

Així, apliquem la condició de l’angle, i s’obtenim les possibles pendents:


2
m2 1 
2
m2  2
 m1 8  5 3
cos 60        m  16m  11 0  
5 · 1 m 2
 2   5 · 1  m2   m2  8  5 3

427
 
 
 
 
 
 

Geometria analítica
Geometria analítica 7
 
 
 
 
 
 

Llavors, tenim que:

m1  8  5 3  rAC : (8  5 3) x  y  (10  5 3)  0



m2  8  5 3  rAD : (8  5 3) x  y  (10  5 3)  0

L’equació genèrica de les rectes que busquem, passen per les y  3 m( x  2)  mx  y  (3  2m) 0 :

 m  8  5 3  rBD : (8  5 3) x  y  ( 13  10 3)  0
Així, tenim  1
 m2  8  5 3  rCB : (8  5 3) x  y  ( 13  10 3)  0

Fent les corresponents interseccions s’obté els punts C i D, vèrtexs dels dos triangles possibles:

(8  5 3) x  y  (10  5 3) 0  3 3 1 17 3 
C  rAC  rCB    ...  C   ,  
(8  5 3) x  y  (13  10 3) 0  2 10 2 10 

(8  5 3) x  y  (10  5 3) 0  3 3 1 17 3 
D  rAD  rBD    ...  D   ,   
(8  5 3) x  y  (13  10 3) 0  2 10 2 10 

 
163. El centre d’un hexàgon regular és el punt A(6, –2) i un costat es troba sobre la recta d’equació
–4x + 3y + 5 = 0. Determina’n les coordenades dels vèrtexs i l’àrea.

Calculem la longitud de l’apotema:


|�4 � � � 3 � ���� � 5|
���� �� � �5
���4�� � 3�

� � ��√�
Determinem la longitud del costat: � � �5� � � � → � � �
� �

��√�
���� �� ��
Determinem l’àrea: � � � �
� 50√3��
� �

��√�
Calculem els punts sabent que la distància entre ells és � i que pertanyen a la recta donada:

� 4√3
10√3 � � � �� � √3 � �� �� � 1 � 3
� 3 � ��� � �� � ��� � �� →
�4� � 3� � 5 � 0 � 4√3
��� � �√3 � �� �� � 1 �
� 3

�√� �√�
� �√3 � ��1 � � � ��√3 � ��1 � �
� �

428
 
 
 
 
 
 

Geometría analítica 77
 
 
 
 
 
 

Altres dos vèrtexs són simètrics als vèrtexs calculats respecte de A.

�√�
√����� �� � �� �√�
��, �2� � � , � � � �10 � √3, �� � �
� � �

�√�
�√����� �� � �� �√�
��, �2� � � , � � � �10 � √3, �� � �
� � �

Per calcular la resta de vèrtexs tenim en consideració la longitud dels costats.


� 4√3 10√3
���10 � √3 � �� � ��� � 3 � �� � 3

� � �, � � �2
�� 8√3
� � � � 2√3 � �, � � �2
4√3 10√3 3
� ��√3 � 2 � ��� � �1 � � �� �
� 3 3


� 4√3 10√3
���10 � √3 � �� � ��� � 3 � �� � 3

� � �, � � �2
�� 8√3
� � � � �2√3 � �, � � � �2
4√3 10√3 3
� ���√3 � 2 � ��� � �1 � � �� �
� 3 3

�√� �√�
� �2√3 � �, � 2� � ��2√3 � �, � � 2�
� �

 
164. En Joan i la Betlem es miren l’un a l’altra a través d'un mirall situat segons la recta d’equació
y = –x + 2. La Betlem es troba en el punt (–9, –1), i en Joan en el (–4, 3). Quines coordenades té
el punt M on miren?

 
Determinem el vector director de la recta donada:
�� � �1, �1�
�����
Un vector perpendicular és �
������ � �1,1�.
La recta, perpendicular al mirall, que passa per on hi ha la Betlem és:
� � �� � �

� � �1 � �
Determinem el punt de tall de les dues rectes:
�1 � �� � � � �� � 2 � �� � � � � ��3,��
Determinem el punt simètric respecte del mirall, B’:
��� ��1
��3,�� � � , � � �� � �3,11�
2 2

429
 
 
 
 
 
 

Geometria analítica
Geometria analítica 7
 
 
 
 
 
 

Repetim el procés amb l’altre punt donat i obtenim J’:


� � �4 � �

� �3��
3 5 9
3�� �4���2�� � � �� , �
2 2 2
5 9 ��4 ��3
�� , � � � , � � ����1,��
2 2 2 2
Determinem la recta que passa per (-9,1) i per J’:
� � �9 � ��

� � �1 � ��
Calculem la recta que passa per (-4,3) i per B’:
� � �4 � ��

� � 3 � ��
Determinem el punt de tall de ler rectes.
�9 � �� � �4 � �� 4 1
� ��� ���
�1 � �� � 3 � �� 5 5
�� ��
El punt de tall és: �� , �
� �

165. La recta –2 x + y = –3 és la mediatriu d’un segment AB amb A(–2, 3). Quines són les
coordenades del punt B?
La recta perpendicular a la mediatriu que conté el punt A, és a dir, la recta que conté al segment AB
és:

vAB (2, 1) 
  r : x  2y 
4
A(2,3) 

x  2y  4 
El seu punt de tall amb la mediatriu és   C  2,1 , que és el punt mitjà del segment AB.
2 x  y 3

Llavors, les coordenades de B, punt simètric de A respecte de C són:


B  2 · 2  2, 2 · 1  3    6,  1 

430
 
 
 
 
 
 

Geometría analítica 77
 
 
 
 
 
 

AMPLIA 

166. Escull la resposta adequada.

□ La recta y  mx i passa per l’orígen de coordenades. Perquè divideixi el triangle donat en altres
dos de la mateixa superfície, el punt mitjà del costat oposat:
(1,1)  (6m, 0)  1  6m 1 
M   , 
2  2 2

 1
 m1  
1 1  6m  2 1
M  r : y mx  m ·  6m2  m  1 0    m1  m2 
2 2 m  1 6
 2 3
□ Sigui x la longitud del segment BD. Llavors:
B  x x x C
D E   
5
x2  (2 x)
2
k (3x)2  5x
2
9kx2  k
9
 
□ AB  (4, 4)  AB 16  16 4 2
 
AC  (2014, 4020)  AC  20142  40202 
4 496, 29
 
BC  (2010, 4016)  BC  20102  40162 
4490,92

▪ Base: Segment AC  b 4496, 29


▪ Altura:
Es calcula la recta perpendicular a la recta rAC ; després, el punt d’intersecció entre elles; i, per
acabar, la distància entre el punt obtingut i B:
rAC : 2 010 x  1007 y 
8040 .

(2 010, 1007) 
vh 

  rh :1007 x  2 010 y 
8040
B(0, 4) 

431
 
 
 
 
 
 

Geometria analítica
Geometria analítica 7
 
 
 
 
 
 

D  rAC  rh  1,595; 4, 799   h  BD  1, 784

b · h 4 496, 29 · 1, 784
Així, l’àrea seràA   4010,546
2 2
Per tant, la resposta més aproximada és 4010.
□ Primer es calcula l’angle entre les dues rectes donades:
3 1 5
cos    cos      63, 435
2 · 10 5
A continuación s’obté el punt de tall entre aquestes rectes:
y  x 
  4 x  4  P  1,1
y 3x  4

D’altra banda, l’equació de la recta reflectida, donada en la seva forma punt-pendent és:
1 m( x  1)  mx  y  m 
y  1 0

L’angle entre la recta reflectida i la recta y   x ha de ser l’obtingut abans:

 5
2 2  m1  3
 m 1  1 m 2  1  2m 2 

      3m  10 m  3  0   1
2  2
 5   2  2m  5 2  2m  m2  3

1
El pendent de la recta donada és 3, per tant, el pendent de la recta reflectida és .
3
1
Llavors, l’equació de la recta reflectida és y  1  ( x  1)  3 y  x  4
3
□ Repetim de forma general el procediment de l’apartat anterior, s’obté que l’equació de la recta
x b
reflectida és y  .
a a

432
 
 
 
 
 
 

Geometría analítica 77
 
 
 
 
 
 

�� i ���, perquè els seus mòduls coincideixin amb


167. Calcula l’angle que han de formar dos vectors �
el mòdul de la seva diferència, � �� . És a dir, |�
�� � � � ��| � �� �� � ���� .
��� � ��

�� � ��� ?
I perquè coincideixin amb el mòdul de la seva suma, �

 
�� � ��� ��

�� � ��� ��

�� � ��� � �� � �� � � ��

|�
��| � �� � � � �

��� � �� � � � �
��

��� � ���� � ��� � ��� � �� � ��� � �� � � � � � 2�� � � � � � � � 2��

Igualant, tenim que:

� � �� �
� � � � � � � � � � � � 2�� � � � � � � � 2�� → � �� � ��

Calculem l’angle que formen:

�� � �� 1
��� � � � � � � ���
�� � � �� � √� � � �� 2

Perquè els mòduls de dos vectors del mateix mòdul coincideixin amb la seva diferència han de formar
un angle de 60º.

��� � ���� � ��� � ��� � �� � ��� � �� � � � � � 2�� � � � � � � � 2��

Igualant, tenim que:

� � �� �
� � � � � � � � � � � � 2�� � � � � � � � 2�� → � �� � ��
��

Calculem l’angle que formen:

�� � �� 1
��� � � � � � � � 12��
�� � � �� � √� � � �� 2

Perquè els mòduls de dos vectors del mateix mòdul coincideixin amb la seva suma, han de formar un
angle de 120º.

   

433
 
 
 
 
 
 

Geometria analítica
Geometria analítica 7
 
 
 
 
 
 

168. Demostra que si dos vectors � �� tenen el mateix mòdul, llavors �


������ �� i �
�� � � � �� formen un angle
�� � � �
recte. Dedueix d’aquest resultat que les diagonals d’un rombe són perpendiculars.

 
Apliquem els resultats de l’activitat anterior:

�� � ��� ��

�� � ��� ��

|�
�� � ��| � ��� � ��� � �� � ��� � �� � � � � � ��� � � � � � � � ���

|�
�� � ��| � ��� � ��� � �� � ��� � �� � � � � � ��� � � � � � � � ���

�� � � � � � ��� � � � � � � � ��� � �� � � � � � ��� � � � � � � � ��� � � � �� � ��

�� � ��
��� � � � � � � � ���
�� � � � � � √� � � � �

Siguin �
������� els vectors dels costats del rombe, que tenen el mateix mòdul.

Les diagonals del rombe s’obtenen, respectivament, sumant i restant �


������� , per tant, les diagonals són
perpendiculars.

169. La recta d’Euler d’un triangle és la recta que conté l’ortocentre, el circumcentre i el baricentre
del triangle. S’anomena així en honor del matemàtic suís Leonhard Euler, que va descobrir
aquest fet a mitjan segle XVIII.

 
Determina la recta d’Euler en el triangle de vèrtexs A(–5, 6), B(–1, 4) i C (3, –2) i comprova que
tant el baricentre G, l’ortocentre H i el circumcentre es troben en aquesta recta i que verifiquen
la relació GH = 2GO.
Calculem el baricentre, G, i per això addicionem les coordenades dels punts i dividim per tres:G(-1,0).
Per trobar l’ortocentre, calculem una recta perpendicular al costat AB que passi per C:

434
 
 
 
 
 
 

Geometría analítica 77
 
 
 
 
 
 
��3 ��2

5 2
Determinem una recta perpendicular al costat AC i que passi per B:
��1 ��4

1 1
El punt de tall d’aquestes rectes és l’ortocentre:
1 10
� �� , � �
3 3
Per trobar el circumcentre, calculem la recta que passa pel punt mitjà del costat AB i és perpendicular
al mateix:
��3 ��1

5 2
Determinem la recta que passa pel punt mitjà del costat AC i és perpendicular al mateix:
��1 ��2

1 1
El punt de tall d’aquestes rectes és el circumcentre:
� �
� �� , �
� �
Calculem la recta que passa per GH:
��1 �
� � 5� � � � 5 � 0
1 �5
Com que O verifica les equacions de l’equació de la recta, els tres punts estan alineats.
Determinem la distància GH:


1 10 � √104
|��| � ��� � 1� � �� � �
3 3 3
Calculem la distància GO:

4 �
5 � √26
|��| � ��� � 1� � � � �
3 3 3
Per tant, es verifica la relació, ja que resulta que:
√104 2√26
|��| � � � 2|��|
3 3

435
 
 
 
 
 
 

Geometria analítica
Geometria analítica 7
 
 
 
 
 
 

170. Pels punts mitjans dels costats d’un triangle ABC tracem les mitjanes i unim els punts que
trisequen el tercer costat amb el vèrtex oposat. Així, a l’interior s’obté un llacet (dos triangle
units per un vèrtex).
Es pot calcular la fracció de superfície total del triangle que representa el llacet?

 
Per començar, se suposa que l’àrea del triangle donat és 1, és a dir, AABC  1 .

Les mitjanes divideixen al triangle en sis parts iguals, així:


1 1 1
AAMF
 AFNB
  2·
AAFB
6 6 3
Es tracen des de A, C, P i Q perpendiculars sobre la mitjana BM, on X, Y, Z i V són els seus
respectius peus.

Observant la figura, es tenen les següents relacions:

▪ AMY  CMX . Com que AM  MC  AY  XC  h .

2 1
▪ AYB  PZB  QVB . Com que AP  PQ  QB  PZ  h i QV  h .
3 3
2 2 2 3
▪ PEZ  XCE . Com que PZ XC  PE EC  PE PC i EC  PC .
3 3 5 5
1 1 1
▪ QHV  XCH . Com que QV  QC  QH  HC  QH  QC .
3 3 4
Fent el mateix procediment sobre la mitjana AN:
2 3 1
QG QC GB QC
 PD PC
5 5 4
Veiem la mitjana PC, i els punts D i E:

436
 
 
 
 
 
 

Geometría analítica 77
 
 
 
 
 
 

 5
 PD  20 PC

2 1 3  3
DE  PE  PD    PC  PC   DE  PC
5 4 20  20
 12
 EC  20 PC

Tracem la línia auxiliar AE, i s’observa que els triangles ACE, AED i ADP tenen la mateixa altura. Així:
AADP PD 5 AAED DE 3 AACE EC 12
     
AACP PC 20 AACP PC 20 AACP PC 20

 5
 AADP  60

1  3
Com que AACP    AAED 
3  60
 12
 AACE  60

1 6
AAME
D’altra banda, com que EM és mitjana del triangle ACE,  AACE . Llavors:
2 60
1 1 3 1
 AAMF  AAME  AAED  ADEF    ADEF  ADEF 
6 10 60 60
1
Anàlogament, AFGH  , i així:
60
1 1 1
APajarita   
60 60 30
Així doncs, l’àrea del triangle ABC és 30 cops l’àrea del llacet.

   

437
 
 
 
 
 
 

Geometria analítica
Geometria analítica 7
 
 
 
 
 
 

MATEMÀTIQUES A LA TEVA VIDA 
1. Per què et sembla que influeixen el vent i les marees en el rumb del vaixell sinistrat?
El vent i les marees poden variar el rumb, allunyant o apropant el vaixell cap a la costa. Quant més temps passi 
el vaixell a la deriva, més probable és que s’esdevingui. 

2. Quina relació hi ha entre el vector ሬሬሬሬሬሬԦ


ࡻࡼ i els vectors ሬሬሬሬሬሬԦ
࡭࡮ǡ ࡮ࡼ ሬሬሬሬሬሬԦ?
ሬሬሬሬሬሬԦ࢏ࡻ࡭
   
OP  OA  AB  BP  

3. Trobes que és possible aplicar un mètode semblant si volguessin interceptar un vaixell


sospitós de contraban albirat des d’un avió?
Sí, sempre que l’avió es posicioni sobre el vaixell, i emeti el senyal de ràdio a la base dels
guardacostes.

4. Determina les equacions de les rectes descrites a continuació:


a) La direcció del vaixell guardacostes quan va a efectuar el rescat.
b) La recta que uneix la base amb el primer punt de contacte.
c) La recta que uneix la base amb el segon punt de contacte.
d) La direcció del vaixell sinistrat quan va a la deriva.
Siguin A(a1 , a2 ), B(b1 , b2 ), P( p1 , p2 ) y O(0, 0) . Llavors:


a) La direcció que porta el vaixell guardacostes és la direcció del vector OP  P  O  ( p1 , p2 ) .

p2
La recta que marca aquesta direcció és y  x.
p1

a2
b) La recta que uneix la base amb el punt A, és y  x.
a1

b2
c) La recta que uneix la base amb el punt B, és y  x.
b1


d) La direcció del vaixell quan va a la deriva és la del vector AB  (b1  a1 , b2  a2 ) . La recta
corresponent és:

b2  a2 b a b a
y x 1 2 2 1
b1  a1 b1  a1

438
 
 
 
 
 
 

Geometría analítica 77
 
 
 
 
 
 

5. Assenyala els punts de tall que tenen les rectes anteriors entre elles.
El punt O(0, 0) és el punt de tall entre les rectes dels apartats a), b) i c).

▪ El punt A és el punt de tall entre les rectes rOA i rAB .

▪ El punt B és el punt de tall entre les rectes rOB i rAB .

▪ El punt P és el punt de tall entre les rectes rOP i rAB , es troba alineat amb A i B.

439
 
 
 
 
 
 

8
 
 
 
 
 
 

Llocs geomètrics. Còniques


 

 
Llocs geomètrics. Còniques   
 
 
 
 
 

ACTIVITATS 
 
1. Si en lloc d’una superfície cònica fem servir un cilindre, quines còniques podem obtenir?
Si el pla és perpendicular a la generatriu del cilindre, la secció és una circumferència.

Si no és perpendicular, la secció és una el·lipse.

2. Justifica per què la paràbola és una secció cònica que no té dues branques.
Perquè el pla únicament talla un dels cons de la superfície cònica.

3. Dibuixa el lloc geomètric dels punts que equidisten de dues rectes:


a) Que és tallen en un punt. b) Que són paral·leles.
a) El lloc geomètric està format per les dues bisectrius dels angles que formen les rectes en tallar-se.
 

 
b) El lloc geomètric és una altra recta paral·lela a ambdues.

 
 
4. Determina el lloc geomètric dels punts del pla tals que la seva suma de coordenades
cartesianes és igual a 10.
I si la condició del lloc geomètric és que el seu producte sigui 10?
Els punts que compleixen la primera condició formen una recta d’equació x + y = 10.
Els punts que compleixen la segona condició formen una hipèrbole equilàtera d’equació xy = 10.
 
5. La distància que hi ha entre els focus d’una el·lipse és de 6 cm. Calcula la mida de l’eix menor
tenint en compte que l’eix major fa 10 cm.

441
 
 
 
 
 
 

Llocs geomètrics.
Llocs Còniques
geomètrics. Còniques 8
 
 
 
 
 
 

2c = 6 cm → c = 3 cm
2a = 10 cm → a = 5 cm
Atès que a2 = b2 + c2 → b = 4 cm
Així, la mida de l’eix menor és de 8 cm.
 
6. Un punt d’una el·lipse dista de cadascun dels focus 6 cm i 7 cm, i l’eix menor fa 6,6 cm. Calcula
la mida de l’eix major i la distància entre els focus.
2a = d(P,F) + d(P,F’) = 6 + 7 = 13 cm
L’eix major mesura 13 cm.
2a = 13 cm → a= 6,5 cm 2b = 6,6 cm → b = 3,3 cm
Atès que a = b + c → c = 5,6 cm
2 2 2

Així doncs, la distància entre els focus mesura 2c = 11,2 cm.


 
7. Calcula l’equació d’una el·lipse que té els punts F(5, 0) i F' (–5, 0) com a focus i dos dels seus
vèrtexs se situen en els punts A(8, 0) i A' (–8, 0).

��� �� ��
� → � � √39 → � �1
��� 64 39

8. Determina l’equació de l’el·lipse que té de vèrtexs els punts següents:


(–4, 0) (0, –2) (4, 0) (0, 2)

��4 � ��
� → � �1
� � 2 16 4

9. Calcula les excentricitats d’aquestes el·lipses:


�� �� �� ��
a) � �� b) � ��
��� ��� ��� �

�� �� � � 12
a) � �1→�
��� ��� � � 11
√��
Atès que a2 = b2 + c2 → c = √23 →e= � ��39
��

�� �� � � 1�
b) � � 1�
��� � ��3
�√� �√�
Atès que a2 = b2 + c2 → c = √216 � 6√6 → � � � � ��9�
�� �

442
 
 
 
 
 
 

Llocs geomètrics. Còniques 88


 
 
 
 
 
 

10. D’una el·lipse sabem que l’excentricitat és 0,6 i que dos dels vèrtexs se situen en els punts F(6,
0) i F'(–6, 0). Determina’n els vèrtexs.
c
a  6    e  0,6   → c  3,6 
a

Atès que a2 = b2 + c2 → b2 = 23,04 → B(0; 4,8) i B'(0; -4,8)


11. En una hipèrbole, la distància entre els vèrtexs és de 8 cm si B(0, 3) i el seu punt simètric és B'
(0, –3). Calcula els focus de la hipèrbole.
2a  8 → a  4     b  3 
Atès que c2 = a2 + b2 → c = 5 → F(5, 0) i F'(- 5, 0)

12. En una hipèrbole, la distància entre els focus és de 25 unitats i la distància entre els vèrtexs és
de 16 unitats. Determina B i B'.
2c  25 → c  12,5        2a  16 → a  8 
Atès que c2  a2  b2 → b  9,6 → B(0; 9,6) i B'(0; 9,6) 
 
13. Determina l’equació de la hipèrbole que té els focus en els punts (–2, 0) i (2, 0) i els vèrtexs en
(–1, 0) i (1, 0).

��� �� ��
� → � � √3 → � �1
��1 1 3

14. Calcula l’equació de la hipèrbole que té els focus en els punts (5, 0) i (–5, 0) i que passa pel
�√�
punt ( 6, ).

c  5 

Atès que c2  a2  b2 → 25  a2  b2 → b2  25  a2 

x2 y2 x2 y2
Així doncs, l’equació de la hipèrbole és:  - 2 =1 → - =1 
a2 b a 2
25 - a2

æ 3 5 ö÷÷ 36 45
Atès que el punt  ççç6, ÷ pertany a la hipèrbole: 2 - = 1 → 4a4  289a2  3 600  0 
çè 2 ø ÷ a 100 - 4 a2

225
Tenim dues solucions per a2:  i 16 
4  

Com que b2  25  a2  0 → a2  16 → b2  9 

x2 y2
L’equació de la hipèrbole és:  - =1 
16 9

   

443
 
 
 
 
 
 

Llocs geomètrics.
Llocs Còniques
geomètrics. Còniques 8
 
 
 
 
 
 

15. Determina els focus i els vèrtexs de la hipèrbole que té l’equació següent:

 
2 2
x y
- =1 
64 225

a2  64 → a  8 → A(8, 0) i A'(8, 0) 

b2  225 → b  15 → B(0, 15) i B'(0, 15) 

Atès que c2  a2  b2 → c2  289 → c  17 → F(17, 0) i F'(17, 0) 

 
16. Calcula l’excentricitat de les hipèrboles que estan determinades per aquestes equacions:

 
a)  a   576 → a  24   
2
    b   49 → b  7 
2

Atès que c2  a2  b2 → c  25 


c 25
e        1,04 
a 24
�� �� � � ��
b)  � � �� → �  
��� �� ���
√���
Atès que c2  a2  b2 → c  √157 → � � � 1�1� 
��

17. Determina l’equació de la paràbola amb el vèrtex a l’origen de coordenades i focus F(0, 2).
p = 4 → x2 = 8y
 
18. Calcula l’equació de la paràbola que té el vèrtex a l’origen de coordenades i focus F (2, 0).
p = 4 → y2 = 8x

19. Determina l’equació de la circumferència amb centre C (–3, 1) i que passa per l’origen de
coordenades.

� � ���� �� � ���� � 0�� � �1 � 0�� � � √10


L’equació de la circumferència és: (x + 3)2 + (y – 1)2 = 10 → x2 + y2 + 6x – 2y = 0.

 
20. Comprova si aquesta equació correspon a una circumferència x2 + y2 – 6x + 2y + 11 = 0.
A  6 → a  3        B  2 → b  1 

C  11  a2  b2  r2  10  r2 → r2  1 → No té solució.

Per tant, aquesta equació no correspon a una circumferència.

444
 
 
 
 
 
 

Llocs geomètrics. Còniques 88


 
 
 
 
 
 

21. Estudia la posició relativa de les circumferències que tens a continuació


x2 + y2 – 9 = 0 x2 + y2 – 2x – 2y + 1 = 0
� ��� ��
x2 + y2 – 9 = 0 →� �
�� � �

�2 ��� ��
x2 + y2 – 2x – 2y + 1 = 0 → �
�� � �
���� � �� � � ��� � ��� � �� � ��� � √2 � �� � �� � 2
Les circumferències són interiors.

22. Troba una circumferència tangent interior a la circumferència que té l’equació següent:
(x – 3)2 + (y + 1)2 = 4
Resposta oberta.
Una de les circumferències tangents interiors és (x – 2)2 + (y + 1)2 = 1
 
23. Determina la posició relativa de la circumferència x2 + y2 – 6x + 4y + 4 = 0 i els eixos de
coordenades.
���� �2�
x2 + y2 – 6x + 4y + 4 = 0 →�
�� � �

|��|
La distància del centre a l’eix d’abscisses és: ���� �� � �2
√�� ���

Atès que és més petit que el radi, l’eix és secant a la circumferència.


|�|
La distància del centre a l’eix d’ordenades és: ���� �� � ��
√�� ���

Com que el radi té la mateixa mesura, l’eix és tangent a la circumferència.


 
24. Troba tres rectes no paral·leles que siguin secant, tangent i exterior a la circumferència
x2 + (y – 3)2 = 36.
Resposta oberta
Una recta secant és: x – y = 0
Una recta tangent és: y + 3 = 0
Una recta exterior és: x – 7 = 0
 

   

445
 
 
 
 
 
 

Llocs geomètrics.
Llocs Còniques
geomètrics. Còniques 8
 
 
 
 
 
 

SABER FER 
 
25. Determina el lloc geomètric dels punts que equidisten de la recta y = 5x – 2 i de l'eix Y.
Prenem P(x, y) un punt genèric del lloc geomètric

5x - y - 2 5x - y - 2
d(P, r)   2 2
         d(P, OY)  │x│ 
5 + (-1) 26

5x - y - 2
Atès que d(P, r)  d(P, OY) → = x  5 x – y – 2 = 26 x  (5 - 26 ) x – y – 2 = 0   
26

 5 x – y – 2 = – 26 x  (5 + 26 ) x – y – 2 = 0  

El lloc geomètric són les dues equacions obtingudes que, atès que són de grau 1, equivalen a dues
rectes que són les bisectrius dels angles que formen la recta donada i l’eix Y.

26. Determina el lloc geomètric dels punts que equidisten de la recta 3y = x + 1 i de l’eix X.

Prenem P(x, y) un punt genèric del lloc geomètric.

x - 3y + 1 x - 3y + 1
d(P, r)            d(P, OX)  │y│ 
12 + (-3)2 10
 

x - 3y + 1
Atès que d(P, r)  d(P, OX) →  = y  x – 3 y + 1 = 10 y  x – (3 + 10 ) y + 1 = 0  
10

x – 3 y + 1 = – 10 y  x – (3 – 10 ) y + 1 = 0  

El lloc geomètric són les dues equacions obtingudes que, atès que és de grau 1, equivalen a dues
rectes que són les bisectrius dels angles que formen la recta donada i l’eix X.

27. Calcula l’equació d’una el·lipse que té excentricitat 0,8 i distància focal 8.
c 4
2c  8 → c  4                  e= = = 0,8 → a  5 
a a

Atès que a2  b2  c2 → b2  9 

x2 y2 x2 y2
L’equació de l’el·lipse és del tipus:  + 2 =1 → + = 1 
a2 b 25 9

   

446
 
 
 
 
 
 

Llocs geomètrics. Còniques 88


 
 
 
 
 
 

28. Calcula l’equació d’una el·lipse amb eixos paral·lels als eixos X i Y, de centre C (4, 1) i vèrtexs
A' (1, 1) i B ' (4, –1).
Es trasllada el centre C(4, 1) a l’origen de coordenades:

A'(1, 1) → A''(1  4, 1  1)  (3, 0) → a  3 

B'(4, 1) → B''(4  4, 1  1)  (0, 2) → b  2 

L’equació de l’el·lipse és del tipus:


2 2
x2 y2 ( x - 4) ( y - 1)
+ 2 =1 → + = 1 
a2 b 9 4

29. Calcula l’equació de la hipèrbole amb eixos paral·lels als eixos X i Y si saps que el centre és C
(–1, 2), un dels focus és F (3, 2) ) i l’excentricitat és 1,5.
Es trasllada el centre C(1, 2) a l’origen de coordenades:

F(3, 2) → F''(3  (1), 2  2)  (4, 0) → c  4 

c 4 8 80
e        1,5 → a               c2  a2  b2 → b2    
a a 3 9

L’equació de la hipèrbole és del tipus:


2 2
x2 y2 9 ( x + 1) 9 ( y - 2)
- 2 =1 → - =1 
a2 b 64 80

30. Troba l’equació de la hipèrbole amb eixos paral·lels als eixos X i Y si saps que el centre és C(4,
0), un dels focus és F (1, –3) i l’excentricitat és 2.
Es trasllada el centre C(4, 0) a l’origen de coordenades:

F(1, 0) → F''(1  4, 0)  (3, 0) → c  3 

c 3 3 27
e        2 → a                 c2  a2  b2 → b2    
a a 2 4

L’equació de la hipèrbole és del tipus:


2
x2 y2 4 ( x - 4) 4y 2
- 2 =1 → - =1 
a2 b 9 27

31. Calcula l’equació d’una paràbola de vèrtex V (–2, 1) i de directriu la recta y = –1.

Es trasllada el vèrtex V(–2, 1) a l’origen de coordenades.

p
y   1 → y'   1  1   2   - → p  4 
2

447
 
 
 
 
 
 

Llocs geomètrics.
Llocs Còniques
geomètrics. Còniques 8
 
 
 
 
 
 

L’equació de la paràbola és del tipus:

(x  2)2  2p(y  1) → (x  2)2  8(y  1) → x2  4x  8y  12  0 

32. Determina l’equació d’una paràbola de vèrtex V (3, 5) i de directriu la recta y = 1.


Es trasllada el vèrtex V(3, 5) a l’origen de coordenades.

p
y  1 → y'  1  5  4   - → p  8 
2

L’equació de la paràbola és del tipus:

(x  3)2  2p(y  5) → (x  3)2  16(y  5) → x2  6x  16y  89  0 

33. Calcula l’equació general de la circumferència que passa pels punts P (1, 4), Q (1, 0) i R (3, 2).
Atès que x2  y2  Ax  By  C  0: 

P (1, 4)  A + 4B + C = –17 üïï


ïï
Q (1, 0 )  A + C = –1  ý  A = –2B = –4C = 1  x + y – 2 x – 4 y + 1 = 0  
2 2
ïï
R (1, 4)  3 A + 2B + C = –13ïïþ

34. Calcula el centre i el radi de la circumferència que passa pels punts P (–4, 4), Q (–5, 1) i R (–1,
3).
Atès que (x  a)2  (y  b)2  r2: 

ìï 15
P ( –4, 4)  ( –4 – a) + (4 – b) = r 2 üïï ïïï a = -
2 2

16 + a2
+ 8 a + 16 + b 2
– 8 b = r ï 10
ïïï ïï ïï
1+ a2 – 2a + b2 = r 2 ïý  ïí b = 2
2
Q(1, 0)  (1– a) + b2 = r 2 ý  
ïï ïï ïï
2 2 ïï 1 + a 2
– 2 a + 16 + b 2
– 8 b = r 2ï ï 41
R (1, 4)  (1– a) + (4 – b) = r 2
ïþ ï ï
þ ïr = 2
ïïî 4

æ 15 ö÷ 41
C(a, b) → C ççç- , 2÷         r    
è 10 ÷ø 2

35. Calcula l’equació d’una circumferència que passa pels punts A (1, –2) i B (–3, 0) i que té el
centre a la recta d’equació y = 5 – 2x.
Es calcula el lloc geomètric dels punts P(x, y) que són equidistants de A i B:

d(A, P)  d(B, P) → ( x - 1)2 + ( y + 2)2 = ( x + 3)2 + y 2 →

→ x2  2x  1  y2  4y  4  x2  6x  9  y2 → 8x  4y   4 → 2x  y   1 

El centre serà la intersecció d’aquest lloc geomètric i la recta donada:

ü
2 x – y = –1ï
ï ìx = 1
ï
ýï
í  C(1, 3)           d(C, A)   (1- 1)2 + (-2 - 3)2   5 → r  5 
2x + y = 5 ïþ ï
ï ïy = 3
î

448
 
 
 
 
 
 

Llocs geomètrics. Còniques 88


 
 
 
 
 
 

L’equació de la circumferència: (x  1)2  (y  3)2  25 → x2  y2  2x  6y  15  0 

 
36. Calcula el centre i el radi d’una circumferència que passa pels punts A (1, 1) i B (2, 4) i que té el
centre a la recta d’equació 3x + y – 11 = 0.
Es calcula el lloc geomètric dels punts P(x, y) que són equidistants de A i B:

d(A, P)  d(B, P) → ( x - 1)2 + ( y - 1)2 = ( x - 2)2 + ( y - 4)2 → x2  2x  1  y2  2y  1  x2  4x  4  y2  8y  

16→

→ 2x  6y  18 → x  3y  9 

El centre serà la intersecció d’aquest lloc geomètric i la recta donada:

x + 3y = 9 üï
ï ìx = 3
ï
ýï
í  C(3, 2)           d (C, A) = (1- 3)2 + (1- 2)2 = 5  r = 5  
3 x + y = 11ï
ï ï
þ ïy = 2
î

L’equació de la circumferència: (x  3)2  (y  2)2  5 → x2  y2  6x  4y  8  0 

 
37. Identifica quines còniques són les que tenen aquestes equacions.
�� ��
a) x2 + y2 + 11 = 4y – 8x c) x2 – 2x – 10y = 19 e) ���
��� ��
��
b) � ��� � � d) x2 – 4y = 0 f ) x2 + y2 + 6y = 7
��
a) Apareixen les dues variables al quadrat amb coeficient +1; per tant, és una circumferència:

x2  y2  8x  4y  11  0 → A  8, B   4 y C  11 

A   2a → a   4          B   2b → b  2       C  a2  b2  r2 → r2  9 

La cònica és una circumferència de centre C(-4, 2) i r = 3.

b) Apareixen les dues variables al quadrat amb mateix signe i diferent coeficient, per tant és una
el·lipse:

x2 y2 1 143 143
+ = 1  a = 4, b =         c2 = c=  
16 1 3 9 3
9

æ 143 ö÷ æ 143 ö÷÷


La cònica és una el·lipse de focus F ççç , 0÷÷  i F 'ççç-
÷ø
, 0 ÷ . 
çè 3 çè 3 ø÷

c) Apareix una variable al quadrat i l’altra amb grau 1; per tant, és una paràbola:

x2  2x  1  10y  19  1 → (x  1)2  10(y  2) 

La cònica és una paràbola de vèrtex V(1, 2) i directriu paral·lela a l’eix Y.

d) Apareix una variable al quadrat i l’altra amb grau 1; per tant, és una paràbola:

449
 
 
 
 
 
 

Llocs geomètrics.
Llocs Còniques
geomètrics. Còniques 8
 
 
 
 
 
 

x2  4y 

La cònica és una paràbola de vèrtex l’origen de coordenades i directriu paral·lela a l’eix Y.

e) Apareixen les dues variables al quadrat amb signe diferent, per tant és una hipèrbole:

x2 y2
2
- 2 =1 → a  15, b  9 
15 9

Atès que  a2  b2  c2 → c 2 = 306 = 3 54  

La cònica és una hipèrbole de focus F (3 54, 0)  i  F '(-3 54, 0) . 

f) Apareixen les dues variables al quadrat amb coeficient + 1; per tant, és una circumferència:

x2  y2  6y  7  0 → A  0, B  6 y C   7 

A   2a → a  0B  2b → b  3C  a2  b2  r2 → r2  16 

La cònica és una circumferència de centre C(0, –3) i r = 4.

38. Estudia la posició dels punts A(0, 2) i B (4, 1) respecte de la circumferència x2 + (y – m)2 = 16,
segons el valor de m.
Centre i radi de la circumferència: C(0, m) i r = 4.

Posició de A(0, 2) amb respecte a la circumferència donada:

d(C, A)   (0  0)2  (2  m )2    m2 - 4m + 4  

m2 - 4m + 4 = 4 → m2  4m  12  0 → m1  6 y m2  2 

m2 - 4m + 4 > 4 → m2  4m  12  0 → m  2 y m  6 

m2 - 4m + 4 < 4 → m2  4m  12  0 → 2  m  6 

Si m  2 o m  6 → El punt A és exterior.

Si m  6 o m  2 → El punt A pertany a la circumferència.

Si 2  m  6 → El punt A és interior.

Posició de B(4, 1) amb respecte a la circumferència donada:

d(C, B)   (4 - 0)2 + (1- m)2    m2 - 2m + 17  

m2 - 2m + 17 = 4 → m2  2m  1  0 → m  1 

m2 - 2m + 17 > 4 → m2  2m  1  0 → m  1 

m2 - 2m + 17 < 4 → m2  2m  1  0 → No té solució. 

450
 
 
 
 
 
 

Llocs geomètrics. Còniques 88


 
 
 
 
 
 

Si m  1 → El punt B pertany a la circumferència. 

Si m  1 → El punt B és exterior.

39. Donada la circumferència (x – m)2 + (y – 1)2 = 9, estudia la posició dels punts A(0, –2) i B (–5, 1)
respecte de la circumferència en funció del valor de m.
Centre i radi de la circumferència: C(m, 1) y r  3. 

Posició de A(0, 2) amb respecte a la circumferència dada:

d(C, A)   (0 - m)2 + (-2 - 1)2    m2 + 9  

m2 + 9 = 3 → m2  0 → m  0 

m2 + 9 > 3 → m2  0 → m  0 

m2 + 9 < 3 → m2  0 → No té solució.

Si m  0 → El punt A pertany a la circumferència.

Si m  0 → El punt A és exterior.

Posició de B(5, 1)amb respecte a la circumferència dada: 

d(C, B)   (-5 - m)2 + (1- 1)2    m2 + 10 m + 25  

m2 + 10 m + 25 = 3 → m2  10m  16  0 → m1  8 y m2  2 

m2 + 10 m + 25 > 3 → m2  10m  16  0 → m  8 y m  2 

m2 + 10m + 25 < 3 → m2  10m  16  0 → 8  m  2 

Si m < -8 o m > 2 → El punt B és exterior.

Si m  8 o m  2 → El punt B pertany a la circumferència. 

Si 8  m  2 → El punt B és interior. 

   

451
 
 
 
 
 
 

Llocs geomètrics.
Llocs Còniques
geomètrics. Còniques 8
 
 
 
 
 
 

ACTIVITATS 
 

40. Determina el lloc geomètric dels punts que equidisten dels punts A i B.
a) A(–6, 0) i B(–1, 0) c) A(3, –5) i B(7, 1)
b) A(–2, –1) i B(4, 1) d) A(0, –2) i B(0, 7)
a) Sigui P(x, y) un punt equidistant de A i B, aleshores d(A, P)  d(B, P). 

d ( A, P ) = ( x - (-6))2 + ( y - 0)2 = x 2 + y 2 + 12 x + 36 d ( B, P ) = ( x - (-1))2 + ( y - 0)2 = x2 + y 2 + 2x + 1  


     
7
x 2 + y 2 + 12 x + 36 = x 2 + y 2 + 2 x + 1  12 x + 36 = 2 x + 1  x = -
2
7
El lloc geomètric és la recta, mediatriu del segment AB, paral·lela a l’eix Y amb equació x = - . 
2

b) Sigui P(x, y) un punt equidistant de A i B, aleshores d(A, P)  d(B, P). 

d ( A, P ) = ( x - (-2))2 + ( y - (-1))2 = x 2 + y 2 + 4 x + 2 y + 5   

d ( B, P ) = ( x - 4)2 + ( y - 1)2 = x 2 + y 2 - 8 x - 2 y + 17   

x 2 + y 2 + 4 x + 2y + 5 = x 2 + y 2 - 8 x - 2 y + 17  4 x + 2 y + 5 = –8 x – 2 y + 17  3 x + y – 3 = 0  

El lloc geomètric és la recta, mediatriu del segment AB, que té per equació 3x  y  3  0. 

c) Sigui P(x, y) un punt equidistant de A i B, aleshores d(A, P)  d(B, P). 

d(A, P)   ( x - 3)2 + ( y - (-5))2    x 2 + y 2 - 6 x + 10 y + 34  

d(B, P) = ( x - 7)2 + ( y - 1)2    x 2 + y 2 - 14 x - 2 y + 50  

x 2 + y 2 - 6 x + 10 y + 34 = x 2 + y 2 - 14 x - 2 y + 50  –6 x + 10 y + 34 = –14 x – 2 y + 50  2 x + 3 y – 4 = 0   

El lloc geomètric és la recta, mediatriu del segment AB, que té per equació 2x  3y  4  0. 

d) Sigui P(x, y) un punt equidistant de A i B, aleshores d(A, P) = d(B, P):

d ( A, P ) = ( x - 0)2 + ( y - (-2))2 = x 2 + y 2 + 4y + 4  

d ( B, P ) = ( x - 0)2 + ( y - 7)2 = x 2 + y 2 - 14 y + 49  

5
x 2 + y 2 + 4 y + 4 = x 2 + y 2 - 14 y + 49  4 y + 4 = –14 y + 49  y =  
2

5
El lloc geomètric és la recta, mediatriu del segment AB, paral·lela a l’eix X amb equació y = .
2

   

452
 
 
 
 
 
 

Llocs geomètrics. Còniques 88


 
 
 
 
 
 

41. Calcula la mediatriu del segment que té com a extrems A(5, –2) i B(4, 3).
Sigui P(x, y) un punt de la mediatriu, aleshores d(P, A)  d(P, B): 

d(A, P)   ( x - 5)2 + ( y - (-2))2    x 2 + y 2 - 10 x + 4 y + 29  

d(B, P) = ( x - 4)2 + ( y - 3)2    x 2 + y 2 - 8 x - 6 y + 25  

d(A, P)  d(B, P) → x 2 + y 2 - 10 x + 4 y + 29    x 2 + y 2 - 8 x - 6 y + 25 → 10x  4y  29  8x  6y  25 →

→ x  5y  2  0 

La mediatriu és la recta x  5y  2  0. 

42. Determina el lloc geomètric dels punts P del pla la distància dels quals a A(–3, 0) sigui el doble
de la distància a B(1, 0). Identifica la figura que en resulta.
Sigui P(x, y) un punt del lloc geomètric, aleshores d(A, P)  2d(B, P). 

d(A, P)   ( x - (-3))2 + ( y - 0)2    x 2 + y 2 + 6 x + 9  

d(B, P) = ( x - 1)2 + ( y - 0)2    x 2 + y 2 - 2 x + 1  

x2 + y2 + 6x + 9    2 x 2 + y 2 - 2 x + 1 → x2  y2  6x  9  4(x2  y2  2x  1) → 3x2  3y2  14x  5  0 

La figura que trobem és la circumferència que té per equació:

14 5
x2 + y2 – x – =0 
3 3

43. Calcula el lloc geomètric dels punts que disten 4 unitats de la recta r: 4x – 2y + 5 = 0.

 
|�������|
Sigui (x, y) un punt del lloc geomètric, aleshores:  � � 
��� ������

�� � 2� � 5 � �√5 � 0
→ |�� � 2� � 5| � �√20 → |�� � 2� � 5| � �√5 → �  
��� � 2� � 5 � �√5 � 0

   

453
 
 
 
 
 
 

Llocs geomètrics.
Llocs Còniques
geomètrics. Còniques 8
 
 
 
 
 
 

44. Determina el lloc geomètric dels punts que disten 2 unitats de la recta r: x – 3y + 1 = 0.
Sigui P(x, y) un punt del lloc geomètric, aleshores d(r, P)  2. 

x - 3y + 1 ì
ï x – 3 y + 1– 2 10 = 0
 = 2  x – 3 y + 1 = 2 10  ï
í  
2
1 + (-3) 2 ïï- x + 3 y – 1– 2 10 = 0
ï
î

45. Troba el lloc geomètric dels punts que equidisten dels parells de rectes següents:
a) 3x – 4y – 26 = 0 c) 2x – 3y + 3 = 0
12 x + 5y + 1 = 0 3x – 2 = 0
b) –2 x + 7y + 9 = 0 d) 2y – 5 = 0
4x – 14y + 11 = 0 2 x – 3y + 5 = 0

El lloc geomètric està format per les dues bisectrius de les rectes.

Atès que les rectes són paral·leles, el lloc geomètric és una altra recta paral·lela a les mateixes.

2 x - 3y + 3 3x - 2
c)  =  
22 + (-3)2 32 + 0 2

ì2x - 3y + 3 3 x - 2
ï
ï
ï = ï(6 – 3 13 ) x – 9 y + 9 + 2 13 = 0
ì
ï 13 3 ì6 x – 9 y + 9 = 3 13 x – 2 13
ï ï
íï ïí ï
í  
î( )
ï 2 x - 3 y + 3 -3 x + 2 ïï6 x – 9 y + 9 = -3 13 x + 2 13
ï ï 6 + 3 13 x – 9 y + 9 – 2 13 = 0
ï
ï = ï
î ï
ï 13 3
îï

El lloc geomètric el formen les dues bisectrius de les rectes.

2y - 5 2 x - 3y + 5
d)  2 2
=  
2 +0 22 + (-3)2

ì
ï 2y - 5 2 x - 3 y + 5
ï
ïï –4 x + (6 + 2 13 ) y – 10 – 5 13 = 0
ï = ì ì
ï 2 13 ï2 13 y – 5 13 = 4 x – 6 y + 10
ï
í ï
í  ïí  
ï
ïï 2 y - 5 = -2 x + 3 y - 5 ï
ï
î2 13 y – 5 13 = -4 x + 6 y 110 ïï
ïï ( )
ï4 x + –6 + 2 13 y + 10 – 5 13 = 0
î
î 2
ï 13

El lloc geomètric el formen les dues bisectrius de les rectes.

454
 
 
 
 
 
 

Llocs geomètrics. Còniques 88


 
 
 
 
 
 

46. Determina les bisectrius de les rectes següents:


3x – 2y – 1 = 0 4x + 2y – 6 = 0
ì3 x - 2y - 1 2 x + y - 3
ï
ï
ï =
3 x - 2y - 1 4 x + 2y - 6 ï 13 5
= ï
í  
3 2 + (-2)2 4 2 + 22 ï 3 x - 2 y - 1 -2 x - y + 3
ï
ï
ï =
ï
î 13 5

ï( ) ( )
ìï 3 5 x – 2 5 y – 5 = 2 13 x + 13 y – 3 13 ìï 3 5 – 2 13 x – 2 5 + 13 y – 5 + 3 13 = 0
ïí  ïí  
ïî( ) ( )
ï3 5 x – 2 5 y – 5 = -2 13 x - 13 y + 3 13 ïï 3 5 + 2 13 x – 2 5 – 13 y – 5 – 3 13 = 0
ï
ïî

47. Calcula l’equació del lloc geomètric dels punts del pla que tenen la mateixa distància a l’eix X
que al punt A(3, 2).
Sigui P(x, y) un punt del lloc geomètric, aleshores d(OX, P)  d(A, P). 

d(OX, P)  │y│ 

d(A, P)   ( x - 3)2 + ( y - 2)2 → x 2 + y 2 - 6 x - 4 y + 13   │y│ → x 2 - 6 x - 4 y + 13   0 

El lloc geomètric és una paràbola amb la directriu paral·lela a l’eix X.

48. Troba el lloc geomètric dels punts del pla que verifiquen que el triangle ABP sigui rectangle en
P, amb A(–3, 1) i B(4, 3). De quina figura es tracta?
ABP és triangle rectangle en P(x, y) si els seus costats AP i BP són perpendiculars:

AP  (x  3, y  1)       BP  (x  4, y  3) 

Seran perpendiculars si el seu producte escalar és 0, així doncs:

AP  BP  (x  3)(x  4)  (y  1)(y  3)  0 → x2  y2  x  4y  9  0 

El lloc geomètric és una circumferència que té per equació x2  y2  x  4y  9  0 i el seu centre és el


punt mig del segment AB i el seu radi mesura la meitat del seu mòdul.

49. Calcula el lloc geomètric dels punts la distància dels quals al punt A(2, –3) és la mateixa que el
quadrat de la seva distància a l’origen de coordenades.
Sigui P(x, y) un punt del lloc geomètric, aleshores d(A, P)  d(O, P)2. 

d(A, P)   ( x - 2)2 + ( y - (-3))2    x 2 + y 2 - 4 x + 6 y + 13  

d(O, P)   x 2 + y 2  

x 2 + y 2 - 4 x + 6 y + 13   x2  y2 → x2  y2  4x  6y  13  x4  2x2y2  y4 →

455
 
 
 
 
 
 

Llocs geomètrics.
Llocs Còniques
geomètrics. Còniques 8
 
 
 
 
 
 

→ x4  y4  x2  y2  2x2y2  4x  6y  13  0 

50. Determina el lloc geomètric dels punts del pla Q que verifiquen que el punt mitjà del segment
PQ és un punt de la recta que té d’equació 2x + 4y – 5 = 0, amb P(2, 6).
æ x + 2 y + 6 ÷ö
Sigui Q(x, y) un punt del lloc geomètric. El punt mig de PQ és  ççç , ÷ . 
è 2 2 ÷ø

æ x + 2 ö÷ æ y + 6 ÷ö
Aquest punt pertany a la recta → 2 ççç ÷   4 ççç ÷   5  0 → x  2y  9  0 
è 2 ø÷ è 2 ÷ø

El lloc geomètric és una recta paral·lela a 2x  4y  5  0 amb equació x  2y  9  0. 

51. Calcula l’equació del lloc geomètric dels punts del pla tals que la suma dels quadrats de les
seves distàncies als punts A(3, 1) i B(5, 1) és igual a 8.
Sigui P(x, y) un punt del lloc geomètric, aleshores d(A, P)2  d(P, B)2  8. 

d(A, P)   ( x - 3)2 + ( y - 1)2    x 2 + y 2 - 6 x - 2 y + 10  

d(B, P)   ( x - 5)2 + ( y - 1)2    x 2 + y 2 - 10 x - 2 y + 26  

x2  y2  6x  2y  10  x2  y2  10x  2y  26  8 → x2  y2  8x  2y  14  0 

El lloc geomètric és una circumferència que té per equació x2  y2  8x  2y  14  0. 

52. Calcula el lloc geomètric dels punts del pla P tals que la diferència de quadrats de les
distàncies als punts A(–1, 0) i B(3, 2) és 4. De quina figura es tracta?
Sigui P(x, y) un punt del lloc geomètric, aleshores d(A, P)2  d(P, B)2  4. 

d(A, P)   ( x - (-1))2 + y 2    x 2 + y 2 + 2 x + 1  

d(B, P)   ( x - 3)2 + ( y - 2)2    x 2 + y 2 - 6 x - 4 y + 13  

x2  y2  2x  1  (x2  y2  6x  4y  13)  4 → 2x  y  4  0 

El lloc geomètric és una recta que té per equació 2x  4  y  0. 

53. Determina els vèrtexs, els focus i les excentricitats de les el·lipses següents:
࢞૛ ࢟૛
a) ൅ ൌ૚ c) 25x2 + 16y2 = 1600
૛૞ ૢ

࢞૛ ࢟૛ ࢞૛ ࢟૛
b) ൅ ൌ૚ d) ൅ ൌ૚
૚૟ ૛૞ ૛૞ ૚૟

456
 
 
 
 
 
 

Llocs geomètrics. Còniques 88


 
 
 
 
 
 

�� �� � � 5 � ��5, 0��������5, 0�
a) � �1��
�� � � � 3 � ��0, 3�������0, �3�

Atès que a2 = b2 + c2 → c = 4 → F(4, 0) F’(-4, 0)


L’excentricitat és: e = = 0,8

�� �� � � 4 � ��4, 0��������4, 0�
b) � �1��
�� �� � � 5 � ��0, 5�������0, �5�

Atès que a2 = b2 + c2 → c = 3 → F(0, 4) F’(0, -4)


L’excentricitat és: e = = 0,6

�� �� � � 8 � ��8, 0��������8, 0�
c) �5� � � 1�� � � 1�00 � � �1��
�� ��� � � 10 � ��0, 10�������0, �10�

Atès que a2 = b2 + c2 → c = 6 → F(0, 6) F’(0, -6)


L’excentricitat és: e = = 0,6
��

�� �� � � 5 � ��5, 0��������5, 0�
d) � �1��
�� �� � � 4 � ��0, 4�������0, �4�

Atès que a2 = b2 + c2 → c = 3 → F(3, 0) F’(-3, 0)


L’excentricitat és: e = = 0,6

54. Calcula els vèrtexs, els focus i les excentricitats de les el·lipses següents:
�� ��
a) � �� c) 9x2 + 25y2 = 900
� ��

��
b) � ��� � ��� d) x2 + 2y2 = 16

x2 y2
a)  + =6 
6 54

a = 3 6  A(3 6, 0) , A'( –3 6, 0)  

b = 6  B(0, 6 ) , B’(0, – 6 )  

Atès que a2  b2  c2 → c2  48 → c = 4 3  F (4 3, 0) , F '( –4 3, 0)  

4 3 2 2
L’excentricitat és:  e = = = 0,94  
3 6 3

   

457
 
 
 
 
 
 

Llocs geomètrics.
Llocs Còniques
geomètrics. Còniques 8
 
 
 
 
 
 

x2
b)  + 5 y 2 = 245  
5

a  35 → A(35, 0), A'(35, 0) 

b  7 → B(0, 7), B'(0, 7) 

Atès que a2  b2  c2 → c2  1 176 →  c = 14 6  F (14 6, 0) , F '( –14 6, 0)  

L’excentricitat és:  e = 14 6
= 0,98  
35

�� �� � � 10 � ��10, 0��������10, 0�
c) �� � � 2�� � � �00 � � �1��
��� �� � � 6 � ��0, 6�������0, �8�

Atès que a2 = b2 + c2 → c = 8 → F(8, 0) F’(-8, 0)


L’excentricitat és: e = = 0,8
��

�� �� � � 4 � ��4, 0��������4, 0�
d) � � � 2� � � 16 � � �1��
�� � � � √8 � 2√2 � ��0, 2√2�������0, �2√2�

Atès que a2 = b2 + c2 → c = 2√2 → F(2√2, 0) F’(-2√2, 0)

2√2 √2
L’excentricitat és: e = � = 0,707
� �

 
55. Calcula l’equació de l’el·lipse que compleix les condicions en cadascun d’aquests casos:
a) La distància focal és 4 i el semieix menor és 3.
b) La semidistància focal és 3 i l’eix major és 10.
√�
c) Passa pel punt (8, 3) i l’excentricitat és

d) Passa pel punt (–4, 1) i l’eix menor és 6.
a) c  2 i b  3 
Atès que a2  b2  c2 → a2  13 
x2 y2
Equació de l’el·lipse:  + = 1 
13 9

b)  c  3 y a  5 
Atès que a2  b2  c2 → b2  16 
x2 y2
Equació de l’el·lipse:  + = 1 
25 16

c 3 a 3
c)  e= = c=  
a 2 2

a2 x2 y2 2 2
Atès que a2  b2  c2 → b2 =  2 + 2 = 1 → x 2 + 4 y2 = 1  
4 a a a a
4

82 4 ⋅ 32 a2
Atès que (8, 3) és un punt de l’el·lipse:  + 2 =1 → a2  100 → b2 = = 25  
a2 a 4

458
 
 
 
 
 
 

Llocs geomètrics. Còniques 88


 
 
 
 
 
 

x2 y2
Equació de l’el·lipse:  + = 1 
100 25

(-4)2 12
d) Atès que (4, 1) és un punt de l’el·lipse i b  3:  + 2 =1 → a2  18 
a2 3

x2 y2
Equació de l’el·lipse:  + = 1 
18 9

 
56. Troba les equacions de les el·lipses que compleixen les condicions següents:
a) L’excentricitat és 0,6 i l’eix major fa 20.

b) Els focus són (6, 0) i (–6, 0) i l’excentricitat és

c) Passa pel punt (3, 2) i l’eix gran és 10.
d) Els focus són a (4, 0) i (–4, 0) i els dos vèrtexs són (5, 0) i (–5, 0).
a) 2a = 20 → a = 10
Si e = 0,6 → c = 6
�� ��
Atès que a2  b2  c2 → b  8 → � � �
��� ��

b) c = 6

Si e = → a = 18

�� ��
Atès que a2  b2  c2 → b  12√2 → � � �
��� ���

c) 2a = 10 → a = 5
(3, 2) és un punt de l’el·lipse, així doncs:
9 4 4 16 2 25 5
� �1→ � →� � →��
25 � 2
� 2 25 4 2
Així, l’equació de l’el·lipse és:
�� ��
� � 1 → � � � 4� � � 25
25 ��

��4
d) �
��5
�� ��
Atès que a2  b2  c2 → b  3 → � � �
�� �

 
57. Determina l’equació que respon als llocs geomètrics següents:
a) Punts amb suma de distàncies a P(–4, 0) i Q(4, 0) igual a 10.
b) Punts amb suma de distàncies a P(–5, 0) i Q(5, 0) igual a 21.
a) Sigui R(x, y) un punt del lloc geomètric, aleshores d(P, R)  d(Q, R)  10. 

d(P, R)   ( x - (-4))2 + ( y - 0)2  

d(Q, R)   ( x - 4)2 + ( y - 0)2  

459
 
 
 
 
 
 

Llocs geomètrics.
Llocs Còniques
geomètrics. Còniques 8
 
 
 
 
 
 

( x + 4)2 + y 2    ( x - 4)2 + y 2   10 → ( x + 4)2 + y 2   10   ( x - 4)2 + y 2 →

→ (x  4)2  y2  100  (x  4)2  y2  20 ( x - 4)2 + y 2 → 

→ 20 ( x - 4)2 + y 2   100  16x → 400(x2  y2  8x  16)  10 000  3 200x  256x2 → 

→ 144x2  400y2  3 600 

x2 y2
L’equació del lloc geomètric és una el·lipse d’equació:  + = 1 
25 9

b) Sigui R(x, y) punt del lloc geomètric, aleshores, d(P, R)  d(Q, R)  21 

d(P, R)   ( x - (-5))2 + ( y - 0)2  

d(Q, R)   ( x - 5)2 + ( y - 0)2  

( x + 5)2 + y 2 + ( x - 5)2 + y 2 = 21 → ( x + 5)2 + y 2 = 21- ( x - 5)2 + y 2   

→ (x  5)2  y2  212  (x  5)2  y2   42 ( x - 5)2 + y 2 → 

→ 42 ( x - 5)2 + y 2   441  20x → 1 764(x2  y2  10x  25)  194 481  17 640x  400x2 → 

→ 1 364x2  1 764y2  150 381 

4 x 2 4y 2
L’equació del lloc geomètric és una el·lipse d’equació: + = 1 
441 341

 
58. Escriu l’equació de l’el·lipse amb centre a l’origen de coordenades, focus sobre l’eix X i que
√� √�
passa pels punts ��, � i �√�, �.
� �

x2 y2
L’equació de l’el·lipse és del tipus  2
+ 2 = 1 . 
a b
2
æ 3 ÷ö
çç ÷
æ 3 ö÷ 2 çç 2 ÷÷
ç 1 è ø 1 3
Atès que çç1, ÷ és un punt de l’el·lipse:  2 + 2 = 1 → 2 + 2 = 1 → 4b2  3a2  4a2b2 
÷
è 2 ø a b a 4b
2
æ 2 ö÷
ççç ÷÷
æ 2 ÷÷ö çè 2 ø÷
2
ç 2 2 2
Atès que  çç 2, ÷÷ és un punt de l’el·lipse:  2 + 2 = 1 → 2 + 2 = 1 → 8b2  2a2  4a2b2 
è 2 ø a b a 4b

Igualem les dues equacions: 4b2  3a2  8b2  2a2 → a2  4b2 


1 3 a2
Substituïm en la primera equació:  2
+ 2 = 1 → a2  4,  b2 = → b2  1 
a a 4

x2 y2
Equació de l’el·lipse:  + = 1 
4 1

 
59. Determina l’equació reduïda d’una el·lipse amb distància focal igual a 8 tal que l’àrea del
rectangle construït sobre els seus eixos de 48√� unitats quadrades.

460
 
 
 
 
 
 

Llocs geomètrics. Còniques 88


 
 
 
 
 
 

12 5
c  4 →  2 a ⋅ 2 b = 48 5b=  
a
2
æ 12 5 ö÷
Atès que a2  b2  c2 → a2 = ççç ÷ + 16 → a4  16a2  720  0 → a2  36 
è a ÷ø÷
2
æ 12 5 ö÷
Atès que  b 2 = ççç ÷÷ → b2  20 
çè a ø÷

x2 y2
Equació de l’el·lipse:  + = 1 
36 20

 
60. Una el·lipse és tangent als costats del rectangle definit per les rectes y = 8, y = –8, x = 10 i x = –
10. Troba'n l’equació i les coordenades de cinc punts.

� � �� �� ��
� → � � � 
��� ��� ��

Cinc punts de l’el·lipse són: A(10, 0), A’(-10, 0), B(0, 8), B’(0, -8) i C(5, 4√3)

61. Calcula la posició relativa de l’el·lipse d’equació x2 + 2y2 = 3 amb la recta d’equació x + 2y – 1 =
0.
x  2y  1  0 → x  1  2y 

Substituïm a l’equació de l’el·lipse: x2  2y2  3 → (1  2y)2  2y2  3 → 3y2  2y  1  0 

∆  16  0 → L’equació té dues solucions; per tant, l’el·lipse i la recta són secants.

62. Escriu l’equació del lloc geomètric dels punts tals que la suma de distàncies a (0, –12) i a (0,
12) és igual a 26.

 
63. Escriu en la forma general l’equació de l’el·lipse 4x + 9y – 8x + 36y + 4 = 0.
2 2

Calcula’n també els focus i l’excentricitat.

461
 
 
 
 
 
 

Llocs geomètrics.
Llocs Còniques
geomètrics. Còniques 8
 
 
 
 
 
 

4(x2  2x)  9(y2  4y)  4  0 

4(x2  2x  1)  9(y2  4y  4)  4  4  36 → 4(x  1)2  9(y  2)2  36 

En la forma general:

( x - 1)2 ( y + 2)2
+ =1 → a  3, b  2 
9 4

Atès que a2  b2  c2 → c2  5 → c = 5  

c 5
Excentricitat: e= =  
a 3

Atès que C(1, 2) y  c = 5 → F (1+ 5,-2)  y  F '(1– 5, - 2) . 

 
64. Escriu en la forma general l’equació de l’el·lipse següent, troba’n els focus i calcula’n
l’excentricitat:
x2 + 400y2 = 6x – 800y – 309
(x2  6x)  400(y2  2y)  309  0 

(x2  6x  9)  400(y2  2y  1)  309  9  400 → (x  3)2  400(y  1)2  100 

Forma general: 

( x - 3)2 ( y + 1)2 1
+ =1 → a  10,  b =  
100 1 2
4

399
Atès que a2  b2  c2 → c2   → c   399  
4 2

Excentricitat:  e = c = 399
 
a 20

399 æ 399 ÷ö æ 399 ö÷ æ 6 + 399 ÷ö æ 6 - 399 ö÷


Atès que C(3, 1) y c   → F ççç3 + , - 1÷÷ y  F 'ççç3 - ,-1÷÷ → F ççç , - 1÷÷ y  F 'ççç
÷ø
,-1÷÷  
÷ø
2 çè 2 ÷ø çè 2 ÷ø çè 2 çè 2

 
65. Determina els focus, els vèrtexs, les asímptotes i les excentricitats de les hipèrboles següents:
�� �� �� ��
a) � �� d) � ��
�� � �� ��
�� ��
b) � �� e) ��� � ���� � ���
�� ��

c) ���� � ���� � ���� f) �� � ��� � ��

�� �� � � 5 � ��5, 0��������5, 0�
a) � ����  
�� � � � 3 � ��0, 3������0, �3�
Atès que c2  a2  b2 → c  √34 → F(√34, 0) F’(�√34, 0)  

462
 
 
 
 
 
 

Llocs geomètrics. Còniques 88


 
 
 
 
 
 
√��
L’excentricitat és: e =    = 1,16  



�� � � �

Les asímptotes són � � � � � � �   

��� � � � �

�� �� � � 5 � ��0, 5� ���0, �5�


b) � �1��  
�� �� � � 4 � ��4, 0� ����4, 0�
Atès que c2  a2  b2 → c  √41 → F(√41, 0) F’(�√41, 0)  

√��
L’excentricitat és: e =    = 1,28  



�� � � �

Les asímptotes són � � � � � � �   

��� � � � �

�� �� � � 10 � ��0, 10� ���0, �10�


c) 16� � � 25� � � 1600 � � � 1�  
��� �� � � 8 � ��8, 0� ����8, 0�
Atès que c2  a2  b2 → c  2√41 → F(0, 2√41) F’(0, �2√41)  

�√��
L’excentricitat és: e =    = 1,28  
��

ì
ï 4
ï
ï r: y = x
b ï 5
Les asímptotes són y =  x  ïí  
a ï
ï 4
ï
ï r ': y = - x
ï
î 5

�� �� � � 5 � ��5, 0� ����5, 0�
d) � �1��  
�� �� � � 4 � ��0, 4� ���0, �4�
Atès que c2  a2  b2 → c  √41 → F(√41, 0) F’(�√41, 0)  

√��
L’excentricitat és: e =    = 1,28  



�� � � �

Les asímptotes són � � � � � � �   

��� � � � �

�� �� � � 10 � ��0, 10� ���0, �10�


e) �� � � 25� � � �00 � � � 1�  
��� �� � � 6 � ��6, 0� ����6, 0�
Atès que c2  a2  b2 → c  2√34 → F( 2√34, 0) F’(�2√34, 0)  

�√��
L’excentricitat és: e =    = 1,16  
��

ì
ï 3
ï
ïr: y = x
b ï 5
  Les asímptotes són  y =  x  ïí  
a ï
ï 3
ï
ïr ': y = - x
ï
î 5

�� �� � � 4 � ��4, 0� ����4, 0�
f) � � � 2� � � 16 � � � 1�  
�� � � � √8 � ��0, 2√2� ���0, �2√2�
Atès que c2  a2  b2 → c  2√6 → F( 2√6, 0) F’(�2√6, 0)  

463
 
 
 
 
 
 

Llocs geomètrics.
Llocs Còniques
geomètrics. Còniques 8
 
 
 
 
 
 
�√�
L’excentricitat és: e =    = 1,22 

ìï
ïïr : y = 2 x
b ï 2
Les asímptotes són y =  x  ïí  
a ïï
ïïr ': y = - 2 x
ïî 2

 
66. Calcula l’equació de la hipèrbole que compleix les condicions de cada cas:
a) La distància entre els focus és 12 i la corba Passa pel punt P(8, 14).
b) La distància focal és 34 i la distància d’un focus al vèrtex més proper és 2.
c) Passa pels punts (4, √� ) I (2√�, 2).
a) c  6 
x2 y2
Atès que c2  a2  b2 → b2  36  a2 → - =1 
a 2 36 - a 2

82 14 2
I P(8, 14) pertany a la hipèrbole:  2
- =1 → a4  296a2  2 304  0 → a12  8 y a22  288 
a 36 - a2

Així doncs b2  0, a2  8 → b2  28 


x2 y2
- =1 
8 28

b)  c  17 

a  17  2  15 

Atès que c2  a2  b2 → b2  64 

x2 y2
Equació de la hipèrbole:  - =1 
225 64
2
42 8 16 8 8 a2
c)  2
- 2 = 1 → 2 - 2 = 1 → 16b2  8a2  a2b2 → b2    
a b a b 16 - a2

(2 3 )
2
22 12 4
2
- = 1 → 2 - 2 = 1 
a b2 a b

12 4(16 - a2 )
Se substitueix en la segona equació:  - =1 → a2  8 → b2  8 
a2 8 a2

x2 y2
Equació de la hipèrbole:  - =1
8 8  

464
 
 
 
 
 
 

Llocs geomètrics. Còniques 88


 
 
 
 
 
 

67. Troba les equacions de les hipèrboles que compleixen les condicions següents:
a) Les asímptotes són y = 2x i y = –2x i un focus té de coordenades (3√�, 0).
b) Els focus són (–5, 0) i (5, 0), i la distància entre els vèrtexs és 8.
� �
c) Les asímptotes són y = x i y = – x, i passa pel punt (3√��, 5).
� �
d) Un focus és (6, 0) i l’excentricitat és 1,2.

a) � 2 → � � 2� � � 3√5

2 2
Atès que c2  a2  b2 → 45  a2 +4 a2→ a2 = 9 → � � 3 → b=6 → x - y = 1  
9 36

b) c = 5 2a = 8 → a = 4
2 2
Atès que c2  a2  b2 → 25  16 +b2→ b2 = 9 → � � 3 → x -y =1 
16 9
� �
c) � → � � 3�  
� �

(3√29� 5� és un punt de la hipèrbole: 


261 25 36

� � � 1 → � � 1 → � � � � → � � 2 → � � 6 
9� � 9�
�� ��
Així doncs, l’equació de la hipèrbole és:  � � 1 
�� �

d) c = 6 � � � 1�2 → � � 5 

2 2
Atès que c2  a2  b2 → b  √11 → x -y =1 
25 11

 
68. Determina els focus, els vèrtexs, les asímptotes i l’excentricitat de les hipèrboles següents no
centrades a l’origen:
������ ������ �� ������
a) � �� c) � ��
�� � �� �
������ �� ������ ������
b) � �� d) � ��
�� � �� �

a)  C(5, 4) 

a2  10 → a   10 → Vèrtex: A(5   10 , 4), A'(5   10 , 4) 

b2  9 → Atès que c2  a2  b2 → c2  19 → c   19 → Focus: F(5   19 , 4), F'(5   19 , 4) 

c 19
Excentricitat:  e = =  
a 10

b 3 10 x
=
3 10
→ Asímptotes:  r : y = ,  r ': y = - 3 10 x
 
a 10 10 10

b)  C(0, 3) 

a2  25 → a  5 → Vèrtex: A(0, 3  5), A'(0, 3  5) → A(0, 2), A'(0, 8) 

b2  7 → Atès que c2  a2  b2 → c2  32 → c   4 2 → Focus: F(0, 3   4 2 ), F'(0, 3   4 2 ) 

465
 
 
 
 
 
 

Llocs geomètrics.
Llocs Còniques
geomètrics. Còniques 8
 
 
 
 
 
 

Excentricitat:  e = c = 4 2
 
a 5

a 5 7
= → Asímptotes:  r : y = 5 7
x ,  r ': y = - 5 7
x  
b 7 7 7

c) C(4, 0) 

a2  11 → a   11 → Vèrtex: A(4,  11 ), A'(4,  - 11 ) 

b2  9 → Atès que c2  a2  b2 → c2  20 → c   2 5 → Focus: F(4,  2 5 ), F'(4,  -2 5 ) 

Excentricitat:  e = c = 2 55
 
a 11

a 11 11 11
= → Asímptotes:  r : y = x ,  r ': y = - x  
b 3 3 3

d)  C(4, 1) 

a2  14 → a   14 → Vèrtexs: A(4   14 , 1), A'(4   14 , 1) 

b2  9 → Atès que c2  a2  b2 → c2  23 → c   23 → Focus: F(4   23 , 1), F'(4   23 , 1) 

c 23
Excentricitat:  e = =  
a 14

b 3 14 3 14
= → Asímptotes:  r : y = x ,  r ': y = - 3 14
x  
a 14 14 14

 
69. Troba l’equació de la hipèrbole no centrada a l’origen que compleix les condicions de cada
cas:

a) El centre és C(1, 0), l’excentricitat és i passa pel punt P(4, 3).

b) El centre és C(2, 3), un vèrtex és (–3, 3) i un focus és (9, 3).
c) El centre és C(–5, 2), un vèrtex és (0, 2) i l’excentricitat és 2.
 

c 4 4a
a)  e = = → c    
a 3 3

7a2
Atès que c2  a2  b2 → b2    
9

( x - 1)2 y2 2 2
L’equació de la hipèrbole és de la forma:  2
- 2 = 1 → ( x -2 1) - 9 y 2 = 1
a 7a a 7a  
9

(4 - 1)2 9 ⋅ 3 2
P(4, 3) pertany a la hipèrbole:  - =1 → a2  0 → Impossible, no existeix la hipèrbole. 
a2 7a2

b)  C(2, 3) 

466
 
 
 
 
 
 

Llocs geomètrics. Còniques 88


 
 
 
 
 
 

A(3, 3) → A(2  5, 3) → a  5 

F(9, 3) → F(2  7, 3) → c  7 

Atès que c2  a2  b2 → b2  24 

( x - 2)2 ( y - 3)2
Equació de la hipèrbole:  - =1 
25 24

c) C(-5, 2)

A(0, 2) → A(0  5, 2) → a  5 

e  2 c  e  a → c  10 

Atès que c2  a2  b2 → b2  75 

( x + 5)2 ( y - 2)2
Equació de la hipèrbole:  - = 1 
25 75

70. Determina l’equació d’una hipèrbole que té eix focal 18 i que passa pel punt P(15, 4).

2c  18 → c  9 

c2  a2  b2 → 81  a2  b2 

x2 y2 225 16
2
- = 1 2 - = 1  a2 = 161- 4 481  
a 81- a2 a 81- a2

Per tant, l’equació de la hipèrbole demanada és:

x2 y2
- = 1 
161 - 4 481 4 481 - 80

71. Escriu l’equació del lloc geomètric dels punts que tenen 10 com a diferència de les distàncies
a (0, –12) i a (0, 12).

 
 
467
 
 
 
 
 
 

Llocs geomètrics.
Llocs Còniques
geomètrics. Còniques 8
 
 
 
 
 
 

72. Considerem els punts A(-5, -1) i A' (–5, –11). Quina equació té el lloc geomètric que formen els
punts la diferència de distàncies dels quals a A i A' és 8?

Sigui P(x, y) un punt del lloc geomètric, aleshores d(A, P) - d(A', P) = 8.

( x + 5)2 + ( y + 1)2   ( x + 5)2 + ( y + 11)2   8 

→ ( x + 5)2 + ( y + 1)2   8   ( x + 5)2 + ( y + 11)2 → 

→ ( x + 5)2 + ( y + 1)2   82  (x  5)2  (y  11)2  16 ( x + 5)2 + ( y + 11)2 → 

→ 5y  46  4 ( x + 5)2 + ( y + 11)2 → (5y  46)2  16(x  5)2  16(y  11)2→ 

→ 25y2  460y  2 116  16x2  160x  400  16y2  352y  1 936 → 

→ 9(y2  12y)  16(x2  10x)  220  0 → 9(y2  12y  36)  16(x2  10x  25)  220  324  400 → 

9( y + 6)2 16( x + 5)2


→ 9(y  6)2  16(x  5)2  144 → - = 1 
144 144

( y + 6)2 ( x + 5)2
El lloc geomètric és una hipèrbole amb eix paral·lel a OY i equació: - =1 
16 19

73. Calcula l’equació de la paràbola amb vèrtex a l’origen de coordenades, que passa pel punt (3, –
6) i l’eix de la qual coincideix amb l’eix d’abscisses.
L’equació de la paràbola amb V(0, 0) i eix en l’eix X és de la forma: y2 = 2px

Atès que (3, 6) pertany a la paràbola: 

(6)2  2p3 → p  6 

Equació de la paràbola: y2  12x 

74. Determina el focus i la directriu de les paràboles següents i representa-les gràficament:


a) y2 = 10x d) x2 = y
b) y2 = 7x e) y2 = –10x
c) x2 = 6y f ) x2 = –6y

a) 2p = 10 → p = 5 → � � � ��


Directriu: � � �

468
 
 
 
 
 
 

Llocs geomètrics. Còniques 88


 
 
 
 
 
 
� �
b) 2p = 7 → p = → � � , 0�
� �


Directriu: � � �

3
c) 2p = 6 → p = 3 → � �0, � �
2


Directriu: � � �

� 1
d) 2p = 1 → p = → � �0, � �
� 4


Directriu: � � �

5
e) 2p = -10 → p = -5 → � �� , 0�
2


Directriu: � �

3
e) 2p = -6 → p = -3 → � �0, � �
2

469
 
 
 
 
 
 

Llocs geomètrics.
Llocs Còniques
geomètrics. Còniques 8
 
 
 
 
 
 


Directriu: � �

 
75. Calcula l’equació de la paràbola en cadascun dels casos següents:
a) Focus a (5, 0) i la directriu és x = –5.
b) Focus a (0, 2) i la directriu és y = –2.
a)  P(x, y) punt de la paràbola → d(P, F)  d(P, s) → ( x - 5)2 + y 2   │x  5│ →

→ x2  10x  25  y2  x2  10x  25 → y2  20x 

b)  P(x, y) punt de la paràbola → d(P, F)  d(P, s) → x 2 + ( y - 2)2   │y  2│ → 

→ x2  y2  4y  4  y2  4y  4 → x2  8y 

76. Determina l’equació reduïda de la paràbola amb vèrtex a l’origen de coordenades que compleix
les condicions de cada cas:
a) Passa pel punt (–3, 8) i la directriu és horitzontal.
b) Passa pel punt (5, –4) i la directriu és vertical.
a) L’equació de la paràbola és del tipus: x2 = 2py
� �
Atès que passa pel punt (–3, 8): 9 = 2p · 8 → p = → x2 = y
�� �
b) L’equació de la paràbola és del tipus: y2 = 2px
� ��
Atès que passa pel punt (5, –4): 16 = 2p · 5 → p = → y2 = x
� �

 
77. Troba l’equació de la paràbola que té el vèrtex en el punt V(–1, 3), passa pel punt (2, –2) i l’eix
és paral·lel a l’eix Y.
L’equació és del tipus:

(x  1)2  2p(y  3) 

Atès que (2, 2) pertany a la paràbola:

9
(2  1)2  2p(2  3) → p   -  
10

Equació de la paràbola:

9
(x  1)2   - ( y - 3) → 5x2  10x  9y  22 
5

78. Determina els vèrtexs, els focus i les directrius de les paràboles següents:
470
 
 
 
 
 
 

Llocs geomètrics. Còniques 88


 
 
 
 
 
 

a) y2 = 2(x – 3) d) (x – 3)2 = 8(y + 1)


b) (y – 1)2 = 4(x – 4) e) x2 = –4(y + 1)
c) x2 = 6(y – 2) f) (y + 3)2 = –8(x – 1)
æ7 ö 5
a)  V(3, 0)     2p  2 → p  1 → F ççç , 0 ÷÷÷         Directriu: x    
è2 ø 2

b)  V(4, 1)     2p  4 → p  2 → F(5, 1)         Directriu: x  3 

æ 7 ÷ö 1
c)  V(0, 2)     2p  6 → p  3 → F ççç 0, ÷        Directriu: y    
è 2 ÷ø 2

d)  V(3, 1)      2p  8 → p  4 → F(3, 1)         Directriu: y  3 

e)  V(0, 1)      2p  4 → p  2 → F(0, 2)       Directriu: y  0 

f) V(1, 3)      2p  8 → p  4 → F(1, 3)      Directriu: x  3 

 
79. Calcula els vèrtexs, els focus i les directrius de les paràboles següents:
a) (x – 5)2 = 14(y – 3) d) (x + 4)2 = –4(y + 7)
b) (y + 6)2 = 24(x – 1) e) (y – 2)2 = –18x
c) (y + 4)2 = –8(x – 6) f) x2 = –9(y + 1)
æ 7 ö÷ æ 13 ö 7 1
a)  V(5, 3)     2p  14 → p  7 → Focus: F ççç5, 3 + ÷  F ççç5, ÷÷÷     Directriu: y  3   → y   -  
è 2 ø÷ è 2ø 2 2

12
b)  V(1, 6)    2p  24 → p  12 → Focus:  F æççç1 + 12 , - 6 ö÷÷÷ → F(7, 6)  Directriu: x  1   → x  5 
è 2 ø 2

-4
c)  V(6, 4)    2p   8 → p   4 → Focus:  F æççç6 + -4 , - 4ö÷÷÷ → F(4, 4)  Directriu: x  6   → x  8 
è 2 ø 2

-2
d) V(4, 7)    2p   4 → p   2 → Focus:  F æççç- 4, - 7 + - 2 ö÷÷÷ → F(4, 8)  Directriu: y   7   → y  6 
è 2 ø 2

-9 9
e) V(0, 2)     2p   18 → p   9 → Focus:  F æççç0 + - 9 , 2ö÷÷÷  F æççç- 9 , 2÷÷÷ö   Directriu: x  0   → x    
è 2 ø è 2 ø 2 2

-2
f)  V(0, 1)    2p   4 → p   2 → Focus:  F æççç0, - 1 + - 2 ö÷÷÷ → F(0, 2)  Directriu: y  1   → y  0 
è 2 ø 2

 
80. Troba l’equació de la paràbola en cada cas.
a) Vèrtex a (–5, 8) i focus a (–5, 2).
b) Vèrtex a (3, 7) i focus a (0, 7).
c) Focus a (3, 5) i la directriu és x = 10.
d) Vèrtex a (2, 3) i la directriu és x = –3.
a) Equació del tipus: (x  5)2  2p(y  8) 

æ pö p
F ççç- 5, 8 + ÷÷÷ → 2  8   → p   12 
è 2ø 2

Equació de la paràbola:

471
 
 
 
 
 
 

Llocs geomètrics.
Llocs Còniques
geomètrics. Còniques 8
 
 
 
 
 
 

(x  5)2  2(12)(y  8) → (x  5)2   24(y  8) → x2  10x  24y  167  0 

b)  Equació del tipus: (y  7)2  2p(x  3) 

æ p ö p
F ççç 3 + , 7÷÷÷ → 0  3   → p  6 
è 2 ø 2

Equació de la paràbola:

 (y  7)2  2(6)(x  3) → y2  14y  12x  13  0 

c) Sigui P(x, y) un punt de la paràbola, aleshores d(P, F)  d(P, s): 

( x - 3)2 + ( y - 5)2   │x  10│ → x2  6x  9  y2  10y  25  x2  20x  100 

Equació de la paràbola:

y2  10y  14x  66  0 

9
d)  s: x  3 → Equació del tipus: (y  3)2  2p(x  2) y s: x  2   -  
2

p
3  2   - → p  10 
2

Equació de la paràbola:

 (y  3)2  2  10(x  2) → (y  3)2  20(x  2) → y2  6y  20x  49  0 

 
81. Calcula l’equació de la paràbola que té l’eix paral·lel a l’eix Y i que passa pels punts A(2, –4),
B(–2, 4) i C(–3, 2).
La paràbola és del tipus (x  h)2  2p(y  k), i V(h, k) 

Atès que A, B i C pertanyen a la paràbola:

(2 – h) = 2 p ( –4 – k )  h2 – 4 h + 8 p + 2 pk + 4 = 0üïïï
2

ï
(  –2 – h) = 2 p (4 – k )  h2 + 4 h – 8 p + 2 pk + 4 = 0ïýï
2  
ï
( –3 – h) = 2 p (2 – k )  h2 + 6 h – 4 p + 2 pk + 9 = 0ïïïþ
2

Es resta la primera equació a les altres dues:

5 5
8h – 16 p = 0  ïüï h = 2p  10 ⋅ 2p – 12p + 5 = 0  p = – h = –
ý 8 4 
10 h – 12p + 5 = 0ïïþ

Se substitueix p i h a la primera equació:


2
æ –5 ÷ö æ –5 ö æ –5 ö æ –5 öæ –5 ö 89
ççç ÷÷ – 4 ççç ÷÷÷ + 8 ççç ÷÷÷ + 2ççç ÷÷÷ççç ÷÷÷ + 4 = 0  k =  
è4ø è4ø è8ø è 8 øè 4 ø 20
2
æ 5ö 5æ 89 ö
Equació de la paràbola:  ççç x + ÷÷÷ = – ççç y – ÷÷÷  
è 4ø 4è 20 ø

472
 
 
 
 
 
 

Llocs geomètrics. Còniques 88


 
 
 
 
 
 

82. Determina l’equació del lloc geomètric dels punts que equidisten del punt P(3, 1) i de la recta r:
3x – 4y + 5 = 0.

 
Sigui P(x,y) un punt del lloc geomètric. 

 
 
83. En cadascun dels casos següents, escriu l’equació de la circumferència corresponent:
� √�
a) C(0, 0); r = e) C(2, –1); r =
� �

b) C(0, 7); r = 14 f ) C(6, 8); r = √�



c) C(5, 0); r = 7 g) C(12, 4); r =

d) C(7, –1); r = 5 h) C(–4, –3); r = 8
2
æ3ö
a)  x2  y2   ççç ÷÷÷ → Equació de la circumferència: 25x2  25y2  9 
è5ø

b)  x2  (y  7)2  142 → Equació de la circumferència: x2  y2  14y  147  0 

c)  (x  5)2  y2  72 → Equació de la circumferència: x2  y2  10x  24  0 

d)  (x  7)2  (y  1)2  52 → Equació de la circumferència: x2  y2  14x  2y  25  0 

2
æ 2 ö÷
e)  (x  2)2  (y  1)2   ççç ÷ → Equació de la circumferència: 9x2  9y2  36x  18y  43  0 
è 3 ÷ø÷

2
f)  (x  6)2  (y  8)2   ( 2 ) → Equació de la circumferència: x2  y2  12x  16y  98  0 

2
æ 1ö
g)  (x  12)2  (y  4)2   ççç ÷÷÷ → Equació de la circumferència: 4x2  4y2  96x  32y  639  0 
è2ø

h)  (x  4)2  (y  3)2  82 → Equació de la circumferència: x2  y2  8x  6y  39  0 

84. Determina l’equació de la circumferència en cadascun dels casos següents:


a) Centre en (–2, –5) i passa per (6, 1).

473
 
 
 
 
 
 

Llocs geomètrics.
Llocs Còniques
geomètrics. Còniques 8
 
 
 
 
 
 

b) Centre en (7, 2) i passa per (0, 0).


c) Tangent a l’eix Y i centre en (4, –2).
d) Tangent a l’eix X i centre en (0, –3).
e) Els extrems d’un diàmetre són (5, –4) i (–5, 1).
f ) Els punts (7, 6) i (1, 3) són diametralment oposats.
a) L’equació és del tipus: (x  2)2  (y  5)2  r2 

(6, 1) pertany a la circumferència: (6  2)2  (1  5)2  r2 → r  10 

Equació de la circumferència: (x  2)2  (y  5)2  102 → x2  y2  4x  10y  71  0 

b) L’equació és del tipus: (x  7)2  (y  2)2  r2 

(0, 0) pertany a la circumferència: (7)2  (2)2  r2 → r   53  

2
Equació de la circumferència: (x  7)2  (y  2)2   ( 53 ) → x2  y2  14x  4y  0 

c) L’equació és del tipus: (x  4)2  (y  2)2  r2 

d(C, OY)  4  r → Equació de la circumferència: (x  4)2  (y  2)2  42 → x2  y2  8x  4y  4  0 

d) L’equació és del tipus: x2  (y  3)2  r2 

d(C, OX)  3  r → Equació de la circumferència: x2  (y  3)2  32 → x2  y2  6y  0 

e) Longitud del diàmetre: d((5, 4),(5, 1))   (-5 - 5)2 + (1+ 4)2    5 5 → r   5 5


 
2

æ 5 + (- 5) - 4 + 1ö÷ æ 3ö æ 3ö
çç , ÷    ççç0, - ÷÷÷ → C ççç 0, - ÷÷÷  
çè 2 2 ø÷ è 2ø è 2ø

2 2
æ 3ö æ 5 5 ö÷
Equació de la circumferència: x2   ççç y + ÷÷÷    ççç ÷ → 4x2  4y2  12y  116  0 
è 2ø è 2 ÷÷ø

f) Longitud del diàmetre: d((7, 6), (1, 3))   (1- 7)2 + (3 + 6)2    3 13 → r   3 13


 
2

æ 7 + 1 -6 + 3 ÷ö æ 3ö æ 3 ö÷
çç , ÷    çç4, - ÷÷÷ → C ççç 4, - ÷ 
çè 2 2 ÷ø çè 2ø è 2 ÷ø
2
æ 3ö æ 3 13 ö÷2

Equació de la circumferència: (x  4)2   ççç y + ÷÷÷    ççç ÷ → 4x2  4y2  32x  12y  44  0 


è 2 ø è 2 ÷ø÷

 
85. Calcula l’equació d’una circumferència amb centre en (–1, 6) i que passa pel punt (3, –3).
Digues si el punt (–2, –8) està situat en aquesta circumferència.
L’equació de la circumferència és del tipus: ( x + 1)2 + (y – 6)2 = r2
Si passa pel punt (3, -3): ( 3 + 1)2 + (– 3 – 6)2 = 97 → r = √97
L’equació simplificada és: x2 + y2 + 2x – 12y – 60 = 0
Substituïm: (–2)2 + (–8)2 + 2(–2) – 12(–8) – 60 = 100 � 0
(–2, –8) No pertany a la circumferència.

474
 
 
 
 
 
 

Llocs geomètrics. Còniques 88


 
 
 
 
 
 

86. Troba en cada cas l’equació de la circumferència que compleix aquestes condicions:
a) Passa pels punts (–3, 7) i (11, 3) ) i el radi és de 2 unitats.
b) Passa pels punts (5, 4) i (–2, 3) ) i el radi és de 5 unitats.
a) La distància entre els dos punts és més gran que 4, així doncs no poden ser els dos dins una
circumferència de diàmetre 4.

b) L’equació és de la forma: (x  a)2  (y  b)2  25 

2 2
Passa per (5, 4)  (5 – a ) + (4 – b) = 25  a 2 + b2 – 10a – 8b + 16 = 0 üï
ï
ý 
Passa per ( –2, 3)  ( –2 – a ) + (3 – b) = 25  a + b + 4a – 6b – 12 = 0ïïïþ
2 2 2 2

Restem la segona equació a la primera:

14a  2b  28  0 → b  14  7a 

Substituïm a la primera equació:

a2  (14  7a)2  10a  8(14  7a)  16  0 → a2  3a  2  0 → a1  1 y a2  2 

Si a  1 → b  7 i l’equació de la circumferència és (x  1)2  (y  7)2  25 → x2  y2  2x  14y  25  0 

Si a  2 → b  0 i l’equació de la circumferència és (x  2)2  y2  25 → x2  y2  4x  21  0 

 
87. Estudia si les equacions següents corresponen a circumferències. En cas afirmatiu, calcula’n
el centre i el radi.
a) x2 + y2 – 9x + 6y + 41 = 0 d) x2 + y2 – x + 12y + 41 = 0
b) x2 + y2 + 8x – 10y + 3 = 0 e) x2 + y2 + 2 x + 6y + 10 = 0
c) x2 + y2 + 10x + 12y + 33 = 0 f) x2 + y2 + 8x + 10y + 41 = 0
2
æ 9ö 47
a)  ççç x - ÷÷÷   (y  3)2   - → No és una circumferència. 
è 2ø 4

b)  (x  4)2  (y  5)2  38 → Correspon a una circumferència amb C(4, 5) i r   38 . 

c)  (x  5)2  (y  6)2  28 → Correspon a una circumferència amb C(5, 6) i r   2 7 . 

2
æ 1ö 19
d)  ççç x - ÷÷÷   (y  6)2   - → No és una circumferència. 
è 2ø 4

e)  (x  1)2  (y  3)2  0 → No és una circumferència. 

f)  (x  4)2  (y  5)2  0 → No és una circumferència. 

 
88. Calcula en els casos següents l’equació de la circumferència que passa pels punts A, B i C:
a) A(2, 8), B(2, –5) i C(0, 0)
b) A(4, 0), B(–1, –2) i C(1, 4)
c) A(0, 2), B(1, 7) i C(–3, –4)

475
 
 
 
 
 
 

Llocs geomètrics.
Llocs Còniques
geomètrics. Còniques 8
 
 
 
 
 
 

a) L’equació és del tipus: x2  y2  Dx  Ey  F  0 

Atès que D, E, F pertanyen a la circumferència:

22  82  D2  E8  F  0 → 2D  8E  F  68  0 

22  (5)2  D2  E(5)  F  0 → 2D  5E  F  29  0 

F  0 

2D + 8 E + 68 = 0ü
ï
ï
ý 
2D – 5E + 29 = 0ïï
þ

Restem la segona equació a la primera: 13E  39  0 → E  3 

Substituïm en la primera: 2D  8  (3)  68  0 → D  22 

Equació de la circumferència: x2  y2  22x  3y  0 

b) L’equació és del tipus: x2  y2  Dx  Ey  F  0 

Atès que D, E, F pertanyen a la circumferència:

42  0  4D  0  F  0 → 4D  F  16  0 → F  4D  16 

(1)2  (2)2  D(1)  E(2)  F  0 → D  2E  F  5  0 

1  42  D  E4  F  0 → D  4E  F  17  0 

Substituïm en les dues equacions:

D  2E  4D  16  5  0 → 5D  2E  11  0 

D  4E  4D  16  17  0 → 3D  4E  1  0 

21 124 19
D   -         F         E   -  
13 13 13

21x 19 y 124
Equació de la circumferència: x2  y2           0 → 13x2  13y2  21x  19y  124  0 
13 13 13

c) L’equació és del tipus: x2  y2  Dx  Ey  F  0 

Atès que D, E, F pertanyen a la circumferència:

0  22  0  E2  F  0 → 2E  F  4  0 → F  2E  4 

1  72  D  E7  F  0 → D  7E  F  50  0 

(3)2  (4)2  D(3)  E(4)  F  0 → 3D  4E  F  25  0 

Substituïm en les dues equacions:

53 94
D  7E  2E  4  50  0 → D  5E  46  0      E   - → F    
3 3

476
 
 
 
 
 
 

Llocs geomètrics. Còniques 88


 
 
 
 
 
 
127
3D  4E  2E  4  25  0 → 3D  6E  21  0    D    
3

127 x 53 y 94
Equació de la circumferència:  x 2 + y 2 + –  + = 0 → 3x2  3y2  127x  53y  94  0 
3 3 3

 
89. Determina el punt de la circumferència de centre (2, 8) i radi 5 que es trobi més pròxim a
cadascun dels punts següents:
a) P(7, 18) b) Q(–1, 6) c) R(5, 4)
L’equació de la circumferència és: ( x - 2)2 + ( y - 8)2 = 25  x 2 + y 2 - 4 x - 16 y + 43 = 0  

a) La recta no passa per P i el centre de la circumferència és: 2x – y + 4 = 0 

� � � � � � �� � 1�� � �3 � �
� → � � � �� � 1 � � → � � � � √5 
� � �� � �

Les coordenades del punt més pròxim són: A(2+√5, 8+2√5) 

b) La recta que passa per Q i el centre de la circumferència és: 2x – 3y + 20 = 0 

� � � � � � �� � 1�� � �3 � � �� � 15√13
� �� � �� → 13� � � 5�� � 1�3 � � → � �  
�� 13
3

�����√��
Les coordenades del punt més pròxim són: M( , 24�1�√13) 
��

c) 52 + 42 – 4 · 5 – 16 · 4 +43 = 0 → R pertany a la circumferència i coincideix amb el punt demanat.  

 
90. Donada l’equació de la circumferència
x2 + y2 + 4x + 2y – 20 = 0
calcula l’equació de les rectes tangent i normal en el punt (2, 2).
A  4 → a  2            B  2 → b  1            Centre  (2, 1) 

C  20 → Atès que C  a2  b2  r2 → r  5 

La recta normal passa pel punt (2, 2) i el centre (2, 1) → 3x  4y  2  0 

La recta tangent passa pel punt (2, 2) i és perpendicular a la normal → 4x  3y  14  0 

 
91. Calcula l’equació de les rectes que passen per P(5, –3) i que són tangents a la circumferència
x2 + y2 = 1.

477
 
 
 
 
 
 

Llocs geomètrics.
Llocs Còniques
geomètrics. Còniques 8
 
 
 
 
 
 

 
L’equació de la tangent és del tipus: y  Ax  B 

Atès que és tangent a la circumferència, la intersecció entre la recta i la circumferència és un únic


punt:

x2  (Ax  B)2  1 → (1  A2)x2  2ABx  B2  1  0 

Discriminant: (2AB)2  4(1  A2)(B2  1)  0 → B2  A2  1  0 

Atès que P(5, 3) pertany a la tangent: 3  A5  B → B  3  5A 

Substituïm B en l’equació del discriminant es té 12A2  15A  4  0, i resolem aquesta equació:

- 15 + 33 3 - 5 33 - 15 - 33 3 + 5 33
A=  B= A= B=  
24 24 24 24

  33
    3 - 5   33
Rectes tangents:  y = - 15 + x+  24 y = (- 15 + 33 ) x + (3 - 5 33 )  
24 24

- 15 - 33 3 + 5 33
Rectes tangents: y= x+  24 y = (- 15 - 33 ) x + (3 + 5 33 )  
24 24

92. Determina l’equació de la recta tangent a la circumferència de radi 3 i centre en (5, –2), que és
paral·lela a la recta 2 x + y – 11 = 0.
Les rectes paral·leles a la recta donada són del tipus: 2x + y + k = 0

�� � 5�� � �� � ��� � � � � � � � � ��� � �� � �� � �


� →� →
�� � � � � � � � � ��� � �

→5� � � ��� � ���� � � � � �� � �� � ��

Si la recta és tangent, aleshores la intersecció amb la circumferència és un sol punt, així doncs
l’equació de segon grau ha de tenir una solució única.

��� � ���� � ���� � � �� � ��� � � → � � � ��� � �� � � → � � �� � �√5

�� � � � � � �√5 � �
Les dues rectes que compleixen les condicions són:�
�� � � � � � �√5 � �

 
93. Determina l’equació de la circumferència de centre (1, –3) que és tangent a la recta 3x + y – 2 =
0.

478
 
 
 
 
 
 

Llocs geomètrics. Còniques 88


 
 
 
 
 
 

3 -3 -2 2 10
r = d(C, t ) = = =  
32 + 12 10 5

2
L’equació de la circumferència és del tipus: (x  1)2  (y  3)2   → 5x2  5y2  10x  30y  48  0 
5

 
94. Troba l’equació de la circumferència de centre (3, –4) que és tangent a la recta següent:
3x + 4y – 18 = 0
9 - 16 - 18
r  d(C, t)     5 
32 + 42

L’equació de la circumferència és del tipus: (x  3)2  (y  4)2  25 → x2  y2  6x  8y  0 


 
95. Calcula l’equació de la circumferència de centre (4, –2) que és tangent a la recta x – 3y = – 5.

4+6+5 15 3 10
r = d ( C, t ) = = =
12 + 3 2 10 2
 
45
L’equació de la circumferència és del tipus: (x  4)2  (y  2)2   → 2x2  2y2  16x  8y  5  0 
2

 
96. Troba l’equació de la circumferència tangent a la recta x + y + 5 = 0 en (–3, –2) i que té el centre
sobre l’eix Y.
0+k +5 k +5 k +5 2
C (0, k )  r = d (C, t ) = = =  
2
1 +1 2
2 2

2 2
2(k + 5) (k + 5)
L’equació de la circumferència és del tipus: x   (y  k)    2 2
→ x   (y  k)   
2 2
 
4 2

2
(k + 5)
(3, 2) pertany a la circumferència: (3)2  (2  k)2   → k  1 
2

Equació de la circumferència: x2  (y  1)2  18 → x2  y2  2y  17  0 

 
97. Calcula l’equació de la circumferència que passa pels punts A(3, 9) i B(8, 4) i que té el centre a
la recta d’equació 2 x – 5y + 14 = 0.
El centre equidista dels punts A i B, aleshores, es troba a la mediatriu del segment AB. Sigui P(x, y)
un punt de aquesta mediatriu, aleshores d(A, P)  d(B, P). 

( x - 3)2 + ( y - 9)2    ( x - 8)2 + ( y - 4)2 → x2  6x  y2  18y  90  x2  16x  y2  8y  80 → x  y  1  0 

479
 
 
 
 
 
 

Llocs geomètrics.
Llocs Còniques
geomètrics. Còniques 8
 
 
 
 
 
 

El centre és la intersecció d’aquesta recta i la recta donada:

ü
x – y + 1 = 0 ï
ï
ý → C(3, 4) → r   (3  3)   (4  9)   25 
2 2 2
2 x – 5 y + 14 = 0 ï
ï
þ

Equació de la circumferència: (x  3)2  (y  4)2  25 → x2  y2  6x  8y  0 

 
98. Determina l’equació de la circumferència tangent a la recta 2 x – 5y + 1 = 0 en el punt (2, 1) i
que té el centre a la recta d’equació x + y = 0.
El centre és sobre la recta x  y  0, per tant, les seves coordenades són de la forma C(h, h). 
2 h + 5h + 1 7h + 1
C(h, h) → r  d(C, t)       
22 + 52 29

2
(7 h + 1)
L’equació de la circumferència és del tipus: (x  h)2  (y  h)2    
29
2
(7 h + 1)
(2, 1) pertany a la circumferència: (2  h)2  (1  h)2   → h2  8h  16  0 → h  4 
29

Equació de la circumferència: (x  4)2  (y  4)2  29 → x2  y2  8x  8y  3  0 


 
99. Calcula l’equació de la circumferència tangent a la recta x + y + 4 = 0 en el punt (–3, –1) i que té
el centre a la recta d’equació 3x – 4y + 2 = 0.
æ ö
El centre és sobre la recta 3x  4y  2  0, per tant, les seves coordenades són de la forma  C ççç h, 3 h + 2 ÷÷÷ . 
è 4 ø

3h + 2
h+ +4
æ 3 h + 2 ÷ö 4 7 h + 18
C ççç h, ÷ → r  d(C, t)       
è 4 ÷ø 12 + 12 4 2

2
æ 3h + 2 ÷ö
2
(7h + 18)
L’equació de la circumferència és del tipus: (x  h)2   ççç y - ÷÷     
è 4 ø 32

2
æ 3 h + 2 ö÷
2
(7h + 18)
(3, 1) pertany a la circumferència: (3  h)2   ççç-1- ÷    → h2  12h  36  0 → h  6 
è 4 ÷ø 32

Equació de la circumferència: (x  6)2  (y  4)2  18 → x2  y2  12x  8y  34  0 

100. Determina l’equació de la circumferència tangent a la recta 3x – 2y + 4 = 0 en el punt (2, 5) i que


té el centre sobre l’eix X.
3h + 4 3h + 4
C(h, 0) → r  d(C, t)       
3 2 + 22 13

2
( 3 h + 4)
L’equació de la circumferència és del tipus: (x  h)2  y2    
13

2
( 3 h + 4) 19
(2, 5) pertany a la circumferència: (2  h)2  52   → h    
13 2

480
 
 
 
 
 
 

Llocs geomètrics. Còniques 88


 
 
 
 
 
 
2
æ 19 ö 4225
Equació de la circumferència: ççç x - ÷÷÷   y2   → 52x2  52y2  988x  468  0 
è 2ø 52

101. Calcula l’equació de la circumferència tangent a la recta 4x + 3y – 4 = 0 en el punt (4, –4) i que
té el centre a la recta d’equació x – y – 7 = 0.
El centre és en la recta x  y  7  0, per tant, les seves coordenades són de la forma C(h, h  7). 

4h + 3( h - 7) - 4 7 h – 25
C(h, h  7) → r  d(C, t)       
2
4 +3 2 5

2
(7h - 25)
L’equació de la circumferència és del tipus: (x  h)2  (y  h  7)2    
25

2
(7h - 25)
 (4, 4) pertany a la circumferència: (4  h)2  (4  h  7)2   → h  0 
25

Equació de la circumferència: x2  (y  7)2  25 

102. Troba els punts d’intersecció de la circumferència x2 + y2 – 4x + 6y – 28 = 0 amb aquestes


rectes:
a) r: x + 9y – 16 = 0 b) s: x + y + 2 = 0 c) t: 4x – 5y – 23 = 0

a)  
Els punts d’intersecció són: (7, 1) i (–2, 2) 

b)  

Els punts d’intersecció són: (6, ‐8) i (‐3, 1) 

c)  

Els punts d’intersecció són: (7, 1) i (‐3, ‐7) 

103. Segons el valor de m, estudia la posició relativa de la recta y = mx + 2 respecte de la


circumferència (x – 3)2 + (y – 2)2 = 4.
(x  3)2  (mx  2  2)2  4 → (1  m2)x2  6x  5  0 

Discriminant: 36  4  (1  m2)  5  16  20m2 

4
16  20m2  0 → m2   → m    2 5 → Tangent.
5 5

481
 
 
 
 
 
 

Llocs geomètrics.
Llocs Còniques
geomètrics. Còniques 8
 
 
 
 
 
 

16  20m2  0 → m   - 2 5
y m   2 5
→ Exterior. 
5 5

2 5
16  20m2  0 → -   m   2 5
→ Secant.
5 5

 
104. En funció del valor que prengui m, estudia la posició relativa de la recta y = mx + 2 respecte de
la circumferència x2 + 4x + y2 – 6y – 12 = 0.
x2  4x  (mx  2)2  6(mx  2)  12  0 → (1  m2)x2  (4  2m)x  20  0 

Discriminant: (4  2m)2  4  (20)  (1  m2)  84m2  16m  96 

84m2  16m  96  0 → No té solució. 

84m2  16m  96  0 → No té solució. 

84m2  16m  96  0 → Sempre → La recta és secant per qualsevol valor de m.

 
105. Calcula el valor que ha de tenir k perquè la recta r i la circumferència C siguin tangents.
a) r: 3x + 2y + k = 0 C: x2 + y2 – 6x + 8y – 6 = 0
b) r: 4x + 2y + k = 0 C: x2 + y2 + 6x + 10y – 1 = 0
2y + k
a)  x   -  
3

2
æ ö
çç 2 y + k ÷÷   y2  6 æçç 2 y + k ö÷÷   8y  6  0 → 13y2  (108  4k)y  k2  18k  54  0 
çè 3 ÷ø çè 3 ÷ø

Perquè la recta sigui tangent a la circumferència, la solució ha de ser única:

Discriminant: (108  4k)2  4  13  (k2  18k  54)  0 → k   -1 403  

2y + k
b)  x   -  
4

2
æ 2 y + k ö÷ æ 2 y + k ö÷
çç- ÷   y2  6 çç- ÷   10y  1  0 → 20y2  (112  4k)y  k2  24k  16  0 
èç 4 ÷ø çè 4 ÷ø

Perquè la recta sigui tangent a la circumferència, la solució ha de ser única:

Discriminant: (112  4k)2  4  20  (k2  24k  16)  0 → k   22  10 7  

   

482
 
 
 
 
 
 

Llocs geomètrics. Còniques 88


 
 
 
 
 
 

106. Determina el valor del coeficient C en la circumferència x2 + y2 + 10x + 2y + C = 0 perquè sigui


tangent a la recta 2x + 3y = 0.
��� � ���� � ����� � ���� � ��� � ��
� → ��� � � ��� � �� � �
�� � �� � �

Si la recta és tangent, aleshores la intersecció amb la circumferència és un sol punt, així doncs
l’equació de segon grau ha de tenir una única solució.

6084 – 468C = 0 → C = 13

 
107. Estudia la posició relativa dels parells de circumferències següents:
a) C1: x2 + y2 – 8x + 5y – 7 = 0 d) C1: x2 + y2 + 4x + 2y + 4 = 0
C2: x2 + y2 – 8x + 5y – 15 = 0 C2: x2 + y2 – 2x – 8y – 32 = 0
b) C1: x2 + y2 – 7x – 11y – 3 = 0 e) C1: x2 + y2 + 4x – 2y – 15 = 0
C2: x2 + y2 – 7x – 10y + 2 = 0 C2: x2 + y2 – 12 x – 10y + 45 = 0
c) C1: x2 + y2 + 8x + 4y – 11 = 0 f ) C1: x2 + y2 + 8x – 4y + 18 = 0
C2: x2 + y2 – 6x – 8y + 21 = 0 C2: x2 + y2 – 2x + 6y – 22 = 0
2
æ 5ö 117 æ 5ö 117
a)  C1: (x  4)2   ççç y + ÷÷÷    → Centre ççç4, – ÷÷÷ y r1    
è 2ø 4 è 2ø   2

2
æ 5ö 149 æ 5 ö÷ 149
C2: (x  4)2   ççç y + ÷÷÷    → Centre çç4, – ÷  y r2    
è 2ø 4 çè 2 ÷ø 2

El centre és el mateix i r2  r1 → Són concèntriques. 


2 2
æ 7ö æ 11ö 182
b)  C1:  ççç x – ÷÷÷    ççç y – ÷÷÷  → Centre  æççç 7 , 11ö÷÷÷ y r1   182
 
è 2ø è 2ø 4 è2 2 ø 2
 
2
æ 7ö 141 æ7 ö
C2:  ççç x – ÷÷÷   (y  5)2   → Centre ççç , 5÷÷÷  y r2   141  
è 2ø 4 è2 ø 2

2
æ 11ö÷ 1
d(C1, C2)   0 + ççç5 - ÷    r1  r2 → Són interiors.
è 2 ÷ø 2

c)  C1: (x  4)2  (y  2)2  31 → Centre (4, 2) y r1   31  

C2: (x  3)2  (y  4)2  4 → Centre (3, 4) y r2  2 

d(C1, C2)   (-4 - 3)2 + (-2 - 4)2    85   r1  r2 → Són exteriors. 

d)  C1: (x  2)2  (y  1)2  1 → Centre (2, 1) y r1  1 

C2: (x  1)2  (y  4)2  49 → Centre (1, 4) y r2  7 

d(C1, C2)   (1+ 2)2 + (4 + 1)2   5,83  r2  r1 → Són interiors. 

e)  C1: (x  2)2  (y  1)2  20 → Centre (2, 1) y r1   2 5  

C2: (x  6)2  (y  5)2  16 → Centre (6, 5) y r2  4 

483
 
 
 
 
 
 

Llocs geomètrics.
Llocs Còniques
geomètrics. Còniques 8
 
 
 
 
 
 

d(C1, C2)   (6 + 2)2 + (5 - 1)2    4 5   r2  r1 → Són exteriors. 

f)  C1: (x  4)2  (y  2)2  2 → Centre (4, 2) y r1   2  

C2: (x  1)2  (y  3)2  32 → Centre (1, 3) y r2   32  

d(C1, C2)   (1+ 4)2 + (-3 - 2)2 = 5 2   r2  r1 → Són tangents exteriors. 

108. Calcula els punts d’intersecció de les circumferències següents:


a) C1: x2 + y2 – 6x – 6y + 13 = 0 b) C1: x2 + y2 + 6x – 4y + 8 = 0
C2: x2 + y2 – 8x + 11 = 0 C2: x2 + y2 + 4x – 10y + 24 = 0
a) C1: (x  3)2  (y  3)2  5 → Centre (3, 3) y r1   5  

C2: (x  4)2  y2  5 → Centre (4, 0) y r1   5  

d(C1, C2)   (4 - 3)2 + (0 - 3)2    10   r1  r2 y  10   r1  r2 → Secants (dos punts d’intersecció). 

Es resol el sistema format per C1 i C2: 

Restem C2 a C1: x  3y  1  0 → x  3y  1 

Substituïm en C2: (3y  1)2  y2  8(3y  1)  11  0 → y2  3y  2  0 → y1  1, y2  2 → x1  2, x2  5 

Punts d’intersecció: (2, 1) i (5, 2) 

b) C1: (x  3)2  (y  2)2  5 → Centre (3, 2) y r1   5  

C2: (x  2)2  (y  5)2  5 → Centre (2, 5) y r1   5  

d(C1, C2)   (-2 + 3)2 + (5 - 2)2    10   r1  r2 y  10   r1  r2 → Secants (dos punts d’intersecció). 

Es resol el sistema format per C1 i C2: 

Restem C2 a C1: x  3y  8  0 → x  3y  8 

Substituïm en C1: (8  3y)2  y2  6(8  3y)  4y  8  0 → y2  7y  12  0 → y1  3, y2  4 → x1  1, x2  4 

Punts d’intersecció: (1, 3) i (4, 4) 

109. Estudia la posició del punt P(–3, 2) respecte de la circumferència (x – 1)2 + (y + 3)2 = k, segons
el valor de k.

C(1, 3)      r   k     d(P, C)   (1+ 3)2 + (-3 - 2)2    41  

Si k  41 → P pertany a la circumferència.

Si k  41 → P és interior.

Si 0  k  41 → P és exterior. 

484
 
 
 
 
 
 

Llocs geomètrics. Còniques 88


 
 
 
 
 
 

 
110. Troba el lloc geomètric dels centres de les circumferències que passen, alhora, pels punts (4,
1) i (–2, 5). De quina figura es tracta?
Els centres de les circumferències són a la mateixa distència tots dos punts; per tant, formen la
mediatriu del segment que determinen.

ඥሺ‫ ݔ‬െ Ͷሻଶ ൅ ሺ‫ ݕ‬െ ͳሻଶ ൌ ඥሺ‫ ݔ‬൅ ʹሻଶ ൅ ሺ‫ ݕ‬െ ͷሻଶ ՜ ‫ ݔ‬ଶ െ ͺ‫ ݔ‬൅ ͳ͸ ൅ ‫ ݕ‬ଶ െ ʹ‫ ݕ‬൅ ͳ
ൌ ‫ ݔ‬ଶ ൅ Ͷ‫ ݔ‬൅ Ͷ ൅ ‫ ݕ‬ଶ െ ͳͲ‫ ݕ‬൅ ʹͷ ՜ ͵‫ ݔ‬െ ʹ‫ ݕ‬൅ ͵ ൌ Ͳ 
 
111. Determina el lloc geomètric dels punts tals que la seva diferència de quadrats als punts (1, –2) i
(0, 3) és sempre de 5 unitats.
Sigui P(x, y) un punt del lloc geomètric, aleshores  ( x - 1)2 + ( y + 2)2 - x 2 - ( y - 3)2 = 5  → 2x  10y  9  0 

 
112. Calcula la longitud de la corda que forma la recta x + 2y – 5 = 0 sobre la circumferència de x2 +
y2 = 25.
Determinar la intersecció de la recta i la circumferència:
x  5  2y 
(5  2y)2  y2  25 → y2  4y  0 → y1  0, y2  4 → x1  5, x2  3 
Els punts d’intersecció de la recta i la circumferència són A(5, 0) y B(3, 4). 

Longitud de la corda: d(A, B)   (5 + 3)2 + (0 + 4)2    4 5  


 
113. Calcula l’àrea de la circumferència que està determinada per l’equació següent:
x2 + y2 – 3x + 5y – 14 = 0
2 2
æ ö æ ö
çç x – 3 ÷÷ + çç y + 5 ÷÷ = 90 → r2   45  
èç 2 ø÷ çè 2 ÷ø 4 2

45p
Àrea = p⋅ r 2 =  
2

 
114. Troba l’àrea de la corona circular formada per les circumferències C1 i C2.
C1: x2 + y2 – 4x + 6y + 12 = 0
C2: x2 + y2 – 4x + 6y + 9 = 0
C1: (x  2)2  (y  3)2  1 → r12  1 → A1 = p ⋅ r 2 = p  

C2: (x  2)2  (y  3)2  4 → r22  4 → A2 = p ⋅ r 2 = 4p  

Àrea corona circular = A2 – A1 = 4 p – p = 3p   

   

485
 
 
 
 
 
 

Llocs geomètrics.
Llocs Còniques
geomètrics. Còniques 8
 
 
 
 
 
 

115. Escriu l’equació de la circumferència que passa pels punts A (1, –2) i B (5, –3) i que té el centre
a la recta r : x – y + 2 = 0.

 
Atès que el centre equidista dels punts, cerquem a la mediatriu de AB:

P(x, y) punt de la mediatriu: d(A, P)  d(B, P) → ( x - 1)2 + ( y + 2)2 = ( x - 5)2 + ( y + 3)2 → 8x  2y  29  0 

El centre és el punt d’intersecció de la mediatriu i la recta donada:

8 x – 2 y – 29 = 0ïïü x = y – 2
ïý
ï 15 11 æ 11 15 ö
x – y + 2 = 0ïþï 8 ( y – 2) – 2 y – 29 = 0  y =  x =  C çç , ÷÷÷  
2 2 çè 2 2 ø

2 2
æ11 ö æ15 ö 442
r  d(A, C)   ççç - 1÷÷÷ + ççç + 2÷÷÷ =  
è2 ø è2 ø 4

2 2
æ 11ö æ 15 ö 221
Equació de la circumferència:  ççç x - ÷÷÷ + ççç y - ÷÷÷ =  
è 2ø è 2ø 2

 
116. Un triangle té els costats sobre les rectes d’equacions r: 2 x + y + 2 = 0, s: 2 x – y – 10 = 0 i t : x
+ 2y – 5 = 0. Calcula l’equació de la circumferència inscrita en aquest triangle.

 
Atès que (3, 4), (2, 6) pertanyen a r → r: 2x  y  2  0 

Atès que (5, 0) y (2, 6) pertanyen a s → s: 2x  y  10  0 

Atès que (5, 0) y (3, 4) pertanyen a t → t: x  2y  5  0 

El centre de la circumferència és la intersecció de les bisectrius de les interseccions de les rectes.

Sigui P(x, y) un punt de la bisectriu de l’angle format per les rectes r i t, aleshores d(P, r)  d(P, t): 

2x + y + 2 x + 2y - 5
2 2
    
2 +1 2 2 + 12

486
 
 
 
 
 
 

Llocs geomètrics. Còniques 88


 
 
 
 
 
 

La solució són dues equacions. Escollim la recta adequada que passa per la circumferència:

2x  y  2  x  2y  5 → x  y  1  0 

Sigui P(x, y) un punt de la bisectriu de l’angle format per les rectes s i t, aleshores d(P, s) = d(P, t):

2 x - y - 10 x + 2y - 5
2 2
    
2 +1 2 2 + 12

La solució són dues equacions. Escollim la recta adequada que passa per la circumferència:

2x  y  10  x  2y  5 → x  3y  5  0 

x + y – 1 = 0ïü
ï
Intersecció de les dues rectes: ý → x  2, y   1 → Centre de la circumferència: C(2, 1) 
x – 3 y – 5 = 0ï
ï
þ

2-2-5 5
Radi de la circumferència: r  d(C, t)   = = 5 
2
2 +1 2
5

Equació de la circumferència: (x  2)2  (y  1)2  5 → x2  y2  4x  2y  0 

117. Troba l’equació de la circumferència circumscrita al triangle de vèrtex A(1, 0), B(–3, 1) i C(5, 2).

 
Determina les coordenades del circumcentre del triangle ABC.
L’equació és de la forma: x2  y2  Ax  By  C  0 

Atès que A(1, 0) pertany a la circumferència: 1  0  A  C  0 → A  C  1  0 

Atès que B(3, 1) pertany a la circumferència: 9  1  3A  B  C  0 →  3A  B  C  10  0 

Atès que C(5, 2) pertany a la circumferència: 25  4  5A  2B  C  0 → 5A  2B  C  29  0 

5 37 1
Resolem el sistema: A   - , B   - , C   -  
6 3 6

5 37 1
Equació de la circumferència: x2  y2   x   y      0 → 6x2  6y2  5x  74y  1  0 
6 3 6

487
 
 
 
 
 
 

Llocs geomètrics.
Llocs Còniques
geomètrics. Còniques 8
 
 
 
 
 
 

El circumcentre del triangle és el centre de la circumferència:

2 2 2 2 2 2
æ 5ö æ 37 ö 1 æ 5 ö æ 37 ö æ 5ö æ 37 ö 709 æ 5 37 ö
ççç x – ÷÷÷ + ççç y – ÷÷÷ = + ççç ÷÷÷ + ççç ÷÷÷ → ççç x – ÷÷÷ + ççç y – ÷÷÷ = → Circumcentre:  ççç , ÷÷÷  
è 12 ø è 6 ø 6 è 12 ø è 12 ø è 12 ø è 6 ø 72 è 12 6 ø

 
118. Si dibuixes quatre punts sobre una circumferència i els uneixes, segons veiem en la figura,
resulta que:

 
Comprova-ho prenent els punts M(5, 1), N(4, 2), P(–3, -5) i Q(–2, 2) i demostrant que es troben
en la mateixa circumferència. Determina’n l’equació. Després, calcula el punt T i prova que es
verifica la igualtat inicial.
Sigui l’equació de la circumferència de la forma: x2 + y2 + Ax + By + C = 0.

Obtenim l’equació de la circumferència que passa per M, N i P.


Atès que (-2)2 +22 – 2(–2) + 4·2 – 20 = 0, Q pertany també a la circumferència.
௫ିହ ௬ିଵ
L’equació de la recta que passa per M i Q és: ൌ ՜ ‫ ݔ‬൅ ͹‫ ݕ‬െ ͳʹ ൌ Ͳ
ି଻ ଵ
௫ିସ ௬ିଶ
L’equació de la recta que passa per N i P és: ൌ ՜‫ݔ‬െ‫ݕ‬െʹൌͲ
ି଻ ି଻

   

488
 
 
 
 
 
 

Llocs geomètrics. Còniques 88


 
 
 
 
 
 

119. Troba l’equació de les circumferències tangents als eixos de coordenades que passen pel punt
(–1, –3).

Els centres de les circumferències es troben sobre la bisectriu de l’angle format pels eixos: x – y = 0 →
Els centres seran de la forma C(h, h).

L’equació de la circumferència és de la forma: (x  h)2  (y  h)2  r2 

La intersecció de les circumferències amb els eixos és un únic punt.

Amb l’eix Y: x  0 → (0  h)2  (y  h)2  r2 → y2  2hy  2h2  r2  0 

Atès que la solució és única → 4h2  4(2h2  r2)  0 → h2  r2 

L’equació queda de la forma: x2  y2  2hx  2hy  h2  0 

P(1, 3) pertany a la circumferència: (1)2  (3)2  2h(1)  2h(3)  h2  0 → h  r  -4  6  

Equacions de les circumferències:

C1:  ( x + 4 + 6 ) + ( y + 4 + 6 ) = ( –4 – 6 ) C2:  ( x + 4 – 6 ) + ( y + 4 – 6 ) = ( –4 + 6 )  
2 2 2 2 2 2

      

120. Donades les rectes r: 3x + 4y - 10 = 0, s: 5x – 12y + 2 = 0 i la circumferència x2 + y2 – 20x + 84 =


0.

a) Comprova que les dues rectes són tangents a la circumferència.


b) Troba el punt P d’intersecció de les dues rectes, el punt C, que és el centre de la
circumferència, i els punt A i A', en els quals les rectes són tangents a la circumferència.
c) Si anomenem d la distància que separa P de C, la distància de P a Q és d – r, i la distància de
P a Q' és d + r. Demostra que |��| � |���| � |��|� .

489
 
 
 
 
 
 

Llocs geomètrics.
Llocs Còniques
geomètrics. Còniques 8
 
 
 
 
 
 

 
∆ = 0→ L’equació té una solució, per tant, la recta és tangent a la circumferència.

∆ = 0→ L’equació té una solució, per tant, la recta és tangent a la circumferència.

121. Donades les rectes r: 3x + 4y + 7 = 0 i s: 12 x – 5y + 7 = 0, indica si podem traçar una


circumferència de centre (4, 4) que sigui tangent a les dues rectes. I amb centre en el punt (10,
2)? Escriu l’equació d’aquesta circumferència en el cas que la resposta hagi estat afirmativa.
Circumferència de centre (4, 4):

���������
� ���
√�� ���
� ���������� ��
→ La circumferència no es pot dibuixar, ja que el centre no es troba a la mateixa
� ��
���� ����� � ��

distància de les dues rectes.

Circumferència de centre (10, 2):

����������
� ���
√�� ���
� �����������
→ El centre és a la mateixa distància de les dues rectes.
� ���
���� ����� �

L’equació de la circumferència és:

(x – 4)2 + (y – 4)2 = 81 → x2 + y2 – 8x – 8y – 49 = 0 

490
 
 
 
 
 
 

Llocs geomètrics. Còniques 88


 
 
 
 
 
 

122. L’arc del pont de la figura té forma de mitja el·lipse amb les mides que veiem.

Quina és l’equació de l’el·lipse que forma l’arc?


Considerem el centre O(0, 0) en el vèrtex de la paràbola.

L’equació de la paràbola és de la forma: x2  2py 

25
Són punts de la paràbola: (10, 6) y (10, 6) → 102  2p(6) → p   -  
3

50
Equació de la paràbola: x2   - y 
3

123. Donada l’el·lipse d’equació 9x2 + 7y2 – 63 = 0, determina la longitud de la corda que forma
aquesta el·lipse amb la recta d’equació 2x – y – 1 = 0.
Aïllem y a l’equació de la recta: y = 2x – 1

Calculem els punts d’intersecció de la recta i l’el·lipse substituint a l’equació de l’el·lipse:

9x2  7(2x  1)2  63  0 → 37x2  28x  56  0 → x   14  18 7


, y   - 9  36 7
 
37 37

æ14 + 18 7 -9 + 36 7 ö÷ æ 14 - 18 7 -9 - 36 7 ö÷
Els punts són:  ççç , ÷÷  y  çç
÷ø ççè
, ÷÷  
÷ø
èç 37 37 37 37

Longitud de la corda és la distància entre els punts:


æ
ç 14 + 18 7 14 - 18 7 ö÷ ççæ -9 + 36 7 -9 - 36 7 ö÷
2 2
362 ⋅ 7 722 ⋅ 7 362 ⋅ (1 + 4) ⋅ 7
ççç - ÷÷ + çç - ÷÷    +       36 35  
ççè 37 37 ÷ø ç 37 37 ÷ø 372 372 372 37
è

 
࢞૛ ࢟૛
124. Calcula la recta tangent a l’el·lipse ൅ ൌ ૚ que passa pel punt P(0, 8).
૛૞ ૚૟

 
(Les rectes que passen per (0, 8) tenen com a equació y = mx + 8).
491
 
 
 
 
 
 

Llocs geomètrics.
Llocs Còniques
geomètrics. Còniques 8
 
 
 
 
 
 

y = mx + 8

La recta és tangent a l’el·lipse si l’equació té una única solució, és a dir, si el discriminant de l’equació
és igual a zero.

Hi ha dues rectes tangents a l’el·lipse que passen pel punt (0, 8):

125. Els enginyers de la NASA tenen la intenció d’enviar una sonda espacial a la Lluna. El
departament d’enginyeria ha manifestat que el millor moment per llançar la sonda és quan la
Terra es troba en el punt més allunyat del Sol.
Determina la màxima distància entre la Terra i el Sol sabent que l’òrbita terrestre al voltant del
Sol és una el·lipse, amb el Sol en un dels focus, que la longitud de l’eix major és de 241 428
000 km i que l’excentricitat de l’òrbita és 0,016. Determina també una equació de l’el·lipse que
representa l’òrbita de la Terra.
 
2a  241 428 000 → a  120 714 000 
c
e  0,016   → c  0,016  120 714 000  1 931 424 
a

La distància màxima entre la Terra i el Sol és quan la Terra es


troba sobre l’eix focal. La seva distància en aquest punt és: a  c  
122 645 424 km 
Atès que a2  b2  c2 → b2  14 568 139 397 332 224 
x2 y2
Equació de l’el·lipse:  + = 1 
14 571 869 796 000 000 14 568 139 397 332 224

 
126. Calcula els punts d’intersecció de les còniques següents:
a) L’el·lipse 2x2 – 3y2 = 6 i la hipèrbole 6x2 + y2 = 58.

b) Les paràboles y2 = 9x, x2 = y

a) De la hipèrbole s’obté: y2  58  6x2 
Substituint a l’equació de l’el·lipse: 2x2  3(58  6x2)  6 → 20x2  180  0 → x2  9 → x  3 
Els punts de tall són: (3, 2), (3, 2), (3, 2), (3, 2) 
� � � ��
b) � � → 9x2 – 9x = 0 → x(x3 – 1) = 0
�� � �
  Els punts de tall són: (0, 0) i (1, 3) 

492
 
 
 
 
 
 

Llocs geomètrics. Còniques 88


 
 
 
 
 
 

127. Una hipèrbole en la qual es compleix que a = b diem que és equilàtera. Suposem que té el
centre en (0, 0) i que l’eix focal és horitzontal. Calcula’n l’equació i troba les coordenades dels
focus en funció de a. Determina’n les equacions de les asímptotes.

 
�� ��
L’equació de la hipèrbole equilàtera és del tipus: � � � → � � � � � � ��
�� ��

Atès que a = b i c2 = a2 + b2 → c = √2a → F�√2a, 0) F’��√2a, 0)

a = b, per tant les equacions de les asímptotes són: y = � x

 
128. Comprova que la hipèrbole amb focus (4√�, 4√�) i (–4√�, –4√�) i constant 8√� és equilàtera.
Comprova que (8, 2) està situat en aquesta hipèrbole. Troba’n l’equació.

 
a = b, per tant la hipèrbole és equilàtera.

 
→ 2xy = 32 → xy = 16 és l’equació de la hipèrbole.

8·2 = 16 → (8, 2) és un punt de la hipèrbole.

   

493
 
 
 
 
 
 

Llocs geomètrics.
Llocs Còniques
geomètrics. Còniques 8
 
 
 
 
 
 

129. Un cable està aguantat per dos suports a la mateixa alçada a una distància l’un de l’altre de
240 m. Si el cable penja 30 m en el centre en forma de paràbola, calcula’n l’equació situant
l’origen en el punt més baix del cable.

 
Considerem el centre O(0, 0) en el vèrtex de la paràbola. L’equació de la paràbola és de la forma:  
x2  2py 
Són punts de la paràbola: (120, 30) i (120, 30) → 1202  2p30 → p  240 
Equació de la paràbola: x2  480y 
 
130. Un pont com el de la imatge, construït com un arc
parabòlic, uneix una distància de 320 m. Si l’altura de
l’arc a 128 m del centre del pont, mesurada des del
suport dels pilars, és de 32 m, quina alçada té l’arc que
forma el pont en el centre?
Considerem el centre O(0, 0) en el vèrtex de la paràbola.
L’equació de la paràbola és del tipus: x2 = 2py
æ 320 ö
Els punts  ççç , – h÷÷÷   (160, h) y (128, (h  32))  (128, 32  h) pertanyen a la paràbola, per tant:
è 2 ø

25 600 12 800 üïï


160 2 = 2 p ( – h)  p = =- ïï
-2 h h ïý → 12 800    - 8 192 →
16 384 8 192 ïï h 32 - h
128 = 2 p (32 – h)  p =
2
= ï
2(32 - h) 32 - h ïïþ

→ 409 600  4 608h → h » 89 m  
 
131. Un dissenyador d’una antena electromagnètica parabòlica de 20 m de diàmetre per rastrejar
espais de prova vol situar el focus 10 m per damunt del vèrtex.

494
 
 
 
 
 
 

Llocs geomètrics. Còniques 88


 
 
 
 
 
 

Escriu l’equació de la paràbola que forma la secció de l’antena.


L’equació de la paràbola és de la forma: x2  2py       F(0, 10) → p  20 

Equació de la paràbola: x2  40y 

 
132. Les bisectrius dels quatre quadrants tallen la paràbola y = x2 – 3x en tres punts. Calcula l’àrea
del triangle que formen.
 
‫ݕ‬ൌ‫ݔ‬
Les bisectrius dels quatre quadrants tenen com a equacions: ቄ‫ ݕ‬ൌ  െ‫ ݔ‬ 
‫ݕ‬ൌ‫ݔ‬
൜ ՜  ‫ ݔ‬ଶ െ Ͷ‫ ݔ‬ൌ Ͳ 
‫ ݕ‬ൌ  ‫ ݔ‬ଶ െ ͵‫ݔ‬
Els punts d’intersecció són: (0, 0) i (4, 4) 
‫ ݕ‬ൌ െ‫ݔ‬
൜ ՜  ‫ ݔ‬ଶ െ ʹ‫ ݔ‬ൌ Ͳ 
‫ ݕ‬ൌ  ‫ ݔ‬ଶ െ ͵‫ݔ‬
Els punts d’intersecció són: (0, 0) i (2, -2)
Atès que en el vèrtex O l’angle és recte, l’àrea del triangle és:
OA ⋅ OC 8 ⋅ 32
= = 8 u2  
2 2

 
133. Com que la paràbola és el lloc geomètric dels punts que equidisten d’una recta anomenada
directriu i un punt anomenat focus, fes servir la definició per calcular l’equació d’una paràbola
que tingui com a directriu la recta r: 3x + 4y = 0 i com a focus el punt F(–1, 0).

 
 
134. Determina els focus, els vèrtexs i les directrius de les paràboles següents:
a) y = x2 + 2x + 1 c) y = 4x2 – 8x + 12
b) 4y = –x2 + 8x – 6 d) y = 6x2 + 9x – 10

495
 
 
 
 
 
 

Llocs geomètrics.
Llocs Còniques
geomètrics. Còniques 8
 
 
 
 
 
 

(Recorda que, en una paràbola del tipus y = ax2 + bx + c, la directriu és horitzontal i el vèrtex és

un punt d’abscissa െ ).
૛ࢇ

 
a) y  (x  1)   2

1 æ 1 ö÷ 1
Vèrtex: (1, 0)        2p = 1 p =  F ççç- 1, ÷        Directriu:  y = -  
2 è 4 ÷ø 4

æ 5ö
b)  4y  10  (x2  8x  16) → - 4 ççç y - ÷÷÷   (x  4)  
2
è 2ø

7
Vèrtex:  æççç4, 5 ö÷÷÷         æ
2 p = - 4  p = - 2  F ççç 4,
3 ö÷
÷        Directriu:  y =  
è 2ø è 2 ø÷ 2

1
c) y  8  4(x2  2x  1) → ( y - 8)   (x  1)2 
4

1 1 æ 129 ö÷ 127
Vèrtex: (1, 8)        2p =  p =  F ççç1, ÷        Directriu:  y =  
4 8 è 16 ÷ø 16
2
107 æ 3 9 ö 1æ 107 ö÷ çæ 3ö
d)  y + = 6 ççç x 2 + x + ÷÷÷  ççç y + ÷ = ç x + ÷÷÷  
8 è 2 16 ø 6 è 8 ÷ø èç 4ø

æ 3 107 ÷ö 1 1 æ 3 40 ö -161
Vèrtex:  ççç- , - ÷    2p =  p=  F çç- , - ÷÷÷      Directriu:  y =  
è 4 8 ÷ø 6 12 çè 4 3ø 12

135. Determina quins tipus de còniques són les següents, identifica’n els elements i dibuixa'n una
representació gràfica aproximada:
a) x2 + y2 + 2x + 6y + 1 = 0
b) x2 – 4y2 + 16y – 32 = 0
c) 16x2 + 9y2 – 32x + 54y – 47 = 0
d) x2 + 6x – 4y + 17 = 0
e) x2 – y2 + 3x + 5y + 3 = 0
a) x2 + y2 + 2x + 6y + 1 = 0
(x+1)2 + (y+3)2 = 9

La cònica és una circumferència de centre C(–1, –3) i radi 3.

496
 
 
 
 
 
 

Llocs geomètrics. Còniques 88


 
 
 
 
 
 
�� ������
b) x2 – 4y2 + 16y – 32 = 0 → x2 – 4(y – 2)2 = 16 → � ��
�� �
La cònica és una hipèrbole de centre C(0, 2).
���� 2�
a2 = 16 → a = 4 → �
������ 2�

� �√20� 2�
Si c2 = a2 + b2 → � � √20 → �  
�� ��√20� 2�
c) 16x2 + 9y2 – 32x + 54y – 47 = 0 → 16(x – 1)2 + 9(y + 3)2 = 144
������ ������
→ � � � → La cònica és una el·lipse de centre C ��� –��
� ��

���� ��
Atès que a2 = 16 → a = 4 → �
����� �7�
���2� ���
Si b2 = 9 → b = 3 → �
����� ���

� ��� �� � √7�
Atès que a2 = b2 + c2 → c = √7 → �
�� ��� �� � √7�
d) x2 + 6y2 – 4y + 17 = 0 → (x + 3)2 = 4(y – 2)

→ � � 2 � �� � ���

La cònica és una paràbola de vèrtex V �–�� 2��

� � ��
Atès que 2p = →p= → � ���� �
� � �

��
La directriu és la recta y =

497
 
 
 
 
 
 

Llocs geomètrics.
Llocs Còniques
geomètrics. Còniques 8
 
 
 
 
 
 
2 2 2 2
æ 3ö æ 5ö æ 5ö æ 3ö
e) x2  y2  3x  5y  3  0 → ççç x + ÷÷÷ – ççç y – ÷÷÷ = –7 → ççç y – ÷÷÷ – ççç x + ÷÷÷ = 7  
è 2ø è 2ø è 2ø è 2ø

æ 3 5ö
La cònica és una hipèrbole de centre  ççç – , ÷÷÷  i l’eix focal paral·lel a l’eix Y:
è 2 2ø

æ –3 5 ö æ 3 5 + 2 7 ÷ö æ 3 5 ö æ 3 5 - 2 7 ö÷
ç
a  b  7 → A ççç , + 7 ÷÷÷    çç- ç
, ÷÷÷ ,  A 'ççç – , – 7 ÷÷÷    ç-
çç 2 ,
÷÷  
è2 2 ø çè 2 2 ø è 2 2 ø è 2 ø÷

æ 3 5 + 2 14 ö÷ æ 3 5 - 2 14 ö÷
ç
c2  a2  b2 → c   14 → F ç- ç
÷÷ ,  F 'ç- ÷÷  
çç 2 , ÷ø çç 2 ,
è 2 è 2 ø÷

AMPLIA 
 
136. Escull la resposta adequada.

□  Les circumferències són tangents als eixos, per tant la distància a aquests és el seu radi (r); atès
que són també tangents exteriors a la circumferència, la distància entre els seus centres és r + 1 (la
suma dels seus radis).

Tenim un triangle rectangle; emprant Pitàgores:

(r  1)2  (3  r)2  r2 → r2  8r  8  0 → r  4   2 → La seva suma és 8. 

□  El sistema que forma la intersecció de les dues circumferències: 

x 2 – 4 x + y 2 – 2y = 0 ü
ï
ï
ý 
x + 2 x + y + 2 y – 18 = 0ï
2 2
ï
þ

Si restem la segona equació a la primera obtindrem la recta:

3x  2y  9 

□  Distància mínima:  52 + 22 + 102 + 62 - 4    29 + 136 - 4 » 13  

□  Resolem el sistema format per les dues equacions i trobem la seva intersecció:

k
De la segona equació tenim y = . I aïllem en la primera:
x

2
æk ö k 2  k 4 - 4k 2
x2   ççç ÷÷÷   k2 → x4  k2x2  k2  0 → x2   → No té solució si k4  4k2  0 → k2  4 
èxø 2

498
 
 
 
 
 
 

Llocs geomètrics. Còniques 88


 
 
 
 
 
 

Els valors enters que compleixen aquesta condició són l’1 i el –1. El 0 no es considera, ja que en
ambdues equacions resulta l’origen. Per tant, la resposta seria 2.

137. Donats els punts A(2, 1) i B(6, 4), determina el lloc geomètric dels punts P que verifiquen que
l’àrea del triangle ABP és de 10 unitats quadrades.
d(A, B) = √4� � 3�  = 5 

Si l’àrea del triangle mesura 10, aleshores l’altura ha de mesurar 4.

��� ���
Sigui r la recta determinada per A i per B:  � → 3� � 4� � 2 � � 
� �

|�������| 3� � 4� � 2 � 2�
d(P, r) = 4 →  �4→�  
�32 ���4�2 �3� � 4� � 2 � 2�

3� � 4� � 22 � �
El lloc geomètric està format per dues rectes paral·leles: �
3� � 4� � �� � �

138. Descriu el lloc geomètric dels punts que verifiquen l’equació següent:
x2 – y2 – 4x + 6y – 5 = 0
x2 – y2 – 4x + 6y – 5 = 0 → (x2 – 4x + 4) – (y2 – 6y + 9) – 5 = 4 – 9
��2���3 �������
→ (x – 2)2 – (y – 3)2 = 0 → � →�
� � 2 � �� � 3 �������
El lloc geomètric està format per dues rectes perpendiculars.
 
139. Considera un punt Q d’una circumferència amb el centre a l’origen i radi r, i un altre punt P
amb la mateixa abscissa que Q. Calcula el lloc geomètric dels punts P si sabem que la raó de
les ordenades de P i Q és k.
 
 
Q(a, y1) compleix  a2 + y 12 = r 2  y 1 = r 2 - a2 . 

Sabem que  y2 = ky1 , amb k ¹ 0 . Aïllant, obtenim:

2 2 2 2
y 22 æ aö æ y ö æxö æ y ö
y 2 = k ⋅ r 2 - a2  a2 + = r 2  ççç ÷÷÷ + ççç 2 ÷÷÷ = 1  ççç ÷÷÷ + ççç ÷÷÷ = 1  
k2 è r ø è kr ø è r ø è kr ø

Així, el lloc geomètric demanat seria una el·lipse amb un semieix de longitud el
radi de la circumferència, l’altre de longitud la constant k multiplicada pel radi.

   

499
 
 
 
 
 
 

Llocs geomètrics.
Llocs Còniques
geomètrics. Còniques 8
 
 
 
 
 
 

140. Si un senyal incideix sobre una antena parabòlica en direcció perpendicular a la seva directriu,
es reflecteix com si xoqués contra la recta tangent en aquest punt.

 
Demostra que el senyal reflectit passa sempre pel focus de la paràbola.
Les antenes parabòliques compleixen la seva missió gràcies a aquesta propietat, ja que
concentren el senyal en un únic punt, el focus.

 
Es considera un punt F, una recta s i un punt M que pertany a la mateixa.

Si dibuixem la perpendicular a d que passa per M, aquesta recta talla a la mediatriu del segment MF
തതതതത ൌ ܲ‫ܨ‬
en un P, el qual pertany a la paràbola de focus F i directriu s per ser ‫ܲܯ‬ തതതത . Atès que la recta és
perpendicular, la mesura del segment MP coincideix amb la distància del punt P a la recta s.

Aquesta mediatriu és la tangent a la paràbola de focus F i directriu s. A més, atès que formes el
෢ ݅‫ܶܲܨ‬
senyal incident i reflectit, els angles ܴܲܵ ෣són iguals. I com que són oposats pel vèrtex, els angles
෣ ݅ܶܲ‫ܨ‬
‫ܶܲܯ‬ ෣ també són iguals.

Per tant, un senyal que incideix perpendicularment a la directriu es reflecteix passant pel focus F.

141. Fem lliscar un quadrat de 10 cm de costat pel pla XY de manera que els vèrtexs d’un dels
costats sempre estan en contacte amb els eixos de coordenades, un amb l’eix X i l’altre amb
l’eix Y. Determina el lloc geomètric que descriuen els elements següents en aquest moviment:
a) El punt mitjà del costat de contacte amb els eixos.
b) El centre del quadrat.
c) Els vèrtexs del costat de contacte i de l’oposat en el primer quadrant.

500
 
 
 
 
 
 

Llocs geomètrics. Còniques 88


 
 
 
 
 
 

a) Sigui L(x, y) un punt d’aquest lloc geomètric:

Com que és el punt mig del segment PQ → P(2x, 0) y Q(0, 2y) 

Apliquem Pitàgores al triangle OPQ:

10 2 - 4 x 2
d(O, Q)   10 2 - 4 x 2 → y   → 4y2  100  4x2 → x2  y2  25 
2

El lloc geomètric és una circumferència de centre (0, 0) i radi 5.

b) Sigui L(x, y) un punt d’aquest lloc geomètric:

Diagonal
Atès que és el centre del rectangle la distància d(C, P) = d(C, Q) =
2

Diagonal
Apliquem Pitàgores al quadrat s’obté: Diagonal   10 2 →    5 2  
2

(5 2 )
2
Apliquem Pitàgores al triangle CEP: d(E, P)   - y2  

Repetim el procediment pel triangle amb base en l’eix Y, així:

(5 2 ) (5 2 )
2 2
- x 2 → (5 2 ) - y 2    (5 2 ) - x 2 → x2  y2 
2 2
- y 2   

Les coordenades del centre del quadrat són iguals movent-se aquest en un segment de les
bisectrius dels quadrants:

C(c1, c2)  (±5α, ±5α) essent a Î éêë1, 2 ùúû . 

c) ▪ Vèrtex Q: el seu lloc geomètric és la recta: x = 0 en el interval [-10, 10].

▪ Vèrtex P: el seu lloc geomètric és la recta: y = 0 en el interval [-10, 10].

▪ Vèrtex R:

Sigui L(x, y) un punt del lloc geomètric. S’aplica Pitàgores als triangles ratllats del dibuix per obtenir
els costats del triangle ombrejat.

Repetim el procediment aplicant Pitàgores: 100   ( y - 100 - x 2 )    ( x - 200 - y 2 )  


2 2

El lloc geomètric és l’equació: 2x 200 - y 2   2y 100 - x 2   300  0 

▪  Vèrtex S: 

Sigui L(x, y) un punt del lloc geomètric. Apliquem Pitàgores als triangles ratllats del dibuix per
obtenir els costats del triangle ombrejat.

Repetim el procediment aplicant Pitàgores: 100   ( y - 200 - x 2 )    ( x - 100 - y 2 )  


2 2

El lloc geomètric és l’equació: 2y 200 - x 2   2x 100 - y 2   300  0 

501
 
 
 
 
 
 

Llocs geomètrics.
Llocs Còniques
geomètrics. Còniques 8
 
 
 
 
 
 

MATEMÀTIQUES A LA TEVA VIDA 
 
1. Què és una antena parabòlica i per a què es fa servir?
Una antena parabòlica és una superfície metàl·lica amb forma de paraboloide de revolució que
serveix de reflector dels senyals i un element radiant situat en el focus que rep els senyals reflectits.

Usos: Televisió via satèl·lit.

2. Explica com es forma un paraboloide de revolució. És una figura plana?


Es forma en girar una paràbola al voltant d’un eix.
No és una figura plana, és tridimensional.

3. Explica la propietat que permet a les antenes parabòliques rebre i emetre de manera òptima
senyals via satèl·lit.
Atès que són parabòliques reflecteixen els senyals transmesos que incideixen paral·lels a l’eix i es
concentren en el focus, on són convertits per un receptor al format adequat.

4. Enumera alguns avantatges i inconvenients d’utilitzar antenes parabòliques per a Internet.


▪ Avantatges: sense fils; accessible per a les persones que no poden optar a una altra tecnologia.
▪ Inconvenients: la recepció pot alterar-se segons les condicions meteorològiques; la velocitat pot
ser més lenta si s’envia o rep senyal a grans distàncies; pot ser una mica més car que d’altres
tecnologies.

5. Si una antena parabòlica fa 1 m de diàmetre d’obertura i el receptor, ubicat en el focus, és a 25


cm del vèrtex, quina profunditat té?
L’equació de la paràbola és del tipus: x2  2py 

F(0, 25) → p  50 → x2  100y 

Atès que el diàmetre és 1 m = 100 cm, un dels extrems és x = 50 cm:

502  100y → y  25 cm 

La profunditat de la paràbola és de 25 centímetres.

6. Dibuixa la gràfica de la paràbola que genera l’antena de l’activitat anterior si gira sobre el seu
eix.

x2  100y 

 
 

502
Funcions
Funcions  9
 

ACTIVITATS 
1. Justifica si les gràfiques següents corresponen a funcions

 
a) La gràfica correspon a una funció, perquè a cada valor de x li correspon un únic valor de y.

b) La gràfica no correspon a una funció, perquè hi ha valors de x als quals els corresponen més d’un
valor de y.

2. Indica en cada cas si la relació entre les dues magnituds és una funció o no ho és.  

a) La quantitat de fruita que compra una família, en quilos, i el nombre de peces de fruita que hi
entren.

b) La quantitat de fruita que compra una família, en quilos, i el preu de la compra.

c) La quantitat de fruita que compra una família, en quilos, i el preu d’un quilo de fruita.

a) No es tracta d’una funció, atès que la mida i el pes de cada fruita és variable.

b) És una funció, atès que per a cada quantitat de fruita comprada hi ha un únic preu segons el pes en
quilos.

c) No és una funció.

3. Determina el domini de les funcions següents:

a) ���� � ��� � �� � � e) ���� � ����� � ��

�� ��
b) ���� � f) ���� � √�� � �
�� ��


c) ���� � g) ���� � ����� � ���
�� ��

� � ��
d) ���� � √� � � h) ���� � ��

 
a) Dom f = � c) Dom f = � e) Dom f = � g) Dom f = (16, +∞)
é1 ö
b) Dom f = � - {-2, 2} d) Dom f = � f) Dom f = ê , + ¥÷÷÷ h) Dom f = ��
êë 3 ø

4. Calcula el domini i el recorregut de la funció que tens a la dreta.

503
Funcions
Funcions 9

 
 
Dom f = [‐4,3] ⋃ (4,6) 
Im f = [‐3,2] ⋃ {4} 
 
5. Estudia la simetria de les funcions següents:

�� �� �� ����� �� ��
a) ���� � b) ���� � c) ����� �
�� �� ���

(- x )2 - 1 x2 -1 x2 -1
a) f(-x) = = = - = -f(x) → f(x) és imparell.
2(- x ) -2 x 2x

(- x )2 - 6(- x ) - 7 x2 + 6x - 7
b) f(-x) = 2
= → f(x) no és parell ni imparell.
(- x ) x2

(- x )4 - 5 x4 - 5
c) f(-x) = 2
= = f(x) → f(x) és parell.
3(- x ) 3x2

 
6. Completa la gràfica d’aquesta funció periòdica de període 3.

 
 

504
Funcions 99

7. Representa sobre els mateixos eixos les funcions f(x) = 3x – 1 i g(x) = 5x + 4. Calcula’n el punt en
comú.

El punt d’intersecció és:

૞ ૚ૠ
൬െ ǡ െ ൰
૛ ૛

 
 
8. Representa gràficament aquestes funcions quadràtiques:

a) f(x) = –3x2 – x – 1 b) f(x) = x2 + 2 x – 2

 
 
9. Tenint en compte la gràfica de f(x), expressa g(x) utilitzant f(x).

 
a) g(x) = f(x) – 3 b) g(x) = f(x + 2)

10. Representa gràficament la funció:

f(x) = x2 – 2x + 1

A partir d’aquesta funció, representa les següents:

a) g(x) = x2 – 2x + 3

505
Funcions
Funcions 9

b) h(x) = x2 – 2x – 2

c) i(x) = (x – 1)2 – 2(x – 1) + 1

d) j(x) = x2 + 2x + 1

e) k(x) = – x2 + 2x – 1
f(x)

g(x)

j(x) i(x) 

a) g(x) = f(x) + 2
b) h(x) = f(x) – 3
c) i(x) = f(x – 1)
h(x) 
d) j(x) = f(–x) k(x) 

e) k(x) = –f(x)
 
11. Representa gràficament aquestes funcions:

૜ ૜ ି૝ ି૚
a) ࢌሺ࢞ሻ ൌ b) ࢌሺ࢞ሻ ൌ െ c) ࢌሺ‫ܠ‬ሻ ൌ d) ࢌሺ‫ܠ‬ሻ ൌ
࢞ ࢞ ࢞ ࢞

a) c)
 
 
 
 
 
 
 
 
b) d)
 
 
 
 
 
 
 
   

506
Funcions 99

12. Representa gràficament les funcions següents:

� � � ��
a) ���� � b) ���� � � c) ���� � d) ���� �
�� �� �� ��

a) c)
 
 
 
 
 
 
 
b)                                 d)  
 
 
 
 
 
 
 
13. Calcula el domini d’aquestes funcions amb radicals:

a) ���� � √�� � �
b) ���� � √�� � ��

a) Dom f =  b) Dom f = (– ∞ , – 6] U [6, +∞)


 
14. Representa gràficament les funcions següents:

a) ���� � √� � � b) ���� � √� � � c) ���� � √� � �

15. Calcula la funció inversa de les funcions que hi ha a continuació:

� �
a) ���� � � � � c) ���� � � � �
� �

507
Funcions
Funcions 9


b) ���� � �� � � � d) ���� � √� � � �

y y
a)  x   -   1 → f 1(x)  2x  2              c)   x   - 2  → f 1(x)  2x2  4 
2 2

b)  x  y2  4 → f 1(x)   4 - x               d)  x   3 y 2 - 1  → f 1(x)   x 3 + 1  

 
���
16. Determina quina és la funció inversa de ���� � i efectua les accions següents:

a) Representa les funcions f(x) i f -1(x).


b) Comprova si les gràfiques són simètriques respecte de la recta y = x.
7�� 7 7
�� � �� � 7 � � � �� � � � 7 � � � � � �� ��� �  
� ��� ���
 
a)

 
b) Les funcions són simètriques respecte a la recta y = x.
 
17. Sense dibuixar la gràfica, justifica si les funcions següents són creixents o decreixents:

a) ���� � �� �� c) ���� � �� ��

� � �
b) ���� � � � d) ���� � √�

a) Creixent, ja que a > 1. c) Decreixent, ja que a < 1.


b) Decreixent, ja que a < 1. d) Creixent, ja que a > 1.
 
18. Representa gràficament aquestes funcions:

� �
a) ���� � �� c) ���� � �� �� e) ���� � �� �

� � � ��
b) ���� � ���� � d) ���� � � � f) ���� � � �
� �

508
Funcions 99

a) c) e) a < 0  no es pot representar.

      

b) d) f)

        
 
19. Sense dibuixar la gràfica, justifica si les funcions següents són creixents o decreixents:

a) ���� � ������ � c) ���� � ���� � e) ���� � ���√� �

b) ���� � ���� � d) ���� � ��� ��� � f) ���� � ������ �


a) Creixent, atès que a > 1. d) Decreixent, atès que a < 1.


b) Decreixent, atès que a < 1. e) Creixent, atès que a > 1.
c) Creixent, atès que a > 1. f) Creixent, atès que a > 1.
20. Representa gràficament les funcions que tens a continuació:

 
a) c) e)

             

509
Funcions
Funcions 9

b) d) f)

             
 
21. Representa gràficament aquestes funcions:

� �
a) ���� � ��� �� � � b) ���� � ��� �� � �
� �

 
 

 
� �
f�x� � ��� �� � � � ����� ���� � ��� �� � � � �����
� �

22. Representa aquestes funcions:


a) ���� � �� �� � � b) ���� � ���� � ��

a)          b) 

            
   

510
Funcions 99

23. Calcula les expressions següents:

2
a) arc sin b) arc cos 0 c) arc tg (-1)
2

2 p p p
a) arc sin = b) arc cos 0 = c) arc tg (-1) = –
2 4 2 4

 
24. Representa gràficament aquestes funcions:


a) f(x) = arc cos��� � � b) f(x) = arc sin (x – π)

               
 
25. Representa gràficament aquesta funció definida a trossos:

��������������������������������������� � ��
���� � ��� � ����������������� � � � � � �
�� � ������������������������������ � �

Descriu-ne les característiques principals.

 
 
▪ Primer interval (–∞, –2):
f(x) = 2 → Recta horitzontal, paral·lela a l’eix X a y = 2 amb extrem a
(-2, 2).
▪ Segon interval [-2, 0]:
f(x) = x2 – 7 → Paràbola amb mínim (a = 1 > 0) al vèrtex (0, –7) i
amb extrems a (–2, –3) i (0, –7).
▪ Tercer interval (0, +∞):
f(x) = –7 – x → Recta decreixent (m = –1 < 0) amb extrem a f(0) =
(0, –7).

26. Representa la gràfica d’aquesta funció i descriu-ne les característiques més importants:


���������������������������������������������������� � �
���� � � ��
� � �� � ������������������������� � � � �
������������������������������������������������������ � �

511
Funcions
Funcions 9

 
▪ Primer interval (–∞, 2]:
x
f(x) = → Decreixent a aquest interval, amb extrem a (2, 1).
2

▪ Segon interval (2, 4]:


f(x) = x2 – 6x + 9 → Paràbola amb mínim (a = 1 > 0) al vèrtex (3, 0) i amb extrems a (2, 1) i (4, 1).
▪ Tercer interval (4, +∞):
f(x) = 5 → Recta horitzontal, paral·lela a l’eix X a y = 5 amb extrem a (4, 5).
 
27. El servei de correus cobra 0,30 € pels primers 25 g de tramesa i, a partir d’aquesta quantitat,
cobra 0,20 € per cada 25 g, o fracció, de pes extra. Representa la gràfica del que costa enviar
cartes fins a 150 g.

ìï0,30 si x Î (0, 25]


ïï
ïï0,30 + 0,20 si x Î (25, 50 ] éxù
f ( x ) = ïí  f ( x ) = 0,30 + 20 ⋅ ê ú  
ïï ï0,30 + 0,20 ⋅ 2 si x Î ( 50, 75 ] ëê 25 ûú
ïï
ïî...
 
 
28. La funció que associa a cada nombre la seva part decimal és:

f(x) = x – [x]
Representa la funció i analitza’n les propietats.

Dom f = � Im f = [0, 1)

La funció no és contínua. Tots els nombres enters són punts de discontinuitat inevitable de salt finit.

És periodica, de període 1. No és simètrica.

És creixent en (k, k + 1), essent k ∈ �.

No té màxims ni mínims.

512
Funcions 99

29. Determina el valor de les funcions següents en el punt x = – 2, tenint en compte que:


���� � ���� � ��� � �
���


a) (f – g)(x) b) (f · g)(x) c) � � ���

x -3 x 3 - 3 x 2 + 2 x + 1 -3(-2)3 - 3(-2)2 + 2(-2) + 1


a)  (f  g)(x)  f(x)  g(x)    (3x2  1)    → (f  g)(2)     9 
x +1 x +1 (-2) + 1

æ x ö ÷÷ (3x2  1)   3x3 - x 3(-2)3 - (-2)


b)  (f  g)(x)  f(x)  g(x)   ççç  → (f  g)(2)     22 
è x + 1ø÷ x +1 (-2) + 1

x
æ f ö÷ f ( x ) x æf ö (-2) 2
c)  ççç ÷÷÷( x )         2+ 1    3
x  →  ççç ÷÷÷÷( x )        
è gø g( x ) 3x -1 3x + 3x2 - x -1 è gø 3(-2)3 + 3(-2)2 - (-2) - 1 11

 
30. Calcula el valor de les funcions següents en els punts indicats, tenint en compte que:

�� ��
���� � √� � ���� �
���

a) (f · g)(4) b) � �(–1) c) ��� ����

x2 + 3 42 + 3 19 608
a)  (f ⋅ g )( x ) = x 5 ⋅  (f ⋅ g )(4) = 45 ⋅ = (2)2⋅5 ⋅ =  
x +1 4 +1 5 5

� �� � ������� �
b) � � ��� � �� ��

� � � ��
���


No existeix � � ����, perquè ������  no és real ja que el radicant és negatiu. 

2
c)  (f 2 )( x ) = ( x 5 ) = x 5  (f 2 )(2) = 25 = 32   
 

31. Donades les funcions ���� � � �� �������������� � ,
����

calcula el valor de les composicions que tens a continuació a x = 2.

a) (f ◦ g)(x) c) (f ◦ f)(x)

b) (g ◦ f)(x) d) (g ◦ g)(x)

æ 1ö 1
a) (f ◦ g)(2) = f(g(2)) = f ççç ÷÷÷ = c) (f ◦ f)(2) = f(f(2)) = f(4) = 16
è3ø 9

1 æ 1ö
b) (g ◦ f)(2) = g(f(2)) = g(4) = d) (g ◦ g)(2) = g(g(2)) = g ççç ÷÷÷ = - 3
7 è3ø

   

513
Funcions
Funcions 9

32. Si f(x) = √��� i g(x) = x – 4, determina primer la composició de funcions i, després, calcula el valor
de les funcions següents en els punts indicats:

a) (f ◦ g)(5) b) (g ◦ f)(5)

A partir dels resultats, justifica si la composició de funcions és commutativa.

a) (f ◦ g)(x) � f(g(x))= f(x ‐ 4) = �2�� � ���  
(f ◦ g)(5) � √2 

b) (g ◦ f)(x) � g(f(x))= g(√2�3 ) = √2� � � � 
(g ◦ f)(5) � √250 – 4 = 5√10 � � 
(f ◦ g)(5) � (g ◦ f)(5) → La composició de funcions no és commutativa. 
      
 

   

514
Funcions 99

SABER FER 

33. Calcula el domini de ���� � � � √� � �

1
està definida si x � 0.
x

x + 3 està definida si x � 3 � 0 → x � -3.

Dom f = [-3, 0) U [0, +∞) = [-3, +∞) - {0}

 
34. Determina el període de f (x) = cos 2x.

cos x  cos (x  2π) → f(x)  cos 2x  cos (2x  2π)  cos (2(x  π))  f(x  π) → Període  π 
 
35. Representa gràficament aquestes funcions:

a) ���� � �� b) ���� � ���

 
a)                        b) 

              
 
36. A partir de la gràfica de f (x), representa:

a) g(x) = k(x) – 3 b) g(x) = f(x + 2)

         
 

515
Funcions
Funcions 9

37. Representa la funció f(x) = 2 log x.

 
 
 
38. Representa gràficament les funcions següents:

ି૚ ૛࢞ା૛
a) ࢌሺ࢞ሻ ൌ b) ࢌሺ࢞ሻ ൌ
࢞ି૚ ࢞ି૛

 
1 6 6
a)  f(x)  g(x  1) amb g(x)                 b)  f(x)  2    → g(x)    → f(x)  2  g(x  2) 
x x -2 x

                    
 
39. Representa la gràfica de les funcions inverses d’aquestes funcions:

a) f(x) = x2 b) f(x) = log(x – 1)

a)  y  x2 → x  y2 → y    x  → f  1(x)    x  


No és una funció, ja que per a cada valor de x, es tenen dos valors y. 
b)  y  log x → x  log y → 10x  y → f 1(x)  10x 

     
40. Representa gràficament la funció exponencial f(x) = 32x – 3.
516
Funcions 99

g(x)  9x → f(x)  g(x)  3 

 
 
41. Dibuixa la gràfica de les funcions següents:

૚ ૜࢞
a) ࢌሺ࢞ሻ ൌ ૜૛࢞ b) ࢌሺ࢞ሻ ൌ ቀ ቁ

x
æ 1 ö÷
a) f(x) = 9x b) f(x) = ççç ÷
è27 ø÷

                    
 
42. Dibuixa la gràfica de la funció f(x) = log 10x2.

f(x)  log 10x2  log 10  2 log x  1  2 log x 

43. Dibuixa la gràfica de la funció f(x) =|sin x|; en l’interval [0, 2ૈ].

ïìsin x si x Î [0, p]
f ( x) = ïí  
ïïî-sin x si x Î (p, 2p]

517
Funcions
Funcions 9

 
 
44. Dibuixa la gràfica de la funció f(x) = 2x – |x|.

ïì x si x ³ 0
f ( x ) = ïí  
ïïî3 x si x < 0

 
 
45. Expressa aquestes funcions com a composició de funcions més senzilles:


a) ࢌሺ࢞ሻ ൌ ࢙࢏࢔૛ ሺ࢞૛ ൅ ૚ሻ b) ࢌሺ࢞ሻ ൌ ට
࢞ି૚

a) f1(x) = x2 + 1 f2(x) = sin x f3(x) = x2 → f(x) = (f3 ◦ f2 ◦ f1)(x)


1
b) f1(x) = x – 1 f2(x) = f3(x) = x → f(x) = (f3 ◦ f2 ◦ f1)(x)
x

   

518
Funcions 99

ACTIVITATS FINALS 
 
46. Digues si aquestes gràfiques corresponen a una funció:

 
 
a) La gràfica correspon a una funció perquè a cada valor de x li correspon un únic valor de y.
b) La gràfica no correspon a una funció perquè hi ha valors de x als quals els correspon dos valors de y.
c) La gràfica no correspon a una funció perquè hi ha valors de x als quals els correspon dos valors de y.
d) La gràfica correspon a una funció perquè a cada valor de x li correspon un únic valor de y.
 
47. Fixa’t en aquests exemples de tarifes telefòniques:

 Establiment de trucada 0,15 € i 3 cènt. el minut o fracció.

 Cada minut o fracció, 5 cènt.

 Cada minut o fracció entre setmana, 4 cènt., i si és cap de setmana, 3 cènt.

Completa una taula com la següent per a cada tipus de tarifa i determina si defineixen una funció.

En cas afirmatiu, dibuixa’n la gràfica.

Minuts  1 5 7 9 12
Preu (€) 
 
▪ Tarifació 1:

Minuts  1 5 7 9 12
Preu (€)  0,18 0,30 0,36 0,42 0,51
 
Correspon a una funció perquè per a cada durada de la trucada li correspon un únic preu.

minuts
 

519
Funcions
Funcions 9

▪ Tarifació 2:

Minuts  1 5 7 9 12
Preu (€)  0,05 0,25 0,35 0,45 0,60
 
Correspon a una funció perquè per a cada durada de la trucada li correspon un únic preu.

minuts
 
▪ Tarifació 3:
– Entre setmana:

Minuts  1 5 7 9 12
Preu (€)  0,04 0,20 0,28 0,36 0,48
 
– Cap de setmana:

Minuts  1 5 7 9 12
Preu (€)  0,03 0,15 0,21 0, 27 0,36

No correspon a una funció perquè a cada durada de la trucada li corresponen dos preus, depenent del
dia de la setmana en què es faci la trucada.
 
48. Comprova si els punts x = –3, x = 0, x = 2 pertanyen al domini d’aquestes funcions:

a) ���� � �� � �� � � c) ���� � √��� � �

����
b) ���� � d) ���� � ����� � ��
�� ���

a) Dom f =  → Els tres punts pertanyen al domini de la funció.


b) x2 + 3x = 0 → Dom f =  � {0, �3 } → Només x = 2 pertany al domini de la funció.
c) –2(–3) + 1 = 7 > 0 → x = �3 pertany al domini.
0 + 1 = 1 > 0 → x = 0 pertany al domini.
�2 · 2 + 1 = �3 < 0 → x = 2 no pertany al domini.
d) – (–3) - 4 = �1 < 0 → x = �3 no pertany al domini.
0 � 4 = �4 < 0 → x = 0 no pertany al domini.
�2 �4 = –6 < 0 → x = 2 no pertany al domini.
 
49. Estudia si els valors de l’ordenada, y, estan inclosos en els recorreguts d’aquestes funcions:

a) y = 3, y = 2, y = –5 per a f(x) = √�� � �

b) y = 0, y = 30, y = –3 per a f(x) = x2 – 5x + 6

520
Funcions 99

a) √�� � � � � → �� � � � � → � � � → � � � ∈ �� �

7
√3� � 3 � 3 → 3� � 3 � � → � � → � � 3 ∈ �� �
3
y = – 5 ∉ Im f, perquè l’arrel no pot prendre valors negatius.
b) � � � �� � � � � → � � � � � � � → � � � ∈ �� �

� � � �� � � � 3� → � � � �� � �� � � → � � � � � � �3 → � � 3� ∈ �� �
� � � �� � � � �3 → � � � �� � � � � → � � ��� � �
→ L’equació no té solucions → y = – 3 ∈ Im f.

50. Calcula el domini d’aquestes funcions:

������� ����
a) ���� � c) ���� �
� �� ��

������ ���
b) ���� � d) ���� �
��� �� �����

a) Dom f =  b) Dom f =   {5} c) Dom f =  d) Dom f =   {2}


 
51. Estudia el domini de les funcions següents:

a) ���� � √�� � � c) ���� � √�� � � � �

b) ���� � √�� � � � �� d) ���� � √��� � �

é7 ö
a) Dom f = ê , + ¥÷÷÷ c) Dom f = (∞, 3] U [2, +∞)
ëê 3 ø

é1 ö
b) Dom f =  d) Dom f = (∞, 0] U ê , + ¥÷÷÷
êë 3 ø

52. Escriu el domini d’aquestes funcions:

a) f (x) = log4 (x – 4) c) f (x) = 3ln x

b) f (x) = cos (1 – x) d) f (x) = sin (x – �)

a) Dom f = (4,+∞) c) Dom f = (0, +∞)

b) Dom f =  d) Dom f = 

 
53. Determina el domini d’aquestes funcions:

a) ���� � √� � � � √� � � b) ���� � √�� � � � √� � �

a) Dom f = [ -1, 8]

b) Dom f = ∅

521
Funcions
Funcions 9

54. Determina si aquestes funcions tenen algun tipus de simetria:

a) f (x) = x3 – 3x c) f (x) = x2 – x

b) f (x) = x4 – 1 d) f (x) = x4 – 2x2

a) f(–x ) = (–x)3 – 3 (–x) = –x3 + 3x = –f(x) → La funció és simètrica respecte de l’origen de coordenades.
b) f(–x) = (–x)4 – 1 = x4 – 1 = f(x) → La funció és parell, és simètrica respecte de l’eix Y.
c) f(–x) = (–x)2 – (–x) = x2 + x → La funció no és simètrica.
d) f(–x) = (–x)4 – 2(–x)2 = x4 – 2x2 = f(x) → La funció és parell, és simètrica respecte de l’eix Y.
 
55. Indica el tipus de simetria d’aquestes funcions:

૜࢞૛ ି࢞ ૛࢞૜ ି࢞
a) ࢌሺ࢞ሻ ൌ b) ࢌሺ࢞ሻ ൌ
࢞ ࢞૛ ା૚

3(- x)2 - (- x ) 3 x2 + x 3 x2 + x
a)  f(x)         -  → La funció no té simetria. 
(- x ) -x x

2(- x)3 - (- x ) -2 x 3 + x
b)  f(x)   2
   2   f(x) → La funció és imparell, és simètrica respecte de l’origen de
(- x ) + 1 x +1

coordenades.

56. Determina el període d’aquestes funcions:

 
a) T = 3 0 =3 b) T = 2  0 = 2

57. Considera la funció que relaciona el temps, en dies, amb la superfície visible de la Lluna. Digues
si és una funció periòdica. Si ho és, indica’n el període.

La superfície visible depèn de les fases a la rotació de la Lluna al voltant de la Terra, la funció és
periòdica. El període és de 28 dies.

 
58. Escriu l’expressió algèbrica de les funcions representades i calcula’n el pendent i l’ordenada en
l’origen.

૛ ૚ ૠ ૞
a) ࢌሺ࢞ሻ ൌ ࢞ െ c) ࢌሺ࢞ሻ ൌ e) ࢌሺ࢞ሻ ൌ െ૜࢞ ൅
૜ ૛ ૛ ૛

૛࢞ି૜ ૛
b) ࢌሺ࢞ሻ ൌ െ૛ d) ࢌሺ࢞ሻ ൌ f) ࢌሺ࢞ሻ ൌ െ ࢞
૞ ૜

522
Funcions 99

a)                     c)                      e) 

             
b)                      d)                    f) 

             
 

59. Escriu l’expressió algèbrica de les funcions representades i calcula’n el pendent i l’ordenada en
l’origen.

 
f(x)  2 →     Pendent: m  0    Ordenada a l’origen: n  2 
g(x)  x  3 → Pendent: m  1   Ordenada a l’origen: n  3 
h(x)  x  1 →  Pendent: m  1   Ordenada a l’origen: n  1 
1 1
i(x)   x  →    Pendent:  m     Ordenada a l’origen: n  0 
3 3

 
60. Representa aquestes funcions en els mateixos eixos de coordenades i relaciona l’obertura de les
branques de cada paràbola amb el coeficient de x2.

a) ࢌሺ࢞ሻ ൌ ‫ ܠ‬૛ c) ࢎሺ࢞ሻ ൌ ૛‫ ܠ‬૛

૚ ૚
b) ࢍሺ࢞ሻ ൌ ‫ ܠ‬૛ d) ࢏ሺ࢞ሻ ൌ ‫ ܠ‬૛
૛ ૝

523
Funcions
Funcions 9

 
f(x)  g(x) 

h(x) 
i(x) 

 
 
 
 
 
L’obertura és més petita quan el coeficient és més gran.
61. Determina les coordenades del vèrtex de les funcions quadràtiques següents:

a) f(x) = x2 – 6x + 10 c) f(x) = x2 – 4

b) f(x) = – x2 – 4x + 10 d) f(x) = –x2 – 4x + 2

æ -(-6) 62 - 4 ⋅ 1⋅ 10 ö÷ æ -4 ⋅ 1⋅ (-4) ÷ö
a)  V çç ,- ÷÷   V(3, 1)          c)  V ççç0, - ÷   V(0, 4) 
çè 2 ⋅ 1 4 ⋅1 ÷ø è 4 ⋅ 1 ÷ø

æ -(-4) 42 - 4 ⋅ (-1) ⋅ 10 ÷ö æ -(-4) 42 - 4 ⋅ (-1) ⋅ 2 ö÷


b)  V ççç ,- ÷÷   V(2, 14)       d)  V ççç ,- ÷÷÷   V(2, 6) 
èç 2 ⋅ (-1) 4 ⋅ (-1) ø÷ èç 2 ⋅ (-1) 4 ⋅ (-1) ø

 
62. Representa gràficament les paràboles següents:

a) f(x) = x2 + x – 6 c) f(x) = x2 – 3x + 5

b) f(x) = x2 – 10x + 25 d) f(x) = –x2 – 3x +1

a)                              c) 

                    

524
Funcions 99

b)                               d) 

                    
63. Associa cada funció amb la seva gràfica.

a) ࢌሺ࢞ሻ ൌ െ࢞૛

b) ࢍሺ࢞ሻ ൌ െ࢞૛ ൅ ૜

c) ࢎሺ࢞ሻ ൌ െ࢞૛ െ ૜

d) ࢏ሺ࢞ሻ ൌ െ૛࢞૛

 
a) Blava. b) Morada. c) Vermella. d) Verda.
 
64. Relaciona cada gràfica amb la seva expressió algèbrica.

࢞૛
a) ࢟ ൌ ൅ ૜࢞ െ ૚

b) ࢟ ൌ ૛࢞૛ െ ૛࢞ ൅ ૚

࢞૛
c) ࢟ ൌ െ െ࢞൅૛

d) ࢟ ൌ െ૛࢞૛ ൅ ࢞ െ ૚

525
Funcions
Funcions 9


a) y = f(x), perquè si a = > 0, la paràbola és oberta cap amunt i c = –1.

b) y = g(x), si a = 2 > 0, la paràbola és oberta cap amunt i c = 1.



c) y = h(x), perquè si a = െ < 0, la paràbola és oberta cap avall i c = 2.

d) y = i(x), ja que si a = –2 < 0, la paràbola és oberta cap avall i c = –1.

65. Representa els mateixos eixos de coordenades les funcions següents:

a) f(x) = x2 c) h(x) = (x – 4)2

b) g(x) = x2 + 3 d) i(x) = 2x2

En què s’assemblen i en què es diferencien aquestes funcions?

i(x)

f(x)

g(x)

h(x) 

 
Totes són paràboles i el vèrtex és un mínim, ja que el coeficient de x2 és més gran que 0.
S’obtenen desplaçant la paràbola x2 o ampliant/reduint l’obertura de les seves branques.

66. Fes la gràfica de la funció f(x) = x2 + 2x. Determina l’expressió algèbrica de les funcions següents i
representa-les:

a) f(x – 2) b) f(x) – 4 c) f(x + 1) d) f(x) + 2

Hi ha cap relació entre aquestes gràfiques?

       

526
Funcions 99

     
 
67. Construeix la taula de valors i dibuixa la gràfica d’aquestes funcions:

a) f(x) = x3 + 2 x2 + 3 b) f(x) = – x3 + 6x + 1

 
 
68. Representa les funcions polinòmiques següents i indica’n els punts de tall amb els eixos:

a) f(x) = x3 – x2 – 9x + 9 b) f(x) = 2x3 – 9x2 + x + 12

c) f(x) = x3 – 2x2 – 7x – 4 d) f(x) = x3 – 2x2 – 2x – 3

527
Funcions
Funcions 9

a) Punts de tall amb l’eix X: (–3, 0), (1, 0) i (3, 0).

Punt de tall amb l’eix Y: (0, 9)


b) Punts de tall amb l’eix X: (–1, 0), ቀ ǡ ૙ቁ i (4, 0).

Punt de tall amb l’eix Y: (0, 9)

c) Punts de tall amb l’eix X: (–1, 0) i (4, 0).

Punt de tall amb l’eix Y: (0, 4)

d) Punts de tall amb l’eix X: (3, 0).

Punt de tall amb l’eix Y: (0, –3)

 
   

528
Funcions 99

69. Calcula i representa les funcions polinòmiques de grau mínim que passen pels punts següents:


a) A(0, 0), B ቀ૞ǡ ቁ i C(–2, –1)

b) A(3, 0), B(4, 1) i C(5, 0)

c) A(1, 0), B(2, 1), C(3, 0) i D(4, 1)

x
a) Els punts A, B y C es troben alineats. La funció que passa per ells és f ( x ) = .
2

     
b) Sigui f(x) = ax + bx + c
2

L’expressió de la funció és: f(x) = – x2 + 8x – 15

c) Sigui f(x) = ax3 + bx2 + cx + d

529
Funcions
Funcions 9

L’expressió de la funció és:

70. Associa cada gràfica amb la funció corresponent.


a) ࢌሺ࢞ሻ ൌ
࢞ା૜


b) ࢍሺ࢞ሻ ൌ
࢞ି૝


c) ࢎሺ࢞ሻ ൌ ൅૛

 
a) Blava. b) Verda. c) Vermella.

71. Relaciona cada gràfica amb la seva funció.


a) ࢌሺ࢞ሻ ൌ ൅૜
࢞ି૛


b) ࢍሺ࢞ሻ ൌ െ૛
࢞ା૝

 
a) Vermella. b) Verda.

530
Funcions 99


72. Donada la funció ���� � , determina l’expressió algèbrica de les funcions següents:

a) g(x) = f(x – 3) d) g(x) = f (x) + 3

b) g(x) = f(x + 1) e) g(x) = f (–x)

c) g(x) = f(x) – 2 f) g(x) = – f(x)

� � ���� � �
a) ���� � c) ���� � � � � e) ���� � ��
��� � � �� �

� � ���� �
b) ���� � d) ���� � � � � f) ���� � �
��� � � �

 
73. Sense representar-les, escriu la relació que hi ha entre les gràfiques d’aquestes funcions i la de
��
���� � .

�� �� ��
a) ���� � b) ���� � �� c) i��� � �
��� � �

a) La gràfica de g(x) s’obté desplaçant la gràfica de f(x) 4 unitats a l’esquerra sobre l’eix X.

b) La gràfica de h(x) s’obté desplaçant la gràfica de f(x) una unitat cap amunt sobre l’eix Y.

c) La gràfica de i(x) és la simètrica de f(x) amb respecte a l’eix X; que equival a la simètrica respecte
amb l’eix Y.


74. Representa la gràfica de la funció ���� � . A partir d’aquesta gràfica, dibuixa les funcions

següents:

��� ���� �����


a) ���� � b) ���� � c) i��� �
��� ��� ���

3
a)  g(x)     1 → g(x)  f(x  1)  1  f(x)
x +1
h(x)
3
b)  h(x)  2    → h(x)  f(x  1)  2  g(x) 
x -1
3
c)  i(x)  2    → i(x)  f(x  1)  2 
x +1

 
i(x)
 
 
75. Representa gràficament les funcions següents:

� � �
a) ���� � � b) ���� � c) i��� � �
��� ��� ���

531
Funcions
Funcions 9

a) b) c)

         
 
76. Calcula el domini d’aquestes funcions:

a) ���� � √�� � � c) ���� � √�� � � �

b) ���� � ��� � � d) ���� � � � √�

1 é1 ö
a) 3x – 1 � 0 → x → Dom f = ê , + ¥÷÷÷ c) Dom f = 
3 êë 3 ø

b) x2 – 4 � 0 → 2 � x, x � – 2 → Dom f = (– ∞, –2] U [2, +∞) d) Dom f = [0, +∞)


 
77. Quin és el domini d’aquestes funcions amb radicals?

�� √����
a) ���� � b) ���� �
��√��� ��√���

a)  x  5  0 → x  5     x - 5   2 → x  5  4 → x  9 

Dom f  [5, 9) U (9, ∞)  [5, ∞)  {9} 
1
b)  3x  1  0 → x   x  1  0 → x  1    x + 1   4 → x  1  16 → x  15 
3     
é1 ö é1 ö
Dom f   ê , 15÷÷÷  U (15, ∞)   ê , + ¥÷÷÷   {15} 
ê3ë ø ê3ë ø

78. Representa gràficament les funcions següents:

� �
a) ���� � c) ���� � � ��
√� √�

� ��
b) ���� �
√���
d) ���� � √��� � �

532
Funcions 99

a) c)

b) d)

 
79. Comprova si aquests parells de funcions són inverses:

���
a) ���� � �� � � ���� �

���
b) ���� � ���� � � � ��


c) ���� � �� � � ���� � √� � �

x +5
a)  x  2y  5 → y    → f 1(x)  g(x) → Són inverses. 
2
3-y
b)  x    → y  3  4x → f 1(x)  g(x) → Són inverses. 
4

c)  x  y3  1 → y   3 x - 1  → f 1(x)  g(x) → Són inverses. 


 
80. Calcula, si és possible, la inversa d’aquestes funcions:

a) ���� � �� � � d) ���� � �� � �

b) ���� � �� � � e) ���� � √� � ��
� �
c) ���� � f) ���� �
��� ���

533
Funcions
Funcions 9

x +1 x +1
a)  x  2y  1 → y    → f 1(x)    
2 2

b)  x  y2  5 → y   x + 5  → No existeix la inversa, f -1(x) no és una funció, perquè per a cada valor de x
s’obtenen dues imatges.

1 1- 2x 1- 2x
c)  x    → y    → f 1(x)    
y +2 x x

d)  x  y2 + y → No existeix la inversa; f -1(x) no és una funció, perquè per a cada valor de x s’obtenen dues
imatges. 

2 - x2 2 - x2
e)  x   2 - 5y  → y    → f 1(x)    
5 5

1 2x + 1 2x + 1
f)  x    → y    → f 1(x)    
y -2 x x

 
81. Dibuixa la gràfica de la inversa de cada funció.

a) c)

b) d)

 
 
 
 
82. Calcula les funcions inverses d’aquestes funcions:

a) ���� � ���� � �� d) ���� � ������


b) ���� � � � � � � e) ���� � |� � �|

������ ������ �
c) ���� � d) ���� �
� �

a)  x  ln(y  3) → ex  y  3 → y  ex 3 → f 1(x)  ex  3 
534
Funcions 99

x -3
b)  x  3  4  5y → log5   y → y  log5(x  3)  log54 → f 1(x)  log5(x  3)  log54 
4
1 + tg y
c)  x    → y  arc tg(2x  1) → f 1(x)  arc tg(2x  1) 
2
arc sin x arc sin x
d)  x  sin 2y → y    → f 1(x)    
2 2

e)  x  │y  1│ → No existeix inversa, ja que per a cada valor de x, f -1(x) hi ha dues imatges; f -1(x) no és
una funció. 
1+ log 3 y
f)  x    → 5x  1  log3y → y  35x  1 → f 1(x)  35x  1 
5
 

83. Associa cada gràfica amb la funció corresponent.

 
࢞ ૚ ࢞
a) ࢌሺ࢞ሻ ൌ ૚૛ b) ࢍሺ࢞ሻ ൌ ૛࢞ c) ࢎሺ࢞ሻ ൌ ቀ ቁ

a) f(x) → Vermella. b) g(x) → Verda. c) h(x) → Morada.

84. Representa en els mateixos eixos de coordenades les funcions exponencials següents:
࢞ ࢞
a) ࢌሺ࢞ሻ ൌ ૛ ࢍሺ࢞ሻ ൌ ૞࢞ ࢎሺ࢞ሻ ൌ ૚૙

૚ ࢞ ૚ ࢞ ࢞
b) ࢌሺ࢞ሻ ൌ ቀ ቁ ࢍሺ࢞ሻ ൌ ቀ ቁ ࢎሺ࢞ሻ ൌ ૚૙
૛ ૞

a) b)
g(x) 
f(x)  g(x) h(x)
h(x)  f(x)

535
Funcions
Funcions 9

85. A partir de la gràfica de la funció exponencial g(x) = 4x, representa les funcions següents:

 
a)  4 x -3
= g ( x - 3)               b)  4 x +1
= g ( x + 1)            c)  1+ f ( x ) = 1+ 4 x  

          

d) - f ( x) = -4
x
e) 2 - f ( x) = 2 - 4
x x
f) f ( x ) - 1 = 4 - 1  
                     

                                              
 
86. A partir de la gràfica de la funció exponencial g(x) = 4x, representa les funcions següents:

૛࢞
a) ࢌሺ࢞ሻ ൌ ૝ b) ࢌሺ࢞ሻ ൌ ૝૛

536
Funcions 99

a) b)

87. Representa en els mateixos eixos de coordenades les funcions logarítmiques següents:

a) ࢌሺ࢞ሻ ൌ ࢒࢕ࢍ૛ ࢞ ࢍሺ࢞ሻ ൌ ࢒࢕ࢍ૞ ࢞ ࢎሺ࢞ሻ ൌ ࢒࢕ࢍ૚૙ ࢞

b) ࢌሺ࢞ሻ ൌ ࢒࢕ࢍ૚ ࢞ ࢍሺ࢞ሻ ൌ ࢒࢕ࢍ૚ ࢞ ࢎሺ࢞ሻ ൌ ࢒࢕ࢍ ૚ ࢞


૛ ૞ ૚૙

a) b)

f(x) 

g(x) 
h(x) 
h(x)
g(x)
f(x)

88. Associa cada gràfica amb la seva funció.

a) Morada. b) Vermella. c) Verda.


89. A partir de la gràfica de la funció logarítmica g(x) = log x, representa les funcions següents:

 
a) d)
537
Funcions
Funcions 9

b) e)

c) f)

90. A partir de la gràfica de la funció logarítmica g(x) = log x representa aquestes funcions:

a) ࢌሺ࢞ሻ ൌ ࢒࢕ࢍ૛࢞ b) ࢌሺ࢞ሻ ൌ ࢒࢕ࢍ

æxö
a) f(x) = log 2x = log 2 + log x b) f(x) = log ççç ÷÷÷ = log x – log 2
è2ø

                
91. Dibuixa la gràfica de la funció g(x) = cos x; a partir d’aquesta funció representa les següents:

538
Funcions 99


a) ࢌሺ࢞ሻ ൌ ࢉ࢕࢙ሺെ࢞ሻ d) ࢌሺ࢞ሻ ൌ ࢉ࢕࢙ ቀ࢞ െ ቁ

b) ࢌሺ࢞ሻ ൌ െࢉ࢕࢙࢞ e) ࢌሺ࢞ሻ ൌ ࢉ࢕࢙࢞ ൅ ૛

c) ࢌሺ࢞ሻ ൌ ࢉ࢕࢙ሺ࢞ ൅ ࣊ሻ f) ࢌሺ࢞ሻ ൌ ૚ െ ࢉ࢕࢙࢞

a) d)

f(x)  g(x)  f(x)
g(x) 

b) e)

f(x)  f(x) 

g(x)  g(x)

c) f)

f(x)
f(x) 

g(x)  g(x)

92. Representa la gràfica de la funció g(x) = sin x; a partir d’aquesta funció representa les següents:


a) ࢌሺ࢞ሻ ൌ ࢙࢏࢔ሺെ࢞ሻ d) ࢌሺ࢞ሻ ൌ ࢙࢏࢔ ቀ࢞ െ ቁ

b) ࢌሺ࢞ሻ ൌ െ࢙࢏࢔࢞ e) ࢌሺ࢞ሻ ൌ ࢙࢏࢔࢞ ൅ ૛

c) ࢌሺ࢞ሻ ൌ ࢙࢏࢔ሺ࢞ ൅ ࣊ሻ f) ࢌሺ࢞ሻ ൌ ૚ െ ࢙࢏࢔࢞

539
Funcions
Funcions 9

a) d)

f(x) 
f(x)

g(x) 
g(x) 

b) e)

f(x) 

f(x) 

g(x)  g(x)

c) f)

f(x)
f(x) 

g(x)  g(x) 

 
93. A partir de les gràfiques de les funcions g(x) = sin x, h(x) = sin 2x, representa les funcions
següents:

a) ࢌሺ࢞ሻ ൌ ࢙࢏࢔૝࢞ b) ࢌሺ࢞ሻ ൌ ࢙࢏࢔

540
Funcions 99

a) b)

g(x)  g(x) 
h(x)  h(x) 
f(x) 
f(x) 


94. A partir de les gràfiques de les funcions g(x) = cos x, h(x) = cos , representa les funcions

següents:

a) ࢌሺ࢞ሻ ൌ ࢉ࢕࢙ b) ࢌሺ࢞ሻ ൌ ࢉ࢕࢙૛࢞

 
a) b)

f(x)  f(x)  h(x) 

g(x)  h(x)  g(x) 

 
95. A partir de la gràfica de la funció trigonométrica g(x) = tg x, representa les funcions següents:

a) ࢌሺ࢞ሻ ൌ ࢚ࢍሺ࢞ ൅ ࣊ሻ b) ࢌሺ࢞ሻ ൌ ૚ െ ࢚ࢍ࢞

541
Funcions
Funcions 9

 
 
96. A partir de la gràfica de la funció g(x) = arcsin x, dibuixa les gràfiques d’aquestes funcions:

a) ࢌሺ࢞ሻ ൌ ૛ െ ࢇ࢘ࢉ࢙࢏࢔࢞ c) ࢌሺ࢞ሻ ൌ ࢇ࢘ࢉ࢙࢏࢔ሺ࢞ ൅ ૚ሻ

૚ ૚
b) ࢌሺ࢞ሻ ൌ ൅ ࢇ࢘ࢉ࢙࢏࢔࢞ d) ࢌሺ࢞ሻ ൌ ࢇ࢘ࢉ࢙࢏࢔ ቀ࢞ െ ቁ
૛ ૛

 
a) c)
 
f(x) 
 
g(x) 
 
  g(x)  f(x)

   
 
 
b) d)
 
 
f(x) 
  f(x)  g(x) 

  g(x) 

 
 
 
97. A partir de la gràfica de la funció g(x) = arccos x, dibuixa les gràfiques d’aquestes funcions:

a) ࢌሺ࢞ሻ ൌ ૚ ൅ ࢇ࢘ࢉࢉ࢕࢙࢞ c) ࢌሺ࢞ሻ ൌ ࢇ࢘ࢉࢉ࢕࢙ሺ࢞ ൅ ૚ሻ

૚ ૚
b) ࢌሺ࢞ሻ ൌ െ ࢇ࢘ࢉࢉ࢕࢙࢞ d) ࢌሺ࢞ሻ ൌ ࢇ࢘ࢉࢉ࢕࢙ሺ࢞ െ ሻ
૛ ૛

a) c)

f(x)  f(x)
g(x) 
g(x) 

542
Funcions 99

b) d)
g(x) 
g(x) 
f(x) 

f(x) 


98. Calcula f(–1 ), f(0) i fቀ ቁ en aquesta funció:

࢞૛ ൅ ૚࢙࢏࢞ ൏ ૙
ࢌሺ࢞ሻ ൌ ቐ࢞૛ ࢙࢏࢞ ൌ ૙
࢞૛ െ ૚࢙࢏࢞ ൐ ૙

▪  x  1: 
  x  0 → f(1)  (1)2  1 → f(1)  2 
▪  x  0: 
  f(0)  02 → f(0)  0 
1
▪  x   : 
2
2
æ 1ö æ 1ö æ 1ö 3
  x  0 →  f ççç ÷÷÷    ççç ÷÷÷   1 →  f ççç ÷÷÷    -  
è2ø è 2ø è2ø 4

 
99. Indica quina de les gràfiques següents correspon a aquesta funció:

࢞ െ ૚࢙࢏࢞ ൏ ૚
ࢌሺ࢞ሻ ൌ ቄ
૛࢞ െ ૜࢙࢏࢞ ൒ ૚

 
A f(x) li correspon la gràfica a), ja que el punt x = 1 pertany al segon tros.
 
100. Representa gràficament les funcions següents:

࢞ െ ૜࢙࢏࢞ ൑ ૙ ࢞ ൅ ૚࢙࢏࢞ ൐ ૙
a) ࢌሺ࢞ሻ ൌ ൝ ૛࢙࢏૙ ൏ ࢞ ൏ ૜ b) ࢌሺ࢞ሻ ൌ ൝ ૚࢙࢏࢞ ൌ ૙
െ࢙࢞࢏࢞ ൒ ૜ െ࢞ ൅ ૚࢙࢏࢞ ൏ ૙

543
Funcions
Funcions 9

a) b)

 
101. Representa aquesta funció:

࢞૛ ൅ ૜࢙࢞࢏࢞ ൏ െ૚
ࢌሺ࢞ሻ ൌ ൝െ૝࢙࢏࢞ ൌ െ૚
െ࢞ ൅ ૜࢙࢏࢞ ൐ െ૚
Estudia el valor que adopta la funció en els punts propers a –1 i completa les taules.

Esquerra de –1 –2 –1,5 –1,1 –1,05


f(x)

Dreta de –1 0 –0,5 –0,9 –0,95


f(x)

Descriu el que li passa a la funció als voltants de –1.

 
 

Esquerra de 1  2  1,5  1,17  1,05  Dreta de 1  0 0,5  0,9  0,95 


f(x)  2  2, 25  2, 09  2, 0457  f(x) 3 3, 5  3, 9  3, 95
 
A la dreta de x = –1, les imatges tendeixen a 4, és a dir, – (–1) + 3.

544
Funcions 99


102. La funció que té g(x) = com a expressió algèbrica s’anomena funció signe de x.
|�|

Troba’n l’expressió algèbrica com una funció definida a trossos i respon les preguntes següents:

a) Quant val si x = 3? c) Quant val si x = –3,4?

b) Quant val si x = –5? d) Quant val si x = 0?

������������������� � �
���� � �
���������������� � �

a) f(3) = 1 b) f(–5) = –1 c) f(–3, 4) = –1 d) No hi ha imatge de x = 0.

103. Representa i descriu les característiques de les funcions següents:


a) ���� � �� � �� � ������������������� � �
�� � ����������������������������� � �

�� �������������������������������������������������� � ��
�����
b) ���� � � �������������������������������� � � � � � �

��� � ������������������������������������������ � �

� ��� ���������������������������������������������� � ��
c) ���� � ��� � ��������������������������� � � � � � �
� �
� �������������������������������������������������� � �
���

a) ▪ Primer interval (∞, 0):


Paràbola, decreixent fins a x = 1 punt al qual se situa el vèrtex (1,
4), i creixent a la resta, acaba al punt (0, 3).

▪ Segon interval [0, �∞):

Recta creixent començant en el punt (3, 0) inclòs.

És continua a tot el domini.

Dom f =  Im f = [4, +∞)

b) ▪ Primer interval (∞, 2]:

Paràbola, decreixent a l’interval finalitzant a (2, 4) inclòs.

▪ Segon interval (2, 1):

Recta decreixent amb extrems a (2, 4) i (1, 3), aquest darrer punt
no inclòs.

545
Funcions
Funcions 9

▪ Tercer interval [3, +∞):

Paràbola decreixent a l’interval començant a (3, 0) amb vèrtex al punt


æ3 9 ö÷
çç , ÷.
çè 2 4 ø÷

Té una discontinuïtat de salt finit en x = 1.

æ 9ù
Dom f =  Im f = ççç-¥, ú È (3, + ¥)
è 4 úû

c) ▪ Primer interval (∞, 2):

Funció racional, decreixent en aquest interval.

▪ Segon interval [2, 2]:

Paràbola amb màxim en (0, 4) i extrems en (2, 0) i (2, 0).

▪ Tercer interval (2, +∞):

Funció racional, decreixent a l’interval.

Té discontinuïtats de salt infinit a x = 2 i x = 2.

Dom f =  Im f = (∞, +∞]

104. Representa i descriu les característiques d’aquestes funcions definides a trossos:

�� ������������������������������������������������������� � �

a) ���� � ���� �������������������������������������������� � � � �
√������������������������������������������������������� � �

�� ������������������������������������� � �
b) ���� � �
������������������������������������� � �

a) Dom f = � – |3| Im f = (–∞, 0] ∪ [2, +∞)

La funció és creixent a l’interval (–∞, 0] ∪ [4, +∞) i és decreixent a


(0, 3) ∪���������

Té un mínim relatiu en x = 4.

No és contínua a x = 0, ni a x = 3, i el punt x = 0 és de
discontinuitat inevitable de salt finit, i el punt x = 3 és de
discontinuitat inevitable de salt infinit.

Té una asímptota vertical a x = 3.

No és simètrica ni periòdica.

546
Funcions 99

b) Dom g = � – |3| Im g = (0, 2]

La funció és creixent a (–∞, 1) ∪ [1, +∞).

No té màxims ni mínims.

No és contínua a x = 1, i aquest punt és de discontinuitat


inevitable de salt finit.

No té asímptotes.

No és simètrica ni periòdica.

105. Representa gràficament les funcions següents:

a) f(x) = |2x – 3| c) f(x) = |x2 + 6x + 5|

b) f(x) = |–3x + 1| d) f(x) = |–x2 + x – 2|

a) c)

b) d)

                   
 
106. Expressa com una funció definida a trossos.

a) f(x) = |x| + |x + 2| c) f(x) = |x – 1| – |1 – x|

b) f(x) = |x + 1| – |1 – x| d) f(x) = |2 x + 1| – |2 – x|

547
Funcions
Funcions 9

 
107. Expressa com una funció definida a trossos.

1
a) x2 - 4 b) c) 3
x -3    
x+2

ìï x 2 - 4 si x £ -2
ìï- x 2 + 4 si x 2 - 4 < 0 ïï
ï
a)  x - 4 = í 2
2
= ï í- x 2
+ 4 si -2< x <2  
ïîï x - 4 si x 2 - 4 ³ 0 ïï 2
ïïî x - 4 si 2 £ x

ïìï 1 ïì 1
ïï- si x + 2 < 0 ïï- si x < -2
1 x + 2 ï x + 2
b)  = ïí = ïí  
x + 2 ïï 1 ïï 1
ï
ï si x + 2 > 0 ïïîï x + 2 si x > - 2
ïx +2
î

ïìï 1 ïì 1
ïï- - sin x si x + 2 < 0 ïï- - sin x si x < -2
1 ï x+2 ï x+2
ï
    - sin x = í =í  
x+2 ï 1 ï 1
ïïï - sin x si x + 2 > 0 ïïï - sin x si x > -2
îï x + 2 îï x + 2

ì- 3 x - 3
ï si x - 3 < 0 ïì- 3 x - 3 si x < 3
c)  3
x -3 =ï
í3 = ïí  
ï si x - 3 ³ 0 ïïîï 3 x - 3
ï x -3
ï
î si x ³ 3

 
����
108. Donades ���� � �������� � � , calcula.
���


a) (f + g)(2) b) (f · g)(1) c) (f – g)(3) d) � � ���

5⋅ 2 -1 29
a)  (f  g)(2)     4 → (f  g)(2)    
2+3 5

5 ⋅ 1- 1
b)  (f  g)(1)     4 → (f  g)(1)  4 
1+ 3

5⋅ 3 -1 5
c)  (f  g)(3)     4 → (f  g)(3)   -  
3+3 3

æf ö 5⋅ 0 -1 æf ö 1
d)  ççç ÷÷÷(0)     : 4 →  ççç ÷÷÷(0)    -  
è g ø÷ 0+3 è g ø÷ 12

 
109. Calcula el domini d’aquestes funcions:

���� � √�� � � ���� � √�� � ��

548
Funcions 99

Utilitza el resultat que has obtingut per calcular el domini de les funcions següents:

� �
a) (f + g)(x) b) (f · g)(x) c) � � ��� d) � � ���
� �

Dom f = (–∞, –2] ∪ [2, +∞)

Dom g = [–5, 5]

a) Dom (f + g) = [–5, –2] ∪ [2, 5]

b) Dom (f · g) = [–5, –2] ∪ [2, 5]


c) Dom � � =([–5, –2] ∪ [2, 5)


d) Dom � � = [–5, –2) ∪ (2, 5]

110. Donades les funcions:


f(x) = 3x – 1 g(x) = h(x) = √� � �
���

defineix les funcions següents:

a) �� � ����� e) � � ���

b) �� � �� � ������ f ) ��� � �����

c) �� � ����� g) �� � � � �� ����

� ���
d) � � ��� h) � � ���
� �

1 3x2 - 7x + 3
a)  (f  g)(x)  f(x)  g(x)  3x  1    → (f  g)(x)    
x -2 x -2

1 3 x 2 - 7 x + 1- ( x - 2) x + 1
b)  (f  (g  h))(x)  f(x)  (g  h)(x)  f(x)  g(x)  h(x)  3x  1      x + 1     
x -2 x -2

1 3x -1
c)  (f  g)(x)  f(x)  g(x)  (3x  1)       
x -2 x -2

ægö g( x ) 1 x +1
d)  ççç ÷÷÷( x )       : x + 1    2  
è ø h h( x ) x -2 x - x -2

e)  f 2(x)  (3x  1)2  9x2  6x  1 

f)  (h2  f)(x)  h2(x)  f(x)  x  1  3x  1  4x 

3x -1 x2 + 2x - 3
g)  (g  f  h2)(x)  (f  g)(x)  h2(x)     x  1    
x -2 x -2

æ f + g ö÷ (f + g )(x) 3x2 - 7x + 3 3x2 - 7x + 3 (3 x 2 - 7 x + 3) x + 1


h)  ççç ÷÷( x )       : x + 1        
è h ø h( x ) x -2 ( x - 2) x + 1 x2 - x - 2

   

549
Funcions
Funcions 9


111. Donades les funcions ���� � ��� � ���������������� � � �, calcula:

a) �������� b) �������� c) �������� d) ��������

2
æ5 ⋅ 2ö
a)  (f ◦ g)(2)  f(g(2))  4 ççç ÷÷÷   11  111 
è 2 ø

5 135
b)  (g ◦ f)(2)  g(f(2))    (4  22  11)    
2 2

c)  (f ◦ f)(2)  f(f(2))  4(4  22  11)  11  119 
5 5 25
d)  (g ◦ g)(2)  g(g(2))        2   . 
2 2 2

 
112. Calcula (f º g)(2) si f i g són funcions que compleixen que f(10) + 5 = 0, g(2) – 10 = 0.

f(10)  5, g(2)  10 → (f ◦ g)(2)  f(g(2))  f(10)  5 
 
113. Donades les funcions f(x) = 4 i g(x) = – 2x2 + 6x, calcula f º g i g º f.

(f ◦ g)(x)  f(g(x))  4        (g ◦ f)(x)  g(f(x))  g(4)  2  42  6  4  8 
 
114. Donades les funcions f(x) = √� i g(x) = x2 – 5, calcula f –1 º g i g º f –1.

f 1(x)  x2 
(f 1 ◦ g)(x)  f ‐1(g(x))  (x2  5)2  x4  10x2  25 
(g ◦ f 1)(x)  g(f 1(x))  (x2)2  5  x4  5 
 
115. Comprova amb les funcions f(x) = √� � � i g(x) = 3x – 2 que la composició de funcions no és
commutativa. Calcula el domini de f º g i de g º f.

�������� � ������� � ���� � �� � √�� � �

�������� � ������� � ��√� � �� � �√� � � � �

�������� � �������� → La composició de funcions no és commutativa.


��������� � � � ���

��������� � ���� ���

 
116. Per a la funció h(x), troba dues funcions f(x) i g(x) que verifiquin que (f º g)(x) = h(x).

a) ���� � √� � � c) ���� � ��� � ���


b) ���� � √� � � d) ���� � � ��
���

550
Funcions 99

a) f(x)   x     g(x)  x  3          c) f(x)  x4      g(x)  3x  1 


1
b)  f(x)  x  3  g(x)   x            d) f(x)   x + 1     g(x)    
x -2

 
117. Explica de quina manera hem de compondré aquestes funcions:

ࢌሺ࢞ሻ ൌ ξ࢞૛ ൅ ૝ ࢍሺ࢞ሻ ൌ ૞࢞ ൅ ૚ ࢎሺ࢞ሻ ൌ
࢞ା૚

per obtenir les funcions següents:


࢞ା૚૚
a) ࢏ሺ࢞ሻ ൌ ૞ඥ࢞૛ ൅ ૝ ൅ ૚ c) ࢏ሺ࢞ሻ ൌ
࢞ା૚

b) ࢏ሺ࢞ሻ ൌ ૛૞࢞ ൅ ૟ d) ࢏ሺ࢞ሻ ൌ ඥ࢞૛ ൅ ૡ

d) (f  f )( x ) = f (f ( x )) = f ( x 2 + 4 ) = ( )
2
x2 + 4 +4 = x2 + 8

118. Un objecte que es llança cap amunt pot arribar a una altura màxima determinada per la funció, h(t)
= v0t - 4,9t2 + h0, en què v0 és la velocitat inicial de l’objecte, h0 és l’altura inicial, des d’on s’inicia
el moviment, i t és el temps.

Si llancen un coet pirotècnic des d’una plataforma situada a 2 m del terra amb una velocitat inicial
de 40 m/s:

a) A quina altura màxima arribarà el coet?

b) Si el programen perquè exploti als 5 s del llançament, a quina altura es produirà l’explosió?

a) h0 = 2 m v0 = 40 m/s → h(t) = 40t - 4,9t2 + 2


L’altura màxima és al vèrtex de la paràbola: (4,9; 80,35) → 80,35 m serà l’altura màxima que assolirà el
coet.
b) t = 5 s → h(t) = 40 · 5 – 4,9 · 52 + 2 = 79,5 m
   

551
Funcions
Funcions 9

119. El preu en euros d’un article perible que comencen a vendre el primer dia d’un mes determinat
varia amb el temps, en dies, segons la funció:


൅ ૡ࢙࢏૙ ൑ ࢚ ൑ ૝

ࡼሺ࢚ሻ ൌ ൞ ૛
െ࢚
൅ ૛࢚ ൅ ૞࢙࢏૝ ൏ ࢚ ൑ ૚૙

a) Quin és el preu inicial de l’article?

b) Dibuixa la gràfica de la funció P(t).

a)  t  0 → P(0)  8 → El preu és de 8 €.       


b)

P(x)

        

120. Un estudi sobre el medi ambient ha estimat que el nivell mitjà de monòxid de carboni en l’aire és
M(x) = 1 + 0,5x parts per milió quan el nombre de persones és x milers. Si la població en milers en
el moment t és:

P(t) = 200 + 257 + 0,3t2

 
a) Escriu la funció que expressa el nivell de monòxid de carboni en l’aire com una funció de
temps.

b) Calcula per a t = 10 el nivell de monòxid de carboni.

a) En aquest cas, per calcular quin és el nivell de monòxid de carboni en funció del temps hem de dur a
terme la composició de les dues funcions, així:

552
Funcions 99

( M  P)(t ) = M (P (t )) = 1+ 0,5(457 + 0,3t 2 )  

b) Per a t = 10, el nivell de monòxid de carboni serà

M (10) = 1+ 0,5(457 + 0,3 ⋅ 102 ) = 1+ 243,5 = 244,5  

121. El dibuix representa un sistema de tancs d’aigua, un dels quals és un tanc cúbic de costat x i
l’altre és un tanc d’altura x i base quadrada de costat b.

Determina una funció que expressi el volum total del sistema de tancs d’acord amb la longitud de
les arestes dels tancs.

 
El volum total ve determinat per la fórmula V ( x , b) = x + xb .
3 2

122. Per repoblar un llac introdueixen inicialment 50 individus d’una espècie de peixos que triplica el
nombre de membres cada dos mesos.

 
 

a) Quina és la fórmula de la funció que representa el creixement de la població de peixos tenint


en compte els mesos?

b) Quants peixos hi ha al cap de 4 anys?

c) Després de quant de temps la població de peixos serà de 1.000 individus?

a) Tenint en consideració les dades inicials, com que la població inicial són 50 peixos, al cap de dos
mesos hi haurà 150 peixos; als 4, 450; i als sis, 1.350, i així successivament:
m
f ( m) = 50 ⋅ 3 2

553
Funcions
Funcions 9

b) Per calcular quants peixos hi ha al cap de quatre anys, es pot emprar la mateixa fórmula de l’apartat
anterior, convertint prèviament els anys a mesos: 4 anys = 48 mesos.

50 ⋅ 324 = 14 121476 824 050


Per tant, després de 4 anys haurà peixos.
c) Per estimar quant temps es necessita, es calcula:
m m
50 ⋅ 3 2 = 1000  3 2 = 20  m = 2 log 3 20 = 5,45

I s’aproxima al següent mes, és a dir, 6 mesos.


 

123. En un llac hi ha una espècie de peix gran que s’alimenta d’una espècie de peixos més petits que,
al mateix temps, s’alimenta de plàncton. El nombre de peixos grans és una funció f(x) de la
quantitat x de peixos petits; el nombre de peixos petits és una funció g(y) de la quantitat y de
plàncton del llac. Expressa la población de peixos grans considerant el plàncton del llac si:


ࢌሺ࢞ሻ ൌ ૜૙ ൅ ට ࢍሺ࢟ሻ ൌ ૝࢟ െ ૚
૚૛૙

En aquest cas, l’únic que haurem de fer és la composició d’una funció amb l’altra, o el que és el mateix:
4y - 1
(f  g )( y ) = f (g ( y )) = 30 +
120

124. Segons la llei del refredament de Newton, la temperatura d’un objecte segueix la funció:

ࢌሺ࢚ሻ ൌ ࢀ ൅ ሺ࡯ െ ࢚ሻ ൉ ࢋି࢑࢚

en què T és la temperatura ambient, C és la temperatura inicial, t és el temps transcorregut i k


indica la taxa de refredament de l’objecte per unitat de temps.

Un objecte amb una temperatura de 40 ºC es deixa a l’aire lliure a una temperatura de 25 ºC i al


cap de 10 minuts la temperatura de l’objecte és de 34 ºC. Quant de temps ha de passar perquè
l’objecte es refredi fins a una temperatura de 30 ºC?

f (10) = 34 = 25 + (40 - 10) ⋅ e-10 k  k = 0,12 30 = 25 + (40 - t ) ⋅ e-0,12t  t » 13,802 min

125. Una ONG ha calculat que el nombre de persones ingressades en els hospitals després d’un
tsunami segueix aproximadament aquesta fórmula:

૚૚૙
ࡼሺ࢚ሻ ൌ ૚ ൅ ࢚ ‫ א‬ሺ૙ǡ ૜૙ሻ
࢚૛ ൅ ૚૙

en què P és el nombre de persones hospitalitzades, en milers, i t és el nombre de dies


transcorreguts des del tsunami.

a) Quantes persones hi haurà hospitalitzades el primer dia?


554
Funcions 99

b) I quantes n’hi haurà al cap de tres setmanes?

c) Si la capacitat hospitalària d’una illa de l’àrea afectada és de 2 000 llits, fins a quin dia es va
ultrapassar aquesta capacitat?

a) 11 000 persones

b) 1 243 persones

૚૚૙
c) ૚ ൅ ൌ ૛ ՜  ࢚૛ ൅ ૚૛૙ ൌ ࢚૛ െ ૚૙૙ ൌ ૙ ՜ ࢚ ൌ േ૚૙
࢚૛ ା૚૙

Atès que el nombre de persones hospitalitzades decreix segons el nombre de dies, la capacitat
d’hospitalització va ser desbordada fins al desè dia.

   

555
Funcions
Funcions 9

AMPLIA 
 
126. Escull la resposta adequada.

□ De les dades de l’enunciat es pot obtenir que si es formula la paràbola com ax 2 + bx + c , així:
b ïü
- = 2ïï
2a ïïï -b = 4 aüïï a = 2 ü ï
ï
ï ï
2 a + 2b + c = 1 ý  4 a + 2b + c = 1 ïý  b = -8ïý
2 ï
ïï ïï ï
16 a - 16 a + c = 9ïï c = 9 ïïþ c = 9 ïïï
þ
ïï
ïþ

La paràbola passa pel punt (1, 3), fruit de substituir x = 1 en 2x2 – 8x + 9.


□ Com que 3x – 1 = 5 → x = 2, substituint a la fórmula, amb la qual cosa s’obté el valor:
f (5) = f (3 ⋅ 2 - 1) = 23 + 2 + 1 = 8 + 2 + 1 = 11

p
□ El més senzill per resoldre aquest exercici és proposar un sistema d’equacions amb x = i amb
2
p
x = - . Això condueix a:
2

æpö æ pö üï
f çç ÷÷÷ + 2f çç- ÷÷÷ = 1 ïï
çè 2 ø çè 2 ø ïï 1ª-2⋅2ª æpö æpö
ý ¾¾¾-3f ççç ÷÷÷ = 3  f ççç ÷÷÷ = -1  
æ p ö÷ æ p ö÷ ïï è2ø è 2ø
f çç- ÷÷ + 2f çç ÷÷ = -1ïï
çè 2 ø çè 2 ø ïïþ

□ Si el primer que es fa és calcular la inversa de f(x), d’aquesta manera es veurà quin domini té
-1 -1
f ( x ) = 3 x + 5  x = 3f (x)
+ 5  x - 5 = 3f (x)
 f -1 ( x ) = log 3 ( x - 5 )  Dom f -1 = (5, + ¥ )

□ Per obtenir el nombre de vegades que f talla amb l’eix X, n’hi ha prou amb calcular les arrels de f(x).
1 1
f ( x) = sin = 0  x = , n ¹ 0, n Î 
x pn
1
Per quins valors s’obtindrà 1 > x = > 0,01 ?
np

N’hi haurà prou amb resoldre la inequació i comprovar a partir de quina n no s’aplica:
1 100
x= > 0,01  > n  n < 31,8 → Talla 31 vegades.
pn p

556
Funcions 99

127. Considera les funcions:

ࢋ࢞ െ ࢋି࢞ ࢋ࢞ ൅ ࢋି࢞
ࢌሺ࢞ሻ ൌ ࢍሺ࢞ሻ ൌ
૛ ૛

comprova que es compleix que

ሾࢍሺ࢞ሻሿ૛ െ ሾࢌሺ࢞ሻሿ૛ ൌ ૚
2 2
æ e x + e- x ö÷ æ e x - e- x ÷ö e2 x e-2 x 1 æç e2 x e-2 x 1 ÷ö
[ g ( x )] - [f ( x )] = ççç ÷÷ - çç - ÷÷   
2 2
÷÷ = + + - çç +
è 2 ø÷ è ç 2 ÷ø 4 4 2 è 4 4 2 ø÷

e2 x e-2 x 1 e2 x e-2 x 1 1 1
 + + - - + = + =1
4 4 2 4 4 2 2 2
 
 

128. Justifica per a quins valors de x es fa més gran la diferència ξ࢞૛ ൅ ૚ െ ȁ࢞ȁ.

La diferència assoleix el valor més gran quan x = 0. 
 

 
 

129. Determina quantes solucions tenen les equacions següents en l’interval [-ૈ, ࣊]?


a) ex = 2 – x 2 b) ln x = –x c) sin x =

       
Té dues solucions. Té una solució. Té tres solucions.
 

130. Les busques d’un rellotge fan 20 cm i 30 cm. Si en aquest moment les busques es troben entre
les 12.00 h i les 12.30 h, contesta les preguntes següents:

a) Expressa l’angle que formen d’acord amb el temps, t, mesurat en minuts.

b) Calcula l’àrea del triangle que es crea quan s’uneixen els extrems en funció de t.

557
Funcions
Funcions 9

Pot adoptar el valor 0?

A quina hora arriba al valor màxim?

c) Expressa la distància entre els extrems de les busques segons t.

a) Com que la busca que marca les hores triga 12 hores a fer la volta completa ( 2� radians), la velocitat
�� �
és: �� � � ��������
��� ���

�� �
De manera anàloga, la velocitat de l’altra busca és: �� � � ��������
�� ��

L’angle que formen ambdues busques és la diferència entre els angles recorreguts per cadascuna, en
funció del temps t transcorregut:

� � ���
�� �� �� �����
�� ��� ���

� ��� ���
b) � � � �� � �� � ��� � �� � �������� ���
� ��� ���

Aquesta funció s’anul·la si l’angle medeix kπ radians, amb k ∈ �. A l’interval de temps demanat
aquesta condició es compleix únicament a les 12 h (α=0).


Com que el valor més gran de la funció sinus s’assoleix quan l’angle és radians, s’ha de calcular a

quina hora l’angle format té aquesta amplitud.

��� �
� � � � � � ��� ��� L’àrea és màxima a les 12 hores i 16,36 minuts.
��� �

c) Pel teorema del cosinus, la distància entre les busques és:

��� ���
� � ���� � ��� � � � �� � �� � ��� � �� � ����� � ���� � ��� � ��
��� ���

���
� ����� � �� � ��� � ��
���

   

558
Funcions 99

131. Representa gràficament la funció següent en l’interval –8 ≤ x ≤ 8.

f (x) = |||x – 1| – 2| – 3|

 
 

132. Considera una funció per a la qual


ࢌሺ࢞ሻ ൅ ࢌ ቀ ቁ ൌ ࢞ si x és diferent de 0 i d’1.
૚ି࢞

Quin és el valor de f(2)?

Sabem que la fórmula és vàlida sempre que x és diferent de 0 i de 1.La millor idea és provar resoldre:
f(2)  f(1)  2 → f(2)  2  f(1) 
Encara manca saber el valor de f(–1). Com que x = –1 és diferent de 0 i de 1, es pot tornar a emprar
l’equació per veure el valor de f(–1), perquè possiblement això ajudi:
æ 1ö æ 1ö æ 1ö
f (-1) + f çç ÷÷÷ = -1  f (-1) = -1- f çç ÷÷÷  f (2) = 3 + f çç ÷÷÷
èç 2 ø çè 2 ø çè 2 ø
 
æ 1ö 1
Ara el valor desconegut és f ççç ÷÷÷ , no obstant, com que x = és diferent de 0 i de 1, es pot tornar a emprar
è 2ø 2
l’equació, i obtenim:
æ 1ö 1 æ 1ö 1 1 7 7
f çç ÷÷÷ + f (2) =  f çç ÷÷÷ = - f (2)  f (2) = 3 + - f (2)  2f (2) =  f (2) =  
çè 2 ø 2 ç
è 2ø 2 2 2 4

133. Tenim una col·lecció d’esferes idèntiques que amunteguem formant un tetraedre amb totes les
arestes igual a n esferes.

Calcula, tenint en compte n, el nombre total de punts de tangència (contactes) que hi ha entre les
esferes de la pila.

Podem sumar els contactes de cada bola i després dividir per 2. Tenim doncs:
1. 1 bola a cada un dels 4 vèrtex, amb tres contactes cadascuna: 4 · 3 = 12
2. n – 2 boles internes a cadascuna de les sis arestes, amb 6 contactes cadascuna: 6 ⋅ 6 ⋅ ( n - 2) = 36n - 72

3. T(n – 3) boles internes de cada cara, per 4 cares i 9 contactes cadascuna:


 

559
Funcions
Funcions 9

36 ( n - 3)( n - 2)
36 ⋅ T ( n - 3) = = 18 n2 - 90 n + 108  
2

n ( n + 1)
On T ( n) = = 1 + 2 + ... + n és l’enèsim nombre triangular.
2

4. P(n – 4) boles internes, amb 12 contactes cadascuna, essent P ( n) = T (1) + T (2) + ... + T ( n) l’enèsim
nombre piramidal:
n
1 n 2 1 æ n( n + 1)(2n + 1) n ( n + 1)ö÷ 1 1
P ( n) = å T ( k ) =
k =1
å k + k = 2 ççççè
2 k =1 6
+
2
÷÷ = n ( n + 1)(2n + 1 + 3) = n( n + 1)( n + 2) 
ø÷ 12 6

5. Llavors, 12P ( n - 4) = 2( n - 4)( n - 3)( n - 2) = 2n3 - 18n2 + 52n - 48 i en sumar els diferents tipus de boles
segons els seus contactes, i dividint entre dos per evitar repetir, obtenim:
( + 36 n - 72 + 18 n2 - 90 n + 108 + 2n3 - 18n2 + 52n - 48)
 12
= n3 - n = n ( n2 - 1) 
2
 
Per tant, en funció del nombre n de boles per costat, hi haurà n(n – 1) contactes, per tetraedres amb
2

costats formats per un nombre d’esferes n = 1, 2, …


   

560
Funcions 99

MATEMÀTIQUES A LA TEVA VIDA 
1. Quin gruix té l’estratosfera a l’equador?

L’estratosfera té a l’equador un gruix de 35 km.


 

2. Quina relació hi ha entre la temperatura i les capes de l’atmosfera?

Proporcionalitat directa, on la temperatura baixa a mesura que l’altura puja, a raó de 6oC.
 

3. A quina capa s'enregistra la temperatura més baixa?

A l’estratosfera, on s’assoleixen temperatures de fins a –53ºC.


 

4. D’acord amb el gràfic, quina temperatura té l’atmosfera a 110 km d’altura?

A 110 km d’altura es parla de ser dins de la termosfera, on hi pot haver fins a 1 500ºC, quan el Sol és
actiu.
 

5. On es troba la capa d’ozó? Investiga quina és la situació actual de la capa d’ozó i per què és tan
important recuperar-la i conservar-la.

La capa d’ozó es troba situada a l’estratosfera.

Actualment la capa d’ozó es troba en una situació de desgast greu degut als coneguts gasos d’efecte
hivernacle, cosa que provoca una filtració més petita als raigs ultraviolats, i que comporta un creixement
de problemes diversos com la fosa dels pols, problemes perquè les plantes facin la fotosíntesi,
melanomes, cataractes oculars...

Sense aquesta capa tots aquests problemes s’incrementarien de forma exponencial, cosa que implica un
dany enorme per a l’ecosistema i unes conseqüències horribles per a la humanitat. Per tant, cal
conscienciar totes les persones perquè es fomenti la seva recuperació i conservació, i tenir un món millor
on viure el dia de demà.

6. Si assumim que la temperatura a nivell del mar és de 20 ºC, quina temperatura hi deu haver
aproximadament al cim de l’Everest, a 8.848 m d’altura?

Sabent que a la troposfera per cada kilòmetre que s’ascendeix es perden 6 ºC, i amb la condició que a
nivell del mar hi ha 20ºC, si aproximem l’altura de l’Everest a 9 000 m, que són 9 km, implica que haurà
perdut 54ºC en l’ascens, per tant la temperatura que s’haurà de tenir al cim serà de 20 – 54 = –34oC.

561
Límit d’una funció
Límit d’una funció  10
10 

ACTIVITATS 
1. Calcula els termes a100, a101 i a102 de la successió amb aquest terme general:
���
�� �
��
49 99 25
a100 =             a101 =             a102 =  
5 000 10 201 2601

 
2. Determina el terme general d’aquestes successions:
a) 1, 3, 5, 7, … c) 0, 2, 6, 12, 20, …
� � �� � � �� ��
b) , , ,… d) , , , ,…
� �� �� � � � ��

a) an = 2 n - 1 c) an = n ( n - 1)
4n - 1 -2 n + 5
b) an = d) an =
5 ⋅ 3 n-1 n2

3. Determina el límit de les successions següents amb la calculadora:


���
a) �� � �������� c) �� � �� � �� e) �� �

� �
b) �� � �� d) �� � �, �� f) �� � � � � �

a) nlim
¥
an = 1 d) nlim
¥
an = 0

b) lim an = +¥ e) lim an = 1
n ¥ n ¥

c) nlim
¥
an = -¥ f) nlim
¥
an = -7

4. Escriu successions que tinguin com a límits els valors que hi ha indicats a continuació:
a) 0 d) 4 g) No tingui límit.
b) +3 e) -3 h) Un nombre a.
c) -3 f ) 0,5 i) ��
Resposta oberta. Per exemple:
4 4n - 1
a) an = d) an = g) an = (-1)n
5n + 1 n + 10

1- 3n an2
b) an = 3n3 - 7 e) an = h) an = 2
n ( n - 1)

n+5 a2 n
c) an = 4 - n f) an = i)
2n n-5

   

563
Límit d’una
Límit d’una funció
funció 10

5. Determina els límits següents:


a) ��� √� c) ��� ��
��� ���
� � �
b) ��� �� �� � �� � �� d) ��� � �
��� � ��� �

a) nlim
¥
n = +¥ c) nlim
¥
n3 = +¥

n
æ ö æ 3ö
b) lim ççç- 1 n 2 - 2 n + 7÷÷÷ = -¥ d) nlim çç ÷÷ = +¥
¥ èç 2 ø÷
n ¥ è 2 ø

6. Calcula els límits de les successions que tenen com a termes generals els següents:
10
8n (n -1)
a) b) 10
2n 2 + 3n -1 ( n + 2)
10
8n ( n - 1)
a) nlim =0 b) nlim 10
=1
¥ 2n2 + 3 n - 1 ¥
( n + 2)

 
7. Determina els límits següents:
æ 4n5 - n 2 3n2 -1÷ö 3n2 +1
a) lim ççç 6 - 2 ÷÷ d) nlim
n¥ çè 2n +1 n +1 ÷ø
2
¥ n +n+2

æ 3n2 -1 4n 4 ö÷
e) lim ççç n + 1 - n -1÷÷÷
æ ö
b) lim ççç 3 ⋅ 4 ÷÷
n¥ è ç n 2n + 3÷ø n¥ è n - 1 n + 1ø

n2 + 7 æ n3 (1- n) 4 ö÷
c) lim ln f) lim ççç 4 + 2 ÷÷
n¥ 2n n¥ ç 2n - n -1
è n ÷ø

æ 4n5 - n2 3n2 - 1ö
÷÷ = -3 3n2 + 1
a)  nlim çç 6 - 2 ÷              d)  nlim = 3 
¥ çè 2n + 1 n + 1 ø÷ ¥ 2
n +n+2

æ 3n2 - 1 4n4 ö÷ æ ö
b) nlim çç ⋅ ÷÷ = 0 e) lim ççç n + 1 - n - 1÷÷÷ = 0
¥ çè n
3
2n4 + 3 ÷ø n ¥ è n - 1 n + 1ø

n2 + 7 æ n3 (1- n) 4ö 1
c) nlim ln = +¥ f) nlim çç + ÷÷÷ = -
¥ 2n ¥ ç
çè 2 n4 - n - 1 n2 ÷ø 2

 
8. Calcula aquests límits:
æ n4 +1 ÷÷ -n3 ö 2 n 2 -1

a) nlim çç
¥ ç
+ 3 ÷ d) lim 9 n
2
+1

èç 2n +1 3n + n +1÷ø
4 n¥

æ 2n3 +1 ÷ö n+1

b) nlim
¥
ln ççç ÷
çè 2n3 + n ÷÷ø
e) n¥
lim 0,1 n
2

n+1
æ 2 ö (n + 1)(n -1) æ 2n 2 + 1ö÷ n2
c) lim ln ççç ÷÷÷⋅ f) lim ççç ÷÷
n¥ ç è n ÷ø
2 2
n¥ ènø (n + 1)

æ n4 + 1 -n3 ö 1 æ 2 ö ( n + 1)( n - 1)
n +1

a) nlim çç + ÷÷ = c) lim lnççç ÷÷÷ ⋅ → No existeix. e) nlim 2


0,1 n = 1
ç 2n4 + 1 3n3 + n + 1÷ø÷ 6
¥ è n ¥ è nø ( n + 1)
2 ¥

564
Límit d’una funció 1010

n +1
æ 2n3 + 1 ö÷ 2 n2 -1
æ 2 n2 + 1ö÷ n2
b) nlim lnçç 3 ÷= 0 d) nlim 9 n2 + 1
= 81 f) lim çç ÷÷ = 1
¥ çè 2n + n ÷÷ø ¥ n ¥ ç
è n
2 ÷ø

9. Calcula els límits següents:


n
æ n +1 2n 2 ö÷ æ5ö 1
a) nlim çç - ÷÷ c) nlim çç ÷÷
ç 2 ÷ø 3n - 2
¥ ç èç-n - 5 n + 2 ø÷ ¥ è

æ-2n +1 ÷÷ n2 ö -n - 3
b) nlim çç - d) nlim
¥ ç
÷
èç n - 5 n + 2 ÷ø
2 n
¥
(0,3)
æ n +1 ÷÷ = lim2n2 ö n +1 2n2
a) nlim çç - - lim = -1- ¥ = -¥
÷
¥ çè -n - 5 n + 2 ÷ø n¥ -n - 5 n¥ n + 2

æ -2n + 1 ÷÷ = lim n2 ö -2n + 1 n2


b) nlim çç 2 - ÷ - lim = 0 - ¥ = -¥
¥ ç
è n -5 n + 2 ÷ø n¥ n2 - 5 n¥ n + 2

n n
æ5ö 1 æ5ö 1
c) nlim ççç ÷÷÷ = lim çç ÷÷ ⋅ lim = +¥⋅ 0 → Indeterminació.
¥ è 2 ø 3n - 2 n¥ èç 2 ø÷ n¥ 3 n - 2

-n - 3 lim - n - 3 -¥
d) nlim n
= n ¥
n
= = -¥
¥
(0,3) lim (0,3)
n ¥
0

10. Determina aquests límits:


n
n æ 1 ÷ö
a) lim c) nlim çç ÷
n¥ n-2 ¥ çè n + 5÷ø
n
3n 2 - 6 æ 1- n ö
÷÷
b) lim log d) nlim çç
n¥ 3n 2 ¥ çè n - 2 ø÷

n lim n
n n æ 1 ö æ
÷ = ç lim 1 ön¥
÷
a) lim = lim = 1=1 c) nlim çç
¥ èç n + 5 ÷÷ø èççn¥ n + 5 ÷÷ø
= 0+¥ = 0
n ¥ n-2 n ¥ n - 2

n lim n
3 n2 - 6 3 n2 - 6 æ 1- n ö÷ æ 1- n ö÷n¥
b) nlim d) nlim çç ÷ = ççç lim
¥
log = log lim = log 1 = 0 ÷ = (-1) → No existeix.
¥ 3n 2 n¥ 3 n2 ¥ èç n - 2 ø÷ è n¥ n - 2 ø÷

 
11. Calcula els límits següents:
3 2
a) lim n - 2n + 6
n¥ n-3

n3 + 3n2
b) nlim
¥
n3 - n 2 - 5

n 3 - 2 n2 + 6 +¥
a) nlim 
¥ n-3 +¥

n3 - 2n 2 + 6 3
lim  El grau més gran de n és .
n¥ n-3 2

n3 - 2 n2 + 6
n3 - 2 n2 + 6
3 2
3 lim
n - 2n + 6 n 2 n ¥ n3 1
lim = lim = = = +¥
n ¥ n-3 n ¥ n-3 n-3 0
3
lim 3
n ¥
n 2
n2

565
Límit d’una
Límit d’una funció
funció 10

n3 + 3n2 +¥
b) nlim 
¥ 3
n - n -5 2 +¥

n3 + 3n2
lim  El grau més gran de n és 3.
n¥
n3 - n 2 - 5

n 3 + 3 n2 n3 + 3 n2
lim
n3 + 3 n2 n3 n ¥ n3 1
lim = lim = = = +¥
n ¥
n3 - n2 - 5 n ¥
n - n2 - 5
3
n - n2 - 5
3 0
lim
n3 n ¥ n6

12. Determina aquests límits:


3 4
a) lim 8n +2 3n -1 b) lim 3 n+3
n¥ 3n -1 n¥
- 3n 3 + 5 n 2 + 7 n - 1

3
8n4 + 3n - 1 +¥
a) nlim 
¥ 3n2 - 1 +¥
3
8 n 4 + 3n - 1
lim  El grau més gran de n es 2.
n¥ 3n 2 - 1
3
8 n4 + 3 n - 1
8 n4 + 3 n - 1
3
n2
lim = lim =0
n ¥ 3 n2 - 1 n ¥ 3 n2 - 1
2
n
n+3 +¥
b) lim 3 
n ¥ 3 2
-3 n + 5 n + 7 n - 1 -¥

n+3
lim  El grau més gran de n es 1.
n¥ 3
-3n 3 + 5n 2 + 7 n - 1

n+3
n+3 n 1
lim = lim =3
n ¥ 3 3 2
-3 n + 5n + 7n - 1 n¥ 3 3 2
-3 n + 5n + 7n - 1 -3
n

 
13. Determina els límits següents:
æ n 2 +1÷÷ n2 ö
a) nlim ççç - ÷
¥ çè n n +1÷ø

æ1- 2n2 -3n2 ö


÷÷
b) nlim çç
¥ ççè 3n - 2 - n - 4 ÷÷ø

æ n3 + 2 n3 + 2n +1÷ö
c) nlim çç 2 - ÷÷
¥ çèç 4n - n -1 n2 +1 ø÷

æ n2 + 1 n2 ÷ö æ n2 + 1 n2 ö÷ n2 + n + 1
lim çç - ÷÷  ¥ - ¥ çç - ÷÷ =
n¥ çè n n + 1÷ø çè n n + 1÷ø n2 + n
a)
æ n2 + 1 n2 ö÷ n2 + n + 1
lim ççç - ÷÷ = lim =1
n¥ è n n + 1ø÷ n ¥ n2 + n
 
æ1- 2n2 -3n2 ö÷
lim çç 1 - 2 n2 -3 n2 7 n3 + 2 n2 + n - 4
- ÷÷  -¥ + ¥ - =
n¥ çè 3n - 2 n - 4 ÷ø 3n - 2 n - 4 3 n2 - 14 n + 8
b)

566
Límit d’una funció 1010

æ1- 2n2 -3n2 ö÷ 7n3 + 2n2 + n - 4


lim çç - ÷ = lim = +¥
n¥ çè 3n - 2 n - 4 ÷÷ø n¥ 3n2 - 14n + 8

æ n3 + 2 n3 + 2n + 1÷ö
lim çç 2 - ÷÷  ¥ - ¥   n3 + 2 n 3 + 2 n + 1 - 3 n5 + n 4 - 6 n 3 + 3 n + 3
- =
n¥ èç 4n - n - 1 n2 + 1 ø÷ 2
4n - n - 1 n2 + 1 n4 - n 3 + 3 n 2 - n - 1
c)
æ n3 + 2 n3 + 2n + 1÷ö -3n5 + n4 - 6n3 + 3n + 3
lim çç 2 - ÷÷ = lim = -¥  
n¥ çè 4n - n - 1 n2 + 1 ÷ø n¥ n4 - n3 + 3n2 - n - 1

 
14. Calcula aquests límits:
a) lim ( n - 2 - n ) c) lim ( 3n + 1 - n)
n¥ n¥

b) lim (2n - n 2 -1) d) lim ( n 2 + 4n -1 - n 2 + 1)


n¥ n¥

a) nlim
¥
( n - 2 - n)  ¥ - ¥
-2
lim ( n - 2 - n ) = lim =0
n ¥ n¥ n-2 + n

b) nlim
¥
(2n - n2 - 1)  ¥ - ¥
3 n2 + 1
n ¥
(
lim 2 n - n2 - 1 = lim ) n ¥
2 n + n2 - 1
= +¥

c) nlim
¥
( 3n + 1 - n)  ¥ - ¥
-n 2 + 3 n + 1
lim ( 3 n + 1 - n) = lim = -¥
n ¥ n ¥ 3n + 1 + n

d) nlim
¥
( n2 + 4n - 1 - n2 + 1)  ¥ - ¥
4n - 2
n ¥
lim ( n2 + 4 n - 1 - n2 + 1 = lim ) n ¥
n2 + 4 n - 1 + n2 + 1
=2

 
15. Calcula aquests límits:
n 2n
æ 1 ö5 æ 1 ö÷ 3
a) nlim çç1 + ÷÷÷ c) nlim çç1- ÷
¥ ç
è nø ¥ çè n ø÷
n-1
n-1
æ 3ö æ 1ö 3
b) nlim çç1 + ÷÷ d) nlim çç1- ÷÷÷
¥ çè n ÷ø ¥ çè 2n ø

n 1
n
æ 1 ö5 æ 1 ö5 éæ 1ö ù 5
n 1

a) nlim çç1+ ÷÷÷  1


¥
lim çç1+ ÷÷÷ = lim êêçç1+ ÷÷÷ úú = e 5
¥ ç n¥ çè nø ç
è nø n ¥
êëè n ø úû

3
é nùn ( ⋅ n-1)

êæç ö÷3 ú
æ 3ö
n-1
æ 3ö
n-1 êç 1 ÷÷ ú
ç
b) nlim çç1 + ÷÷÷  1¥ lim çç1 + ÷÷÷ = lim êêçç1 + ÷÷÷ úú = e3
¥ çè nø n ¥ ç
è nø ç n ÷÷ ú
n ¥
êçç ÷÷ ú
êèç 3 ø
ëê ûú

2n 2
2n -
æ 1ö 3 æ 1ö 3 éæ 1ö ù
-n 3 -
2

c) nlim çç1- ÷÷÷  1


¥
lim çç1- ÷÷÷ = lim êêçç1+ ÷÷÷ úú =e 3
¥ ç
è nø n¥ èç nø n¥ è ç -n ø ûú
ëê

567
Límit d’una
Límit d’una funció
funció 10

1 n-1
- ⋅
2n 3
n-1 n-1
æ 1ö 3 æ 1ö 3 éæ 1 ö÷ ùú
-2 n
-
1

d) nlim çç1- ÷÷  1¥ lim çç1- ÷÷÷ = lim êêçç1+ ÷ =e 6


¥ èç 2n ÷ø n¥ èç 2n ø n¥ ç è -2n ø úú
÷
ëê û

16. Determina els límits següents:


n 3n-2
æ 5ö æ 3ö
a) nlim çç1 + ÷÷ c) nlim çç1 + ÷÷
¥ çè n ÷ø ¥ çè 2n ÷ø

n 2n
æ ÷÷ 2 ö æ n ö
÷÷
b) nlim çç1- d) nlim çç1 +
¥ çè n +1÷ø ¥ çè n2 +1÷ø

5
é nù
êçæ ö5 ú
÷
æ 5ö
n
æ 5ö
n êç 1 ÷÷ ú
ç
a) nlim çç1 + ÷÷  1¥ lim çç1 + ÷÷÷ = lim êêçç1 + ÷÷÷ úú = e5
¥ çè n ÷ø ç
n ¥ è nø n ¥ ç
êçç n ÷÷ ú
÷
êèç
êë 5 ÷ø úú
û
2n
-
é -
n +1 ù n +1
êçæ ö 2 ú
÷
æ 2 ö
n
æ 2 ÷ö
n êç 1 ÷÷÷ ú
ç
b) nlim çç1- ÷÷  1¥ lim çç1- ÷ = lim êêçç1 + ÷
÷ ú
ú = e-2
¥ çè n + 1÷ø n ¥ ç
è ÷
n + 1ø n ¥
êç ç n + 1 ÷
÷÷ ú
êçèç -
êë 2 ø÷ ú
úû

3(3 n- 2)
é 2n ù 2n
êæç ö÷ 3 ú
æ 3ö
3 n- 2
æ 3ö
3 n- 2 êç 1 ÷÷ úú
÷ 9
ç
c) nlim ççç1+ ÷÷÷  1¥ lim çç1 + ÷÷÷ = lim êêçç1 + ÷ = e2
¥ è 2n ø ç
n ¥ è 2n ø n ¥ ç
êçç 2n ÷÷÷ úú
÷
êçè
êë 3 ø÷ úú
û
2 n2
é n2 +1 ù n2 +1
êæ ö÷ n ú
ê ç ÷ ú
êççç
2n
1 ÷÷÷ ú
2n
æ n ö æ n ö÷
d) nlim çç1 + 2 ÷÷  1¥ lim çç1 + 2 ÷ = lim ê ç 1 + ÷ ú = e2
¥ ç
è n + 1÷ø n ¥ ç
è n + 1÷ø n ¥ êç
ç n2 + 1÷÷÷ ú
êçç ú
êè n ø÷ ú
êë ûú

17. Calcula aquests límits:

3 x 2 + 2 x -1 3x ⋅( x 2 -1)
a) lim b) xlim
x+¥ x - 2 - 6x2 +¥ 2 - x2

3x2 + 2x -1 1 3 x ⋅ ( x 2 - 1)
a) lim =- b) x lim = -¥
x +¥ x - 2 - 6 x 2 2 +¥ 2 - x2

 
18. Troba els límits següents:
3 x2
æ 3 x -1 ö
a) lim
x+¥
( x -1 - x ) b) lim ççç ÷÷
x+¥ è 2 + 3 x ø÷

çæ 3 ö
÷÷ 3 x 2
é æç 3 x + 2 ÷ö ù èçç- 3 x + 2 ÷ø÷⋅
- ÷
êæç ö÷ççè 3 ÷ø ú
æ 3 x - 1ö
3 x2 êç ÷
÷ ú
ç 1
a) lim ( x - 1 - x ) = -¥ b) x +¥
lim çç ÷÷ = lim êêçç1 + ÷÷÷ ú
ú =0
x  +¥ èç 2 + 3 x ø÷ x +¥ ç
êçç - 3 x + 2 ÷÷÷ ú
êçè 3 ÷ø ú
ë û

568
Límit d’una funció 1010

19. Determina els límits laterals de les funcions següents en els punts indicats:
-2 x + 1
a) a x=1 b) x a x=0
x -1 x

-2 x + 1 -2 x + 1
a) lim = -¥ lim = +¥ b) lim x = 1 lim
x
= -1
x 1+ x -1 x 1- x -1 x 0+
x x  0- x

20. Indica quins són els límits laterals en x = -2 de la funció següent:


�� � ��������� � ��
���� � �
� � �� ��������� � ��

lim f ( x ) = lim- 3 x + 4 = -2          lim f ( x ) = lim+ 1- x 2 = -3  


x -2- x -2 x -2+ x -2

 
21. Calcula els límits de les funcions en els punts indicats.
���� ���
a) ���� � �� ����� a x = 1 b) ���� � ��� a x = -1

3x -1
a) lim f ( x ) = lim =1
x 1 x 1 x2 - 2x + 3

ì
ï lim 2 - x = -¥
ï
2 - x ïïï x- 1- x + 1
b) xlim f ( x ) = lim =í  No existeix lim f ( x ).
- 1 x - 1 x + 1 ï
ï 2- x x - 1
ï
ï lim = +¥
ï x - 1+ x + 1
î

 
��� ���
22. Justifica si existeix o no el límit en x = 2, en x = 3 i en x = 4 de la funció ���� � � .
��� ���


lim� ���� � ��
lim ���� → ��→� → No existeix lim ����.
�→� � lim� ���� � �� �→�
�→�

��
lim ���� →
�→� �

53
lim ���� →
�→� 14

23. Calcula el límit de f (x) en x = 0 i en x = 3.


� � ��������� � �
���� � � � �

��������� � �
���
lim f ( x ) = lim- 3 - x = 3 üïï
x0- x0 ïï
x-2 ý  No existeix lim f ( x ).  
lim+ f ( x ) = lim+ = -2ïïï x0
x0 x0 x + 1 ïþ
x -2 1
lim f ( x ) = lim+ =  
x 3 x 3 x +1 4

24. Determina el valor dels límits que tens a continuació.

569
Límit d’una
Límit d’una funció
funció 10

࢞૜ ି૛࢞૛ ା࢞ ࢞૝ ି૚૟
a) ‫ܕܑܔ‬ b) ‫ܕܑܔ‬
࢞՜૚ ࢞૛ ି૜࢞ା૛ ࢞՜ି૛ ࢞૜ ାૡ

 
 
25. Posa un exemple d’una funció que tingui com a asímptotes horitzontals aquestes rectes:
a) y = 1 b) y = 2 c) y = 3
Resposta oberta. Per exemple:
2x -1 2x2 3x2 - 5
a) y = 1  f ( x ) = b) y = 2  f ( x ) = 2 c) y = 3  f ( x ) =
2x + 8 ( x + 1) 2
x + 7x - 3

 
26. Esbrina si una funció pot tenir dues asímptotes horitzontals. I tres?
▪ Sí, n’hi ha prou que els límits quan x → +∞ i quan x → -∞ sigui diferents de +∞ o -∞ i diferents entre si.
▪ No.
 
27. Calcula les asímptotes horitzontals de les funcions que hi ha a continuació.
3 x 2 -1 2 x -1
a) f ( x) = c) f ( x) =
2 x 2 -1 x

1- 2 x 3 x2 +1
b) f ( x) = d) f ( x) =
4 x3 - 2 x 2
x - 2x - 3

3 x 2 -1 3 3
a) xlim f ( x) = lim 2
=  f ( x) té una asímptota horitzontal quan y = .  
¥ x¥ 2 x -1 2 2
1- 2 x 3 1 1
b)  xlim f ( x ) = lim 3
= -  f ( x ) té una asímptota horitzontal quan y = - .  
¥ x¥ 4x - 2x 2 2
2 x -1
c)  xlim f ( x ) = lim = 2  f ( x ) té una asímptota horitzontal quan y = 2.  
¥ x¥ x
x2 +1
d)  xlim f ( x ) = lim 2
= 1  f ( x ) té una asímptota horitzontal quan y = 1.  
¥ x¥ x - 2x - 3
   

570
Límit d’una funció 1010

28. Troba les asímptotes verticals de les funcions següents:


� ���� �� ��
a) ���� � b) ���� � c) ���� �
�� �� ��� �� ��

a) Dom f = 
f(x) no té asímptotes verticals.
b) Dom f =  - {-2}

f(x) té una asímptota vertical en x = -2.

lim f ( x) = +¥üï
ï
x-2- ï
ý  f ( x ) té una asímptota vertical a x = -2.
lim+ f ( x) = -¥ï
ï
x-2 ï
þ
c) Dom f =  - {- 1, 1}

lim f ( x) = -¥üï
ï
x-1- ï
ý  f ( x) té una asímptota vertical a x = -1.
lim+ f ( x) = +¥ï
ï
x-1 ï
þ

lim f ( x ) = +¥üïï
x1- ïý  f ( x ) té una asímptota vertical a x = 1.
lim+ f ( x ) = -¥ïï
x1 ïþ
 
29. Calcula l’asímptota obliqua d’aquestes funcions, si en tenen:
�� �� ��� ���
a) ���� � b) ���� � c) ���� �
��� ���� �� ��

f (x) 2
x 2 2
x -x +x
a)  x lim = lim 2 = 1  lim f ( x ) - x = lim =1
+¥ x x +¥ x - x x +¥ x +¥ x -1
 
f(x) té l’asímptota obliqua y  x  1. 
f (x) 2x
b)  x lim = lim
x +¥ x - x 3
=0 
+¥ x

f(x) té una asímptota horitzontal. Per tant, no té asímptota obliqua. 
f (x) 3x3 + 2x 3x3 + 2x - 3x3 + 9x
c)  x lim = lim 3
= 3  lim f ( x ) - 3 x = lim =0
+¥ x x +¥ x - 9 x x +¥ x +¥ x2 - 9
 
f(x) té la asímptota obliqua y  3x. 
 
30. Estudia la continuïtat de les funcions que tens a continuació:
a) ���� � � �� b) ���� � √� � � c) ���� � ���� � �� �
a) Dom f = � - {0} → f(x) és contínua a � - {0}.
b) Dom f = [4, +∞) → f(x) és contínua a [4, +∞).
c) Dom f = (-1, 1) → f(x) és contínua a (-1, 1).

571
Límit d’una
Límit d’una funció
funció 10

31. Estudia la continuïtat de les funcions següents:


૛‫ିܠ‬૚
a) ࢌሺ࢞ሻ ൌ b) ࢌሺ࢞ሻ ൌ ‫࢞܍‬ା૚ c) ࢌሺ࢞ሻ ൌ ‫࢞ܖܑܛ‬૛ െ ૚
࢞ା૜

a) Dom f =  - {-3}  f ( x) és contínua en  - {-3}.

b) Dom f =   f ( x) és contínua en .

c) Dom f =   f ( x) és contínua en .

32. Estudia la continuïtat de les funcions següents:



࢙࢏࢞ ൑ ૚
‫ܠ‬ା૚ ࢞
a) ࢌሺ࢞ሻ ൌ b) ࢌሺ࢞ሻ ൌ ቐ࢙࢞࢏૚ ൏ ࢞ ൏ ૝
࢞ି૛
૞࢙࢏࢞ ൒ ૝
a) Dom f = Թ - {2}

lim f ( x) = -¥ üïï
x2- ïý  No existeix lim f ( x) i f ( x) no és contínua quan x = 2.
lim+ f ( x) = +¥ïï x2
x2 ïþ
La discontinuïtat és inevitable de salt infinit. La funció té una asímptota vertical a x = 2.

b) Dom f = Թ - {0}

lim f ( x ) = -¥ ü
ï
ï
x0- ï
ý  No existeix lim f ( x ) i f ( x ) no és contínua quan x = 0.
lim+ f ( x ) = +¥ï
ï x0
x0 ï
þ
La discontinuïtat és inevitable de salt infinit. La funció té una asímptota vertical a x = 0.

lim f ( x ) = 1üïï
x1- ïý  $ lim f ( x ) =
1
lim+ f ( x ) = 1ïï x1
x1 ïþ

Com que $ f (1)=xlim


1
f ( x) , la funció és contínua quan x = 1.

lim f ( x ) = 4üïï
x4- ïý  No existeix lim f ( x ) i f ( x ) no és contínua quan x = 4.
lim f ( x ) = 5ïï x4
x4+ ïþ

La discontinuïtat és inevitable de salt finit.

33. Estudia la continuïtat d’aquestes funcions i indica el tipus de discontinuïtat que presenten:
࢞ ࢞૛
࢙࢏࢞ ൏ ૚ ૛ ࢙࢏࢞ ൑ ૚
a) ࢌሺ࢞ሻ ൌ ቊ ࢞ି૚ b) ࢌሺ࢞ሻ ൌ ቐ ࢞ ૚ି૜࢞ା૛
૛࢞ െ ૜࢙࢏࢞ ൒ ૚ ࢙࢏࢞ ൐ ૚
૛࢞ି૚

572
Límit d’una funció 1010

a) Dom f = 

lim f ( x) = -¥üïï
x1- ïý  No existeix lim f ( x) y f ( x) no és contínua quan x = 1.
lim+ f ( x) = -1 ïï x1
x1 ïþ
La discontinuïtat és de salt infinit. La funció té una asímptota vertical en x = 1.

b) Dom f = 

lim f ( x) = +¥üïï
x1- ïý  No existeix lim f ( x) y f ( x) no és contínua quan x = 1.
lim f ( x) = 1 ïï x1
x1+ ïþ
La discontinuïtat és de salt infinit. La funció té una asímptota vertical en x = 1.

   

573
Límit d’una
Límit d’una funció
funció 10

SABER FER 
 
34. Calcula els límits següents:
3
2n3 + 3n -1
a) lim
n¥
1 + 2n 2

2
b) lim n -3 6n + 1
n¥ 7n - 7

3 2
c) lim 2n + n + 7
n¥ 8n - 1

3
2 n3 + 3 n - 1 +¥
3
2n3 + 3n -1
a) nlim  lim  El grau més gran de n és 1.
¥
1 + 2 n2 +¥ n¥
1 + 2n 2

3
2 n3 + 3 n - 1 2 n3 + 3 n - 1
3 3
3 lim 3
2n + 3 n - 1 n n ¥ n3 2 1
lim = lim = = =6
n ¥
1 + 2 n2 n ¥
1 + 2 n2 1 + 2n 2
2 2
lim
n n ¥ n2

n2 - 6n + 1 +¥ n 2 - 6n + 1
b) nlim  lim  El grau més gran de n és 3.
¥ 7n3 - 7 +¥ n¥ 7 n3 - 7

n2 - 6 n + 1 n2 - 6 n + 1
2 lim
n - 6n + 1 n3 n ¥ n6 0
lim = lim = = =0
n ¥ 7 n3 - 7 n ¥ 7 n3 - 7 3
7n - 7 7
lim
n3 n ¥ n3

3
2n2 + n + 7 +¥
3
2n 2 + n + 7
c) nlim  lim  El grau més gran de n és 1.
¥ 8n - 1 +¥ n¥ 8n - 1

3
2 n2 + n + 7 2 n2 + n + 7
3 lim
2 n2 + n + 7
3
n n ¥ n3 0
lim = lim = = =0
n ¥ 8n - 1 n ¥ 8n - 1 8n - 1 8
lim
n n ¥ n

 
35. Calcula aquests límits:
n 2 -3
æ ö
3n
æ 2n -1 ö÷ æ 1- n 2 ö÷
3n

a) nlim çç1 + 2n ÷÷ b) nlim çç ÷ c) nlim çç ÷÷


¥ çè n 2 -1÷ø ¥ çè 5 + 2n ÷ø ¥ çèç -n + 2 ø÷
2

2n
⋅3n
é n2 -1 ù n2 -1
êæ ö÷ 2 n ú
ê ç ÷ ú
êçç
3n
1 ÷÷÷ ú
3n
æ 2n ö æ 2n ÷ö
a) nlim çç1 + 2 ÷÷  1¥ lim çç1 + 2 ÷ = lim êçç1 + 2 ÷ ú = e6
¥ ç
è n - 1÷ø n ¥ ç
è n - 1÷ø n ¥ ê ç n - 1÷÷÷ ú
êççç ú
êè 2n ÷ø ú
êë úû

574
Límit d’una funció 1010

6
é 5+2 n ù -
5+2 n
(
⋅ n2 - 3 )
-
êæ ÷ö 6 úú
n2 - 3 n -32
êçç ÷
b) nlim çç
æ 2n - 1 ö÷  1¥
æ 2 n - 1 ö÷
lim çç
ç
= lim êêçç1 +
1 ÷÷÷ ú = e-¥ = 0
¥ èç 5 + 2n ø÷÷ ç 5 + 2 n ø÷÷
n ¥ è n ¥ ç
êçç 5 + 2 n ÷÷
÷
ú
ú
- ÷
êçè
êë 6 ÷ø ú
úû

1
⋅3 n
æ 1- n2 ö÷
3n
æ 1- n2 ÷ö
3n éæ 1 ÷ö
n 2 - 2 ù n2 - 2

c) nlim çç ÷÷  1¥ lim ççç 2 ÷÷ = lim êêçç1 + 2 ÷ ú


ú = e0 = 1
¥ çè -n + 2 ÷ø
2 n ¥ è -n + 2 ÷ n ¥ ç n - 2 ÷ø
ø êëè úû

36. Calcula el valor dels límits següents:

x 2 -16
lim
a) x4 2
x + 16

5x - 9
b) lim
x¢3 2x2 + 6 x

x 2 - 16
a) lim =0
x 4 x 2 + 16

5x - 9 ü
ï
lim = -¥ï
ï
2
2x + 6x
x-3- ï
ï 5x - 9
ý  lim no existeix.
b) 5x - 9 ï x-3 2 x 2 + 6 x
lim ï
= +¥ï
2
x-3+ 2 x + 6 x ï
ï
þ

37. Determina els límits següents:


2 2
a) lim x + 1 - 1 b) lim x + 5 - 3
x 0 x x 2 x-2

2
x2 + 1-1 x
a) lim x + 1 - 1  0 lim = lim =0
x 0 x 0 x 0 x x  0 2
x +1+1

2
x2 + 5 - 3 x +2 2
b) lim x + 5 - 3  0 lim = lim =
x2 x -2 0 x 2 x -2 x 2
x2 + 5 + 3 3

38. Representa la gràfica d’una funció que talla els eixos en l’origen, sempre és creixent, té asímptotes
verticals en x = –1 i x = 3 i una d’horitzontal en y = 0.
Resposta oberta. Per exemple:

x
f ( x) = -  
x2 - 2x - 3

 
 
   

575
Límit d’una
Límit d’una funció
funció 10

39. Calcula les branques infinites de les funcions següents:


3 x 4 - 2 x 2 + 5 x -1 2 x 4 + 3x
a) f ( x) = c) f ( x) =
1- x 1- x 2

5x + x2 - 3 5 - 2 x - x3
b) f ( x) = d) f ( x) =
4 + 3x 1- x

3x4 - 2x2 + 5x -1 3x4 - 2x2 + 5x -1


a) lim = -¥ lim = +¥
x +¥ 1- x x -¥ 1- x

5x + x2 - 3 5x + x2 - 3
b) lim = +¥ lim = -¥
x +¥ 4 + 3x x -¥ 4 + 3x

2x4 + 3x 2x4 + 3x
c) lim
x +¥ 1- x 2
= -¥ lim
x -¥ 1- x 2
= -¥

5 - 2x - x3 5 - 2x - x3
d) lim = +¥ lim = +¥
x +¥ 1- x x -¥ 1- x

40. Troba les asímptotes horitzontals i obliqües d’aquestes funcions:


2 x 4 + 3x x2 - 3 2 x - x3 3x -1
a) f ( x) = b) f ( x) = c) f ( x) = d) f ( x) =
1- x 2 -3 x 2 x3 - x x2 - 5x

2 x 4 + 3x
a) xlim f ( x ) = lim = -¥  f ( x ) no té asímptotes horitzontals ni obliqües.
¥ x¥ 1- x 2

f ( x) x2 - 3 1
b) xlim = lim 2
=-
¥ x x ¥ -3 x 3

x2 - 3 1 1
lim f ( x ) = lim + x = 0  f ( x ) té una asímptota obliqua a y = - x.
x¥ x¥ - 3 x 3 3

f ( x) 2x - x3
c) xlim = lim 3
x ¥ x - x
= -1
¥ x

2 x - x3
lim f ( x ) = lim + x = 0  f ( x ) té una asímptota obliqua a y = -x.
x¥ x¥ x 2 -1

3 x -1
d) xlim f ( x ) = lim = 0  f ( x ) té una asímptota horitzontal a y = 0.
¥ x¥ x2 - 5x

41. Estudia la continuïtat d’aquestes funcions i determina els tipus de discontinuïtat que presenten:
�� � ������������������������������� � � √������������������������������������� � �

a) ���� � � ���������������������������� � � � � b)����� � ������������������������������� � � � �
��� �
�� � �������������������������������� � � ����������������������������������� � �
���

576
Límit d’una funció 1010

a) Dom f = 

lim f ( x) = 1 üï
x2- ïï
ý  No hi ha lim f ( x ) i f ( x ) no és contínua a x = 2.
lim+ f ( x ) = +¥ïï x2
x2 ïþ

La discontinuïtat és de salt infinit. La funció té una asímptota vertical a x = 2.

lim f ( x) = 1üïï
x3- ïý  f ( x ) és contínua a x = 3.
lim f ( x ) = 1ïï
x3+ ïþ

b) Dom f = [0, + ¥) - {4}

lim f ( x )  No existeix.ü
ï
ï
x0- ï
ý  f ( x) no és contínua a x = 0.
lim+ f ( x ) = 0 ï
ï
x 0 ï
þ
lim f ( x) = 6 ïüï
x2- ïï
1 ýï  No existeix lim f ( x) y f ( x) no és contínua a x = 2.
lim+ f ( x) = - ïï x2
x2 2 ïþ
La discontinuïtat és de salt finit.
lim f ( x) = -¥üï
ï
x4- ï
ý  No existeix lim f ( x) y f ( x) no és contínua en x = 4.
lim+ f ( x) = +¥ï
ï x4
x4 ï
þ
La discontinuïtat és de salt infinit. La funció té una asímptota vertical en x = 4.

42. Calcula el valor de k perquè aquestes funcions siguin contínues:


 
૜ି࢑
૛࢞ െ ૜࢙࢏࢞ ൏ ૚ ࢙࢏࢞ ൏ ૙
a) ࢌሺ࢞ሻ ൌ ቄ ૛ b) ࢌሺ࢞ሻ ൌ ቊ࢞ି૛
࢑࢞ ൅ ૛࢞ െ ૚࢙࢏࢞ ൒ ૚ ૛
࢞ െ ૚࢙࢏࢞ ൒ ૙
a) f ( x ) és contínua a x = 1 si es verifica que f (1) = lim
x1
f ( x) :

lim f ( x ) = -1 ü
ï
x 1- ï
ï
ï
lim+ f ( x ) = -k + 1ï
ý  -k + 1 = -1  k = 2  
x 1 ï
ï
f (1) = -k + 1 ï
ï
ï
þ

b) f ( x ) és contínua a x = 0 si es verifica que f (0) = lim


x0
f ( x) :

3 - k ïüï
lim f ( x ) = ï
x  0- -2 ïï
ï 3-k
lim f ( x ) = -1 ïý  = -1  k = 1  
x  0+ ïï -2
f (0 ) = -1 ï
ïï
ïïþ

577
Límit d’una
Límit d’una funció
funció 10

ACTIVITATS 
43. Calcula el terme general de les successions que tenen els primers termes següents:
� � � � � �
a) �, , , , … c) � , �, , , …
� � � � � �
� � � � � � �
b) � , , , ,… d) �, , , ,…
� � � �� � �� ��
2
n -1 ( n - 1) - 1
a) an = c) an =
n +1 n +1
n
(-1) n +1
b) an = d) an =
2 n
n3

44. Per mitjà de la calculadora, troba el límit d’aquesta successió definida de manera recurrent:
����� � �
�� � � �� �
����� � �

�� ���
�� � � �� � � �,����� … �� � � �,������ …
�� ����
� ���
�� � � �,����� … �� � � �,����� …
� ���

����
45. Copia i completa aquesta taula, en què �� �
����

Quin és ��� �� ?
���

n  1  10 100 1 000 10 000


an  0,1818  0,4462  0,4942  0,4994  0,4999 
1
lim an =  
n ¥ 2

 
����
46. Copia i completa aquesta taula, en què �� �
�� ��

 
Quin és ��� �� ? 
���

n  1  10 100 1 000 10 000


an  1  0,4857  0,0501  0,0050  0,0005 
lim an = 0  
n¥

 
 

578
Límit d’una funció 1010

47. Determina els límits següents:

3n6 +1 3n5 + n3 - n 4
a) nlim d) nlim
¥ 5n4 + 3n3 + 2n -1 ¥ 1- 4n - n5

1- n3 + 2n - 5n2 n (4n 2 -1)


b) nlim e) nlim
¥ 2n3 - 3n2 + 6n -1 ¥ n + n 2 - 2n3

4n2 - 5n4 + 2n -1 (n4 + 4n3 + 2) n2


c) nlim f) nlim  
¥ 3n - 2n3 + 5 - n4 ¥ n5 + 3n 4 + 4

3 n6 + 1 3 n5 + n3 - n4
a) nlim = +¥ d) nlim = -3
¥ 5 n + 3 n3 + 2 n - 1
4 ¥ 1- 4 n - n5

1- n3 + 2 n - 5 n2 1 n(4 n2 - 1)
b) nlim =- e) nlim = -2
¥ 2 n3 - 3 n2 + 6 n - 1 2 ¥ n + n2 - 2n3

4 n2 - 5 n4 + 2 n - 1 (n4 + 4n3 + 2) n2
c) nlim =5 f) nlim = +¥
¥ 3 n - 2 n3 + 5 - n4 ¥ n5 + 3n4 + 4

48. Calcula els resultats dels límits següents:

4n 2 + n - 3 2n + 3n 2 +1
a) nlim d) nlim
¥ 3n +1 ¥
3(n4 + n +1)

8n 2 + 3n + 2 2n + 1 + n 2
b) nlim 3 4 e) nlim
¥
n + 2n ¥ 3n + 5

5 - 2n + 6 n 3 n3 - 3n + 2
c) nlim f) nlim
¥ n2 - n - 6 ¥ n2 + 5

4 n2 + n - 3 +¥ 4 n2 + n - 3 2
a) nlim  lim =
¥ 3n + 1 +¥ n¥ 3n + 1 3

8n2 + 3n + 2 +¥ 8n2 + 3n + 2
b) nlim  lim =4 2
¥
n3 + 2n4 +¥ n¥
n3 + 2n4

5 - 2n + 6 n3 +¥ 5 - 2 n + 6 n3
c) nlim  lim =0
¥ n2 - n - 6 +¥ n ¥ n2 - n - 6

2n + 3n2 + 1 +¥ 2n + 3n2 + 1
d) nlim  lim = 3
¥
3 ( n4 + n + 1) +¥ n¥
3 ( n4 + n + 1)

2n + 1+ n2 +¥ 2n + 1+ n2
e) nlim  lim =1
¥ 3n + 5 +¥ n¥ 3n + 5

n3 - 3 n + 2 +¥ n3 - 3n + 2
f) lim 2
 lim =0
n¥ n +5 +¥ n¥ n2 + 5

 
 
 

579
Límit d’una
Límit d’una funció
funció 10

49. Troba els límits següents d’aquestes successions:


æ n2 + n 5n2 ö
÷÷ æ 3n3 + 4n 4n2 ö÷
a) nlim çç - d) nlim çç - ÷÷
¥ ç
÷ ¥ ç
èç n n +1÷ø çè 2n 7n3 ø÷

æ n2 - n 3 - n2 ö÷ æ 6n 2 + 4n 4n3 ö÷
b) nlim çç ÷÷ e) nlim çç ÷÷
¥ ççè n +1 + n ÷ø ¥ ççè 3n
-
2n2 ÷ø

æ 3n2 -2n3 ö æ 2 3 ö
c) nlim ççç + ÷÷ f) nlim çç -7 n + 4 + 5n + n ÷÷
¥ èç n - 5 n + 3 ÷ø÷ ¥ èçç 2 - n3 ÷
3 - n 4 ø÷

æ n2 + n 5n2 ÷ö æ n2 + n 5n2 ö÷ -4n2 + 2n + 1


a) nlim çç - ÷÷  ¥ - ¥ lim ççç - ÷÷ = lim = -¥
¥ ç
è n n + 1÷ø n¥ è n n + 1ø÷ n¥ n +1

æ n2 - n 3 - n2 ö÷ æ n2 - n 3 - n2 ö÷ -2n2 + 3n + 3
b) nlim çç lim çç
+ ÷÷ ¥ - ¥ + ÷÷÷= nlim = -2
¥ çè n +1 n ÷ø n¥ çè n + 1 n ø ¥ n2 + n

æ 3n2 -2n3 ö æ 3n2 -2n3 ö÷ -2n4 + 13 n3 + 9n2


c) nlim çç + ÷÷ ¥ - ¥ lim çç + ÷÷ = lim = -¥
¥ èç n - 5 n + 3 ÷ø÷ n¥ èç n - 5 n + 3 ø÷ n¥ n2 - 2n - 15

æ 3n3 + 4n 4n2 ö÷ 3n3 + 4n 4n2


d) nlim çç - 3÷
÷ = lim - lim 3 = +¥ - 0 = +¥
¥ ç
è 2n 7n ø÷ n ¥ 2n n ¥ 7n

æ 6n2 + 4n 4 n3 ö÷ æ 6n2 + 4n 4n3 ÷ö 4 4


e) nlim çç lim çç
- ÷÷  ¥ - ¥ - 2 ÷÷ = lim =
¥ çè 3n 2n2 ÷ø n¥ çè 3n 2n ø÷ n ¥ 3 3

æ -7n2 + 4 5n3 + n ÷ö -7n2 + 4 5n3 + n


f) lim ççç + ÷÷ = lim + lim = 0+0 = 0
n¥ è 2 - n 3
3 - n ø÷ n¥ 2 - n
4 3 n¥ 3 - n4

50. Determina el valor d’aquests límits:


a) ��� �� � √�� � �� � ��
���

b) ��� �√��� � � � �� � ���


���

c) ��� ��� � √���� � ��


���

a) lim �� � √�� � �� � �� � � � �
���

��� � �
lim �� � ��� � �� � �� � lim � ��
��� ��� � � √�� � �� � �

b) lim �√��� � � � �� � ��� � � � �


���

���� � � � ��
��� ����� � � � �� � ��� � ��� � ��
��� ��� √��� � � � �� � ��

c) lim ��� � √���� � �� � � � �


���

��
��� ��� � ����� � �� � ��� � �
��� ��� �� � √���� � �

580
Límit d’una funció 1010

51. Calcula els límits següents:


n-1 n
æ 1 ö æ n 2 -1 ö
a) nlim ççç1 + ÷÷ c) lim ççç 2 ÷÷÷
¥ è 2n + 3÷ø n¥ ç è n + 1ø÷

n 2 +1 n
æ 2n - 3 ö÷ æ n ö
b) lim ççç ÷ d) nlim ççç2 - ÷÷
n¥ è 2n + 1 ÷ ø ¥ è n +1÷ø
n-1 n-1
æ 1 ÷÷ ö æ 1 ö÷ 1
a)  nlim çç1 +  1¥         lim çç1 + ÷ = e2  
¥ çè 2n + 3 ÷ø n¥ çè 2n + 3 ÷ø

n2 + 1 n2 + 1
æ 2n - 3 ö
÷÷ æ 2n - 3 ö÷
b)  lim ççç ÷  1¥           lim çç ÷ =0  
n ¥ è 2 n + 1ø è 2n + 1ø÷
n ¥ ç

n n
æ n2 - 1ö æ n2 - 1ö÷
c)  nlim çç ÷÷  1¥           lim çç 2 ÷ = 1 
¥ èç n2 + 1÷÷ø n¥ èç n + 1÷ø÷

n n
æ n ö
÷÷  1¥    æ n ÷ö
d)  nlim çç2 -       lim çç2 - ÷ =e  
¥ èç n + 1÷ø n¥ çè n + 1÷ø

 
52. Troba els límits següents:

a) ��� �� � ��� � � c) ���
��� ��� �� �����


b) ��� d) ��� ���
��� ��|�| ���


a) lim � � � �� � � � � �� c) lim ��
��� ��� � � �����


b) lim �� d) lim �� � � ��
��� ��|�| ���

53. Calcula els límits de polinomis següents:

a) lim ( x 2 - 3 x + 1)
x+¥

lim (3 x - x 2 - 7 x 4 - 3)
b) x+¥

c) lim (2 x - 3x5 + x2 -1)


x-¥

d) xlim
-¥
(4 x2 - x + 3x4 -1)

e) lim (4 x6 - 3x3 + 2 x2 - 6 x -1)


x-¥

a)  lim ( x 2 - 3 x + 1) = +¥  
x  +¥

b)  lim (3 x - x 2 - 7 x 4 - 3 ) = -¥  
x  +¥

c)  x lim
-¥
(2 x - 3 x 5 + x 2 - 1) = +¥  

581
Límit d’una
Límit d’una funció
funció 10

d)  x lim
-¥
(4 x 2 - x + 3 x 4 - 1) = +¥  

e)  x lim
-¥
(4 x 6 - 3 x 3 + 2 x 2 - 6 x - 1) = +¥  

 
54. Troba el valor d’aquests límits amb valors absoluts:
a) lim - x 2 + 5 x - 1
x+¥

b) xlim
-¥
3x 3 - 2 x 2 + x - 7

c) xlim
-¥
1- x 2 - 5 x 6

a) lim - x 2 + 5 x - 1 = +¥
x  +¥

b) lim 3 x 3 - 2 x 2 + x - 7 = +¥
x -¥

c) lim 1- x 2 - 5 x 6 = +¥
x -¥

 
55. Calcula aquests límits i comprova’n el resultat amb la calculadora:
��� ����� ��� �����
a) ��� c) ���
��� �� ����� ��� ��� ���� ��
��� ���� ���� ��� �����
b) ��� d) ���
��� ��� ����� ��� �� ��� ��

2�2 �����
a) lim �2
��� �2 �����

�� � ��� � ����
b) lim � ��
��� �� � �����

�� � ����� �
c) lim ��
��� �� � ��� � �� �

�� � �����
d) lim ��
��� � � �� � ��

 
56. Calcula els límits següents:

x2 - 5x + 3 (7 + 7 x + 7 x 3 ) x 2
a) xlim c) x+¥
lim
+¥ 2 - x3 + x 2 x 2 ( x 2 - 4 x3 ) - 2 x 5

x ( x3 - 4 + x) 1 + x - 6 x 4 + x3
b) x+¥
lim 4 3
d) x+¥
lim
-7 + 4 x + x 3x + 2 x 2 - 3

x2 - 5x + 3 (7 + 7 x + 7 x 3 ) x 2 -7
a) lim =0 c) x lim =
x +¥ 2 - x3 + x2 +¥ x2 ( x2 - 4x3 )- 2x5 6

x (x3 - 4 + x) 1 1+ x - 6 x 4 + x 3
b) lim 4 3
= d) x lim = -¥
x +¥ -7 + 4 x + x 4 +¥ 3x + 2x2 - 3

582
Límit d’una funció 1010

 
57. Calcula aquests límits amb l’ajut de la calculadora i comprova el resultat que obtens:
x2 + 5x + 7 x3 - 2 x 2 -10 x
a) xlim c) xlim
-¥ 2 x 2 + x +1 -¥ -x 2 + 2 x 3 - x + 3

4 + x - 2 x3 -x 2 + 3x + 21
b) xlim d) xlim
-¥ 2 x 2 - 3x +11 -¥ 5 x 2 - 4 x3 + 2 x

x2 + 5x + 7 1 x 3 - 2 x 2 - 10 x 1
a) lim = c) x lim =
x -¥ 2x2 + x + 1 2 -¥ -x 2 + 2 x 3 - x + 3 2

4 + x - 2x3 - x 2 + 3 x + 21
b) lim = +¥ d) x lim =0
x -¥ 2 x 2 - 3 x + 11 -¥ 5x2 - 4x3 + 2x

 
58. Determina el valor dels límits següents:

2x2 + 3 6x2 - 2
a) lim lim
c) x+¥
x+¥ 5 x -1 3x 2 + 2

4 x2 - 2 x -1 x2 - 2
lim
b) x+¥ d) x+¥
lim
3x 4 - 3x3 +1 x5 - 7 x + 1

2x2 + 3 2 6x2 - 2
a) lim = c) x lim = +¥
x +¥ 5x -1 5 +¥
3x2 + 2

4x2 - 2x -1 4 x2 - 2
b) lim = d) x lim =0
x +¥
3x4 - 3x3 +1 3 +¥
x5 - 7x + 1

 
59. Calcula el resultat d’aquests límits:
�������� ���� ���� ����
a) ��� b) ���
��� �� �� ��� ����

√������ � √�� � ��� � ��� �


a) lim �� b) lim � ��
��� � � �� ��� ����

 
60. Calcula el valor dels límits següents:
æ 2 x2 x3 ö÷
a) lim ççç - 2 ÷÷
x+¥ ç è 2 x + 1 x + 1÷ø

æ 5 x 2 + 1 3 - x 2 ö÷
çç ÷÷
b) xlim
-¥ ç
+
çè x x + 2 ÷ø

æ 3
2 x 2 +1÷ö
çç x ÷÷
c) xlim ç -
-¥ çè x - 2 x 2 x - 4 ø÷
2

æ 2x2 x3 ö
÷÷  ¥ - ¥     æ 2x2 x 3 ö÷ 1
lim ççç
a)  x +¥ - 2 ÷   lim ççç - 2 ÷÷ = -  
è 2 x + 1 x + 1ø÷ x +¥ è 2 x + 1 x + 1ø÷ 2

583
Límit d’una
Límit d’una funció
funció 10

æ5x 2 + 1 3 - x 2 ö÷ æ 5 x 2 + 1 3 - x 2 ö÷
b)  x lim çç + ÷÷  ¥ - ¥      lim çç + ÷÷ = -¥  
-¥ çè x x + 2 ÷ø x -¥çè x x + 2 ÷ø

æ x3 2 x 2 + 1ö æ x3 2 x 2 + 1ö÷
c)  x lim çç - ÷÷  ¥ - ¥     lim çç 2 - ÷= 0  
ç x 2 - 2 x 2 x - 4 ÷ø÷
-¥ è x -¥ èç x - 2 x 2 x - 4 ÷ø÷

 
61. Determina el valor d’aquests límits:

a) lim ( x - x 2 + 4 x ) c) lim ( x2 + 4 - x2 - 3 )
x+¥ x+¥

b) lim
x+¥
( x2 + 6 - x ) d) lim
x+¥
( x2 - 2 x +1 - x 2 - 2 x + 4 )
a)  x lim
+¥
(x - x2 + 4x )  ¥ - ¥
 
-4 x
x  +¥
(
lim x - x 2 + 4 x = lim ) x  +¥
x+ x2 + 4x
= -2  

b)  x lim
+¥
( x 2 + 6 - x)  ¥ - ¥
 
6
lim
x +¥
( x 2 + 6 - x = lim ) x +¥ 2
x +6 + x
=0 

c)  x lim
+¥
( x2 + 4 - x2 - 3)  ¥ - ¥
 
7
lim
x +¥
( x 2 + 4 - x 2 - 3 = lim ) x +¥
x2 + 4 + x2 - 3
=0 

d)  x lim
+¥
( x2 - 2x + 1- x 2 - 2x + 4)  ¥ - ¥
 
-3
lim
x +¥
( x 2 - 2 x + 1 - x 2 - 2 x + 4 = lim ) x  +¥ 2
x - 2x + 1 + x2 - 2x + 4
=0 

 
62. Calcula els límits següents:
x2 3 x-2
æ x -1 ö÷ æ x + 2 ö÷
lim çç lim çç
a) x+¥ çè 3x + 2 ÷÷ø b) x+¥ çè x ÷÷ø

x2
æ x -1 ö ÷ =0  
a)  x lim çç
+¥ èç 3 x + 2 ÷ø÷

3 x -2 3 x -2
æ x + 2 ÷ö æ x + 2 ö÷
b)  x lim çç ÷÷  1¥      lim çç ÷ = e6  
+¥ çè x ø x +¥ çè x ø÷

   

584
Límit d’una funció 1010

63. Calcula aquests límits:


2 x+1
æ 2 x3 - 3x ÷÷ ö x
a) lim ççç 3 2
x+¥ èç 8 x + x - 3 x + 1÷ø÷

3 x 2 -2 x+1
æ 3 x 2 - 2 x + 1ö÷ x 2 -3
b) lim ççç ÷÷
x+¥ èç x - 8 ø÷

2x
æ x3 - x 2 + 3x ÷ö
c) lim ççç x - ÷÷
x+¥ èç x 2 + 1 ÷ø

2 x +1
æ 2x3 - 3x ö÷ x 1
a)  lim çç 3 ÷÷ =  
x +¥ çè 8 x + x - 3 x + 1÷ø
2
16

3 x 2 - 2 x +1
æ 2 ö
çç 3 x - 2 x + 1÷÷
x 2 -3
b)  x lim
+¥ ç
÷÷ = +¥  
è x -8 ø

2x
æ x 3 - x 2 + 3 x ÷ö 1
c)  x lim çç x - ÷÷ = 4  
+¥ çè x 2 + 1 ÷ø e

 
64. Calcula primer els límits laterals i després determina aquests límits:
�� ����� ����
a) ��� d) ���
��� ���� ���� �� ���
���
b) ����� � ���� e) ��� �� � �
��� ���� �

�� ����� �
c) ��� � f) ���
��� ��� ���� √���

� � ����� ����
a) lim �� d) lim ��
��� ���� ���� � � ���
���
b) lim �� � ���� � 3�3 e) lim �� � ���
��� ���� �

� � ����� �
c) lim � � √3 f) lim ��
��� ��� ���� √���

��
65. Amb la calculadora, completa la taula i comprova que si ���� �
���
,
aleshores ������� � ��� �
���

 
   

585
Límit d’una
Límit d’una funció
funció 10

66. Determina els límits indicats per a aquesta funció:


� � ����������� � �
���� � � �  
� � ��������� � �
a) �������� b) ���� ����
��� ���

a)  lim f ( x ) = lim ( x - 1) = 0 b)  lim f ( x ) = lim+ ( x 2 + 1) = 2  


x 1- x 1-
           x  1+ x 1

 
��� �������������������� � �
67. Donada la funció ���� � �√��������������� � � � � , calcula
������������������������� � �
a) �������� b) ���� ���� c)� �������� d) ���� ����
��� ��� ��� ���

a) lim f ( x ) = lim - x = 0 2
c) lim f ( x ) = lim- x = 2
x  0- x  0- x  4- x4

b) lim f ( x ) = lim+ x =0 d) lim f ( x ) = lim 3 = 3


+ +
x  0+ x0 x 4 x 4

68. Fixa’t en la gràfica de la funció i determina els límits següents:

a) �������� b) ������� c)� �������� d) ���� ����


��� ��� ��� ���

a) lim f ( x ) = 1 c) lim f ( x ) = 5
x -2- x  4-

b) lim f ( x ) = -3 d) lim f ( x ) = 5
x -2+ x  4+

 
69. Calcula els límits següents tenint en compte la gràfica de la funció:

 
a) �������� b) ������� c)� ��� ���� d) ��� ����
��� ��� ���� ����

a)  lim f ( x ) = 1 c)  x lim f (x) = 0  


x -2-
                
+¥

1
b)  lim f ( x ) = - d)  x lim f ( x ) = +¥  
x -2+ 2 -¥
               
586
Límit d’una funció 1010

 
70. Calcula els límits següents:
a) ��� ��� � c) ���
��
��� �
�→� �→

��� �
b) ���� �� � d) ���
�→ �→� ��� �

a) lim ��� x � ��
�→�

b) lim� tg x → lim
��
tg x � �� lim� tg x � ��
�→ � �→ �
� � �→

c) lim
��
�i� x � ��
�→

��� � � ��� � ��� �
d) lim →� lim → �� lim → ��
�→� ��� � � �→�� ��� � �→�� ��� �

 
71. Donada la funció f(x) definida a trossos, troba’n els límits:
�� � � �� � � ��

���� � � �� � � � � � � 
���
�� � �� � �� �� � � �
 
a) ��� ���� c) ���� ���� e) ��� ���� g) ���� ����
�→�� �→�� �→�� �→�

b) ��� ���� d) ���� ���� f) ������� h) ���� ����


�→�� �→�� �→� �→�

a)  x lim f ( x ) = -¥       c)  lim f ( x ) = -3      e)  xlim f ( x ) = -3     g)  lim f ( x ) = -5  


-¥ x -2- -2 x  3+

9
b)  x lim f ( x ) = +¥       d)  lim f ( x ) = -3      f)  lim f ( x ) =     h)  lim f ( x)  No existeix.  
+¥ x -2+ x  3- 2 x3

 
72. Resol els límits següents:
3x + 1 x2 - 4
a) lim d) xlim
x0 x2 + 2 x -2 x2 + 2x

x x+4
b) lim 2 e) xlim
x0 x + 2x -2 x2 + 2 x +1

2 x +1 x2 + 4
c) lim f) lim
x0 x 2 -1 x-2 x 2 + 2 x

3x +1 1 3x +1 3x +1
a) xlim  lim = -¥ lim = +¥
0 x2 + 2x 0 x  0- x2 + 2x x  0+ x2 + 2x

x 0 x x 1
b) xlim  lim = lim =
0 x2 + 2x 0 x 0 x 2 + 2 x x  0 x ( x + 2) 2

2x + 1
c) lim = -1
x 0 x2 -1

x2 - 4 0 x2 - 4 ( x - 2)( x + 2)
d) xlim  lim = lim =2
-2 x2 + 2x 0 x -2 x 2 + 2 x x -2 x ( x + 2)

587
Límit d’una
Límit d’una funció
funció 10

x +4
e) xlim =2
-2 x2 + 2x + 1

x2 + 4 8 x2 + 4 x2 + 4
f) lim  lim- = +¥ lim+ = -¥
x -2 x2 + 2x 0 x -2 x2 + 2x x -2 x2 + 2x

 
73. Determina els límits següents:

x 2 -1 x3 - 9 x 2 +15 x + 25
a) lim
x1 x 2 + x - 2 d) lim
x5 x3 - 5x 2 + 2 x -10

x -3 5 x3 - 3x 2
b) lim 2
x3 x - 6 x + 9 e) lim
x 0 2 x3 - x 2 + 3x

3x3 +12 x 2 - x - 4 x 2 - 3x + 2
lim
c) x-4 3 f) lim
x + 7 x 2 +14 x + 8 x1 x - 2 x2 - x + 2
3

x2 -1 0 x2 -1 ( x - 1)( x + 1) 2
a)  lim 2
 lim 2
x 1 x + x - 2
= lim
x 1 ( x - 1)( x + 2)
=  
x 1 x + x -2 0          3

x -3 0 1 1
b)  lim 2
 lim = -¥           lim = +¥  
x 3 x - 6x + 9 0          x  3- x -3 x  3+ x -3

3 x 3 + 12 x 2 - x - 4 0 3 x 3 + 12 x 2 - x - 4 (3 x 2 - 1)( x + 4) 47
c)  xlim  lim
x -4 x 3 + 7 x 2 + 14 x + 8
= lim
x -4 ( x + 1)( x + 2)( x + 4 )
=  
-4 x 3 + 7 x 2 + 14 x + 8 0       6

2
x 3 - 9 x 2 + 15 x + 25 0 x 3 - 9 x 2 + 15 x + 25 ( x + 1)( x - 5)
d)  lim  lim = lim 2 =0  
x 5 x 3 - 5 x 2 + 2 x - 10 0       
x 5 x 3 - 5 x 2 + 2 x - 10
( )(
x 5 x + 2 x - 5
)

5x3 - 3x2 0 5x3 - 3x2 x 2 (5 x - 3 )


e)  lim  lim = lim =0 
x 0 2x3 - x2 + 3x 0         
x 0 2 x 3 - x 2 + 3 x x  0 x (2 x 2 - x + 3 )

x2 - 3x + 2 0 x2 - 3x + 2 ( x - 1)( x - 2) 1
f)  lim  lim
x 1 x - 2 x 2 - x + 2
3
= lim
x 1 ( x - 1)( x - 2)( x + 1)
=  
x 1 x - 2x2 - x + 2
3
0        2

 
74. Calcula aquests límits:

x+2- 2 x2 + 5 - 3
a) lim d) lim
x0 x x 2 x2 - 4

x - 3 -1 2 x -1
b) lim e) lim
x4 x 2 -16 x1 2x - 2

x + 1 -1 x 2 - 3x - 4
c) lim f) lim
x0 x x4 2 x +1 - 3

x +2- 2 0 x +2- 2 1 2
a)  lim             lim = lim =  
x 0 x 0 x 0 x x 0 x +2 + 2 4

588
Límit d’una funció 1010

x - 3 -1 0 x - 3 -1 1 1
b)  lim              lim = lim =  
x4 x 2 - 16 0 x 4 x 2 - 16 x 4
( x + 4)( x - 3 + 1) 16

x + 1-1 0 x + 1-1 1 1
c)  lim              lim = lim =  
x 0 x 0 x 0 x x 0 x +1+1 2

2
x2 + 5 - 3 1 1
d)  lim x 2+ 5 - 3  0 lim = lim =  
x 2 x -4 0 x 2 x2 - 4 x 2
x2 + 5 + 3 6
          

2x -1 1 2x -1 2 x -1
e)  lim               lim = +¥         lim  No existeix.  
x 1 2x - 2 0 x 1+ 2x - 2 x1- 2x - 2

2
x2 - 3x - 4 ( x + 1)( 2 x + 1 + 3)
f)  lim x - 3 x - 4  0             lim = lim = 15  
x4 2x + 1- 3 0 x 4 2 x + 1 - 3 x 4 2

 
75. Calcula el límit que hi ha a continuació:

x + 3 - 2x
lim
x1 x -1

x + 3 - 2x 0
lim 
x 1 x -1 0
 
x + 3 - 2x ( x + 1)( x + 3 - 4 x 2 ) x + 3 - 4x2 x +1 1 ( x - 1)(-4 x - 3) 7
lim = lim = lim ⋅ lim = ⋅ lim =-  
x 1 x -1 x 1 (
x - 1)( x + 3 + 2 x ) x 1 x -1 x 1 x + 3 + 2 x 2 x 1 x -1 2

76. Troba el límit següent:


3
x-3 2
lim
x2 x-2

3
x -3 2 0
lim   
x 2 x -2 0

3
3
x -3 2 x 2 - 3 22 x -2 3
2
lim = lim = lim =  
( x - 2)( x + 2 ) x  2 ( x - 2)( 3 x + 3 2 )
2
x 2 x -2 x 2 3 3
8

 
77. Calcula el valor dels límits següents:

x- a ( x3 + 2x) -(c3 + 2a)


a) lim c) lim
x a x-a xc x -c

x+b - b x 2 + ( d - 4) x - 4 d
b) lim   d) xlim
xb x -d x 2 + ( d -1) x - d

a) ▪ Si a ് 0:
x- a 0 x- a ( x - a) a
lim  lim = lim =
 
xa x-a 0       
x a x -a x a
( x - a)( x + a ) 2 a
 
589
Límit d’una
Límit d’una funció
funció 10

  ▪ Si a  0: 
x 0 x x 1 1
lim  lim = lim = lim 
x 0 x 0          x0 x x0 x x x0 x 0
 
x 1 x
lim = lim = +¥    lim  No existeix.  
x  0+ x x 0 x x0- x

b) ▪ Si b ് 0:
x +b- b 2b - b b
lim = lim = ( 2 - 1)
xb x x b b b
 
    ▪ Si b  0: 

x
lim = +¥  És el mateix límit de l'apartat anterior.  
x0+ x
c) ▪ Si c  a i c  0: 

( x 3 + 2 x ) - (c3 + 2a) 2(c - a) ( x 3 + 2 x ) - (c3 + 2a)


lim  lim =¥
x c x -c 0 x c x -c
      
    ▪ Si c  a i c  0: 

( x 3 + 2 x ) - 2a -2a ( x 3 + 2 x ) - 2a
lim  lim =¥
x 0 x 0 x 0 x
           
    ▪ Si c  a i c  0: 

( x 3 + 2 x ) - (c3 + 2c) 0 ( x 3 + 2 x ) - ( c 3 + 2c ) ( x - c )( x 2 + cx + (2 + c 2 ))
lim  lim = lim = 3c 2 + 2
x c x -c 0        x c x -c x c x -c
 
    ▪ Si c  a i c  0: 
x3 + 2x 0 x3 + 2x
lim  lim = lim x 2 + 2 = 2  
x0 x 0 x 0 x x 0
              
d) ▪ Si d ് 0:
x 2 + ( d - 4) x - 4 d 0 x 2 + ( d - 4) x - 4 d ( x + d )( x - 4) d + 4
lim  lim = lim =
x -d x + (d - 1) x - d
2
0 x -d x + (d - 1) x - d
2 x -d ( x + d )( x - 1) d + 1
        
    ▪ Si d  0: 
x2 - 4x 0 x2 - 4x x ( x - 4) ( x - 4)
lim  lim = lim = lim =4 
x 0 x2 - x 0              x 0 x 2 - x x 0 x ( x - 1) x 0 ( x - 1)

 
78. Determina aquests límits:

sec 2 x -1
a) lim cos x - 3 sen x c) lim
xp x0 sec x ⋅ sen x

3 3 sen 2 x ⋅ tg 2 x + sen 2 x
b) limp tg x ⋅ cos x d) lim
x
2
x0 tg 2 x

a) lim cos x - 3 sin x  No existeix.


xp

b) limp tg 3 x ⋅ cos3 x  ¥ ⋅ 0
x
2

590
Límit d’una funció 1010

sin3 x
lim tg 3 x ⋅ cos3 x = lim ⋅ cos3 x = lim sin3 x = 1
x
p
x
p
cos 3 x x
p
2 2 2

sec 2 x -1 0
c) lim 
x0 sec x ⋅ sin x 0

1 1- cos 2 x
2 -1
sec x -1 2
cos x 1- cos 2 x sin x
lim = lim cos x = lim = lim = lim =1
x0 sec x ⋅ sin x x0 1 x0 sin x x0 sin x x0 sin x
⋅ sin x
cos x cos x

2 2 2
d) lim sin x ⋅ tg 2 x + sin x  0
x 0 tg x 0

sin2 x sin4 x + cos 2 x ⋅ sin2 x


sin2 x ⋅ + sin2 x
sen2 x ⋅ tg 2 x + sin2 x 2
cos x cos 2 x
lim = lim = lim = lim sin2 x + cos 2 x = 1
x0 tg 2 x x0 sin2 x x0 sin2 x x0

cos 2 x cos 2 x

79. Determina el límit d’aquesta funció quan x tendeix a 0 i quan x tendeix a 3.


��
���� �
�� � ���

Especifica el valor dels límits laterals, si cal.

�� � �� ��
lim → lim � lim �0
�→� � � ��� � � �→� � � ��� � �→� ���
�� 81
lim →
�→� � � � �� � 0
��
lim� � �� ��
� � ��� �
��→� ��
→ No existeix lim .
�→� � � ��� �
lim� � ��
�→� � � ��� �

80. Fixa’t en les taules de valors d’aquesta funció:


��� � ��
���� �
��� � �

591
Límit d’una
Límit d’una funció
funció 10

Digues si és cert que y = 2 és una asímptota. Quan x → +3, indica si la funció es troba per sobre o per
sota de l’asímptota. Què passa quan x → – 3?
Sí, és cert que y = 2 és una asímptota horitzontal.
Quan x tendeix a +∞, la funció és per sota de l’asímptota.
Quan x tendeix a –∞, la funció és per sobre de l’asímptota.
 
81. Construeix una taula com la de l’activitat anterior i determina si y = –3 és una asímptota vertical de la
����
funció ���� � . Indica si la funció està situada per sobre o per sota de l’asímptota.
���

x  1  10 100 1 000 10 000 


f(x)   0,5  2,5455  2,9505  2,9950  2,9995 
 

x  2  10  100  1 000  10 000 


f(x)   2,6666  3,5556  3,0505  3,0050  3,0005 
 
Sí, és cert que y = –3 és una asímptota horitzontal.
Quan x tendeix a +∞, la funció és per sobre de l’asímptota.
Quan x tendeix a –∞, la funció és per sota de l’asímptota.

82. Fixa’t en les taules de valors d’aquesta funció:


�� � �
���� �
���

 
Indica si és cert que x = 3 és una asímptota vertical. Quan � → �� , digues si la branca infinita de la
funció tendeix a +∞ o a –∞. Què passa quan � → �� ?
Sí, és cert que x = 3 és una asímptota vertical.
Quan x tendeix a 3 per l’esquerra, la branca infinita de la funció tendeix a –∞.
Quan x tendeix a 3 per la dreta, la branca infinita de la funció tendeix a +∞.

83. Construeix una taula com la de l’activitat anterior i determina si x = –2 és una asímptota vertical de la
�� ���
funció ����� � . Indica si la branca infinita de la funció tendeix a +∞ o a –∞.
���

x  1  1,5  1,9  1,99  1,999  1,9999 


f(x)   4  13,5  93,1  993,01  9 993,001 
 
99 993,0001 
 
x  2,0001  2,001  2,01  2,1  2,5 
f(x)   10 0007  10 007,001  1 007,01  107,1  27,5 

592
Límit d’una funció 1010

Sí, és cert que x = –1 és una asímptota vertical.


Quan x tendeix a –2 per l’esquerra, la branca infinita de la funció tendeix a –∞.
Quan x tendeix a –2 per la dreta, la branca infinita de la funció tendeix a +∞.
 
84. Observa aquesta taula de valors de la funció:

Aquesta és la taula de valors de la recta y = 2 x + 6.

 
La recta és una asímptota de f(x)? Quina posició té la funció respecte de la recta quan x tendeix a
+∞? Investiga la posició relativa de totes dues quan x tendeix a –∞.

Sí, es cert que y = 2x + 3 és una asímptota obliqua.


Quan x tendeix a +∞, la funció es troba situada per sobre de l’asímptota.
Si x = - 1000, f(x) – 2x – 3 > 0, i quan x tendeix a –∞, la funció es troba situada per sobre de l’asímptota.

85. Construeix una taula com la de l’activitat anterior i determina si y = 4x + 2 és una asímptota obliqua
��� ��
de la funció ���� � . Indica si la funció està situada per sobre o per sota de l’asímptota.
����

 
x 10  100 1 000 10 000
f(x)   42,2632  402,0251  4 002,0025  40 002,0003 
 
Aquesta és la taula de valors de la recta y = 4x + 2.
x 10  100 1 000 10 000
y  42  402  4 002  40 002 
Sí, és cert que y = 4x + 2 és una asímptota obliqua.
Quan x tendeix a +∞, la funció es troba situada per sobre de l’asímptota.
Si x = 1 000 → f ( x ) - 4 x - 2 < 0 , i quan x tendeix cap a ∞ la funció es troba per sota de l’asímptota.

 
86. Calcula les asímptotes horitzontals i verticals de les funcions següents:
� ���� ���
a) ���� � c) ���� � e) ���� �
��� ����� �� ��
� ���� ���
b) ���� � d) ���� � f) ���� �
�� �� �� �� �� ��

593
Límit d’una
Límit d’una funció
funció 10

a)  Dom f =  - {-3}


 
lim f ( x ) = +¥ü ï
ï
x-3- ï
ý  La funció té una asímptota vertical a x = -3.
lim+ f ( x ) = -¥ïï
x-3 þï
 
x
lim f ( x ) = lim = 1  f ( x ) té una asímptota horitzontal a y = 1.  
x¥ x¥ x + 3

b)  Dom f =  - {- 1, 1}
 
lim f ( x) = +¥üï
ï
x-1- ï
ý  La funció té una asímptota vertical a x = -1.
lim+ f ( x) = -¥ï
ï
x-1 ï
þ
 
lim f ( x) = -¥üïï
x1- ïý  La funció té una asímptota vertical a x = 1.
lim+ f ( x) = +¥ïï
x1 ïþ
 
2
lim f ( x) = lim 2 = 0  f ( x) té una asímptota horitzontal a y = 0.  
x¥ x¥ x -1

c)  Dom f     → f(x) no té asímptotes verticals. 


-x + 5
lim f ( x) = lim = 0  f ( x) té una asímptota horitzontal a y = 0.  
x¥ x¥ -4 - x 2

d)  Dom f =  - {- 3, 3 }
 
lim - f ( x) = -¥üïï
x- 3 ï
ý  La funció té una asímptota vertical a x = - 3.
lim + f ( x) = +¥ïï
x- 3 ïïþ
 
lim- f ( x ) = -¥ ü
ï
ï
x 3 ï
ý  La funció té una asímptota vertical a x = 3.
lim+ f ( x ) = +¥ïï
x 3 ï
ï
þ
 
3x -1
lim f ( x) = lim = 0  f ( )
x té una asímptota horitzontal a y = 0.  
x¥ x¥ x2 - 3
e)  Dom f =  - {-2, 2}
 
ü
lim f ( x) = -¥ïï
x-2- ïý  La funció té una asímptota vertical a x = -2.
lim+ f ( x) = +¥ïï
x-2 ïþ
 
lim f ( x) = -¥üïï
x2- ïý  La funció té una asímptota vertical a x = 2.
lim+ f ( x ) = +¥ïï
x2 ïþ
 
x +1
lim f ( x ) = lim 2 = 0  f ( x ) té una asímptota horitzontal a y = 0.
x¥ x¥ x - 4
 
f)  Dom f     → f(x) no té asímptotes verticals. 

2x2
lim f ( x ) = lim = 2  f ( x ) té una asímptota horitzontal a y = 2.  
x¥ x¥ x2 +1

   

594
Límit d’una funció 1010

87. Determina les asímptotes obliqües de les funcions següents:


2
3x3
a) f ( x ) = x 2 + 5 x c) f ( x ) = 5 -
3
x + 2x 2 x + 8x

2 x2 - 4 2 3
b) f ( x) = d) f ( x ) = x2 - 6 x
x -1 x + x -1

f ( x) x2 + 5x üï
lim = lim =1 ïï
x+¥ x 2
x+¥ x + 2 x ïï
ïý  f ( x) té una asímptota obliqua en y = x + 3.
a)  æ x + 5x
2 ö÷ 3 x ï  
lim ççç - x÷÷ = lim = 3ïïï
x+¥ ç è x+2 ÷ø x+¥ x + 2 ïïþ

f ( x) 2 x2 - 4 üï
lim = lim =2 ïï
x+¥ x 2
x+¥ x - x ïï
ïý  f ( x) té una asímptota obliqua en y = 2 x + 2.
b)  æ 2 x2 - 4 ö÷ 2x - 4 ï  
ç
lim çç - 2 x÷÷ = lim = 2ïïï
x+¥ çè x -1 ø÷ x+¥ x -1 ïïþ

f ( x) 5 - 3x 3 3 üï
lim = lim =- ïï
ïïï
3
x+¥ x x+¥ 2 x + 8x 2 3
c)  æ 5 - 3x3 3 ö÷ ý  f ( x) té una asímptota obliqua en y = - x.  
12 x - 5 ï 2
lim ççç 2 + x÷ = lim = 0ïïï
ç 2 x + 8 2 ø÷÷ x+¥ 2 x 2 + 8
ïïþ
x+¥ è

f ( x) x 2 - 6 x3 ü
ï
lim = lim = -6 ï
ï
x+¥ x 32
x+¥ x + x - x ï
ï
ï
ý  f ( x) té una asímptota obliqua en y = -6 x + 7.  
d)  æ x 2 - 6 x3 ö÷ 7 x 2
- 6 x ï
lim ççç 2 + 6 x÷÷ = lim 2 = 7ïï
ï
x+¥ èç x + x -1 ø÷ x+¥ x + x -1 ï
ï
þ
 
88. Determina les asímptotes de les funcions següents:
2 x -1 x3 + 1
a) f ( x) = 2
c) f ( x) =
x - 5x + 6 x2 - 2x

x2 + 8 x2 - 5x + 4
b) f ( x ) = d) f ( x ) =
x + 2 x 2 - 11x - 12
3
x - 3 x 2 - 13 x + 15
3

a)  Dom f =  - {2, 3}


 
lim f ( x ) = +¥üïï
x2- ïý  La funció té una asímptota vertical a x = 2.
lim+ f ( x ) = -¥ïï
x2 ïþ
 
lim- f ( x) = -¥üïï
x3 ïý  La funció té una asímptota vertical a x = 3.
lim+ f ( x ) = +¥ïï
x3 ïþ
 
2 x -1
lim f ( x) = lim 2 = 0  f ( x) té una asímptota horitzontal a y = 0.  
x¥ x¥ x - 5 x + 6

b)  Dom f =  - {-4, - 1, 3}


 
lim- f ( x) = -¥ü
ï
ï
x-4 ïý  La funció té una asímptota vertical a x = -4.
lim+ f ( x) = +¥ïï
x-4 ï
þ
 

595
Límit d’una
Límit d’una funció
funció 10

lim f ( x) = +¥üï
ï
x-1- ï
ý  La funció té una asímptota vertical a x = -1.  
lim+ f ( x) = -¥ï
ï
x-1 ï
þ

lim f ( x) = -¥üï
ï
x3- ï
ý  La funció té una asímptota vertical a x = 3.
lim+ f ( x) = +¥ï
ï
x3 ï
þ
 
2
x +8
lim f ( x) = lim = 0  f ( x) té una asímptota horitzontal a y = 0.  
x¥ x¥ x + 2 x 2 -11x -12
3

c)  Dom f =  - {0, 2}


 
lim- f ( x) = +¥ü
ï
ï
x0 ï
ý  La funció té una asímptota vertical a x = 0.
lim+ f ( x) = -¥ï
ï
x0 ï
þ
 
lim f ( x) = -¥üï
ï
x2- ï
ý  La funció té una asímptota vertical a x = 2.
lim+ f ( x) = +¥ï
ï
x2 ï
þ
 
x3 + 1
lim f ( x ) = lim 2 = ¥  f ( x ) no té asímptotes horizontals.
x¥ x¥ x - 2 x
 
f ( x) x +1 3 ü
ï
lim = lim 3 =1 ïï
x+¥ x x+¥ x - 2 x 2 ïï
ïý  f ( x) té una asímptota obliqua a y = x + 2.
æ x3 +1 ö÷ 2 x 2
+ 1 ï  
lim ççç 2 - x÷÷ = lim 2 = 2ïïï
x+¥ è ç x - 2x ø÷ x+¥ x - 2 x ïïþ

x2 - 5x + 4 ( x - 1)( x - 4)
d)  f ( x ) = =
x - 3 x 2 - 13 x + 15 ( x - 1)( x - 5)( x + 3)
3

 
Dom f =  - {- 3, 1, 5}
 
lim f ( x ) = -¥ ü
ï
ï
x-3- ïý  La funció té una asímpota vertical a x = -3.
 
lim+ f ( x ) = +¥ïï
x-3 ï
þ

lim f ( x ) = +¥üïï
x1- ïý  La funció té una asímptota vertical a x = 1.
 
lim+ f ( x) = +¥ïï
x1 ïþ

lim f ( x) = -¥ üï
ï
x5- ï
ý  La funció té una asímptota vertical a x = 5.  
lim+ f ( x ) = +¥ï
ï
x5 ï
þ

x2 - 5x + 4
lim f ( x) = lim = 0  f ( x) té una asímptota horitzontal a y = 0.  
x¥ x¥ x3 - 3x 2 -13x + 15
 
89. Calcula les asímptotes de les funcions següents:
� √����
a) ���� � � c) ���� �
��� ���

��� ��� ��
b) ���� � d) ���� �
��� �� ���

596
Límit d’una funció 1010

a)  Dom f = (-¥, 0 ] È (3, + ¥)


 

lim f ( x)  No existeix.üïï
x3- ïý  La funció té una asímptota vertical a x = 3.
lim+ f ( x) = +¥ ïï
x3 ïþ
 
x
lim f ( x ) = lim = 1  f ( x) té una asímptota horitzontal a y = 1.  
x¥ x¥ x -3

b)  Dom f = (-¥, - 3) È (3, + ¥)


 

x +3 ( x + 3)( x 2 - 9 )
0 lim = lim =0
lim f ( x )  ( x + 3)( x - 3)
x -3 0     
x -3
x2 - 9 x -3
 
lim f ( x)  No existeix.ü
ï
ï
x3- ï
ý  La funció té una asímptota vertical a x = 3.
lim+ f ( x) = +¥ ï
ï
x3 ï
þ  
x +3
lim f ( x) = lim = 1  f ( x) té una asímptota horitzontal a y = 1.
x+¥ x+¥
x2 - 9
 
x +3
lim f ( x ) = lim = -1  f ( x ) té una asímptota horitzontal a y = -1.
x-¥ x-¥
x2 - 9
 
é ö
c)  Dom f = ê 3 , 2÷÷÷ È (2, + ¥ )  
êë 2 ø

lim f ( x) = -¥ ü
ï
ï
x2- ï
ý  La funció té una asímptota vertical a x = 2.
lim+ f ( x) = +¥ï
ï
x2 ï
þ  

2x - 3
lim f ( x) = lim = 0  f ( x) té una asímptota horitzontal a y = 0.
x+¥ x+¥ x-2
 
d)  Dom f =  - {- 3}
 

lim f ( x) = -¥üï
ï
x-3- ïý  La funció té una asímptota vertical a x = -3.
lim+ f ( x) = +¥ïï
x-3 ï
þ  

x 4 +1
lim f ( x) = lim = ¥  f ( x) no té asímptotes horizontals.
x¥ x¥ x +3
 
f ( x) x +1 4 ü
ï
ïï
lim = lim 2 =1 ïï
x+¥ x x+¥ x + 3x ï
æ x4 +1 ý  f ( x ) té una asímptota obliqua a y = x - 3.
çç ÷ö÷ x 4 + 1 - x 2 - 3x ïï
lim ç - x÷÷ = lim = -3ïï  
x+¥ ççè x + 3 ÷ø x+¥ x+3 ïï
þï
   

597
Límit d’una
Límit d’una funció
funció 10

 
90. Calcula les asímptotes d’aquestes funcions i la posició de les branques infinites:
�� ���� ������ �� ���� ������
a) ���� � c) ���� �
��� �� ��
�� ���� ������ �� ���� ������
b) ���� � d) ���� �
�� ���� ������

�� ���� ������ �� ���� ������


c) ���� � e) ���� �
��� �� ��

a) Dom f = � - {-3}
� � � �� � � ��� � � ����
lim →
�→�� ��� 0
� � ��� � ������
lim � � �∞
���
� �→�� � � ���������� → La funció té una asímptota vertical a x = 3. Per la esquerra la
lim � � �∞
�→�� ���

branca infinita de la funció tendeix a +∞, i per la dreta tendeix a –∞.

� � ��� � ������
lim � �∞ →  La funció no té asímptota horitzontal. 
�→�� ���

���� � � ��� � ������


lim � lim � �∞ →  La funció no té asímptota obliqua. 
�→�� � �→�� � � ���

b) Dom f = � - {-3,2}
� � � �� � � ��� � � ����
lim →  
�→�� �� � � � � 0
� � ��� � ������
lim � � �∞
� � ����
��→�� � � ��� � ������ → La funció té una asímptota vertical a x = –3. Per la esquerra la branca
lim � � � �∞
�→�� � ����

infinita de la funció tendeix a –∞, i per la dreta tendeix a +∞.

� � � �� � � ��� � � 0
lim →
�→� �� � � � � 0
� � � �� � � ��� � � �� � ���� � � �� � ��
��� � ��� � 0
�→� �� � � � � �→� �� � ���� � ��

→ La funció no té asímptota vertical en x = 2.

� � ��� � ������
��� � �∞ → La funció no té asímptota horitzontal.
�→�� � � ����

� ���� � � � �� � � ��� � �
� lim � lim ��
�→�� � �→�� �� � � � �
� � � →
� lim �� � �� � ��� � � � �� � lim ��� � ��� � � � ��
��→∞ �� � � � � �→∞ �� � � � �

→ La funció té una asímptota obliqua: y = x – 7.

598
Límit d’una funció 1010

� � ��� � ������ �
c) lim →  
�→� ��� �

� � � �� � � ��� � � �� � ���� � � �� � ��
lim � lim �0→
�→� ��� �→� ���
→ La funció no té asímptota vertical a x = 2.

� � ��� � ������
lim � �� → La funció no té asímptota horitzontal
�→�� ���
���� � � ��� � ������
lim � lim � �� → La funció no té asímptota obliqua.
�→�� � �→�� � � ���

d) Dom f = �  {2,2}
� � � �� � � ��� � � ���
lim →  
�→�� �� � � 0
� � � �� � � ��� � �
� lim � ��
�→�� � �� � � → 
� � � � �� � � ��� � �
lim �
��→�� � ��
�� � �
La funció té una asímptota vertical a x = –2.

Per la esquerra la branca infinita de la funció tendeix a ∞, i per la dreta tendeix a +∞.

� � � �� � � ��� � � 0
lim →
�→� �� � � 0
� � � �� � � ��� � � �� � ���� � � �� � ��
lim � lim �0→
�→� �� � � �→� �� � ���� � ��
→ La funció no té asímptota vertical a x = 2.

� � ��� � ������
lim � �� → La funció no té asímptota horitzontal
�→�� � � ��
���� � � ��� � ������
lim � lim ��
�→�� � �→�� � � ���
� →
� � ��� � ������ ��� � ������
lim � � �� � lim � ��
�→�� � � �� �→�� � � ��

→ La funció té una asímptota obliqua: y = x – 6.

e) Dom f = �  {2}
� � � �� � � ��� � � 0
lim →  
�→� �� � ��� 0
� � ��� � ������ ������
lim � lim � 0 →  La funció no té asímptota vertical a x = 2.
�→� ������ �→� ������
� � ��� � ������
lim � �� →  La funció no té asímptota horitzontal. 
�→�� ������

���� � � ��� � ������


lim � lim ��
�→�� � �→�� � � ��� � ���
� →
� � ��� � ������ ��� � �����
lim � � �� � lim � ��
�→�� � � ����� �→�� � � �����

599
Límit d’una
Límit d’una funció
funció 10

→ La funció té una asímptota obliqua: y = x – 2.

f(x) – x + 2 = 0 → L’expressió de la funció coincideix amb l’equació de l’asímptota excepte a x = 2.

f) Dom f = �
� � � �� � � ��� � �
lim → �∞ → 
�→�� �� � �

→La funció no té asímptota horitzontal.

���� � � ��� � ������


lim � lim ��
�→�� � �→�� � � ���
� � � ��� � ������ ��� � �����

lim � � �� � lim � ��
�→�� � � �� �→�� � � ��

→ La funció té una asímptota obliqua: y = x – 6.

Si x = 1 000, f(x) – x + 6 > 0, i quan x tendeix a +∞ la funció és troba situada per sobre de l’asímptota.

Si x = 1 000, f(x) – x + 6 < 0, i quan x tendeix a ∞ la funció és troba situada per sota de l’asímptota.
 
91. Calcula les branques infinites i les asímptotes d’aquestes funcions:
a) f(x) = x2 + 5x – 1 c) f(x) = log x
b) f(x) = 2x – 1 d) f(x) = tg x
a) Dom f = � → La funció no té asímptota vertical.
lim �� � � �� � �� � �∞ → La funció no té asímptota horitzontal.
�→�

���� �� � ������
lim � lim � �∞ → La funció no té asímptota obliqua.
�→� � �→� �

b)   La gràfica de la funció és una recta, així doncs, no té asímptotes. Branques parabòliques: 


ì lim f ( x ) = +¥
ï
ï
ï
í
x +¥
 
ï lim f ( x ) = -¥
ï
ï
î x -¥

c) Dom f = (0,+∞)
lim l�� � � � ∞ →  La funció no té asímptota vertical a x = 0.
�→��

La branca infinita de la funció tendeix a ∞.

lim l�� � � � ∞ →  La funció no té asímptota horitzontal. 


�→��

���� ��� �
lim � lim � � →  La funció no té asímptota obliqua.
�→�� � �→�� �


d) Dom f = � - { � ��� � � ��


lim� �� � →   
�→ �

600
Límit d’una funció 1010

lim
��
���� � ��
�� �
� lim ���� � ��→ La funció té una asímptota vertical a x = .
2

��
��
2

Per l’esquerra la branca infinita de la funció tendeix cap a +∞, i per la dreta, cap a ∞.

Com que es tracta d’una funció periòdica, de període π, tots els punts que no pertanyen al domini són
asímptotes del mateix tipus. Per tant, la funció no té asímptotes horitzontals ni obliqües.

92. Determina les asímptotes de les funcions següents:


|����| �
a) ���� � b) ���� � |���|

ïìï 3 - 2 x 3
ïï si x <
2x - 3
a)  f ( x ) = = ïí x 2 
x ïïï 2 x - 3 si x ³
3
ïîï x 2

3- 2x ïü
lim f ( x) = lim- = -¥ ïï
x0- x0x ïï
ý  La funció té una asímptota vertical a x = 0.
3- 2x ï
lim+ f ( x ) = lim+ = +¥ïïï
x0 x0 x ïþ  
3- 2x
lim f ( x ) = lim = -2  f ( x ) té una asímptota horitzontal a y = -2.
x-¥ x-¥ x
 
2x - 3
lim f ( x) = lim = 2  f ( x) té una asímptota horitzontal a y = 2.
x+¥ x+¥ x
 
ìï x
x ïïï si x < 1
b)  f ( x ) = = ïí1- x  
x - 1 ïï x
ïï si x ³ 1
îï x - 1

x ü
ï
lim f ( x) = lim- = +¥ï
ï
x1- 1- x
x1 ï
ï
ý  La funció té una asímptota vertical a x = 1.  
x ï
ï
lim+ f ( x) = lim+ = +¥ï
ï
x1 x1 x -1 ï
þ
x
lim f ( x ) = lim = -1  f ( x ) té una asímptota horitzontal a y = -1.  
x-¥ x-¥ 1- x
x
lim f ( x) = lim = 1  f ( x) té una asímptota horitzontal a y = 1.  
x+¥ x+¥ x -1
93. Estudia la continuïtat de la funció següent en els punts x = 2, x = 0 i x = 2.

601
Límit d’una
Límit d’una funció
funció 10

lim f ( x ) = 4üïï
x2- ïý  La funció no és contínua a x = 2.
lim+ f ( x ) = 5ïï
x2 ïþ
És un punt de discontinuïtat de salt finit.

lim f ( x) = 0üï
ï
x0- ï
ý  La funció és contínua a x = 0.
lim+ f ( x)= 0 ï
ï
x0 ï
þ

lim f ( x ) = -5üï
ï
x-2- ïý  La funció no és contínua a x = -2.
lim+ f ( x )= -4 ïï
x-2 ï
þ
És un punt de discontinuïtat de salt finit.
 
94. Determina els punts de discontinuïtat de cadascuna de les funcions següents i indica de quin tipus
són:

 
a) La funció es defineix de la forma:

1
f ( x ) = x + n "x Î [ n, n + 1) , "n Î   
2

Cada x Î  es tracta d’un punt de discontinuïtat de salt finit.

b) Els punts de discontinuïtat són x = 0 i x = 3, ambdós punts tenen una discontinuïtat de salt finit.

c) L’únic punt de discontinuïtat és x = 2, té una discontinuïtat de salt infinit.

d) Els punts de discontinuïtat són x = 1 i x = 2, ambdós punts tenen una discontinuïtat evitable.

95. Determina si les funcions següents són contínues en el punt indicat:


a) ���� � � � �� ������� � ��      c)����� � √� � ��������� � � 
� ��
b) ���� � ��� � ���������� �     d)����� � �������� � �� 
� �� ����

602
Límit d’una funció 1010

lim f ( x) = 0ïüï f (-1) = 0 ïüï


x-1- ïý  ý  f ( x) és contínua a x = -1.
a) f ( x) = 0ïï
lim f ( x) = 0ïï xlim
x-1+ ïþ -1 ïþ

5üï æ1ö 5 ü
ï
lim- f ( x) = ïï f çç ÷÷÷ = ï
1 4 ï çè 2 ø 4 ï
ï
x ï ïï 1
2 ï
ý  ý  f ( x) és contínua a x = .
b) 5ï 5ï 2
lim+ f ( x) = ïï lim1 f ( x) = ï
ï
x
1 4ïï
ï
x 4ï
þï
ï
2 ï
þ 2

c) f(x) no està definida a x = 1 → f(x) no és contínua a x = 1.

d) f(x) no està definida en x = - 2→ f(x) no és contínua en x = -2.

96. Completa, a la llibreta, la taula per a aquesta funció:


࢞૛ െ ૛࢞ െ ૜
ࢌሺ࢞ሻ ൌ
࢞െ૜

 
Comprova que el límit, quan x tendeix a 3, és:
‫ࢌ ܕܑܔ‬ሺ࢞ሻ ൌ ૝
࢔՜૜

Quant val f (3)? Fes una representació de la funció.


Quina diferència hi ha entre les gràfiques de f (x) i de y = x + 1?

 
2
x - 2x - 3 ( x - 3)( x + 1)
lim = lim = lim ( x + 1) = 4  
x 3 x -3 x 3 x -3 x 3

No existeix f(3).

 
La gràfica de f(x) coincideix amb la gràfica de la recta y = x + 1, excepte al punt x = 3.

603
Límit d’una
Límit d’una funció
funció 10

97. Determina els punts de discontinuïtat de les funcions següents:


࢞ା૛ ࢞૛ ି૚
a) ࢌሺ࢞ሻ ൌ c) ࢌሺ࢞ሻ ൌ
‫ܠ‬૛ ା૜‫ܠ‬ା૞ ‫ܠ‬૛ ିૢ
࢞ି૜ ૛࢞૛ ା૞࢞ି૜
b) ࢌሺ࢞ሻ ൌ d) ࢌሺ࢞ሻ ൌ
‫ܠ‬૛ ି‫ିܠ‬૛૙ ‫ܠ‬૝ ି૝࢞૛ ା૜
a)  Dom f = 
 
La funció és contínua per a tot  .

b)  Dom f =  - {-4, 5}


 
lim f ( x ) = -¥ ü
ï
ï
x-4- ï
ý  No existeix xlim f ( x ).  
lim+ f ( x ) = +¥ï
ï -4
x-4 ï
þ

x = 4 és un punt de discontinuïtat de salt infinit.


lim f ( x) = -¥üïï
x5- ïý  No existeix lim f ( x).
 
lim+ f ( x) = +¥ï ïï x5
x5 þ
x = 5 és un punt de discontinuïtat de salt infinit.
c)  Dom f =  - {- 3, 3}
 
lim f ( x) = +¥üï
ï
x-3- ï
ý  No existeix xlim f ( x).
lim+ f ( x) = -¥ï
ï -3
x-3 ï
þ
x = –3 és un punt de discontinuïtat de salt infinit.
lim f ( x ) = -¥üïï
x3- ïý  No existeix lim f ( x).
lim+ f ( x) = +¥ïï x3
x3 ïþ
x = 3 és un punt de discontinuïtat de salt infinit.
d)  Dom f =  - {- 3, - 1, 1, 3 }
 
lim - f ( x ) = -¥ ü
ï
ï
x- 3 ï
ý  No existeix lim f ( x ).
lim + f ( x ) = +¥ïï x- 3
ï
ï
þ
x- 3
 
x = - 3 és un punt de discontinuïtat de salt infinit.

lim f ( x ) = +¥üïï
x-1- ïý  No existeix lim f ( x ).
lim+ f ( x) = -¥ïï x-1
x-1 ïþ  
x = –1 és un punt de discontinuïtat de salt infinit.
lim f ( x) = +¥üïï
x1- ïý  No existeix lim f ( x).
 
lim+ f ( x) = -¥ïï x1
x1 ïþ
x = 1 és un punt de discontinuïtat de salt infinit.

604
Límit d’una funció 1010

lim- f ( x) = -¥ü
ï
ï
x 3 ï
ý  No existeix lim f ( x).  
lim+ f ( x) = +¥ï
ï x 3
x 3 ï
ï
þ
x =  3  és un punt de discontinuïtat de salt infinit. 
 
98. Indica els punts de discontinuïtat d’aquestes funcions:
���
a) ���� � √� � � d) ���� � �
���

� √���
b) ���� � √� � �� e) ���� �
���

� ��� ��
c) ���� � f) ���� �
√��� ��� ��

a) Dom f = (-¥, 4] La funció és contínua en (-¥, 4 ] .

b) Dom f =  La funció és contínua en tot  .

Dom f = (-1, + ¥)
La funció és contínua en (
-1, + ¥)
c) .

Dom f = (-¥, - 1) È [1, + ¥) -¥, - 1) È [1, + ¥)


d) La funció és contínua en ( .

Dom f = [1, + ¥)
La funció és contínua en [
1, + ¥)
e) .

Dom f = (-¥, - 1) È (1, + ¥)


La funció és contínua en (
-¥, - 1) È (1, + ¥)
f) .
 
99. Determina els punts de discontinuïtat de les funcions següents:
� ���
a) ���� � e) ���� �
��� �� ������
���
b) ���� � f) ���� � √� � �
�� �����

c) ���� � √� � � g) ���� � √�� � �� � �


d) ���� � √� � �� � �� h) ���� � √�� � �� � �
a) Dom f = � - {-3}

1
lim � ��
�→��� ���
� → 
1
lim � � ��
�→�� � � �

→ No existeix lim ���� , i x = 3 és un punt de discontinuïtat inevitable de salt infinit.


�→��

b) Dom f = � → No hi ha punts de discontinuïtat.

c) Dom f = ������→ No hi ha punts de discontinuïtat.

d) Dom f = ���1�→ No hi ha punts de discontinuïtat.

e) Dom f = � -�����→ No hi ha punts de discontinuïtat.

605
Límit d’una
Límit d’una funció
funció 10

��� �
lim →
�→� � � ������ �

���
lim� � ��
�→� � �
� �� � ��
� →
���
lim� � � ��
�→� � � �� � ��

→ No existeix lim ����, i x = 3 és un punt de discontinuïtat inevitable de salt infinit.


�→�

f) Dom f = ������→ No hi ha punts de discontinuïtat.

g) Dom f = (-∞, -2] ∪ [4, +∞) → No hi ha punts de discontinuïtat al seu domini.

g) Dom f = � → No hi ha punts de discontinuïtat.

 
100. Estudia la continuïtat de la funció que assigna a cada nombre la seva part entera.
f(x) = [x]
Especifica els tipus de discontinuïtats que presenta aquesta funció.

 
Per a cada x Î  , la funció té una discontinuïtat de salt finit. Per exemple, si x = 1 → f(1) = 1.

lim f ( x) = 0üïï
x1- ïý  No existeix lim f ( x).
 
lim+ f ( x) = 1 ïï x1
x1 ïþ

 
101. Estudia la continuïtat de les funcions següents en el punt x = 2 i, si són discontínues, indica el tipus
de discontinuïtat que tenen:
�� � � �� � � ��
a) ���� � �����
�� � � ��
���

b) ���� � �� � �� � � �� � � ��
� �� � � ��
� � �� �� � � ��
c) ���� � �� �� � � ��
�� � �� � � �� � � ��
a) f(2)  1 
lim f ( x ) = 1ïü
x -2- ïï
ý  xlim f ( x ) = 1 
lim+ f ( x ) = 1ïï -2
x -2 ïþ

La funció és contínua en x = 2.


606
Límit d’una funció 1010

b) f(2)  2 
lim x 2 - 5 x + 2 = 16  
x -2

A x  2, la funció té una discontinuïtat evitable. 

c) f(2)  3 

lim f ( x ) = 7üï
ï
x -2- ï
ý  xlim f ( x) = 7  
lim+ f ( x ) = 7ï
ï -2
x -2 ï
þ

A x  2, la funció té una discontinuïtat evitable. 

102. Defineix d’una altra manera les funcions següents perquè siguin contínues en tots els nombres
reals:

࢙࢏࢞ ൏ ૚

a) ࢌሺ࢞ሻ ൌ ቐ૚࢙࢏࢞ ൌ ૚
૜࢞ െ ૚࢙࢏࢞ ൐ ૚
࢞૛ ି૚૟
b) ࢌሺ࢞ሻ ൌ ቊ ࢙࢏࢞ ് ૝
࢞ି૝
૟࢙࢏࢞ ൌ ૝
a) f(x) no es pot redefinir perquè sigui contínua a tots els nombres reals, atès que a x = 0 té una
discontinuïtat de salt infinit.

x 2 - 16 0 x 2 - 16
b) lim  lim = lim x + 4 = 8
x 4 x -4 0 x 4 x -4 x 4

Perquè f(x) sigui contínua a tots els nombres reals, redefinim la funció a f(4)  8.
 
 
103. Calcula el valor que ha de tenir a perquè les funcions següents siguin contínues:
૜࢞ െ ૝࢙࢏࢞ ൏ ૚
a) ࢌሺ࢞ሻ ൌ ቐ ࢇ࢙࢏࢞ ൌ ૚
࢞ି૜
࢙࢏࢞ ൐ ૚

࢞૛ ି૝
b) ࢌሺ࢞ሻ ൌ ቊ ࢞ା૛ ࢙࢏࢞ ് െ૛
ࢇ࢙࢏࢞ ൌ െ૛
a)  f(1)  a 
lim f ( x ) = -1ïü
x 1- ï
ï
ý  limf ( x ) = -1
lim+ f ( x ) = -1ï
ï x 1
x 1 ï
þ

La funció és contínua si f (1)  = lim


x 1
f ( x )  a = -1 .

0 x2 - 4
b)  xlim f (x)       lim f ( x ) = lim = lim x - 2 = -4
-2 0 x -2 x -2 x +2 x -2
 
La funció és contínua si f (-2) = xlim
-2
f ( x )  a = -4 .

 
104. Determina el valor de a perquè aquestes funcions siguin contínues:

607
Límit d’una
Límit d’una funció
funció 10


a) ���� � ��� �� � � �
�� � � �� � � �
� �
b) ���� � �� �� � � � � � �� � � ��
�� � � � � �� � � ��
a)  f(2)  12 

lim f ( x) = 12 üï
ï
ýï  El límit existeix si 6 + a = 12  a = 6.
x2-

lim f ( x) = 6 + aïï
x2+ ïþ

b)  f(1)  3   

lim f ( x) = -3 üï
ï
ýï  El límit existeix si 2a -1 = -3  a = -1.  
x-1-

lim f ( x) = 2a -1ïï
x-1+ ïþ
 
105. Quin valor ha de tenir a perquè les funcions siguin contínues?
���
a) ���� � � � �� � � ��
�� � � �� � � ��
��
�� �� � � ��
b) ���� � � ��
��� ��� � �� �� � � ��

a) ���2� � 2�� �

1 ��1
lim � ���� � lim � 2 � 1 17
� �→�� �→�� 8 → ∃ lim ���� �� � �2� � 2 → � � �
lim � ���� � lim � ��� � 2� � �2� � 2 �→�� 8 16
�→�� �→��

b) ���2� � 1
��
lim � ���� � lim � ��
�1 3
� �→�� �→�� 2� → ∃ lim ���� �� 1 � �����2� � 7� → � � �
lim � ���� � lim � ��� ��� � 7� � �����2� � 7� �→�� 2
�→�� �→��

106. Troba els valors de a i b perquè les funcions següents siguin contínues en tots els nombres reals:
|� � �| �� � � �

a) ���� � � �� � � � � �

� �� � � �
�� � ��� � � �� � � �
b) ���� � ��� � � �� � � � � �
����
�� � � �

a)  f(2)  0 

lim f ( x ) = 0 üïï
x2- ïï 2
2 ýï  El límit existeix si b = 0.
lim f ( x ) = ïï
x2+ b ïþ
No es pot resoldre per a cap valor de b, així
  doncs la funció no és contínua per a cap valor de b,
independentment del valor de a.

608
Límit d’una funció 1010

b)  f(1)  a  5 

lim f ( x) = 2b + 5ü ï
ï
x1- ï
ý  El límit existeix si 2b - a = 0.  
lim+ f ( x ) = a + 5 ï
ï
x1 ï
þ
f(2)  2a  5 

lim f ( x ) = 2a + 5üïï
x2- ï 5
5 ýï  El límit existeix si 2a + 5 = .  
lim f ( x ) = ïï 2
x2+ 2 ïïþ

5 5
La funció és contínua si a = - y b=- .
4 8

107. Calcula el valor de a perquè aquestes funcions siguin contínues:


࢞ െ ૛࢙࢏࢞ ൑ ࢇ
a) ࢌሺ࢞ሻ ൌ ൜
࢒࢕ࢍ૛ ࢙࢞࢏࢞ ൐ ࢇ
࢞૜ ൅ ࢞૛ െ ࢞ െ ૛࢙࢏࢞ ൏ െࢇ
b) ࢌሺ࢞ሻ ൌ ൝࢞૛ െ ૛࢙࢏ࢇ ൑ ࢞ ൑ ࢇ
࢞૜ ൅ ࢞૛ െ ࢞ െ ૛࢙࢏࢞ ൐ ࢇ
lim f ( x) = a - 2 ï ü
xa- ï
ï
a)  f(a)  a  2        ý  El límit existeix si a - 2 = log 2 a  a = 4.
lim+ f ( x ) = log 2 aï
ï
xa ï
þ

lim f ( x ) = -a 3 + a 2 + a - 2ü
ï  
x-a- ï
ï  El límit existeix si - a 3 + a = 0  a = {-1, 0, 1}
b)  f(a)  a   2   
2
  ý
ï  
lim f ( x ) = a 2 - 2 ï
x-a + ï
þ

lim f ( x) = a3 + a 2 - a - 2ü
ï
ï
x-a- ï  El límit existeix si - a3 - a = 0  a = 0
  f(a)  a2  2        ý  
lim+ f ( x) = a - 2
2 ï
ï
x-a ï
þ
Per tant, la funció és contínua quan a = 0.
 
108. Calcula m i n perquè la funció f(x) sigui contínua en R.
૛࢙࢏࢞ ൑ ૚
ࢌሺ࢞ሻ ൌ ൝࢓࢞ ൅ ࢔࢙࢏૚ ൏ ࢞ ൑ ૜
૝࢙࢏࢞ ൐ ૜
lim f ( x ) = 2 ü
ï
x1- ï
ïý  El límit existeix si m + n = 2.
f (1) = 2   
lim+ f ( x ) = m + nïï
x1 ï
þ

lim f ( x) = 3m + nü ïï
ýï  El límit existeix si 3m + n = 4.  
x3-
f (3) = 3m + n 
lim f ( x) = 4 ïï
x3+ ïþ

m + n = 2ïüï
ý  m = 1, n = 1  
3 m + n = 4ïïþ

609
Límit d’una
Límit d’una funció
funció 10

109. Estudia la continuïtat en tot el domini de les funcions. Determina els punts de discontinuïtat que
presenta cadascuna d’aquestes funcions.

a) ���� � ����� � �� c) ���� � �� �� � �

b) ���� � ���� � �� d) ���� � � ���

a) Dom f = � → No hi ha punts de discontinuïtat.

b) Dom f = ���� ��� → No hi ha punts de discontinuïtat.

c) Dom f = � - {π + k π� � � � �


lim �� �� � � � ��
�→�� �
� �
→ No existeix lim ����, la funció no és contínua a x = �.
�→�
lim��� �� � � � ��
�→� �

Es tracta d’un punt de discontinuïtat inevitable de salt infinit.

Com que es tracta d’una funció periòdica, de període �, tots els punts on falla el domini són punts de
discontinuïtat inevitable de salt infinit.

d) Dom f = � → No hi ha punts de discontinuïtat.

 
110. Investiga la continuïtat de les funcions següents:
����
�� � � � �� � � �� � � �
�� ����
a) ���� � ����� c) ���� � �
�� � � � √� � � �� � � �
���

����� � �� �� � � � ��� � �� � � �
b) ���� � � c) ���� � � � ��
� �� � � � ����� � �� �� � � �
a) ▪  Si x  1: 
5x + 1
f (x) = Dom f = (-¥, 1) - {-2}
x 2 - x - 6       

lim f ( x ) = -¥ ü
ï
ï
9 x-2- ï
ý  No existeix xlim f ( x ).  
f (-2)  -
0         lim+ f ( x ) = +¥ï
ï -2
x-2 ï
þ
x = –2 és un punt de discontinuïtat de salt infinit.

▪  Si x  1: 
lim f ( x ) = -1ïü
x 1- ï
ï
f (1) = -1         ý  lim f ( x ) = -1  
lim+ f ( x ) = -1ï
ï x 1
x 1 ï
þ
 
Com que f (1) = lim f ( x), la funció és contínua a x = 1.  
x1

▪  Si x  1: 
3x +1
f (x) = Dom f = (1, + ¥) - {5}  
x - 5       

610
Límit d’una funció 1010

lim f ( x ) = -¥ ü
ïï
x5- ïý  No existeix lim f ( x ).
lim+ f ( x ) = +¥ï ïï x5
x5 þ
x = 5 és un punt de discontinuïtat de salt infinit.
 
La funció és contínua a  - {- 2, 5} .

b) ▪ Si x  5: 

f ( x ) = log ( x - 1) Dom f = (1, 5)  No hi ha punts de discontinuïtat.  


    

▪  Si x  5: 

lim f ( x) = log 4üïï


x5- ïý  No existeix lim f ( x).
f (5) = 3            
lim+ f ( x) = 3 ïï x5
x5 ïþ

La funció no és contínua a x = 5.

▪ Si x  5: 

f ( x) = 3           Dom f = (5, + ¥ )  No hi ha punts de discontinuïtat.  

La funció és contínua a (1, +¥) - {5} .

c) ▪  Si x  0: 

f ( x) = 3 x + 1 Dom f = (-¥, 0)  No hi ha punts de discontinuïtat.  


      

▪ Si x  0: 

lim f ( x ) = 1ïü
x  0- ï
ï
f (0 ) = 1           ý  lim f ( x) = 1  
lim+ f ( x ) = 1ï
ï x 0
x 0 ï
þ

Com que f (0) = lim f ( x), la funció és contínua a x = 0.  


x0

▪  Si x  0: 

f ( x ) = 1- x Dom f = (0, 1]  No hi ha punts de discontinuïtat.  


      

La funció és contínua a (-¥, 1] .

d) ▪  Si x < 0: 

æ 1 ö÷
f ( x ) = log ççç 2 ÷ Dom f = (-¥, 0)  No hi ha discontinuïtats.  
è x + 1÷ø     

611
Límit d’una
Límit d’una funció
funció 10

▪  Si x  0: 

æ 1 ÷ö ü
ï
lim f ( x ) = lim- log çç 2 ï
= 0ï
x  0- x 0 çè x + 1÷÷ø ï  → f és contínua a x  0 
ý
ï
lim+ f ( x ) = lim+ log ( x + 1) = 0 ï ï
x 0 x0 ï
þ

▪  Si x  0: 

f ( x ) = log ( x + 1) Dom f = (0, + ¥ )  No hi ha discontinuïtats.  


     

La funció és contínua a tot  .

 
111. Un atleta ha de recórrer una distància de 100 km. Si el primer dia en fa la meitat; el segon dia, la
meitat del que li falta; el tercer dia, la meitat del que li ha quedat per recórrer el segon dia, i així
successivament, quants dies trigarà l’atleta per recórrer els 100 km? Justifica la resposta.
Mai recorrerà els 100 km, perquè necessitaria un nombre «infinit» de dies per aconseguir-ho.

100 1
El primer dia ha recorregut a1 = = 100 ⋅ = 50 km .
2 2
100 300 3
El segon dia ha recorregut a2 = + 50 = 2 = 100 ⋅ 2 = 75 km .
22 2 2
100 700 7
El tercer dia ha recorregut a3 = + 75 = 3 = 100 ⋅ 3 = 87,5 km .
23 2 2
n
æ 1ö
El terme general de la successió és an = 100 ⋅ 2 - 1
= 100 ⋅ çç1 - n ÷÷÷ , on és el dia enèsim.
ç
n
2 è 2 ø

æ ö
Quan n es fa molt gran la successió tendeix a 100 → lim an = lim 100 ⋅ ççç1- 1n ÷÷÷ = 100
n ¥ n ¥ è 2 ø

112. Calcula ��� �� � �� per a aquestes funcions:


�→�

�� � �
���� � � � � ���� �
��� � ���
�� � 2�� � � � � � 4� � �
� � ���� � ������� � ��� � 2� � �
2�� � 2�� � �0�� � 2� 2� � � 2� � �2
� � � 4� � � 24
lim � � ���� � lim →
�→� �→� 2� � � 2� � �2 0
� � �����
lim� � ��
�� � ������
��→� �� ����� → No existeix el límit.
lim� � ��
�→� �� � ������

 
113. Construeix la gràfica aproximada d’una funció que compleixi les condicions següents:
����
a) ��� ���� � � b) ��� ��
�→�� �→�� �

��� ���� � �� ��� ����� � ��� � �


�→�� �→��

��� ���� � �� ��� ���� � �


�→�� �→��

612
10

613
Límit d’una
Límit d’una funció
funció 10

117. Estudia la continuïtat segons els valors dels paràmetres a i b en aquesta funció:
���
��������������������������� � �
���� � �� � ���
�������������������������������������� � �

lim f ( x ) = 0 ïüï
x  0- ïï
f(0)  b          a ý a= b= 0  
lim f ( x ) = = aïïï
x  0+ 1 ïþ

 
118. El teorema de Bolzano diu:
Si f és una funció contínua en l’interval [a, b] i es verifica que el signe de f(a) és diferent del signe de
f(b), aleshores existeix un punt c ∈ (a, b) tal que f(c) = 0.
Aplica el teorema de Bolzano per demostrar que l’equació x2 + x – 4 = 0 té almenys una solució en
l’interval (1, 2).
12 + 1- 4 = -2 < 0üïï
ý → 
22 + 2 - 4 = 2 > 0 ïïþ

Segons el teorema de Bolzano, existeix com a mínim un x a l’interval (1, 2) tal que x2 + x - 4 = 0. És a dir, hi
ha com a mínim una solució en aquest interval.

119. Aplica el teorema de Bolzano per demostrar que l’equació e-x + 2 = x té alguna solució.
Si definim les funcions f(x) i g(x) com f ( x ) = e- x + 2 i g(x) = x, es tallen en un punt x Î (2, 3) , aleshores

apliquem el teorema de Bolzano a e- x + 2 - x = 0 a l’interval (2, 3).

1 üï
e-2 + 2 - 2 = > 0 ïï
e2 ïï
ý
1 ï
e-3 + 2 - 3 = 3 - 1 < 0ïïï
e ïþ

Pel teorema de Bolzano, hi ha com a mínim un x en l’interval (2, 3) de manera que e-x + 2 = x. És a dir,
existeix com a mínim una solució en aquest interval.

120. Calcula el valor de a perquè el límit tingui valor finit.


��� � �
��� � ��
���� � � �

Amb aquest valor de a, calcula b perquè es verifiqui que:


��� � �
��� � �� � � � �
���� � � �

��� ��
Quina relació hi ha entre la funció ���� � i la recta y = ax + b?
���

2x2 + 3 2 x 2 + 3 - ax 2 + ax (2 - a) x 2 + ax + 3
lim - ax = lim = lim  
x +¥ x -1 x +¥ x -1 x +¥ x -1

614
Límit d’una funció 1010

Perquè tingui límit finit, el grau del numerador i del denominador ha de ser el mateix, així:

2x + 3
2 - a = 0  a = 2  lim =2 
x +¥ x -1

2x2 + 3 2x + 3 2 x + 3 - bx + b (2 - b ) x + (3 + b )
lim - ax - b = lim - b = lim = lim  
x +¥ x -1 x +¥ x - 1 x +¥ x -1 x +¥ x -1

Perquè aquest límit sigui 0, el grau del numerador ha de ser més petit que el del denominador, així:

5
2 - b = 0  b = 2  lim =0
x +¥ x -1
 
2x2 + 3
La relació que hi ha entre ambdues funcions és que y = ax + b és una asímptota obliqua de y = .
x -1

121. La fórmula següent, plantejada per Albert Einstein, expressa la massa, M, d’un cos en relació amb la
seva velocitat, v, en què c és la velocitat de la llum (300 000 km/s).
��
��
√�� � ��
a) Calcula el límit de la massa, M, quan v tendeix a c.
b) Analitza si un cos pot arribar a aconseguir la velocitat de la llum.
��
lim � ��
��� √� � � � �

Perquè la velocitat arribés a ser la de la llum, el cos hauria de tenir una massa infinita.

122. En el servei de traumatologia d’un hospital volen implantar un nou sistema destinat a reduir a curt
termini les llistes d’espera.

615
Límit d’una
Límit d’una funció
funció 10

Preveuen que a partir d’ara la funció següent, P(t), indicarà en cada moment t, en mesos, el
percentatge de pacients que podran operar sense necessitat d’entrar en llista d’espera.

࢚૛ െ ૡ࢚ ൅ ૞૙࢙࢏૙ ൑ ࢚ ൑ ૚૙
ࡼሺ࢚ሻ ൌ ቐ ૜ૡ࢚ െ ૚૙૙
࢙࢏࢚ ൐ ૚૙
૙ǡ ૝࢚

Al cap de molt temps, quin serà aquest percentatge?

38t - 100 38
lim = = 95  Serà atès el 95%.  
t +¥ 0, 4t 0, 4
 
123. Un comercial d’un producte determinat rep, com a sou mensual, una quantitat fixa de 600 € més una
comissió que depèn de l’expressió x2 – x + 1, en què x representa el nombre d’articles que ven.
El comercial s’ha de pagar les despeses pròpies, que són de 50 € més 3 € per cada producte que
ven. Troba la funció que determina el sou mensual del venedor. És una funció contínua?
f ( x ) = 600 + ( x 2 – x + 1) - (50 + 3 x ) = x 2 - 4 x + 551  

No, és una funció discreta definida per a x > 0 i amb x Î  .

   

616
Límit d’una funció 1010

AMPLIA 
 
124. Escull la resposta adequada.

□ La successió és de la forma: an = 2n x - 2n + 1 = 2n ( x - 1) + 1 , on x representa el nombre inicial. Aïllant:

299 ( x - 1) + 1 = 2100 + 1 tenim que x = 3.

□ Es necessiten 4 funcions contínues.

□ Atès que el nombre 1 ocupa una posició, el 2 ocupa dues posicions, etc., la successió cercada ha de
tenir els termes següents:

a1 = 1, a2 = 1 + 2 = 3, a3 = 1 + 2 + 3 = 6...

n ( n + 1)
El terme general és an = . Ara busquem el nombre n que compleixi:
2

n ( n + 1)
2 007 =  n = 62
2

Com que Im ( n2 + 20 n + 20) = (0, + ¥) e Im ( n2 - 20 n + 20) = (0, 20) , aleshores busquem entre els n < 20.

□ Primer ens fixem que per qualsevol n parell f(n) és parell, així que mirem únicament els n imparells. Els
únics primers que surten són f(1) = 41 i f(19) = 761, i si els sumem tenim 802.

□ Definim la successió an = 11n , observem alguns valors:

n = 1 111 = 11 1 n = 2  112 = 121 2 n = 3  113 = 1331  3 n = 4  114 = 14 641  4

···
n = 48  1148  8

125. Calcula els límits següents:


૚ ૚ ૚
a) ‫࢔࢏࢙ ܕܑܔ‬ b) ‫ ܕܑܔ‬ቀ࢞ ൉ ࢙࢏࢔ ቁ c) ‫ ܕܑܔ‬ቀ ൉ ࢉ࢕࢙࢞ቁ
࢔՜૙ ࢞ ࢔՜૙ ࢞ ࢔՜ஶ ࢞

617
Límit d’una
Límit d’una funció
funció 10

Encara que no sapiguem el valor que té el sinus i el cosinus d’un angle quan l’angle tendeix a infinit, si
sabem que és una quantitat acotada, així, tant el sinus com el cosinus d’un angle tenen un valor comprès en
[–1, 1], i en multiplicar per zero una quantitat acotada el resultat és zero.
æ1ö 1
a) lim sin çç ÷÷÷  No existeix. b) lim x ⋅ sin =0 c) xlim
1
⋅ cos x = 0
x0 çè x ø x0 x ¥ x

126. Estudia la continuïtat d’aquestes funcions:


� �
a) ���� � |�| b) ���� � |�� � �| c) ���� �
��� ���

�1 �� � � �
a) ���� � �
1 �� � � �
No existeix f(9).
lim� ���� � �1
��→� → No existeix lim ����, i la funció no és contínua a x = 0.
lim� ���� � 1 �→�
�→�

Es tracta d’un punt de discontinuïtat inevitable de salt finit.


La funció és contínua en � - {0}.

b) ���� � �
� � � 1 �� � � �1 � �� � � 1
�� � � 1 �� � 1 � � � 1
Si x = –1; f(–1) = 0
lim ���� � lim �� � � 1� � �
�→��� �→��
� → lim ���� � �
lim� ���� � lim� ��� � � 1� � � �→��
�→�� �→��

Com que ���� � lim ����, la funció és contínua a x = –1.


�→��

Si x = 1; f(1) = 0
lim ���� � lim��� � � 1� � �
�→�� �→�
� → lim ���� � �
lim� ���� � lim� �� � � 1� � � �→�
�→� �→�

Com que ���� � lim����, la funció és contínua a x = 1.


�→�

La funció és contínua a �.

� �� � � �1 � �� � � 1
c) ���� � �� ��

�� � 1 � � � 1
�� � ��

No existeix f(–1).
1
lim ���� � lim � ��
�→��� �→�� �2 �1
� 1
→ La funció no és contínua a x = –1.
lim� ���� � lim� � ��
�→�� �→�� ��2 �1

És un punt de discontinuïtat inevitable de salt infinit.


No existeix f(1).
1
lim ���� � lim � ��
�→�� �→� ��2 �1
� 1
→ La funció no és contínua a x = 1.
lim� ���� � lim� � ��
�→� �→� �2 �1

És un punt de discontinuïtat inevitable de salt infinit.


La funció és contínua a � – � –1� 1�.

618
Límit d’una funció 1010

127. Si mesurem l’angle x en radians, demostra que

��� �
��� ��
�→� �

Demostra també que si l’angle x es mesura en graus sexagesimals, aleshores:


��� � �
��� �
�→� � ���
 
Com que la mesura de la longitud de l’arc és un valor entre la longitud dels segments AC i AB, aleshores
l’àrea del sector circular és un valor entre l’àrea dels triangles.

Àrea OAC < Àrea de sector< Àrea OAB

� � � ��� � � � � �� �
� ��� �  
2 2

�� ��� � � � �� �� �
� �� �  
2 2 2

��
Simplifiquem dividint entre       : sin x <  x < tg x 

��� � � �� �
Dividim entre sin x:  � �  
��� � ��� � ��� �

��� � ��� � ��� � ��� �


→     � �     → 1� � ��� � 
��� � � �� � �

Fem límits amb x → 0:

�i� � �i� � �i� �


lim 1 � lim

� lim ��� � → 1 � lim �
� 1 → lim �
� 1 , que és el que volíem demostrar.
�→� �→� �→� �→� �→�

Si x es mesura en graus:

� � � ��� � � � � � �� � �� ��� � ��� �� �� �


� ��� � � → � �� �  
2 360 2 2 360 2
�� �
Simplifiquem dividint entre   � �i� � � � � � �� � 
� ���

Dividim entre sin x:

��� � � � �� � � � 1
� � � →1� � �  
�i� � 180 �i� � �i� � 180 �i� � ��� �
180 ��� �
                  →1� � � ��� � 
� �

619
Límit d’una
Límit d’una funció
funció 10

Fem límits amb x → 0:

180 sin � 180 sin �


lim 1 � lim � � lim cos � → 1 � � lim � 1 
�→� �→� � � �→� � �→� �
180 ��� �
                  → � lim � 1 
� �→0 �

sin � �
I aïllant, tenim que:  lim �
� ���  
�→�

 

128. Comprova que ��� �� no existeix.
�→�
1 ü
ï
lim- e x = 0ï
ï 1
x0 ï
ï
ý  No existeix lim e x

1 ï
ï x0
lim e x = +¥ï
ï
x0+ ï
þ
 
129. Calcula el límit següent:
� � � �
��� ��� � ��� � � ��� � � � � ��� �
�→� � � � �

Partim de la fórmula de sin 2x i l’apliquem successivament, obtenim:


� � � � �
sin 2� � 2 sin � cos � � 2� cos � sin ��� � 2� cos � ��� ��� � cos � �
2 2 2 2 2
� � � � �
� � � 2��� cos � ��� ��� � cos � � � � cos � sin �
2 2 2 2 2
� � � � sin 2� sin �
��� ��� � cos � � � � cos � � � � �
2 2 2 2 2��� cos � sin 2� sin
�� ��

��� � ��� � ��� � ��� �


Com que lim � 1, tenim que: lim � � lim � �
�→� � �→� �� ����� �→� �� �� �

x x x x sin x
Per tant,  xlim cos ⋅ cos ⋅ cos 3 ⋅ ...cos n = lim = 1 . 
¥ 2 22 2 2 x¥ x

 
130. Justifica si es pot verificar la continuïtat en el punt x = 0 d’una funció real f(x) de variable real en
aquests casos:
� �
a) Per a n natural, � � � � � i � � � � ��
�� ����

b) Per a x real no negatiu, ���� � �� , i per a x real negatiu, ���� � �.



c) Per a n natural, � � � � �

Perquè una funció sigui contínua en x = 0, s’ha de verificar que, per a qualsevol successió {�� �,

on lim � 0, es verifiqui que lim ���� � � ��0�.


�→� �→�

620
Límit d’una funció 1010

a) En aquest cas es verifica que:


� �
lim � � lim ��
�→� �� �→� ����

� �
No obstant això, la funció f(x) és igual a 1 per a � � i és igual a -1 quan � � .
�� ����

� �
lim � � � � � lim � � � � ��
�→� �� �→� ����

No existeix el límit i la funció no és contínua a x = 0.

b) La funció és:
� �� � � �
���� � �
� � �� � � �

lim� ���� � �
��→� → lim ���� � � � ���� La funció és contínua a x = 0.
lim� � � � � �→�
�→�

c) Perquè f(x) sigui contínua a x = 0 s’ha de verificar que per a qualsevol successió [�� �, amb
lim �� � �, es verifiqui que lim ���� � � ����. Com que únicament es coneix el valor de f(x) per una
�→� �→�

successió de valors de x, no es pot afirmar que la funció sigui contínua.

 
 
 

   

621
Límit d’una funció

622
Límit d’una funció 1010

9. Construeix la gràfica de la funció segle considerant que x adopta valors reals.

 
Siglo 

Año
 

10. Analitza la continuïtat de la funció segle en x = 2 000; per fer-ho, utilitza els límits laterals.

f (2 000 ) = 20 lim f ( x ) = 21 lim f ( x ) = 20  No existeix lim f ( x ). → La funció no és contínua a


x 2 000+ x 2 000- x 2 000

x = 2 000.

623
Derivada d’una funció
Derivada d’una funció  11
11 

ACTIVITATS 
 
1. Calcula la taxa de variació mitjana de la funció f (x) = x2 – x + 3 en els intervals següents:
[2, 3], [2, 4], [2, 5], [2, 6], [3, 5], [3, 6]
f (3 ) - f (2) 9 - 5 f (6) - f (2) 33 - 5
T .V .M .([2, 3 ]) = = =4 T .V .M .([ 2, 6 ]) = = =7  
3-2 1                6-2 4

f (4) - f (2) 15 - 5 f (5) - f (3) 23 - 9


T .V .M .([2, 4 ]) = = =5 T .V .M .([ 3, 5 ]) = = =7  
4-2 2               5-3 2

f (5) - f (2) 23 - 5 f (6) - f (3) 33 - 9


T .V .M .([2, 5 ]) = = =6 T .V .M .([ 3, 6 ]) = = =8 
5-2 3               6-3 3

2. Determina la T.V.M. de la funció f (x) = x2 – x + 3 en els intervals següents:


a) [2, 2 + h] b) [3, 3 + h]
a)  f (2 + h ) - f (2) (2 + h ) 2 - (2 + h ) + 3 - (4 - 2 + 3 ) h2 + 3 h
T .V .M . ([ 2, 2 + h ]) = = = = h+3 
2+ h-2 h h

b)  f (3 + h ) - f (3) (3 + h ) 2 - (3 + h ) + 3 - (9 - 3 + 3 ) h2 + 5h  
T .V .M .([ 3, 3 + h ]) = = = = h+5
3+ h-3 h h

3. Mitjançant la definició, calcula la derivada en x = 2 i en x = –1 d’aquestes funcions:


૚ ૚
a) ࢌሺ࢞ሻ ൌ b) ࢌሺ࢞ሻ ൌ ૛࢞૛ ൅ ࢞ c) ࢌሺ࢞ሻ ൌ
࢞ି૜ ࢞૛

1 1
-
f (2 + h) - f (2) 2 + h - 3 2 - 3 = lim 1 = -1
a)  f ´(2) = hlim = lim
0 2 + h-2 h  0 h h  0 -1 + h
 
1 1
-
f (-1 + h) - f (-1) - 1 + h - 3 - 1 - 3 = lim 4 - 4 + h = - 1
f ´(-1) = lim = lim
h 0 -1 + h - (-1) h 0 h h  0 4 (- 4 + h) h 16
 
2 2
b)  f (2 + h ) - f (2) 2(2 + h ) + (2 + h ) - (2 ⋅ 2 + 2)
f ´(2) = lim = lim =
h0 2+ h-2 h0 h
 
2 2
2(4 + h + 4 h ) + 2 + h - 10 2h + 9h
= lim = lim = lim (2 h + 9) = 9
h0 h h0 h h0
 
f (-1+ h) - f (-1) 2(-1+ h)2 + (-1+ h) - éëê 2 ⋅ (-1)2 + (-1)ùûú
f ´(-1) = lim = lim =
h  0 -1 + h - (-1) h  
h 0

2(1 + h 2 - 2 h ) - 1 + h - 1 2h2 - 3 h
= lim = lim = lim (2 h - 3) = - 3
h0 h h0 h h0
 
1 1
-
f (2 + h) - f (2) (2 + h)2 22 4 - (2 + h)2
c)  f ´(2) = hlim = lim = lim
h  0 4 h(2 + h)2
=
0 2+ h-2 h 0 h
 
4 - (4 + h 2 + 4 h ) -h2 - 4 h -h - 4 1
= lim = lim = lim =-
h  0 4 h (4 + h 2 + 4 h ) h  0 16 h + 4 h 3 + 16 h 2 h  0 16 + 4 h 2 + 16 h 4  
1 1
-
f (-1+ h) - f (-1) (-1+ h)2 (-1)2 1- (1- 2h + h2 ) 2h - h2 2- h
f ´(-1) = lim = lim = lim 2
= lim = lim =2 
h  0 -1 + h - (-1) h 0 h h  0 h(-1+ h) h  0 h(1- 2h + h2 ) h  0 1- 2h + h2

4. Calcula la derivada de la funció f(x) = x3 + 4 en els punts següents:

625
Derivada d’una
Derivada d’una funció
funció 11

a) x = 1 b) x = –4 c) x = 2 d) x = –3
 
(1 + h ) 3 + 4 - (13 + 4) 3 h2 + 3 h + h3
a)  f ´(1) = lim = lim = lim (3 h + 3 + h 2 ) = 3  
h0 h h  0 h h0

(-4 + h)3 + 4 - éêë(-4)3 + 4ùúû (h3 - 12h2 + 48h - 64) + 4 - (-64 + 4)


b)  f ´(-4) = hlim = lim =
0 h h 0 h
 
= lim(h2 -12h + 48) = 48  
h0

(2 + h )3 + 4 - (2 3 + 4) 8 + h 3 + 6 h 2 + 12 h + 4 - 12
c)  f ´(2) = lim = lim = lim ( h 2 + 6 h + 12) = 12  
h0 h h0 h h0

(-3 + h)3 + 4 - éëê(-3)3 + 4ùûú -27 + h3 - 9h2 + 27h + 4 - (-27 + 4)


d)  f ´(-3) = hlim = lim = lim ( h2 - 9h + 27) = 27  
0 h h 0 h h 0

 
5. Determina l’equació de la recta tangent a la funció f (x) = x – x2 en els punts d’abscissa x = 2 i x = –3.
éê2 + h - (2 + h)2 ùú - (2 - 22 ) 2 + h - (4 + 4h + h2 ) - 2 + 4 -3h - h2
f ´(2) = lim ë û = lim = lim = -3
h 0 h h 0 h h 0 h
 
f(2)  2  22  2 
L’equació de la recta tangent al punt P(2, -2) és:
y  (2)  f´(2)  (x  2) → y  2  3(x  2) → y  3x  4 

(-3 + h) - (-3 + h)2 - éëê-3 - (-3)2 ùûú -3 + h - (9 + h2 - 6h) + 12 -h2 + 7h


f ´(-3) = lim = lim = lim =7  
h 0 h h 0 h h  0 h

f(3)  3  (3) 2  12 
L’equació de la recta tangent en el punt P(-3, -12) és:
y  ( 12)  f´(3)  (x  (3)) 
y  12  7(x  3) 
y  7x  9 
 
6. Calcula el pendent de la recta tangent a la gràfica de la funció f(x) = x2 + 4x + 3 en els punts on talla
els eixos X i Y.
Talls amb l’eix X: (-1, 0), (-3, 0)
La derivada f´(a) és el pendent de la recta tangent al punt P(a, f(a)).
(-1+ h)2 + 4(-1+ h) + 3 - êëé(-1)2 + 4 ⋅ (-1) + 3úûù 1+ h2 - 2h - 4 + 4h + 3 - 1+ 4 - 3 h2 + 2h
f ´(-1) = lim = lim = lim =2  
h 0 h h 0 h h 0 h

(-3 + h) + 4(-3 + h) + 3 - éêë(-3)2 + 4 ⋅ (-3) + 3ùúû


2
9 + h2 - 6h - 12 + 4h + 3 - 9 + 12 - 3 h2 - 2h
f ´(-3) = lim = lim = lim = -2  
h 0 h h 0 h h 0 h
Tall amb l’eix Y: (0, 3)
(0 + h)2 + 4(0 + h) + 3 - êëé(0)2 + 4 ⋅ (0) + 3ùúû h2 + 4h + 3 - 3
f ´(0) = lim = lim =4
h 0 h h 0 h
 
   

626
Derivada d’una funció 1111

7. Utilitza la definició per calcular la funció derivada d’aquestes funcions:



a) ���� � ��� b) ���� � √� c) ���� �
���

f ( x + h) - f ( x ) 4( x + h ) 3 - 4 x 3 4( x 3 + 3 x 2 h + 3 xh 2 + h 3 ) - 4 x 3 12 x 2 h + 12 xh 2 + 4 h 3
a) f ´( x ) = lim = lim = lim = lim = 12 x 2
h0 h h0 h h0 h h0 h

f ( x + h) - f ( x ) ( x + h - x )( x + h + x ) x + h- x 1 1
b) f ´( x ) = hlim = lim = lim = lim =
0 h h 0
h( x + h + x ) h 0
h( x + h + x ) h 0 x + h + x 2 x

1 1
-
f ( x + h) - f ( x ) x + h + 3 x + 3 = lim x + 3 - ( x + h + 3) = lim -h 1
c) f ´( x ) = hlim = lim =-
0 h h 0 h h 0 ( x + 3)( x + h + 3)h h  0 ( x + 3)( x + h + 3)h ( x + 3)2

 
8. Determina les derivades segona i tercera de les funcions següents:
a) ���� � �� � ��� b) ���� � �� � � � �
f ( x + h) - f ( x ) ( x + h ) 3 + 4( x + h ) 2 - ( x 3 + 4 x 2 )
a)  f ´( x ) = lim = lim =
h0 h h  0 h
 
x 3 + 3 x 2 h + 3 xh 2 + h 3 + 4( x 2 + h 2 + 2 xh ) - x 3 - 4 x 2 3 x 2 h + 3 xh 2 + h 3 + 4 h 2 + 8 xh
= lim = lim = 3x2 + 8x  
h0 h h  0 h

f ´( x + h ) - f ´( x ) 3( x + h ) 2 + 8( x + h ) - (3 x 2 + 8 x )
f ´´( x ) = lim = lim =  
h0 h h  0 h

3( x 2 + h 2 + 2 xh ) + 8 x + 8 h - 3 x 2 - 8 x 3 h 2 + 6 xh + 8 h
= lim = lim = 6x + 8  
h0 h h  0 h

f ´´( x + h ) - f ´´( x ) 6( x + h ) + 8 - (6 x + 8) 6h
f ´´´( x ) = lim = lim = lim =6 
h0 h h0 h h0 h

f ( x + h) - f ( x ) ( x + h ) 2 - ( x + h ) + 5 - ( x 2 - x + 5)
b)  f ´( x ) = lim = lim =
h0 h h0 h
 
x 2 + h 2 + 2 xh - x - h + 5 - x 2 + x - 5
= lim = lim ( h + 2 x - 1) = 2 x - 1  
h0 h h0

f ´( x + h ) - f ´( x ) 2( x + h ) - 1 - (2 x - 1) 2h
f ´´( x ) = lim = lim = lim =2 
h0 h h0 h h0 h

f ´´( x + h ) - f ´´( x ) 2-2


f ´´´( x ) = lim = lim =0  
h0 h h0 h

 
9. Calcula la derivada d’aquestes funcions:
� �
a) ���� � � b) ���� � �� c) ���� � √� � d) ���� �
��
3 3 1
4 3 3 -1 3 - 3
a) f ´( x ) = 0 c) f ( x) = x = x 4  f ´( x ) = x 4 = x 4
=
4 4 44 x

b) f ´( x ) = 4 ⋅ x 4-1 = 4 x 3 d) f ( x ) = 15 = x -5  f ´( x ) = - 5 x -5-1 = - 5 x -6 = - 5 16
x x

 
10. Determina la derivada d’aquestes funcions:
� � �
a) ���� � √� � b) ���� � � � c) ���� � √� � d) ���� �
��
4 4 3
4 -1 4 - 4
a) f ( x ) = 7 x 4 = x 7  f ´( x ) = x 7 = x 7
=
7 7 7 x37

b) f ´( x ) = 8 ⋅ x 8-1 = 8 x 7

627
Derivada d’una
Derivada d’una funció
funció 11

3 3 2
3 -1 3 - 3
c) f ( x ) = 5 x 3 = x 5  f ´( x ) = x 5 = x 5
=
5 5 55 x 2

d) f ( x ) = 14 = x -4  f ´( x ) = -4 ⋅ x -4-1 = -4 x -5 = -45
x x

11. Calcula la derivada d’aquestes funcions:


a) ���� � �� b) ���� � �� c) ���� � ��
a) f ´( x ) = 2 x ln 2 b) f ´( x ) = 3 x ln 3 c) f ´( x ) = 4 x ln 4

 
12. Determina la derivada d’aquestes funcions:
a) ���� � ��� � � b) ���� � ��� � � c) ���� � ��� � �
a) f ´( x ) =
1 b) f ´( x ) =
1 c) f ´( x ) =
1
x ln 2 x ln 3 x ln 4

 
13. Determina la derivada d’aquestes funcions:
� �
a) ���� � �� � �� � � � � c) ���� � √� � √��

b) ���� � � � � d) ���� � �� � ��������

2 1
1 - 3 - 1 3
a)  f ´( x ) = 3 x 2 + 2 x - 1                     c)  f ´( x ) = x 3 + x 4 = +  
3 4 3 x 3 2
44 x

b)  f ´( x ) = - 12 + 1                       d)  f ´( x ) = 3 x ln 3 - 1
 
x 1+ x 2

 
14. Calcula la derivada de les funcions següents:
� �
a) ���� � ��� � ��� � �� � � c) ���� � � √� � � √��

b) ���� � ������� � ������� d) ���� � � ���
��
2 1
1 - 3 - 7 9
a) f ´( x ) = 6 x 2 + 8 x - 8 c) f ´( x ) = 7 ⋅ x 3 - 3 ⋅ x 4
= -
3 4 3
3 x 2 4
4 x

b) f ´( x) = 2 cos x - 3(-sin x) = 2 cos x + 3 sin x d) f ´( x ) = - 1 2 - 5


2x

Calcula la derivada de les funcions següents:



a) ���� � � √�� e) ���� � ����� � �����
�� ��
b) ���� � � � �� f) ���� �

c) ���� � � � ����� g) ���� � ��� � ��� � �����

d) ���� � � � ���� h) ���� � ��� � �� � ����


1
3 2 2 - 23 x 2 5 3 2
a) f ´( x) = 1⋅ x + x ⋅ x 3
= 3 x2 + = x
3 3 3

b) f ´( x ) = 1⋅ e x + x ⋅ e x
c) f ´( x ) = 1⋅ sen x + x ⋅ cos x

628
Derivada d’una funció 1111

d) f ´( x ) = 1⋅ ln x + x ⋅ 1 = ln x + 1
x

e) f ´( x ) = cos x ⋅ cos x + sen x ⋅ (-sen x ) = cos 2 x - sen2 x

f) f ( x ) = ( x 2 + 1) ⋅ 1  f ´( x ) = 2 x ⋅ 1 + ( x 2 + 1) ⋅ -21 = 2 - 1- 12 = 1- 12
x x x x x

g) f ´( x ) = (2 x + 2) ⋅ sen x + ( x + 2 x ) ⋅ cos x2

ex - x
h) f ´( x ) = ( e x - 1) ⋅ ln x +
x

 
15. Determina la derivada d’aquestes funcions:
a) ���� � ��� � ���� � e) ���� � �� � ����� � �� �����

b) ���� � �� � ���� f) ���� � ���� � � �� � � �
��

c) ���� � √� � ����� g) ���� � ������ � ������ � ����
�����
d) ���� � ����� � ���� h) ���� � ��√� �

1 3x
a) f ´( x ) = 6 x ⋅ log 2 x + 3 x 2 ⋅ = 6 x log 2 x +
x ln 2 ln 2

b) f ´( x ) = e x ⋅ sin x + e x ⋅ cos x = e x ( sin x + cos x )


1
c) f ´( x ) = ⋅ cos x - 3
x ⋅ sin x
3
3 x2

2 1 -sin2 x 1
d) f ´(x) =-sin x ⋅ tg x + cos x ⋅ (1+tg x) =-sin x ⋅ tg x + cos x ⋅ 2
= + = cos x
cos x cos x cos x

e) f ´( x ) = 4 sin x + 4 x cos x + 3 x 2 cos x - x 3 sin x = (4 - x 3 ) sin x + (4 x + 3 x 2 ) cos x

f) f ´( x ) = 1 ⋅ 14 + ln x ⋅ - 45 + 2 x ⋅ e x + x 2 ⋅ e x = 15 (1 - 4 ln x ) + x e x (2 + x )
x x x x

g) f ´( x ) = (cos x + sin x ) tg x + ( sin x - cos x )(1 + tg 2 x )


1 cos x sin x cos x sin x
h) f ´( x ) = 4 ⋅ x + 4 x ⋅ + - 2 =6 x+ - 2
2 x x x x x

 
16. Determina la derivada de les funcions següents:
��� √� �� ����
a) ���� � b) ���� � c) ���� �
� ���� ���

a) f ´( x ) = 1 ⋅ x - ( x2 + 2) ⋅ 1 = - 22
x x

1
⋅ (3 x + 4) - x ⋅ 3
-3 x + 4
b) f ´( x ) = x
2 =
(3 x + 4)2 2 x ⋅ (3 x + 4)2

c) (2 x + 1)( x + 1) - ( x 2 + x - 3 ) ⋅ 1 x2 + 2x + 4
f ´( x ) = 2
=
( x + 1) ( x + 1) 2

   

629
Derivada d’una
Derivada d’una funció
funció 11

17. Calcula la derivada d’aquestes funcions:


����� √� ��������
a) ���� � b) ���� � c) ���� �
�� ����� ���

cos x ⋅ x 3 - sin x ⋅ 3 x 2 x cos x - 3 sin x


a) f ´( x ) = =
x6 x4

1
⋅ cos x - x ⋅ (-sin x)
cos x + 2 x sin x
b) f ´( x) = 2 x =
cos 2 x 2 x cos 2 x

( cos x + 2) ⋅ ( x - 3) - ( sin x + 2 x ) ⋅ 1 ( x - 3) ⋅ cos x - 6 - sin x


c) f ´( x ) = =
( x - 3) 2 ( x - 3) 2

 
18. Determina la derivada de les funcions que hi ha a continuació:

a) ���� � ��� � � b) ���� � ������������ c) ���� � �����


2
2x +1 3(arc sin x)
a) f ´( x) = b) f ´( x) = c) f ´( x ) = 1
2 x2 + x 1- x 2 x

 
19. Calcula la derivada d’aquestes funcions:
� � �����
a) ���� � �������� b) ���� � �� � � c) ���� � ��
���
-5 1
a) f ´( x) = -2 cos x sin x b) f ´( x ) = ⋅ c) f ´( x ) = e x
2
+7 x -4
⋅ (2 x + 7 )
æ x ö
÷÷ ( x - 5)
2
cos 2 ççç ÷
è x - 5ø

   

630
Derivada d’una funció 1111

SABER FER 
 
૜࢞ା࢓
20. Calcula el valor de m en la funció ࢌሺ࢞ሻ ൌ si saps que f '(–1) = 5.
࢓࢞૛
3 ⋅ mx 2 - (3 x + m ) ⋅ 2 mx
f ´( x ) =  
m2 x 4

3 m + 2 m ⋅ (- 3 + m) - 3 m + 2 m 2 3
f ´(- 1) = = = - + 2 = 5  m = -1
m2 m2 m
 
 
࢑࢞ ൅ ૛࢙࢏࢞ ൏ െ૚
21. Estudia la derivabilitat de ࢌሺ࢞ሻ ൌ ቄ en x = –1 segons els valors de k.
࢞૜ െ ࢞ ൅ ࢑࢙࢏࢞ ൒ െ૚
Una funció és derivable si també és contínua, de manera que primer analitzem si la funció és contínua en x
= -1:
ü
lim f ( x ) =-k + 2ï
x -1- ï
ï
ý → Perquè sigui contínua –k + 2 = k → k = 1.
lim+ f ( x ) = k ï
ï
x -1 ï
þ
f(x) únicament és contínua per a k = 1, per tant, pot ser derivable només per aquest valor. Analitzem la
derivabilitat per aquest valor:
ìï1 si x < 1
f ´( x ) = ïí 2
ïïî3 x - 1 si x ³ 1

lim f ´( x ) = 1ïüï
x -1- ïý → f(x) no és derivable per a cap valor de k.
lim+ f ´( x ) = 2ïï
x -1 ïþ


22. Determina l’equació de la recta tangent a la funció f(x) = sin2x – cos x en el punt d’abscissa x = .

æ pö 2p p æ 1ö 3 3
f ´çç ÷÷÷ = 2 cos + sin = 2 ⋅çç- ÷÷÷ + = -1+
çè 3 ø 3 3 ç
è 2ø 2 2

æ pö 2p p 3 -1
f çç ÷÷÷ = sin - cos =
çè 3 ø 3 3 2

æ ö
L’equació de la recta tangent en el punt P ççç p , 3 - 1÷÷÷÷ és:
çè 3 2 ÷ø

æ 3 - 1÷ö æ öæ ö æ ö
y - ççç ÷÷ = çç- 1 + 3 ÷÷÷çç x - p ÷÷ → y = ççç 3 - 1÷÷÷ x + p - 3p + 3 - 1
çè 2 ÷ø ççè 2 ÷è÷øç 3 ÷ø çè 2 ÷ø 3 6 2

 
23. Determina els punts de la funció f(x) = x3 – 3x amb la tangent horitzontal.
f ´( x ) = 3 x 2 - 3 = 0  x = 1  

x = 1  f (1) = -2  A(1, - 2)  

x = -1  f (-1) = 2  B(-1, 2)
 
 
 

631
Derivada d’una
Derivada d’una funció
funció 11

24. Determina quin ha de ser el valor de k en la funció f(x) = (k – 1)x3 + x2 – kx – 4 si les rectes tangents

en x = i en x = –1 són paral·leles.

æ 1ö 1 2
f ´( x ) = 3( k - 1) x 2 + 2 x - k   f ´çç ÷÷÷ = f ´(-1)  3(k -1) + - k = 3(k -1) - 2 - k  k = 2  
        çè 3 ø 9 3

 
�� �� ��
25. Escriu l’equació de la recta tangent a l’el·lipse � �� en el punt x = .
� � �

æ x2 ö 12 6
y = 4 ççç1- ÷÷÷ x= y=  
è 9 ø÷ 5 5
         
1 æ-8 x ö÷ 4x
f ´( x ) = ⋅ çç ÷÷ø = -  
æ 2ö ç è 9 æ
x x2 ö
2 4 ççç1- ÷÷÷ 9 4 ççç1- ÷÷÷
è 9 ÷ø è 9 ÷ø

æ12 ö 8
f ´çç ÷÷÷ = -  
çè 5 ø 9

æ12 6ö÷
L’equació de la recta tangent en el punt  Pççç , ÷  és: 
è 5 5ø÷

6 8 æ 12ö 8 96 6 8 10
y - =- ⋅ çç x - ÷÷÷  y =- x + +  y =- x +  
5 9 çè 5ø 9 45 5 9 3

 
26. Calcula les derivades segona i tercera per a la funció f (x) = e2x + sin x.
f ´( x ) = 2e 2 x + cos x  f ´´( x ) = 4 e 2 x - sin x  f ´´´( x ) = 8e 2 x - cos x  

 
27. Troba la derivada d’aquestes funcions:

a) ���� � ���� � �� � ��� c) ���� �
��� �
� �
b) ���� � √���� d) ���� � �
��� ������

a) f ´( x ) = 3(4 x 2 + 5 x - 2)2 (8 x + 5)
4
1 - cos x
b) f ´( x) = sin 5 x ⋅ cos x =
5 5 5 sin 4 x

c) f ´( x ) = -
3
(1 + tg 2 x )
tg 4 x

4
-1 2 - 7 - 2x
d) f ´( x ) =
3
( x - 7 x - 12) 3 ⋅ (2 x - 7) =
4
3
( x 2 - 7x -12)

28. Determina la derivada d’aquestes funcions:


� ����� �
a) ���� � ��� b) ���� � ������
a) f ´( x ) = 5 3 x
2
-2 x +1
ln 5 ⋅ (6 x - 2)

b) 2 2
f ´( x ) = 7 cos x ln 7 ⋅ 2 x ⋅ (- sin x 2 ) = - 7 cos x ln 7 ⋅ 2 x sin x 2

632
Derivada d’una funció 1111

 
29. Troba la derivada de les funcions següents:

a) ���� � ���√��� b) ���� � �� � � �

1 1 1 1 -2 -2
a) f ´( x ) = ⋅ ⋅2 = b) f ´( x ) = 1 ⋅ 3 =
2x 2 2x 2x x x
x2

30. Determina la derivada d’aquestes funcions:


a) ���� � ����� � �� b) ���� � ���√�
1
a) f ´( x ) = 2 éêë1 + tg 2 (2 x - 5 ) úùû b) f ´( x ) = - sin ( x )
2 x

 
31. Troba la derivada de les funcions següents:

a) ���� � �� � ��� b) ���� � ��� � ���

a) ln f (x) = ln éêë( x + 1) ù = x ln( x + 1)


x é x3ù
úû b) ln f ( x ) = ln ê( x 2 + 1) ú = x 3 ln ( x 2 + 1)
ëê ûú

f ´( x ) 1 f ´( x ) 1 2x4
= 1 ⋅ ln ( x + 1) + x ⋅ = 3 x 2 ⋅ ln ( x 2 + 1) + x 3 ⋅ 2 ⋅ 2 x = 3 x 2 ln ( x 2 + 1) + 2
f(x) x +1 f(x) x +1 x +1

é x ù x é 2 x 4 ùú 2 x3
f ´( x ) = êln( x + 1) + ú ( x + 1) f ´( x ) = ê3 x 2 ln( x 2 + 1) + 2
ú(
x + 1)
êë x + 1úû ê x + 1
ë û

32. Calcula la derivada de les funcions següents:



a) ���� � ����������� � �� b) ���� � ������� � �

2
3x +1 1 -1 -1
a) f ´( x ) = 2
b) f ´( x ) = ⋅ = 2
1- ( x 3 + x )
2
æ 1 ö÷ x 2 x +1
1 + ççç ÷÷
èxø

 
33. Determina la derivada d’aquestes funcions:
� �
a) ���� � ������ b) ���� � ��� √�
1 1 1 1
a) f ´( x ) = 2 x cos x 2 e sen x
2
b) f ´( x ) = ⋅3 ⋅ =
2 ln 3 x x 3 3 x 2 6 x ln 3 x

   

633
Derivada d’una
Derivada d’una funció
funció 11

ACTIVITATS 

34. Completa a la llibreta aquesta taula amb les taxes de variació mitjana de la funció f(x) = 2 x2 – x + 1.

f (- 1) - f (- 3 ) 4 - 22 f (3) - f (0 ) 16 - 1
T .V .M . ([ –3, – 1]) = = = -9 T .V .M .([0, 3 ]) = = =5
- 1 - (- 3 ) 2 3-0 3

f (2) - f (- 5) 7 - 56 f (4) - f (1) 29 - 2


T .V .M .([ –5, 2 ]) = = = -7 T .V .M .([1, 4 ]) = = =9
2 - (- 5) 7 4-1 3

35. Determina la taxa de variació mitjana d’aquesta funció en cadascun dels intervals indicats:

a) [-1, 1] b) [1, 3] c) [-1, 3]

a) T .V .M .([ –1, 1]) = f (1) - f (- 1) = 1 - 2 = - 1


1 - (- 1) 2 2

f (3) - f (1) 4 -1 3
b) T .V .M .([1, 3 ]) = = =
3-1 2 2

c) T .V .M .([ – 1, 3 ]) = f (3) - f (- 1) = 4 - 2 = 1
3 - (- 1) 4 2

36. Calcula la taxa de variació mitjana d’aquestes funcions en els intervals següents:
a) ࢌሺ࢞ሻ ൌ ࢞ ࢇሾെ૚ǡ ૚ሿ

b) ࢌሺ࢞ሻ ൌ ࢞૛ െ ૞ ࢇሾ૜ǡ ૝ሿ


c) ࢌሺ࢞ሻ ൌ ࢙࢏࢔࢞ ࢇ ሾ ǡ ࣊ሿ

a) f (1) - f (- 1) 1 - (- 1)
T .V . M .([ – 1, 1]) = = =1
1 - (- 1) 1 - (- 1)

f (4) - f (3) 7 21
b) T .V .M .([ 3 , 4 ]) = = 7- =
4-3 4 4

æpö
f (p ) - f ççç ÷÷÷
æ é p ù ö÷ è2ø 0 -1 2
c) ç
T .V .M .ç ê , p ú÷÷ = = =-
èç êë 2 úû ø÷ p-
p p p
2 2

634
Derivada d’una funció 1111

37. Troba la taxa de variació mitjana de la funció f(x) = 2x2 – x en l’interval [2, 2 + h]. Utilitza el resultat
per determinar la taxa de variació mitjana de la funció en els intervals que hi ha a continuació:
a) [2, 3] c) [2, 8]
b) [2, 5] d) [2, 10]
��2 � �� � ��2� 2�2 � ��� � �2 � �� � �
�. �. �. ��2, 2 � ��� � � �
2���2 �
� � �� � ��� � � � � � �
� � � � ��

a) T. V. M. ��2, 3�� � � � 2 � 1 � �
b) T. V. M. ��2, 5�� � � � 2 � 3 � 13
c) T. V. M. ��2, 8�� � � � 2 � � � 1�
d) T. V. M. ��2, 10�� � � � 2 � 8 � 23


38. Calcula la taxa de variació mitjana de la funció ���� � en l’interval [1, 1 + h]. Fes servir aquest

resultat per determinar la taxa de variació mitjana de la funció en els intervals següents:
a) [1, 2] c) [1, 5]
b) [1, 3] d) [1, 10]
1 1
f (1 + h) - f (1) - 1- (1 + h)
1 + h 1 -1
= = =
1+ h - 1 h h (1 + h) (1 + h)
-1 1 -1 1
a) h = 1 =- c) h = 4  =-
(1+ 1) 2 (1+ 4) 5

-1 1 -1 1
b) h = 2  =- d) h = 9  =-
(1+ 2) 3 (1+ 9) 10

39. Calcula el valor que ha de tenir a perquè la taxa de variació mitjana de la funció f(x) = 2x2 + ax – 5 en
l’interval [0, 2] sigui 1.

f (2 ) - f (0 ) 3 + 2 a - (- 5 )
= = 4 + a = 1  a = -3  
2-0 2

 
40. Troba dues funcions polinòmiques de segon grau que passin pels punts (0, 4) i (3, 10). Comprova
que la taxa de variació mitjana en l’interval [0, 3] és la mateixa per a les dues funcions.
Resposta oberta. Per exemple:
La funció és del tipus: f(x) = ax2 + bx + c
Atès que la gràfica passa pel punt (0, 4), es verifica que: c = 4
En passar també pel punt (3, 10), es compleix que: 9a + 3b + 4 = 10 → 3a + b = 2
Siguin f(x) = x2 – x + 4 i g(x) = 2x2 – 4x + 4 les funcions demanades.
��3� � ��0� 10 � �
�. �. �. ��0, 3�� � � �2
3�0 3

635
Derivada d’una
Derivada d’una funció
funció 11

��3� � ��0� �0 � �
�� �� �� ��0, 3�� � � ��
3�0 3

41. Per què la taxa de variació mitjana de la funció f(x) = 2 x – 3 en qualsevol interval sempre és 2?
Perquè la gràfica de la funció és una recta de pendent 2, i indica la seva variació en qualsevol interval.
 
42. Aplica la definició de derivada per calcular les derivades de les funcions en els punts indicats.

a) f (x) = 3x – 1 en x = 2 d) ���� � ������������� � �

b) f (x) = x2 + x en x = 3 e) f(x) = (x - 1)2 en x = – 2
����
c) ���� � �������� � �� f) f(x) = x3 + 5 en x = 0

 
f ( h ) - f (0 ) 3
h + 5-5
f)  f ´(0 ) = lim = lim =0  
h0 h h0 h

 
43. A partir de la definició, calcula les funcions derivades de les funcions següents:
a) f(x) = 2 x + 3 d) f(x) = 2x2 – 3x
���� ��
b) ���� � e) ���� �
� �
c) f(x) = x 3
f ) f(x) = (3x2 + 2)2

636
Derivada d’una funció 1111

 
 
44. Calcula la taxa de variació mitjana de la funció f(x) = 2x2 – 2x + 3 en l’interval [1, 1 + h].
a) Utilitza el resultat per determinar la taxa de variació mitjana en els intervals [1, 3], [1, 5] i [1, 8].
b) Calcula el límit quan h tendeix a zero de la taxa de variació mitjana en l’interval [1, 1 + h] i
comprova que equival a f' (1).

 
 
45. Determina la derivada de cadascuna de les funcions següents en el punt indicat:
a) f(x) = (x + 2)2 – 1 en x = 2
b) f(x) = 5 – 2x en x = 0

637
Derivada d’una
Derivada d’una funció
funció 11


c) ���� � en x = 1
���

d) ���� � √� � �� en x =–1

e) ���� � en x = 8
√���
-3 3
a) f ´( x ) = 2 ( x + 2 )  f ´(2 ) = 8 d) f ´( x ) =  f ´(- 1) = -
2 2- 3x 2 5

-5 5
b) f ´( x ) = - 2  f ´(0 ) = - 2 e) f ´( x ) =  f ´(8) = -
3
2 ( x + 1) 54

-7 7
c) f ´( x ) = 2
 f ´(1) = -
( x - 4) 9

 
46. Troba la derivada de la funció de la gràfica en els punts següents:
a) X = –1 b) x = 0 c) x = 1 d) x = 3

 
æ 3 ö÷ ,
La paràbola passa pels punts çç0, ÷ (1, 1), (3, 3). Substituint aquests punts en l’equació quadràtica
çè 2 ø÷
f(x) = ax2 + bx + c, obtenim la funció:
ìï
3 ïü ïï a = 1
= a ⋅ 0 2 + b ⋅ 0 + c ïï ïï 2
2 ï
ïý  ïïí b = - 1 1 2 3
f (x) = x - x +  f ´( x ) = x - 1
1= a + b + c ïï ïï 2 2
ïï ïï 3
3 = 9a + 3b + c c=
ïþ ïïïî 2

a) f ´(-1) = -2 b) f ´(0 ) = - 1 c) f ´(1) = 0 d) f ´(3 ) = 2

 
47. Demostra gràficament que la derivada d’aquesta funció en el punt d’abscissa 3 té un valor inclòs
entre 2 i 3:

 
La derivada de la funció en el punt x = 3 és el pendent de la recta tangent, i observant el dibuix s’obté que
per cada unitat en horitzontal l’avanç vertical es troba comprès entre 2 i 3 unitats.

638
Derivada d’una funció 1111

 
 
48. Calcula el pendent de la recta tangent a cada funció f (x) en els punts següents:
a) f(x) = 3x2 + 4x – 2 en x = –2
b) f(x) = x – x + x 3 2
en x = 3
c) f(x) = 4x – x – 5 2
en x = 0
a) f ´( x ) = 6 x + 4  f ´(-2) = -8 b) f ´( x ) = 3 x 2 - 2 x + 1  f ´(3 ) = 22 c) f ´( x ) = 8 x - 1  f ´(0 ) = - 1

49. Calcula el pendent de les rectes tangents a la corba f (x) = x2 – 4 en els punts de tall amb els eixos X i
Y.
f ´( x ) = 2 x  

La derivada f´(a) és el pendent de la recta tangent en el punt P(a, f(a)).


Talls amb l’eix X: (2, 0), (–2, 0).
f ´( 2 ) = 4 f ´(- 2 ) = - 4

Tall amb l’eix Y: (0, –4).


f ´( 0 ) = 0

 
50. Determina l’equació de la recta tangent a la gràfica de la funció f (x) és horitzontal.
a) f(x) = x3 + 3x2 b) f(x) = x3 + 3x2 + 3x c) f(x) = x3 + 6x2 + 9x – 1
La tangent a la corba f(x) és horitzontal quan el pendent de la recta tangent és zero, és a dir, quan la
derivada és zero.

a) f ´( x ) = 3 x + 6 x = 3 x ( x + 2) = 0  x1 = 0, x 2 = -2
2

És horitzontal als punts (0, 0) i (–2, 4).

b) f ´( x ) = 3 x + 6 x + 3 = 0  x1 = x 2 = -1
2

És horitzontal al punt (–1, –1).


c) f ´( x ) = 3 x 2 + 12 x + 9 = 0  x 1 = - 1, x 2 = - 3

És horitzontal als punts (–1, –5) i (–3, –1).


 
51. Determina l’equació de la recta tangent a la gràfica de la funció f(x) en el punt indicat.
a) f(x) = 3x2 – 1 en x = 1 c) f(x) = x2 – 2x en x = 1

b) f(x) = x3 en x = 2 d) f(x) = en x = –1

639
Derivada d’una
Derivada d’una funció
funció 11

a) f ´( x ) = 6 x  f ´(1) = 6 f (1) = 2

y - 2 = f ´(1) ⋅ ( x - 1)  y - 2 = 6 ⋅ ( x - 1)  y = 6 x - 4

b) f ´( x ) = 3 x 2  f ´(2 ) = 12 f (2 ) = 8

y - 8 = f ´(2 ) ⋅ ( x - 2 )  y - 8 = 12 ⋅ ( x - 2 )  y = 12 x - 16

c) f ´( x ) = 2 x - 2  f ´(1) = 0 f (1) = -1

y - (- 1) = f ´(1) ⋅ ( x - 1)  y + 1 = 0  y = - 1

d) f ´( x ) = - 12  f ´(- 1) = - 1 f (- 1) = - 1
x

y - (- 1) = f ´(- 1) ⋅ éë x - (- 1)ùû  y + 1 = - 1 ⋅ ( x + 1)  y = - x - 2

 
52. Calcula l’equació de la recta tangent que hi ha a les gràfiques següents:

 
a) Hem de trobar l’equació de la recta tangent a la corba en x = 1.
f ´( x ) = 4 x  f ´(1) = 4 f (1) = -2  
           
y - (-2) = f ´(1) ⋅ ( x - 1)  y + 2 = 4 ⋅ ( x - 1)  y = 4 x - 6  

b) Hem de trobar l’equació de la recta tangent a la corba en x = –1.


f ´( x ) = 3 x 2 - 2 x  f ´(- 1) = 5 f (- 1) = - 2  
       
y - (- 2 ) = f ´(- 1) ⋅ éë x - (- 1)ùû  y + 2 = 5 ⋅ ( x + 1)  y = 5 x + 3  

 
53. Escriu l’equació de la recta tangent a la gràfica de la funció f (x) = x2 + 2x – 5.
a) En el punt d’abscissa 2. c) En el punt d’ordenada –2.
b) En el punt d’abscissa –1. d) En el punt de tall amb l’eix Y.
f ´( x ) = 2 x + 2

a) f ´(2 ) = 6 f (2) = 3 c) - 2 = x 2 + 2 x - 5  x 1 = 1, x 2 = - 3

y - 3 = 6 ⋅ ( x - 2)  y = 6 x - 9 f ´(1) = 4  y + 2 = 4 ⋅ ( x - 1)  y = 4 x - 6

f ´(- 3) = - 4  y + 2 = - 4 ⋅ ( x + 3 )  y = - 4 x - 14

b) f ´(- 1) = 0 f (-1) = -6 d) El punt de tall amb l’eix Y és (0, –5):

y - (-6 ) = 0  y = -6 f ´(0 ) = 2  y - (- 5) = 2 ⋅ x  y = 2 x - 5

 
54. Indica l’equació de la recta tangent a la gràfica de la funció f(x) = x3 + 3x2.
a) En els punts de tall amb els eixos X i Y.
b) En el punt d’abscissa 1.
c) En els punts d’ordenada 4.

640
Derivada d’una funció 1111

f ´( x ) = 3 x 2 + 6 x

a) Els punts de tall amb l’eix X són (0, 0) i (-3, 0).


El punt de tall amb l’eix Y és (0, 0).
f ´( 0 ) = 0

La recta tangent en (0, 0) és y = 0.


f ´(- 3 ) = 9  y - 0 = 9 ⋅ éë x - (- 3 )ùû  y = 9 x + 27

b) f ´(1) = 9 f (1) = 4

y - 4 = 9 ⋅ ( x - 1)  y = 9 x - 5

c) 4 = x 3 + 3 x 2  x 1 = 1, x 2 = x 3 = - 2

x1 = 1 coincideix amb l’apartat b).


f ´(- 2 ) = 0  y - 4 = 0  y = 4

55. Escriu l’equació de la recta tangent a la gràfica de la funció f(x) = 2 + ln x en el punt d’abscissa 1.
1
f ´( x ) =  f ´(1) = 1             f (1) = 2 y - 2 = 1⋅ ( x - 1)  y = x + 1  
x        
 
56. Determina l’equació de la recta tangent a la gràfica de la funció f (x) = x3 paral·lela a la recta y = x + 1.
Com que ha de ser paral·lela la recta y = x + 1, el pendent ha de ser 1, és a dir, la derivada ha de valer 1.
1 1
f ´( x ) = 3 x 2 = 1 x1 = , x2 =-  
3 3
æ 1 ö÷ 1 1 1 2
f ççç ÷÷ =  y- = x-  y= x-
çè 3 ø÷ 3 3 3 3 3 3 3
 
æ 1 ÷÷ö 1 æ 1 ÷ö æ 1 ÷ö 2
ç
f ç-  y - çç- ÷ = x - çç-  
÷=- ÷ y = x +
çç
è 3 ÷ø 3 3 èç 3 3 ø÷ èç 3 ø÷ 3 3

 
57. Determina en els casos següents l’equació de la recta tangent a la gràfica de la funció f(x) que sigui
paral·lela a la recta 3x – y + 6 = 0.
a) f(x) = x2 + 4x – 2 c) f(x) = x3 – 4
૜࢞ ૜࢞૛ ା૚
b) ࢌሺ࢞ሻ ൌ d) ࢌሺ࢞ሻ ൌ
૚ି࢞ ࢞
Com que y = 3x + 6, el pendent de la recta és 3.

a) f ´( x ) = 2 x + 4 = 3  x = - 1
2

æ 1ö 15 æ 15ö é æ 1öù 9
f çç- ÷÷÷ = -  y -çç- ÷÷÷ = 3 ⋅ ê x -çç- ÷÷÷ú  y = 3 x -
èç 2ø 4 ç
è 4 ø ê
ë
ç
è 2û øú 4

ì
ï x 1 = 0  f (0 ) = 0  y = 3 x
b) f ´( x ) = (
3 1 - x ) - 3 x ⋅ (- 1) 3
= =3ï
í
2
(1 - x ) (1 - x ) ï x 2 = 2  f (2) = - 6  y - (- 6 ) = 3 ( x - 2 )  y = 3 x - 12
ï
2
î

ì x 1 = 1  f (1) = - 3  y - (- 3 ) = 3 ( x - 1)  y = 3 x - 6
ï
c) f ´( x ) = 3 x 2 = 3  ï
í
ïï x 2 = - 1  f (- 1) = - 5  y - (- 5 ) = 3 é x - (- 1)ù  y = 3 x - 2
ï
î ë û

641
Derivada d’una
Derivada d’una funció
funció 11

6 x ⋅ x - 3x2 -1 3x2 -1
d) f ´( x ) = = = 3  3 x 2 - 1 = 3 x 2  - 1 = 0  No té solució.
x2 x2

58. Calcula l’equació de la recta tangent a la gràfica de la funció ࢌሺ࢞ሻ ൌ paral·lela a la recta y – x = 6.
࢞ା૜

Com que y  x  6, el pendent de la recta és 1. 

x +3- x 3 ( ) ( ) (
ìï x = - 3 + 3  f - 3 + 3 = - 3 + 1  y - - 3 + 1 = x - - 3 + 3
ïï 1 ) 
f ´( x ) = = = 1  í
( )
ï x = -3 - 3  f -3 - 3 = 3 + 1  y = x + 4 + 2 3
2 2
( x + 3) ( x + 3) ï
ï
î 2

 
59. Calcula el vèrtex de les paràboles següents si sabem que aquest punt de la corba té per tangent una
recta paral·lela a l’eix X.
a) f(x) = x2 – 4x + 6 d) f(x) = 2x2 + 4x – 3
b) f(x) = –x2 + 2x – 1 e) f(x) = 3x2 – 6x + 5
c) f(x) = –x2 + 2x + 1 f) f(x) = (x – 1)(2x + 5)
Com que la recta tangent al vèrtex és horitzontal, el pendent ha de ser zero, és a dir, la derivada ha de ser
zero.
a) f ´( x ) = 2 x - 4 = 0  x = 2, f (2 ) = 2 V(2, 2)

b) f ´( x ) = -2 x + 2 = 0  x = 1, f (1) = 0 V(1, 0)

c) f ´( x ) = -2 x + 2 = 0  x = 1, f (1) = 2 V(1, 2)

d) f ´( x ) = 4 x + 4 = 0  x = - 1, f (- 1) = - 5 V(-1, -5)

e) f ´( x ) = 6 x - 6 = 0  x = 1, f (1) = 2 V(1, 2)

f) f ( x ) = ( x - 1)(2 x + 5) = 2 x 2 + 3 x - 5

3 æ 3ö 49 æ 3 49 ö
f ´( x ) = 4 x + 3 = 0  x =- , f çç- ÷÷÷ =- V çç- ,- ÷÷÷
4 è 4øç 8 çè 4 8ø

60. Calcula el punt de tall de les rectes tangents a la corba f (x) en els punts d’abscissa 2 i 0.
a) f(x) = x2 – x c) f(x) = x2 + 1
b) f(x) = x2 – 4x + 2 d) f(x) = ln (x + 1)
a) f ´( x ) = 2 x - 1

f ´(2) = 3ïüï ü
f ´(0 ) = - 1ï
ï  y = -x
ý  y - 2 = 3 ( x - 2)  y = 3 x - 4 ý
f (2) = 2ïïþ f (0 ) = 0 ïï
þ

El punt de tall de les dues rectes és:


y = - x üïï
ý  3 x - 4 = -x  x = 1, y = -1  P(1, - 1)
y = 3 x - 4ïïþ

b) f ´( x ) = 2 x - 4

f ´(2) = 0 ïüï ü
f ´(0 ) = - 4ï
ï  y - 2 = -4 x  y = -4 x + 2
ý  y - (-2) = 0  y = -2 ý
f (2) = -2ïïþ f (0 ) = 2 ïï
þ

El punt de tall de les dues rectes és:


y = -2 ü
ï
ï
ý  y = -2, x = 1  P(1, - 2)
y = -4 x + 2ï
ï
þ

642
Derivada d’una funció 1111

c) f ´( x ) = 2 x

f ´( 2 ) = 4 f (2 ) = 5 y - 5 = 4 ( x - 2)  y = 4 x - 3

f ´( 0 ) = 0 f ( 0) = 1 y -1= 0  y = 1

El punt de tall de les dues rectes és:


ü
y = 4 x - 3ï
ï
ý  y = 1, x = 1  P (1, 1)
y =1 ï
ï
þ
1
d) f ´( x ) =
x +1

1 üï
ï f ´(0 ) = 1 ü
ï
f ´( 2 ) = ï 1 1 2 ï y = x
3 ý  y - ln 3 = ( x - 2 )  y = x - + ln 3 ý
ï
ï 3 3 3 ï
f (0 ) = 0 ï
f (2 ) = ln 3ï þ
ï
þ

El punt de tall de les dues rectes és:


1 2 üï
y= x - + ln 3ïï 3 æ 3 3 ö
3 3 ý  y = x = - 1 + ln 3  P ççç- 1 + ln 3, - 1 + ln 3÷÷÷
ï
ï 2 è 2 2 ø
y=x ï
þ

61. Indica en quins punts de la gràfica de la funció f (x) = x3 – x2 + x la recta tangent té pendent 2.

La recta tangent té pendent 2 en els punts en què la derivada és 2.


æ 1 13 ö
f ´( x ) = 3 x 2 - 2 x + 1 = 2  x1 = 1, x 2 = -
1 A(1, 1), Bçç- , - ÷÷÷
3 çè 3 27ø

 
���
62. Considera la funció ���� � . Calcula els valors de la variable x en cada cas i interpreta
���
geomètricament el que obtens.
a) ����� � � c) ����� � �

b) ����� � � d) ����� �

x + 2 - ( x - 2) 4
f ´( x ) = 2
= 2
 
( x + 2) ( x + 2)

4
a) f ´( x ) = 2
= 1 x1 = 0, x2 = -4
( x + 2)
La recta tangent a la corba té pendent 1 als punts d’abscissa 0 i –4.
4
b) f ´( x ) = 2
= 4  x1 = -1, x2 = -3
( x + 2)
La recta tangent a la corba té pendent 4 als punts d’abscissa –1 i –3.
4
c) f ´( x ) = 2
¹0
( x + 2)
No existeixen punts en els quals la recta tangent a la corba sigui paral∙lela l’eix X. 
4 1
d) f ´( x ) = =  x1 = 2, x2 =-6
2
( x + 2) 4

643
Derivada d’una
Derivada d’una funció
funció 11

La recta tangent a la corba té pendent 1 en els punts d’abscissa 2 i –6.


4

 
����
63. Calcula el valor de k perquè la funció ���� � verifiqui que f'(–1) = 19.
����
� � ��� � �� � ��� � �� � � �� � 10
� � ��� � �

��� � �� ��� � ���
� ��1�
� � �� � 10 � 1� � � � �

 
64. Troba els valors de a i b perquè les funcions f(x) i g(x) tinguin la mateixa recta tangent en x = 3.
f(x) = ax2 – 1 g(x) = x2 + 3x + b
f ´( x ) = 2 ax g ´( x ) = 2 x + 3

Necessitem que coincideixin en el punt x = 3, és a dir, que f (3) = g (3) .

També necessitem que el pendent sigui el mateix en aquest punt, és a dir, que f ´(3 ) = g ´(3 ) .

Resolem el sistema de equacions:

9 a - 1 = 9 + 9 + bïü
ï 3 11
ý a= , b=-
2 ⋅ 3a = 2 ⋅ 3 + 3 ï
ï
þ 2 2

65. Determina l’equació de la recta normal de les funcions f(x) següents en el punt indicat:
a) f(x) = x2 – 2x en x = 2 c) f(x) = x3 en x = 1

b) f(x) = 2 – x2 en x = 3 d) f(x) = en x = –1

a) f ´( x ) = 2 x - 2 f ´( 2 ) = 2 f (2 ) = 0

La recta tangent és: y = 2 ( x - 2 )

La recta normal és: y = - 1 ( x - 2 )


2

b) f ´( x ) = - 2 x f ´( 3 ) = - 6 f (3) = -7

La recta tangent és: y + 7 = - 6 ( x - 3 )

La recta normal és: y + 7 = 1 ( x - 3 )  y = 1 x - 15


6 6 2

c) f ´( x ) = 3 x 2
f ´(1) = 3 f (1) = 1

La recta tangent és: y - 1 = 3 ( x - 1)

La recta normal és: y - 1 = - 1 ( x - 1)  y = - 1 x + 4


3 3 3

d) f ´( x ) = - 12 f ´(-1) = -1 f (- 1) = - 1
x

La recta tangent és: y + 1 = - ( x + 1)

La recta normal és: y + 1 = x + 1  y = x


66. Calcula l’equació de la recta normal a la gràfica de f(x) = x2 que sigui paral·lela a la recta y = 2x – 1.

644
Derivada d’una funció 1111

Com que el pendent de la recta paral·lela a la recta normal és 2, el pendent de la recta tangent haurà ser
1
- .
2

æ 1ö 1
1
f ´( x ) = 2 x = -  x = -
1 f çç- ÷÷÷ =
2 4 çè 4ø 16

1 æ 1ö 9
L’equació de la recta normal és: y - = 2çç x + ÷÷÷  y = 2 x +
16 çè 4ø 16

67. Esbrina les equacions de les rectes perpendiculars que hi ha a la gràfica següent:

 
f ´( x ) = 6 x - 3
2
f ´(- 1) = 3 f (-1) = 1

La recta tangent a la corba és: y - 1 = 3 ( x + 1)  y = 3 x + 4

La recta normal a la corba és: y - 1 = - 1 ( x + 1)  y = - 1 x + 2


3 3 3

 
68. Calcula la recta tangent i la recta normal a les funcions següents en els punts indicats:
a) f(x) = 23x-8 en x = 3
b) f(x) = x2 ln (x + 3) en x = –2
c) f(x) = (3x – 5)6 en x = 2
a) f ´( x ) = 3 ⋅ 2 3 x -8
⋅ ln 2

L’equació de la recta tangent és: y - 2 = 6 ln 2( x - 2)  y = 6 ln 2( x - 2) + 2

L’equació de la recta normal és: y-2=-


1
( x - 2)  y = -
1
( x - 2) + 2
6 ln 2 6 ln 2


b) � � ��� � �� ���� � �� � � � �
���
L’equació de la recta tangent és: y – 0 = 4(x + 2) → y = 4x + 8
� � �
L’equació de la recta normal és: y – 0 = (x + 2) → y = – x –
� � �
� ��� � �
c) � � ���� � �� � � � ����� � ��
L’equació de la recta tangent és: y – 1 = 18(x – 2) → y = 18x – 35
� � ��
L’equació de la recta normal és: y – 1 = – (x – 2) → y = – x +
�� �� �

69. Determina la recta tangent i la recta normal a aquestes funcions en els punts indicats:
���
a) f(x) = √�� � � en x = 5 c) f(x) = �� en x = �

b) f(x) = sin (2 x + r) en x = 0 d) f(x) = ln (x2 + 1) en x = 0

645
Derivada d’una
Derivada d’una funció
funció 11

� � �
a) � � ��� � ��� � ���� � � �
� √����
� � ��
L’equació de la recta tangent és: � � � � �� � �� � � � � � �
� � �

L’equació de la recta normal és: � � � � ���� � �� � � � � ��� � ��


b) � � ��� � ������ � �� � �
L’equació de la recta tangent és: � � 0 � ���� � 0� � � � ���
� �
L’equació de la recta normal és: � � 0 � �� � 0� � � � � �
� �
��� �
c) � � ��� � �� � ��� � � �� �
� �
� � �
L’equació de la recta tangent és: � � 0 � � �� � �� � � � � � �
� � �

L’equació de la recta normal és: � � 0 � ��� � �� � � � ��� � ��


� ��
d) � � ��� � � �� �
� � �� � � ��

L’equació de la recta tangent és: � � 0


L’equació de la recta normal és: � � 0

70. Aplica la derivada de la suma a la funció f(x) = 3x4 – 2x2 – 7x + 5 per calcular:
a) La funció derivada.
b) La derivada en els punts d’abscissa x = –2, x = 0 i x = 1.

a) f ´( x ) = 12 x 3 - 4 x - 7

b) 3
f ´(- 2 ) = 12 ⋅ (- 2 ) - 4 ⋅ (- 2 ) - 7 = - 95 f ´(0 ) = 12 ⋅ 0 3 - 4 ⋅ 0 - 7 = - 7 f ´(1) = 12 ⋅ 13 - 4 ⋅ 1 - 7 = 1

71. Considera la funció f(x) = 3x4 – 5x3 + x – 6, i calcula:


a) f'(x)
b) f'(3), f' (–2) i f'(0)
c) f'(4 - 8) i f'(4) – f'(8). Són iguals?
a) f ´( x ) = 12 x 3 - 15 x 2 + 1

b) f ´(3 ) = 12 ⋅ 3 3 - 15 ⋅ 3 2 + 1 = 190
3 2
f ´(- 2 ) = 12 ⋅ (- 2 ) - 15 ⋅ (- 2 ) + 1 = - 155

f ´(0 ) = 12 ⋅ 0 3 - 15 ⋅ 0 2 + 1 = 1

c) 3 2
f ´(4 - 8 ) = 12 ⋅ (- 4 ) - 15 ⋅ (- 4 ) + 1 = - 1 007

f ´(4 ) = 12 ⋅ 4 3 - 15 ⋅ 4 2 + 1 = 529

f ´(8 ) = 12 ⋅ 8 3 - 15 ⋅ 8 2 + 1 = 5 185

f ´(4 ) - f ´(8 ) = - 4 656 ¹ - 1 007 = f ´(4 - 8 )

   

646
Derivada d’una funció 1111

72. Determina la derivada de les funcions següents:


a) f(x) = –3x3 + 5x2 – x + 5 c) f(x) = x(2 + x2) + 3
b) f(x) = –2(x4 – 9x2) + x d) f(x) = x6 – 10x2 – x – 3
a) f ´( x ) = -9 x 2 + 10 x - 1 c) f ´( x ) = 3 x 2 + 2

b) f ´( x ) = -8 x 3 + 36 x + 1 d) f ´( x ) = 6 x 5 - 20 x + 34
x

73. Fes servir les regles de derivació per calcular la funció derivada de les funcions següents:
a) f(x) = 5x4 + 3x3 – 7x2 + 12x – 1
૜࢞૛ ି૞࢞
b) ࢌሺ࢞ሻ ൌ
࢞ା૚
ି૜࢞૛ ାૡ࢞ିૢ
c) ࢌሺ࢞ሻ ൌ

a) f ´( x ) = 20 x 3 + 9 x 2 - 14 x + 12

(6 x - 5 )( x + 1) - (3 x 2 - 5 x ) 3x2 + 6x - 5
b) f ´( x ) = 2
= 2
( x + 1) ( x + 1)

c) f ´( x ) = - 3 ⋅ 2 x + 8 = - 3 x + 4
2

74. Aplica la derivada del producte a la funció f (x) = (5x2 – 3x) · (x4 – 2x + 5) per calcular:
a) La funció derivada.
b) La derivada en els punts d’abscissa –3, 0 i 2.
a) f ´( x ) = (10 x - 3 )( x 4 - 2 x + 5 ) + (4 x 3 - 2 )(5 x 2 - 3 x ) = 30 x 5 - 15 x 4 - 30 x 2 + 62 x - 15

b) 5 4 2
f ´(- 3 ) = 30 ⋅ (- 3 ) - 15 ⋅ (- 3 ) - 30 ⋅ (- 3 ) + 62 ⋅ (- 3 ) - 15 = - 8 976

f ´(0 ) = 30 ⋅ 0 5 - 15 ⋅ 0 4 - 30 ⋅ 0 2 + 62 ⋅ 0 - 15 = - 15

f ´(2 ) = 30 ⋅ 25 - 15 ⋅ 2 4 - 30 ⋅ 2 2 + 62 ⋅ 2 - 15 = 709

75. Utilitza les regles de derivació per calcular la funció derivada de les funcions següents:
a) f (x) = (3x2 – 1) · 4x
૛࢞ି૜
b) ࢌሺ࢞ሻ ൌ ൫െ૜࢞૛ ൅ ࢞ െ ૚൯ ൉ ቀ ቁ

c) f (x) = 2x · (5x – 3) · (x2 – 3x + 1)

a) f ´( x ) = 6 x ⋅ 4 x + (3 x 2 - 1) ⋅ 4 = 36 x 2 - 4

æ 2 x - 3÷ö æ 2ö -18 x 2 + 22 x - 5
b) f ´( x ) = (-6 x + 1)ççç ÷÷ +(-3 x 2 + x -1)ççç ÷÷÷ =
è 3 ø è 3ø 3

c) f ´( x) = 2éëê(5x -3)( x -3x +1)ùûú +2x éëê5( x -3x +1) +(5x -3)(2x -3)ùûú = 40x -108x +56x -6
2 2 3 2

   

647
Derivada d’una
Derivada d’una funció
funció 11

����
76. Aplica la regla del quocient a la funció ���� � per calcular:
�� ���
a) La funció derivada.
b) La derivada en els punts d’abscissa –1, 1 i 2.
3( x 2 - 5 x ) -(3 x -1)(2 x - 5) -3 x 2 + 2 x - 5
a) f ´( x ) = 2
= 2
( x 2 - 5x ) ( x 2 - 5x)
2
-3 ⋅ (-1) + 2 ⋅ (-1) - 5 10 5
b) f ´(-1) = 2
=- =-
é(-1)2 - 5 ⋅ (-1)ù 36 18
êë úû

- 3 ⋅ 12 + 2 ⋅ 1 - 5 6 3 -3 ⋅ 22 + 2 ⋅ 2 - 5 13
f ´(1) = 2
=- =- f ´(2) = 2
=-
(1 2
- 5 ⋅ 1) 16 8 (2 2
- 5 ⋅ 2) 36

 
77. Utilitza les regles de derivació per calcular la funció derivada de les funcions següents:
��� ����� �
a) ���� � c) ���� �
��� �� ��
��� �� ����
b) ���� � d) ���� �
��� ���� ��� ��

a) (21x 2 - 2)( x - 2) - (7 x 3 - 2 x + 4 ) 14 x 3 - 42 x 2
f ´( x ) = 2
= 2
( x - 2) ( x - 2)

24 x 3 (7 x 2 - x + 3) - 6 x 4 (14 x -1) 84 x 5 -18 x 4 + 72 x 3


b) f ´( x ) = 2
= 2
(7x 2 - x + 3) (7x2 - x + 3)
-4 -8 x
c) f ´( x ) = 2
⋅ 2x = 2
( x 2 - 1) ( x 2 - 1)
(2 x - 1)(2 x 2 - x ) -( x 2 - x + 3)(4 x -1) x 2 - 12 x + 3
d) f ´( x ) = 2
= 2
(2 x 2 - x ) (2 x 2 - x )

78. Efectua la derivada de la funció següent:


��� ���
���� � �
��� ���
( x + 1) - ( x - 1) ( x - 1) - ( x + 1) 2 2 8x  
f ´( x ) = 2
+ 2
= 2
- 2
=- 2
( x + 1) ( x - 1) ( x + 1) ( x - 1) ( x 2 - 1)
 
79. Calcula la derivada de les funcions següents:
� �
a) ���� � √�� c) ���� � √� � �� � √��
� √�
b) ���� � √�� � � √� d) ���� � �
�� √�
4
7 73 x 4 1 -
4
7 -103 1 7
a) f ´( x ) = x 3 = c) f ( x ) = 5 x - x 7  f ´( x ) = x 5 -
10
x = 5 -
3 3 5 10 5 x 4 1010 x 3

1 æ 1 ö÷
⋅ (1 - 3 x ) - x ⋅ ççç- 3 ÷÷
3 1 3 -4 3 x 3 2 x èç 3 x 2 ø÷ 33 x2 - x
b) f ´( x ) = x 2 - x 5 = - 5 d) f ´( x ) = =
2 5 2 5 x4 (1- 3
x)
2
6 6 x 7 (1 - 3 x )
2

648
Derivada d’una funció 1111

80. Determina la derivada de les funcions que hi ha a continuació:


�� ���
a) ���� � � √� 
����
��� ���
b) ���� � �  
��� ���
�� ����� √���
c) ���� � �  
��� �� �

 
(2 x - 4 )(5 x + 2 ) - ( x 2 - 4 x )5 1 5x2 + 4x - 8 1
a) f ´( x ) = 2
- = 2
-
(5 x + 2 ) 2 x (5 x + 2 ) 2 x

x 2 + 2 x -15 (2 x + 2)( x 2 - 3 x + 2) -( x 2 + 2x -15)(2x - 3) -5x 2 + 34 x - 41


b) f ( x ) =  f ´( x ) = 2
= 2
x2 - 3x + 2 ( x 2 - 3 x + 2) ( x 2 - 3 x + 2)
1
x - x +5
(2 x - 3)(2 x 2 - x ) -( x 2 - 3 x - 1)(4 x - 1) 2 x +5 5x 2 + 4 x -1 x + 10
c) f ´( x ) = 2
+ = 2
- 2
(2 x 2 - x ) (2 x - x )
2
x 2 2 x x +5

81. Calcula la derivada d’aquesta funció:

��� � � � � � �
���� � � � � �
�� � � �

2 x 2 + x - 3 çæ 1 ÷ö 2x + 3 1
2
1 æ 2( x + 1) - (2 x + 3)ö÷ 2 - x +1 2
ç ÷
f ( x) = + ç ÷÷ = + 2  f ´( x ) = ⋅ çç ÷÷ - 3 = 2 2 x + 3 x + 1 2 - x 3  
x2 -1 èç x ø x +1 x 2x + 3 çè ( x + 1)
2
ø÷ x ( )
2
x +1

 
82. Troba la derivada de les funcions exponencials i logarítmiques següents:
������
a) ���� � ���� � �� d) ���� �
��
����
b) ���� � �� ����� � � e) ���� � ��
��

c) ���� � ��� � ������ �� f) ���� � ��� � ��

a) f ´( x ) = 1 + e x
x

1 æ 1 ö÷
b) f ´( x ) = 2 x log x + x 2 = x ççç2 log x + ÷÷
x ln 10 çè ln 10 ÷ø

x2 + 3
c) f ´( x ) = 2 x log2 x +
x ln 2

1 x
⋅ e -(ln x + 4) ⋅ ex
x 1- x ln x - 4 x
d) f ´( x ) = =
e2 x xex
1 x
⋅ e -ln x ⋅ ex
x 1- x ln x
e) f ´( x) = 2x
=
e xex
f) f ´( x ) = 5e x - 3 x ln 3

649
Derivada d’una
Derivada d’una funció
funció 11

83. Deriva les funcions trigonomètriques següents:


a) ���� � ����������� d) ���� � ������
b) ���� � � ����� � �

e) ���� � �����������

c) ���� � ������������� f) ���� �
����

a) ����� � ��� � � � ���� �


� ��� ��� � ��������� � ����� ��������
b) ����� � �
�� ��
��� �� ��� � � �
c) � ���� � � ������� � ��� � � �� �� �
��� � � ���� � ��� � � ���� �

d) � ���� � � � ���� � � � �� � ��� ��� � � � ���� � ����� �


� �
e) � ���� � � � ��������� � � � �� � � ��������� �
√��� � √��� �
������� �
f) � ���� �
���� �

 
84. Calcula la derivada de les funcions trigonomètriques següents:
a) ���� � �� � ��������� � ���������
b) ���� � �� � �� ����������
c) ���� � ���������
d) ���� � �� ������
�����
e) ���� �
���

 
 
85. Calcula les sis primeres derivades de les funcions f(x) = sin x i g(x) = cos x.

 
 
86. Determina les tres primeres derivades de les funcions següents:
a) ���� � �� � ��� c) ���� � �����
b) ���� � √�� � �� d) ���� � ���

650
Derivada d’una funció 1111

a) f ´( x ) = 4 x 3 + 21x 2 f ´´( x ) = 12 x 2 + 42 x f ´´´( x ) = 24 x + 42

3x2 - 4 3 x 4 - 24 x 2 -16 -3 ( x 6 - 20 x 4 - 80 x 2 + 64)


b) f ´( x) = f ´´( x ) = 3 f ´´´( x ) = 5
2 x3 -4x 4( x 3 - 4 x ) 2
8 ( x 3 - 4 x )2

c) f ´( x ) = 2 x cos x 2 f ´´( x ) = 2 cos x 2 - 4 x 2 sin x 2 f ´´´( x ) = - 12 x sin x 2 - 8 x 3 cos x 2

d) f ´( x ) = 2 e 2 x f ´´( x ) = 4 e 2 x f ´´´( x ) = 8 e 2 x

87. Calcula les derivades primera, segona i tercera d’aquestes funcions:


a) ���� � �� � ��� � � � � c) ���� � �����
b) ���� � √� � � d) ���� � ������������
a) f ´( x ) = 3 x 2 + 4 x + 1 f ´´( x ) = 6 x + 4 f ´´´( x ) = 6

1 1 3
b) f ´( x ) = f ´´( x ) = - f ´´´( x ) =
2 x-2 4 ( x - 2)
3
8 ( x - 2)
5

c) f ´( x ) = 1 f ´´( x ) = -
1
f ´´´( x ) =
2
x x2 x3

d) f ´( x ) = (cos x - sin x ) e
sin x+cos x

2
f ´´( x ) = (cos x - sin x ) e sin x + cos x - e sin x + cos x ( sin x + cos x )

f ´´´( x) = esin x+cos x éê(cos x - sin x) -3(cos2 x - sin2 x) -(cos x - sin x)ùú
3

ë û

88. Troba els valors on s’anul·la la derivada segona de les funcions següents:
a) f(x) = –3x3 + 4x2 – 3x + 1
b) f(x) = ln (x2 + 2)
4
a) f ´( x ) = - 9 x 2 + 8 x - 3 f ´´( x ) = - 18 x + 8 = 0  x =
9

2x -2 x 2 + 4
b) f ´( x ) = f ´´( x ) = 2
=0 x= 2
x2 + 2 ( x 2 + 2)


89. Donada ���� � , calcula el valor de x en aquests casos:
���
a) f'(x) = 0 b) f"(x) = 0
2 x ( x + 3) - x 2 2
x + 6x
a) f ´( x ) = 2
= 2
= 0  x1 = 0, x 2 = -6
( x + 3) ( x + 3)

(2 x + 6 )( x + 3 ) - 2 ( x 2 + 6 x )( x + 3 )
2
18
b) f ´´( x ) = 4
= 3
¹0
( x + 3) ( x + 3)

No existeix cap valor de x que anul·li la segona derivada.


 
90. Calcula la derivada enèsima, fn(x), d’aquesta funció:

���� �
���

651
Derivada d’una
Derivada d’una funció
funció 11

Calculem les primeres derivades:


-2 -3 -4 -5
f ´( x ) = - 2 ⋅ ( x - 1) f ´´( x ) = 4 ( x - 1) f ´´´( x ) = - 12 ( x - 1) f IV ( x ) = 48 ( x - 1)

La derivada enèsima és del tipus:


n - ( n + 1)
f n ( x ) = (- 1) 2 ⋅ n !( x - 1)

91. Escriu les funcions elementals que componen aquestes funcions i troba’n les derivades:
a) f(x) = ln (2x2) d) f(x) = e3x
b) f(x) = log3 (x2 – 1) e) f(x) = cos (3x – 1)
c) f(x) = 10x+3 f ) f(x) = sin (x2 – 3)

a) f(x) = g[h(x)], on g ( x ) = ln x i h ( x ) = 2 x 2 → f ´( x ) = 2
x

2x
b) f(x) = g[h(x)], on g ( x ) = log 3 x i h ( x ) = x 2 - 1 → f ´( x ) =
( x 2 - 1) ln 3
c) f(x) = g[h(x)], on g ( x ) = 10 x i h ( x ) = x + 3 → f ´( x ) = 10 x + 3 ln 10

d) f(x) = g[h(x)], on g ( x ) = e x i h ( x ) = 3 x → f ´( x ) = 3 e 3 x

e) f(x) = g[h(x)], on g ( x ) = cos x i h( x ) = 3 x - 1 → f ´( x ) = - 3 sin (3 x - 1)

f) f(x) = g[h(x)], on g ( x ) = sin x i h( x ) = x 2 - 3 → f ´( x ) = 2 x co s ( x 2 - 3 )

92. Escriu les funcions que componen les funcions següents i calcula’n les derivades:
a) f(x) = log3 (2x + 1) e) f(x) = 23x – 4

b) f(x) = (3x2 – 3x + 1)4 f) f(x) = √�� � �
c) f(x) = sin √� g) f(x) = cos ln x
d) f(x) = arc tg ex h) f(x) = 3cos x

 
 
93. Aplica la regla de la cadena per calcular les derivades d’aquestes funcions exponencials i
logarítmiques:
�� ���
a) ���� � �� e) ���� � ����
���� ���

652
Derivada d’una funció 1111

� ���
b) ���� � �� ���� f) ���� � �����
��� �� �����
c) ���� � � � g) ���� � �
��� ��
����� ��� ���
d) ���� � h) ���� � �� �
����
��

2 x + 1 3 x (2 x + 1) - 2 x
2 3
4x + 3
a) f ´( x ) = ⋅ =
x 3
(2 x + 1)
2
x (2 x + 1)

ex
b) f ´( x ) = e x ln x +
x

æ 6 x ⋅ x -(3 x 2 - 1)ö÷
÷÷ = e x æçç 3 x + 1ö÷÷
3 x 2 -1 3 x 2 -1 2
çç
c) f ´( x ) = e x
ççç ÷ ç ÷
÷÷ø è x ÷ø
2 2
è x

4e4 x +1(1+ x 4 ) - 4 x 3e4 x +1 4e4 x +1(1- x 3 + x 4 )


d) f ´( x) = 2
= 2
(1+ x4 ) (1+ x4 )
x - 7 1 4 x ( x - 7) - 2 x
2
x - 14
e) f ´( x ) = ⋅ ⋅ =
2 x 2 ln 2 ( x - 7)
2
x ( x - 7) ln 2

1
f) f ´( x ) = 3
(
ln x 4 + 2 )
ln 3 ⋅ ⋅4x3
x4 + 2

3 -x2 - x 2
e - ln x 3 ⋅ e-x - x ⋅ (-2 x - 1) e x 2 + x é3 + (2 x 2 + x ) ln x 3 ù
g) f ´( x ) = x = ëê ûú
2
e-2 x -2 x x
1 1
æ-1ö
e
1
x (3e3 x + e x ) e x - (e3 x + e x ) ⋅ ççèç x 2 ÷÷ø÷ e x 1
h) f ´( x ) = 2
=3+
e3 x + e x x2
e x

 
94. Aplica la regla de la cadena per calcular les derivades d'aquestes funcions trigonomètriques:
���
a) ���� � ������� e) ���� � ����

b) ���� � ������ � �� �
f) ���� � ���√� � �

c) ���� � ������ � ��� g) ���� � �����
��� ����

d) ���� � �����√�� � �� h) ���� � ���
√���

1 æ x -1ö÷
e) f ´( x) =- sinçç
a) f ´( x ) = 6 x cos 3 x 2
x2 çè x ÷÷ø

b) f ´( x ) = - 2 x sin ( x 2 + 1) f) f ´( x ) = (1 + tg 2 x - 1) ⋅
1
2 x -1

c) f ´( x) = éêë1+tg ( x -3x)ùúû ⋅(2x -3)


2 2 æ x ö÷ 3x4 -1
g) f ´( x ) = -cos ççç ÷÷ ⋅
è - x + x - 1ø (- x 4 + x - 1)2
4

1 æ 2 ÷ö 1
d) f ´( x ) = cos x + 3 x ⋅ ⋅ (2x + 3)
2
h) f ´( x ) = çç1 + tg 2 ÷⋅
2
2 x + 3x çè 1- x ÷ø (1- x )3

653
Derivada d’una
Derivada d’una funció
funció 11

95. Calcula la derivada d’aquestes funcions:


����
a) ���� � ��� � ��� � �� d) ���� �
��

�� �� ��� ��
b) ���� � � e) ���� �
�� ��

�� �� �
c) ���� � f) ���� � �� �
�� � �� �

 
 
96. Aplica la regla de la cadena per determinar les derivades d'aquestes funcions potencials:
a) ���� � ���� � �� � ��� f) ���� � �� � ��� ��
� �
b) ���� � � � g) ���� � ���� ��
����
� �
c) ���� � � √�� � �� h) ���� � ���� ��� � �� � ��

d) ���� � �� � ��� �� i) ���� � ��� ��� � ��


� � �
e) ���� � ��� ��� � ��� j) ���� � ����� � ��� �
� �
4
a) f ´( x) = 5(3x -2x +1) (12x -2)
4 3

æ 1- x 3 - x ⋅ -3 x 2 ö÷ 5 x 4 1 + 2 x 3
æ x ö÷
4
çç ( )÷÷ ( )
b) f ´( x ) = 5 ççç ÷ ççç ÷÷÷ =
è 1- x 3 ø÷ ( 1 - x 3 2
) (1 - x 3 6
)
çè ø÷
2
c) f ´( x ) = 5 x 2 ( x 3 - 1)3
3
d) f ´( x) =8(1+2e ) e
x x

3
3 1 2 24 x ln3 ( x 2 - 1)
e) f ´( x ) = 4 ln ( x - 1) ⋅ ⋅ 3( x - 1) ⋅ 2 x =
3 2 2
3
( x 2 - 1) x 2 -1

654
Derivada d’una funció 1111

5
f) f ´( x) =-18e (1-3e )
x x

g) f ´( x ) = 4 x sin x 2 cos x 2 = 2 x sin(2 x 2 )

h) f ´( x ) = - 3 cos 2 ( x 2 - 7 x + 1)⋅ sin ( x 2 - 7 x + 1)⋅ ( 2 x - 7 )

i) f ´( x) = 6x2 tg( x3 -8) ⋅ éê1+tg2 ( x3 -8)ùú


ë û
2
3 æç 1 1 ÷ö æ 1 1 ÷ö
j) f ´( x) = çsin + cos ÷ çççsin - cos ÷
x 2 çè x x ÷ø è x x ÷ø

97. Aplica la regla de la cadena per determinar la funció derivada d’aquestes funcions:
a) ���� � ������� f) ���� � �������

b) ���� � √����� g) ���� � ����√�


c) ���� � ���� ��� h) ���� � ���� ���

d) ���� � �� � ��� �� i) ���� � ��� ��� � ��


e) ���� � ��������� j) ���� � ������� ��

   
 
98. Deriva les funcions següents:
� �
a) ���� � ��√���� f) ���� � ������� � ��
�� ��
b) ���� � ��� � � g) ���� � �������������� ���
���
�����
c) ���� � ���� � ���� �� h) ���� � ���
������

d) ���� � ����������� i) ���� � ���� � ������



e) ���� � �������� � ������ � j) ���� � √���� ���

a) f ´( x ) = e(
2
x +1) ( x + 1)
x
2
2x - 2x + 2
b) f ´( x ) =
x2 -1

æ x
1 ö
c) f ´( x ) = çç 2 e + 1 ÷÷
ççè ÷
x
2 2 e + log 2 3 x x ln 2 ø÷

655
Derivada d’una
Derivada d’una funció
funció 11

d) f ´( x ) = 2 éëln (ln x )ùû ⋅ 1 ⋅ 1


ln x x

e) f ´( x ) = 6 x cos x 2 + 4 sin x cos x


2
6 sin( x3 +1) ⋅( x3 +1) x2
f) f ´( x) = 2
cos2 ( x3 +1)

g) f ´( x) = 2x sin éêësin(tg x )ùúû cos(tg x )(1+tg x )


2 2 2 2

2x cos x2 cos 2x + 2 sin x2sin 2x


h) f ´( x) =
sin x2 cos 2x

i) f ´( x ) = 2e 2 x cos x 2 - 2 xe 2 x sin x 2

-2 x 2 sin x3
j) f ´( x ) =
3
cos x3

 
99. Aplica la regla de la cadena per calcular la derivada d’aquestes funcions:
૛࢞૝ ૜࢞૜ ࢞૛ ࢞ ૛
a) ࢌሺ࢞ሻ ൌ െ െ ൅ ൅
૞ ૛ ૜ ૛ ૜

ି૜
b) ࢌሺ࢞ሻ ൌ ට࢙࢏࢔૛ ࢞ െ ࢉ࢕࢙૛ ሺ૜࢞ െ ૚ሻ ൅ ࢚ࢍ
࢞૛ ା૛

ඥ૝࢞૛ ା૚૙࢞ି૚
c) ࢌሺ࢞ሻ ൌ ࢚ࢍ૝ ൬ ൰
࢞ି૚૟

࢙࢏࢔૛ ࢞ାࢉ࢕࢙૛ ࢞
d) ࢌሺ࢞ሻ ൌ ࢒࢔ ቀ ൅ ࢚ࢍ૛ ࢞ቁ
࢙࢏࢔૛ ࢞ିࢉ࢕࢙૛ ࢞

૜ ࢞૛ ା૝
ඥି૜࢞૛ ା૚૙࢞ି૚
e) ࢌሺ࢞ሻ ൌ ൬ ൰
ඥ࢞૝ ି૝
4
æ 4 3 2 ö æ 3 2 ö
a) f ´( x ) = 5 çç 2 x - 3 x - x + x + 2 ÷÷÷ çç 8 x - 9 x - 2 x + 1 ÷÷÷
çè 5 2 3 2 3ø è 5÷ ç 2 3 2 ø÷

é ù
ê æ 2 æ -3 ÷ öö 1 ú
ê3 sin x cos x + 6 cos (3 x -1) sin (3 x -1) + 6 x çç1 + tg ççç 2 ÷÷÷÷÷÷
2
ú
ê èç è x + 2 øø ( x 2 + 2)2 ú
êë úû
b) f ´( x) =
-3
2 sin3 x - cos 2 (3 x -1) + tg 2
x +2

4x + 5
æ 4 x 2 + 10 x - 1÷öæç æ 4 x 2 + 10 x - 1÷ö÷ö 2
( x - 16) - 4 x 2 + 10 x - 1
ç ÷÷çç1+ tg 2 çç ÷÷÷÷ ⋅ 4 x + 10 x - 1
c) f ´( x ) = 4 tg 3 ççç ÷ç ç ÷÷
èçç ø÷ø÷÷
2
çè x - 16 ø÷èç x - 16 ( x - 16)

æ ö÷
1 çç -2 sin 2x 2 ÷
f ´( x) = ⋅çç +1+ tg x÷÷÷
d) 1 çç( sin2 x - cos2 x)2 ÷
ø÷
2
+ tg x è
sin x -cos2 x
2

æ1 1 ö
e) lnf ( x ) = ( x + 4)ççç ln(-3 x + 10 x - 1) - ln( x - 4)÷÷÷
2 2 4

è3 2 ø

656
Derivada d’una funció 1111

f ´( x ) æ3 2 ö é 4 x 3 úù
ç -3 x + 10 x -1÷÷ ê -6 x + 10
= 2 x lnçç ÷
÷÷ + ( x 2
+ 4) ê - ú
f ( x) ççè êë 3(-3 x + 10 x -1) 2( x -1)úû
2 4
x4 - 4 ø
2
æ æ 3 -3 x 2 + 10 x - 1ö÷ é ùö x +4
ç
ç ç ÷ ê -6 x + 10 2 x 3 ú÷÷ æçç 3 -3 x 2 + 10 x - 1ö÷÷
f ´( x ) = çç2 x lnç ç ÷ + ( x + 4) ê ú÷÷÷ ⋅ çç
2
- 4 ÷
çè ççè x4 - 4 ÷ø÷ êë 3 (-3 x + 10 x - 1) ( x - 1)ú÷
2
ç x4 - 4 ÷ø÷
ûø è

100. Determina els coeficients i els exponents desconeguts perquè es verifiqui que les funcions i les
seves derivades es corresponen.
��
a) f(x) = x3 + ax2 + bx + 6 c) ���� �
��
��� �
f’(x) = 3x2 + 4x – 3 �� ��� � �� � � �
� ��

b) g(x) = a ln x + bx d) ���� � �
√�
� �
g’(x) = �� �� ��� � �
� � √�

a) a = 2, b = - 3 c) a = 2, b = 1
b) a = 3, b = - 5 d) b = 3
 
101. Deriva les funcions trigonomètriques inverses següents:
��
a) f(x) = arc sin x2 d) f(x) = arc sin
���
b) f(x) = arc cos (2x – 1)2 e) f(x) = arc cos e2x
c) f(x) = arc tg √� f) f(x) = arc tg ln x
1 -2
a) f ´( x ) =
2x d) f ´( x ) = ⋅ 2
1- x 4 4x 2
( x - 1)
1- 2
( x - 1)

- 4 (2 x - 1) -2e2 x
b) f ´( x ) = e) f ´( x ) =
1 - (2 x - 1)
4
1- e4 x

1 1 1
c) f ´( x ) = ⋅ f) f ´( x ) =
2 x 1+ x x (1+ ln2 x )

 
102. Calcula les derivades de les funcions següents:
a) f(x) = (x2 + 1)3x e) f(x) = cos3x2x
� ��
b) f(x) x3x2 7x 1 f) f(x) = � √��� � ���
c) f(x) = (3x2 + 1)ln x g) f(x) = (–x10 + 3x5 – 1)sin x
d) f(x) = sinx x2 h) f(x) = e –5x3+ 4x – 1
a) ln f ( x ) = 3 x ln ( x 2 + 1)

f ´( x ) 6x2
= 3 ln( x 2 + 1) + 2
f (x) x +1

657
Derivada d’una
Derivada d’una funció
funció 11

æ 6 x 2 ÷ö 2
f ´( x ) = çç3 ln( x 2 + 1) + 2
3x
÷÷( x + 1)
çè x + 1ø ÷

b) ln f ( x ) = (3 x 2 - 7 x - 1) ln x

f ´( x ) 3x2 - 7x -1
= (6 x - 7) ln x +
f ( x) x

æ 3 x 2 - 7 x - 1ö÷ 3 x 2 -7 x -1
f ´( x ) = çç(6 x - 7) ln x + ÷÷÷ x
çè x ø

c) ln f ( x ) = ln x ln (3 x 2 + 1)

f ´( x ) ln (3 x + 1)
2
6x
= + ln x ⋅
f (x) x 3x2 + 1

æ ö
çln(3 x + 1) 6 x ln x ÷÷ 2
2
ln x
f ´( x) = çç + 2 ÷÷(3 x + 1)
çç x 3 x + 1 ÷
è ø÷

d) ln f ( x ) = x ln ( sin x 2 )

f ´( x) 2 x 2 cos x 2
= ln ( sin x 2 ) +
f ( x) sin x 2

é 2 x2 cos x 2 ùú x 2
f ´( x) = êln ( sinx 2 ) + sin x
ê sinx 2 úû
ë
e) ln f ( x ) = 3 x ln (cos 2 x )

f ´( x) sin 2 x
= 3 ln (cos 2 x) + 6 x
f ( x) cos 2 x

é sin 2 x ùú
f ´( x) = ê3 ln (cos 2 x) + 6 x cos 3 x 2 x
ê cos 2 x úû
ë
x2
f) ln f ( x ) = ln (- x 3 - 15 )
5

f ´( x ) 2 x 3x4
= ln(- x 3 - 15) +
f (x) 5 5 ( x 3 + 15)

æ 3 x 4 ÷÷ö 5 3
ç2x x2
f ´( x ) = çç ln(-x 3 -15) +
ççè 5
÷÷ -x -15
5( x 3 + 15)÷÷ø
( )
g) ln f ( x ) = sin x ln (- x 10 + 3 x 5 - 1)

f ´( x ) (-10 x9 + 15 x 4 ) sin x
= cos x ln (- x10 + 3 x 5 -1) +
f ( x) - x10 + 3 x 5 -1

æ
ç (-10x9 +15x4 ) sin x ö÷÷ 10 5 sin x
f ´( x) = ççcos x ln (-x10 + 3x5 -1) + ÷(-x + 3x -1)
ççè -x10 + 3x5 -1 ÷÷÷ø

658
Derivada d’una funció 1111

h) ln f ( x ) = -5 x 3 + 4 x - 1

f ´( x )
= -15 x 2 + 4
f (x)

f ´( x ) = (- 15 x 2 + 4 ) e - 5 x
3
+ 4 x -1

 
103. Calcula a, b i c en la funció f(x) = ax2 + bx + c, sabent que la seva gràfica passa per (0, –3) i (2, 5), i la
recta tangent en x = –1 és horitzontal.
f (0) = c = -3 f (2 ) = 4 a + 2 b + c = 5

f ´( x ) = 2 ax + b f ´(- 1) = - 2 a + b

c = -3ü
ï
ï
ï
4a + 2b + c = 5 ï
ý  a = 1, b = 2, c = -3
ï
-2a + b = 0 ïï
ï
þ

104. Determina quina és l’equació d’una paràbola que passa pel punt (0, 9) i en el punt (2, 9) té com a
recta tangent y – 6x + 3 = 0?
Sigui f(x) = ax2 + bx + c.

Com que la paràbola passa pel punt (0, 9) → c = 9

I com que també passa pel punt (2, 9) → 4a + 2b + 9 = 9 → 4a + 2b = 0 → b = –2a

Així, resulta que: f(x) = ax2 – 2ax + 9 → f’(x) = 2ax – 2a

Si y = 6x – 3 és la tangent en el punt x = 2, aleshores:

f’(2) = 6 → 4a – 2a = 6 → a = 3

L’equació de la paràbola és: f(x) = 3x2 – 6x + 9

105. Calcula els valors de a, b i c perquè la funció f(x) = x3 + ax2 + bx + c passi per l’origen de
coordenades, la seva recta tangent en x = 1 tingui pendent 3 i la segona derivada en x = –1 sigui
nul·la.
f (0 ) = c = 0  

f ´( x ) = 3 x 2 + 2 ax + b       f ´(1) = 3 + 2 a + b = 3  

f ´´( x ) = 6 x + 2 a           f ´´(- 1) = - 6 + 2 a = 0  

ü
c = 0ï
ï
ï
3 + 2a + b = 3ï
ý  a = 3, b =-6, c = 0  
ï
-6 + 2a = 0ïï
ï
þ

106. Determina els valors de a, b i c perquè la funció f(x) = x3 + ax2 + bx + c passi per (3, 0) i les rectes
tangents a la seva gràfica en x = 2 i x = 4 siguin paral·leles a l’eix X.
f (3 ) = 27 + 9 a + 3 b + c = 0  

f ´( x ) = 3 x 2 + 2 ax + b       f ´(2 ) = 12 + 4 a + b = 0       f ´(4 ) = 48 + 8 a + b = 0  

659
Derivada d’una
Derivada d’una funció
funció 11

27 + 9a + 3b + c = 0ü
ï
ï
ï
12 + 4a + b = 0ï
ý  a = -9, b = 24, c = -18  
ï
48 + 8a + b = 0ï
ï
ï
þ
 
107. Troba a, b i c en la funció f(x) = ax4 + bx + c, sabent que la seva gràfica passa per (1, –1), la recta
tangent en x = 1 és horitzontal i la recta tangent en x = 0 és paral·lela a la recta y = 4x.
f (1) = a + b + c = - 1

f ´( x ) = 4 ax 3 + b f ´(1) = 4 a + b = 0

El pendent de la recta y = 4x és 4, aleshores:


f ´(0 ) = b = 4

a + b + c = -1ïüï
ï
4a + cb = 0 ïý  a = -1, b = 4, c = -4
ï
b = 4 ïïïþ

 
108. Determina els valors de a, b i c perquè la funció f(x) = 2x3 + ax2 + bx + c passi per (1, 6) i les rectes
tangents a la gràfica en x = 1 i x = 2 siguin horitzontals.
f (1) = 2 + a + b + c = 6  

f ´( x ) = 6 x 2 + 2 ax + b       f ´(1) = 6 + 2 a + b = 0      f ´(2) = 24 + 4 a + b = 0  

2 + a + b + c = 6ü
ï
ï
ï
6 + 2a + b = 0ï
ý  a = -9, b = 12, c = 1  
ï
24 + 4a + b = 0ïï
ï
þ

 
109. Justifica si existeix algun punt en el qual la tangent a la gràfica de la corba f(x) sigui horitzontal.
࢞ି૛ ૜࢞
a) ࢌሺ࢞ሻ ൌ c) ࢌሺ࢞ሻ ൌ
࢞ି૜ ࢞ି૜
࢞૛ ି૛ ି࢞
b) ࢌሺ࢞ሻ ൌ d) ࢌሺ࢞ሻ ൌ
࢞ି૜ ࢞૛ ା૚

1 9
a) f ´( x ) =- 2
¹0 c) f ´( x ) =- 2
¹0
( x - 3) ( x - 3)

x2 - 6x + 2 6  28 2
b) f ´( x ) = 2
=0 x=  x1 = 3 + 7, x 2 = 3 - 7 d) f ´( x ) = x - 1 2 = 0  x = 1
( x - 3) 2 (x 2
+ 1)

110. Indica si alguna de les rectes tangents de les funcions següents són paral·leles a la recta
r: y – 2x + 3 = 0.
a) f(x) = x3 – x + 7 c) f(x) = ln x2

b) f(x) = x2 + 4x + 3 d) f(x) =
࢞ା૜
El pendent de la recta r és 2. Per tant, perquè una recta tangent a f(x) sigui paral·lela a r hem de trobar la
solució de l’equació f´(x) = 2.

660
Derivada d’una funció 1111

a) f ´( x ) = 3 x 2 - 1 = 2  x = 1 c) f ´( x ) = 2 = 2  x = 1
x

-1
b) f ´( x ) = 2 x + 4 = 2  x = -1 d) f ´(x) = 2
= 2  Sense solució.
( x +3)

111. Determina en cada cas els punts en els quals la tangent a la corba f(x) és paral·lela a la bisectriu dels
quadrants primer i tercer.
a) f(x) = x2 – 3x + 1 c) f(x) = x3 – x2

b) f (x) = d) f (x) = x ln x
૚ି࢞
La bisectriu del primer i tercer quadrants compleix amb l’equació y = x i el seu pendent és 1, per tant f´(x) =
1.

a) f ´( x ) = 2 x - 3 = 1  x = 2 c) f ´( x ) = 3 x 2 - 2 x = 1  x1 = - 1 , x 2 = 1
3

1
b) f ´( x ) = 2
= 1  x1 = 0, x2 = 2 d) f '( x ) = ln x + 1 = 1  x = 1
(1- x )

 
112. Troba en cada cas els punts en els quals la tangent a la corba f (x) és paral·lela a la bisectriu dels
quadrants segon i quart.

a) f(x) = c) f(x) = 2x3 – 3x2 + 5


b) f(x) = x3 + 2x2 d) f(x) =
૚ି૛࢞
La bisectriu dels quadrants segon i quart compleix l’equació y = –x i el seu pendent és –1, per tant f´(x) =–1.

a) f ´( x ) = - 12 = - 1  x =  1 c) f ´( x ) = 6 x 2 - 6 x = - 1  x =
3 3
x 6

-2 1 2
b) f ´( x ) = 3 x 2 + 4 x = -1  x1 = - 1 , x 2 = -1 d) f ´( x ) = 2
= -1  x =
3 (2 x - 1) 2

 
113. Considera la funció f(x) = ln x. Calcula els punts de la corba en els quals la recta tangent té el mateix
pendent que les rectes següents:
a) r: y – x = 2 c) t: y + x – 1 = 0
b) s: 4y = 3 – x d) u: 2y – x = –4
1
f ´( x ) =
x

a) r : y - x = 2  y = x + 2  El pendent de la recta r és 1.
1
= 1 x = 1
x

b) s : 4 y = 3 - x  y = 3 - 1 x  El pendent de la recta s és - 1 .
4 4 4

1 1
= -  x = -4
x 4

c) t : y + x - 1 = 0  y = - x + 1  El pendent de la recta t és –1.

661
Derivada d’una
Derivada d’una funció
funció 11

1
= -1  x = -1
x

d) u : 2 y - x = - 4  y = - 2 + 1 x  El pendent de la recta u és 1 .
2 2

1 1
=  x=2
x 2

114. Considera la funció ࢌሺ࢞ሻ ൌ
࢞ା૜
a) Escriu les equacions de les rectes tangents a la gràfica de la funció el pendent de les quals sigui

െ .

b) Indica si la gràfica de f(x) és tangent en algun punt a la recta y + 2x + 2 = 0. Digues si aquest punt
és únic.
2
f ´( x) =- 2
( x + 3)
 
2 1
a) - = -  x 2 + 6 x + 5 = 0  x1 = -1, x2 = -5
2
( x + 3) 2

1 1 1
f (-1) = 1 y - 1 = - ( x + 1)  y = - x +
2 2 2

1 1 7
f (- 5) = -1 y + 1 = - ( x + 5)  y = - x -
2 2 2

b) y + 2x + 2 = 0  y =-2x - 2  El pendent de la recta és –2.


2 2
- 2
= -2  ( x + 3) = 1 x1 = -4, x2 = -2
( x + 3)

Existeixen dos punts on la gràfica és tangent a la recta y + 2 x + 2 = 0 .

115. La recta que té d’equació y = 9x – 14 és tangent a la funció y = x3 – 3x + k. Determina en quin punt


són tangents i troba el valor de k.
Hi ha una sola solució? La funció té dos punts en els quals la tangent és horitzontal. Calcula’ls i
escriu l’equació d’aquestes rectes.
f’(x) = 3x2 – 3.

Quan la recta és tangent: 3x2 - 3 =9 → x2 = 4 → x=േ2

Si x = 2 → y – (2 + k) = 9(x – 2) → y = 9x – 16 + k → k = 2

Si x = –2 → y – (–2 + k) = 9(x + 2) → y = 9x + 16 + k → k = –2

Així, resulta que hi ha dues solucions

Quan la tangent és horitzontal, es compleix que: 3x2 – 3 = 0 → x2 = 1 → x = േ1

Si x = 1 → y – (–2 + k) = 0·(x – 1) → y = – 2 + k

Si x = –1 → y – (2 + k) = 0·(x + 1) → y = 2 + k

662
Derivada d’una funció 1111

116. Indica en quines de les parelles de funcions següents les seves gràfiques són tangents en algun
punt:
a) f(x) = x2 + 3x + 2 c) f(x) = ln x
g(x) = 3x + 2 g(x) = x – 1

b) f(x) = ex d) f(x) = cos x


g(x) = x g(x) = x2 + 1

ü 1 üïï
f ´( x ) = 2 x + 3ï f ´( x ) = ï 1
a) ï  2x + 3 = 3  x = 0
ý c) x ý  = 1 x = 1
g ´( x ) = 3 ï
ï ïï x
þ g ´( x ) = 1 ïïþ

A més, f(0) = g(0) = 2. A més, f(1) = g(1) = 0.


Les seves gràfiques són tangents en el punt x = 0. Les seves gràfiques són tangents al punt x = 1.

f ´( x ) = e x ü
ï f ´( x) = -sin xüïï
b) ï x
ý  e = 1 x = 0 d) ý  -sin x = 2 x  x = 0
g ´( x ) = 1 ï ï
þ
g´( x) = 2 x ïï
þ
Com que f (0 ) = 1 ¹ g (0 ) = 0 . A més, f(0) = g(0) = 1.

Les seves gràfiques no són tangents a cap punt. Les seves gràfiques són tangents al punt x = 0.
 
117. Calcula l’expressió algèbrica d’una funció que passa per (2, 5) sabent que la seva derivada és:
f'(x) = 2x2 + 6x – 3

Si f’(x) = 2x2 + 6x – 3 → f(x) = x3 + 3x2 – 3x + k

Com que la funció passa pel punt (2, 5) → f(2)=5


ଶ ଵଽ
· 8 + 12 – 6 + k = 5 → k = –
ଷ ଷ
ଶ 3 ଵଽ
f(x) = x + 3x2 – 3x –
ଷ ଷ

 
118. La recta tangent a una funció f(x) en el punt d’abscissa x = 2 és y = 5x – 7. Calcula el valor de la
funció i de la seva derivada en el punt d’abscissa 2.
La de la recta és 5, per tant f ´(2 ) = 5 .

Com que la recta és tangent a f en x = 2  f (2) = 5 ⋅ 2 - 7 = 3

119. La recta y = ax + b passa per (1, 6) i (2, 8) i és tangent a la corba g(x) en x = 0. Calcula el valor de g(0)
i g'(0).
6= a+b ü ï
ï
ý  a = 2, b = 4
8 = 2a + bï
ï
þ

L’equació de la recta és y = 2x + 4.

g (0 ) = y (0 ) = 4 g ´(0 ) = y ´(0 ) = 2

663
Derivada d’una
Derivada d’una funció
funció 11

� ��
120. La funció derivada d’una paràbola és una recta que passa pels punts ��, � i ���, � �.
� �

Calcula l’abscissa del vèrtex d’aquesta paràbola.


Atès que l’equació d’una paràbola és y = ax2 + bx + c, la seva derivada és y’ = 2ax + b.
L’equació de la recta que passa pels punts és:

��1 �� 5

�� � →�� 3� �
�2 �� 2
Igualem coeficients, resultant:

2ax = 3x → a =


b=�


� � �
L’abscissa del vèrtex és x = � ��� �
� �
�� �� �

 
121. Si tracem la recta tangent i la normal a la funció f(x) = x3 – 12x2 + 42x – 40, en el punt (3, 5) es forma,
amb els semieixos positius de coordenades, un quadrilàter. Determina’n l’àrea.
f’(x) = 3x2 – 24x + 42

f’(3) = –3

L’equació de la recta tangent en (3, 5) és:

y – 5 = –3(x – 3) → y = –3x + 14

I l’equació de la recta normal és:

� �
y–5= (x – 3) → y = x +4
� �

14
El quadrilàter té els vèrtex: (0, 0), (0, 4), (3, 5) i �
3
, 0�.

Per calcular la seva àrea el descomponem en tres parts:

 El rectangle de vèrtex: (0, 0), (4, 0), (3, 4) i (3, 0) i la seva mida és 14 u2.

 El triangle de vèrtex: (4, 0), (3, 4) i (3, 5) i la seva mida és u2.

�� ��
 El triangle de vèrtex: (3, 5), (3, 0) i � , 0� i la seva mida és u2.
� �
� �� ��
Així doncs l’àrea del quadrilàter és: 12 + + = u2.
� � �

664
Derivada d’una funció 1111

 
 
122. Considera la corba f1(x) = √� � �i la recta d’equació f2(x) = ax. Calcula el valor de a perquè la recta
tangent a f1(x) sigui:
a) Perpendicular a f2(x) en x = 1.
b) Paral·lela a f2(x) en x = –4.
1
f1 ´( x ) = -     f2 ´( x ) = a
2 5- x  

a) f1 ´(1) = - 1 f1 (1) = 2
1 1
y - 2 = - ( x - 1)  y = - x +
9
→ -1=-1 a=4
4 4 4 4 4 a

b) f1 ´(- 4 ) = - 1 f1 (- 4 ) = 3
1 1
y - 3 = - ( x + 4)  y = - x +
7
→ a=-1
6 6 6 3 6

 
123. Determina quant ha de valer a perquè la funció f(x) = x ln x – ax tingui, en el punt d’abscissa e, una
recta tangent paral·lela a la bisectriu del primer quadrant.
La bisectriu del primer quadrant és y = x
Aquesta recta i la recta tangent són paral·leles si els seus pendents són iguals.
El pendent de la recta tangent a la funció, a x = e és:

f’(x) = ln x + x· – a = ln x + 1 – a → f’(e) = 2 – a

Aleshores, tenim que: 2 – a = 1 → a = 1


 
124. Considera la circumferència d’equació x2 + y2 = 20. Escriu l’equació de la recta tangent en el punt
d’abscissa x = 4.
Considerem l’arrel positiva: y = 20 - x 2

-x
y´( x) = y (4 ) = 2 y ´( 4 ) = - 2
20 - x2

y - 2 = - 2 ( x - 4 )  y = - 2 x + 10

125. Considera la funció f(x) = x2 – 4x + 6.


a) Calcula les equacions de les rectes tangents a la seva gràfica paral·leles a les bisectrius dels
quadrants.
b) Calcula el punt de tall entre elles i el de cada una amb l’eix de coordenades X.
c) Calcula l’àrea del triangle que té de vèrtexs els punts que has trobat abans.

a) f ´( x ) = 2 x - 4

665
Derivada d’una
Derivada d’una funció
funció 11

La bisectriu dels quadrants primer i tercer és y = x i té pendent 1.


æ 5ö 9
2x - 4 = 1 x =
5 f çç ÷÷÷ =
2 çè 2ø 4

Anomenem r a la recta paral·lela a la bisectriu dels quadrants primer i tercer:


9 5 1
r: y - = x-  y = x-
4 2 4

La bisectriu dels quadrants segon i quart és y = - x i té pendent -1.


æ 3ö 9
2 x - 4 = -1  x =
3 f çç ÷÷÷ =
2 çè 2ø 4

Anomenem s a la recta paral·lela a la bisectriu dels quadrants segon i quart: 


9 æ 3ö 15
s: y - = -çç x - ÷÷÷  y =-x +
4 ç
è 2ø 4

b) El punt de tall entre les dues rectes, r i s, és:



ï
y = x- ï
ï
4ïý  x = 2, y = 7  P æç2, 7 ÷ö÷
ï
ççè
15ï 4 4 ø÷
y = -x + ï ï
ï
4ïþ
1 1 æ1 ö
El punt de tall de la recta r amb l’eix X és: y =0= x-  x =  P ççç , 0 ÷÷÷
4 4 è4 ø

15 15 æ 15 ö
El punt de tall de la recta s amb l’eix X és: y = 0 = -x +  x=  P ççç , 0 ÷÷÷
4 4 è 4 ø

b⋅h 7 2
c) La base mesura 15 - 1 = 7 i la altura mesura 2. ÀreaT = = u
4 4 2 2 2

126. Considera la funció f(x) = x3 + 3x2 + 4.


a) Calcula els punts en els quals la gràfica de la funció té recta tangent horitzontal.
b) Escriu l’equació de la recta que passa per aquests punts.
c) Calcula els punts de la corba que tenen per recta tangent una paral·lela a aquesta recta.
a) f ´( x ) = 3 x 2 + 6 x = 0  x 1 = 0, x 2 = - 2 f (0 ) = 4 f (-2) = 8

Els punts són (0, 4) i (-2, 8).


b) Si l’equació de la recta és del tipus y = mx + n, tenim:
4=n ü
ï
ï
ý  m = -2, n = 4
ï
8 = -2m + nï
þ

L’equació de la recta és: y = -2x + 4

c) El pendent de la recta és -2: 3 3


f ´( x ) = 3 x 2 + 6 x = - 2  x 1 = - 1 + , x2 = -1-
3 3

૛࢞ିૢ
127. Considera la funció ࢌሺ࢞ሻ ൌ
࢞ି૜

a) Calcula els punts de tall d’aquesta funció amb els eixos de coordenades.
b) Escriu l’equació de la recta tangent a la corba en cadascun d’aquests punts.
c) Troba l’àrea dels triangles que determinen aquestes rectes amb els eixos de coordenades.

666
Derivada d’una funció 1111

3
f ´( x ) = 2
( x - 3)
 
a) Tall amb l’eix X: 2 x - 9 = 0  x = 9 Tall amb l’eix Y: 2 ⋅ 0 - 9 = 3
x-3 2 0-3

æ 9ö 4
b) f ´ççç ÷÷÷ = y=
4
x -6 f ´(0 ) =
1
y=
1
x +3
è 2ø 3 3 3 3

c) Calculem els punts de tall amb els eixos de la recta y = 4 x - 6 :


3

Tall amb l’eix Y: x = 0  y = -6 Tall amb l’eix X: y = 0  x = 9


2

b⋅h 9⋅6 27 2
Àrea del triangle que formen: AT = = = u
2 4 2

1
Calculem els punts de tall amb els eixos de la recta y = x + 3 :
3
Tall amb l’eix Y: x = 0  y = 3 Tall amb l’eix X: y = 0  x = -9
b⋅h 3⋅9 27 2
Àrea del triangle que formen: AT = = = u
2 2 2

128. Llancem verticalment una pilota cap amunt amb una velocitat inicial de 49 m/s des de la part superior
d’un edifici de 39 m d’altura. L’altura de la pilota h(t) sobre el terra després de t segons està
determinada per h(t) = 39 + 49t – 4,9t2. Calcula la velocitat mitjana de la pilota en cadascun dels
intervals següents:
a) [0, 1] b) [4, 6] c) [11, 13]
Què podem deduir sobre el moviment de la pilota?
h (1) - h (0 ) 83,1 - 39 h (6 ) - h (4 ) 156, 6 - 156, 6 h (13 ) - h (11) 0 - 0
a) = = 44,1 m /s b) = = 0 m /s c) = = 0 m /s
1- 0 1 6-4 2 13 - 11 2

En el primer interval la pilota es troba pujant i, per tant, la velocitat mitjana és positiva; en el segon interval la
pilota recorre el mateix tram cap amunt i cap avall; en l’últim interval la pilota es troba a terra i no es mou,
amb velocitat mitjana zero.

129. L’espai que recorre un objecte, en metres i en un temps t, s’expressa amb aquesta fórmula:
e(t) = 4t2 + 2t + 1
a) Quin espai ha recorregut al cap de 4 segons? I al cap de 7 segons?
b) Quina és la velocitat mitjana que ha mantingut entre els 4 i els 7 segons?
a) Als 4 segons: e = 73 m Als 7 segons: e = 211 m
ଶଵଵି଻ଷ
b) T.V.M.([4, 7]) = = 46 m/s
଻ିସ

667
Derivada d’una
Derivada d’una funció
funció 11

130. L’espai, en metres, que recorre un mòbil considerant el temps, en segons, està descrit per
l’expressió:

e(t) = t2 + t

Calcula la velocitat instantània del mòbil al cap de 3 segons.

f’(t) = t + 1 → f’(3) = 5

La velocitat instantània del mòbil als 3 segons és de 5 m/s.

   

668
Derivada d’una funció 1111

AMPLIA 
 
131. Escull la resposta adequada.

□ f ´( x ) = cos x Es defineix entre els valors -1 i 1, per tant la inclinació més gran de la funció és 1.

□ f ´( x ) = 3 x 2 - 4 x f ´(1) = -1 La recta tangent és: y = - ( x - 1) = - x + 1

x 3 - 2 x 2 + 1 = - x + 1  x 3 - 2 x 2 + x = 0  x 1 = 0, x 2 = x 3 = 1 Talla també en el punt (0, 1).

□ f ´( x ) = 2 x - 3 = 3  x = 3  y = 1 y - 1 = 3 ( x - 3)  y = 3 x - 8

La recta tangent és y = 3x - 8.

□ f ´( x ) = - 12 f ´´( x ) =
2
f ´´´( x ) = -
6
f iv ( x ) =
24
x x3 x4 x5

Per tant: f n ( x ) = (- 1) n ⋅ nn +! 1
x

132. Calcula l’expressió algèbrica d’una funció que passa per (2, 5) sabent que la seva derivada és:
f'(x) = 2x2 + 6x – 3

Si f’(x) = 2x2 + 6x – 3 → f(x) = x3 + 3x2 – 3x + k

Atès que la funció passa pel punt (2, 5) → f(2) = 5


� ��
· 8 + 12 – 6 + k = 5 → k = �
� �
� ��
f(x) = x3 + 3x2 – 3x �
� �

133. Si f (x) i g(x) són funcions inverses, és a dir, (g º f )(x) = x, es verifica que (g' º f ' )(x) = x?

No es verifica. Si es consideren les funcions f(x) = x3 i g(x) = √� , s’obté que són inverses atès que
compleixen que: (g º f)(x) = x
� �
No obstant això, tenim que: (g’ º f’)(x) = g’(f’(x)) = g’(3x2) = � � � ��
� ���� � �� �� √��

Per tant f’(x) i g’(x) no són funcions inverses.

669
Derivada d’una
Derivada d’una funció
funció 11

��� �
134. Considera ���� � ��� �� . Estudia si f(x) i f'(x) són constants.
����� �

� ��� ������� ������� � ��� ��� ��� ���


f’(x) = � � � ������ ��� � �
���
��� �
� ������ ��� ����� � ������ ������ ������ �
����� �

��� ��� �

���� ��� �

Atès que f’(x) és constant i no nul·la, la funció f(x) no és constant.

135. Verifica que si un polinomi té una arrel doble, també ho és de la seva derivada.
Resol l’equació 12 x3 – 16x2 + 7x – 1 = 0 sabent que una de les seves arrels és doble.
Si un polinomi té una arrel doble a, aleshores: f(x) = (x – a)2·p(x)
f’(x) = 2(x – a) · p(x) + (x – a)2 · p’(x) = (x – a)[2p(x) + (x – a) · p’(x)]
Per tant, a també és una arrel de la derivada.
Sigui f’(x) = 12x3 – 16x2 + 7x – 1

� �

Com que f’’(x) = 36x2 – 32x + 7, tenim que: 36x2 – 32x + 7 = 0 � �
��
��

I atès que una de les arrels és doble coincideix amb una de les anteriors:
� �
� � � � � → 12x3 – 16x2 + 7x – 1 = �� � � · (12x2 – 10x + 2)
� �

� �

12x2 – 10x + 2 = 0 → �
��

� �
Les solucions de l’equació són: doble i .
� �

 
136. Demostra que la recta tangent a una circumferència en un punt és perpendicular al radi de la
circumferència en aquest punt.

 
 
 
 
 
Sigui una circumferència amb centre a l’origen de coordenades, i de radi r: x2 + y2 = r2
�� �
Si � � √� � � � � → � � � ��
��� � �� � �� � �� �

Aleshores, l’equació de la recta tangent en un punt (a,b) és: � � � � � �� � ��

La recta determinada pel radi de la circumferència que passa per aquest punt és:
670
Derivada d’una funció 1111

‫ݕ ݔ‬ ܾ
ൌ ՜‫ݕ‬ൌ ‫ݔ‬
ܽ ܾ ܽ
௕ ଵ
Les rectes són perpendiculars atès que: ൌെ ೌ
௔ ି

 
137. Demostra que una funció que no és contínua en un punt no pot ser derivable en aquest punt.

Sigui una funció f(x) que no és contínua en x = x0.  xlimf


x
( x) ¹ f ( x0 )
0

Si la funció és derivable en x = x0, aleshores, existeix el límit:


f ( x 0 + h)- f ( x 0 )
lim = l  lim (f ( x 0 + h ) - f ( x 0 )) = l ⋅ lim h  lim (f ( x 0 + h ) - f ( x 0 )) = 0  lim f ( x 0 + h ) = lim f ( x 0 )
h 0 h h 0 h 0 h 0 h 0 h 0

Això no és cert perquè la funció no és contínua en x = x0, i la funció no pot ser derivable en aquest punt.
 
138. Per calcular la derivada de la funció implícita has de suposar que y és una funció derivable respecte
de x i aplicar la regla de la cadena.
Utilitza aquesta tècnica per calcular la derivada de les funcions implícites que tens a continuació:
a) y3 – 2xy2 = 3x3y
b) 3xy + y3 = 2x
a) 3 y 2 y ´-2 y 2 - 2 x 2 yy ´= 9 x 2 y + 3 x 3 y ´ b) 3 y + 3 xy ´+3 y 2 y ´= 2
y ´ (3 y 2 - 4 x y - 3 x 3 ) = 9 x 2 y + 2 y 2 y ´ (3 x + 3 y 2 ) = 2 - 3 y

9 x 2 y + 2y 2 2 - 3y
y ´= y ´=
3y 2 - 4 xy - 3 x 3 3 x + 3y 2

૚ ૚ ૚
139. Demostra que l’equació de la recta tangent a la corba ࢞૛ ൅ ࢟૛ ൌ ࢇ૛ , en què a és un nombre real
positiu, en el punt P (x0, y0), es pot escriure de la manera següent:

 
Derivem implícitament:
1 1 1 y0
+ y ´= 0  y ´=- ⋅2 y y ´( x 0 ) = -
2 x 2 y 2 x x0

Calculem la recta tangent que passa pel punt P(x0, y0) :

y0 y - y0 x - x0 x y y x
y - y0 =- ( x - x0 )  + =0 1 + 1 = 0 + 0
x0 y0 x0 y0 x0
x2 y2
1
y0 x
Comprovem que + 0 = a2 :
y0 x0

y0 x y
+ 0 = 0
x0 + x0 y0
=
(y 0 x0 + x0 y0 ) x0 y0
= x0 + y0 = a2
1

y0 x0 x0 y0 x0 y0

671
Derivada d’una
Derivada d’una funció
funció 11

MATEMÀTIQUES A LA TEVA VIDA 
1. Què és el cost marginal de la producció del qual es parla en el text?
El cost marginal és la derivada del cost total de producció respecte de la producció.

2. Explica amb quin sentit es fa servir el terme entrada en el text anterior.


Els insums o inputs són tots els elements necessaris per produir un bé.

3. Per què el cost marginal de producció es pot considerar com una derivada?
Perquè mesura la taxa de variació del cost entre la variació de la producció.

 
4. Si tenim en compte la funció de cost, quin signe et sembla que deu tenir el paràmetre a?
Positiu.

 
5. Calcula la funció del cost marginal en l’exemple de la manufactura de medicaments si a i b són
nombres reals.
Funció cost: f(x) = 3ax2 + 2bx

6. Quin tipus de funció és el cost marginal?


És una funció quadràtica amb representació del tipus paràbola còncava; considerant que en l’eix
d’abscisses es representa la producció i en l’eix d’ordenades els costos.

672
12
Aplicacions de la derivada.
Aplicacions de la derivada. 
Representació de funcions
Representació de funcions  12  

ACTIVITATS 
1. Dibuixa la gràfica d’una funció creixent en (–∞, –2] ∪ (4,∞] i decreixent en la resta.
Resposta oberta. Per exemple:

 
 
2. Dibuixa la gràfica d’una funció amb dos màxims i un mínim, i cinc punts de tall amb els eixos X i Y.
Resposta oberta. Per exemple:

 
 
3. Dibuixa la gràfica de f(x) = x2. A partir d’aquest gràfica, troba els intervals de creixement i
decreixement de les següents:
a) f(x) = (x – 1)2 c) f(x) = x2 + 1
b) f(x) = (x + 2)2 d) f(x) = x2 – 3
f(x) = x2 decreix en (–∞, 0) i creix en (0, +∞).

673
Aplicacions de la
Aplicacions de la derivada.
derivada. Representació
Representacióde
defuncions
funcions 12

a) f(x) = (x – 1)2

En aquest cas és tracta d’una translació de la funció original una unitat a la dreta, l’interval de
decreixement en aquest cas serà (–∞, 1) i el de creixement serà (1, +∞).

b) f(x) = (x + 2)2

En aquest cas és tracta d’una translació de la funció original dues unitats a l’esquerra, per la qual cosa
l’interval de decreixement serà (–∞, –2) i el de creixement serà (–2, +∞).

c) f(x) = x2 + 1

Aquesta funció és el resultat de traslladar una unitat cap amunt la funció original, per la qual cosa l’interval
de decreixement en aquest cas serà (–∞, 0) i el de creixement serà (0, +∞).

d) f(x) = x2 – 3

En aquest cas, igual que a l’apartat anterior, és produeix una translació vertical de tres unitats cap avall,
així, l’interval de decreixement en aquest cas serà (–∞, 0) i el de creixement serà (0, +∞).

4. Estudia el creixement i el decreixement de les funcions.


a) f(x) = 2x2 + 4x – 3 b) f(x) = –x2 + 6x + 2
a) La derivada és f ´( x) = 4x +4 i 4 x + 4 = 0  x = -1 .

Veiem que 4 x + 4 < 0  x Î (-¥, -1) → En aquest interval la funció decreix.

4 x + 4 > 0  x Î (-1, +¥) → En aquest interval la funció creix.

b) La derivada és f ´( x) =-2x +6 i -2 x + 6 = 0  x = 3 . Veiem que:

-2x + 6 > 0  x Î (-¥, 3) → En aquest interval la funció creix.

-2x + 6 < 0  x Î (3, +¥) → En aquest interval la funció decreix.

5. Determina els màxims i els mínims d’aquestes funcions


a) f(x) = x3 – 3x2 b) f(x) = 2x3 – 3x2 – 36x + 1
a) La seva derivada és f ´( x) = 3 x 2 - 6 x = 3 x( x - 2) , que s’anul·la en x = 0 i x = 2.

És creixent a l’esquerra de 0 i decreixent a la dreta → Màxim en (0, 0).

És decreixent a l’esquerra de 2 i creixent a la dreta → Mínim en (2, –4).

b) La seva derivada és f ´( x) = 6x2 - 6x - 36 = 6( x - 3)( x + 2) i s’anul·la en x = 3 i x = –2.

És creixent a l’esquerra de –2 i decreixent a la dreta → Màxim en (–2, 45).

És decreixent a l’esquerra de 3 i creixent a la dreta → Mínim en (0, –80).

674
Aplicacions de la derivada. Representació de funcions 1212

6. Determina els màxims i els mínims de les funcions següents mitjançant la derivada segona:
a) f(x) = x4 – 4x2 + 2 c) f(x) = x2 + 9x
b) f(x) = x3 – 3x2 + 7 d) f(x) = 2x3 + 9x2 – 4x
a) La seva primera derivada és f ´( x ) = 4 x 3 - 8 x = 4 x ( x 2 - 2) = 4 x ( x - 2 )( x + 2 ) , i és igual a 0 en x = 0, x = –

2 i x= 2.

La segona derivada és f ´´( x ) = 12x2 -8 = 4(3 x - 2) .

2
Per x = - 2 : f ´´(- 2 ) = 4(3 (- 2 ) - 2) = 4(6 - 2) = 16 > 0 → Mínim en x = - 2 .

2 : f ´´( 2 ) = 4 (3 ( 2 ) - 2) = 4(6 - 2) = 16 > 0 → Mínim en x =


2
Per x = 2.

Per x = 0: f ´´(0) = 4(3(0)2 - 2) = 4(0 - 2) =-8 < 0 → Màxim en x = 0.

b) La seva primera derivada és f ´( x ) = 3 x 2 + 6 x = 3 x( x - 2) , i és igual a 0 en x = 0 i x = 2.

La seva segona derivada és f ´´( x) = 6x +6 = 6( x -1) .

Per x = 0: f ´´(0) = 6(0 -1) =-6 <0 → Màxim en x = 0.

Per x = 2: f ´´(2) = 6(2-1) = 6 > 0 → Mínim en x = 2.

-9
c) La seva primera derivada és f ´( x) = 2x +9 , i és igual a 0 en x = .
2

La seva segona derivada és f ´´( x) =2 .

-9 æ- 9 ö -9
Para x = : f ´´ççç ÷÷÷= 2 > 0 → Mínim en x = .
2 è 2 ø 2

-9 + 105 -9 - 105
d) La seva primera derivada és f ´( x ) = 6 x 2 + 18 x - 4 que s’anul·la per x = i x= .
6 6

La seva segona derivada és f ´´( x ) = 12 x + 18 .

-9 + 105 æ -9 + 105 ÷ö -9 + 105


Per x = : f ´´ççç ÷ø÷÷ > 0 → Mínim en x = .
6 è 6 6

-9 - 105 æ -9 - 105 ö÷
Per x = : f ´´ççç ÷÷ < 0 → Màxim en x = -9 - 105 .
6 è 6 ÷ø 6

7. Considera la funció racional següent:


࢞૛
ࢌሺ࢞ሻ ൌ
࢞൅૚
Estudia’n el creixement i troba’n els màxims i els mínims.
2 x ( x + 1) - x 2 2 x 2 + 2 x - x 2 x2 + 2x x ( x + 2)
La seva primera derivada és f ´( x ) = 2
= 2
= = .
( x + 1) ( x + 1) ( x + 1)2 ( x + 1)2

675
Aplicacions de la
Aplicacions de la derivada.
derivada. Representació
Representacióde
defuncions
funcions 12

S’observa que és una funció racional on el denominador és sempre positiu, així que estudiem el signe del
numerador.

x( x + 2) > 0  x Î (-¥, - 2) È (0, +¥) . Per tant, a aquests intervals la funció és creixent.

x( x + 2) < 0  x Î (-2, 0) . Per tant, a aquest interval la funció és decreixent.

x ( x + 2)
Per calcular els valors de x tals que f ´( x) = 0 , fem = 0 , que és equivalent en aquest cas a calcular
( x + 1)2

x( x + 2) = 0 . Aquesta equació es compleix per x = 0 i x = –2.

La seva segona derivada és:

(2 x + 2)( x + 1)2 - 2 x ( x + 2)( x + 1) (2 x + 2)( x + 1) - 2 x ( x + 2) 2


f ´´( x ) = = =
( x + 1)4 ( x + 1)3 ( x + 1)3

2
Per x = 0: f ´´(0) = = 2 > 0 → Mínim en x = 0.
(0 + 1)3

2
Per x = –2: f ´´(-2) = = -2 < 0 → Màxim en x = –2.
(-2 + 1)3

8. Determina on són còncaves i on són convexes les funcions següents:


࢞૛
a) f(x) = 7x3 – x2 – x + 2 c) f(x) =
࢞૛ ା૚
a) Analitzem el signe de f ´´(x):

f ´(x)  21x2  2x  1      f ´´(x)  42x  2 

Cerquem els punts pels quals f ´´(x) s’anul·la, que són els possibles punts d’inflexió:

1
f ´´(x)  42x  2  0 → x =  
21

1
Analitzem el signe de f ´´(x) a l’esquerra i a la dreta de :
21

1  
Per x < : f ´´(0)  2  0 → f(x) és convexa. 
21

1  
Per x > : f ´´(1)  26  0 → f(x) és còncava. 
21

b) Analitzem el signe de f ´´(x):

2x 2 (1 - 3 x 2 )
f ´( x ) = 2
        f ´´( x ) =  
( x 2 + 1) ( x 2 + 1)
3

Cerquem els punts pels quals f ´´(x) s’anul·la, que són els possibles punts d’inflexió:

2 (1 - 3 x 2 ) 3
f ´´( x ) = 3
= 0  → x =   
( x 2 + 1) 3

676
Aplicacions de la derivada. Representació de funcions 1212

3 3 3 3
Analitzem el signe de f ´´(x) en x < - , - <x< i <x:
3 3 3 3

3  
Per  x < - : f ´´(2)  0 → f(x) és convexa.
3

3 3  
Per  - <x< : f ´´(0)  0 → f(x) és còncava.
3 3

3
Per  < x : f ´´(2)  0 → f(x) és còncava. 
3

9. Determina els intervals de concavitat i convexitat de les funcions següents, i comprova el resultat
gràficament:
a) f(x) = –x2 – x + 4 b) f(x) = –x – 5x2
a) f ´(x) = –2x – 1 f ´´(x) = –2

f ´´(x) < 0 per tot valor de x, per tant, és sempre convexa.

b) f ´(x) = –1 – 10x f ´´(x) = –10

f ´´(x) < 0 per tot valor de x, per tant és sempre convexa.

10. Representa la gràfica d’una funció sempre creixent que no tingui asímptotes.
Resposta oberta. Per exemple:

677
Aplicacions de la
Aplicacions de la derivada.
derivada. Representació
Representacióde
defuncions
funcions 12

 
 

11. Dibuixa la gràfica d’una funció sempre decreixent que tingui una asímptota vertical i una
d’horitzontal.
Resposta oberta. Per exemple:

12. Representa la gràfica d’una funció sempre positiva amb un màxim en (4, 6).
Resposta oberta. Per exemple:

13. Dibuixa la gràfica d’una funció de domini [–5, 5], que passi per l’origen i que verifiqui ‫ࢌ ܕܑܔ‬ሺ࢞ሻ ൌ λ i
࢞՜ି૞

‫ࢌ ܕܑܔ‬ሺ࢞ሻ ൌ  െλ
࢞՜૞

Resposta oberta. Per exemple:

678
Aplicacions de la derivada. Representació de funcions 1212

 
 

14. Representa gràficament una funció que compleixi totes les condicions que tens a continuació:
 El seu a domini és � – {3}.
 Talla els eixos en el punt (0, 0).
 Té una asímptota vertical en el punt x = 3 i una d’horitzontal d’altura y = 1.
 Sempre és decreixent.
 És còncava en (–∞, 3) i convexa en (3, +∞).
 Té un únic punt d’inflexió.
Resposta oberta. Per exemple:

 
 
15. Representa gràficament una funció que compleixi totes les condicions que trobes descrites a
continuació:
 El seu a domini és �.
 Talla l’eix X en el punt (–2, 0) i l’eix Y en el punt (0, 1).
 Té dues asímptotes horitzontals, y = 3 i y = –2.
 Sempre és creixent.
 És convexa en (–∞, 1) i còncava en (1, +∞).
 Té un punt d’inflexió en (–1, 0).
No existeix una funció còncava en (–∞, 1) i convexa en (1, +∞) amb un punt d’inflexió en x = –1; el punt
d’inflexió hauria de ser a x = 1.

16. Dibuixa la gràfica d’una funció que té de domini �, que passa per (–2, –1) i (2, 3), és creixent en
(–∞, 0], decreixent en (0, ∞) i té un màxim en (0, 6).

679
Aplicacions de la
Aplicacions de la derivada.
derivada. Representació
Representacióde
defuncions
funcions 12

Resposta oberta. Per exemple:

 
 
17. Representa la gràfica d’una funció amb domini R, que té com a punts de tall (0, 4) i (6, 0), i que
verifica que ‫ࢌ ܕܑܔ‬ሺ࢞ሻ ൌ λ i ‫ࢌ ܕܑܔ‬ሺ࢞ሻ ൌ  െλ
࢞՜ିஶ ࢞՜ஶ

Resposta oberta. Per exemple:

18. Representa gràficament les funcions següents:


a) f(x) = x3 – 3x2 + 2 b) f(x) = x3 – 3x2 + 2x
a) b)
 
 
 
 
 
 
 
19. Representa gràficament aquestes funcions:
a) f(x) = x3 – 4x b) f(x) = x3 – 6x
a) b)

 
 
 
 

680
Aplicacions de la derivada. Representació de funcions 1212

20. Determina quantes asímptotes hi ha en aquestes funcions i quines són:


૜࢞ ࢞૛
a) ࢌሺ࢞ሻ ൌ b) ࢌሺ࢞ሻ ൌ
࢞ି૛ ࢞ି૚

a) ▪ Asímptotes verticals:

3x
El denominador s’anul·la quan x = 2: lim = ¥  f té una asímptota vertical en x = 2.
x 2 x -2

▪ Asímptotes horitzontals:

3x
Com que grau(3x) = grau(x – 2) → Existeix asímptota horitzontal en y = k, on k = xlim = 3  y = 3.
¥ x -2

▪ Asímptotes obliqües:

No existeix asímptota obliqua perquè grau(3x) = grau(x – 2) = 1.

b) ▪ Asímptotes verticals:

El denominador s’anul·la quan x = 1:

x2
lim 2
= ¥  f té una asímptota vertical a x = 1.
x 1 x - 1

x2
lim = ¥  f té una asímptota vertical a x = –1.
2
x -1 x - 1

▪ Asímptotes horitzontals:

x2
Com que grau(x2) = grau(x2 – 1) → Existeix asímptota horitzontal en y = k, on k = xlim 2
= 1 → y = 1.
¥ x -1

▪ Asímptotes obliqües:

No existeix asímptota obliqua perquè grau(x2) = grau(x2 – 1) = 1.

21. Determina les asímptotes obliqües de les funcions següents:


࢞૛ ࢞
a) ࢌሺ࢞ሻ ൌ b) ࢌሺ࢞ሻ ൌ ࢞ ൅
࢞ା૚ ૛ି࢞

f (x) x2 x2
a) m = xlim = lim = lim 2 = 1 m = 1
¥ x x ¥ x ( x + 1) x ¥ x +x

é x2 ù é x2 - x2 - x ù
n = lim [ f ( x ) - mx ] = lim ê - x ú = lim ê ú = lim - x = - 1  n = - 1  
x ¥ ê
x ¥ x + 1 ú x ¥ ê x + 1 ú x ¥ x + 1
ë û ë û

Per tant, l’asímptota obliqua és y = x – 1.

x
x+ 2
b) m = xlim
f (x)
= lim 2 - x = lim 3 x - x = lim 3 - x = 1  m = 1
¥ x x ¥ x x ¥ x (2 - x ) x ¥ 2 - x

é x ù é x ù
n = lim [f ( x ) - mx ] = lim ê x + - x ú = lim ê ú = -1  n = -1  
x ¥ ê
x ¥
ë 2- x ûú x ¥ ëê 2 - x ûú

Per tant, l’asímptota obliqua és y = x – 1.

681
Aplicacions de la
Aplicacions de la derivada.
derivada. Representació
Representacióde
defuncions
funcions 12

22. Representa aquesta funció:


࢞൅૝
ࢌሺ࢞ሻ ൌ
૛࢞ െ ૜

 
 
23. Representa la funció següent:
࢞൅૝
ࢌሺ࢞ሻ ൌ ࢞ െ
࢞െ૜

 
 
   

682
Aplicacions de la derivada. Representació de funcions 1212

SABER FER 

24. Estudia el creixement el decreixement de f (x).


࢞૛ ൅ ૛࢙࢞࢏࢞ ൏ െ૛
ࢌሺ࢞ሻ ൌ ൝ ૚
૚ ൅ ૛ ࢙࢏࢞ ൒ െ૛

Analitzem el domini de f(x) per saber on pot tenir discontinuïtats:

Dom f =  -{0}

Analitzem el signe de f ´(x):

ïìï2 x + 2 si x < -2
f ´( x ) = ïí 2
ïï- si x ³ -2
ïîï x
3

Hem de veure on s’anul·la f ´(x). Analitzem on s’anul·len cadascuna de les parts que la formen:

ü
2 x + 2 = 0  x = –1ï
ï
ï → En x = –1 f ´(x) no pren el valor de la primera funció. Per tant, f ´(x) mai s’anul·la.
2 ý
- 3 ¹0 ï
ï
x ï
þ

Analitzem el signe de f ´(x) si x  2,  - 2 £ x < 0  i x  0: 

Per x  2: f ´(3)  0 → f(x) és decreixent. 

Per  - 2 £ x < 0 : f ´(1)  0 → f(x) és creixent.

Per x  0: f ´(1)  0 → f(x) és decreixent.

25. La funció g(x) = 100 + 5x + x2 proporciona la despesa en euros d’una empresa per fabricar x unitats
d’un producte. Si cada unitat té un preu de venda de 30 €, calcula la funció benefici i quantes
unitats han de produir perquè sigui màxim.
Per començar, si s’entén que l’empresa gana 30 € per cada unitat, l’ingrés serà f(x) = 30x, mentre que la
despesa serà g(x).

Per tant, el benefici serà b ( x ) = f ( x ) - g ( x ) = 30 x - 100 - 5 x - x 2 = 25 x - x 2 - 100 .

25
Primer es calcula b´( x ) = 25- 2 x i s’observa que el candidat a màxim o mínim és x = .
2

25
Es calcula b´´( x ) = -2 i es veu aleshores que x = és un màxim.
2

25
Com que = 1,25 no és un nombre enter, s’observa el valor de b(12) i b(13) i, essent el benefici
2
b(12) = (13) = 56, es conclou que per maximitzar el benefici s’ha de fabricar 12 o 13 unitats.

683
Aplicacions de la
Aplicacions de la derivada.
derivada. Representació
Representacióde
defuncions
funcions 12

26. Determina el valor de a perquè hi hagi un mínim en x = 2 per a f(x) = x2 + ax + a.


Es calcula la derivada de la funció f ´( x ) = 2 x + a i imposem la condició f ´(2) = 0:

2 ⋅ 2 + a = 0  a = -4 , així que el candidat serà a = –4.

Es comprova que és un mínim, per això mirem que la segona derivada és positiva:

f ´´( x ) = 2  f ´´(2) = 2 > 0 . Amb a  4, x  2 és un mínim.

27. Determina com són les funcions la derivada de les quals es representa amb una recta de pendent 3.
Sabem que la derivada de la funció és una recta amb pendent 3, per tant serà de la forma f ´( x ) = 3 x + n .

La funció serà del tipus f ( x ) = ax 2 + bx + c , atès que la seva derivada és f ´( x ) = 2 ax + b .

3
Aleshores tenim que 2a = 3  a = , i b = n , atès que les funcions que compleixin les condicions de
2
3
l’enunciat tindran com equació: f ( x ) = x 2 + nx + c amb n, c Î 
2

28. Troba l’equació de la paràbola que té com a vèrtex (3, –8) si talla l’eix Y en (0, 1).
Una paràbola tindrà com equació f ( x ) = ax 2 + bx + c , i com que passa per (0, 1), al substituir, s’obté c = 1.

Atès que el punt (3, –8) és el vèrtex de la paràbola:

▪ És un punt de l’equació → a3 2 + b3 + 1 = -8

▪ Serà un màxim o un mínim global → f ´(3) = 0 → 6 a + b = 0 .

a32 + b3 + 1= -8ïüï 9a + 3b = -1 üïï


ý ý  9a - 18a = -9  a = 1, b = -6
6a + b = 0 ïþï b = -6aïï
þ

Per tant, f (x) = x2 - 6 x + 1.

29. Representa la derivada de f(x) = x3 – x2 + x – 2.


f ´(x)  3x2  2x  1 

684
Aplicacions de la derivada. Representació de funcions 1212

30. Estudia la concavitat i la convexitat d’aquesta funció:


࢞૛ ൅ ૛࢙࢞࢏࢞ ൏ െ૛
ࢌሺ࢞ሻ ൌ ൝ ૚
૚ ൅ ૛ ࢙࢏࢞ ൒ െ૛

 
Analitzem el domini de f(x) per saber on pot tenir discontinuïtats: 

Dom f =  -{0}  

ïìï2 x + 2 si x < -2 ì2
ï
ï si x < -2
f ´( x ) = ïí 2        f ´´( x ) = ï
í6  
ïï- si x ³ -2 ï
ï si x ³ -2
ïîï x ï x4
3
ï
î

Hem de cercar on s’anul·la f ´´(x). En aquest cas, f ´´( x ) ¹ 0 per qualsevol x, per tant, analitzem el signe de
f´´(x) si x = –2,  - 2 £ x < 0  i x  0: 

Per x  2: f ´´  0 → f(x) és còncava. 

Per  - 2 £ x < 0 : f ´´(1)  0 → f(x) és còncava.

Per x  0: f ´(1)  0 → f(x) és còncava.

31. Estudia la concavitat o la convexitat d’aquesta funció en els punts x = –1, x = 1 i x = 2:

Quan x = –1 la funció és còncava.

Quan x = 1 la funció té un punt d’inflexió.

Quan x = 2 la funció és convexa.

૛ି࢞૛
32. Determina si la funció ࢌሺ࢞ሻ ൌ té asímptotes horitzontals i estudia’n la posició.
࢞૛ ି૚

2 - x2
lim = -1  f té una asímptota horitzontal en y = - 1 .
x ¥ x2 -1

Per determinar la situació de la gràfica respecte de l’asímptota per la dreta, a x resulten valors molt grans, i
els estudiem:
2
Per x = 1 000: f (1 000) = 2 - (1 000)
2
=
2 - 1 000 000 - 999 998
= = - 0,999998 > - 1
(1 000) - 1 1 000 000 - 1 999 999

685
Aplicacions de la
Aplicacions de la derivada.
derivada. Representació
Representacióde
defuncions
funcions 12

Per la dreta, la gràfica se situa per sobre de l’asímptota.

Per determinar la situació de la gràfica respecte de l’asímptota per l’esquerra, a x resulten valors molt
petits, i els estudiem:
2
Per x = –1 000: f (- 1 000) = 2 - (- 1 000)
2
=
- 999 998
= - 0,999998 > - 1
(- 1000) - 1 999 999

Per l’esquerra, la gràfica se situa per sobre de l’asímptota.

࢞૜
33. Calcula les asímptotes horitzontals de la funció ࢌሺ࢞ሻ ൌ i estudia’n la posició.
૛࢞૜ ା૜‫ ܠ‬૛ ା૛
x3 1 1
lim =  f té una asímptota horitzontal en y = .
x ¥ 2x + 3x2 + 2 2
3
2

Per determinar la situació de la gràfica respecte de l’asímptota per la dreta, a x resulten valors molt grans, i
els estudiem:

(1 000)3 1 000 000 000 1


Per x = 1 000: f (1 000) = = = 0,49925... <
2(1 000) + 3(1 000)2 + 2 2 003 000 002
3
2

Per la dreta, la gràfica se situa per sota de l’asímptota.

Per determinar la situació de la gràfica respecte de l’asímptota per l’esquerra, a x resulten valors molt
petits, i els estudiem:

(- 1 000)3 1 000 000 000 1


Per x = –1 000: f (- 1 000) = = = 0,5007... >
2(- 1 000)3 + 3(- 1 000)2 + 2 1 996 999 998 2

Per l’esquerra, la gràfica se situa per sobre de l’asímptota.

‫ܠ‬ା૚
34. Determina si la funció ࢌሺ࢞ሻ ൌ té asímptotes verticals i estudia’n la posició.
‫ ܠ‬૛ ି૝

Quan el denominador pren el valor 0 tenim les asímptotes verticals. En aquest cas:

x 2 - 4 = ( x + 2)( x - 2) = 0  x =  2

x +1
lim =¥ Té una asímptota vertical en x = 2.
x 2 x2 - 4

x +1
lim = ¥  Té una asímptota vertical en x = –2.
x -2 x2 - 4

Per determinar la situació de la gràfica respecte de l’asímptota x = –2 per l’esquerra, s’estudia la funció per
valors molt propers a –2 per l’esquerra de x:

-2,01 + 1
Per x = –2,01: f (-2,01) = < 0  lim- f ( x ) = -¥
(-2,01)2 - 4 x -2

Per determinar la situació de la gràfica respecte de l’asímptota x = –2 per la dreta, s’estudia la funció per
valors molt propers a –2 per la dreta de x:

686
Aplicacions de la derivada. Representació de funcions 1212

-1,99 + 1
Per x = –1,99: f (-1,99) = > 0  lim+ f ( x ) = +¥
(-1,99)2 - 4 x -2

Per determinar la situació de la gràfica respecte de l’asímptota x = 2 per l’esquerra, s’estudia la funció per
valors molt propers a 2 per l’esquerra de x:

1,99 + 1
Per x = 1,99: f (1,99) = < 0  lim- f ( x ) = -¥
(1,99)2 - 4 x 2

Per determinar la situació de la gràfica respecte de l’asímptota x = 2 per la dreta, s’estudia la funció per
valors molt propers a 2 per la dreta de x:

2,01+ 1
Per x = 2,01: f (2,01) = > 0  lim+ f ( x ) = +¥
(2,01)2 - 4 x 2

‫ܠ‬ା૚
35. Calcula les asímptotes verticals de la funció ࢌሺ࢞ሻ ൌ i estudia’n la posició.
‫ ܠ‬૛ ି૝
Quan el denominador pren el valor 0 tenim les asímptotes verticals. En aquest cas:

x 2 - x - 6 = ( x + 2)( x - 3) = 0  x = -2, x = 3 → Té dues asímptotes verticals.

3x - 2
lim = ¥  Té asímptota vertical en x = –2.
x -2 x2 - x - 6

3x - 2
lim =¥ Té asímptota vertical en x = 3.
x 3 x2 - x - 6

Per determinar la situació de la gràfica respecte de l’asímptota x = –2 per l’esquerra, s’estudia la funció per
valors molt propers a –2 per l’esquerra de x:

3(-2,01) - 2
Per x = –2,01: f (-2,01) = < 0  lim- f ( x ) = -¥
(-2,01)2 + 2,01- 6 x -2

Per determinar la situació de la gràfica respecte de l’asímptota x = –2 per la dreta, s’estudia la funció per
valors molt propers a –2 per la dreta de x:

3(-1,99) - 2
Per x = –1,99: f (-1,99) = > 0  lim+ f ( x ) = +¥
(-1,99)2 + 1,99 - 6 x -2

Per determinar la situació de la gràfica respecte de l’asímptota x = 3 per l’esquerra, s’estudia la funció per
valors molt propers a 3 per l’esquerra de x:

3(2,99) - 2
Per x = 2,99: f (2,99) = < 0  lim- f ( x ) = -¥
(2,99)2 - 2,99 - 6 x 3

Para determinar la situació de la gràfica respecte de l’asímptota x = 3 per la dreta, s’estudia la funció per
valors molt propers a 3 per la dreta de x:

3(3,01) - 2
Per x = 3,01: f (3,01) = > 0  lim+ f ( x ) = +¥
(3,01)2 - 3,01- 6 x 3

 
687
Aplicacions de la
Aplicacions de la derivada.
derivada. Representació
Representacióde
defuncions
funcions 12

ACTIVITATS 
 
36. Determina si aquestes funcions creixen o decreixen en els punts indicats:
a) ���� � ���� � ��� � � � � �� � � �
��� �����
b) ���� � �� � � ��

c) ���� � �� � ��� � � � �� � � �

d) ���� � �� � �√� �� � � �

a) ����� � ��� � � �� � �
� � ��� � �� � � � La funció és decreixent en x = 1.
������������ ������ ��� ����
b) � � ��� � �
�� ��

� � ���� � � � � La funció és creixent en x = –2.


c) � � ��� � �� � �� � �


� � �4� � �� �� � � � � � La funció és creixent en x = 4.


d) � � ��� � � �
� √�

� � �9� � � � � La funció és creixent en x = 9.

 
37. Indica si les funcions següents creixen o decreixen en els punts assenyalats:
a) ���� � ��� � �� � � �
b) ���� � ��� � � �� � �� � � �
����� � �
c) ���� � �� � �
�� �� �

d) ���� � ��� � � ��� � �� � � �


� �

a) f ´( x ) = cos x  f ´(p) = cos p =-1< 0  La funció decreix en x = p .

b) f ´( x ) = cos x + 1+ tg 2 x  f ´(0) = cos 0 + 1+ tg 2 0 = 2 > 0  La funció creix en x = 0.

( x 2 -1) cos x - 2 x( sin x + 1) æpö


c) f ´( x ) =  f ´ççç ÷÷÷ < 0  p
La funció decreix en x = .
2 è2ø
( x -1)
2
2

d) f ( x ) = sin 2 x + cos 2 x = 1 " x Î   Funció constant en tot el seu domini i no creix ni decreix en x = 0.

38. Indica els intervals de creixement i decreixement d’aquestes funcions. Hi ha algun màxim o mínim?
a) f(x) = x2 – 4x + 1 d) f(x) = –3x – x2
b) f(x) = –x2 + 4 e) f(x) = x2 – 6x + 5
c) f(x) = 2x2 – 8x f) f(x) = 3x2 – 2x.

688
Aplicacions de la derivada. Representació de funcions 1212

a) f ´( x ) = 2 x - 4  f ´( x ) = 0 quan x = 2.

En (–∞, 2) f ´(x) < 0 → f(x) és decreixent en aquest interval.

En (2, +∞) f ´(x) > 0 → f(x) és creixent en aquest interval.

Per tant, x = 2 és un mínim.

b) f ´( x ) = - 2 x  f ´( x ) = 0 quan x = 0.

En (–∞, 0) f ´(x) < 0 → f(x) és creixent en aquest interval.

En (0, +∞) f ´(x) > 0 → f(x) és decreixent en aquest interval.

Per tant, x = 0 és un màxim.

c) f ´( x ) = 4 x - 8  f ´( x ) = 0 quan x = 2.

En (–∞, 2) f ´(x) < 0 → f(x) és decreixent en aquest interval.

En (2, +∞) f ´(x) > 0 → f(x) és creixent en aquest interval.

Per tant, x = 2 és un mínim.

3
d) f ´( x ) = - 3 - 2 x  f ´( x ) = 0 quan x = - .
2

æ 3ö
En ççç-¥, - ÷÷÷ f ´(x) > 0 → f(x) és creixent en aquest interval.
è 2ø

æ 3 ö
En ççç- , + ¥÷÷÷ f ´(x) < 0 → f(x) és decreixent en aquest interval.
è 2 ø

3
Per tant, x = - és un màxim.
2

e) f ´( x ) = 2 x - 6  f ´( x ) = 0 quan x = 3.

En (–∞, 3) f ´(x) < 0 → f(x) és decreixent en aquest interval.

En (3, +∞) f ´(x) > 0 → f(x) és creixent en aquest interval.

Per tant, x = 3 és un mínim.

1
f) f ´( x ) = 6 x - 2  f ´( x ) = 0 quan x = .
3

æ 1ö
En ççç-¥, ÷÷÷ f ´(x) < 0 → f(x) és decreixent en aquest interval.
è 3ø

æ1 ö
En ççç , + ¥÷÷÷ f ´(x) > 0 → f(x) és creixent en aquest interval.
è3 ø

1
Per tant, x = és un mínim.
3

689
Aplicacions de la
Aplicacions de la derivada.
derivada. Representació
Representacióde
defuncions
funcions 12

39. Estudia els intervals de creixement i decreixement de les funcions següents i troba’n els possibles
màxims o mínims:
a) f(x) = x3 – 6x2 + 9x + 1 d) f(x) = 2x3 + 3x2 – 36x + 8
b) f(x) = x3 – 3x2 – 9x + 1 e) f(x) = 2x3 + 3x2 + 6x – 12
c) f(x) = 2x3 – 3x
a) f ´( x ) = 3 x 2 - 12 x + 9  f ´( x ) = 0 quan x = 1 i x = 3. Dividim el domini en 3 intervals.

En (–∞, 1) f ´(x) > 0 → f(x) és creixent en aquest interval.

En (1, 3) f ´(x) = 0 → f(x) és decreixent en aquest interval.

En (3, +∞) f ´(x) > 0 → f(x) és creixent en aquest interval.

En x  1 té un màxim i en x  3 un mínim. 

b) f ´( x ) = 3 x 2 - 6 x - 9  f ´( x ) = 0 quan x = –1 i x = 3. Dividim el domini en 3 intervals.

En (–∞, –1) f ´(x) > 0 → f(x) és creixent en aquest interval.

En (–1, 3) f ´(x) = 0 → f(x) és decreixent en aquest interval.

En (3, +∞) f ´(x) > 0 → f(x) és creixent en aquest interval.

En x = –1 té un màxim i en x = 3 un mínim.

c) f ´( x ) = 3 x 2 - 3  f ´( x ) = 0 quan x = 1 .

En (–∞, –1) f ´(x) > 0 → f(x) és creixent en aquest interval.

En (–1, 1) f ´(x) < 0 → f(x) és decreixent en aquest interval.

En (1, +∞) f ´(x) > 0 → f(x) és creixent en aquest interval.

En x = –1 té un màxim i en x = 1 un mínim.

d) f ´( x ) = 6 x 2 + 6 x - 36  f ´( x ) = 0 quan x = 2 i x = –3.

En (–∞, –3) f ´(x) > 0 → f(x) és creixent en aquest interval.

En (–3, 2) f ´(x) < 0 → f(x) és decreixent en aquest interval.

En (2, +∞) f ´(x) > 0 → f(x) és creixent en aquest interval.

En x = –3 té un màxim, i en x = 2 un mínim.

e) f ´( x ) = 6 x 2 + 6 x + 6 > 0  f ´( x ) > 0 "x Î 

La funció és creixent en tot la seva domini i no té màxims ni mínims.

40. Determina els intervals de creixement i decreixement de les funcions següents:


a) ࢌሺ࢞ሻ ൌ ࢞૝ െ ૝࢞ b) ࢌሺ࢞ሻ ൌ ૢ࢞૝ െ ૛࢞૜ െ ૟࢞૛  

690
Aplicacions de la derivada. Representació de funcions 1212

���
c) ���� � � �� � ��� � � d)  ���� � �� � ���    

a) f ´( x ) = 4 x 3 - 4  f ´( x ) = 0 quan x = 1.

En (–∞, 1) f ´(x) < 0 → f(x) és decreixent en aquest interval.

En (1, +∞) f ´(x) > 0 → f(x) és creixent en aquest interval.

1 2
b) f ´( x ) = 36 x 3 - 6 x 2 - 12 x  f ´( x ) = 0 quan x = 0, x = - i x = .
2 3

æ 1ö
En ççç-¥, - ÷÷÷ f ´(x) < 0 → f(x) és decreixent en aquest interval.
è 2ø

æ 1 ö
En ççç- , 0÷÷÷ f ´(x) > 0 → f(x) és creixent en aquest interval.
è 2 ø

æ 2ö
En ççç0, ÷÷÷ f ´(x) < 0 → f(x) és decreixent en aquest interval.
è 3ø

æ2 ö
En ççç , + ¥÷÷÷ és creixent en aquest interval.
è 3 ø

c) f ´( x ) = 3 x 3 - 3 x 2 - 6 x  f ´( x ) = 0 quan x = 0, x = –1 i x = 2.

En (–∞, –1) f ´(x) < 0 → f(x) és decreixent en aquest interval.

En (–1, 0) f ´(x) > 0 → f(x) és creixent en aquest interval.

En (0, 2) f ´(x) < 0 → f(x) és decreixent en aquest interval.

En (2, +∞) f ´(x) > 0 → f(x) és creixent en aquest interval.

d) f ´( x ) = 4 x ( x + 1)( x - 1)  f ´( x ) = 0 quan x = 0, x = –1 i x = 1.

En (–∞, –1) f ´(x) < 0 → f(x) és decreixent en aquest interval.

En (–1, 0) f ´(x) > 0 → f(x) és creixent en aquest interval.

En (0, 1) f ´(x) < 0 → f(x) és decreixent en aquest interval.

En (1, +∞) f ´(x) > 0 → f(x) és creixent en aquest interval.

41. Estudia el creixement d’aquestes funcions:


a) f(x) = x3 b) f(x) = x4 c) f(x) = √�
a) 2
f ´( x ) = 3 x  f ´( x ) = 0 quan x = 0.

En (–∞, 0) f ´(x) > 0 → f(x) és creixent en aquest interval.

En (0, +∞) f ´(x) > 0 → f(x) és creixent en aquest interval.

691
Aplicacions de la derivada. Representació de funcions

<

>

692
12

693
Aplicacions de la
Aplicacions de la derivada.
derivada. Representació
Representacióde
defuncions
funcions 12

ïì f ´( x ) < 0 si x < - 1  f decreix üï


ïïì2 x + 2 si x £ 2 ïïï ýï  f té un mínim en x = - 1
d) f ´( x ) = íï- 3  í f ´( x ) > 0 si - 1 < x £ 2  f creix ïï
þ  
îï si x > 2 ïïï
ïî f ´( x ) < 0 si 2 < x  f decreix

 
45. Analitza en quins intervals creixen i en quins decreixen les funcions que tens a continuació:
�� ��
a) ���� � c) ���� �
�� �� ���
��� ���
b) ���� � d) ���� �
��� ���

 
2x
a) f ´( x ) =  f ´( x ) = 0 quan x = 0.
( x 2 + 1)2

Com que el denominador és sempre positiu, únicament comprovarem el signe del numerador, per tant:

2x
f ´( x ) = < 0  x Î (-¥, 0)  En aquest interval f(x) decreix.
( x 2 + 1)2

2x
f ´( x ) = > 0  x Î (0, + ¥)  En aquest interval f(x) creix.
( x 2 + 1)2

1
b) Dom f ( x ) =  - {- 3 } f ´( x ) = > 0 en tot el domini, per tant f(x) és sempre creixent.
( x + 3)2

2
c) Dom f ( x ) =  - {-1} f ´( x ) = > 0 en tot el domini, per tant f(x) és sempre creixent.
( x + 1)2

2
d) Dom f ( x ) =  - {2} f ´( x ) = - < 0 en tot el domini, per tant f(x) és sempre decreixent.
( x - 2)2

46. Determina si és cert o no ho és que la funció f(x) = x3 sempre és creixent. Explica què passa en
l’origen de coordenades.
f’(x) = 3x2
Si x ≠ 0 → f’(x) > 0→ f(x) és creixent en (–∞, 0) ∪ (0, +∞)
Si x = 0 → f’(0) = 0→ La funció no és creixent ni decreixent en aquest punt.

47. Estudia el creixement i el decreixement de les funcions següents en els punts indicats:
a) ���� � |�� � �| en x = 1 i x = –4.
b) ���� � |�� � �| en x = 0 i x = 5.
c) ���� � |��� � ��| en x = –1 i x = 2.
��
d) ���� � � � �� en x = –2 i x = 0.

ìï ïì æ 2ö
ïï-3x - 2 si x < - 2
ìï
ïï-3 si x < - 2 ïï f ( x) decreix en çç-¥, - ÷÷÷.
ïï 3 ï 3 ï ç
è 3ø
a)  f ( x) = íï  f ´( x) = ïí ï í  
ïï3x + 2 2 ï
ïï3 2 ï æ ö
ïï f ( x) creix en ç- 2 , + ¥÷÷.
ï si x ³ - ï si x > - ï çè
ç ÷
îï 3 îï 3 ï î 3 ø

694
Aplicacions de la derivada. Representació de funcions 1212

En x = 1 creix i en x = –4 decreix.

ïì x - 3 si x > 3 ïì1 si x > 3 ïìï f ( x ) creix en (3, + ¥ ).


f ( x) = ïí  f ´( x ) = ï
í í
ïî
ï3 - x si x £ 3 ïî
ï- 1 si x < 3 ïîï f ( x ) decreix en (-¥ , 3).
b)   

En x = 0 decreix i en x = 5 creix.

ìï x - 3 si x > 3 ìï1 si x > 3 ìï f ( x ) creix en (3, + ¥ ).


f ( x) = ïí  f ´( x ) = ïí ïí
ïî
ï3 - x si x £ 3 ï
ï- 1 si x < 3 î
î ï
ï f ( x ) decreix en (-¥ , 3).  
c) 

En x = –1 decreix i en x = 2 decreix.

ïì ìï æ 2ö
ïï-3x - 2 si x < - 2 ïï-3 si x < - 2 ïï f ( x) decreix en çç-¥, - ÷÷÷.
ïì
ï 2 3 ï 2 3 ïï ç
è 3ø
f ( x) = íï  f ´( x) = íï í
ï 3x + 2 2 ï3 2 ïï æ 2 ö
ïïï si x ³ - ïïï si x > - ïï f ( x) creix en ççç- , + ¥÷÷÷.
d)  ï 2
î 3 ï2
î 3 î
ï è 3 ø
 

En x = –2 creix i en x = 0 creix.
 
48. Estudia el creixement i el decreixement de les funcions que tens a continuació en els punts
indicats:
a) ���� � |�� � �| en x = –3, x = –1 i x = 4
b) ���� � |��� � ��| en x = –2, x = 2 i x = 6
���
c) ���� � � � en x = –10, x = –5 i x = 1

ì x 2 - 3 si x £ - 3
ï ì2 x
ï si x £ - 3
ï
ï ï
ï
ï ï
a) f ( x ) = íï3 - x si - 3 < x < 3  f ´( x ) = íï-2 x si - 3 < x < 3
2

ï
ï 2 ï
ï
ïïî x - 3 si 3 £ x î2 x
ï
ï si 3 £ x

En x = –3: f ´(–3) = –6 < 0 → f(x) decreix.

En x = –1: f ´(–1) = 2 > 0 → f(x) creix.

En x = 4: : f ´(4) = 8 > 0 → f(x) creix.

ïìï x 2 - 3 x si x £ 0 ïìï2 x - 3 si x £ 0
ï
ï ïï
b) f ( x ) = í
ïï 2
3 x - x 2
si 0 < x < 3  f ´( x ) = í3 - 2 x si 0 < x < 3
ïï
ïîï x - 3 x si 3 £ x îïï2 x - 3 si 3 £ x

En x = –2: f ´(–2) = –7 < 0 → f(x) decreix.

En x = 2: f ´(2) = –1 < 0 → f(x) decreix.

En x = 6: f ´(6) = 9 > 0 → f(x) creix.

ì x +7
ï ïìï 1
ï
ï - si x < -7 ï- si x < -7 ì
ï
ï 4 ïïí 4 ï f ( x) decreix en (-¥, - 7).
c) f ( x ) = í  f ´( x ) =  ïí
ïï x + 7 ïï 1 ï f ( x) creix en (-7, +¥).
ïï si x ³ -7 ïï si x > -7 îï
îï 4 îï 4

695
Aplicacions de la
Aplicacions de la derivada.
derivada. Representació
Representacióde
defuncions
funcions 12

En x = –10 decreix, en x = –5 i x = 1 creix.

49. Determina els intervals de creixement i decreixement de les funcions següents:


a) ���� � |�� � � � �| d) ���� � ��|�|

b) ���� � � � e) ���� � |�� �|


c) ���� � � � f) ���� � ���|�| � ��
�� ��

ì
ïï 2 æ 1- 21ùú éê 1 + 21 ö
ïx - x - 5 si x Î ççç-¥, È , + ¥÷÷÷
ï
ï çè 2 ûúú ëêê 2 ÷ø
a) f ( x ) = x 2 - x - 5 = ïíï
ï 2 æ1- 21 1+ 21ö÷
ï ï - x + x + 5 si x Î ççç , ÷÷
ï
ï çè 2 2 ÷ø
ï
î

ïì ïìï 1
ïïï æ 1- 21÷ö÷ çæ 1 + 21 ö ïï2 x - 1 > 0 si x > 2
ïï2 x - 1 ç ÷ ï
si x Î çç-¥, ÷Èç , + ¥÷÷÷  í
ïï è 2 ø÷ èç 2 ø ïï 1
ïï ïï2 x - 1 < 0 si x <
ï
ï ïî 2
f ´( x ) = í
ïï ìï
ïï ïï-2 x + 1 > 0 si x < 1
ïï-2 x + 1 si x Î æçç 1- 21 , 1 + 21ö÷÷  ïï 2
÷ í
ïïï ççè 2 2 ÷ø ïï 1
ïï ïï-2 x + 1 < 0 si x >
ïî ïî 2

æ 1- 21ö÷÷ æç 1 1 + 21ö÷÷ æ 1- 21 1 ö÷ æ 1 + 21 ö÷
Per tant, la funció decreix en ççç-¥, ÷Èç ,
ç
÷ i creix en çç , ÷÷ È ççç , + ¥÷÷ .
çè 2 ÷ø èçç 2 2 ÷ø çè 2 2 ø÷ èç 2 ø÷

ìï 1 ì
ï1
ï- si x < 0 ï si x < 0
1 ïïï x ï
ï x2 ïìf ´( x ) > 0 si x < 0
b) f ( x ) = = íï  f ´( x ) =ïí  ïí
x ï1 ï 1 ïf ´( x ) < 0 si x > 0
ï
ï si x ³ 0 ïïï- 2 si x > 0 îï
ï
î x îï x

Per tant, la funció creix en l’interval (–∞, 0) i decreix en l’interval (0, +∞).

ì
ï x2 -1
ì
ï x ï
ï si x < 0
ï
ï- si x < 0 ï
ïï( x 2 + 1)
2
x ï x2 +1 ì
ï f ´( x ) < 0 en x Î (-1, 0 )È (1, + ¥)
c) f ( x ) = 2 ï
=í  f ´( x ) = ï í ïí
ï x
x +1 ï ï 1- x 2
ï ï f ´( x ) > 0 en x Î(-¥, - 1) È (0, 1)
ï
ï 2
si x ³ 0 ï
ï si x > 0 îï
ïx +1
î ïï( x 2 + 1)2
ïî

La funció decreix en (-1, 0) È (1, + ¥) i creix en (-¥, - 1) È (0, 1) .

ì
ïï 1
ï
si x < 0
ì
ï- - x si x < 0 ïï 2 - x ìïf ´( x ) < 0 si x > 0
d) f ( x ) = - x = ïí  f ´( x ) = í ïí
ï
ï- x si x ³ 0 ï
ïï- 1 ïf ´( x ) > 0 si x < 0
ï
î
ï si x > 0 îï
î 2 x
ï
ï

La funció creix en l’interval (–∞, 0) i decreix en (0, +∞).

ìï 1
ïï- si 0 < x < 1 ì
ìïï-ln x si 0 < x £ 1 ï x ïf ´( x ) < 0 si 0 < x < 1
e) f ( x ) = ln x = í  f ´( x ) = ïí  ïí
îïf ´( x ) > 0 si x > 1
ïîïln x si x > 1 ï 1 ï
ïïï si x > 1
îï x

La funció decreix en l’interval (0, 1) i creix en (1, +∞).

696
Aplicacions de la derivada. Representació de funcions 1212

ìï
ïï- 1 si x < 0
ïìln(1- x ) si x < 0 ï 1- x ïìf ´( x ) < 0 si (-¥, 0)
f) f ( x ) = ln( x + 1) = ïí  f ´( x ) = ïí  ïí
ïln( x + 1) si x ³ 0 ï
ïï 1 ï f ´( x ) > 0 si (0, + ¥)
îï si x > 0 îï
ïïî x + 1

La funció decreix en l’interval (–∞, 0) i creix en l’interval (0, +∞).

50. Determina els màxims i els mínims de les funcions següents utilitzant la derivada segona:
� � ����� ��
a) ���� � d) ���� �
��� �� ��
�� � � �����
b) ���� � e) ���� �  
��� ���

� � �� ��
c) ���� �   f)  ���� �  
� �� ��
x ( x - 2)
a) f ´( x ) =  f ´( x ) = 0 quan x = 0 i x = 2.
( x - 1)2

2 ìEn x = 0  f ´´(0) = -2 < 0 ìïx = 0 màxim


ï
f ´´( x) =  ïí ï
í
( x -1)3
ïïEn x = 2  f ´´( 2) = 2 > 0 ï
ïx = 2 mínim
î
î

x (4 - x )
b) f ´( x ) =  f ´( x ) = 0 quan x = 0 i x = 4.
(2 - x )2

8 ïìEn x = 0  f ´´(0) = 1 > 0 ì


ïx = 0 mínim
f ´´( x) =  íï  íï  
îïEn x = 4  f ´´(4) = -1 < 0 î
(2 - x)3
ï ïïx = 2 màxim

( x + 3)( x - 3)
c) f ´( x ) =  f '( x ) =0 quan x = 3 .
x2

ìï
ïEn x = -3  f ´´(-3) = - 2 < 0
18 ïïï 3 ïì x = -3 màxim
f ´´( x) = 3  í  ïí
x ï
ïïEn x = 3  f ´´(3) = 2 > 0 ïïî x = 3 mínim  
ïîï 3

x 2 ( x 2 + 12)
d) f ´( x ) =  f ´( x ) = 0 quan x = 0.
( x 2 + 4)2

8 x ( x 2 - 12)
f ´´( x ) = -  En x = 0 f ´´(0 ) = 0  
( x 2 + 4)3

Mitjançant aquest mètode, no podem concloure si és un màxim o un mínim.

x ( x + 2)
e) f ´( x ) =  f ´( x ) = 0 quan x = 0 i x = –2.
( x + 1)2

ì
f ´´( x) =
2
 ïíEn x = 0  f ´´(0) = 2 > 0
ï ìx = 0 mínim
ï
 ïí  
îEn x = -2  f ´´(-2) = -2 < 0 ï
( x +1)3
ïï îïx = 2 màxim

8( x 2 - 1)
f) f ´( x ) = -  f ´( x ) = 0 quan x =  1 .
( x 2 + 1)2

697
Aplicacions de la
Aplicacions de la derivada.
derivada. Representació
Representacióde
defuncions
funcions 12

16 x( x 2 - 3) ïìïEn x = -1  f ´´(-1) = 4 > 0 ï ìx = -1 mínim


f ´´( x) = í  ïí  
2
( x +1) 3
ï
îïEn x = 1  f ´´()
1 = -4 < 0 ï
ïx = 1 màxim
î

51. Troba els màxims i els mínims de les funcions següents per mitjà de la derivada segona:
a) ���� � �� � � d) ���� � ��� � ��� � �
����
b) ���� � e) ���� � ��� � ���

c) ���� � �� � � �� f) ���� � �� � ���

a) f ´( x) = 3 > 0  f ´( x) ¹ 0 "x Î   No existeix màxim ni mínim per aquesta funció.

1
b) f ´( x ) = - < 0  f ´( x ) ¹ 0 " x Î   No existeix màxim ni mínim per aquesta funció.
2

c) f ´( x ) = 6 x 2 - 6 = 6( x 2 - 1)  f ´( x ) = 0 quan x 2 - 1 = 0  x =  1

ìï f ´´(- 1) = - 12 < 0 ìïï x = - 1 màxim


f ´´( x ) = 12 x  íï í
ïîï f ´´(1) = 12 > 0 ïîï x = 1 mínim

d) f ´(x) = 4x -12 = 4(x -3)  f ´(x) = 0 quan 4(x -3) = 0  x = 3

f ´´( x ) = 4  f ´´(3) = 4 > 0  x = 3 mínim

e) f ´( x ) = 8 x 3 - 8 x = 8 x ( x 2 - 1)  f ´( x ) = 0 quan 8 x ( x 2 - 1) = 0  x = 0 i x =  1

ïìï f ´´(0) = -8 < 0 ïìï x = 0 màxim


ïï ï
f ´´( x) = 8(3x -1)  í f ´´(-1) = 16 > 0  íï x = -1 mínim
2
ï
ï ï
ïï x = 1 mínim
ï f ´´(1) = 16 > 0
ï
î îï

3
f) f ´( x) = 4 x3 - 6 x2 = 2 x2 (2 x - 3)  f ´( x) = 0 per x = 0 i x = .
2

ì
ïï f ´´(0) = 0 ì
ïx = 0 no es pot decidir amb aquest mètode.
ï
ï
f ´´( x) = 12 x( x -1)  í æ 3 ö÷ ï
í 3  
ï
ï f ´´ççç ÷÷ = 9 > 0 ï
ï x = mínim
ï
ï
î è 2 ø ï
î 2

52. Fes servir la derivada segona per calcular l’expressió algèbrica de les coordenades del vèrtex
d’una paràbola genèrica del tipus f (x) = ax2 + bx + c.
Com que el vèrtex és un màxim o un mínim, ha de complir l’equació f(x) = 0. A més, i per tant en el vèrtex,
quan igualem ambdues equacions, s’obté:

b æ -b ö b2
0 = 2ax + b  x = - , i en substituir tenim f ççç ÷÷÷ = c - , ara ja es pot donar una expressió algèbrica pel
2a è 2a ø 4a

vèrtex d’una paràbola:

698
Aplicacions de la derivada. Representació de funcions 1212

æ b 2 ö
çç- , c - b ÷÷ en funció dels paràmetres a, b i c.
çè 2 a ÷
4a ø

53. Determina els valors de a, b i c perquè la paràbola f (x) = ax2 + bx + c tingui el vèrtex en el punt (–1,
–8) i talli l’eix Y en (0, –5).
æ b 2 ö
çç- , c - b ÷÷ = (- 1, 8) i 5  a02  b0  c → c  5, per tant:
çè 2 a ÷
4a ø  
b ïü
- =- 1ïï ü
b = 2a ï
ïï b = 2aü
ï
ïï b = 2aü
ï
ïï b = 6üï
2a ïýï  2
ý  (2a) 2
ý  4 a2 ý ï
ý 
ïï -5 - b
b 2
= -8ïï =3 ïï = 3 ï
ï a = 3þï
ï
-5 - = -8ï 4 a ï
ï
þ 4 a ï
ï
þ 4 a ï
ï
þ
4a ïïþ

 
54. Determina el vèrtex de les paràboles següents tenint en compte que, en aquest vèrtex, la tangent a
la corba és horitzontal. Indica si es tracta d’un màxim o un mínim.
a) ࢌሺ࢞ሻ ൌ ૜࢞૛ െ ૟࢞ ൅ ૞ d) ࢌሺ࢞ሻ ൌ െ࢞૛ ൅ ૝࢞
b) ࢌሺ࢞ሻ ൌ െ૛࢞૛ ൅ ૟࢞ ൅ ૢ e) ࢌሺ࢞ሻ ൌ ૜࢞૛ ൅ ࢞ ൅ ૢ
࢞૛ ା૝࢞ା૜
c) ࢌሺ࢞ሻ ൌ ࢞૛ ൅ ૛࢞ െ ૞ f) ࢌሺ࢞ሻ ൌ

Si la tangent a la corba és horitzontal en el vèrtex, significa que la primera derivada (pendent de la tangent
en aquest punt) és igual a zero.

a) f ´( x ) = 0  6 x - 6 = 0  x = 1

f ´´( x) = 6  f ´´(1) = 6 > 0  x = 1 mínim

3
b) f ´( x ) = 0  -4 x + 6 = 0  x =
2

æ 3ö 3
f ´´( x) = -4  f ´´ççç ÷÷÷ = -4 < 0  x = màxim
è 2ø 2

c) f ´( x ) = 0  2 x + 2 = 0  x = - 1

f ´´( x) = 2  f ´´(-1) = 2 > 0  x = -1 mínim

d) f ´( x ) = 0  - 2 x + 4 = 0  x = 2

f ´´( x) = -2  f ´´(2) = -2 < 0  x = 2 màxim

-1
e) f ´( x ) = 0  6 x + 1 = 0  x =
6

æ -1ö -1
f ´´( x) = 6  f ´´ççç ÷÷÷ = 6 > 0  x = mínim
è 6 ø 6

f) f ´( x ) = 0  x + 2 = 0  x = - 2

f ´´( x) = 1  f ´´(-2) = 1 > 0  x = -2 mínim  

699
Aplicacions de la
Aplicacions de la derivada.
derivada. Representació
Representacióde
defuncions
funcions 12

55. Troba els valors de a i b perquè la funció f(x) = x3 + ax + b tingui un mínim en (1, 5).
En substituir en la funció, tenim que 5 = 13 + a1 + b = 1 + a + b  4 = a + b , i atès que es té mínim en (1, 5),
tenim que f ´( x ) = 3 x 2 + a  f ´(1) = 0  0 = 3 + a  a = - 3 , i s’obté:

a+b= 4 üï ì
ï ïb = 7
ýï
í
a = -3ï
ï ï
þ ï a = -3
î

56. Calcula els valors de a, b i c en la funció f(x) = ax3 + bx2 + x + c si saps que la seva gràfica passa per
(–1, 9) i en els punts d’abscissa x = 0 i x = –2 té tangent horitzontal.
És impossible que en x = 0 la derivada d’aquesta funció hi hagi tangent horitzontal, atès que
f ´( x ) = 3ax 2 + 2bx + 1 i f ´(0) = 1, per tant, es conclou que no existeix una funció amb aquestes

característiques.

57. Determina els valors de a, b i c perquè la funció f(x) = ax3 + bx + c passi pel punt (2, 6) i tingui un
mínim en (1, 2).
Passa per (2, 6)  6 = a(2)3 + b(2) + c  6 = 8a + 2b + c

Passa per (1, 2)  2 = a(1)3 + b(1) + c  2 = a + b + c

Com que a més en (1, 2) té un mínim, és obligat que:

f ´( x ) = 3ax 2 + b  f ´(1) = 3a + b = 0

Així que es tenen tres equacions i tres incògnites:

8 a + 2b + c = 6üï 8 a + 2b + c = 6 üï ìïa = 1
ïï ïï 8 a - 6 a + c = 6üïï 2a + c = 6üïï ïïï
a + b + c = 2ýï  a + b + c = 2 ýï  ý  3
ý íb = -3  f ( x ) = x - 3 x + 4

ï
ï ï
ï a - 3 a + c = 2 ï
þï -2 a + c = 2 ï ï
þï ïïc = 4
3 a + b = 0ï
ïþ b = -3 aï
ïþ îï

Es comprova que x = 1 és un mínim, atès que f ´´( x ) = 6 x  f ´´(1) = 6 > 0 .

58. Calcula a, b i c perquè la funció f(x) = ax3 + bx2 + cx + d tingui dos punts de tall amb l’eix de les X en
ି૚૙
x = –4 i x = –3, i dos punts de tangent horitzontal en x = –4 i x = . Indica si aquests dos últims

punts són màxims o mínims.

Com que la seva gràfica passa per (–4, 0) i (–3, 0), s’obtenen les equacions:

ìï0 = a(-4)3 + b(-4)2 + c(-4) + d  0 = -64 a + 16b - 4c + d


ïí
ïïî0 = a(-3)3 + b(-3)2 + c(-3) + d  0 = -27a + 9b - 3c + d

10
A més, se sap que la seva derivada s’anul·la en els punts x = –4 i x = - , així s’obtenen les equacions:
3

700
Aplicacions de la derivada. Representació de funcions 1212

ìï0 = 3 a(-4)2 + 2b(-4) + c  0 = 48 a - 8b + c


ïï
ïí 2
ïï0 = 3 a æçç- 10 ö÷÷ + 2b æçç- 10 ÷÷ö + c  0 = 100 a - 20 b + 3c
ïïî èç 3 ø÷ èç 3 ÷ø

ì-64 a + 16b - 4c + d = 0
ï
ï
ï
ï -27a + 9b - 3c + d = 0
S’obtenen quatre equacions amb quatre incògnites: ïí
ï
ï 48 a - 8b + c = 0
ï
ï
ï
ï
î 100 a - 20 b + 3c = 0

El sistema és compatible indeterminat, per tant no existeix una única solució. Deixem d com a variable
lliure:

5d 11d d
c= ,b= ,a=
6 48 48

59. Estudia el creixement, el decreixement i els màxims i els mínims d’aquesta funció:

૚૜ ૛
ࢌሺ࢞ሻ ൌ ඨ૛࢞૜ െ ࢞ െ ૞࢞ 

ìï 13 2 ü éê 13 - 329 ùú éê 13 + 329 ö÷
Dom f = í x Î  : 2 x 3 - x - 5 x > 0ï
ý= ê , 0ú È ê , + ¥÷÷
ïîï 2 ï
þï ëê 8 8 ÷ø
ûú ëê

6 x2 -13x - 5 1 5
f ´( x) =  f ´( x) = 0 quan 6 x2 -13x - 5 = 0  x = - y x = .
13 2 3 2
2 x3 - x - 5x
2

5
Dels dos possibles candidats a punt crític, descartem x = per no trobar–se dins el domini de la funció.
2

1
Calculem la segona derivada per comprovar si x = - és un màxim o un mínim:
3

12 x 4 - 52 x3 - 60 x 2 - 25 æ 1ö 3 1
f ´´( x) =  f ´´ççç- ÷÷÷ = -51 < 0  x = - màxim
2
2( x(4 x -13x -10)) 3/2 è 3ø 94 3

æ 13 - 329 1ö÷ æ 1 ö
La funció creix en l’interval ççç , - ÷÷÷ , i decreix en l’interval ççç- , 0÷÷÷ .
è 8 3ø è 3 ø

60. Calcula els valors de a, b i c perquè la funció f(x) = x3 + ax2 + b tingui un màxim en un punt en què
talla l’eix X i un mínim en el punt de tall amb l’eix Y, i compleix que a + b = –1.
Els punts de tall són el (0, b) i el (x0, 0), on x0 serà una arrel del polinomi x 3 + ax 2 + b .

En aquests punts, la derivada s’anul·la; en el cas de (0, b) és un mínim, així que la segona derivada en
aquest és positiva; en el cas (x0, 0) és un màxim, així que la segona derivada en aquest és negativa:

701
Aplicacions de la
Aplicacions de la derivada.
derivada. Representació
Representacióde
defuncions
funcions 12

ìïf ´(0 ) = 3 ⋅ 0 2 + 2 a ⋅ 0 = 0
ïï ì
ìïf ´( x ) = 3 x 2 + 2ax ïïf ´´(0 ) = 6 ⋅ 0 + 2 a = 2 a > 0 ïïï x 0 = 0
ïí  ïí  í 2
ïîïf ´´( x ) = 6 x + 2a ïïïf ´( x 0 ) = 3 x 0 + 2 ax 0 = 0 ïïï x 0 = - a
2

ïïf ´´( x ) = 6 x + 2 a < 0 î 3


ïî 0 0

Es rebutja x0 = 0 perquè, en aquest cas, f ´´( x0 ) = 6 x0 + 2a = 2a < 0 , que seria contradictori amb f ´´(0) = 2a > 0 .

3 2
æ 2 a ö÷ æ ö 3 3 3
Per tant, çç- ÷ + a çç- 2 a ÷÷ + b = 0  - 8 a + 12 a + b = 0  4 a + b = 0 .
çè 3 ø÷ çè 3 ø÷ 27 27 27

Per últim, atès que es compleix l’equació a + b = -1 , es resol el sistema:

4a 3 ïü ìïa = -3 y b = 4  Es descarta perquè 2a > 0.


+ b = 0ïï 3
ïï
27  4 a - 27 a + 27 = 0 
ïïa = 3 y b = -1
ý í
ï
a + b = 1 ïïïþ ïî 2 2

Així que la funció cercada serà:

3 1
f ( x) = x3 + x2 -  
2 2

61. Indica quins han de ser els valors de a, b, c i d perquè la funció f(x) = ax3 + bx2 + cx + d compleixi
que:
Tingui un màxim en el vèrtex de la paràbola g(x) = x2 + 12x + 202.
Tingui un mínim en el vèrtex de la paràbola h(x) = –2x2 + 8x – 98.
b b
El màxim tindrà com a coordenades x = - = -6 i g (-6) = 238 i el mínim x = - = 2 i h(2) = -90 .
2a 2a

Per tant:

ìïf ´(-6) = 108 a - 12b + c = 0


ïï
ìïf ´( x ) = 3 a x 2 + 2bx + c ïïf ´´(-6) = -36 a + 2b < 0 ïì108 a - 12b + c = 0
ïí  ïí  ïí
ïîïf ´´( x ) = 6 ax + 2b ïïï
ïf ´(2) = 12 a + 4b + c = 0 ïîï12a + 4 b + c = 0
ïîïf ´´(2) = 12a + 2b > 0

ïì-216 a + 36b - 6c + d = 238


D’altra banda, també tenim que: ïí
ïïî8 a + 4 b + 2c + d = -90

Per tant, tenim un sistema de quatre equacions amb quatre incògnites, i quan el resolem s’obté els valors
dels paràmetres.

108 a - 12b + c = 0 ïüï


ïï
12a + 4b + c = 0 ï 41 123 369 -155
ý a= ,b= ,c=- ,d=
-216 a + 36b - 6c + d = 238 ïï 32 16 8 4
ï
8 a + 4b + 2c + d = -90ïïïþ

123 2 369 155


Per tant, la funció cercada serà: f ( x ) = 32 x 3 + x - x-
16 8 4

702
Aplicacions de la derivada. Representació de funcions 1212

62. Determina els punts de les gràfiques d’aquestes funcions, que tenen la tangent horitzontal:
a) f(x) = 3x2 – 15x + 13
b) f(x) = 2x3 + 3x2 – 36x + 8
c) f(x) = 2x3 + 3x2 + 6x – 12
�� �����
d) ���� �
���
���
e) ���� �
���
La tangent és horitzontal si el pendent és igual a zero.
a) f’(x) = 6x – 15

6x – 15 = 0 → x =

b) f’(x) = 6x2 + 6x – 36
��2
6x2 + 6x – 36 = 0 → x2 + x – 6 = 0 → �
� � ��
c) f’(x) = 6x2 + 6x + 6
6x2 + 6x + 6 = 0 → x2 + x + 1 = 0
L’equació no té solució, així que no hi ha punts que tinguin tangent horitzontal.
�������������� � ������ � � ���
d) ����� � �
������ ������

� � ��� ��0
� 0 � � � � 2� � 0 � �
������ � � �2
��������� ��
e) ����� � �
������ ������

��
� 0 � �� � 0
�� � 2��

L’equació no té solució, i no hi ha punts que tinguin tangent horitzontal.

  

63. Indica en quins punts de les gràfiques d’aquestes funcions la tangent és horitzontal. Determina si
són màxims o mínims.
�� ����� �� ��
a) ���� � c) ���� �
��� �
�� ��
b) ���� � d) ���� �
��� �� ��

�������������� � ������ � � ���


a) ����� � � ������
������

� � ��� ��0
� 0 � � � � 2� � 0 � �
������ ��2
2
������ �
�� � ���

� �� �2� � 2 � 0 � ���� � 2 té un mínim.

� �� �0� � �2 � 0 � ���� � 0 té un màxim.

703
Aplicacions de la
Aplicacions de la derivada.
derivada. Representació
Representacióde
defuncions
funcions 12

��������� � �� ���� �
b) ����� � � ������
������

� 0 � 4� � � � � 0 � �� � 4
���� �
������ ��0
8
������ �
�2 � ���

� �� �0� � 1 � 0 � ���� � �1 té un mínim.

� �� �4� � �1 � 0 � ���� � 1 té un màxim.

������� � ��� � � ��
c) ����� � �
�� ��
� � ��
��
� 0 � � � � � � 0 � � � �3

2
2��2 � �� � �� � �� � 2� 18
������ � � 3
�4 �

� �� ��3� � � � 0 � ���� � �3 té un màxim.


� �� �3� � � 0 � ���� � 3 té un mínim.

�� � ��� � ����� � ��� � � ���� �


d) ����� � �
�� � ���� �� � ����

� � ���� �
� 0 � � � � 12� � � 0 � � � 0
�� � ����

2 4
�4�3 � 24�� � ��2 � 4� � �� � 12�2 � � 2��2 � 4� � 2� ��� � 8�3
������ � 4
� 3
��2 � 4� 2
�� � 4�
� �� �0� � 0 � ���� � 0 no té un màxim ni un mínim.

   

704
Aplicacions de la derivada. Representació de funcions 1212

64. Indica tota la informació que sigui possible sobre el punt x = a de la funció f(x) en els casos
següents:
ࢌԢሺࢇሻ ൌ ૙ ࢌԢሺࢇሻ ൌ ૙
a) ൜ d) ൜
ࢌᇱᇱ ሺࢇሻ±࢙࢔ࢋࢍࢇ࢚࢏࢜ࢇ ࢌᇱᇱ ሺࢇሻ  ൌ ૝
ࢌሺࢇሻ ൌ ૙ ࢌԢሺࢇሻ ൌ ૙
b) ൜ e) ൜
ࢌԢሺࢇሻ  ൌ ૙ ࢌᇱᇱ ሺࢇሻ  ൌ ૙
ࢌԢሺࢇሻ ൌ ૙ ࢌԢሺࢇሻ ൌ ૙
c) ቐ ࢌሺࢇሻ  ൌ ૜ f) ቐࢌᇱ ሺ࢞ሻ ൐ ૙࢖ࢋ࢘࢞ ൏ ࢇ
ࢌሺ࢞ሻ ൑ ૜࢙ࢋ࢓࢖࢘ࢋ ࢌᇱ ሺ࢞ሻ ൏ ૙࢖ࢋ࢘࢞ ൐ ࢇ
a) x = a és un màxim.

b) x = a és un punt crític, no obstant no sabem si és màxim o mínim.

c) x = a és un punt crític i com que f ( x ) £ 3 sempre i f(a) = 3 és un màxim.

d) x = a és un mínim.

e) No es pot decidir si hi ha màxim o mínim en x = a.

f) x = a és un màxim, perquè f ´(a) = 0, creix en x < a i decreix en x > a.

 
65. Assenyala els intervals de creixement i decreixement, els màxims i els mínims, com també els
valors, de les funcions següents:

 
a) Decreix en (-¥, -1) È (0, 1) i creix en (-1, 0) È (1, +¥) .

Té màxim en x = 0, i el valor és –1.

Té dos mínims, en x = –1 i x = 1, ambos amb valor –2.

705
Aplicacions de la
Aplicacions de la derivada.
derivada. Representació
Representacióde
defuncions
funcions 12

b) Decreix en (-¥, 0) È (2, + ¥) i creix en (0, 2) .

Té un mínim en x = 0, i el valor és –3, i un màxim en x = 2, i el valor és 1.

c) Creix en (-¥, - 1) È (-1, 1) i decreix en (1, 3) È (3, + ¥) .

Té un màxim en x = 1, i el valor és, aproximadament, –0,25.

66. Calcula els valors indicats per a la funció f(x) = x4 – 2x3 – 3x2 + 4x + 4 i dedueix tota la informació
que es pugui a partir d’aquests valors:
a) f'(–1), f'(–1,5), f'(–0,5), f(–1) b) f'(0,5), f(0,5), f'(0,25), f(1) c) f(2), f'(2), f'(1,5), f'(2,5)
3 2
f ´( x ) = 4 x - 6 x - 6 x + 4

a) f ´(-1) =-4 -6 +6 +4 = 0

27 27
f ´(-1,5) = - - + 9 + 4 = 13 - 27 = -14
2 2

1 3
f ´(-0,5) = - - + 3 + 4 = 7 - 2 = 5
2 2

f (- 1) = 1 + 2 - 3 - 4 + 4 = 0

La gràfica talla amb l’eix X en x = –1. És un punt crític, perquè el valor de la derivada és 0.

A l’esquerra la funció decreix perquè el valor de la derivada és negatiu, i a la dreta creix perquè és
positiu.

Per tant, (–1, 0) té un mínim.

1 3
b) f ´(0,5) = - - 3 + 4 = 0
2 2

1 1 3 81
f (0,5) = - - +2+4 =
16 4 4 16

1 3 3 65 30 35
f ´(0,25) = - - +4 = - =
16 8 2 16 16 16

f (1) = 1 - 2 - 3 + 4 + 4 = 9 - 5 = 4

Té un punt crític en x = 0,5 perquè el valor de la derivada és 0. A l’esquerra la funció creix, perquè el
81
valor de la derivada és positiu, i a la dreta decreix, perquè >4 .
16

æ 81ö
Per tant, en ççç0,5; ÷÷÷ és un mínim.
è 16 ø

c) f (2) = 16 - 16 - 12 + 8 + 4 = 0

f ´(2) = 32 - 24 - 12 + 4 = 0

706
Aplicacions de la derivada. Representació de funcions 1212

27 27
f ´(1,5) = - - 9 + 4 = -5
2 2

125 75 30 8 133 - 105 28


f ´(2,5) = - - + = = = 14
2 2 2 2 2 2

La gràfica talla amb l’eix X en x = 2. És un punt crític, perquè el valor de la derivada és 0.

A l’esquerra la funció decreix, perquè el valor de la derivada és negatiu, i a la dreta creix, perquè és
positiu.

Per tant, en (2, 0) és un mínim.

67. Troba els màxims i els mínims d’aquestes funcions:


a) f(x) = (x – 1)2 · (x + 3)2
b) f(x) = (x – 2)2 · (x – 1)2
c) f(x) = (x + 2)2 · (x – 1)2
a) f ´( x ) = 4( x -1)( x +1)( x + 3)  f ´( x ) = 0  x = 1, x =-1 y x =-3

ìï f ´´(-1) = -16 < 0  x = -1 màxim


ïï
f ´´( x) = 4(3x 2 + 6 x -1)  ïí f ´´(1) = 32 > 0  x = 1 mínim
ïï
ïïî f ´´(-3) = 32 > 0  x = -3 màxim

3
b) f ´( x ) = 2( x - 1)( x - 2)(2 x - 3)  f ´( x ) = 0 « x = 1, x = 2 y x =
2

ì f ´´(1) = 2 > 0  x = 1 mínim


ï
ï
ï
ï
ï æ 3ö 3
f ´´( x ) = 2(6 x -18 x + 13)  í f ´´ççç ÷÷÷ = -1 < 0  x = màxim
2
ï
ï è 2 ø 2
ï
ï
ï
ï
î f ´´(2) = 2 > 0  x = 2 mínim

1
c)  f ´( x ) = 2( x - 1)( x + 2)(2 x + 1)  f ´( x ) = 0  x = 1, x = -2 y x = -  
2

ì f ´´(-2) = 18 > 0  x = -2 mínim


ï
ï
ï
ï
ï æ 1ö 1
f ´´( x ) = 6(2 x + 2 x -1)  í f ´´ççç- ÷÷÷ = -9 < 0  x = - màxim  
2
ï
ï è 2 ø 2
ï
ï
ï
ï
î f ´´(1) = 18 > 0  x = 1 mínim

68. Descompon el nombre 20 en dos sumands de manera que la suma dels seus quadrats sigui
mínima.
݂ሺ‫ݔ‬ሻ ൌ ʹ‫ ݔ‬ଶ െ ͶͲ‫ ݔ‬൅ ͶͲͲ
ʹͲ ൌ ‫ ݔ‬൅ ‫ݕ‬ ʹͲ െ ‫ ݔ‬ൌ ‫ݕ‬ ᇱ ሺ‫ݔ‬ሻ
ቐ ଶ ՜ቐ ଶ ՜ ቐ݂ ൌ Ͷ‫ ݔ‬െ ͶͲ ൌ Ͳ ՜ ‫ ݔ‬ൌ ͳͲ
‫ ݔ‬൅ ‫ ݕ‬ଶ ൌ ݂ሺ‫ݔ‬ǡ ‫ݕ‬ሻ ‫ ݔ‬൅ ሺʹͲ െ ‫ݔ‬ሻଶ ൌ ݂ሺ‫ݔ‬ሻ
݂ ᇱᇱ ሺ‫ݔ‬ሻ ൌ Ͷ ൐ Ͳ•‡’”‡

Com que la segona derivada és positiva sempre, tenim que x = 10 és un mínim.

y = 20 - x  y = 10  La descomposició demanada és 20 = 10 + 10.

707
Aplicacions de la
Aplicacions de la derivada.
derivada. Representació
Representacióde
defuncions
funcions 12

69. Troba el nombre positiu que fa mínima la suma d’aquest mateix nombre i el quàdruple del seu
invers.
4
��� � � ��� � � � � � � � � � 2
� 4 �� �
� � � ���� 8
� � �� ��� � � � � �� �2� � � � � � ��� � 2���������������

Descartem x = –2 perquè el nombre que cerquem ha de ser positiu.

70. De tots els triangles rectangles en els quals els catets sumen 10, quin té l’àrea més gran?
ì 10 = x + y ì
ï
ï
ï ï10 - x = y
ï
ïì f ´( x) = 0  5 - x = 0  x = 5
í xy ïí x(10 - x) ï í
ïï = f ( x, y ) ïï = f ( x) îïï f ´´( x) = -1 < 0 sempre  En x = 5 hi ha un màxim.  
ï2
î îï 2

y = 10 - x  y = 5  El triangle amb l’àrea més gran és el que té els catets x = 5 i y = 5.

71. Entre tots els triangles rectangles d’hipotenusa 10, troba el que té l’àrea més gran.
ì
ïï10 - x = y
2 2
ì
ïï10 = x + y
2 2

ï ï
ï 10
í xy
ïï = f ( x , y )
 í x (10 - x 2 )
ï
ìf '( x ) = 0  10 - 3 x 2 = 0  x =
ï
ï  
ï = f ( x )  í 3
ïîï 2 ï 2 ï
ï
î îïf ''( x ) = -6 x

æ 10 ö÷
f ´´çç ÷ = -6 ⋅ 10 < 0  x = 10 és un màxim.  
çè 3 ÷÷ø 3 3

10 20 20
y 2 = 10 - x 2  y 2 = 10 - = y=  El triangle amb l’àrea més gran és el que té els catets
3 3 3
10 20
x= iy= . 
3 3
 
72. Volen construir el marc d’una tanca publicitària rectangular de 12 m2; el metre lineal de tram
horitzontal costa 1,50 € i el metre lineal de tram vertical costa 2 €.
a) Determina les dimensions de la tanca perquè el cost sigui mínim.
b) Quant val el marc?
ì ì
ï
ïï y = 12
2
ïìï xy = 12
ï ïïf ´( x ) = 3 x - 48 = 0  x = 4
ï ï x ï 2x2
a) í3 ï
í ï
í
ïïï x + 2 y = f ( x , y ) ï
ï 3 x 24
ïï +
ï
= f(x) ï
48
ïïf ´´( x ) = 3  f ´´(4) =
48
>0
î2
î 2
ï x ï
î x 64

12
A x = 4 hi ha un mínim. y =  y = 4  Les dimensions de la tanca són 4 m de llarg i 3 m d’ample.
x

b) El cost del marc serà 1,5 + 2 · 3 = 12 €.

708
Aplicacions de la derivada. Representació de funcions 1212

73. Amb 200 m de tela metàl·lica volen tancar un recinte format per un rectangle i dos semicercles, de
la mateixa manera que indica la figura.

 
Determina les dimensions de x i y perquè l’àrea tancada sigui màxima.
py 2
El perímetre del recinte es determina per la fórmula py + 2 x = 200 , i l’àrea per + xy , que és el que
4
volem maximitzar, per tant:
ìï
ì ïï x = 100 - py ìï
ï f ´( y ) = 100 - py = 0  y = 200
ïïpy + 2 x = 200 ï 2 ïï 2 p  = 200 =
ïí 2 ï
í 2 í ï y ,x 0
ï py ï æ ö ï
ïï + xy = f ( x, y ) ï p y py -p p
îï 4 ïï + y ççç100 - ÷÷÷ = f ( y ) ïïï f ´´( y ) = < 0 sempre
îï 4 è 2ø ï
î 2

 
74. Una empresa decideix llançar una campanya de publicitat d’un dels seus productes amb l’edició
d’un text que ocupa 18 cm2 en pàgines rectangulars impreses a una cara, amb els marges superior
i inferior de 2 cm, i els laterals, d’1 cm.
Calcula les dimensions de la pàgina perquè el consum de paper sigui mínim.
ì
ï 10 + 4 x
ï
ïy =
ïïì( x - 2)( y - 4) = 18 ï x -2
í  ïí  
ïïî xy = f ( x , y ) ï
ï æ 10 + 4 x ö÷
ï x çç ÷ = f ( x )
ï ç ÷
î è x -2 ø
ï

4 ( x 2 - 4 x - 5) ì
ï x = -1  Es descarta.
f ´( x) = 0  = 0  íï  
( x - 2) 2 ïï
îx = 5
72 8
f ´´( x) = 3
 f ´´(5) = > 0  x = 5 és un mínim.  
( x - 2) 3

10 + 4 x
y=  y = 10  Les dimensions de la fulla són x = 5, y = 10.
x -2
 
75. Necessitem tancar la màxima àrea possible dins d’un terreny rectangular de 500 m de perímetre.
a) Quines longituds té aquest rectangle?
b) Quines serien les dimensions del rectangle si un dels costats limita amb un riu i no cal posar
tanca en aquesta banda?
ìïï2 x + 2 y = 500 ìï y = 250 - x
a) í ïí
ïîï xy = f ( x , y ) ï
ï x (250 - x ) = f ( x )
î

f ´( x ) = 250 - 2 x  f ´( x ) = 0  x = 125

f ´´( x ) = -2 < 0 sempre  x = 125 és un màxim.

y = 250 - x  y = 125  Les dimensions del rectangle 125125 m, i s’obté un quadrat.

709
Aplicacions de la
Aplicacions de la derivada.
derivada. Representació
Representacióde
defuncions
funcions 12

ì
ï 2 x + y = 500 ì y = 500 - 2 x
ï
b) ï
í ïí
ï
ï
î xy = f ( x , y ) ïï x (500 - 2 x ) = f ( x )
î

f ´( x ) = 500 - 4 x  f ´( x ) = 0  x = 125

f ´´( x) = -4 < 0 sempre  x = 125 és màxim.

y = 250 - 2 x  y = 250  Les dimensions del rectangle cercat són 125250 m.

76. El perímetre d’aquesta figura és 5 m. Calcula les mides x i y perquè l’àrea tancada sigui màxima.

 
2
æ xö
pçç ÷÷÷
æ pö è 2ø px 2
El perímetre de la figura és 2 y + ççç1+ ÷÷÷ x = 5 i la seva àrea és xy + = xy + . Per tant, s’ha de
è 2ø 2 8
resoldre el següent problema de maximització:
ïìï æ pö ïìï 5 xæ pö
2 y + çç1 + ÷÷÷ x = 5 y = - çç1 + ÷÷÷
ïïï èç 2ø ïï 2 2 ç
è 2ø
í  íï
ï
ïï px 2 ï
ï 5 x ç 4 + p ö÷ 2
æ
ïï xy + 8 = f ( x , y ) ïïïf ( x ) = 2 - ççè 8 ÷ø÷ x
î ïî

5 æ 4 + p ÷ö 10
f ´( x ) = - 2 x çç =0 x=
2 çè 8 ÷÷ø 4+p

æ 4 +p ö÷
f ´´( x) = -2çç < 0 sempre
çè 8 ÷÷ø

10 5
Per tant, per x = iy= l’àrea és màxima.
4+p 4+p

 
77. Troba els punts de la corba y2 = 2x la distància dels quals al punt (6, 0) és mínima.
La distància del punt (6, 0) a un punt arbitrari de la corba ( x ,  2 x ) compleix l’expressió de la fórmula

d = ( x - 6)2 + ( 2 x )2 , on d representa el mòdul del vector que té per extrems ambdós punts.

ì
ïï f ( x) = x 2 -10 x + 36 ì
ïï x -5
ï ï f ´( x) = 0  =0 x=5
ï
ï ï
ïï x 2
- 10 x + 36
ï x -5
ï f ´( x) =
ï í
ï 11
í x 2 -10 x + 36 ïï f ´´( x) = > 0 per x = 5  x = 5 és un mínim.
ï
ï ï 3
ï
ï 11 ï
ï
î ( x 2
- 10 x + 36 )
ï f ´´( x) =
ï
ï 3
ï
î ( x 2
- 10 x + 36 )
 

Els punts demanats són els de coordenades x = 5, y =  10 .

710
Aplicacions de la derivada. Representació de funcions 1212

78. Entre totes les rectes que passen pel punt (2, 3), troba la que determina el triangle d’àrea mínima
limitat per aquesta recta i la part positiva dels eixos X i Y.
La recta, en passar pel punt (2, 3) tindrà equació y = mx + (3 - 2 m) , on m és el pendent. Això vol dir que el
triangle cercat tindrà catets de distància els talls de la recta amb els eixos.

xy
D’altra banda, s’ha de maximitzar l’àrea, que serà definida per la fórmula , així obtenim el següent
2
problema de maximització:

ìï
ïï x = 2m - 3
ïìï y = mx + (3 - 2m) ïï m
ï ï
í xy  ïí y = 3 - 2m
ïï = f ( x, y ) ïï
ïî 2 ïï -4 m2 + 12m - 9
ïïf ( m) =
ïî 2m

ìï 2
ïï f ´´(m) = -4m + 4m - 9
ìï 2 ïï m3
ïï f ´(m) = -8m + 18 ïï
ïï 4m 2 ï æ 3ö 3
í  ïí f ´´çç ÷÷÷ < 0  m = es descarta.
ïï-8m + 18
2
3 ï ï ç
è 2 ø 2
ïï 2
=0m= ïï
îï 4m 2 ïï çæ -3 ÷ö -3
ïï f ´´çç ÷÷ > 0  m = dóna un àrea mínima.
ïî è 2 ø 2

3
Per tant la recta buscada serà y = - x + 6 .
2

79. Representa gràficament una funció que compleixi les condicions següents:
 El seu domini és � i no té asímptotes.
 Creix en (–∞, 0) , (2, +∞) i decreix en (0, 2).
 En els punts d’abscissa x = 0 i x = 2 la tangent a la corba és horitzontal.
Resposta oberta. Per exemple:

 
 
80. Representa gràficament una funció que verifiqui les característiques següents:
 El seu domini és �.
 Sempre creix i no té asímptotes.
 Talla l’eix Y en (0, –8). A més, en el punt (2, 0) té tangent horitzontal i f"(2) = 0.
Resposta oberta. Per exemple:

711
Aplicacions de la
Aplicacions de la derivada.
derivada. Representació
Representacióde
defuncions
funcions 12

 
81. Representa gràficament una funció que compleixi les característiques següents:
 El seu domini és �.
 Té una asímptota horitzontal en y = 3.
 Talla els eixos únicament en (0, 0), punt en el qual té un mínim.
 Decreix en (–∞, 0) i creix en (0, +∞).
Resposta oberta. Per exemple:

82. Representa gràficament una funció que compleixi les condicions següents:
 El seu domini és � – { –3 }.
 Té dues asímptotes, una de vertical en x = –3 i una d’horitzontal en y = –1.
 Sempre creix.
 Talla els eixos en (–6, 0) i (0, –4).
Resposta oberta. Per exemple:

   

712
Aplicacions de la derivada. Representació de funcions 1212

83. Representa gràficament una funció que verifiqui les característiques següents:
 El seu domini és �.
 Talla els eixos en (–1, 0), (–2, 0) i (0, 0).
�� ��
 Creix en � , � i decreix en la resta.
� �
�� � �� ��
 � , � o és un màxim i � , � és un mínim.
� � � �

Resposta oberta. Per exemple:

84. Representa una funció amb aquestes característiques:


 El seu domini és � – {–3, 3 }.
 Té asímptotes verticals en x = –3 i x = 3.
 f'(x) < 0 en tot el domini.

 
 
85. Representa gràficament una funció que tingui les característiques següents:
 El seu domini és � – { 0 }.
 Té una asímptota vertical en x = 0.
 Té una asímptota obliqua en y = x.
 f'(x) > 0 en els intervals (–∞, –2) i en (0, +∞),
f'(x) < 0 en l’interval (–2, 0) i f'(–2) = 0.
Resposta oberta. Per exemple:

713
Aplicacions de la
Aplicacions de la derivada.
derivada. Representació
Representacióde
defuncions
funcions 12

 
 
86. Considera la funció:
f(x) = 4x3 + 15x2 – 18x + 10
a) Determina els màxims i els mínims de la funció.
b) Calcula ��� ���� � ��� ���� i lim f (x).
���� ���

c) Dibuixa un esbós de la gràfica de la funció.


a) f’(x) = 12x2 + 30x – 18

��
12x2 + 30x – 18 = 0 → 2x2 + 5x2 – 3 = 0 → � �
� � ��
f’’(x) = 24x + 30
� �
f’’� � = 42 > 0 → En x = existeix un mínim.
� �

f’’���� = –42 < 0 → En x = –3 existeix un màxim.


b) lim ���� � ��
����

lim ���� � �
���

c)

87. Donada la funció f(x) = x3 + 6x2 – 36x + 29, resol aquestes qüestions:
a) Determina’n el domini.
b) Calcula’n les asímptotes.
c) Té punts de tall amb els eixos? Quins són?
d) Determina’n els intervals de creixement i decreixement.
e) Calcula’n els màxims i els mínims.
f ) Representa la funció.

714
Aplicacions de la derivada. Representació de funcions 1212

a) Dom f = �
b) lim ���� � ��
����

La funció no té asímptota horitzontal.


����
lim � ��
���� �

La funció no té asímptota obliqua.


c) Si x = 0 → y = 29
Si y = 0 → x3 + 6x2 – 36x + 29 = 0 → (x – 1)(x2 + 7x – 29) = 0
���
→� ���√���
��

d) f’(x)= 3x + 12x – 36
2

���
3x2 + 12x – 36 = 0 → x2 + 4x – 12 = 0 → �
� � ��

f(x) és creixent en (–∞, –6) ∪ (2, +∞).


f(x) és decreixent en (–6, 2).
e) Mínim: (2, –11)
Màxim: (–6, 245)
f)

88. Representa gràficament les funcions següents i estudia’n el creixement, el decreixement, els
màxims els mínims, i la concavitat i la convexitat:
� �
a) f(x) = x3 + x2 – 2x – 1
� �
b) f(x) = x3 – x
c) f(x) = x3 – x2
d) f(x) = x3 – 2x2
e) f(x) = x3 – 3x2
1
a) f ´( x ) = 2 x 2 + 3 x - 2  f ´( x ) = 0  x = y x = -2
2

æ1 ö æ 1ö
f(x) creix en (-¥, - 2) È ççç , + ¥÷÷÷ i decreix en ççç-2, ÷÷÷ .
è2 ø è 2ø

715
Aplicacions de la derivada. Representació de funcions

716
Aplicacions de la derivada. Representació de funcions 1212

e) f ´( x ) = 3x 2 - 6 x  f ´( x ) = 0  x = 0 y x = 2

f(x) creix en (-¥, 0) È (2, +¥) i decreix en (0, 2) .

f(x) té un mínim en x = 2 i un màxim en x = 0.

f ´´(x) = 6x – 6 → f ´´(x) = 0 → x = 1.

f(x) és convexa en (-¥, 1) i còncava en (1, + ¥) .

 
89. Representa gràficament les funcions següents i estudia’n el creixement, el decreixement, els
màxims i els mínims, i la concavitat i la convexitat:
a) f(x) = x3 – x2 – 2x
b) f(x) = x3 – x2 – x + 2

c) f(x) = x3 – 2x2 + 3x – 1

1 7
a) f ´( x ) = 3 x 2 - 2 x - 2  f ´( x ) = 0  x =
3

æ 1- 7 ÷ö÷ æç 1 + 7 ÷ö æ 1- 7 1 + 7 ö÷
f(x) creix en ççç-¥, ÷Èç , + ¥÷÷÷ i decreix en ççç , ÷÷ .
è 3 ø÷ çè 3 ø è 3 3 ø÷

1+ 7 1- 7
f(x) té un mínim en x = i un màxim en x = .
3 3

1
f ´´(x) = 6x – 2 → f ´´(x) = 0 → x =
3
æ 1ö æ1 ö
f(x) és convexa en ççç-¥, ÷÷÷ i còncava en ççç , + ¥÷÷÷ .
è 3ø è3 ø

1
b) f ´( x ) = 3 x 2 - 2 x - 1 f ´( x ) = 0  x = - y x = 1
3
æ 1ö æ 1 ö
f(x) creix en ççç-¥,- ÷÷÷ È (1, + ¥) i decreix en ççç- , 1÷÷÷ .
è 3ø è 3 ø

1
f(x) té un mínim en x = 1 i un màxim en x = - .
3
1
f ´´(x) = 6x – 2 → f ´´(x) = 0 → x =
3
æ 1ö æ1 ö
f(x) és convexa en ççç-¥, ÷÷÷ i còncava en çç , + ¥÷÷ .
è 3ø çè 3 ø÷

c) f ´( x ) = x 2 - 4 x + 3  f ´( x ) = 0  x = 1 y x = 3

f(x) creix en (-¥, 1) È (3, +¥) i decreix en (1, 3) .

f(x) té un mínim en x = 3 i un màxim en x = 1.

f ´´(x) = 2x – 4 → f ´´(x) = 0 → x = 2.

f(x) és convexa en (-¥, 2) i còncava en (2, + ¥) .

717
Aplicacions de la
Aplicacions de la derivada.
derivada. Representació
Representacióde
defuncions
funcions 12

 
90. Estudia i representa aquestes funcions polinòmiques:
a) f(x) = 3x4 – 4x3 – 36x2 + 10
b) f(x) = x3 – 6x2 + 12 x – 5
c) f(x) = 3x4 – 4x3 – 48x2 + 144x + 212
a) Dom f = �
La funció no té asímptotes.
f’(x) = 12x3 – 12x2 – 72x
���
12x3 – 12x2 – 72x = 0 → x(x2 – x – 6) = 0 → � � � �
� � �2

                               

f(x) és creixent en (–2, 0) ∪ (3, +∞) i és decreixent en (–∞, –2) ∪ (0, 3).
Mínims: (–2, –54) i (3, –179)
Màxim: (0, 10)
b) Dom f = �
La funció no té asímptotes.
f’(x) = 3x2 – 12x + 12
3x2 – 12x + 12 = 0 → x2 – 4x + 4 = 0 →(x – 2)2 = 0 → x = 2
f’(x) > 0 → f(x) és creixent en �
f’(2) = 0 → En x = 2 no té ni un màxim ni un mínim.
f’’(x) = 6x – 12.
f’’(x) > 0 si x > 2 → f(x) és còncava en ( 2, +∞).

f’’(x) < 0 si x < 2 → f(x) és convexa en ( –∞, 2).        


c) Dom f = �
La funció no té asímptotes.
f’(x) = 12x3 – 12x2 – 96x + 144
��2
12x3 – 12x2 – 96x + 144 = 0 → �
� � ��

718
Aplicacions de la derivada. Representació de funcions 1212

f(x) és creixent en (–3, 2) ∪ (2, +∞) i és decreixent en (–∞, –3).


Mínim: (–3, –301)
f’’(x) = 36x2 – 24x – 96.
���
36x2 – 24x – 96 = 0 → 3x2 – 2x – 8 = 0 → �� � � �� �

f’’(x) > 0 si x < 2 → f(x) és còncava en (2, +∞���


� �
f’’(x) < 0 si � < x < 2 → f(x) és convexa en (� , 2���
� �
� �
f’’(x) < 0 si x < � → f(x) és convexa en (�∞� � ���
� �

   
����
91. Donada la funció ���� � , resol aquestes qüestions:
���
a) Determina’n el domini.
b) Troba’n les asímptotes.
c) Determina’n els intervals de creixement i decreixement.
d) Calcula’n els màxims i els mínims.
e) Estudia’n la concavitat i la convexitat.
f ) Representa la funció.

a) Dom f = ���������
����
lim
� ���
� �∞
b) ��→� ���� → x = – 4 és una asímptota vertical.
lim� � �∞
�→� ���
����
lim � � � → y = 3 és una asímptota horitzontal.
�→� ���

Com que té asímptota horitzontal, la funció no té asímptota obliqua.


������������� ��
c) � � ��� � �
������ ������

� � ��� � � → f(x) és creixent en (–∞, –4) ∪ (–4, +∞).


d) La funció no té màxims ni mínims.
��������� ��
e) ������ � � � ������ � � → La funció és sempre convexa.
������

f)

719
Aplicacions de la
Aplicacions de la derivada.
derivada. Representació
Representacióde
defuncions
funcions 12

 
 

92. Troba els màxims i els mínims de la funció següent: ���� �
���
Determina’n les equacions de les asímptotes i la posició de la corba respecte d’aquestes
asímptotes. Dibuixa, també, un esbós de la gràfica de la funció.
� � �� � �� � � ��
� � ��� � �

�� � �� �� � ���
No té màxims ni mínims, � � ��� � �� → La funció és decreixent.
Dom f = � – {4}.
lim� ���� � ��
��→� → x = 4 és una asímptota vertical. 
lim� ���� � ��
�→�

lim ���� � � →és una asímptota horitzontal. 


�→��

Si x = 1 000 → f(x) > 1

Quan x tendeix a +∞, la funció es troba per sobre de l’asímptota.

Si x = – 1 000 → f(x) < 1

Quan x tendeix a – ∞, la funció es troba per sobre de l’asímptota.

 
 
93. Calcula les asímptotes d’aquestes funcions i dibuixa’n la representació gràfica:

��� �� �� ��
a) ���� � b) ���� � c) ���� �
��� ��� �� ��

x +3
a) Asímptota horitzontal: xlim = 1 y = 1
¥ x -4

Asímptota vertical: x - 4 = 0  x = 4

720
Aplicacions de la derivada. Representació de funcions 1212

 
6x
b) Asímptota horitzontal: xlim =6 y =6
¥ x -4
Asímptota vertical: x - 4 = 0  x = 4

 
x2 -1
c) Asímptota horitzontal: xlim = 1 y = 1
¥ x2 + 1
2
Asímptota vertical: x + 1 > 0 "x Î   No existeix.

 
 
��
94. Considera la funció: ���� �
����
a) Troba els màxims i els mínims de la funció.
b) Determina les equacions de les asímptotes i la posició de la corba respecte d’aquestes
asímptotes.
c) Construeix un esbós de la gràfica de la funció.
�� � ����� � ��� � ����� �� � �� �
a) � � ��� � �
���� � �� ���� � ��

3� � � � � ���
� �� � 3� � � � � � � � �

�� � � � � � � �√3
��� � �� � ��� � � � �� � �3� � � � � � � ��� � � � ������ �� � �� �
� � ′��� � �
�� � � � �� �� � � � ��

721
Aplicacions de la
Aplicacions de la derivada.
derivada. Representació
Representacióde
defuncions
funcions 12

�√�
� �� ��√3� � � 0 → En x = �√3 hi ha un mínim.

� �� �0� � 0 → En x = 0 no hi ha màxim ni un mínim.

�√�
� �� �√3� � � � 0 → En x = √3 hi ha un màxim.

b) Dom f = � – {–1, 1}

lim� ���� � ��
��→�� → x = – 1 és una asímptota vertical.
lim� ���� � ��
�→��

lim� ���� � ��
��→� → x = 1 és una asímptota vertical.
lim� ���� � ��
�→�

lim ���� � ��� → No hi ha asímptota horitzontal.


�→��

� ���� ��
� � � lim
� lim � ��
�→�� � �→�� � � � �

�� � lim ����� � ��� � lim � � �� � � � ��
� � �� � lim �0
�→�� �→�� � � � � �→�� � � ��

Asímptota obliqua: y = – x

Si x = 1 000 → f(x) – ( –x) < 0 → Quan x tendeix a +∞, la funció es troba per sota de l’asímptota.

Si x = –1 000 → f(x) – ( –x) > 0 → Quan x tendeix a –∞, la funció es troba per sobre de l’asímptota.

c)

95. Estudia la posició de la gràfica d’aquestes funcions respecte de les asímptotes respectives i fes–ne
la representació gràfica:
�� �� ��
a) ���� � c) ���� �
��� ���
�� �� ��
b) ���� � d) ���� �
��� �� ��
a) Té una asímptota vertical en x = 3 i no té asímptota horitzontal.

Té una asímptota obliqua en y = x + 3.

x2 + 2
En x < 3: < x + 3  La gràfica es troba per sota de l’asímptota.
x -3

722
Aplicacions de la derivada. Representació de funcions 1212

x2 + 2
En x > 3: > x + 3  La gràfica es troba per sobre de l’asímptota.
x -3

ìï x2 + 2
ïï lim = -¥
ïï x  3- x -3
í
ïï x2 + 2
ïï lim+ = +¥
ïî x  3 x -3
 
 

b) Té una asímptota vertical en x = 3 i no té asímptota horitzontal.

Té una asímptota obliqua en y = x + 3.

x2 - 2
En x < 3: < x + 3  La gràfica es troba per sota de l’asímptota.
x -3

x2 - 2
En x > 3: > x + 3  La gràfica es troba per sobre de l’asímptota.
x -3

ìï x2 - 2
ïï lim = -¥
ïï x  3- x -3
í
ïï x2 - 2
ïï lim+ = +¥
îï x  3 x -3

c) Té asímptota vertical en x = 1 i no té asímptota horitzontal. Té


asímptota obliqua en y = x + 1.
x2
En x < 1: < x + 1  La gràfica es troba per sota de l’asímptota.
x -1

x2
En x > 1: > x + 1  La gràfica es troba per sobre de l’asímptota.
x -1
ìï x2
ïï lim = -¥
ïï x 1- x -1
í
ïï x2
ïï lim+ = +¥
îï x 1 x -1

d) Té asímptota horitzontal en y = 1 i no té asímptota vertical ni obliqua.


x2
"x Î   2
< 1  La gràfica es troba per sota de l’asímptota.
x +1

   

723
Aplicacions de la
Aplicacions de la derivada.
derivada. Representació
Representacióde
defuncions
funcions 12

96. Estudia i representa les funcions racionals següents:


���� ��
a) ���� � ���
c) ���� �
���
�� ����� �� �����
b) ���� � d) ���� �
��� �� ����

a) Dom f = � – {2}.
����
lim
� ���
� ��
��→� ���� → x = 2 és una asímptota vertical.
lim� � ��
�→� ���
����
lim � 5 → y = 5 és una asímptota horitzontal.
�→�� ���

Com que té asímptota horitzontal, la funció no té asímptota obliqua.


Punt de tall amb l’eix X: �� � ��


Punt de tall amb l’eix Y: ��� � �

5�� � �� � �5� � �� ���


� � ��� � �
�� � ��� �� � ���

� � ��� � � → ���� és decreixent en (–∞, 2) ∪ (2, +∞).

La funció no té ni màxims ni mínims.

 
b) Dom f = � – {3}.
� � �����
lim� � ��
���
��→� �� ����� → x = 3 és una asímptota vertical.
lim� � ��
�→� ���

� � �����
lim � �� → La funció no té cap asímptota horitzontal.
�→�� ���

� ���� � � � �� � �
� � � lim
� lim ��
�→�� � �→�� � � � ��

�� � lim ����� � ��� � lim �� � �� � � � �� � lim � � � � �
� �→�� �→�� ��� �→�� � � �

→ Asímptota obliqua: y = x + 1

Punt de tall amb l’eix X: (1, 0)


Punt de tall amb l’eix Y:��� � �

�������������� � ������ � � �����


� � ��� � �
������ ������

724
Aplicacions de la derivada. Representació de funcions 1212

���
� � � �� � � � �� → �
���

f(x) és creixent en (–∞, 1) ∪ (5, +∞) i és decreixent en (1, 3) ∪ (3, 5).

Màxim: (1, 0) Mínim: (5, 8)

c) Dom f = � – {2}.
��
lim
� ���
� �∞
��→� �� → x = 2 és una asímptota vertical.
lim� � �∞
�→� ���
��
lim � �∞ → La funció no té cap asímptota horitzontal.
�→�� ���

� ���� ��
� � � lim
� lim � ��
�→�� � �→�� � � �

�� � lim ����� � ��� � lim � � � �� � lim �� � ��
� �→�� �→�� � � � �→�� � � �

→ Asímptota obliqua: y = – x – 2

Punt de tall amb els eixos: (0, 0)

��������� � ���� �
� � ��� � �
������ ������

���
�� � � � � �� → �
���

f(x) és decreixent en (–∞, 0) ∪ (4, +∞) i és creixent en (0, 2) ∪ (2, 4).

Màxim: (4, –8) Mínim: (0, 0)  

725
Aplicacions de la
Aplicacions de la derivada.
derivada. Representació
Representacióde
defuncions
funcions 12

d) Dom f = � – {–3, 2}.


�� � �����
lim � � ��
� � ����
��→�� �������� → x = –3 és una asímptota vertical.
lim � � � ��
�→�� � ����

�� � �����
lim� � � ��
� ����
��→� ��� ����� → x = 2 és una asímptota vertical.
lim� � � ��
�→� � ����

�� � �����
lim � 1 → y = 1 és una asímptota horitzontal.
�→�� � � ����

Com que té asímptota horitzontal, la funció no té asímptota obliqua.

Punt de tall amb l’eix X: �1, 0��i���2, 0�


Punt de tall amb l’eix Y: ���, �

�1�� � �
� � ��� �
�� � � � � ���

1
�1�� � � � 0 → � � � �
2

� �
���� és creixent en (–∞, –3) ∪ ���, �i és decreixent en �� , 2�∪ �2, ���.
� �

� ��
Màxim: �� , �
� ��

   
   

726
Aplicacions de la derivada. Representació de funcions 1212

97. Representa aquestes funcions racionals i estudia’n les característiques:


� �� �����
a) ���� � d) ���� �
�� ����� ��
�� ����� ��
b) ���� � e) ���� �
�� ����� �� ����
��� ���
c) ���� � f) ���� �
�� ����� ����������

a) Dom f = � – {–1, 4}.



lim
� �� �����
� ��
��→�� � → x = –1 és una asímptota vertical.
lim � � ��
�→�� �� �����


lim� �� �����
� ��
��→� � → x = 4 és una asímptota vertical.
lim� � ��
�→� �� �����

lim � 0 → y = 0 és una asímptota horitzontal.
�→�� �� �����

Com que té asímptota horitzontal, la funció no té asímptota obliqua.

Punt de tall amb els eixos: �0, 0�

� � � 3� � � � ���� � 3� �� � � �
� � ��� � �
�� � � 3� � ��� �� � � 3� � ���

� � ��� � 0 → ���� és decreixent en (–∞, –1) ∪ (–1, 4) ∪ (4, +∞).

La funció no té ni màxims ni mínims.

 
b) Dom f = � – {–1}.
�� �����
lim � � ��
�� �����
��→�� � → x = –1 és una asímptota vertical.
� �����
lim � � � ��
�→�� � �����
�� �����
lim � � → y = 1 és una asímptota horitzontal.
�→�� �� �����

Com que té asímptota horitzontal, la funció no té asímptota obliqua.

Punt de tall amb l’eix Y: �3, 0�

��� � ���� � � �� � �� � �� � � �� � 3���� � �� ��� � �


� � ��� � � �
�� � � �� � ��� �� � �� � ���

��� � � � 0 → � � �� � ��m��

727
Aplicacions de la
Aplicacions de la derivada.
derivada. Representació
Representacióde
defuncions
funcions 12

� � ��� � ������ � �� → ���� és creixent en (–∞, –1).

� � ��� � ������ � �� → ���� és decreixent en (–1, +∞).

La funció no té màxims ni mínims.

 
c) Dom f = � – {–1, 4}.
���
lim � � ��
�� �����
��→�� ��� → x = –1 és una asímptota vertical.
lim � � ��
�→�� �� �����
���
lim� �� �����
� ��
��→� ���
→ x = 4 és una asímptota vertical.
lim� � ��
�→� �� �����
���
lim � � → y = 0 és una asímptota horitzontal.
�→�� �� �����

Com que té asímptota horitzontal, la funció no té asímptota obliqua.

Punt de tall amb l’eix X: ���� ��


Punt de tall amb l’eix Y: ��� � �

� � � �� � � � �� � ����� � �� �� � � ��� � ��
� � ��� � �
�� � � �� � ��� �� � � �� � ���

�� � � ��� � �� � � ���
� � → �� � � ��� � �� � � → �
�� � � �� � ��� ���

���� és creixent en (–11, –1) ∪ (–1, 1) i és decreixent en (–∞, –11) ∪ (1, 4) ∪ (4, +∞).


Mínim: ����� � � Màxim: ��� ���
��

728
Aplicacions de la derivada. Representació de funcions 1212

d) Dom f = � – {0}.
�� �����
lim � � ��
��
��→�� � → x = 0 és una asímptota vertical.
� �����
lim � � � ��
�→�� �

�� �����
lim � � → y = 1 és una asímptota horitzontal.
�→�� ��

Com que té asímptota horitzontal, la funció no té asímptota obliqua.

Punt de tall amb l’eix X: ��√2 � 2, 0��i��√2 � 2, 0�

�2� � �� � � � � �� � � �� � 2� � 2� ��� � �
� � ��� � �
�� � �� ��

��� � � � 0 → � � ��

���� és decreixent en (–∞, –1) ∪ (0, +∞) i és creixent en (–1, 0).

Mínim: (–1, 1)

e) Dom f = � – {–3, 2}.


��
lim
� �� ����
� ��
��→�� �� → x = –3 és una asímptota vertical.
lim � � ��
�→�� �� ����
��
lim� � ��
�� ����
��→� �� → x = 2 és una asímptota vertical.
lim� � ��
�→� �� ����
��
lim � 0 → y = 0 és una asímptota horitzontal.
�→�� �� ����

Com que té asímptota horitzontal, la funció no té asímptota obliqua.

Punt de tall amb els eixos: �0, 0�

2�� � � � � �� � 2��2� � �� �2� � � �2


� � ��� � �
�� � � � � ��� �� � � � � ���

� � ��� � 0 → ���� és decreixent en (–∞, –3) ∪ (–3, 2) ∪ (2, +∞).

729
Aplicacions de la
Aplicacions de la derivada.
derivada. Representació
Representacióde
defuncions
funcions 12

La funció no té màxims ni mínims.

 
f) Dom f = � – {–1, 1}.
���
lim
� ����������
� ��
��→�� ��� → x = –1 és una asímptota vertical.
lim � � ��
�→�� ����������
���
lim
� ����������
� ��
��→� ��� → x = 1 és una asímptota vertical.
lim� � ��
�→� ����������
���
lim � 0 → y = 0 és una asímptota horitzontal.
�→�� ����������

Com que té asímptota horitzontal, la funció no té asímptota obliqua.

Punt de tall amb l’eix X: �3, 0�

Punt de tall amb l’eix Y: �0, 3�

� � � � � �� � 3� � 2� �� � � �� � �
� � ��� � �
�� � � ��� �� � � ���

�� � � �� � �
� 0 → �� � � �� � � � 0 → � � �2√2
�� � � ���

���� és decreixent en (–∞, –1) ∪ (–1, 3 – 2√2 ) ∪ (3+2√2, +∞).

���√� ���√�
Màxim: �3 � 2√2, � Mínim:�3 � 2√2, �
� �

730
Aplicacions de la derivada. Representació de funcions 1212

98. Estudia i representa les funcions següents:


࢞૝ ା࢞૛ ା૚ ࢞૛ ି૚
a) ࢌሺ࢞ሻ ൌ c) ࢌሺ࢞ሻ ൌ
࢞૜ ି૚ ࢞૜ ା૚
࢞૛ ି૜࢞ା૛ ࢞૛ ା૚
b) ࢌሺ࢞ሻ ൌ d) ࢌሺ࢞ሻ ൌ
࢞૜ ି૚ ࢞૜ ା૚

a) El domini es defineix per tots els nombres que no anul·len el denominador. Per tant, Dom f =  -{1} .

No té punts de tall amb l’eix X, atès que f ( x) ¹ 0 "x Î  .

Punt de tall amb l’eix Y:

x = 0 → f(0) = –1 → El punt de tall és (0, –1).

No té asímptotes horitzontals.

Té asímptotes verticals per x = 1 .

2 x ( x 4 - 2 x 2 - 3)
f ´( x ) = 2
( x 2 - 1)

2 x ( x 4 - 2 x 2 - 3) ïì x = 0 ï (
ìïf ´( x ) > 0 en (- 3, - 1) È (-1, 0) È 3, + ¥
)
= 0  ïí  ïí
( )
ïï x =  3 ïïf ´( x ) < 0 en -¥, - 3 È (0, 1) È (1, + 3 )
2
( x 2 - 1) î ïî

La funció decreix en (-¥, - 3 ) È (0, 1) È (1, + 3 ) i creix en (- 3, - 1) È (-1, 0) È ( 3, + ¥) .

2( x 6 - 3 x 4 + 15 x 2 + 3)
ES calcula la segona derivada f ´´( x ) = 3
i s’avaluen als punts crítics:
( x 2 + 1)

f ´´(- 3) = 12 > 0  x = - 3 mínim

f ´´(0) =-6 x = 0 máxim

f ´´( 3 ) = 12 > 0  x = 3 mínim

 
b) El domini es defineix per tots els nombres que no anul·len el denominador. Per tant, Dom f =  -{1} .

Punt de tall amb l’eix X:

f ( x) = 0  x = 2  El punt de tall és (2, 0).

Punt de tall amb l’eix I:

x = 0 f (0) =-2  El punt de tall és (0, –2).

No té asímptotes verticals.

731
Aplicacions de la
Aplicacions de la derivada.
derivada. Representació
Representacióde
defuncions
funcions 12

Té asímptota horitzontal en y = 0.

-x 2 + 4 x + 3 ï f ´( x) < 0 en x Î (-¥, 2 - 7 ) È (2 + 7, + ¥)  f ( x) creix.


ì
ï
f ´( x ) = ï
í
î f ´( x) > 0 en x Î (2 - 7, 2 + 7 )  f ( x) decreix.
2
ï
( x 2 + x + 1) ï
ï

2( x3 - 6 x 2 + 9 x -1) ìï f ´´(2 - 7) > 0  x = 2 - 7 és mínim.


f ´´( x) =  ïí
( x2 + x +1)
3
ïï f ´´(2 + 7) < 0  x = 2 + 7 és màxim.
ïî

c) El domini es defineix per tots els nombres que no anul·len el denominador. Per tant, Dom f =  -{-1} .

Punt de tall amb l’eix X:

f ( x ) = 0  x = 1 El punt de tall és (1, 0).

Punt de tall amb l’eix I:

x = 0 f (0) =-1 El punt de tall és (0, –1).

Té asímptota horitzontal en y = 0.

x ( x - 2) ìï f ´( x) < 0 en x Î (-¥, 0) È (2, +¥)  f ( x ) creix.


f ´( x ) = -  íï
( x 2 - x + 1) ïïî f ´( x) > 0 en x Î (0, 2)  f ( x) decreix.
2

ïìï f ´´(0) = 2 > 0  x = 0 mínim


2( x3 - 3x 2 +1)
f ´´( x) =  ïí
3
ïï f ´´(2) = -2 < 0  x = 2 màxim
( x2 - x +1) ïî 9

732
Aplicacions de la derivada. Representació de funcions 1212

d) El domini es defineix per tots els nombres que no anul·len el denominador. Per tant, Dom f =  -{-1} .

No té punts de tall amb l’eix X, atès que f ( x) ¹ 0 "x Î  .

Punt de tall amb l’eix Y:

x = 0 f (0) =1El punt de tall és (0, 1).

Té asímptota vertical en x = –1.

Té asímptota horitzontal en y = 0.

x ( x 3 + 3 x 2 - 2) ìï f ´( x) < 0 en x Î (-¥, -1) È (-1, 0) È (0,596; + ¥)  f ( x) creix


f ´( x) = -  íï
2
ïïî f ´( x) > 0 en x Î (0; 0,596)  f ( x) decreix
( x3 + 1)

x ( x 3 + 3 x - 2)
Atès que f ´( x ) = - 2
s’anul·la en x = 0 i en x = 0,596. Calculem la segona derivada:
( x 3 + 1)

2( x 6 + 6 x 4 - 7 x 3 - 3 x + 1)
f ´´( x ) = 3
, que avaluant-la als punts crítics, obtenim:
( x 2 + 1)
ì
ï
ï f ´´(0,596) = -1, 650 < 0  x = 0,596 màxim
í
ï
î f ´´(0) = 2 > 0  x = 0 mínim
ï

   
 
99. Relaciona cadascuna de les característiques següents amb la funció que les tingui:
 Té com a màxim (0, –1).
 Té dues asímptotes verticals.
 La tangent és horitzontal per a x = 1 i x = –1.
 Té com a màxim (1, 4).
 Sempre és creixent.
࢞૜ ା࢞૛ ା࢞ ࢞૛ ା૚
ࢌሺ࢞ሻ ൌ ࢏ሺ࢞ሻ ൌ
૚૙ ࢞ି૜
࢞૛ ା૚
ࢍሺ࢞ሻ ൌ ࢞૝ െ ૛࢞૛ െ ૚ jሺ࢞ሻ ൌ
࢞૛ ି૛

ࢎሺ࢞ሻ ൌ െ࢞ ൅ ૜࢞ ൅ ૛૜
kሺ࢞ሻ ൌ െ࢞૛ െ ૛࢞ െ ૚
▪ El seu màxim és (0, –1) i la tangent és horitzontal quan x = 1 i x = –1.

Són característiques de g(x) = x4 – 2x2 – 1:

733
Aplicacions de la
Aplicacions de la derivada.
derivada. Representació
Representacióde
defuncions
funcions 12

g´( x ) = 4 x 3 - 4 x  g´( x ) = 0  x =1 y x = 0

La derivada en els punts x = 1 i x = –1 és 0; per tant, la tangent serà horitzontal a aquests punts.

g´´(x) = 12x2 - 4  g´(0) =-4 < 0  x = 0 màxim g(0) =-1  (0, -1) màxim

▪ Té dues asímptotes verticals.

x2 + 1
És característica de j( x ) = .
x2 - 2

Les asímptotes horitzontals hi són quan el denominador s’anul·la. En aquest cas hi ha dues asímptotes:
en x = –2 i en x = 2.

▪ El seu màxim és (1, 4) i la tangent és horitzontal quan x = 1 i x = –1.

Són característiques de h(x) = –x3 + 3x + 2

ìïh´( x) = -3x 2 + 3  h´( x) = 0  x = 1


ïï
ïíh´´( x) = -6  h´´(1) = -6 < 0  x = 1 màxim
ïï
ïïh(1) = 4  (1, 4) màxim
î

▪ És creixent sempre.

x3 + x2 + x
És característica de f ( x ) = .
10

3x2 + 2x + 1
f ´( x ) = > 0 "x Î   f ( x ) sempre creixent.
10

i(x) i k(x) no compleix cap de les característiques.

100. Estudia i representa les funcions següents:


a) ࢌሺ࢞ሻ ൌ ૚ ൅ ࢋ࢞ା૜ c) ࢌሺ࢞ሻ ൌ ૜ ൉ ࢋ૚ି࢞
ࢋ૛࢞ ૜ିࢋ࢞
b) ࢌሺ࢞ሻ ൌ d) ࢌሺ࢞ሻ ൌ
૛ ૛

a) El domini de les funcions exponencials coincideix amb el domini del seu


exponent. Atès que el seu exponent és un polinomi, Domf =  .

No té punts de tall amb l’eix X.


Punt de tall amb l’eix Y:
x = 0  f (0) = 1+ e3  El punt de tall és (0, 1 + e3).

Té asímptota horitzontal en y = 1 quan x → –∞.


No té asímptotes verticals.
f ( x ) = 1+ e x +3  f ´( x ) = e x +3 > 0 "x Î   f(x) és sempre creixent.

734
Aplicacions de la derivada. Representació de funcions 1212

b) El domini de les funcions exponencials coincideix amb el domini del seu


exponent. Atès que el seu exponent és un polinomi, Domf =  .

No té punts de tall amb l’eix X.


Punt de tall amb l’eix Y:
e0 1 æ 1ö
x = 0  f (0) = =  El punt de tall és ççç0, ÷÷÷ .
2 2 è 2ø

Té asímptota horitzontal en y = 0 quan x → –∞.


No té asímptotes verticals.
e2 x
f (x) =  f ´( x ) = e2 x > 0 "x Î   f(x) és sempre creixent.
2

c) El domini de les funcions exponencials coincideix amb el domini del seu


exponent. Com que el seu exponent és un polinomi, Domf =  .
No té punts de tall amb l’eix X.
Punt de tall amb l’eix Y:
x = 0 f (0) = 3e  El punt de tall és (0, 3e).
Té asímptota horitzontal en y = 0 quan x → +∞.
No té asímptotes verticals.
f ( x ) = 3e1- x  f ´( x ) = -3e1- x < 0 "x Î   f(x) és sempre decreixent.

d) El domini de les funcions exponencials coincideix amb el domini del seu


exponent. Atès que el seu exponent és un polinomi, Domf =  .

Punt de tall amb l’eix X:


3 - ex
0=  3 = e x  x = ln 3  El punt de tall és (ln 3, 0).
2
uw
Punt de tall amb l’eix Y:
3 - e0 2
x = 0  f (0) = = = 1  El punt de tall és (0, 1).
2 2
3
Té asímptota horitzontal en y = quan x → –∞.
2
No té asímptotes verticals.
3 - ex -e x
f (x) =  f ´( x ) = < 0 "x Î   f(x) és sempre decreixent.
2 2

101. Estudia i representa aquestes funcions:


૛ ૚ ࢞ି૚
a) ࢌሺ࢞ሻ ൌ ൉ ૛࢞ା૛ c) ࢌሺ࢞ሻ ൌ ૝ ൅ ቀ ቁ
૞ ૜

૚ ࢞ା૚ ૚ ૛ି࢞
b) ࢌሺ࢞ሻ ൌ ૜ ൉ ቀ ቁ d) ࢌሺ࢞ሻ ൌ ቀ ቁ
૛ ૝

735
Aplicacions de la
Aplicacions de la derivada.
derivada. Representació
Representacióde
defuncions
funcions 12

a) El domini de les funcions exponencials coincideix amb el domini del seu exponent. Atès que el seu
exponent és un polinomi, Domf =  .

No té punts de tall amb l’eix X.

Punt de tall amb l’eix Y:

2 8 æ 8ö
x = 0  f (0) = ⋅ 22 =  El punt de tall és ççç0, ÷÷÷ .
5 5 è 5ø

Té asímptota horitzontal en y = 0 quan x  -¥ .

No té asímptotes verticals.

2 2
f ( x ) = 2x +2  f ´( x ) = 2x +2 ln 2 > 0 "x Î   f(x) és sempre creixent i, com que f ´(x) no s’anul·la a cap punt,
5 5
no té màxims ni mínims.

b) El domini de les funcions exponencials coincideix amb el domini del seu exponent. Atès que el seu
exponent és un polinomi, Domf =  .

No té punts de tall amb l’eix X.

Punt de tall amb l’eix Y:

1
æ 1ö 3 æ 3ö
x = 0  f (0) = 3 ççç ÷÷÷ =  El punt de tall és ççç0, ÷÷÷ .
è 2ø 2 è ø 2

Té asímptota horitzontal en y = 0 quan x → +∞.

No té asímptotes verticals.

x +1
æ 1ö
f ( x ) = 3 ççç ÷÷÷  f ´( x ) = -2- x -1 ln 8 < 0 "x Î   f(x) és sempre decreixent i,
è 2ø

com que f ´(x) no s’anul·la a cap punt, no té màxims ni mínims.

c) El domini de les funcions exponencials coincideix amb el domini del seu exponent. Atès que el seu
exponent és un polinomi, Domf =  .

No té punts de tall amb l’eix X.

Punt de tall amb l’eix Y:

-1
æ 1ö
x = 0  f (0) = 4 + ççç ÷÷÷ = 7  El punt de tall és (0, 7).
è3ø

Té asímptota horitzontal en y = 4 quan x → +∞.

No té asímptotes verticals.

x -1
æ 1ö
f ( x ) = 4 + ççç ÷÷÷  f ´( x ) = -31- x ln 3 < 0 "x Î   f(x) és sempre decreixent i,
è3ø

com que f ´(x) no s’anul·la a cap punt, no té màxims ni mínims.

736
Aplicacions de la derivada. Representació de funcions 1212

d) El domini de les funcions exponencials coincideix amb el domini del seu exponent. Atès que el seu
exponent és un polinomi, Domf =  .

No té punts de tall amb l’eix X.

Punt de tall amb l’eix Y:

4x 1 æ 1ö
x = 0  f (0) = =  El punt de tall és ççç0, ÷÷÷ .
16 16 è 16 ø

Té asímptota horitzontal en y = 0 quan x → –∞.

No té asímptotes verticals.

ln 4 x
f ( x ) = 4 x -2  f ´( x ) = 4 > 0 "x Î   f(x) és sempre creixent i, com que
16
f ´(x) no s’anul·la a cap punt, no té màxims ni mínims.

102. Estudia i representa les funcions següents:


a) ࢌሺ࢞ሻ ൌ ࢒࢕ࢍ૜ ሺ૛࢞ሻ d) ࢌሺ࢞ሻ ൌ ࢒࢕ࢍ૞ ሺ࢞ െ ૚ሻ

b) ࢌሺ࢞ሻ ൌ ࢒࢕ࢍ૛ ሺ࢞ ൅ ૚ሻ e) ࢌሺ࢞ሻ ൌ ࢒࢔ ቀ ቁ


c) ࢌሺ࢞ሻ ൌ ࢒࢕ࢍ૚ ࢞ f) ࢌሺ࢞ሻ ൌ ࢒࢔ ቀ ቁ
૜ ࢞૛ ା૚

a) La funció logarítmica és definida quan el seu argument és més gran que 0.

2x > 0  x > 0  Dom f = (0, +¥)

Punts de tall amb l’eix X:

1 æ1 ö
0 = log3 2 x  2 x = 30 = 1 x =  El punt de tall és ççç , 0÷÷÷ .
2 è2 ø

No té punts de tall amb l’eix Y.

No té asímptotes horitzontals.

Té asímptota vertical en x = 0.

1
f ( x ) = log 3 2 x  f ´( x ) = > 0 "x > 0  f(x) és sempre creixent.
x ln 3

b) La funció logarítmica és definida quan el seu argument és més gran que 0.

x +1> 0  x >-1 Dom f = (-1, +¥)

Punts de tall amb l’eix X:

0 = log 2 ( x + 1)  x + 1 = 20 = 1  x = 0  El punt de tall és (0, 0).

737
Aplicacions de la
Aplicacions de la derivada.
derivada. Representació
Representacióde
defuncions
funcions 12

Punt de tall amb l’eix Y:


x = 0  f (0) = log2 (0 +1) = log2 1= 0  El punt de tall és (0, 0).

No té asímptotes horitzontals.

Té asímptota vertical en x = –1.

1
f ( x ) = log 2 ( x + 1)  f ´( x ) = > 0 "x > -1  f(x) és sempre creixent.
( x + 1) ln 2

c) La funció logarítmica és definida quan el seu argument és més gran que 0.

x > 0  Dom f = (0, +¥)

Punts de tall amb l’eix X:

0
æ 1ö
0 = log 1 x  x = ççç ÷÷÷ = 1  x = 1  El punt de tall és (1, 0).
3
è3ø

No té punts de tall amb l’eix Y.

No té asímptotes horitzontals.

Té asímptota vertical en x = 0.

1
f ( x ) = log 1 x  f ´( x ) = < 0 "x > 0  f(x) és sempre decreixent i,
æ 1ö
3 x lnçç ÷÷÷
è3ø

atès que f ´(x) no s’anul·la a cap punt, no té màxims ni mínims.

d) La funció logarítmica és definida quan el seu argument és més gran que 0.

x -1> 0  x > 1 Dom f = (1, +¥)

Punts de tall amb l’eix X:

0 = log5 ( x - 1)  x - 1 = 50 = 1  x = 2  El punt de tall és (2, 0).

No té punts de tall amb l’eix Y.

No té asímptotes horitzontals.

Té asímptota vertical en x = 1.

1
f ( x ) = log 5 x  f ´( x ) = > 0 "x > 0  f(x) és sempre creixent i, atès
x ln 5

que f ´(x) no s’anul·la a cap punt, no té màxims ni mínims.

e) La funció logarítmica és definida quan el seu argument és més gran que 0.

1
> 0  x > 0  Dom f = (0, + ¥)
x

Punts de tall amb l’eix X:

738
Aplicacions de la derivada. Representació de funcions 1212

æ 1ö
0 = lnççç ÷÷÷  x = 1  El punt de tall és (1, 0).
èxø

No té punts de tall amb l’eix Y.

No té asímptotes horitzontals.

Té asímptota vertical en x = 0.

æ 1ö 1
f ( x ) = lnççç ÷÷÷  f ´( x ) = - < 0 "x > 0  f(x) és sempre decreixent i, atès
èxø x

que f ´(x) no s’anul·la a cap punt, no té màxims ni mínims.

f) La funció logarítmica és definida quan el seu argument és més gran que 0.

1
> 0 "x Î   Dom f = 
x2 +1

Punts de tall amb l’eix X:

æ 1 ö÷ 1
0 = lnççç 2 ÷ 2 = e0 = 1  x = 0  El punt de tall és (0, 0).
è x + 1ø÷ x +1

Punts de tall amb l’eix Y:

æ 1 ö÷
x = 0  f (0) = lnççç 2 ÷ = ln 1 = 0  El punt de tall és (0, 0).
è 0 + 1ø÷

No té asímptotes horitzontals.

No té asímptotes verticals.

ìï
ï f ´( x) = - 2 x > 0 en x Î (-¥, 0)  f ( x) es creixent.
æ 1 ö÷ ïïï x2 +1
f ( x) = ln ççç 2 ÷í
÷
è x + 1ø ïï 2x
ïï f ´( x) = - 2 < 0 en x Î (0, + ¥)  f ( x) es decreixent.
ïî x +1

Atès que en (0, 0) la funció creix per l’esquerra i decreix per la dreta, és un màxim.

2x
Para x ¹ 0, f ´( x ) = - ¹ 0  No existeixen més màxims o mínims.
x2 +1

103. En una empresa han estimat que la despesa en electricitat, de les 8 h a les 17 h, segueix aquesta
funció: E(t) = 0,01t3 – 0,36t2 + 4,05t – 10 en què t pertany a l’interval (8, 17).
a) Quin és el consum elèctric a les 10 h? I a les 16 h?

739
Aplicacions de la
Aplicacions de la derivada.
derivada. Representació
Representacióde
defuncions
funcions 12

b) En quin moment del dia el consum és màxim? I mínim?


c) Determina les hores del dia en les quals el consum s’incrementa.
a) E(10) = 4,5

E(16) = 3,6

b) E’(t) = 0,03t2 – 0,72t + 4,05

���
0,03t2 – 0,72t + 4,05 = 0 → �
� � ��

E’’(t) = 0,06t – 0,72

E’’(9) = –0,18 < 0 → En t = 9 hi ha un màxim.

E’’(15) = 0,18 > 0 → En t = 15 hi ha un mínim.

Per tant, el consum és màxim a les 9 hores i és mínim a les 15 hores.

c) Atès que t = 9 és un màxim, el consum creix de les 8 hores fins a les 9 hores.

De la mateixa manera, com que t = 15 és un mínim, el consum creix de 15 a 17 hores.

104. Els beneficis de dues empreses, A i B, estan determinats per les funcions fA i fB, en què x es
mesura en anys.
��� ������
�� ��� � �� ��� �
�� ���� �� ����

Efectua l’estudi de les qüestions que tens plantejades a continuació:


a) Durant quant de temps obtenen guanys?
b) Quins són els màxims beneficis que obtenen i quan es produeixen?
c) Quina de les dues empreses comença a notar abans un descens dels beneficis?
d) En algun moment tenen pèrdues?
a) S’obtindran guanys sempre que els guanys siguin superiors a 0:

75 x 100 x + 4
> 0 "x > 0 > 0 "x > 0
x 2 + 100 x 2 + 150

És a dir, sempre tindran guanys.

75( x 2 - 100) -4(25 x 2 + 2 x - 3 750) 1


b) fA ´( x ) = - = 0  x = 10 fB ´( x ) = = 0  x = (1 + 93 751)
( x 2 + 100)2 ( x 2 + 150)2 25

Per comprovar si és màxim analitzem la segona derivada.

150 x ( x 2 - 300) 3
fA ´´( x ) =  fA ´´(10) = - <0
( x 2 + 100)3 80

8(25 x 3 + 3 x 2 - 11250 x - 150) æ1 ö


fB ´´( x ) =  fB ´´ççç (2 + 93751)÷÷÷ » -0,026 < 0
( x 2 + 150)3 è 25 ø

Atès que el candidat té signe negatiu en la segona derivada per ambdues funcions, és un màxim.

740
Aplicacions de la derivada. Representació de funcions 1212

Els màxims guanys són:

15 æ1 ö 1
fA (10) = fB ççç (2 + 93 751)÷÷÷ = (1 + 93 751)
4 è 25 ø 75

15 1
c) Atès que < (1+ 93 751) , la primera comença a notar abans que la segona el descens dels guanys.
4 75

d) Mai tenen pèrdues ja que les funcions són sempre més grans que 0.

105. Un banc ofereix als seus clients un pla en el qual el seu capital s’incrementaria segons la funció
f(x) = 0,0001x4 – 0,04x2, en què x es mesura en milers d’euros. Efectua un estudi per determinar a
partir de quina quantitat és rendible aquesta operació, si el banc imposa un límit de 50.000 €
d’inversió.
æxö
4
x2 ïì x = 0 ìïf ( x ) < 0 en (0, 20 )
f ( x ) = çç ÷÷÷ - = 0  ïí  ïí
çè 10 ø 25 ïîï x = 20 ïïf ( x ) > 0 en (20, 50 )
î

Es calcula la primera derivada, veiem els intervals de f(x) on creix i a quins decreix.

4x3 2x ïì x = 0 f ´( x ) < 0  0 < x < 14,14213


f ´( x ) = - = 0  ïí 
10 4 25 ï
ïî x = 200 = 14,14213 f ´( x ) > 0  14,14213 < x < 50

12 x2 2
f ´´( x) = -  f ´´(14,14213) > 0  x = 14,14213 és mínim
10 000 25

A partir de 20 000 € és, rendible ja que la funció és positiva i creixent.


 
106. La temperatura, en graus centígrads, al llarg d’una nit freda d’hivern en una població ha variat
�� �����
d’acord amb la funció ���� � , en què t ∈ [0, 8] és el temps, mesurat en hores.

a) Quina temperatura hi havia a la una de la matinada? I a les cinc de la matinada?
b) A quina hora hi havia 0 °C? I –3 °C?
c) En quin interval de temps la temperatura baixava? Durant quin interval pujava?
d) En quin període de temps la temperatura ha estat negativa? En quin període ha estat positiva?
e) Quina ha estat la temperatura màxima? I la mínima?
f ) Construeix una gràfica que il·lustri la variació de temperatura en aquestes vuit hores.
1- 5 - 6 10
a) f (1) = = - = -5 ºC
2 2

25 - 25 - 6 6
f (5) = = - = -3 º C
2 2

t 2 - 5t - 6
b) 0 =  t 2 - 5t - 6 = 0  t = 6
2

t 2 - 5t - 6 ìït = 0
-3 =  t 2 - 5t - 6 = - 6  t 2 - 5t = 0  ïí
2 ïïît = 5

741
Estadística unidimensional
Estadística unidimensional  13
10
13  

ACTIVITATS
 
1. En una revista llegim que el pastor alemany té una alçada mitjana de 55 cm. Et sembla que han
mesurat tots els pastors alemanys del planeta? Explica com penses que han arribat a aquesta
conclusió.
No s’ha mesurat a tots els pastors alemanys.
Per arribar a aquesta conclusió s’ha fet un estudi estadístic on s’haurà mesurat una mostra representativa
de pastors alemanys.

2. Analitza si té sentit escollir una mostra per portar a terme un estudi estadístic sobre l’alçada d’un
grup de 10 alumnes.
No té sentit perquè mesurar 10 alumnes no és gaire costós.
 
3. Indica el tipus de variable estadística que estudiem en els casos següents:
a) La quantitat de llibres que llegeix cadascun dels teus companys de classe.
b) El color de la samarreta dels clubs esportius d’una ciutat.
c) La distància que tenen fins a la feina dels veïns d’un edifici.
a) Quantitativa discreta.
b) Qualitativa.
c) Quantitativa contínua.
 
4. En una classe amb 30 alumnes elaboren una enquesta sobre la quantitat d’aplicacions que han
comprat durant l’últim mes per al telèfon intel·ligent, i han obtingut les respostes següents:

 
Organitza aquestes dades en una taula de freqüències.

Nre aplicacions  fi  hi  Fi  Hi 


6  2 0,067 2 0,067
5  2 0,067 4 0,134
4  1 0,033 5 0,167
3  7 0,233 12 0,4
2  6 0,2 18 0,6
1  9 0,3 27 0,9
0  3 0,1 30 1
Total  30 1

743
Estadística unidimensional
Estadística unidimensional 13

5. Completa la taula de freqüències de l’estatura del grup de persones que tens a continuació:

Quin percentatge de persones fan entre 165 cm i 175 cm?

 
El percentatge de persones que fan entre 165 cm i 175 cm és del 27 %.
 
6. En el poliesportiu del barri han format un equip juvenil d’atletisme. Els membres d’aquest equip
tenen les edats següents:

 
Representa aquestes dades en el gràfic que consideris adequat utilitzant les freqüències relatives.

Edat fi hi
13 2 0,1
14 4 0,2
15 5 0,25
16 6 0,3
17 3 0,15
Total 20 1
 
Com que volem utilitzar les freqüències relatives, dibuixem un diagrama de sectors.
14
Edat 

15 
13 

17 

16
 

744
Estadística unidimensional 1313

7. Sobre un total de 722 703 cotxes, els cinc models més venuts l’any 2013 són els següents.
Representa’ls de la manera que consideris més adequada.

Representem les dades en un diagrama de barres:

 
 
8. Completa la taula de freqüències següent i dibuixa’n l’histograma i el polígon de freqüències:

 
xi  [50, 60) [60, 70) [70, 80) [80, 90) Total 
fi  25 30 15 5 75 
hi  0,333 0,4 0,2 0,067 1 
 
fi 

xi 
 
   

745
Estadística unidimensional
Estadística unidimensional 13

9. La quantitat de vegades que han anat al cinema 20 persones ha estat:

 
Agrupa les dades en intervals d’amplitud 5 i dibuixa’n un histograma.

Nre. vegades [5, 10)  [10, 15) [15, 20) [20, 25) [25, 30) [30, 35)  Total 

fi  5  5 4 2 2 2  20 
hi  0,25  0,25 0,2 0,1 0,1 0,1  1 
Nº 
persones 

Nº 
vegades
 
 
10. Una enquesta efectuada a 10 pilots, en la qual els pregunten sobre el nombre de vols setmanals que
fan, mostra les dades següents:

Calcula’n i interpreta’n les mesures de centralització.


▪ Mo = 1 → El més freqüent és que els pilots volin una vegada per setmana.

▪ Me = 1 → El valor central és 1 vol, és a dir, hi ha tants pilots que volen una o més vegades com els que
volen una o menys vegades.

0 ⋅ 2 + 1⋅ 4 + 2 ⋅ 3 + 3 ⋅ 1
▪ x= = 1,3 vols de mitjana que fa cada pilot per setmana.
10

11. A partir d’aquestes dades relatives a la quantitat d’hores d’estudi setmanals d’un grup de 30
alumnes, construeix-ne la taula de freqüències sense agrupar les dades en intervals i calcula’n les
mesures de centralització:

Interpreta els resultats que n’has obtingut.

746
Estadística unidimensional 1313

Hores d’estudi 0  1 2 3 4 5  Total 
fi  5  8 7 5 4 1  30 
FI  5  13 20 25 29 30   
hi  0,167  0,267 0,233 0,167 0,133 0,033  1 

Calculem les mesures de centralització:

▪ Mo = 1 hora → El més freqüent és que els alumnes estudiin una hora per setmana.

▪ Me = 2 → El valor central és 2 hores, és a dir, hi ha tants alumnes que estudien 2 o més hores com que
estudien 2 o menys hores.

0 ⋅ 5 + 1⋅ 8 + 2 ⋅ 7 + 3 ⋅ 5 + 4 ⋅ 4 + 5 ⋅ 1 
▪ x= = 1,93 hores a la setmana de mitjana estudien els alumnes.
30

12. Calcula les mesures de centralització d’aquestes dades:

 
Les marques de classe són, respectivament, 2, 4, 6, 8 i 10. Així:
2 ⋅ 3 + 4 ⋅ 7 + 6 ⋅ 8 + 8 ⋅ 4 + 10 ⋅ 3
Mo = 6 Me = 6 x= = 5,76
25

13. Determina les mesures de centralització d’aquestes dades:

Les marques de classe són, respectivament, 2, 6, 10, 14 i 18. Així:


2 ⋅ 5 + 6 ⋅ 2 + 10 ⋅ 1 + 14 ⋅ 4 + 18 ⋅ 6 
Mo =18 Me = 14 x= = 10,8
18

14. Calcula i interpreta els quartils de les notes de l’examen de Matemàtiques que reflecteix aquesta
taula:

 
 
Calculem les freqüències acumulades:

F1 = 3 F2 = 9 F3 = 18 F4 = 26 F5 = 37

F6 = 50 F7 = 65 F8 = 76 F9 = 85 F10 = 90

747
Estadística unidimensional
Estadística unidimensional 13

25 % de 90 = 90 · 0,25 = 22,5 50 % de 90 = 90 · 0,5 = 45 75 % de 90 = 90 · 0,75 = 67, 5

Q1 → De les freqüències acumulades, la primera que és més gran que 22,5 és F4, per tant, Q1 = 4.

Q2 → De les freqüències acumulades, la primera que és més gran que 45 és F6, per tant, Q2 = Me = 6.

Q3 → De les freqüències acumulades, la primera que és més gran que 67,5 és F8, per tant, Q1 = 8.

15. Fixa’t en el pes de nadons en el moment de néixer i calcula’n els percentils P27, P50 i P90.

Completem la taula amb les freqüències acumulades:

Pes (kg)  [2; 2,5)  [2,5; 3) [3; 3,5) [3,5; 4) [4; 4,5) Total 


xi  2,25  2,75 3,25 3,75 4,25  
fi  3  6 9 8 11 37 
Fi  3  9 18 26 37  
 
27% de 37 = 9,99 50% de 37 = 18,5 90% de 37 = 33,3
P27 → De les freqüències acumulades, la primera que és més gran que 9,99 és F3, per tant, P27 = 3,25.
P50 → De les freqüències acumulades, la primera que és més gran que 18,5 és F4, per tant, P50 = 3,75.
P90 → De les freqüències acumulades, la primera que és més gran que 33,3 és F5, per tant, P90 = 4,25.
 
16. Calcula les mesures de dispersió de les dades següents:

El nombre total de dades és N = 16.


1⋅ 1 + 2 ⋅ 5 + 3 ⋅ 7 + 4 ⋅ 2 + 5 ⋅ 1
x= = 2,8125
16

Rang = 5 – 1 = 4
1- 2,8125 ⋅ 1 + 2 - 2,8125 ⋅ 5 + 3 - 2,8125 ⋅ 7 + 4 - 2,8125 ⋅ 2 + 5 - 2,8125 ⋅ 1
DM = = 0,734375
16

12 ⋅ 1 + 22 ⋅ 5 + 32 ⋅ 7 + 42 ⋅ 2 + 52 ⋅ 1
s2 = - 2,81252 = 0,9
16

s = 0,95

0,95
CV = = 0,34
2,8125

17. Determina les mesures de dispersió d’aquestes dades:

748
Estadística unidimensional 1313

 
Les marques de classe són:

x1 = 2,5 x2 = 7,5 x3 = 12,5 x4 = 17,5

El nombre total de dades és N = 17.

2,5 ⋅ 5 + 7,5 ⋅ 6 + 12,5 ⋅ 4 + 17,5 ⋅ 2


x= = 8, 38
17

Rang = 20 – 0 = 20

2,5 - 3,38 ⋅ 5 + 7,5 - 3,38 ⋅ 6 + 12,5 - 3,38 ⋅ 4 + 17,5 - 3,38 ⋅ 2


DM = = 4, 08
17

2,52 ⋅ 5 + 7,52 ⋅ 6 + 12,52 ⋅ 4 + 17,52 ⋅ 2 4,92


s2 = - 8,382 = 24,22 s = 4,92 CV = = 0,59
17 8,38

18. Després de fer un estudi estadístic s’han obtingut les dades següents:

Calcula les mesures de centralització i de dispersió d’aquestes dades. Quines conclusions en pots
extreure si compares conjuntament les mesures que has calculat?
Contruïm la taula de freqüències:

xi 1 2 5 7 8 9 10 Total
fi 3 1 1 1 1 2 1 10
Fi 3 4 5 6 7 9 10

El nombre total de dades és N = 10.


1⋅ 3 + 2 ⋅ 1 + 5 ⋅ 1 + 7 ⋅ 1 + 8 ⋅ 1 + 10 ⋅ 2 + 10 ⋅ 1
Mo = 1 Me = 5 x= = 5,3
10

Rang = 10 – 1 = 9
1- 5,3 ⋅ 3 + 2 - 5,3 ⋅ 1 + 2 - 5,3 ⋅ 1 + 2 - 5,3 ⋅ 1 + 5 - 5,3 ⋅ 1 + 7 - 5,3 ⋅ 1 + 8 - 5,3 ⋅ 1 + 9 - 5,3 ⋅ 2 + 10 - 5,3 ⋅ 1
DM = = 3,3
10

12 ⋅ 3 + 22 ⋅ 1 + 52 ⋅ 1 + 72 ⋅ 1 + 82 ⋅ 1 + 92 ⋅ 2 + 10 2 ⋅ 1 3,55
s2 = - 5,32 = 12, 61 s = 3,55 CV = = 0,67
10 5,3

El valor mitjà de les dades és 5,3 però podem deduir que, atès que la desviació típica i el coeficient de
variació són elevats, les dades són molt disperses respecte de la mitjana.

749
Estadística unidimensional
Estadística unidimensional 13

19. La taula següent mostra l’antiguitat dels vehicles matriculats en una comarca:

Calcula’n i analitza’n les mesures estadístiques.


Completem la taula de freqüències:

Antiguitat (anys) [0, 4) [4, 8) [8, 10) [10, 12) Total

xi 2 6 9 11

fi 346 521 382 151 1 400


Fi 346 867 1 249 1 400

El nombre total de dades és N = 1 400.

2 ⋅ 346 + 6 ⋅ 521 + 9 ⋅ 382 + 11⋅ 151


Mo  6          Me  6        x= = 6,367  
1 400

Rang = 12 – 0 = 12

2 - 6,367 ⋅ 346 + 6 - 6,367 ⋅ 521 + 9 - 6, 367 ⋅ 382 + 11 - 6,367 ⋅ 151


DM = = 2,43  
1 400

2 2 ⋅ 346 + 6 2 ⋅ 521 + 9 2 ⋅ 382 + 112 ⋅ 151 2,995


s2 = - 6,367 2 = 8,97         s = 2,995     CV = = 0,47  
1 400 6,367

L’antiguitat mitjana dels vehicles del municipi és 6,367 anys, també podem deduir que, atès l’elevat valor de
la desviació típica i del coeficient de variació, les dades es troben força disperses amb respecte de la
mitjana.

20. Estudia la dispersió de les dades d’aquesta taula:

Les dades són freqüències relatives donades en percentatges. Completem la taula de freqüències:

Antiguitat (anys) [0, 5) [5, 15) [15, 35) [35, 65) 


xi  2,5 10 25 50
hi  0,22 0,5 0,25 0,03
%  22 50 25 3
 

Calculem les mesures estadístiques:

750
Estadística unidimensional 1313

n
x = å hi ⋅ x i = 2,5 ⋅ 0,22 + 10 ⋅ 0,5 + 25 ⋅ 0,25 + 50 ⋅ 0,03 = 13,3
i =1

n
s2 = å x i 2 ⋅ hi - x = (2,52 ⋅ 0,22 + 10 2 ⋅ 0,5 + 252 ⋅ 0,25 + 50 2 ⋅ 0,03) - 13,32 = 105,735
i =1 s = 10,28

10,28
CV = = 0,77
13,3

Podem deduir que les dades es troben molt disperses respecte de la mitjana, que és 13,3.
 
21. Indica en quina de les mostres hi ha les dades més concentrades i si hi ha alguna dada atípica.
Mostra A: 86962367
Mostra B: 11 13 15 13 11 12 13 22
▪ Mostra A:

xi  2  3  6 7 8 9 Total 
fi  1  1  3 1 1 1 8 
 
2 ⋅ 1+ 3 ⋅ 1+ 6 ⋅ 3 + 7 ⋅ 1+ 8 ⋅ 1+ 9 ⋅ 1
xA = = 5, 875  
8

2 2 ⋅ 1 + 3 2 ⋅ 1 + 6 2 ⋅ 3 + 72 ⋅ 1 + 8 2 ⋅ 1 + 9 2 ⋅ 1 2,2
s2 A = - 5,8752 = 4,86 s A = 2,2      CVA = = 0,38  
8         5,875

Analitzem l’existència de dades atípiques. Les dades que no es trobin en l’interval podrien considerar-se
atípiques:

( x A - 3s A , x A + 3s A ) = (-0,725; 15,475)  → No hi ha dades atípiques. 

▪ Mostra B:

xi  11  12 13 15 22 Total 


fi  2  1 3 1 1 8 
 
11⋅ 2 + 12 ⋅ 1 + 13 ⋅ 3 + 15 ⋅ 1 + 22 ⋅ 1
xB = = 13, 75
8  
112 ⋅ 2 + 122 ⋅ 1 + 132 ⋅ 3 + 152 ⋅ 1 + 222 ⋅ 1 3,345
s2 B = - 13,752 = 11,1875 CVB = = 0,24
sB = 3,345 13,75
8             
Analitzem l’existència de dades atípiques. Les dades que no es trobin en aquest interval es podrien
considerar atípiques:

( x B - 3sB , x B + 3sB ) = (3,715; 23,785) → No hi ha dades atípiques.

Analizant les mesures estadístiques obtingudes, podem concloure que les dades es concentren al voltant de
la mitjana en la mostra B.

 
SABER FER

751
Estadística unidimensional
Estadística unidimensional 13

22. Agrupa aquestes dades en intervals i organitza-les en una taula de freqüències:

Calculem el nombre d’intervals que utilizarem:

30 » 5,5  6 intervals  
Calculem l’amplitud de cada interval:
Max - Min 81- 34
= » 8,58  → L’amplitud dels intervals serà 9. 
N 30

Contruïm la taula de freqüències:

Dades  xi  fi  Fi  hi  Hi 

[34, 43)  38,5 6 6 0,2 0,2


[43, 52)  47,5 5 11 0,167 0,367 
[52, 61)  56,5 6 17 0,2 0,567 
[61, 70)  65,5 7 24 0,233 0,8
[70, 79)  74,5 4 28 0,133 0,933 
[79, 88)  83,5 2 30 0,067 1
Total    30 1
 

23. En un estudi sobre el comerç exterior, aquesta taula recopila les dades obtingudes, en milions
d’euros, en importacions i exportacions durant un any:

 
Construeix-ne un diagrama de barres adossades i interpreta’l.

752
Estadística unidimensional 1313

Milions de € 

Exportacions 
Importacions 

Alimentació  Automòbils  Altres béns  Prod.  Prod.  Prod.  Maquinària Transport  Altres béns del 


i beguda  de consum  agricultura  energètics  industrials  capital 
 
Aquest diagrama contrasta les exportacions i importacions fetes en un país en un any. Les importacions
tenen molt més pes que les exportacions, excepte en Alimentació i begudes, destaquen les exportacions, i
en Automòbils i Transport, es troba a nivell semblant. El país pateix un dèficit comercial.

 
24. Dibuixa i analitza la piràmide de població de l’any 2013.

 
 
▪ L’esperança de vida és força alta.

▪ Les dones tenen una esperança de vida més gran que els homes.

753
Estadística unidimensional
Estadística unidimensional 13

▪ La major part de la població té entre 30 i 54 anys i és escasa la població de menys de 20 anys, això
augura un envelliment de la població.

 
Dones                    Edats 
                Homes
 
 
Hombres
 
 
 
 
 
 
 
 
 
 
  Milions 
d’habitants 
 
25. Estudia la simetria de les dades.

Completem la taula amb les freqüències acumulades:

xi  1  2  3  4 5 6 7 8  Total

fi  1  1  1  2 3 5 9 10  32

Fi  1  2  3  5 8 13 22 32   

Calculem els cuartils:

25 % de 32  8        50 % de 32  16      75 % de 32  24 

Q1 → De les freqüències acumulades, la primera que és més gran que 8 és F5, per tant, Q1 = 5.

Q2 → De les freqüències acumulades, la primera que és més gran que 16 és F7, per tant, Q2 = Me = 7.

Q3 → De les freqüències acumulades, la primera que és més gran que 24 és F8, per tant, Q3 = 8.

754
Estadística unidimensional 1313

Dibuixem el diagrama de caixa:

Q1  5  Q2  Me  7  Q3  8   

Les dades més grans que la mitjana s’agrupen en l’interval d’amplitud més petita que les dades inferiors,
així donc, les dades presenten una asimetria per la dreta.

26. Determina les mesures estadístiques, mitjançant la calculadora, de la taula següent, que mostra la
despesa en aigua dels veïns d’un edifici:

 
 
Les marques de classe de cada interval són:
x1  22,5      x2  27,5      x3  32,5      x4  37,5      x5  42,5    x6  47,5 
6,283
x = 33,88          s2 = 39,477  s = 6,283            CV = = 0,185  
33,88

 
27. Les mesures estadístiques de les notes d’una classe son aquestes: ࢞
ഥ = 5,7; Me = 4; Mo = 4; ો = 1,70.
Interpreta-les.
La nota mitjana de la classe ha estat 5,7, tot i que la majoria ha obtingut un 4 i hi ha tanta gent que ha
obtingut un 4 o més com un 4 o menys.

1,70
CV = = 0,30 no és molt alt, el que indica que les notes s’han concentrat sobre el 5,7.
5,7

Les tres mesures centrals no són molt pròximes, el que podria significar que hi ha hagut majoria de
suspesos.
 
28. Les dades mostren les precipitacions, en ℓ /m2, en dues ciutats les últimes 6 setmanes.

A i B pertanyen a la mateixa zona climàtica? Per què?


Fem un estudi estadístic sobre cada una de les ciutats:

Ciutat A: Ciutat B:

6,07 36,057
x A = 21,667      s A = 6,07     CVA = = 0,28     x B = 22,833   sB = 36,057    CVB = = 1,58  
21,667 22,833

755
Estadística unidimensional
Estadística unidimensional 13

Malgrat que la mitjana de precipitacions és molt semblant en les dues ciutats, el coeficient de variació és
moltíssim més petit a la ciutat A que a la B, la qual cosa significa que les precipitacions setmanals són molt
properes a la mitjana, o el que és el mateix, són més regulars. Això ens permet concloure que A i B no
pertanyen a la mateixa zona climàtica.

 
29. Completa la taula i calcula’n les mesures estadístiques.

Dades  xi fi  Fi fi ∙ xi fi ∙ xi2


[0, 2)  1 5  5 5 5
[2, 4)  3 2  7 6 18
[4, 6)  5 8  15 40 200 x =5 
[6, 8)  7 8  23 56 392 s2 = 6,08  
s = 2,466  
[8, 10)  9 2  25 18 162
2,466
CV = = 0, 493
Total    25  125 777 5  
 
30. Construeix un histograma amb aquestes dades:

 
fi 

xi 
 
 
   

756
Estadística unidimensional 1313

ACTIVITATS
 
31. Volem fer un estudi sobre pràctica esportiva en els alumnes d’ESO del nostre institut; els volem
preguntar quants dies al mes fan alguna activitat esportiva.
a) Quina mostra escolliries per efectuar l’estudi?
b) De quina grandària és la mostra?
c) Quina és la població?
a) S’haurien d’escollir nois i noies dels quatre cursos d’ESO.

b) Dependrà de la quantitat d’alumnes de l’institut.

c) Tots els alumnes d’ESO d’aquest institut.


 

32. Indica el tipus de variable estadística que estudiem i justifica, en cada cas, què seria més
aconsellable: estudiar una mostra o la població.
a) Els diners que gasten en una setmana els teus amics.
b) La talla de camisa de tots els habitants de la teva ciutat.
c) El color d’ulls dels membres de la teva família.
d) La temperatura a les 12 del migdia d’avui de tots els pobles de la teva comarca.
e) L’alçada dels jugadors de bàsquet de la selecció catalana.
f ) El nivell de colesterol a la sang de tots els catalans.
g) L’edat de tots els alumnes d’una escola.
a) Quantitativa contínua. Estudiar la població.

b) Quantitativa discreta. Estudiar una mostra.

c) Qualitativa. Estudiar la població.

d) Quantitativa contínua. Estudiar una mostra.

e) Quantitativa contínua. Estudiar una mostra.

f) Quantitativa contínua. Estudiar una mostra.

g) Quantitativa discreta. Estudiar una mostra.

33. L’entrenador d’un club de futbol ha apuntat la quantitat de targetes vermelles que han vist al llarg de
la temporada tots els integrants de la plantilla. Les dades són:

a) Organitza els resultats en una taula de freqüències.


b) Quin significat té en aquest cas la columna de les freqüències acumulades?
   

757
Estadística unidimensional
Estadística unidimensional 13

a) Nre targetes  fi  Fi  hi  Hi 


vermelles 
  0  6  6  0,24 0,24
  1  8  14  0,32 0,56
  2  6  20  0,24 0,8
  3  1  21  0,04 0,84
  4  3  24  0,12 0,96
  5  1  25  0,04 1
  Total  25   1

b) El nombre de jugadors que van tenir menys d’i targetes.

34. S’han perdut algunes dades d’aquesta taula amb les poblacions de naixement d’un grup de
persones.
La podries completar?

Població fi hi Percentatge
Amposta 12 0,2 20 %
Cervera 9 0,15 15 %
Collbató 21 0,35 35 %
Gandesa 6 0,1 10 %
Terrassa 12 0,2 20%
Total 60 1 100%

35. Completa la taula relativa a la marca de telèfon mòbil d’un grup d’alumnes.

758
Estadística unidimensional 1313

Marca fi hi Percentatge
Apple 4 0,16 16 %
Samsumg 12 0,48 48 %
LG 3 0,35 12 %
Sony 5 0,1 20 %
Shen Wai 1 0,04 4%
Total 25 1 100%
 
36. Determina la moda, la mitjana i la mitjana aritmètica de les sèries de dades següents:
a) 7, 9, 7, 9, 7, 9, 7, 9 b) 2, 4, 8, 4, 5, 5, 7 c) 12, 2, 6, 5, 9, 5, 3
d) 3, 5, 9, 5, 6, 6, 9, 5 e) 6, 6, 5, 5, 4, 8, 4, 2
a)
xi  7  9 Total 
fi  4  4 8 
Fi  4  8  
 
Mo  {7, 9}      Me  8        x =8 

b)
xi  2  4 5  7 8 Total
fi  1  2 2  1 1 7
Fi  1  3 5  6 7
 
Mo  {4, 5}        Me  5        x =5 

c)
xi  2  3 5  6 9 12 Total
fi  1  1 2  1 1 1 7
Fi  1  2 4  5 6 7
 
Mo  5          Me  5        x =6 

d)
xi  3  5 6  9 Total
fi  1  3 2  2 7
Fi  1  4 6  8
 
Mo  5          Me  5        x = 5,5  

   

759
Estadística unidimensional
Estadística unidimensional 13

e)
xi  2  4 5  6 8 Total
fi  1  2 2  2 1 8
Fi  1  3 5  7 8
 
Mo  {4, 5, 6}       Me  5        x =5 
 

37. Per a una mostra de 572 conductors d’autobús han registrat el nombre d’accidents en els quals han
estat implicats durant 4 anys.

a) Elabora’n una taula de freqüències.


b) Calcula’n les mesures de centralització.
c) Determina els valors de les mesures de dispersió.
a)
xi  0  1  2  3 4 5 6 7 8  9  Total
fi  121  156 130  60 57 23 12 9 3  1  572
Fi  121  277 407  467 524 547 559 568 571  572 
hi  0,212  0,273 0,227  0,105 0,1 0,04 0,021 0,016 0,005  0,001  1
Hi  0,212  0,485 0,712  0,817 0,917 0,957 0,978 0,994 0,999  1 
 
1107
b)  Mo  1    Me  2      x= = 1, 935  
572

760,164 3 849 1,727


c) DM = = 1,329       s2 = - 1,9352 = 2,984        s = 1,727      CV = = 0,89  
572 572 1,935
 

38. El nombre de germans d’un grup d’alumnes de 1r de Batxillerat, exclosos els alumnes, està recopilat
a la taula següent:

Troba’n les mesures de centralització.


El nombre total de dades és N = 30.
43 
Mo  1 germà          Me  1 germà       x= = 1,43  germans
30

   

760
Estadística unidimensional 1313

39. Afegeix una dada a aquest conjunt perquè la mitjana sigui 6. Digues si la mitjana varia.
333444555666777
3 ⋅ (3 + 4 + 5 + 6 + 7)
La mitjana de les dades actuals és x = =5
15

N = 16  6 ⋅ 16 = 96 96 - 3 ⋅ (3 + 4 + 5 + 6 + 7) = 21 → La nova dada hauria de ser 21.

40. En una enquesta han preguntat per la quantitat de televisors que hi ha a les llars. Les respostes
estan reflectides a la taula següent. Determina’n les mesures de centralització: mitjana, mediana i
moda.

 
 
El nombre total de dades és N = 25.
41
x= = 1,64 televisors Me = 2 televisors Mo = 2 televisors
25

41. A la taula següent hi ha aplegada informació relativa a les edats dels socis infantils i juvenils d’un
club de natació:

a) Calcula’n la mitjana aritmètica.


b) Calcula’n la mediana.
c) Determina’n la moda.

Les marques de classe dels intervals són:


x1 = 6,5 x2 = 9,5 x3 = 12,5 x4 = 15,5 x5 = 18,5
El nombre total de dades és N = 140.
1 690
a) x = = 12,071 anys b) Me = 12,5 anys c) Mo = 6,5 anys
140

42. Donades les dades següents, troba una quantitat tal que sumada o restada a cadascuna de les dades
faci que la mitjana sigui 1.

 
31
Calculem la mitjana de les dades: x = = 3,1
10

La quantitat que hem de sumar a les dades perquè la mitjana sigui 1 serà: 1 = 3,1 = –2,1.
Per tant, l’hi hem de restar 2,1 a totes les dades perquè la seva mitjana sigui 1.

761
Estadística unidimensional
Estadística unidimensional 13

43. En un gimnàs han ingressat aquest any 160 socis nous. Les seves edats es distribueixen tal com
podem veure a la taula següent.
Determina els percentils 30, 40, 60, 80 i 90.

Completem la taula amb les freqüències acumulades:

Edat  18  19  20 21 22 23 24 Total 


fi  22  36  40 24 16 12 10 160 
Fi  22  58  98 122 138 150 160  
 
30 % de 160  48 → P30  19            80 % de 160  128 → P80  22 
40 % de 160  64 → P40  20            90 % de 160  144 → P90  23 
60 % de 160  96 → P60  20 
 
44. El tutor de 1r A ha fet una relació dels alumnes del grup ajuntats pel nombre de suspensos que han
tingut en la primera avaluació.

 
 
Calcula’n:
a) La mitjana aritmètica. c) La variància.
b) La desviació mitjana. d) La desviació típica.
38 94
a) x = = 1,1875 c) s2 = - 1,18752 = 1,527
32 32
31,875
b) DM = = 0,996 d) s = 1,236
32

 
45. Les marques, en centímetres, dels saltadors d’altura d’un equip juvenil són les següents:
192 178 186 202 199 181 188 185 179 194
Determina’n:
a) La mitjana aritmètica. c) La variància.
b) La desviació mitjana. d) La desviació típica.
1884 355556
a)  x = = 188,4                c)  s2 = - 188,42 = 61,04  
10 10
66, 8
b)  DM = = 6,68               d)  s = 7,8128  
10

762
Estadística unidimensional 1313

46. Un supermercat porta a terme un estudi de mercat per adequar la seva oferta amb els gustos dels
clients.
Avui han controlat la quantitat d’articles diferents que portaven al carret els clients quan passaven
per la caixa. Han resumit les dades de la manera següent:

a) Troba la quantitat mitjana d’articles diferents que porta cada carret que passa per les caixes del
supermercat.
b) Calcula’n també les mesures de dispersió.

c) Interpreta els resultats que has obtingut.


Les marques de classe són:

x1  2,5      x2  7,5      x3  12,5      x4  17,5      x5  22,5   x6  27,5 

I el nombre total de dades és: N = 114.

1 020
a) x = = 8, 947 artícles 
114

488,421 112 512,5 5,45


b) DM = = 4,284        s2 = - 8,9472 = 29,7       s = 5,45    CV = = 0,61  
114 114 8,947

c) Cada client porta de mitjana 8,947 artícles, gairebé 9. No obstant, el coeficient de variació indica que les
dades no són gaire concentrades a la mitjana.

47. El seleccionador català de bàsquet femení valora incloure una jugadora a l’equip. N’ha estudiat els
últims 10 partits que ha jugat i ha apuntat el nombre de minuts que ha jugat i els punts que ha
encistellat. Ho pots veure en aquesta taula:

763
Estadística unidimensional
Estadística unidimensional 13

a) Calcula la mitjana aritmètica i la desviació típica de les variables que ha estudiat.


b) Determina el coeficient de variació en les variables per estimar quina de les dues és més dispersa.
a) La mitjana i la desviació típica dels minuts jugats són:
290 8 798
xM = = 29 sM 2 = - 292 = 38, 8  sM = 6,229
10             10  
La mitjana i la desviació típica dels punts anotats són:
176
xP = = 17,6  
10
3 618
sP 2 = - 17,62 = 52,04  sP = 7,214  
10

b) El coeficient de variació mitjana dels minuts jugats és:


6,226
    CVM = = 0,215 . 
29

El coeficient de variació mitjana dels punts anotats és:


7,214
CVP = = 0,41 . 
17,6

CVP > CVM → Els punts anotats són més dispersos que els minuts jugats.
 
48. Una empresa elabora un estudi sobre el salari anual dels seus treballadors; els resultats estan
recopilats a la taula següent:

a) Calcula’n les mesures de centralització.


b) Determina’n les mesures de dispersió.
Completem la taula de freqüències:

Salari 
[10, 15)  [15, 20)  [20, 25)  [25, 30)  [30, 35)  [35, 40)  Total 
(milers d’€) 

xi  12,5  17,5  22,5 27,5 32,5 37,5   


fi  21  32  85 90 25 5  258 
Fi  21  53  138 228 253 258   
 
a)  Mo  27,5     Me  27,5 
6 210
    x= = 24, 07  
258
1173,256
b)  DM = = 4,55  
258
157 612,5
    s2 = - 24,072 = 31,547  
258

    s = 5,617  

764
Estadística unidimensional 1313

5,617
    cv = = 0,233  
24,07

 
49. Analitza els salaris mensuals d’aquestes dues empreses mitjançant les mesures de centralització i
les mesures de dispersió:

 
Les marques de classe són, pels dos estudis, les següents:

x1  800        x2  1 200       x3  1 600       x4  1 900    x5  2 200 

▪   Empresa A: 

El nombre total de treballadors és NA  100. 

Les mesures de centralització són:

MoA  1 600 €     MeA  1 600 € 

155 000
xA = = 1550 €
100  

Les mesures de dispersió són:

36 500
DMA = = 365  
100
260 000 000
sA 2 = - 1550 2 = 197 500     sA = 444,41  
100  
444,41
CVA = = 0,287  
1 550

▪ Empresa B:

El nombre total de treballadors és NB = 109.

Les mesures de centralització són:

MoB  1 600 €     MeB  1 600 € 

167 600
xB = = 1537,61 €
109  

Les mesures de dispersió són:

765
Estadística unidimensional
Estadística unidimensional 13

35 009,17
DMB = = 321,185  
109

274 240 000


sB 2 = - 1537,612 = 151 704,402     sB = 389,45  
109
389,45
CVB = = 0,253  
1537,61

 
50. La Xènia treballa d’operadora telefònica a l’empresa de telecomunicacions Lemon. Li han encarregat
que efectuï una enquesta a tots els clients amb qui parli.
Els fa només tres preguntes:
X) Quant de temps ha passat des que ha marcat el nostre número d’atenció al client i el moment en
què hem resolt el seu problema?
Y) Valoració personal del servei prestat: MB (molt bé), B (bé), R (regular), M (malament) i MM (molt
malament).
Z) Quantitat d’anys que fa que és client de l’empresa de telecomunicacions Lemon.
L’operadora ha resumit les dades dels 40 últims enquestats en una taula per passar-les al
departament de màrqueting.

a) Agafa cadascuna de les variables i fes-ne un recompte (si hi ha molts valors diferents, agrupa’ls
en classes).
b) Confecciona la taula de freqüències amb cadascuna.
c) Fes el gràfic que et sembli més adequat en cada cas.

766
Estadística unidimensional 1313

d) Determina’n, si és possible, les mesures de centralització.


e) Troba els quartils inferior i superior de cadascuna.
f ) Si és possible, determina les mesures de dispersió de cadascuna de les tres sèries.
g) Entre la primera variable i la tercera, determina quina és més dispersa.
a) ▪ Variable X:

Agrupem les dades en intervals de classe. El nombre d’intervals serà 40 = 6,3 » 7 , i la seva mida serà

23 - 1
= 3,5 » 5 .
40

Minuts  [1, 6)  [6, 11)  [11, 16) [16, 21) [21, 26) Total 

xi  3,5  8,5  13,5 18,5 23,5  


fi  14  12  8 3 3 40 
 
▪ Variable Y:

Valoració  MB  B  R M MM Total 

fi  10  14  8 5 3 40 


 
▪ Variable Z:

Anys  1  2 3  4 5 6 7 8 9 Total 

fi  3  7 10  10 4 3 2 0 1 40 


 
b) ▪ Variable X:

Minuts  [1, 6)  [6, 11)  [11, 16) [16, 21) [21, 26) Total 

xi  3,5  8,5  13,5 18,5 23,5  


fi  14  12  8 3 3 40 
Fi  14  26  34 37 40  
hi  0,35  0,3  0,2 0,075 0,075 1 
Hi  0,35  0,65  0,85 0,925 1  
 
▪ Variable Y:

Valoració  MB  B  R M MM Total 

fi  10  14  8 5 3 40 


hi  0,25  0,35  0,2 0,125 0,075  
▪ Variable Z:

Anys  1  2 3  4 5 6 7 8 9  Total 

767
Estadística unidimensional
Estadística unidimensional 13

fi  3  7 10  10 4 3 2 0 1  40 


Fi  3  10  20  30 34 37 39 39 40   
hi  0,075  0,175  0,25  0,25 0,1 0,075 0,05 0 0,025  1 
Hi  0,075  0,25  0,5  0,75 0,85 0,925 0,975 0,975 1   

c) ▪ Variable X:
Nre. clients 

Minuts 
 
▪ Variable Y:
Nre. clients 

Satisfacció
 
▪ Variable Z:
Nre. clients 

Anys 
 
d) ▪ Variable X:
385
MoX  3,5 minuts     MeX  8,5 minuts       xx = = 9,625  minuts 
40

▪ Variable Y:
És una variable qualitativa, per tant l’única mesura de centralització que podem calcular és la moda:
MoY  B 

768
Estadística unidimensional 1313

▪ Variable Z:
148
MoZ  {3, 4}          MeZ  3,5              xZ = = 3,7  
40

e) 25 % de 40 = 10 i 75 % de 40 = 30.
▪ Variable X:
QX1  3,5            QX3  13,5 
▪ Variable Y:
És una variable qualitativa, per tant no té sentit calcular els cuartils.
▪ Variable Z:
QZ1  2,5            QZ3  4,5 
f) ▪ Variable X:
198,5 5 180 6,07
DMX = = 4,96       sX 2 = - 9,6252 = 36,86    sX = 6,07   CVX = = 0,63  
40 40 9,625

▪ Variable Y:
És una variable qualitativa, per tant no es poden calcular les mesures de dispersió.
▪ Variable Z:
54 668 1,735
DMZ = = 1,35         sZ 2 = - 3,72 = 3,01       sZ = 1,735   CVZ = = 0,469  
40 40 3,7

g) CVX > CVZ → La variable X és més dispersa que la Z.


 
51. El 15 % dels científics d’un centre d’investigació parlen en un sol idioma. El 48 % poden parlar en
dos idiomes diferents. A més, hi ha un 28 % de científics que són capaços de comunicar-se en tres
llengües, i la resta parlen quatre idiomes o més.
Elabora una taula de freqüències amb aquestes dades. Determina si hi ha algun tipus de freqüència
que contesti la pregunta de quants científics del centre parlen menys de tres llengües. Justifica la
resposta.

Nre.  1 2 3 4 o   Total


idiomes 
%  15 48 28 9 100
hi  0,15 0,48 0,28 0,09 1
Hi  0,15 0,63 0,91 1
 
Sí, la freqüència relativa acumulada H2 = 0,63 = 63 %.
   

769
Estadística unidimensional
Estadística unidimensional 13

 
52. Calcula la mitjana aritmètica, la desviació típica i el coeficient de variació de les dades d’aquesta
sèrie:
21 34 28 26 24 25 20 30
a) Escriu una altra sèrie estadística amb la mitjana igual al doble de l’anterior i el coeficient de
variació més petit.
b) Escriu una altra sèrie diferent amb la mitjana igual a la meitat que la primera i el coeficient de
variació més gran.
208 5 558 4,33
x= = 26         s2 = - 262 = 18,75  s = 4, 33       CV = = 0, 167  
8 8 26

a) Resposta oberta. Per exemple:

Si sumem la mitjana a totes les dades, la sèrie resultant tindrà de mitjana el doble de la inicial i la seva
desviació típica no variarà, per tant, el seu coeficient de variació disminuirá:

47     69      54      52      50      51      46      56 

416 21 782 4,33


x= = 52 = 2 ⋅ 26      s2 = - 56 2 = 18,75  s = 4, 33          CV = = 0, 083  
8 8 52

b) Resposta oberta. Per exemple:

26
La mitjana ha de ser = 13 . De la mateixa manera que a l’apartat anterior, si restem 13 a totes les dades,
2

la sèrie obtinguda tindrà una mitjana que és la meitat de la inicial i la seva desviació típica no variarà, així el
seu coeficient de variació augmentarà:

8      21      15      13      11      12      7      17 

104 26 1502 4,33


x= = 13 =        s2 = - 132 = 18,75  s = 4, 33       CV = = 0, 333  
8 2 8 13

53. El seleccionador català de bàsquet estudia d’una manera molt acurada les característiques de tots
els seus jugadors. A la taula següent hi ha reflectides la quantitat de faltes personals que ha comès
un jugador en concret en els últims 50 partits:

Com pots comprovar, s’han perdut un parell de dades. Intenta recuperar-les si sabem que la
quantitat mitjana de faltes personals d’aquest jugador ha estat de 2,62.
Siguin x i y, respectivament, la primera i la segona dada desconeguda.

Sabem que la suma del total de partits és 50, és a dir,:

4  6  x  14  y  1  50 → x  y  25 

770
Estadística unidimensional 1313

També sabem que la mitjana de faltes personals és 2,62, és a dir,:

5 ⋅ 4 + 4 ⋅ 6 + 3 ⋅ x + 2 ⋅ 14 + 1⋅ y + 0 ⋅ 1
x= = 2,62  3 x + y = 59  
50

Amb les equacions obtingudes plantegem un sistema d’equacions i el resolem:

ü
x + y = 25ï
ï
ý  x = 17, y = 8  
3 x + y = 59ï
ï
þ

El jugador va cometre 3 faltes personals en 17 partits i 1 falta en 8 partits.

54. Les notes que en Daniel ha obtingut en Matemàtiques durant aquest trimestre són 4, 7, 9, 7 i 5.
a) Calcula la mitjana aritmètica, la moda i la mediana d’aquesta sèrie.
b) Encara li falta fer un examen. Quina nota hauria d’obtenir perquè la nota mitjana fos de 6,5?
c) Quines condicions hauria de complir aquesta nota que falta perquè la nova sèrie tingués la
mateixa mediana?
d) Quines condicions hauria de complir aquesta nota que falta perquè la nova sèrie tingués la
mateixa moda?
32
a)  x = = 6,4        Mo  7      Me  7 
5

b) Sigui x la nota que hauria d’obtenir. La nova mitjana serà:


32 + x
x= = 6,5  x = 7  és la nota que hauria d’obtenir en Daniel al pròxim examen. 
6

c) Ser més gran o igual que la mediana inicial.

d) Ser el valor de la moda o un valor diferent a les altres notes obtingudes.

55. Un entrenador d’un club d’atletisme ha format dos grups de corredors en la categoria de 100 m
llisos, un de masculí i un de femení. La mitjana de les marques de l’equip masculí és de 12,42 s, amb
una desviació típica d’1,2 s. La mitjana de l’equip femení és de 13,04 s, amb una desviació típica
d’1,05 s.
S’incorpora una parella al club d’atletisme; ell té una marca de 13,03 s, i ella, de 13,24 s. Si
l’entrenador només pot escollir un dels atletes, quin dels dos té millor marca en la seva categoria?
Analitzem quan allunyades de les mitjanes marques es troben les marques pel que fa a la desviació típica:

13,03 - x 13,03 - 12,42 13,24 - x 13,24 - 13,04


Home: = = 0,51 Dona: = = 0,19
s 1,2 s 1,05

El valor és més petit per a les dones, per tant, dels dos grups qui millor marca té a la seva categoria.

 
56. En Ricard es prepara per participar en la seva primera mitja marató. Cada dia surt a entrenar i apunta
la distancia que corre. Avui, que és divendres, ja ha acabat l’entrenament i pensa en quant hauria de
córrer durant el cap de setmana. A la taula següent hi ha reflectides les seves anotacions:
771
Estadística unidimensional
Estadística unidimensional 13

a) Determina la mitjana aritmètica i la desviació típica de la sèrie setmanal.


b) Afegeix dues possibles distàncies per recórrer dissabte i diumenge de manera que es mantingui
la mateixa mitjana aritmètica però que disminueixi la desviació típica.
c) Determina quines dues distàncies podríem escriure per a dissabte i diumenge de manera que es
mantingui la mitjana aritmètica però que augmentés la desviació.
d) Indica si seria possible afegir dues distàncies per a dissabte i diumenge perquè augmenti la
mitjana aritmètica però es mantingui la desviació típica.
e) Justifica si podries fer el mateix afegint dues distàncies més de manera que disminuís la mitjana
aritmètica sense que augmentés la desviació típica.
84 1 454
a) x = = 16,8 km s2 = - 16,8 2 = 8,56  s = 2,926 km
5 5

b) Per mantenir la mitjana, les noves dades, x i y, han de complir:

84 + x + y
x= = 16,8  x + y = 33,6  y = 33,6 - x  
7

Si es vol disminuir la desviació típica mantenint la mitjana, prenem dos valors molt pròxims a la mitjana,
per exemple, la propia mitjana:

2 018,48
x  16,8 km → y  16,8 km →  s2 = - 16,82 = 6,114  s = 2, 473 km  
7

c) Partim de la mateixa idea de l’apartat anterior, prenem dos valors molt llunyans de la mitjana. Per
exemple:

2 517,76
x  1 km → y  32,6 km →  s2 = - 16,8 2 = 77,44  s = 8,8 km  
7

d) Sí, n’hi hauria prou amb afegir dos valors que estiguin per sobre de la mitjana i a una distància més petita
que la desviació típica. Per exemple, 17 i 18:

119 2 067
x= = 17 km          s2 = - 172 = 6,286  s = 2,507 km  
7 7

e) Sí, n’hi hauria prou amb afegir dos valors que estiguin per sota de la mitjana i a una distància més petita
que la desviació típica. Per exemple, 15 i 14:

113 1875
x= = 16,14 km        s2 = - 16,142 = 7, 265  s = 2,7 km  
7 7

 
57. Un grup de persones participen en l’assaig clínic d’un medicament i per això els vigilen el nivell de
colesterol a la sang que presenten. Els resultats de l’última anàlisi han estat els que estan recopilats
a la taula següent:

772
Estadística unidimensional 1313

 
a) Calcula la mitjana aritmètica i la desviació típica que correspon a aquestes dades.
b) L’equip mèdic vol seleccionar un interval de nivells de colesterol que contingui el 95 % de les
persones i que estigui centrat en la mitjana, és a dir, que sigui del tipus (�
� – 2σ, � + 2σ). Quin són els
extrems d’aquest interval?
a) Les marques de classe són:
x1 = 170 x2 = 190 x3 = 210 x4 = 230 x5 = 250
El nombre total de dades és N = 120.
����� 5956000
�� � � 221� 6� �� � � 221� 6�� � �9�� 2� � �� � 22��
120 ���
b) L’interval seria:
(221,6667 – 2 · 22,3; 221,6667 + 2 · 22,3) = (177,0667; 266,2667)
 
58. L’Antoni, la Sara, la Cèlia i en Robert són quatre amics jubilats; tots quatre treballaven junts
d’estadístics al Centre d’Estadística Corporativa. Els agrada trobar-se i es diverteixen proposant
problemes.
–Ja sabeu que, curiosament, tenim el mateix nombre de néts. Avui he donat una gratificació a
cadascun dels meus
–comença en Robert–. Com que tenen edats molt diferents, he donat també diferents quantitats a
cada un.
–No sé quant els has donat, però sí que sé que no ets gaire generós; jo donaré quatre euros més que
tu a cadascun dels meus –diu la Cèlia.
–Doncs jo donaré a cadascun dels meus néts la quarta part del que els ha donar en Robert als seus,
que jo sé que no és garrepa –afirma la Sara una mica picada amb la seva amiga.
–Com que jo també crec que en Robert és una mica ronyós, donaré als meus néts quatre vegades
més que ell –contesta l’Antoni, que és el més fatxenda de tots.
–Molt bé, farem càlculs i l’estadística posarà cadascú al seu lloc –diu en Robert, una mica empipat–.
La pregunta és: si anomeno m la mitjana de les gratificacions que jo he donat, i s, la desviació típica,
quines són les mitjanes aritmètiques i les desviacions típiques de les quantitats que donareu
cadascun de vosaltres?
Pots posar una mica de pau entre aquests amics efectuant tu els càlculs?

773
Estadística unidimensional
Estadística unidimensional 13

Pista. Posa una sèrie d’exemple per a en Robert i efectua els càlculs. Després, construeix les sèries
dels altres i torna a fer els càlculs. Estableix una hipòtesi general i mira de trobar una demostració
per a aquesta hipòtesi.
La mitjana, m, i la desviació típica, s, de les propines d’en Robert són:

n n

åf ⋅ x i i åf ⋅ x i i
2

m= i =1
s= i =1
- m2
N N

La mitjana i la desviació típica de les propines de la Cèlia seran:


n n n

å f ⋅ ( x + 4) å f ⋅ xi i i i å f ⋅4 i
N
x Cèlia = i=1
= i=1
+ i=1
= m + 4⋅ = m+4
N N N N

n n n

2
å f ⋅ ( x + 4) i i
2

2
å f ⋅ (x i i
2
+ 4 xi + 16)
2
å f ⋅ (x
i i
2
+ 4 xi + 16)
sCèlia = i=1
- ( m + 4) = i=1
- ( m + 4) = i=1
- ( m + 4) 2 =
N N N

n n n n n

åf ⋅ x i i
2
+ å fi ⋅ 8 x i + å fi ⋅ 16
2
åf ⋅ x i i
2

2
åf ⋅ x i i
2

= i =1 i =1 i =1
- ( m + 4) = i =1
+ 8 m + 16 - ( m + 8 m + 16) = i =1
- m2 = s 2
N N N

sCèlia = s

La mitjana i la desviació típica de les propines de la Sara seran:

n
æ xi ö n

å f ⋅çççè 4 ÷÷ø÷ i
1 å f ⋅x i i
m
xSara = i=1
= ⋅ i=1
=
N 4 N 4

2
n
æx ö æ n ö÷
å fi ⋅ ççç i ÷÷÷ 2 2
çç
çç å fi ⋅ xi 2 ÷÷ 2 2
è4ø æ mö æ1ö ÷ æ1ö æsö s
sSara 2 = i=1
-ççç ÷÷÷ = ççç ÷÷÷ ⋅ çç i=1 - m2 ÷÷÷ = ççç ÷÷÷ ⋅ s 2 = ççç ÷÷÷  sSara =
N è 4 ø è 4ø è N ø è 4ø è 4ø 4

La mitjana i la desviació típica de les propines de l’Antoni es calculen d’igual manera a les de la Sara:

n n

å f ⋅ 4x i i å f ⋅ 4x i i
x Antoni = i=1
= 4m s Antoni 2 = i=1
- 4 m 2 = (4 s ) 2  s Antoni = 4 s
N N

774
Estadística unidimensional 1313

59. El tutor d’un curs compara les notes de dues alumnes seves, la Núria i l’Ester. Mira les notes a la
taula següent:

a) Calcula les mesures aritmètiques i les desviacions típiques de les dues assignatures per a
cadascuna de les noies. Explica què passa.
b) Qui té les notes més disperses? Quina mesura has calculat per justificar les teves afirmacions?
a) ▪ Núria:

Matemàtiques → x MMat = 6,5 sMMat = 2,5

Llengua → x MLlen = 5 sMLlen = 0,82

▪ Ester:

Matemàtiques → x EMat = 6,83 sEMat = 1,07

Llengua → xELlen = 8 sELlen = 0,82

b) Calculem els coeficients de variació per analizar la dispersió de les notes:

▪ Núria:

Matemàtiques → CVMMat = 0,38 Llengua → CVMLen = 0,16

▪ Ester:

Matemàtiques → CVEMat = 0,16 Llengua → CVELen = 0,1

Els coeficients de variació més grans són els de la Núria; per tant les seves notes són més disperses.

   

775
Estadística unidimensional
Estadística unidimensional 13

60. Als participants en un curs de submarinisme els han mesurat el temps que resisteixen sense
respirar. El resum dels resultats està presentat en aquesta taula:

a) Completa la taula de freqüències afegint-hi columnes per a la freqüència acumulada, la freqüència


relativa i el percentatge.
b) Representa les dades mitjançant el gràfic estadístic que et sembli més adequat a les
característiques de les dades.
c) Determina’n les mesures de centralització.
d) Calcula també les mesures de dispersió de la sèrie.
e) A l’escola de submarinisme afirmen que el 80 % de les persones aguanten més d’un minut sense
respirar. En aquest grup, quin percentatge ho compleixen?
f ) També afirmen que el 5 % de les persones es mantenen durant més d’un minut i mig sense
respirar. Determina si això és cert en aquest grup.

 
a)

Temps (s)  xi  fi Fi hi Hi % 


[40, 52)  46  10  10  0,08  0,08  8 
[52, 56)  54  28  38  0,224  0,304  22,4 
[56, 64)  60  25  63  0,2  0,504  20 
[64, 76)  70  44  107  0,352  0,856  35,2 
[76, 88)  82  11  118  0,088  0,944  8,8 
[88, 100)  94  7  125  0,056  1  5,6 
Total   125 1 100 

776
Estadística unidimensional 1313

b)
Nre. 
submarinistes 

Temps (s)   
8 112
c) Mo  70      Me  60    x= = 64,896  
125

1 232,896 544 224 11,929


d) DM = = 9,863        s2 = - 64,8962 = 142,3 2     s = 11,929      CV = = 0,184  
125 125 64,896

e) Com a mínim, aguanten més d’un minut sense respirar 35,2  8,8  5,6  49,6 %. 


Podrien ser més, però un dels intervals és [56, 64) i per tant no podem saber quants d’aquests intervals van
resistir el minut.

f) Hi ha un 5,6 % d’alumnes que resisteixen 88 s; les dades no permeten ser més precisos.

61. A un laboratori han arribat 24 ampolles d’aigua, 12 ampolles d’un litre i 12 ampolles de mig litre, per
analitzar-ne el contingut en sals.
Han obtingut les dades següents, expressades en mil·ligrams.
 Ampolles d’un litre:
46 25 27 30 48 40
27 44 37 62 56 29
 Ampolles de mig litre:
76 75 49 59 33 52
54 45 66 69 34 53
a) Classifica la variable estadística de concentració de sals.
b) És convenient adoptar intervals a l’hora de fer una taula?
a) La variable és quantitativa contínua.
b) Per fer la taula es poden agrupar les dades en intervals i facilitar-ne així l’estudi.
�����
√24 � 4�� � intervals � ���4�
√��

Contingut (mg) fi
[25, 36) 7
[36, 47) 5
[47, 58) 6
[58, 69) 3
[69, 80) 3
 

777
Estadística unidimensional
Estadística unidimensional 13

 
 

   

778
Estadística unidimensional 1313

PER APROFUNDIR
 
62. Escull la resposta adequada.

 
□ Atès que la mitjana és 20, la suma de les edats de tota la família és múltiple de 20. Així, si a és el
nombre de membres de la família podem escriure:

48 + edat mare + edat fills = 20a

Si no se suma l’edat del pare, la suma de les edats de la resta es podrà definir per:

Edat mare + Edat fills = 16(a – 1)

Com que s’han de complir ambdues condicions:

20 ⋅ a =16( a - 1) + 48  4a = 32  a = 8  

Si el total de membres de la família són 8, tindran 6 fills.

□ En una successió qualsevol de nombres enters consecutius i imparell, la mitjana coincidirà amb la
mediana, atès que la mitat de les dades seran en el punt mig. Per tant:

75 75
x= = = 73  
49 72

□ Si definim x com el nombre d’alumnes de 1r de Batxillerat, tenim:

2x → alumnes de 4t d’ESO 4x → alumnes de 3r d’ESO

12(4 x ) + 15(2 x ) + 10 x 88 x 88
La suma de les notes dividit pel nombre d’alumnes serà: = = .
x + 2x + 4x 7x 7

88
Per tant, la puntuació mitjana dels participants serà .
7

779
Estadística unidimensional
Estadística unidimensional 13

x
□ Si definim x com la suma del pes de les quatre patates, el pes mig per patata serà . Quan s’afegeix la
4
x
patata gegant, el pes mig passa a ser ; per tant, si se redefineix el seu pes com y, tenim:
2

3
x
x+y x 5 3 12
= y= x-xy = x 2 = =6 
5 2 2 2 x 2
4

63. Tenim 120 dades que hem classificat en tres grups i les volem representar per mitjà d’un diagrama
de sectors. Després d’analitzats aquests grups, hem arribat a les conclusions següents:
 Sector 1: representa el primer conjunt de dades, i inclou el 60 % del diagrama.
 Sector 2: format pel segon grup de dades, està representat per un angle de 90°.
 Sector 3: representa el tercer grup de dades.
Construeix-ne el diagrama i calcula la quantitat de dades que configura cada sector.
Si el 60 % correspon al primer conjunt de dades, atès que es disposa de 120 dades, en aquest grup es
tindrà: 0,6 · 120 = 72 dades.
Atès que el segon sector és de 90º correspon a una freqüència relativa de 0,25; per tant, al segon grup hi
ha: 0,25 · 120 = 30 dades.
Així, el tercer conjunt de dades es forma per 120 – 72 – 30 = 18 dades.

 
 
64. Una variable estadística té com a mitjana aritmètica m, i com a desviació típica, d. Investiga què
passa amb aquests dos paràmetres si:
a) Sumem 4 a tots els nombres.
b) Restem 4 a tots els nombres.
c) Multipliquem per 4 tots els nombres.
d) Dividim entre 4 tots els nombres.
(Posa’n una sèrie d’exemples i efectua els càlculs. Fes una hipòtesi general i mira de trobar la
demostració d’aquesta hipòtesi.)
a) Si sumem 4 a tots els valors, la mitjana és m + 4 i la desviació típica és d.
Per exemple, es considera la sèrie: 5 6 8 9
ଶ଼ ଶ଴଺
‫ݔ‬ҧ ൌ ൌ͹ ߪൌට െ ͹ଶ ൌ ඥʹǡͷ ൌ ͳǡͷͺ
ସ ସ

En sumar 4 a tots els nombres: 9 10 12 13


ସସ ସଽସ
‫ݔ‬ҧ ൌ ൌ ͳͳ ߪൌට െ ͳͳଶ ൌ ඥʹǡͷ ൌ ͳǡͷͺ
ସ ସ

780
Estadística unidimensional 1313

b) Si restem 4 a tots els valors, la mitjana és m – 4 i la desviació típica és d.


Tenint en consideració la sèrie anterior, si restem 4 a tots els valors: 1 2 4 5
�� ��
�� � �� ��� � �� � ��,� � 1,��
� �

c) Si multipliquem per 4 tots els valors, la mitjana és 4m i la desviació típica és 4d.


La nova sèrie és: 20 24 32 36
��� ����
�� � � �� ��� � ��� � √40 � 6,��
� �

� �
d) Si dividim per 4 tots els valors, la mitjana és m i la desviació típica és d.
� �

La nova sèrie és: 1,25 1,5 2 2,25


� ��,���
�� � � 1,�� ��� � 1,��� � √0,16 � 0,��
� �

 
65. Han fet un estudi del professorat de Batxillerat a escala nacional. Aquest estudi indica que entre els
docents menors de 40 anys hi ha més dones que homes; en concret, es troben en relació 11 a 10.
És a dir, per cada 11 dones que exerceixen la docència en aquesta etapa educativa, hi ha 10 homes
que també desenvolupen aquesta feina en el mateix nivell.
Si l’edat mitjana de les professores és de 34 anys i la dels professors és de 32 anys, quina és la
mitjana d’edat dels docents menors de 40 anys en Batxillerat?

������������� ����
�� � � � ��,0� anys 
������� ���
 

66. Les puntuacions mitjanes en un concurs dels nois i les noies separadament, i els nois i les noies de
manera conjunta, de dos centres A i B, sobre una puntuació màxima de 150 punts, són les que es
mostren a la taula següent:

Quina ha estat la mitjana de les noies dels dos centres alhora?


Si x és el nombre de nois del centre A i y és el nombre de noies:
�1� � �6�
� �4 � �� � �� � 0
���
De manera anàloga, si m és el nombre de nois del centre B i n el nombre de noies:
�1� � �0�
� �4 � �� � 6� � 0
���
I com que x és el nombre de nois del centre A i m és el nombre de nois del centre B
�1� � �1�
� �� � �� � �� � 0
���

781
Estadística unidimensional
Estadística unidimensional 13

3� � 2� � � 2 8 4
� � � 2� � � � � � � � � � � � � � �
3 3 3
4� � � � �
Així la mitjana de les noies dels dos centres és:

��� � �� � � 228� � 3��� 588�

� � � � 84
�� � 3� � 4� ��

 
67. La mitjana d’un conjunt de 12 dades és 6, i la mitjana d’un altre conjunt amb 13 dades és 5,5. Quina
seria la mitjana si unim totes les dades en un únic conjunt de 25 dades?
� � �2 � 5�5 � �3
�� � � 5��4
25

 
   

782
Estadística unidimensional 1313

MATEMÀTIQUES A LA TEVA VIDA


 
1. Quin ha estat l’objectiu principal de l’enquesta?
El principal objetiu de l’enquesta és caracteritzar les práctiques i els escenaris de consum de béns i serveis
culturals de la població d’un país.
2. Descriu la mostra que han adoptat per a l’estudi.
La mostra era formada per 8 275 persones més grans de 4 anys de 2 415 llars de 14 municipis del país.

3. Tenint en compte els resultats, et sembla que una empresa que decidís vendre llibres hauria de tenir
una orientació cap el públic femení?
Sí, es pot deduir que les dones de l’enquesta llegeixen més llibres que els homes.

4. Classifica les variables estadístiques que apareixen en el text.


«Nombre de llibres que llegeixen per any la població del país.»

«Nombre de llibres que llegeixen per any les dones del país.»

«Nombre de llibres que llegeixen per any els homes del país.»

5. Elabora una taula de freqüències utilitzant les dades que estan representades en el diagrama.

Nre. llibres  1  2‐4 5‐7 8‐10 11 o   Total 

Homes (%)  1,9 2,28 5,16 23,69 13,37 46,4 

Dones (%)  2,5 2,86 5,97 24,87 17,40 53,6 

Total (%)  4,40 5,14 11,13 48,56 30,77 100 

6. Calcula les freqüències només per a homes i només per a dones.

Nre. llibres  1  2‐4 5‐7 8‐10 11 o   Total 

Homes (%)  4,09 4,91 11,12 51,06 28,82 100 

Nre. llibres  1  2‐4 5‐7 8‐10 11 o   Total 

Dones (%)  4,66 5,34 11,14 46,4 32,46 100 

7. Determina la mitjana de la quantitat de llibres que han llegit les persones enquestades.
Encara que sigui una variable discreta, les dades vénen donades en intervals, així doncs hem de calcular
les marques de classe:
x1  1        x2  3        x3  6        x4  9      x5  11 
En l’últim interval considerem el valor més petit, atès que és l’únic valor segur:
1⋅ 4,4 + 3 ⋅ 5,14 + 6 ⋅ 11,13 + 9 ⋅ 48,56 + 11⋅ 30, 77
x= = 8,62  llibres 
100

 
783
Estadística bidimensional
Estadística bidimensional  14
14
10  

ACTIVITATS

1. Considera aquestes variables bidimensionals i escriu les variables unidimensionals corresponents i


tres parells de valors que les determinen.
a) Edat i sexe de les persones que assisteixen a un concert.
b) Mida d’un fitxer informàtic i temps que es triga a copiar-lo.
c) Pes i talla dels peus dels alumnes d’una classe.
a) X → Edat, en anys, dels assistents al concert.
Y → Sexe dels assistents.
(20, dona) (25, home) (28, dona)
b)  X → Mida, en kb, de l’arxiu informàtic. 
Y → Temps, en s, que es triga en copiar-lo.
(220, 35) (158, 24) (285, 42)
c) X → Pes, en quilos, dels alumnes d’una classe.
Y → Talla, en centímetres, dels alumnes d’una classe.
(61, 155) (76, 172) (56, 160)
 
2. En una classe amb 24 alumnes han efectuat un estudi sobre el nombre d’hores diàries d’estudi, X, i
el nombre de suspensos, Y, i han obtingut els resultats següents:
(2, 1) (0, 7) (1, 3) (3, 1) (3, 0) (1, 2) (1, 1) (2, 0)
(3, 0) (2, 7) (1, 0) (2, 1) (3, 1) (1, 4) (1, 2) (2, 1)
(2, 0) (3, 1) (2, 2) (1, 0) (1, 2) (2, 1) (0, 6) (2, 0)
Ordena aquestes dades en una taula de doble entrada.


0  1  2  3  Total 

0  0  2  3  2  7 

1  0  1  4  3  8 

2  0  3  1  0  4 

3  0  1  0  0  1 

4  0  1  0  0  1 

5  0  0  0  0  0 

6  1  0  0  0  1 

7  1  0  1  0  2 

Total  2  8  9  5  24 

785
Estadística bidimensional
Estadística bidimensional 14

3. Construeix una taula de doble entrada i les taules de freqüències marginals corresponents
d’aquestes dades:
(1, 2) (1, 3) (1, 1) (2, 2) (3, 1) (1, 1) (3, 3) (2, 1) (2, 1) (3, 2)
 


1  2  3  Total 

1  2  1  1  4 

2  1  2  1  4 

3  1  0  1  2 

Total  4  3  3  10 
 

X  1  2  3  Total  Y  1  2  3  Total 

fi  4  3  3  10  fi  4  4  2  10 


 
 
4. Amb la taula de doble entrada de l’activitat anterior, construeix les taules condicionades següents:
a) Y/X = 1 b) X/Y = 3

a)  b)
Y/X  1  1  2  3  Total  X/Y  3  1  2  3  Total 

fi  2  1  1  4  fi  1  0  1  2 
      
 
5. Construeix la taula de doble entrada i les taules marginals per a les dades següents, (X, Y ):
(16, 5) (17, 4) (18, 6) (16, 6) (14, 8)
(17, 3) (14, 5) (13, 4) (14, 8) (15, 8)
Representa el diagrama de dispersió de la variable estadística bidimensional anterior.

 
 
   

786
Estadística bidimensional 1414

Taula de freqüències Taula de freqüències


marginals de X marginals de Y

     
 
6. Elabora un diagrama de dispersió tenint en compte la freqüència de cada parella de dades.

                       
 
7. Representa’n el núvol de punts i analitza’n la dependència.

                       
El núvol de punts s’aproxima poc a una recta i quan la variable X creix, la variable Y decreix, així doncs,
existeix una dependència lineal dèbil i negativa entre les dues variables.

787
Estadística bidimensional
Estadística bidimensional 14

8. Dibuixa’n el núvol de punts i analitza’n la dependència.

                       
El núvol de punts s’aproxima poc a una recta i quan la variable X creix, la variable Y creix, podem dir que
existeix una dependència lineal dèbil i positiva entre les dues variables.

 
9. Tenir gos o gat, X, influeix per aprovar Matemàtiques, Y?


Si  No 

Aprovat     

Suspès     
 
Completem la taula amb els totals i analitzem si les files i les columnes són proporcionals entre sí:


Sí  No  Total 

Aprovat  10  15  25 

Suspès  30  45  75 

Total  40  60  100 

10 15 25 1 30 45 75 3 10 30 40 2 15 45 75 3
= = = = = = = = = = = =
40 60 100 4 40 60 100 4 25 75 100 5 25 75 100 4

Sí són proporcionals, així doncs tenir mascota no influeix per aprovar Matemàtiques.
 
10. Completa la taula següent a la llibreta perquè siguin independents:

788
Estadística bidimensional 1414

El nombre total de dades és 500. La taula és:


A  B  Total 

C  x11  x12  175 

D  x22  x22  325 

Total  200  300  500 

Les variables són independents si les files i les columnes són independents entre elles, així doncs:
ìï x11 175 ⋅ 200 ì
ï =
175
 x11 = = 70 ï x 21 = 325  x 21 = 325 ⋅ 200 = 130
ï
x11 x12 175 ïïï 200 500 500 x 21 x 22 325 ï
ï 200 500 500
= = í      = = ï
í  
200 300 500 ïï x12 175 175 ⋅ 300 200 300 500 ï ï x 22 325 325 ⋅ 300
ïï =  x12 = = 105 ï
ï =  x 22 = = 195
ïî 300 500 500 î 300 500
ï 500

Per tant, la taula restarà com es veu a continuació:


A  B  Total 

C  70  105  175 

D  130  195  325 

Total  200  300  500 


 
11. Determina la covariància d’aquesta variable:

 
 
N

åx ⋅y i i
1279
s XY = j =1
-x⋅y = - 169,31 = -9, 44  
N 8

 
12. Calcula la covariància de la variable següent:

 
N

åx ⋅y
j =1
i i
2 765
s XY = -x⋅y = - 386,30 = 8,70
N 7  
   

789
Estadística bidimensional
Estadística bidimensional 14

13. Representa el diagrama de dispersió i calcula el coeficient de correlació d’aquesta variable:

 
Quina relació pots descriure entre les variables?

 
Hiha una dependència lineal forta, perquè els valors, encara que no s’ajusten a una recta, es troben molt
pròxims; el coeficient de correlació és rXY = 0,82, que és proper a 1, i a més rXY > 0, per tant és positiva.

 
14. Aquesta taula mostra la renda per càpita en milers d’euros, X, i l’esperança de vida en anys, Y, en 9
països.

Indica quina relació pots descriure entre les variables.


x = 21,11 y = 69,11

sX = 15,40 sY = 8,37 sXY = 126,11

rXY = 0,98

El coeficient de correlació és rXY = 0,98, que és molt proper a 1 i determina una dependència lineal forta, i a
més rXY > 0, per tant és positiva.

 
15. Troba la recta de regressió de Y sobre X i representa-la juntament amb el diagrama de dispersió
d’aquesta taula:

 
x = 8,5 y = 25,5 s2X = 28,3 sXY = 85,3  
                
s XY 85,3
y-y = ( x - x )  y - 25,5 = ( x - 8,5)  y = 3,01x - 0,12
s2X 28,3

790
Estadística bidimensional 1414

                         
16. Representa el diagrama de dispersió i determina la recta de regressió de Y sobre X de la taula
següent:

 
x = 40,7 y = 174,5 2
s = 3,79 sXY = 0,59  
           X
    
s XY 0,59
y-y = ( x - x )  y - 174,5 = ( x - 40,7)  y = 3,62 x + 27,10  
s2X 3,79

                         
 
17. Determina les dues rectes de regressió i indica la relació que hi ha entre les variables següents:

�� ��
a) �� � � 12 �� � � �,2
� �
��� ���
��� � � 12� � 3,6�� ��� � � 12 � �,2 � �2,2
� �

�,�
Recta de regressió de Y sobre X: � � �,2 � � �� � 12� � � � ��,61� � 11,52
�,�

99
��� � � �,2� � 2,16
5
�,�
Recta de regressió de X sobre Y: x�12 � � �� � �,2� � � � �1,�2� � 16,2�
�,��

�� � √3,6 � 1,�9 �� � √2,16 � 1,��


�,�
��� � �
�,����,��
� ��,�9 � La dependència és dèbil i negativa.
��� ���
b) �� � � 12,5 �� � � 1�,5
�� ��

791
Estadística bidimensional
Estadística bidimensional 14

���� ����
��� � � 12�5� � ��25�� ��� � � 12�5 � 1��5 � ���5
�� ��

����
Recta de regressió de Y sobre X: � � 1��5 � � �� � 12�5� � � � 1�2�� � 1
����

2325
��� � � 1��5� � 22�25
10
����
Recta de regressió de X sobre Y: x�12�5 � �� � 1��5� � � � 0�35� � ���3
�����

�� � ���25 � 2�5 �� � �22�25 � ���2


����
��� � � � 0��� � La dependència és dèbil i positiva.
��������

 
18. La taula següent mostra el temps que han fet 6 atletes en una carrera i les hores d’entrenament que
hi han dedicat durant l’últim mes:

Temps  11,03  12,91 10,98 11,26 11,12  11,18

Hores 
175  35  201  150  190  100 
d’entrenament 

Determina les dues rectes de regressió i indica la relació que hi ha entre les variables.

x = 40,7 y = 174,5 s2X = 3,79 sXY = 0,59

ìï
ïï y - y = s XY ( x - x )  y - 141,83 = -
40,77
( x - 11,41)  y = 991,42 - 74,44 x
ïï s2X 0,55
í
ïï s 40,77
ïï x - x = XY ( y - y )  x - 11,41 = - ( y - 141,83)  x = 12,85 - 0,01y
ïïî sY2 4 030,17

Les variables són linealment dependents, amb una dependència dèbil i negativa.
 
19. En un estudi estadístic, el coeficient de correlació entre dues variables X i Y és –0,8. Sabem que � =
20, �� = 4, � = 8 i �� = 1.
a) Determina les dues rectes de regressió, representa-les i analitza la correlació que hi ha entre les
variables.
b) Si x = 30, quina és l’estimació de y?
���
a) �0�� � � ��� � �3�2
���
���
Recta de regressió de Y sobre X: � � � � � �� � 20� � � � �0�2� � 12
��
���
Recta de regressió de X sobre Y: � � 20 � � �� � �� � � � �3�2� � �5��

792
Estadística bidimensional 1414

 
 
La dependència és forta i negativa
y = –0,2 · 30 + 12 = 6

20. Aquesta taula mostra el pes, X, i l’alçada, Y, de 10 persones. Fes l’estimació del pes d’una altra
persona que fa 1,90 m.

 
x = 70,8 y = 1,70 sY2 = 0,001 s XY = 1,02

s XY 1,02
x-x = ( y - y )  x - 70,8 = - ( y - 1,70 )  x = 120 y - 132,6
sY2 0,001

Per y = 1,90 m l’estimació considerada per la recta serà:

x = 120 ⋅ 1,90 - 132,6 = 228 - 132,6 = 95,40 kg

   

793
Estadística bidimensional
Estadística bidimensional 14

SABER FER
 
21. Utilitza la calculadora per estudiar la correlació entre aquestes dues variables estadístiques:

 
Establim el Mode Regressió Lineal a la calculadora.

Introduïm les dades i la calculadora dóna la covariància de les variables i les seves desviacions típiques
marginals:

s XY = -1,16 sX = 3,136 sY = 9,247

Calculem la correlació:

-1,16
rXY = = -0,04
3,136 ⋅ 9,247

22. Una empresa vol fer un estudi estadístic sobre la despesa que fa en publicitat (X, en milers d’euros) i
les vendes de productes que llancen al mercat (Y, en milers d’unitats).
(20, 23) (17, 20) (11, 24) (24, 4) (23, 27) (3, 21) (15, 19) (21, 1) (19, 7) (12, 12) (19, 24) (7, 7) (21, 8) (12, 19)
(11, 25) (8, 13) (17, 14) (2, 21) (9, 2) (21, 11) (3, 14) (12, 18) (21, 1) (8, 12) (12, 12) (6, 29) (12, 19) (14, 21)
(3, 12) (15, 15)
Agrupa la despesa en publicitat i les vendes de productes en intervals amb la mateixa amplitud 10.
 


[2, 11]  [12, 21]  [22, 31]  Total 

[1, 10]  2  4  2 8
[11, 20]  4  10  0 14
[21, 30]  4  3  1 8
Total  10  17  3 30
 

   

794
Estadística bidimensional 1414

23. La taula que tens a continuació reflecteix un estudi estadístic en el qual s’analitza la relació entre les
variables X i Y.
Construeix les taules de freqüències marginals de cadascuna de les variables.

X  [0, 5)  [5, 10)  [10, 15)  [15, 20)  [20, 25)  Total 

fi  7  10  8  9  8  42 


 
Y  [0, 5)  [5, 10)  [10, 15)  [15, 20)  [20, 25)  [25, 30)  [30, 35)  [35, 40)  Total   

fi  4  3  8  5  7  3  6  6  42   
 
24. Els resultats d’un estudi sobre la relació de dues variables estadístiques X i Y són els següents:

a) Quantes dades hi ha amb X més petit que 16?


b) Quants valors de Y són més petits que 100?
c) Quantes dades hi ha amb X entre 12 i 18?
a) 104 b) 42 c) 152
   

795
Estadística bidimensional
Estadística bidimensional 14

25. Un pagès vol relacionar la quantitat d’aigua (en m3) i la collita de blat que ha obtingut (en tones). Per
fer-ho, ha apuntat les dades de les últimes cinc collites.
Determina’n la recta de regressió amb la calculadora.

 
Establim el Mode Regressió Lineal a la calculadora.
Introduïm les dades i la calculadora troba directament la mitjana i la desviació típica.
x = 8,6 y = 30,66 sXY =-30,82 sX = 3,28 sY = 9,66
                      
a = 3,11ü
ï
ï
ý  y = 3,11x + 3,88  
b = 3,88ï
ï
þ

 
26. Estudia la correlació entre aquestes variables mitjançant la calculadora.
Determina’n la recta de regressió i justifica si té sentit estimar el valor de Y si la variable X adopta el
valor 18.

 
Establim el Mode Regressió Lineal a la calculadora.
Introduïm les dades i la calculadora troba directament la mitjana i la desviació típica.
x = 14,4 y = 33,9
             

sXY = 0,16 sX = 1,58 sY = 2,23  rXY = 0,04


            

a = 0,06ïüï
ý  y = 0,06 x + 33  
b = 33 ïïþ

No té gaire sentit estimar el valor de Y per quan X pren el valor 18, atès que la correlació es molt propera a
0, per la qual cosa gairebé no existeix dependència.

 
27. La recta de regressió que relaciona dues variables estadístiques és y = 3,2x – 1,7. Si sabem que ࢟
ഥ=
2,34, quin és el valor de ࢞
ഥ?
x + 1,7
y = 3,2 x - 1,7  y = 3,2 x – 1,7  = x  x = 1,26
3,2

 
28. La recta de regressió que relaciona dues variables és y = 3,2x - 1,7. Si sabem que ࢞
ഥ = 2,87, podem
afirmar que, com més gran sigui X, més gran serà Y ?
La recta de regressió indica en aquest cas que existeix una dependència lineal creixent, perquè el pendent
de la recta de regressió és positiva.

Per tant, s’observa que si es trien valors grans per a X, el valor per a Y també creixerà.

796
Estadística bidimensional 1414

Per exemple:

y = 3,2 x - 1,7  y = 3,2 ⋅ 2,87 - 1,7 = 7,48

 
29. Dibuixa el núvol de punts i analitza’n la dependència.

El núvol de punts s’apropa força a una recta i quan la variable X creix, la variable Y també creix; per tant, hi
ha una dependència lineal forta i positiva entre les variables.

30. Calcula la recta de regressió i el valor esperat per a x = 4.

 
Calculem la recta de regressió de X sobre Y:

25 23
x= =5 y= = 4,6 sX 2 = 8 s X = 2,83 sY = 2,498 sXY = 6,8
5 5

6,8
rXY = = 0,96
2,83 ⋅ 2,498

L’equació de la recta de regressió és:

6,8
y - 4,6 = ( x - 5)  y = 0,85 x + 0,35
8

Per tant, el valor esperat per x = 4 serà:

y = 0,85 · 4 + 0,35 = 3,75

Com que rXY es troba molt a prop de 1, l’aproximació és força adequada.

797
Estadística bidimensional
Estadística bidimensional 14

ACTIVITATS
 
31. Representa el núvol de punts associat a aquestes variables bidimensionals i determina si hi ha
dependència entre les variables que les formen; si és així, indica de quin tipus és.
a) (2, 2) (3, 6) (5, 10) (6, 14) (8, 19) (9, 23) (10, 25)
b) (5, 2) (6, 0) (8, -2) (10, -7) (11, -9) (13, -13) (15, -17)
c) (120, 60) (122, 75) (126, 60) (128, 90) (130, 50) (132, 100) (136, 70)

 
Depend. lineal positiva forta Depend. lineal negativa forta. Depend. lineal positiva dèbil.

32. Representa el núvol de punts associat a aquestes variables bidimensionals i determina si hi ha


dependència entre les variables que les formen.
En cas afirmatiu, digues com és.

 
a) La dependència és forta c) No s’aprecia dependència
i positiva entre les variables E i F.

 
   

798
Estadística bidimensional 1414

b) La dependència és forta c) No s’aprecia dependència


i negativa entre les variables G i H.

 
 
33. Troba les freqüències marginals associades a aquestes taules de doble entrada i calcula les mesures
estadístiques de cada variable de manera separada:

 
 
a)
X  1  2  3  4  Total 

fi  8  8  8  11  35 


 

Y  1  2  3  4  Total 

fi  12  6  4  13  35 


92 88
x= = 2,629        y= = 2,514  
35 35

799
Estadística bidimensional
Estadística bidimensional 14

b)
X  [0, 2)  [2, 4)  [4, 6)  [6, 8)  Total 

xi  1  3  5  7   

fi  43  45  42  60  190 


 

Y  10  20  30  40  50  60  Total 

fi  36  24  26  48  27  29  190 

808 6630
x= = 4,253        y= = 34,895   
190 190

c)
X  º  2  3  4  5  6  7  8  9  10  Total 

fi  5  14  14  17  27  18  21  17  19  13  165 
 

Y  [0, 10)  [10, 20)  [20, 30)  [30, 40)  Total 

yi  5  15  25  35   

fi  39  47  44  46  165 

970       3 335    
x= = 5,878   y= = 20,212
165 165
 
34. Les notes de Llengua i d’Anglès dels 30 alumnes d’una classe en l’última avaluació han estat les
següents:
Llengua: 37875 25954 35363
85776 24949 76717
Anglès: 2 6 10 6 4 25955 24151
10 4 7 8 4 25959 85707
a) Construeix la taula de doble entrada amb aquestes dades.
b) Dibuixa’n el diagrama de dispersió.
a)
Notes 
0  1  2  3  4  5  6  7  8  9  10  Total 
Assignatura 

Llengua  0  1  2  4  3  5  3  7  2  3  0  30 
Anglès  1  2  4  0  4  7  2  3  2  3  2  30 

Total  1  3  6  4  7  12  5  10  4  6  2  60 


   

800
Estadística bidimensional 1414

b) Nota 

  Llengua  Anglès   
 
35. Construeix la taula de doble entrada corresponent a partir del diagrama de dispersió, tenint en
compte la freqüència de les dades que figuren entre parèntesis.

 
 

 
 
36. A partir d’aquests diagrames de dispersió, construeix les taules de doble entrada corresponents:

801
Estadística bidimensional
Estadística bidimensional 14

a)

1  2  3  4  5  6  7  Total 

1  1  1  0  1  0  0  0  3 

2  0  1  0  1  0  0  0  2 

3  0  0  1  1  0  0  0  2 

4  0  0  0  0  1  0  0  1 

5  0  0  0  1  0  0  0  1 

6  0  0  0  1  1  0  0  2 

7  0  1  0  1  1  0  0  3 

8  0  0  0  0  0  1  0  1 

9  0  0  0  0  0  1  1  2 

10  0  0  0  0  0  1  0  1 

11  0  0  0  0  0  0  1  1 

Total  1  3  1  6  3  3  2  19 
 
b)


1  2  3  4  5  6  7  8  Total 

3  0  0  0  0  0  1  0  1  2 

5  0  0  0  0  1  0  1  0  2 

6  0  1  0  0  1  0  0  0  2 

7  0  0  1  0  1  0  0  1  3 

8  0  0  1  1  0  0  0  0  2 

9  0  0  1  0  0  1  0  0  2 

10  1  1  0  0  0  0  0  0  2 

11  0  1  0  0  0  0  0  0  1 

Total  1  3  3  1  3  2  1  2  16 
 

37. A partir dels diagrames de dispersió, determina si hi ha o no dependència lineal entre les variables i,
en cas afirmatiu, si és forta o dèbil i si és positiva o negativa.

802
Estadística bidimensional 1414

 
a) No hi ha dependència lineal.

b) La dependència lineal és forta i negativa.

c) La dependència lineal és dèbil i positiva.

d) La dependència lineal és forta i positiva.

 
38. Volen fer un estudi per determinar la influència de la velocitat del vent en el fet que una plaga afecti
durant un temps concret un tipus de planta. Per fer-ho, han mesurat la velocitat en metres per segon,
X, i el temps en hores, Y. Aquesta taula mostra els valors ordenats de manera ascendent respecte
del valor de X.

Construeix-ne el diagrama de dispersió i indica si hi ha dependència entre les variables; en cas


afirmatiu, assenyala de quin tipus és.
 

 
El núvol de punts no s’apropa gaire a una recta i quan la variable X creix, la variable Y creix, per tant hi ha
una dependència lineal dèbil i positiva entre les variables.

803
Estadística bidimensional
Estadística bidimensional 14

39. La taula mostra la quantitat de quadres que han pintat els alumnes d’un taller sobre paisatges i
natures mortes.

 
a) Determina les taules de freqüències marginals de paisatges i natures mortes.
b) Calcula les mitjanes i les desviacions típiques de cadascuna de les variables.
c) Dibuixa el diagrama de dispersió corresponent a la variable bidimensional.
d) A partir del diagrama de dispersió, indica si hi ha una dependència entre les variables.
a) Taula de freqüències marginals Taula de freqüències marginals
dels paisatges de les natures mortes.

 
b) ‫ݔ‬ҧ = 5,47 ‫ݕ‬ത = 5,82
ߪ௫ = 1,15 ߪ௬ = 1,19
c)

 
d) Observant el diagrama de dispersió, podem deduir que hi ha una dependència lineal dèbil positiva entre
les variables.

804
Estadística bidimensional 1414

40. La taula següent reflecteix els ingressos familiars mensuals d’una família en cents d’euros, X, i els
metres quadrats de l’habitatge familiar, Y.

a) Indica les distribucions marginals de les variables.


b) Determina les mesures i les desviacions típiques de cadascuna de les variables.
c) Dibuixa el diagrama de dispersió que correspon a la variable bidimensional.
d) A partir del diagrama de dispersió, determina si hi ha una dependència entre les variables.
a)
X  [0, 50) [50, 100)  [100, 200) [200, 250) [250, 300)  Total 
fi  41  98  58 48 20 265 
 

Y  [0, 5)  [5, 10)  [10, 15) [15, 20) [20, 25)  Total 


fi  50  74  64 55 22 265 
 
41⋅ 25 + 98 ⋅ 75 + 58 ⋅ 150 + 48 ⋅ 225 + 20 ⋅ 275 33375
b)  x = = = 125,94  
265 265
50 ⋅ 2,5 + 74 ⋅ 7,5 + 64 ⋅ 12,5 + 55 ⋅ 17,5 + 22 ⋅ 2,25 2 937,5
y= = = 11,08  
265 265
N

åf ⋅ x i
2
i
5 819506
sX = i =1
- x2 = - 125,94 = 21 960,4 - 125,94 = 21834,46  
N 265
N

åf ⋅ y i
2
i
2 42456,25
sY = i =1
-y = - 11,08 = 160,21- 11,08 = 149,13  
N 265

c)

    
d) El núvol de punts es dispersa i els punts no es troben massa junts. HI ha una dependència lineal dèbil i
creixent.

805
Estadística bidimensional
Estadística bidimensional 14

41. Calcula la covariància i el coeficient per a les variables bidimensionals de les taules següents:

 
 
 
42. Determina la covariància i el coeficient de correlació corresponents a aquestes variables
estadístiques:

 
 
 
 
43. Una companyia d’assegurances vol relacionar el nombre de vehicles que circulen per una autopista
determinada a més de 115 km/h, X, i el nombre d’accidents que tenen lloc en aquesta carretera, Y.
Durant una setmana han obtingut els resultats següents:

Nre. vehicles  150 180  100 80 200 170 400 


Nre. accidents  5  7  2 1 9 6 15 
 
a) Determina les mitjanes i les desviacions típiques de les variables nombre de vehicles i nombre
d’accidents.
b) Calcula la covariància de la variable bidimensional.
c) Troba el coeficient de correlació i interpreta’n el valor.
ì
ï x = 182,85 ìï y = 6,43
a) ïí ïí
ïs X = 105
ï ïïîsY = 4,69
î

b) sXY = 480,24

c) rXY = 0,98 és gairebé 1. Hi ha una dependència lineal forta i, com que és més gran que 0, positiva.

806
Estadística bidimensional 1414

44. Han efectuat una enquesta a 30 treballadors d’una empresa. Entre les dades recopilades consta el
salari anual, en milers d’euros, i els anys d’educació.
Amb el diagrama de dispersió corresponent obtenen aquest núvol de punts:

Analitza com ha de ser el signe de la covariància i el del coeficient de correlació. Justifica les
respostes.
Tant la covariància com el coeficient de correlació han de ser positius, perquè el núvol de punts és creixent
quan creixen les variables.

 
45. Han agafat 6 mostres de sèrum i han apuntat en una taula el temps que feia que estaven preparades i
el nombre de bacteris que hi han trobat en un mil·lilitre.

 
a) Determina el valor de les mitjanes i les desviacions típiques de les variables, el nombre d’hores i
de bacteris.
b) Calcula la covariància de la variable bidimensional.
c) Calcula el coeficient de correlació i interpreta’n el valor.
ì
ï x = 1,83 ì
ï y = 33,83
a) ïí ï
í b) sXY = 18,77 c) rXY = 0,97
ï ï
îs X = 0,83
ï î Y = 19,71
ïs

 
46. En una classe porten a terme una enquesta sobre la quantitat de treballs que han fet i el grau de
satisfacció, de l’1 al 5, amb el professor de l’assignatura.

807
Estadística bidimensional
Estadística bidimensional 14

Analitza com ha de ser el signe de la covariància i el del coeficient de correlació. Justifica les
respostes.
El signe de la covariància ha de ser positiu, i el coeficient de correlació és positiu i proper a 0, perquè creix
la variable "satisfacció" quan creix el treball dut a terme, però és proper a 0 perquè les dades es troben molt
disperses.

 
47. Relaciona aquests diagrames de dispersió amb el coeficient de correlació corresponent:
a) 0 b) 0,3 c) -1 d) 1 e) 0 f) -0,6
 
c) perquè el pendent és negatiu i els punts es troben sobre la recta de regressió.

d) perquè el pendent és positiu i els punts es troben sobre la recta de regressió.

f) perquè el pendent és negatiu i els punts no es troben sobre la recta de regressió.

b) perquè el pendent és positiu i els punts no es troben sobre la recta de regressió.

a) perquè el pendent és 0.

e) perquè el pendent és 0.
 
 

808
Estadística bidimensional 1414

48. Traça a mà alçada, i sense fer càlculs, la recta de regressió de les variables bidimensionals
següents:
 

 
 
a) b) c) d)

                 
 
49. Determina la recta de regressió de Y sobre X i la recta de regressió de X sobre Y corresponents a
aquestes taules:

 
ì
ï x = 4,2 y = -3,2
a) ïí 2  s XY = -10,7
ï
ïs = 7,7 s 2
Y = 133,7
î X

Recta de regressió de Y sobre X: Recta de regressió de X sobre Y:


10,7 10,7
y + 3,2 = - ( x - 4,2)  y = -1,39 x + 2,64 x - 4,2 = - ( y + 3,2)  x = -0,08 y + 3,94
7,7 133,7

ì
ï x = 10 y = 23,93
b) íï 2  s XY = 6,08
ï 2
îs X = 4,08 sY = 9,20
ï

Recta de regressió de Y sobre X: Recta de regressió de X sobre Y:


6,08 6,08
y - 23,93 = ( x - 10 )  y = 1,49 x + 9,03 x - 10 = ( y - 23,93)  x = 0,66 y - 5,82
4,08 9,20

   

809
Estadística bidimensional
Estadística bidimensional 14

50. Troba cinc punts de la recta y = 4x + 6.


a) Calcula’n el coeficient de correlació corresponent i explica el resultat.
b) Determina’n les dues rectes de regressió.
Resposta oberta.

a) �� � � �� � � �� � √2 � 1�41 �� � √32 � ���� ��� � 8


��� � 1 � La dependència és lineal.
b) Recta de regresió de Y sobre X:
8
� � � � �� � �� � � � 4� � �
2
Recta de regresió de X sobre Y:
8 1 3
���� �� � �� � � � � �
32 4 2
 
51. Si sabem que � � �, ��� � �,���� � � i que la recta de regressió de Y sobre X és y = 4 – 0,68x, troba
la recta de regressió de X sobre Y.
Per calcular la recta de regressió de X sobre Y es necessiten també y i s XY , que s’obtenen de la recta de
regressió de Y sobre X i de les dades de l’enunciat.
s XY s
-0,68 =  -0,68 = XY  s XY = 6·(-0,68) = - 4,08
sX2 6

s XY s
-4 = x - y  y = XY x + 4 = -0,68·3 + 4 = 1,96
sX2 sX2

4,08
La recta de regressió de X sobre Y és: x - 3 = - ( y - 1,96)  x = 4 - 0,51y
8

 
52. Si sabem que x = 3,2, y = 1,2 i la recta de regressió passa pel punt (3,9, 3,8), determina la recta de
regressió de Y sobre X.
s XY
y-y =
s2X
( x - x )  y - 1,2 = ssXY2 ( x - 3,2)  y = ssXY2 x - (3,2) ssXY2 + 1,2
X X X

s XY
Atès que passa pel punt (3,9; 3,8), se substitueixen i s’obté el valor de :
s2X

s XY s s s
3,8 = (3,9) - (3,2) XY + 1,2  2,6 = (0,7) XY  XY = 3,71
s2X s2X s2X s2X

Així doncs la recta de regressió serà: y = 3,71x - 10,69


   

810
Estadística bidimensional 1414

53. Han fet un estudi sobre la relació entre la quantitat de mesos que una persona està apuntada a un
centre esportiu i el nivell de satisfacció amb les instal·lacions del centre, mesurat de 0 a 10. Per fer-
lo, han escollit 10 persones a l’atzar i han apuntat els resultats següents:

 
a) Calcula el valor del coeficient de correlació.
b) Determina’n la recta de regressió.
ìs XY = 6,69
ï
ï
ï s 6,69 ìï x = 7,8 s
a) ïís X = 3,39  rXY = XY = = 0,95 b) ïí 2 (
 y - y = XY x - x )  y = 0,58 x + 1,57
ï
ï s X sY 3,39 ⋅ 2,08 ïï y = 6,1 sX
ïs ïî
î Y = 2,08
ï

 
54. Una companyia discogràfica ha recopilat informació sobre 42 dels grups musicals que porta, el
nombre de concerts i les vendes de discos, en milers, i han obtingut la taula següent:

   
 
Determina’n la recta de regressió i explica si hi ha dependència lineal o no n’hi ha.
ì ì 2X = 248,59
ïï x = 35,71  ïïs  s XY = -436,55  
í í 2
ïï y = 13,45 ï îsY = 56,24
ï
ï
î

436,55
La recta de regressió és: y - 13,45 = - ( x - 35,71)  y = 76,16 - 1,76 x
248,59

Hi ha dependència lineal negativa, perquè el pendent és diferent de 0.


 
55. Una confraria de pescadors enregistra la quantitat de sardines que arriben al port, en quilograms, i el
preu de la subhasta a la llotja, en euros per quilo, i han obtingut els resultats següents:

 
a) Determina’n la correlació lineal i interpreta’n el valor.
b) Escriu l’equació de la recta de regressió.
c) Fes una estimació de quin seria el preu de les sardines si un dia se’n pesquessin 1 300 kg.

811
Estadística bidimensional
Estadística bidimensional 14

a) rXY = –0,98

Existeix dependència lineal negativa, i és propera a – 1 perquè els seus valors són molt propers a la
recta de regressió.

b) La recta de regressió és y = 2,96 - 0,01x .

c) L’estimació s’obté en calcular en la recta de regressió el valor de y quan s’introdueix el valor desitjat de x,
és a dir, x = 1 300: y = 2,96 - 0,001⋅ 1300 = 2,96 - 1,3 = 1,66

56. Un club de bàsquet ha publicat l’estadística següent referida al seu primer equip:
 Nre. de jugadors: 10
 Estatures: x = 1,96 cm, ોx = 7,06
 Pesos: y = 90,1 kg, ોy = 6,92
 Covariància: ોxy = 36,7
a) Troba’n el coeficient de correlació.
b) Calcula’n la recta de regressió de Y sobre X.
c) Si l’equip fitxa un jugador que fa 205 cm, quin pes esperaríem que fes?
Justifica la resposta.
s XY 36,7
a) rXY = = = 0,75
s X sY 7,06·6,92

36,7
b) y - 90,1 = ( x - 1,96)  y = 0,73 x + 88,66
49,84

c) L’estimació s’obté calculant en la recta de regressió el valor de y quan s’introdueix com a valor de
x = 2,05: y = 0,73 ⋅ (2,05) + 88,66 = 90,15

57. La Mercè i en Dídac viuen al mateix carrer, però en voreres oposades. Tots dos tenen un termòmetre
al balcó i, com que la Mercè es pensa que el seu està espatllat, decideixen controlar la temperatura
exterior, en °C, durant una setmana i a la mateixa hora del dia.
Han apuntat els resultats en una taula.

 
a) Indica si les dues variables estan relacionades. Et sembla que ho haurien d’estar?
b) Justifica si amb aquestes dades es pot obtenir alguna conclusió sobre el termòmetre de la Mercè.
a) ‫ݔ‬ҧ ൌ ʹʹǡͶ͵ ‫ݕ‬ത ൌ ͳͺǡͷ͹ ߪ௫ ൌ ʹǡͺ͸ ߪ௬ ൌ ͳǡ͸ͻ ߪ௫௬ ൌ െʹǡͲͻ͹
‫ݎ‬௫௬ ൌ െͲǡͶ͵ ՜ La dependència és dèbil i negativa.

Les dues variables estan poc relacionades, atès que els termòmetres es troben en costats oposats de la
vorera reben diferents exposicions solars.

b) Atès que la dependència és dèbil, no es pot concloure res sobre el termòmetre de la Mercè.

812
Estadística bidimensional 1414

58. Tenim dues variables bidimensionals representades pels núvols de punts següents:
I) II)

 
a) Escull-ne els coeficients de correlació i justifica-ho.
–0,92 0,95 0,6 –0,65
b) Determina quines són les equacions de les dues rectes de regressió corresponents. Justifica la
resposta.
y = 3x + 0,2 y = –0,6x + 10
y = 1,3x + 0,9 y = –2x + 12,6
a) El coeficient de correlació de les variables representades a la gràfica I és 0,95; perquè el núvol de punts
mostra una dependència entre les variables forta i positiva. El coeficient de correlació de les variables
representades a la gràfica II és –0,65; atès que la dependència entre les variables és dèbil i negativa.
b) La recta de regressió de la gràfica I és y = 1,3x + 0,9; ja que el pendent de la recta representada és un
valor proper a 1. La recta de la regressió de la gràfica II és y = –0,6x + 10, atès que el valor de l’ordenada de
la recta representada és 10.
 
59. Tenim la variable bidimensional següent:

 
Investiga el que passa amb la covariància i el coeficient de correlació en cada cas.
a) Sumem 10 a tots els valors de la variable X.
b) Sumem 10 a tots els valors de X i de Y.
c) Multipliquem per 4 tots els valors de la variable X.
d) Multipliquem per 4 tots els valors de X i de Y.

813
Estadística bidimensional
Estadística bidimensional 14

 
 
60. En una empresa de missatgeria han anotat el termini d’entrega, en hores, que anunciava en les
trameses i el termini real d’entrega, també en hores, i han obtingut la taula següent:

 
a) Representa les dades en un núvol de punts.
b) Dibuixa’n aproximadament la recta de regressió.
c) Calcula’n el coeficient de correlació.
d) Troba’n la recta de regressió, dibuixa-la i compara-la amb el valor obtingut en l’aproximació.
Termini  Termini 
a) i b) real (h)  real (h)

Termini anunciat (h) Termini anunciat (h) 
    
 
s XY
c)  rXY = = 0,99
s X sY

d)  x = 16,5      y = 11,75     s X 2 = 177,75     sXY = 102,75   

102,75
y - 11,75 = ( x - 16,5)  y = 0,58 x + 2,18   
177,75

   

814
Estadística bidimensional 1414

61. Volen estudiar la repercussió que té la pluja en la quantitat de visites que es fan a un parc
d’atraccions. Per portar-ho a terme, observen, en els últims deu anys, la quantitat de dies de pluja
que hi ha hagut durant la temporada en què el parc és obert, X, i la quantitat de visites durant la
temporada, Y.

 
a) Determina’n el coeficient de correlació i interpreta’n el valor obtingut.
b) Troba la recta de regressió que explica la quantitat de visitants d’acord amb els dies de pluja.
c) Quina és la previsió de visites per a l’any vinent si hi ha una predicció que serà un any poc plujós
amb només 12 dies de pluja en la temporada d’obertura?
d) Determina la recta de regressió de X sobre Y.
e) Representa’n el núvol de punts i les dues rectes de regressió en els mateixos eixos.
s XY
a) rXY = = -0,94 → Hi ha una correlació lineal forta, i el signe negatiu implica que quan creix la variable
s X sY
X, la Y decreix.
22,82
b) y - 33,9 = - ( x - 17,6)  y = 54,87 - 1,19 x
19,16

c) Amb dotze dies de pluja la previsió seguirà la fórmula y = 54,87 - 1,19 ⋅ 12 = 40,59 .
d) x = 42,93 - 0,75y
e)

 
62. La taula següent mostra les dades obtingudes després de preguntar pels ingressos familiars
mensuals, X, en euros, i els metres quadrats dels habitatges, Y.
 

815
Estadística bidimensional
Estadística bidimensional 14

a) Calcula’n les distribucions marginals.


b) Determina’n la covariància, el coeficient de correlació i les dues rectes de regressió.
a) 
X  [0, 500)  [500, 1 000) [1 000, 1 500) [1 500, 2 000) [2 000, 2 500)  Total
Freqüències  41  98 58 48 20  265
Dist. Marginal  0,15  0,37 0,22 0,18 0,08  1

Y  [30, 50)  [50, 70) [70, 90) [90, 110) [110, 130)  Total


Freqüències  50 74 64 55 22  265
Dist. Marginal  0,19 0,28 0,24 0,21 0,08  1

b) sXY = 25 000 rXY = 1

ì
ï y = 30 + 0,04 x
Rectes de regressió:  ïí
ï
î x = 25 y - 750
ï

63. Una marca de bicicletes de competició ofereix la comparativa següent entre el preu i el pes de les
seves bicicletes:

Pes (Kg)  6,45 6,50 6,70 7,35 7,15  7,25


Preu  6.000  4.700 4.200 3.300 2.700  2.600

Determina’n la covariància i el coeficient de correlació.


Com és la relació entre les dues variables?
X → Pes en quilograms de les bicicletes
Y → Preu en euros de les bicicletes
sXY =- 470 rXY =- 0,90

Hi ha una relació de dependència lineal forta i negativa entre les dues variables, perquè quan la variable X
creix, la Y decreix.

64. Les notes en idiomes, francès i anglès, de 10 estudiants escollits a l’atzar d’un centre escolar han
estat les següents:

a) Representa el núvol de punts corresponent a aquesta distribució. Quina hipòtesi podem fer
considerant la representació?
b)Troba els paràmetres de la recta de regressió Y/X i la de X/Y i interpreta els coeficients calculats.
c) Representa’n les dues rectes de regressió juntament amb el núvol de punts.

816
Estadística bidimensional 1414

d) Per a un alumne que hagi tret un 4 en anglès, quin pronòstic de nota té per a francès?
e) Per a un alumne que ha tret un 9 en francès, quin pronòstic de nota té per a anglès?
a) Notes 
francès 

Notes anglès 

Hipòtesi: la nota d’anglès va a ser semblant a la de francès.

ì
ï y = 0,71 + 0,84 x
b) Rectes de regressió: ïí
ï
î x = 0,79 y + 1,77
ï

Com que els pendents de les rectes són molt semblants, això implica que les dades són molt properes a les
rectes de regressió.

c)     Notes 
francès 

Notes anglès 
 
d) Si un alumne ha obtingut un 4 en anglès, s’espera que en francès obtingui un y = 0,71 + 0,84 ⋅ 4 = 4,07 .
e) Si un alumne ha obtingut una 9 en francès, s’espera que en anglès obtingui un x = 0,79 ⋅ 9 + 1,77 = 8,88 .
 
65. Un inversor de borsa vol predir l’evolució que tindrà l’índex de la borsa.
Ha arribat a la conclusió que el comportament de l’índex un dia és el mateix que ha tingut el dia
anterior la cotització de l’empresa JJ&TO.
Investiga si això és correcte a partir de les cotitzacions d’aquesta empresa durant una setmana i els
valors que ha aconseguit l’índex l’endemà.

 
 

817
Estadística bidimensional
Estadística bidimensional 14

a) Quina cotització tindrà JJ&TO el dia anterior en què l’índex de la borsa arribi als 14 000 punts?
b) Si un dia JJ&TO té una cotització de 24 €, quin valor podem esperar que aconsegueixi l’índex
l’endemà?
�� � 20,2� �� � ��73�,29

�� � √2,77 � �,66 �� � √366 809,62 � 60�,6�

��� � 977,26

��� � 0,97 �La dependència és forta i positiva.

a) Recta de regressió de X sobre Y:


977,26
� � 20,2� � �� � ��73�,29� � � � 0,0027� � ��,��
366809,62

� � ��000 � � � 0,0027 � ��000 � ��,�� � 26,36

b) Recta de regressió de Y sobre X:


977,26
� � ��73�,29 � �� � 20,2�� � � � 3�2,8� � ��93,62
2,77
� � 2� � � � 3�2,8 � 2� � ��93,62 � 3873,�8
 

66. Volen estudiar fer pagar un impost a les empreses químiques que sigui proporcional a les emissions
de sofre que aboquen a l’atmosfera. Han experimentat diversos procediments per mesurar aquestes
emissions, però no n’han trobat cap de fiable. Al final, han decidit investigar algun mètode indirecte.
Es pensa que l’emissió de sofre pot estar relacionada amb el consum elèctric, amb el consum
d’aigua o amb el volum de les xemeneies de les fabriques. Per valorar-ho, han fet un estudi en un
medi controlat. En podem veure els resultats en aquesta taula:

818
Estadística bidimensional 1414

Quina de les mesures estadístiques es relaciona mes evidentment amb les emissions de sofre?
Justifica-ho.
El volum de les ximeneies és la variable que més es relaciona amb la quantitat d’emisions de sofre.

 
 
67. Han fet un test de memòria, X, i un test d’atenció, Y, a diversos alumnes, amb els resultats següents:

 
a) Calcula’n la covariància i el coeficient de correlació.
b) Determina’n les dues rectes de regressió.
c) Si és factible, fes una estimació sobre la puntuació que obtindrà un alumne que ha tret 33 en
atenció.
d) Si és factible, fes una estimació sobre la puntuació que obtindrà un alumne que ha aconseguit 27
en memòria.

 
   

819
Estadística bidimensional
Estadística bidimensional 14

b) Recta de regressió de Y sobre X: 

 
Recta de regressió de X sobre Y: 

 
 
68. Per a una distribució bidimensional coneixem les dades següents:
r = 0,7 �x = 1,2 �=4
Recta de regressió X sobre Y: x = 0,44y + 0,6
Calcula’n aquests valors:
a) La mitjana de X. c) La variància de X.
b) La recta de regressió de Y/X. d) La covariància de X i Y.
a) x = 0,44 y + 0,6 = 0,44 ⋅ 4 + 0,6 = 2,36

s XY
b) De la recta de regressió obtenim que 0,7 = .
sY2

s XY
Pel coeficient de correlació tenim que rXY = = 0,44 .
s X sY

Quan trobem sY es pot construir la recta de regressió:

s XY üï
= 0,44ïï
s X sY ïï s XY = 0,44 ⋅ s X sY üï
ý ïý  0,44 ⋅ s s = 0,7 ⋅ s2  s = 0,44 ⋅ s X  s = 0,75
s XY ï ï X Y Y Y Y
ï s XY = 0,7⋅ sY2 ï
þ 0,7
0,7 = 2 ïï
sY ïïþ

Per tant, la covariància serà sXY = 0,25 i la recta de regressió és y = 0,17 x + 3,59 .

c) La variància de X és s2X = 1,44 .

d) La covariància de X i Y és s XY = 0,25 .

69. Determina el coeficient de correlació de la variable bidimensional que te com a rectes de regressió:
Recta de Y sobre X: 2x – y – 1 = 0
Recta de X sobre Y: 9x – 4y – 9 = 0
a) Troba la mitjana aritmètica de cadascuna de les variables.
b) Podries calcular la desviació típica de Y si sabem que la de la variable X és √�?

820
Estadística bidimensional 1414

a) Les mitjanes es calculen fàcilment al solucionar el sistema d’equacions de l’enunciat:

2 x – y – 1 = 0üï
ï 8 x – 4 y – 4 = 0ü
ï - x + 5 = 0üïï ìï
ï ïx = 5
ý ý ýí
9 x – 4 y – 9 = 0þ ï
ï 9 x – 4 y – 9 = 0ïï
þ y = 2 x – 1 ïþ ï
ï ïî y = 9

b) De les dues rectes s’obté:

s XY üï
= 2 ïï
sX2
ïï s XY s s XY s XY s 2 2 2
ý ⋅ XY = ⋅ = XY = 2 ⋅ = = rXY
s XY 4 ïï s2X sY2 s2X sY2 s X sY 3 3
= ï
sY2 9 ïïïþ

Solucionant el sistema anterior, s’obté la desviació típica de Y:

s XY ü
ï
= 2 ïï s XY = 8 ü ï
4 ïï ï
ïý  s2 = 36  s = 6
ý 9
s XY 4 ïï s = s 2

ï Y Y
= ï 4 XY
ï
þ
sY2
9 ïþï

70. En els Jocs Olímpics de Londres 2012, els Estats Units van guanyar la final de basquet. Les
estadístiques dels jugadors nord-americans van ser les següents:

a) Representa el núvol de punts de la variable punts marcats (PT) enfront de l’alçada dels jugadors
(AL).
b) Calcula la recta de regressió de la variable alçada dels jugadors (AL) enfront dels punts marcats
(PT).
c) És raonable pensar que l’alçada explica els punts marcats? Justifica la resposta.
d) Representa el núvol de punts de la variable punts marcats (PT) enfront dels minuts jugats (MJ).
e) Calcula la recta de regressió de la variable minuts jugats (MJ) enfront dels punts marcats (PT).
f ) És raonable pensar que el nombre de minuts explica els punts marcats? Justifica la resposta.

821
Estadística bidimensional
Estadística bidimensional 14

a) Punts 
(PT) 

Alçada (AL) 
   

b) La recta de regressió serà y = 36,89 x - 65,44 .

c) La recta de regressió dona una aproximació lineal dels punts i d’un jugador d’altura x en un partit. Té
pendent positiva, així doncs, serà creixent i ens permet deduir que quan més gran és l’altura, més punts es
marquen.

d)
Punts 
  (PT) 

 
 
 
 
 
  Minuts jugats (MJ) 

e) La recta de regressió serà y = 0,61x - 1,81 .

f) La recta de regressió reprèn una aproximació lineal dels punts y d’un jugador que juga x minuts en un
partit. És una recta amb pendent positiva, de manera que serà sempre creixent i permet comprovar que un
jugador, quant més jugui, més punts marcarà per l’equip.

   

822
Estadística bidimensional 1414

AMPLIA
 
71. Escull la resposta adequada.

□ Siguin C i D el nombre total d’estudiants classificats i desclassificats, respectivament. Considerem les


següents definicions:

C=
åC i
= 83 D=
å D = 55 i

C D

Volem calcular el percentatge de classificats, és a dir, el coeficient, A, pel qual s’ha de multiplicar el nombre
total d’estudiants per aconseguir els que s’han classificat:
1- A
C = A(C + D )  C - AC = AD  D = C
A

Sigui x la mitjana de les puntuacions de tots els estudiants. Aleshores:

x=
åC + åD
i i
=
åC i
+
åDi
=
åC i
+
å Di = A ⋅ å Ci + (1- A) ⋅ å Di = AC + (1- A)D 
C+D C+D C+D 1- A A C D
C+ C D+D
A 1- A

21
76 = 83 A + (1 - A)55  28 A = 21  A = = 0,75 = 75 %
28

□ Siguin m i h el nombre total de dones i homes, respectivament. Considerem les següents definicions:

m=
å m = 34 i
h=
å h = 32
i

m h

A més tenim que 11 = m .


10 h

Sigui x la mitjana d’edat de la población total. Aleshores:

x=
åm + åh i i
=
åm i
+
åh i
=
åm i
+
å hi = 11⋅ å mi + 10 ⋅ å hi = 11 ⋅ m + 10 ⋅ h =
m+h m+h m+h 10 11 21m 21h 21 21
m+ m h+h
11 10

11⋅ 34 + 10 ⋅ 32 694
= =
21 21

823
Estadística bidimensional
Estadística bidimensional 14

□ Siguin B i E el nombre d’alumnes presentats a l’examen de Batxillerat i ESO, respectivament, i P la


puntuació que han aconseguit tots els alumnes de l’ESO. Considerem les definicions següents:

B=
å B = 83
i
E=
åE i
=
E⋅P
=P
B E E

A més tenim que 10 % de (B + E ) = E  10 ⋅ ( B + E ) = E  B = 9 E .


100

x=
åB + åE i i
=
åBi
+
åE i
=
åB i
+
åE i
=
9 ⋅ å Bi
+
åh i
=
9⋅ B + E

B+E B+E B+E 1 9E + E 10 B 10 E 10
B+ B
9

9 ⋅ 83 + P
 84 =  84 ⋅ 10 - 9 ⋅ 83 = P  P = 93 és la puntuació dels alumnes d’ESO.
10

72. Troba la relació que hi ha entre el coeficient de correlació lineal d’una distribució bidimensional i els
pendents de les seves rectes de regressió.
Els pendents de les rectes de regressió són:

Aleshores, tenim que:

 
 
73. Determina si és possible que la recta de regressió de X sobre Y i la recta de regressió de Y sobre X
siguin paral·leles. I perpendiculars?
No és possible que siguin paral·leles, atès que tenen sempre un punt comú ( ሺ‫ݔ‬ҧ ǡ ‫ݕ‬തሻ
Són perpendiculars si la correlació és nul·la.

74. En dos estudis estadístics que han portat a terme sobre les dades d’una variable bidimensional, les
rectes de regressió han estat les següents:
En el primer estudi, la recta de regressió de Y sobre X és: 8x – 3y – 61 = 0
i la de X sobre Y és: x – y + 18 = 0.
 

824
Estadística bidimensional 1414

En l’altre estudi, les rectes de regressió són, respectivament:


8x – 5y + 20 = 0 5x – 2y – 10 = 0

 
Si coneixem � = 23, � = 41 i r = 0,8, comprova quin dels dos estudis és vàlid.

El primer estudi és el correcte, atès que les rectes es tallen en el punt (�� �� �� )

 
75. Considera dues variables estadístiques, X i Y.
Sabem que:
 La recta de regressió de Y sobre X passa pels punts (1, 3) i (2, 5).
 La recta de regressió de X sobre Y té pendent 3 i l’ordenada en l’origen és 2.
 La variància de Y és 3.
Calcula les mesures estadístiques de cadascuna de les variables i el coeficient de correlació.
La recta que passa pels punts (1, 3) i (2, 5) té com a equació: y = 2x + 1
L’equació de l’altra recta és: y = 3x + 2
�� � � � 1 � �
� → � � �1� � � �1
�� � � � � � �
�� � �1
Aleshores, el resultat és �
�� � �1
El coeficient de correlació és igual a l’arrel quadrada del producte del pendent de la recta de regressió de Y
sobre X per la inversa del pendent de la recta de regressió de X sobre Y:

1 ��� ��� ���


� � �� � �� � � � �
�� �� �� �� � ��


Per tant, tenim que: � � �� � � ���16�

El sistema d’equacions formades per les dues rectes de regressió és:


���
� �� � �
� ���
� → � � � � � → ��� � 6��� → �� � √6��
� �� � � ��
����

Com que la variància de Y és 3: ��� �

825
Estadística bidimensional
Estadística bidimensional 14

76. Investiga com varia el coeficient de correlació entre dues variables estadístiques quan multipliquem
les dades relatives a una de les variables per una quantitat constant, k.
I si multipliquem les dues variables per la mateixa constant?
Què passa si multipliquem cada variable per una constant diferent?
Quan multipliquem les dades d’una variable per una quantitat constant k, les seves mesures estadístiques
verifiquen que:

 
Aleshores la covariància entre les dues variables és:

Així doncs, el coeficient de correlació és:

݇ ൉ ߪ௑௒ ߪ௑௒
ൌ ൌ ‫ݎ‬௑௒  
݇ ൉ ߪ௑ ൉ ߪ௒ ߪ௑ ൉ ߪ௒

Si es multipliquem les dades de les dues variables per la mateixa constant k, aleshores el coeficient de
correlació és:

݇ ଶ ൉ ߪ௑௒ ߪ௑௒
ൌ ൌ ‫ݎ‬௑௒  
݇ ൉ ߪ௑ ൉ ݇ ൉ ߪ௒ ߪ௑ ൉ ߪ௒

I si multipliquem la segona variable per una constant m:

݇ ൉ ݉ ൉ ߪ௑௒ ߪ௑௒
ൌ ൌ ‫ݎ‬௑௒  
݇ ൉ ߪ௑ ൉ ݉ ൉ ߪ௒ ߪ௑ ൉ ߪ௒

 
 
 

826
Estadística bidimensional 1414

MATEMÀTIQUES A LA TEVA VIDA

1. Explica què entens per estacionalitat.


L’estacionalitat, en aquest cas, és la variació de les dades en funció de l’època de l’any, que es repiteix
cíclicament any rere any.

 
2. En quin dels gràfics estadístics que acompanyen el text es veu la dependència de dues variables?
Com s’hi veu?
En el segon gràfic. Mitjançant un diagrama de dispersió que relaciona l’edat dels clients i la despesa en
euros que duen a terme.

3. Explica quin tipus de gràfics estadístics surten en el text.


El primer i el tercer són polígons de freqüència i el segon és un diagrama de dispersió.

4. Posa un exemple d’estacionalitat.


Resposta oberta. Per exemple, el cabal del riu Duero al seu pas per Zamora.
 
5. Uns quants dies a l’any, al març o l’abril, l’ocupació hotelera es dispara, encara que no sempre és
durant els mateixos dies. Justifica per què és així.
Perquè en aquests mesos tenen lloc les vacances de Setmana Santa que se celebren la setmana anterior al
primer diumenge posterior a la primera lluna plena després l’equinocci de març.

6. Posa un exemple en què es produeixi un fenomen d’estacionalitat.


Resposta oberta. Per exemple, la migració de les cigonyes

 
   

827
Notes


Notes


Notes


Notes


© 2016 by Grup Promotor/Santillana Educación, S. L.
C/ Frederic Mompou, 11 (Vila Olímpica)
08005 Barcelona
Printed in Spain

CP: 570499

You might also like